You are on page 1of 927

Calculus

Third Edition
Gujarat Technological University 2017
About the Authors
Ravish R Singh is presently Academic Advisor at Thakur Educational
Trust, Mumbai. He obtained a BE degree from University of Mumbai
in 1991, an MTech degree from IIT Bombay in 2001, and a PhD
degree from Faculty of Technology, University of Mumbai, in 2013.
He has published several books with McGraw Hill Education (India)
on varied subjects like Engineering Mathematics, Applied
Mathematics, Electrical Networks, Network Analysis and Synthesis,
Basic, Electrical Engineering, Basic Electrical and Electronics
Engineering, etc., for all-India curricula as well as regional curricula of some universities
like Gujarat Technological University, Mumbai University, Pune University, Jawaharlal
Nehru Technological University, Anna University, Uttarakhand Technical University,
and Dr A P J Abdul Kalam Technical University. Dr Singh is a member of IEEE, ISTE,
and IETE, and has published research papers in national and international journals. His
fields of interest include Circuits, Signals and Systems, and Engineering Mathematics.

Mukul Bhatt is presently Assistant Professor, Department of


Humanities and Sciences, at Thakur College of Engineering and
Technology, Mumbai. She obtained her MSc (Mathematics) degree
from H N B Garhwal University in 1992, and a PhD degree from
Faculty of Science, PAHER University, Udaipur, Rajasthan in 2017.
She has published several books with McGraw Hill Education (India)
Private Limited on Engineering Mathematics and Applied Mathematics
for all-India curricula as well as regional curricula of some universities like Gujarat
Technological University, Mumbai University, Pune University, Jawaharlal Nehru
Technological University, Anna University, Uttarakhand Technical University, and
Uttar Pradesh Technical University. Dr. Bhatt has twenty six years of teaching
experience at various levels in engineering colleges and her fields of interest include
Integral Calculus, Complex Analysis, and Operation Research. She is a member of
ISTE.
Calculus
Third Edition
Gujarat Technological University 2017

Ravish R Singh
Academic Advisor
Thakur Educational Trust
Mumbai, Maharashtra

Mukul Bhatt
Assistant Professor
Department of Humanities and Sciences
Thakur College of Engineering and Technology
Mumbai, Maharashtra

McGraw Hill Education (India) Private Limited


Chennai

McGraw Hill Education Offices


Chennai new York St Louis San Francisco auckland Bogotá Caracas
Kuala Lumpur Lisbon London Madrid Mexico City Milan Montreal
San Juan Santiago Singapore Sydney Tokyo Toronto
McGraw Hill Education (India) Private Limited
Published by McGraw Hill Education (India) Private Limited
444/1, Sri Ekambara Naicker Industrial Estate, Alapakkam, Porur, Chennai 600 116
Calculus, 3e, GTU–2017
Copyright © 2018, 2017, 2015 by McGraw Hill Education (India) Private Limited.
No part of this publication may be reproduced or distributed in any form or by any means, electronic,
mechanical, photocopying, recording, or otherwise or stored in a database or retrieval system without
the prior written permission of the publishers. The program listing (if any) may be entered, stored and
executed in a computer system, but they may not be reproduced for publication.
This edition can be exported from India only by the publishers,
McGraw Hill Education (India) Private Limited.
ISBN 13: 978-93-5260-731-0
ISBN 10: 93-5260-731-7
Managing Director: Kaushik Bellani
Director—Science & Engineering Portfolio: Vibha Mahajan
Senior Portfolio Manager: Hemant Jha
Portfolio Manager: Navneet Kumar
Senior Manager—Content Development: Shalini Jha
Content Developer: Sahil Thorpe
Production Head: Satinder S Baveja
Copy Editor: Taranpreet Kaur
Assistant Manager—Production: Anuj K Shriwastava
General Manager—Production: Rajender P Ghansela
Manager—Production: Reji Kumar

Information contained in this work has been obtained by McGraw Hill Education (India), from sources
believed to be reliable. However, neither McGraw Hill Education (India) nor its authors guarantee the
accuracy or completeness of any information published herein, and neither McGraw Hill Education
(India) nor its authors shall be responsible for any errors, omissions, or damages arising out of use of
this information. This work is published with the understanding that McGraw Hill Education (India) and
its authors are supplying information but are not attempting to render engineering or other professional
services. If such services are required, the assistance of an appropriate professional should be sought.

Typeset at Text-o-Graphics, B-1/56, Aravali Apartment, Sector-34, Noida 201 301, and printed at

Cover Printer:

Visit us at: www.mheducation.co.in


Dedicated

to

My Mother

Late Shrimati Premsheela Singh

(Ravish R Singh)

to

My Sister

Vijay Luxmi Hemdan

(Mukul Bhatt)
Contents
Preface xi
Roadmap to the syllabus xv

Unit i infinite SeqUenCeS And SerieS

1. Sequences and Series 1.1–115


1.1 Introduction 1.1
1.2 Sequence 1.1
1.3 Infinite Series 1.8
1.4 The nth Term Test for Divergence 1.9
1.5 Geometric Series 1.10
1.6 Standard Limits 1.16
1.7 Comparison Test 1.17
1.8 D’Alembert’s Ratio Test 1.36
1.9 Cauchy’s Root Test 1.63
1.10 Cauchy’s Integral Test 1.71
1.11 Alternating Series 1.77
1.12 Absolute Convergence of a Series 1.84
1.13 Uniform Convergence of a Series 1.91
1.14 Power Series 1.94
Points to Remember 1.109
Multiple Choice Questions 1.111

2. taylor's and Maclaurin's Series 2.1–70


2.1 Introduction 2.1
2.2 Taylor’s Series 2.1
2.3 Maclaurin’s Series 2.27
Points to Remember 2.67
Multiple Choice Questions 2.68

Unit 2 CUrVe SKetCHinG

3. Curve Sketching 3.1–46


3.1 Introduction 3.1
3.2 Monotonic Functions 3.1
3.3 Concavity, Convexity and Points of Inflection of a Curve 3.1
viii Contents

3.4 Maxima and Minima 3.2


3.5 Tracing of Cartesian Curves 3.4
3.6 Tracing of Parametric Curves 3.24
3.7 Tracing of Polar Curves 3.31
Multiple Choice Questions 3.43

Unit 3 indeterMinAte forMS & iMproper inteGrAlS

4. indeterminate forms 4.1–81


4.1 Introduction 4.1
4.2 L’Hospital’s Rule 4.1
0
4.3 Type 1 : Form 4.2
0

4.4 Type 2: Form 4.24

4.5 Type 3 : 0 × • Form 4.31
4.6 Type 4 : • − • Form 4.38
4.7 Type 5 : 1•, • 0, 0 0 Form 4.46
4.8 Solving Indeterminate Forms Using Expansion 4.68
Points to Remember 4.79
Multiple Choice Questions 4.79

5. improper integrals 5.1–24


5.1 Introduction 5.1
5.2 Improper Integrals 5.1
5.3 Improper Integrals of the First Kind 5.1
5.4 Improper Integrals of the Second Kind 5.9
5.5 Improper Integral of the Third Kind 5.16
5.6 Convergence and Divergence of Improper Integrals 5.17
Multiple Choice Questions 5.22

Unit 4 AppliCAtionS of inteGrAtion

6. Applications of integration 6.1–43


6.1 Introduction 6.1
6.2 Volume by Slicing 6.1
6.3 Volume of Solid of Revolution 6.6
6.4 Volume by Cylindrical Shells 6.33
Multiple Choice Questions 6.41
Contents ix

Unit 5 pArtiAl deriVAtiVeS

7. partial derivatives 7.1–169


7.1 Introduction 7.1
7.2 Functions of Two or More Variables 7.1
7.3 Limit and Continuity of Functions of Several Variables 7.2
7.4 Partial Derivatives 7.11
7.5 Higher-Order Partial Derivatives 7.11
7.6 Variables to be Treated as Constants 7.55
7.7 Total Derivatives 7.70
7.8 Implicit Differentiation 7.105
7.9 Euler’s Theorem for Homogeneous Functions 7.114
Points to Remember 7.162
Multiple Choice Questions 7.163

8. Applications of partial derivatives 8.1–121


8.1 Introduction 8.1
8.2 Tangent Plane and Normal to a Surface 8.1
8.3 Linear Approximation or Linearization 8.10
8.4 Errors and Approximations 8.13
8.5 Maximum and Minimum Values by Second Derivative Test 8.21
8.6 Maximum and Minimum Values with Constrained Variables 8.38
8.7 Method of Lagrangian Multipliers 8.49
8.8 Taylor’s Formula For Two Variables 8.81
8.9 Jacobians 8.97
Points to Remember 8.115
Multiple Choice Questions 8.117

Unit 6 MUltiple inteGrAlS

9. Multiple integrals 9.1–194


9.1 Introduction 9.1
9.2 Double Integrals Over Rectangles 9.1
9.3 Change of Order of Integration 9.31
9.4 Double Integrals in Polar Coordinates 9.66
9.5 Change of Variables 9.77
9.6 Triple Integrals 9.109
9.7 Area as Double Integral 9.141
9.8 Volume as Triple Integral 9.169
Points to Remember 9.188
Multiple Choice Questions 9.190
x Contents

Appendix 1 Differential Formulae A1.1


Appendix 2 Integral Formulae A2.1–A2.3
Appendix 3 Standard Curves A3.3–A3.5

GTU Solved Question Paper – Winter 2016 SQP.1–SQP.7


GTU Solved Question Paper – Summer 2017 SQP.1–SQP.7

Index I.1–I.3
preface
Calculus is a key area of study in any engineering course. A sound knowledge of this
subject will help engineering students develop analytical skills, and thus enable them
to solve numerical problems encountered in real life, as well as apply mathematical
principles to physical problems, particularly in the field of engineering.

Users
This book will be useful for first-year engineering students of Gujarat Technological
University (GTU). It covers the complete GTU syllabus for the course on Calculus,
which is common to all the engineering branches.

Objective
Crisp, complete explanations of concepts seek to enable students to easily understand,
and thereby build a strong foundation in Calculus and its applications. The tutorial
approach followed in this text will help them develop a logical perspective to solving
problems. Solved question papers will help students understand the patterns of
examinations.

FeatUres
Each topic has been explained from the examination point of view, wherein the theory
is presented in an easy-to-understand student-friendly style. Full coverage of concepts
is supported by numerous solved examples with varied complexity levels. The
solutions of examples are set following a ‘tutorial’ approach, which will make it easy
for students from any background to easily grasp the concepts. Exercises with answers
immediately follow the solved examples enforcing a practice-based approach. We
hope that the students will gain logical understanding from solved problems and then
reiterate it through solving similar exercise problems themselves. The unique blend of
theory and application caters to the requirements of both the students and the faculty.

HigHligHts
∑ Crisp content strictly as per the latest GTU syllabus of Calculus
∑ Excellent coverage with lucid presentation style
∑ New! Multiple Choice Questions are incorporated within each chapter as per
GTU paper style
∑ Solutions of GTU examination papers from 2010 to 2017 are present
appropriately within the chapters
xii Preface

∑ Solution to latest Summer 2017 and Winter 2016 GTU question paper are
placed at the end of the book
∑ Rich exam-oriented pedagogy:
 180+ Enhanced New Multiple Choice Questions
 475+ Solved Examples within chapters
 150+ Exam oriented Solved GTU Questions tagged within chapters
 1450+ Unsolved Exercises

cHapter OrganisatiOn
The content spans the following nine chapters which wholly and sequentially cover
each module of the syllabus.
∑ Chapter 1 explains Sequences and Series.
∑ Chapter 2 presents Taylor’s and Maclaurin’s Series.
∑ Chapter 3 discusses Curve Tracing.
∑ Chapter 4 presents Indeterminate Forms.
∑ Chapter 5 covers Improper Integrals.
∑ Chapter 6 deals with Applications of Integration.
∑ Chapter 7 presents Partial Derivatives.
∑ Chapter 8 explains Applications of Partial Derivatives.
∑ Chapter 9 discusses Multiple Integrals.
∑ Appendices 1, 2, and 3 provide Differential Formulae, Integral Formulae,
and Standard Curves, respectively.

Acknowledgements
Special thanks to the reviewers mentioned here for providing encouraging comments and
valuable suggestions regarding improvement of manuscript.
Manokamna Agrawal Silver Oak College of Engineering and Technology,
Ahmedabad, Gujarat
Prakash Kumar Patel Babaria Institute of Technology, Vadodara, Gujarat
Hiren Bhatt Marwadi Education Foundation Group of Institutions, Rajkot,
Gujarat
Bhavini Pandya Sardar Vallabhbhai Patel Institute of Technology, Anand,
Gujarat
Usha Bag Shree L R Tiwari College of Engineering, Maharashtra.
We would also like to thank all the staff at McGraw Hill Education (India), especially
Vibha Mahajan, Shalini Jha, Sahil Thorpe, Satinder Singh Baveja, Taranpreet Kaur
and Anuj Shriwastava for coordinating with us during the editorial, copyediting, and
production stages of this book.
Preface xiii

Our acknowledgements would be incomplete without a mention of the contribution of


all our family members. We extend a heartfelt thanks to them for always motivating and
supporting us throughout the project.
Constructive suggestions for the improvement of the book will always be welcome.
Ravish R Singh
Mukul Bhatt

publisher’s note
Remember to write to us. We look forward to receiving your feedback, comments and
ideas to enhance the quality of this book.You can reach us at info.india@mheducation.com.
Please mention the title and authors’ name as the subject. In case you spot piracy of
this book, please do let us know.
roadmap to the Syllabus
This text is useful for: Calculus (2110014)
as taught in Gujarat Technological University

Unit 1: Infinite Sequences and Series


∑ Introduction of convergence, divergence of sequences and infinite series
∑ The nth term test for divergence; Integral test
∑ Comparison test, Ratio test, Root test
∑ Alternating series, Absolute convergence, Conditional convergence
∑ Power series and Radius of convergence
∑ Taylor’s series
∑ Maclaurin’s series

Go to:
CHApter 1. Sequences and Series
CHApter 2. taylor's and Maclaurin's series

Unit 2: Curve Sketching


∑ Concavity
∑ Curve sketching
∑ Polar coordinates, Relation between polar and Cartesian coordinates
∑ Graphs in polar coordinates

Go to:
CHApter 3. Curve Sketching

Unit 3:
indeterminate forms:
∑ Indeterminate form: (0/0, •/•, • ◊ 0, • - •)
∑ Indeterminate form: (00, •0, 1•)
improper integral:
∑ Improper integrals of Type-I and Type-II
∑ Convergence and divergence of improper integrals

Go to:
CHApter 4. indeterminate forms
CHApter 5. improper integrals
xvi Roadmap to the Syllabus

Unit 4: Applications of integration


∑ Volume by slicing
∑ Volume of solids of revolution by disk method
∑ Volume of solids of revolutions by washer method
∑ Volume by cylindrical shell

Go to:
CHApter 6. Applications of integration

Unit 5: partial derivatives


∑ Function of two variables, graphs, level curves
∑ Limit, continuity of function of several variables
∑ Partial derivatives and Clairaut's theorem
∑ Tangent plane, Normal line
∑ Linear approximation, Total differential
∑ Chain rule, Implicit differentiation
∑ Euler’s theorem for homogeneous function
∑ Maximum and minimum values by second derivative test
∑ Lagrange multipliers
∑ Taylor’s formula for two variables

Go to:
CHApter 7. partial derivatives
CHApter 8. Applications of partial derivatives

Unit 6: Multiple integrals


∑ Double integrals over rectangles and Fubini’s theorem, Properties of double
integrals
∑ Double integrals over general region
∑ Double integrals in polar coordinates
∑ Triple Integrals, Triple integrals in cylindrical coordinates
∑ Triple integrals in spherical coordinates
∑ Change of order of integration
∑ Jacobian of several variables, Change of variable in multiple integrals

Go to:
CHApter 9. Multiple integrals
Unit 1
Infinite Sequences
and Series
CHAPTER
Sequences
1
and Series
1.1 INTRODUCTION
In this chapter, we will learn about the convergence and divergence of sequence and
series. There are various methods to test the convergence and divergence of an infinite
series. We will study Comparison Test, D’Alembert’s ratio test, Cauchy’s root test
and Cauchy’s integral test. We will also study alternating series, absolute and uniform
convergence of the series and power series.

1.2 SEQUENCE

An ordered set of real numbers as u1, u2, u3, ……..un, …… is called a sequence and
is denoted by {un}. If the number of terms in a sequence is infinite, it is said to be an
infinite sequence, otherwise it is a finite sequence and un is called the nth term of the
sequence.

1.2.1 Limit of a Sequence


A sequence {un} tends to a finite number l as n Æ • if for every Œ > 0 there exists an
integer m such that, | un - l | < Πfor all n > m, i.e., lim un = l.
n Æ•

1.2.2 Convergence, Divergence and Oscillation of a


Sequence
(i) If the sequence {un} has a finite limit, i.e., lim un is finite, the sequence is said
n Æ•
to be convergent.
Ï ¸
Ô 1 Ô
e.g. {{un } = Ì ˝
Ô1 + 1 Ô
Ó n˛
lim un = 1
n Æ•

Since limit is finite, the sequence is convergent.


1.2 Chapter 1 Sequences and Series

(ii) If the sequence {un} has infinite limit, i.e., lim un is infinite, the sequence is said
n Æ•
to be divergent.
e.g. {un} = {{22 n + 1}
lim un = •
n Æ•

Since limit is infinite, the sequence is divergent.


(iii) If the limit of the sequence {un} is not unique, the sequence is said to be
oscillatory.
1
e.g. {{un} = ( -1)n +
2n
lim un = 1, if n is even
n Æ•
= –1, if n is odd
Since limit is not unique, the sequence is oscillatory.

1.2.3 Monotonic Sequence


A sequence is said to be monotonically increasing if unn++1 ≥ un for each value of n and
is monotonically decreasing if unn++1 £ un for each value of n. The sequence is called
alternating sequence if the terms are alternate positive and negative.
e.g. (i) 1, 2, 3, 4, … is a monotonically increasing sequence.
1 1 1
(ii) 1, , , , … is a monotonically decreasing sequence.
2 3 4
(iii) 1, –2, 3, – 4, … is an alternating sequence.

1.2.4 Bounded Sequence


A sequence {un} is said to be a bounded sequence if there exists numbers m and M
such that m < un < M for all n.
Note 1: Every convergent sequence is bounded but the converse is not true.
Note 2: A monotonic increasing sequence converges if it is bounded above and
diverges to + • if it is not bounded above.
Note 3: A monotonic decreasing sequence converges if it is bounded below and
diverges to – • if it is not bounded below.
Note 4: If sequence {un} and {vn} converges to l1 and l2 respectively then
(i) Sequence {un + vn} converges to l1 + l2
(ii) Sequence {un . vn} converges to l1◊ l2
Ïu ¸ l
(iii) Sequence Ì n ˝ converges to 1 provided l2 π 0
v
Ó n˛ l2
1.2 Sequence 1.3

Example 1
ÏÔ n2 + n ¸Ô̧Ô
Test the convergence of the sequence Ì ˝.
ÔÓ 2 n - n Ô˛
2

Solution
n2 + n
Let un =
2n2 - n
n2 + n
lim un = lim
n Æ• 2n2 - n
n Æ•

1
1+
= lim n
n Æ• 1
2-
n
1
=
2
Hence, {un} is convergent.

Example 2
Test the convergence of the sequence {tanh n}.
Solution
Let un = tanh n
lim un = lim tanh n
n Æ• n Æ•
sinh n
= lim
n Æ• cosh n

en - e- n
= lim
n Æ• en + e- n
e2 n - 1
= liim
m
n Æ• e2 n + 1
1
1 - 2n
= lim e
n Æ• 1
1 + 2n
e
=1
Hence, {un} is convergent.

Example 3
Test the convergence of the sequence {2n}.
1.4 Chapter 1 Sequences and Series

Solution
Let un= 2n
lim un = lim 2 n
n Æ• n Æ•
=•
Hence, {un} is divergent.

Example 4
Test the convergence of the sequence 2 - ( -1)n . { }
Solution
Let un= 2 –(–1)n
lim un = lim 2 - (-
( 1)n
n Æ• n Æ•
=2–1=1 , if n is even
= 2 –(–1) = 3 , if n is odd
Since limit is not unique, the sequence {un} is oscillatory.

Example 5
1 1 1
Show that the sequence {un} whose nth term is un = 1 + + 2 +  + n ,
3 3 3
is monotonic increasing and bounded. Is it convergent?
Solution
1 1 1
un = 1 + + ++ n
3 32 3
1 1 1 1
un +1 = 1 + + 2 +  + n + n +1
3 3 3 3
1
unn++1 - un = n +1
>0
3

Hence, {un} is monotonic increasing sequence.


1 1 1
Also, un = 1 + + ++ n
3 32 3
Ê 1 ˆ
1 Á 1 - n +1 ˜
Ë 3 ¯
=
1
1-
3
1.2 Sequence 1.5

3Ê 1 ˆ 3
= Á 1 - n +1 ˜ <
2 Ë 3 ¯ 2
3
{un} is bounded above by .
2
Since {un} is monotonic increasing and bounded above, it is convergent.

Example 6
1 2 1
Show that the sequence {un} whose nth term is un = + + …+ ,
1! 2 ! n!
n ΠN, is monotonic increasing and bounded. Is it convergent?
Solution
1 1 1
un = + + …+
1! 2 ! n!
1 1 1 1
un +1 = + + … + +
1! 2 ! nn!! (n + 1)!
1
unn++1 - un = >0
(n + 1)!
un+1 > un
Hence, {un} is a monotonic increasing sequence.
1 1 1 1
Also, un = + + ++
1! 2 ! 3! n!
1 1 1
= 1+ + ++
2 ! 3! n!
1 1 1
< 1+ + 2 ++ n
2 2 2
Ê 1 ˆ
1 Á 1 - n +1 ˜
Ë 2 ¯
< [Using sum of G.P]
1
1-
2
Ê 1 ˆ
< 2 Á 1 - n +1 ˜
Ë 2 ¯
<2
{un} is bounded above by 2.
Since {un} is monotonic increasing and bounded above, it is convergent.

Example 7
Ï n ¸
Show that the sequence Ì 2 ˝ is monotonic decreasing and bounded.
Ó n + 1˛
Is it convergent?
1.6 Chapter 1 Sequences and Series

Solution
n
Let un =
n2 + 1
n +1
un +1 =
(n + 1)2 + 1
n +1 n
unn++1 - un = -
(n + 1) + 12
n +1
2

(n + 1)(n + 1) - n(n2 + 2 n + 2)
2
=
(n2 + 2 n + 2)(nn2 + 1)
-nn2 - n + 1
-
= <0
(n + 2 n + 2)(n2 + 1)
2

Hence, {un} is a monotonic decreasing sequence.


n
Also, un = >0
n +1
2

{un} is bounded below by 0.


Since {un} is monotonic decreasing and bounded below, it is convergent.

1.2.5 Sandwich Theorem for Sequences


Let {un}, {vn} and {wn} be three sequences such that un £ vn £ wn for all n.
If lim un = lim
lim wn = l, then lim vn = l
n Æ• n Æ• n Æ•

Example 1
sin n
Show that the sequence {un}, where un = converges to zero.
n
Solution
We know that
-1 £ sin n £ 1
1 sin n 1
- £ £
n n n
Ê 1ˆ
lim Á - ˜ = 0
n Æ• Ë n ¯

Ê 1ˆ
lim Á ˜ = 0
n Æ• Ën¯
By sandwich theorem,
sin n
lim =0
n Æ• n
Hence, {un} converges to zero.
1.2 Sequence 1.7

Example 2
If x Œ R with | x | < 1 then prove that xn Æ 0 as n Æ •.
Solution
For x = 0, xn = 0
1
For x π 0, let | x | =
1+ y
1
| x |n =
(1 + y)n
1
=
n(n -1
- 1) y 2
1 + ny + +
2!
1 È ÏÔ n(n - 1) y 2 ¸Ô̧Ô ˘
< Í∵ Ì1 + ny + + ˝ > ny ˙
ny ÍÎ ÓÔ 2! ˛Ô ˙˚
n 1
0< x <
ny
1
lim =0
n Æ• ny
By sandwich theorem,
n
lim x =0
n Æ•
n
Hence, x Æ 0 as n Æ •

EXERCISE 1.1
1. Test the convergence of the following sequences:
2n + 1
(i) (ii) 2 + (0.1)n (iii) 1 + (–1)n
1 - 3n
n2 Ê 1ˆ
(iv) en (v) 1 + (–1)n (vi) sin Á ˜
2n - 1 Ë n ¯
(vii) tan–1 n
 Ans. : (i) convergent (ii) convergent
 
 (iii) divergent (iv) divergent
ivergentt 
 (v) oscillatory (vi) convergent 
 
 (vii) convergent 
1.8 Chapter 1 Sequences and Series

2. Determine whether the following sequences are monotonically increasing/


decreasing, bounded or convergent/divergent.
1 2n − 7
(i) 1 + (ii)
n 3n + 2
 Ans. : (i) decreasing, bounded, convergent 
 
 (ii) increasing, bounded
ed, convergent 

1 1 1
3. Show that the sequence {un}, where un = 1 + + +  + ; n ≥ 2,
is convergent. 1! 2 ! (n − 1)!
n
 n + 1
4. Does the sequence {un} convergent where un =  ?
 n − 1
[ Ans.: yes ]
Ans. : yes

1.3 INFINITE SERIES


If u1, u2, u3, . . . un, . . . is an infinite sequence of real numbers, then the sum of the terms
of the sequence, u1 + u2 + u3 + … + un + … • is called an infinite series.

The infinite series u1 + u2 + u3 + . . . + un + . . . • is usually denoted by  un or Sun.
n =1
The sum of its first n terms is denoted by Sn and is also known as nth partial sum of Sun.

1.3.1 Convergence, Divergence and Oscillation of Infinite


Series
Consider the infinite series Σun = u1 + u2 + u3 + … un + … ∞ and let the sum of the first
n terms be Sn = u1 + u2 + u3 + . . . + un. As n Æ •, three possibilities arise for Sn:
(i) If Sn tends to a finite limit as n Æ •, the series Sun is said to be convergent.
(ii) If Sn tends to ± • as n Æ •, the series Sun is said to be divergent.
(iii) If Sn does not tend to a unique limit as n Æ •, i.e., limit does not exist, the
series Sun is said to be oscillatory.

1.3.2 Properties of Infinite Series


1. The convergence or divergence of an infinite series remains unaffected:
(i) by addition or removal of a finite number of terms
(ii) by multiplication of each term with a finite number
2. If two series Sun and Svn are convergent, then SS(((u
un + vn ) is also convergent.
3. If two series Sun and Svn are divergent, then S(un+ vn) may be convergent.
1.4 The nth Term Test for Divergence 1.9

4. If each term of a series Sun of positive terms does not exceed the corresponding
term of a convergent series Svn of positive terms, then Sun is convergent.
5. If each term of a series Sun of positive terms exceeds the corresponding term of
a divergent series Svn of positive terms, then Sun is divergent.

Example 1
Test the convergence of the series
1 1 1
+ + +
1◊ 2 2 ◊ 3 3 ◊ 4 [Summer 2014]
Solution
1
un =
n(n + 1)
n + 1- n
=
n(n + 1)
1 1
= -
n n +1
Ê 1ˆ Ê 1 1ˆ Ê 1 1 ˆ Ê1 1 ˆ
Sn = Á 1 - ˜ + Á - ˜ + Á - ˜ +  + Á -
Ë 2 ¯ Ë 2 3¯ Ë 3 4 ¯ Ë n n + 1˜¯
1
= 1-
n +1
Ê 1 ˆ
lim Sn = lim Á 1 -
n Æ• n Æ• Ë n + 1˜¯
= 1- 0
= 1 [finite]
Hence, the series Sun is convergent.

1.4 THE nth TERM TEST FOR DIVERGENCE


If a positive term series Sun is convergent, then nlim un = 0.
Æ•

The converse of this result is not true, i.e., if lim un = 0, it is not necessary that the
n Æ•
series will be convergent.

1 1 1
e.g. Â un = 1 + 2
+
3
+…+
n
+…•
1.10 Chapter 1 Sequences and Series

1
lim un = lim =0
n Æ• n Æ• n
1 1 1 1 1 1
Now, Sn = 1 + + +…+ > 1+ + +…+
2 3 n n n n
n
Sn >
n
Sn > n

and lim n = •
n Æ•

Thus, the series is divergent.


Hence, lim un = 0 is a necessary but not sufficient condition for convergence
n→∞
of Sun.

If lim un π 0 or lim un does not exist, then Sun is divergent.


n Æ• n Æ•

1.5 GEOMETRIC SERIES


Consider the geometric series a + ar + ar 2 +  + ar n -1 +  ... (1)
n −1
S n = a + ar + ar +  + ar
2

(1 − r n )
aa(1
= , if r < 1
1− r
a (r n − 1)
a(
= , if r > 1
r −1

(i) When | r | < 1, lim r n = 0


n Æ•

a
lim Sn = is finite.
n Æ• 1- r
Hence, the series is convergent.
(ii) When r > 1, lim r n Æ ∞
nÆ∞

a(r n - 1)
lim Sn = lim Æ∞
nÆ∞ nÆ∞ r -1
Hence, the series is divergent.
(iii) When r = 1,
Sn = a + a + a +  = na
lim Sn Æ ∞
nÆ∞

Hence, the series is divergent.


1.5 Geometric Series 1.11

(iv) When r = –1,


Sn = a - a + a -  ( -1)n -1 a
= 0, if n is even
= a, if n is odd
Hence, the series is oscillatory.
(v) When r < –1, let r = –k where k > 0
a[1 - ( - k )n ]
lim Sn = lim
n Æ• n Æ• 1+ k
a[1 - ( -1)n k n ]
= lim
n Æ• 1+ k
= – •, if n is even
= + •, if n is odd
Hence, the series is oscillatory.
From all the above cases, we conclude that the geometric series (1) is
(i) convergent if | r | < 1
(ii) divergent if r ≥ 1
(iii) oscillatory if r £ –1

1 1 1 1
Note: The p series  np =
1 p
+
2 p
+
3p
+  is
n =1
(i) convergent if p > 1
(ii) divergent if p £ 1

Example 1
2 4 8 16
Prove that 1 + + + + +  converges and find its sum.
3 9 27 81
[Winter 2014]
Solution
2
The given series is geometric series with a = 1 and r = .
3
• • n -1
2 4 8 Ê 2ˆ
1+ + + +  = Â ar n -1 = Â Á ˜
3 9 27 n =1
Ë ¯
n =1 3
2
r = <1
3
Hence, the series is convergent.
a 1
Sn = = =3
1- r 2
1-
3
1.12 Chapter 1 Sequences and Series

Example 2
10 20 40
Test the convergence of the series 5 - + - +
3 9 27
[Summer 2017]
Solution
2
The given series is geometric series with a = 5 and r = - .
3

10 20 40
5- + - +  = Â ar n - 1
3 9 27 n =1
• n -1
Ê 2ˆ
= Â 5Á- ˜
n =1
Ë 3¯

2 2
r = - = <1
3 3
Hence, the series is convergent.
a
Sn =
1- r
5
=
Ê 2ˆ
1- Á- ˜
Ë 3¯
5
=
2
1+
3
5
=
5
3
=3

Example 3
• 3n -1 - 1
Find the sum of the series  . [Summer 2015]
n =1 6 n -1
Solution

3n -1 - 1 • ÈÊ 1 ˆ n -1 Ê 1 ˆ n -1 ˘
 6 n -1
=  ÍÍÁË 2 ˜¯ -Á ˜ ˙
Ë 6¯ ˙
n =1 n =1Î ˚
• n -1 • n -1
Ê 1ˆ Ê 1ˆ
=  ÁË 2 ˜¯ -  ÁË 6 ˜¯
n =1 n =1
1.5 Geometric Series 1.13

Ê 1 1 1 ˆ Ê 1 1 ˆ
= Á 1 + + + + ˜ - Á 1 + + + ˜
Ë 2 4 8 ¯ Ë 6 36 ¯
1 1
Both the series are geometric series with a = 1 and r = and r = respectively.
2 6
a1 a
Sn = - 2
1 - r1 1 - r2
1 1
= -
1 1
1- 1-
2 6
6
= 2-
5
4
=
5

Example 4

Let S = Â n a n where a < 1. Find the value of a in (0, 1) such that
n =1
S = 2a. [Winter 2016]
Solution
S = a + 2a2 + 3a3 + ... ... (1)
aS = a2 + 2a3 + 3a4 + ... ... (2)
Subtracting Eq. (2) from Eq. (1),
S(1 – a) = a + a2 + a3 + ...
a
S(1 - a ) =
1-a
a
S=
(1 - a )2
If S = 2 a,
a
2a =
(1 - a )2
1
(1 - a )2 =
2
1
1-a =
2
a = 0.2929
1.14 Chapter 1 Sequences and Series

Example 5
• 2n + 5
Investigate the convergence of the series  .
n =1 3n
[Summer 2015]
Solution

2n + 5 • ÈÊ 2 ˆ n Ê 1ˆ ˘
n

 3n
=  ÍÁË 3 ˜¯ ÁË 3n ˜¯ ˙˙
Í + 5
n =1 n =1Î ˚

• n • n
Ê 2ˆ Ê 1ˆ
=  ÁË 3 ˜¯ +  5 ÁË 3 ˜¯
n =1 n =1
• n -1 • n -1
Ê 2ˆ Ê 2ˆ Ê 5ˆ Ê 1ˆ
=  ÁË 3 ˜¯ ÁË 3 ˜¯ +  ÁË 3 ˜¯ ÁË 3 ˜¯
n =1 n =1

2 1
Both the series are geometric series with r1 = and r2 = respectively.
3 3
| r1 | < 1 and | r2 | < 1
Hence, both the series are convergent.
2
2
a 3
S1 = 1 = = 3 =2
1 - r1 2 1
1-
3 3
55
a 3 5
S2 = 2 = = 3 =
1 - r2 1 2 2
1-
3 3

2n + 5 5 9
Hence, Â 3 n
= 2+ = .
2 2
n =1

Example 6
A ball is dropped from ‘a’ meters above a flat surface. Each time the
ball hits the surface after falling a distance h, it rebounds a distance rh
where 0 < r < 1. Find the total distance the ball travels up and down,
2
when a = 6 m and r = m. [Winter 2016]
3
1.5 Geometric Series 1.15

Solution
Total distance (h) = a + 2ar + 2ar2 + 2ar3 + ...
2 ar
=a+
1- r
a(1 + r )
=
1- r
2
Here, a=6m r= m
3
Ê 2ˆ
6 Á1 + ˜
Ë 3¯ 5 3
h= = 6 ◊ ◊ = 30 m
Ê 2ˆ 2 1
ÁË 1 - 3 ˜¯

Example 7
The figure 1.1 shows the first seven of a sequence of squares. The
outermost square has an area of 4m2. Each of the other squares is
obtained by joining the midpoints of the sides of the squares in the
infinite sequence. Find sum of the areas of all the squares in the infinite
sequence. [Winter 2015]
Solution
Since each square is obtained by joining the midpoints of the square before its, area of
each square is half the area of previous square.
1
Area of 2nd square = (area of outermost square)
2
1
= (4) = 2
2
1
Area of 3rd square = (area of 2 nd square)
2
Fig. 1.1
1
= (2) = 1
2

1
Area of 4th square = (area of 3rd square)
2
1 1
= (1) =
2 2
and so on.
1.16 Chapter 1 Sequences and Series

Sum(s) of areas of all squares in the infinite sequence is


1
S = 4 + 2 +1+ +
2
Ê 1 1 1 ˆ
= 4 Á 1 + + + + ˜
Ë 2 4 8 ¯

Ê 1 1 1 ˆ
= 4 Á 1 + + 2 + 3 + ˜
Ë 2 2 2 ¯

1
which is an infinite geometric series with a = 1, r = .
2

Ê a ˆ
S = 4Á
Ë 1 - r ˜¯

Ê ˆ
Á 1 ˜
= 4Á

Á 1- ˜
Ë 2¯
=8

1.6 STANDARD LIMITS

log n
(i) lim =0 (vi) lim x n = 0 if x < 1
n Æ• n n Æ•

n
Ê 1ˆ
(ii) lim Á 1 + ˜ = e (vii) lim x n = • if x > 1
n Æ• Ë n¯ n Æ•

1
xn
(iii) lim (n) n = 1 (viii) lim = 0 for all x
n Æ• n Æ• n!
1
Ê a n - 1ˆ
(iv) lim (n !) n = • (ix) lim Á = log a
n Æ• nÆ0 Ë n ˜¯

1 1
Ê n!ˆ n 1 an -1
(v) lim Á ˜ = (x) lim = log a
n Æ• Ë n ¯ e n Æ• 1
n
1.7 Comparison Test 1.17

1.7 COMPARISON TEST


u
If Sun and Svn are series of positive terms such that lim n = l (finite and non-zero)
n Æ• vn
then both series converge or diverge together.

Proof un
lim =l
nƕ vn

By definition of limit, for a positive number Œ, however small, there exists an integer
m such that
un
-l < Πfor all n > m
vn
un
- Œ<
< - l <Πfor all n > m
vn
un
l - Œ<
< < l+ Πfor all n > m
vn

Neglecting the first m terms of Sun and Svn,


un
l - Œ<
< < l+ Πfor all n ... (1)
vn
Case I If Svn is convergent then lim (v1 + v2 + v3 +  + vn ) = finite = k , say
n Æ•
From Eq. (1),
un
< l+ ∈
vn
un < (l + ∈)vn for all n
lim (u1 + u2 + u3 +  + un ) < (l + Œ) lim (v1 + v2 + v3 +  + vn )
n Æ• n Æ•
lim ((uu1 + u2 + u3 +  + un ) < ((ll + Œ)k (finite)
n Æ•

Hence, Sun is also convergent.


Case II If Svn is divergent then
lim (v1 + v2 + v3 + ... + vn ) Æ • ... (2)
n Æ•
From Eq. (1),
un
l - Œ<
vn
un > (l - Œ)vn for all n
lim (u1 + u2 + u3 +  + un ) > (l - Œ) lim (v1 + v2 + v3 +  + vn )
n Æ• n Æ•
1.18 Chapter 1 Sequences and Series

lim (u1 + u2 + u3 +  + un ) Æ • [From Eq. (2)]


n Æ•
Hence, Sun is also divergent.

Example 1 • n
Test the convergence of the series  2
.
n=1 n +1
Solution
n
Let un =
n +12

1
= 3
Ê 1ˆ
n 2 Á1 + 2 ˜
Ë n ¯
1
Let vn = 3
n2
un 1
lim = lim
n ƕ vn nƕ 1
1+
n2
=1 [finite and non-zero]
1 3
and Svn = Â 3
is convergent as p = > 1.
2
n2
Hence, by comparison test, Sun is also convergent.

Example 2

22nn - 1
Test the convergence of the series  n(n + 1)(n + 2) .
n =1

Solution
2n - 1
Let un =
n(n + 1)(n + 2)
Ê 1ˆ
ÁË 2 - n ˜¯
=
Ê 1ˆ Ê 2ˆ
n2 Á 1 + ˜ Á 1 + ˜
Ë n¯ Ë n¯
1
Let vn =
n2
1.7 Comparison Test 1.19

Ê 1ˆ
Á 2- ˜
u Ë n¯
lim n = lim
n Æ• vn n Æ• Ê 1ˆ Ê 2ˆ
ÁË 1 + n ˜¯ ÁË 1 + n ˜¯
= 2 [finite and non-zero]
1
and the series Svn = Â is convergent as p = 2 > 1.
n2
Hence, by comparison test, Sun is also convergent.

Example 3
• 1
Test the convergence of the series  .
n =1 1 + 2
2
+ 32 +  + n2
[Winter 2015]
Solution
1
Let un =
1 + 2 + 3 +  + n2
2 2

6
=
n(n + 1)(2 n + 1)
6
=
Ê 1 ˆÊ 1ˆ
n3 Á 1 + ˜ Á 2 + ˜
Ë n¯ Ë n¯
1
Let vn =
n3
un 6
lim = lim
n Æ• vn n Æ• Ê 1ˆ Ê 1ˆ
ÁË 1 + n ˜¯ ÁË 2 + n ˜¯
= 6 [finite and non-zero]
1
and Svn = Â is convergent as p = 3 > 1.
n3
Hence, by comparison test, Sun is also convergent.

Example 4
• n+2
Test the convergence of the series  .
n =1 ( n + 1) n
1.20 Chapter 1 Sequences and Series

Solution
n+2
Let un =
(n + 1) n
2
1+
= n
1
 1
n 2 1 + 
 n
1
Let vn = 1
n2
Ê 2ˆ
un ÁË 1 + n ˜¯
lim = lim
n Æ• vn n Æ• Ê 1ˆ
ÁË 1 + n ˜¯
= 1 [finite and non-zero]
1 1
and Svn = Â is divergent as p = < 1.
1 2
n2
Hence, by comparison test, Sun is also divergent.

Example 5
• 2n + 1
Is the series  convergent or divergent? [Summer 2015]
n =1 ( n + 1)
2

Solution
2n + 1
Let un =
(n + 1)2
Ê 1ˆ
nÁ2 + ˜
Ë n¯
= 2
Ê 1ˆ
n2 Á 1 + ˜
Ë n¯
Ê 1ˆ
2+ ˜
1 ÁË n¯
=
n Ê 1ˆ2
ÁË 1 + n ˜¯

1
Let vn =
n
1.7 Comparison Test 1.21

Ê 1ˆ
un ÁË 2 + n ˜¯
lim = lim 2
n Æ• vn n Æ•
Ê 1ˆ
ÁË 1 +
n ˜¯
2
= =2 [finite and non-zero]
1
1
and Svn = Â is divergent as p = 1.
n
Hence, by comparison test, Sun is also divergent.

Example 6 1
• (2 n2 - 1) 3
Test the convergence of the series  1
. [Winter 2013]
n =1
(3n3 + 2 n + 5) 4
Solution 1
2 n2 - 1) 3
((2
Let un = 1
3n3 + 2 n + 5) 4
((3
1
2
Ê 1 ˆ3
n3 ÁË 2 - 2 ˜¯
n
= 1
3
Ê 2 5 ˆ4
n4 ÁË 3 + 2 + 3 ˜¯
n n
1
Ê 1 ˆ3
ÁË 2 - 2 ˜¯
n
= 1
1
Ê 2 5 ˆ4
n 12 ÁË 3 + 2 + 3 ˜¯
n n
1
Let vn = 1
n 12
1
Ê 1 ˆ3
un ÁË 2 - 2 ˜¯
n
lim = lim 1
n ƕ vn nƕ
Ê 2 5 ˆ4
ÁË 3 + 2 + 3 ˜¯
n n
1
(2 ) 3
= 1
[finite and non-zero]
(3) 4
1.22 Chapter 1 Sequences and Series

1 1
and Svn = Â 1
is divergent as p = < 1.
12
n 12

Hence, by comparison test, Sun is also divergent.

Example 7
1 2 3 4
Test the convergence of the series + + + +.
1 3 3◊5 5◊7 7◊9
1◊
Solution
n
Let un =
2 n - 1)((2
((2 2 n + 1)
1
=
Ê 1ˆ Ê 1ˆ
nÁ2 - ˜ Á2 + ˜
Ë n¯ Ë n¯
1
Let vn =
n
un 1
lim = lim
n Æ• vn n Æ• Ê 1ˆ Ê 1ˆ
ÁË 2 - n ˜¯ ÁË 2 + n ˜¯

1
= [finite and non - zero]
4

1
and Svn = Â is divergent as p = 1.
n
Hence, by comparison test, Sun is also divergent.

Example 8
1 1 1
Test the convergence of the series p
+ p
+ + .
3 5 7p
Solution
1
Let un =
(2 n + 1) p
1
= p
Ê 1ˆ
np Á 2 + ˜
Ë n¯
1
Let vn =
np
1.7 Comparison Test 1.23

un 1
lim = lim p
Æ• vn Æ•
nƕ
n n
Ê 1ˆ
ÁË 2 + n ˜¯
1
= [finite and non-zero]
2p
1
and Svn = Â is convergent if p > 1 and divergent if p £ 1.
np
Hence, by comparison test, Sun is also convergent if p > 1 and divergent if p £ 1.

Example 9
Test the convergence of the series 2 + 3 + 4 + 5 +  + n + 1 + .
1 8 27 64 n3
Solution
n +1
Let un =
n3
1 Ê 1ˆ
= 2 Á1 + ˜
n Ë n¯
1
Let vn =
n2
un Ê 1ˆ
lim = lim 1 + ˜
n Æ• vn nÆ• ÁË n¯
=1 [finite and non-zero]
1
and Svn = Â is convergent as p = 2 > 1.
n2
Hence, by comparison test, Sun is also convergent.

Example 10
1 2 3
Test the convergence of the series + + +.
1+ 2 1+ 2 3 1+ 3 4
Solution
n
Let un =
1+ n n +1
n
=
3 Ê ˆ
Á 1 1˜
n 2 + 1+
Á 3 n˜
Ë n2 ¯
1.24 Chapter 1 Sequences and Series

1
=
1 Ê ˆ
Á 1 1˜
n 2 + 1+
Á 3 n˜
Ën 2 ¯
1
Let vn = 1
n2
un 1
lim = lim
n Æ• vn nÆ• Ê ˆ
Á 1 + 1+ 1 ˜
Á 3 n˜
Ë n2 ¯
= 1 [finite and non-zero]
1 1
and Svn = 1
is divergent as p = < 1.
2
n2
Hence, by comparison test, Sun is also divergent.

Example 11
1 2 3
Test the convergence of the series 3
+ 3
+ +.
2 3 43
Solution
n
Let un =
(n + 1)3
1
n2
= 3
( n + 1) 2
1
n2
= 3
3
Ê 1ˆ 2
n2 ÁË 1 + n ˜¯

1
= 3
Ê 1ˆ 2
n Á1 + ˜
Ë n¯
1
Let vn =
n
1.7 Comparison Test 1.25

un 1
lim = lim 3
n ƕ vn nƕ
Ê 1ˆ 2
ÁË 1 + n ˜¯
=1 [finite and non-zero]
1
and Svn = Â is divergent as p = 1.
n
Hence, by comparison test, Sun is also divergent.

Example 12
14 24 34
Test the convergence of the series 3
+ 3
+ +.
1 2 33
Solution
nth term of the numerator = a + (n – 1)d = 14 + (n – 1)10 = 10n + 4
nth term of the denominator = n3
10 n + 4
un =
n3
1 Ê 4ˆ
= 2 Á 10 + ˜
n Ë n¯
1
Let vn =
n2
un Ê 4ˆ
lim = lim 10 + ˜
n Æ• vn nÆ• ÁË n¯
= 10 [finite and non-zero]
1
and Svn = is convergent as p = 2 > 1.
n2
Hence, by comparison test, Sun is also convergent.

Example 13
1 2 3
Test the convergence of the series + + + .
a ◊1 + b 2
a◊2 + b
2
a ◊ 32 + b
Solution
n
Let un =
a ◊ n2 + b
1
=
Ê bˆ
nÁa + 2 ˜
Ë n ¯
1.26 Chapter 1 Sequences and Series

1
Let vn =
n
un 1
lim = lim
n Æ• vn n Æ• Ê bˆ
ÁË a + 2 ˜¯
n
1
= [finite and non-zero]
a
1
and Svn = Â is divergent as p = 1.
n
Hence, by comparison test, Sun is also divergent.

Example 14
1 2 3
Test the convergence of the series + + + .
12 + m 22 + m 32 + m
Solution
n
Let un =
n2 + m
1
=
Ê mˆ
n Á1 + 2 ˜
Ë n ¯
1
Let vn =
n
un 1
lim = lim
n Æ• vn n Æ• Ê mˆ
ÁË 1 + 2 ˜¯
n
= 1 [finite and non-zero]
1
and Svn = Â is divergent as p = 1.
n
Hence, by comparison test, Sun is also divergent.

Example 15
2 ◊ 13 + 5 2 ◊ 23 + 5 2 ◊ n3 + 5
Test the convergence of the series + + + + .
4 ◊ 15 + 1 4 ◊ 25 + 1 4 ◊ n5 + 1
Solution
2 n3 + 5
Let un =
4n5 + 1
1.7 Comparison Test 1.27

Ê 5ˆ
ÁË 2 + 3 ˜¯
n
=
2Ê 1ˆ
n Á4+ 5 ˜
Ë n ¯
1
Let vn =
n2
Ê 5ˆ
ÁË 2 + ˜
un n3 ¯
lim = lim
n Æ• vn n Æ• Ê 1ˆ
ÁË 4 + ˜
n5 ¯
2
= [f
[finite and non-zero]
4
1
and Svn = Â is convergent as p = 2 > 1.
n2
Hence, by comparison test, Sun is also convergent.

Example 16
1 2
1◊ 3 4
3◊ 5 6
5◊
Test the convergence of the series + + +.
3 ◊4
2 2
5 ◊6
2 2
72 ◊ 82
[Winter 2013]
Solution
2 n - 1) ◊ 2 n
((2
Let un = [Using A.P.]
2 n + 1)2 ((2
((2 2 n + 2 )2
Ê 1ˆ
ÁË 2 - n ˜¯ ◊ 2
= 2 2
Ê 1ˆ Ê 2ˆ
n2 Á 2 + ˜ Á 2 + ˜
Ë n¯ Ë n¯
1
Let vn =
n2
Ê 1ˆ
2Á2 - ˜
un Ë n¯
lim = lim 2 2
n Æ• vn nÆ• Ê 1ˆ Ê 2ˆ
ÁË 2 + 2 +
n ˜¯ ÁË n ˜¯

1
= [f
[finite and non-zero]
4
1.28 Chapter 1 Sequences and Series

1
and Svn = Â is convergent as p = 2 > 1.
n2
Hence, by comparison test, Sun is also convergent.

Example 17
1 3 5 7
Test the convergence of the series + + + +.
1◊ 2 ◊ 3 2 ◊ 3 ◊ 4 3 ◊ 4 ◊ 5 4 ◊ 5 ◊ 6
[Winter 2014]
Solution
((22 n + 1)
Let un = [Using A.P.]
n(n + 1)(n + 2)
Ê 1ˆ
ÁË 2 + n ˜¯
=
Ê 1ˆ Ê 2ˆ
n2 Á 1 + ˜ Á 1 + ˜
Ë n¯ Ë n¯
1
Let vn = 2
n
Ê 1ˆ
un ÁË 2 + n ˜¯
lim = lim
n Æ• vn n Æ• Ê 1ˆ Ê 2ˆ
ÁË 1 + n ˜¯ ÁË 1 + n ˜¯
= 2 [finite and non-zero]
1
and Svn = Â is convergent as p = 2 > 1.
n2
Hence, by comparison test, Sun is also convergent.

Example 18

Test the convergence of the series Â( n4 + 1 - n4 - 1 . )
n =1
[Summer 2016]
Solution
Let un = n 4 + 1 - n 4 - 1

( )
=
n4 + 1 - n4 - 1
( n4 + 1 + n4 - 1 )
( n +1 + n
4 4
- 1)
1.7 Comparison Test 1.29

(n 4 + 1) - (n 4 - 1)
=
n4 + 1 + n4 - 1
2
=
n4 + 1 + n4 - 1
1 2
= 2◊
n Ê 1 1 ˆ
Á 1+ 4 + 1- 4 ˜
Ë n n ¯
1
Let vn =
n2
un 2
lim = lim
n Æ• vn nÆ• Ê 1 1 ˆ
Á 1+ 4 + 1- 4 ˜
Ë n n ¯
=2 [finite and non-zero]
1
and  vn =  n2 is convergent as p = 2 > 1.

Hence, by comparison test, Sun is also convergent.

Example 19
• 5n3 - 3n
Check for convergence of the series  .
n =1 n2 (n - 2)(n2 + 5)
[Winter 2016]
Solution
5n 3 - 3n
un =
n2 (n - 2)(n2 + 5)
Ê 3ˆ
n3 Á 5 - 2 ˜
Ë n ¯
=
Ê 2ˆ Ê 5ˆ
n5 Á 1 - ˜ Á 1 + 2 ˜
Ë n¯ Ë n ¯
Ê 3ˆ
ÁË 5 - 2 ˜¯
n
=
2Ê 2ˆ Ê 5ˆ
n Á1 - ˜ Á1 + 2 ˜
Ë n¯ Ë n ¯
1.30 Chapter 1 Sequences and Series

1
Let vn =
n2
Ê 3ˆ
Á 5- 2˜
u Ë n ¯
lim n = lim
n Æ• vn n Æ• Ê 2ˆ Ê 5ˆ
ÁË 1 - n ˜¯ ÁË 1 + 2 ˜¯
n
=5 [finite and non zero]
1
and  vn =  n2 is convergent as p = 2 > 1.

Hence, by comparison text, Â un is also convergent.

Example 20
• È1 Ê n + 1ˆ ˘
Test the convergence of the series Â Í n - log ÁË ˜ .
n ¯ ˙˚
n =1 Î
Solution
1 Ê n + 1ˆ
un = - log Á
Ë n ˜¯
Let
n
1 Ê 1ˆ
= - log Á 1 + ˜
n Ë n¯
1 Ê1 1 1 1 ˆ
= - Á - 2 + 3 - 4 + ˜
n Ë n 2n 3n 4n ¯

1 1 1
= - + -
2 n 2 3n 3 4 n 4
1 Ê1 1 1 ˆ
= 2Á - + 2 - ˜
n Ë 2 3n 4n ¯
1
Let vn =
n2
un Ê1 1 1 ˆ
lim = lim - + - ˜
n Æ• vn nÆ• ÁË 2 3n 4 n2 ¯
1
= [f
[finite and non-zero]
2
1
and Svn = Â is convergent as p = 2 > 1.
n2
Hence, by comparison test, Sun is also convergent.
1.7 Comparison Test 1.31

Example 21 • 1
Test the convergence of the series  sin n .
n =1

Solution
1
Let un = sin
n
1 1 1
= - 3
+ -
n 3! n 5! n5
1Ê 1 1 ˆ
=Á 1- + - ˜
n Ë 3! n 2
5! n 4 ¯
1
Let vn =
n
un Ê 1 1 ˆ
lim = lim Á 1 - + - ˜
n Æ• vn n Æ• Ë 3! n 2
5! n 4 ¯
=1 [finite and non-zero]
1
and Svn = Â is divergent as p = 1.
n
Hence, by comparison test, Sun is also divergent.

Example 22
È • 1 ˘
Test the convergence of the series Â Í n3 + 1
n =1 Í
( ) 3 - n ˙.
˙˚
[Summer 2017]
Î
Solution
È 1 ˘
Let un = Í(n3 + 1) 3 - n ˙
ÍÎ ˙˚
1
Ê 1 ˆ3
= n Á1 + 3 ˜ - n
Ë n ¯
È 1Ê1 ˆ 1Ê1 ˆÊ1 ˆ ˘
Í 1 1 3 ÁË 3 - 1˜¯ 1 2 3 ÁË 3 - 1˜¯ ÁË 3 - 2˜¯ 3 ˙
Ê ˆ Ê 1ˆ
= n Í1 + ◊ 3 + ÁË 3 ˜¯ + ÁË 3 ˜¯ + ˙˙ - n
ÍÎ 3 n 2! n 3! n ˚
1 1 1 1 5 1
= ◊ 2 - 2 ◊ 5 + 4 ◊ 8 -
3 n 3 n 3 n
1.32 Chapter 1 Sequences and Series

1 1 1 1 5 1
2
- 2 3
+ 4
◊ -
n n n6
1
Let vn
n2
un 1 1 1 5 1
lim = - + ◊ -
n Æ• vn n Æ• 3 32 3 4 6

1
and non-zero]
3
1
and the series Svn = Â is convergent as p = 2 > 1.
n2
Hence, by comparison test, Sun is also convergent.

Example 23
È 1 1
Test the convergence of the series  .
n 1

Solution
1 1
Let un 3

1
1 ˘
=n
3 ˙
˙

˘
1 ˙
=n ˙
3 2! ˙

1 1 5
= 2
- 5
+ 8
-
3n 3 9n 3 81n 3
1 Ê1 1 5 ˆ
= 2 Á
- + - ˜
Ë 3 9n 81n2 ¯
n3
1
Let vn 2
n3
1.7 Comparison Test 1.33

un Ê1 1 5 ˆ
lim = lim Á - + - ˜
n Æ• vn n Æ• Ë 3 9n 81n 2 ¯

1
= [finite and non-zero]
3
1 2
and Svn = Â 2
is divergent as p = < 1.
3
n3
Hence, by comparison test, Sun is also divergent.

Example 24

n2 + n + 1 − n2 − n + 1
Test the convergence of the series ∑n =1 n
.

Solution
n2 + n + 1 - n2 - n + 1
Let un =
n
1 1
È Ê 1 1 ˆ ˘2 È Ê 1 1 ˆ ˘2
= Í1 + Á + 2 ˜ ˙ - Í1 + Á - + 2 ˜ ˙
Î Ë n n ¯˚ Î Ë n n ¯˚
Expanding using binomial expansion,
È 1Ê1 ˆ ˘
Í 1 1 1 ÁË 2 - 1˜¯ 1 1 2 ˙
Ê ˆ 2 Ê ˆ
un = Í1 + Á + 2 ˜ + ÁË n + ˜ +  ˙
Í 2Ën n ¯ 2! n2 ¯ ˙
Í ˙
Î ˚
È 1Ê1 ˆ ˘
Í 1 ÁË 2 - 1˜¯ 2 ˙
Ê 1 1 ˆ 2 Ê 1 1 ˆ
- Í1 + Á - + 2 ˜ + Á - + 2 ˜ + ˙
Í 2Ë n n ¯ 2! Ë n n ¯ ˙
Í ˙
Î ˚
È 1 1 1Ê 1 2 1ˆ ˘
= Í1 + + 2 - Á 2 + 3 + 4 ˜ + ˙
Î 2 n 2n 8 Ë n n n ¯ ˚
È 1 1 1Ê 1 2 1ˆ ˘
- Í1 - + 2 - Á 2 - 3 + 4 ˜ + ˙
Î 2 n 2n 8 Ë n n n ¯ ˚
1.34 Chapter 1 Sequences and Series

1 1
= - +
n 2 n3
1Ê 1 ˆ
= Á 1 - 2 + ˜
n Ë 2n ¯

1
Let vn =
n
un Ê 1 ˆ
lim = lim Á 1 - 2 + ˜
n Æ• vn n Æ• Ë 2n ¯
=1 [finite and non-zero]
1
and Svn = Â is divergent as p = 1.
n
Hence, by comparison test, Sun is also divergent.

Example 25
Ê n2 + 1 - n ˆ
Test the convergence of the series  Á ˜.
ÁË np ˜¯
Solution
n2 + 1 - n
Let un =
np
È 1 ˘
Í Ê 1 ˆ2 ˙
n Á1 + 2 ˜ - 1
ÍË n ¯ ˙
ÍÎ ˙˚
= p
n
ÈÏ 1Ê1 ˆ 1Ê1 ˆÊ1 ˆ ¸ ˘
Í -1 -1 - 2˜ Ô ˙
n Í ÔÔ 1 2 ÁË 2 ˜¯ 1 2 ÁË 2 ˜¯ ÁË 2 ¯ 1 Ô
= p Ì1 + 2 + ◊ 4 + ◊ 6 + ˝ - 1˙
n Í Ô 2n 2! n 3! n Ô ˙˙
ÍÔ Ô˛ ˚
ÎÓ
n Ê 1 1 1 ˆ
= Á - + - ˜
n p Ë 2 n2 8n 4 16 n6 ¯
1 Ê1 1 1 ˆ
= ÁË 2 - 2 + - ˜
np +1
8n 16 n 4 ¯
1.7 Comparison Test 1.35

1
Let vn = p +1
n
un Ê1 1 1 ˆ
lim = lim Á - 2 + - ˜
n Æ• vn n Æ• Ë 2 8n 16 n 4 ¯
1
= [finite and non-zeero]
2
1
and Svn = Â p +1
is convergent if p + 1 > 1, i.e., p > 0 and divergent if p + 1 £ 1,
n
i.e., p £ 0.
Hence, by comparison test, Sun is also convergent if p > 0 and divergent if p £ 0.

Example 26
1 1 1 1 1
Test the convergence of the series + + + + +,
x x -1 x +1 x - 2 x + 2
where x is a positive fraction.

Solution
Since it is an infinite series, by ignoring the first term, the series can be rewritten as
Ê 1 1 ˆ Ê 1 1 ˆ
 un = ÁË x - 1 + x + 1˜¯ + ÁË x - 2 + x + 2 ˜¯ + 
2x 2x
= + +
x 2 - 12 x 2 - 22
2x

x 2 - n2
2x
un =
x 2 - n2
2x
=
Ê x2 ˆ
n2 Á 2 - 1˜
Ën ¯

1
Let vn =
n2
un 2x
lim = lim
n Æ• vn nÆ• Ê x 2 ˆ
Á 2 - 1˜
Ën ¯
= –2x [finite and non-zero]
1.36 Chapter 1 Sequences and Series

1
and Svn = is convergent as p = 2 > 1.
n2
Hence, by comparison test, Sun is also convergent.

EXERCISE 1.2
1. Test the convergence of the following series:
1
(i) ∑n 2
+1
(ii) ∑( n +1− n )
(iii) ∑( n4 + 1 − n4 − 1 ) (iv) ∑ 

n +1+ n
np 


n p
1  1
(v) ∑ (n + 1) q
(vi) ∑ n
tan  
 n
 1 1
(vii) ∑ tan −1
  (viii) ∑  b
a+ 
.
n  a+
n
n
 Ans. : 
 
 (i) Convergent (ii) Divergent (iii) Convergent 
 1 1 
(iv) Convergen ntt if p < − D ivergent if p ≥ −
Divergent 
 2 2 
 (v) Convergent if p − q + 1 < 0, DivDive rgent if p − q + 1 ≥ 0 
ergen
(vi) Convergent (vii) Divergent 
 
(viii) Convergent if a > 1, D ivergent if a ≤ 1
Divergent 
2. Test the convergence of the series
((1
1+
1 + a)(
)(1 + b) ((2
2 + a)(
)(2 + b) ((3
3 + a)(
)(3 + b)
+ + +….
1⋅ 2 ⋅ 3 2⋅3⋅4 3⋅ 4 ⋅5
[Ans. : Divergent]

1.8 D’ALEMBERT’S RATIO TEST

un +1
If Sun is a positive-term series and lim = l then
n Æ• un

(i) Sun is convergent if l < 1

(ii) Sun is divergent if l > 1


1.8 D’Alembert’s Ratio Test 1.37

Proof
un +1
Case I If lim = l < 1.
n Æ• un

un +1
Consider a number l < r < 1 such that < r for all n > m … (1)
un
Neglecting the first m terms,

 un = um ++11 + um + 2 + um + 3 + … •
n m +1
n=

Ê u u u ˆ
= um +1 Á 1 + m + 2 + m + 3 + m + 4 + …˜
Ë um +1 um +1 um +1 ¯
Ê u u u u u u ˆ
= um +1 Á 1 + m + 2 + m + 3 ◊ m + 2 + m + 4 ◊ m + 3 ◊ m + 2 + …˜
Ë um +1 um + 2 um +1 um + 3 um + 2 um +1 ¯
< um +1
+1 (1 + r + r ◊ r + r ◊ r ◊ r + …) [[Usin
Using Eq. (1)]
= um +1
+1 (1 + r + r + r + …)
2 3

1
= um + 1 ◊ (r < 1)
1-r

• um + 1
\ Â un <
1-r
(f
(finite)
n m +1
n=


Thus, the series  un is convergent.
n = m +1

The nature of a series remains unchanged if we neglect a finite number of terms in the

beginning. Hence, the series  un is convergent.
n=1

un +1
Case II If lim = l > 1,
n Æ• un

un +1
> 1 for all n > m ... (2)
un
1.38 Chapter 1 Sequences and Series

Neglecting the first m terms,



 un um m m m+4 + •
n m +1

Ê um + 2 u u ˆ
= um + + + +4 + …
um +1 um +1 um +1
Ê um + 2 u u u u u
= um + + ◊ m+2 + m+ 4 m+3 +…
um +1 um + 2 um +1 um + 3 um + 2 um +1
> um +1 (1+ + + + …)

\ o terms)
Sn m n
lim m 1 is positive]
n n Æ•

Thus, the series  un is divergent.
n m +1

The nature of a series remains unchanged if we neglect a finite number of terms in the

beginning. Hence, the series  un is divergent.
n
un +1
Note 1: If lim = 1 the ratio test fails, i.e. no conclusion can be drawn about the
n Æ• un
convergence or divergence of the series.
Note 2: It is convenient to use D’Alembert’s ratio test in the following form:
u
If Sun is a positive term series and lim n = l, then
n Æ• un +1

(i) Sun is convergent if l > 1


(ii) Sun is divergent if l < 1
(iii) The ratio test fails if l = 1

Example 1
• 32 n
Test the convergence of the series  . [Summer 2014]
n£0 23 n
Solution
32 n
Let un =
23 n
1.8 D’Alembert’s Ratio Test 1.39

32( n +1) 32 n + 2
un +1 = =
23( n +1) 23 n + 3

un 32 n 23n + 3
= 3n ◊ 2 n + 2
un +1 2 3
23 8
= 2
=
3 9

un 8 8
lim = lim = < 1
n ƕ un +1 nƕ 9 9
Hence, by D’Alembert’s ratio test, the series is divergent.

Example 2
•5n -1
Test the convergence of the series  .
n =1 n !
Solution
5n -1
Let un =
n!
5n
un +1 =
(n + 1)!
n -1
un 5n- 1
(n + 1)!
= ◊
un +1 n! 5n
n +1
=
5
un n +1
lim = lim Æ • >1
n Æ• un +1 n Æ• 5

Hence, by D’Alembert’s ratio test, the series is convergent.

Example 3
• 2n
Test the convergence of the series  . [Winter 2016]
n =1 n + 1
3

Solution
2n
Let un =
n3 + 1
2 n +1
un +1 =
(n + 1)3 + 1
1.40 Chapter 1 Sequences and Series

un 2 n (n + 1)3 + 1
= 3 ◊
un +1 n + 1 2 n +1
3
Ê 1ˆ 1
ÁË 1 + n ˜¯ + 3 1
n
= ◊
1 2
1+ 3
n
3
Ê 1ˆ 1
Á 1+ ˜ + 3
u Ë n¯ n 1
lim n = lim ◊
n Æ• un +1 n Æ• 1 2
1+ 3
n
1
= <1
2
Hence, by D’Alembert’s ratio test, the series is divergent.

Example 4
n!
Test the convergence of the series  .
nn
Solution
n!
Let un =
nn
(n + 1)!
un +1 =
(n + 1)n +1
n!
un nn
=
un +1 (n + 1)!
(n + 1)n +1
(n + 1)(n + 1)n
=
(n + 1)n n
n
Ê 1ˆ
= Á1 + ˜
Ë n¯
n
un Ê 1ˆ
lim = lim Á 1+
1+ ˜
Æ• un +1
nƕ
n n Æ• Ë n¯
= e >1
Hence, by D’Alembert’s ratio test, the series is convergent.
1.8 D’Alembert’s Ratio Test 1.41

Example 5
n !(2) n
Test the convergence of the series ∑ nn .
Solution
n !(2)n
Let un =
nn
)!(2)n +1
(n + 1)!(
un +1 =
(n + 1)n +1
un n ! 2 n (n + 1)n +1
= n ◊
un +1 n )! 2 n +1
(n + 1)!
n
1 Ê n + 1ˆ
=
2 ÁË n ˜¯
n
un 1 Ê 1ˆ
lim = lim Á 1 + ˜
Æ• un +1
nnÆ• 2 n Æ• Ë n¯
e
=
>1
2
Hence, by D’Alembert’s ratio test, the series is convergent.

Example 6
• n3
Test the convergence of the series  .
n =1 ( n - 1)!
Solution
n3
Let un =
(n -1
- 1)!

(n + 1)3
un +1 =
n!
un n3 n!
= ◊
un +1 (n -1
- 1)! (n + 1)3
n3 n(n -1
- 1)!
= ◊
(n -1
- 1)! (n + 1)3
n
= 3
Ê 1ˆ
ÁË 1 + n ˜¯
1.42 Chapter 1 Sequences and Series

un n
lim = lim Æ • >1
n Æ• un +1 nÆ• Ê 1 ˆ 3
ÁË 1 + n ˜¯

Hence, by D’Alembert’s ratio test, the series is convergent.

Example 7

(n + 1) n
Test the convergence of the series ∑ .
n =1 n!
Solution
(n + 1)n
Let un =
n!
(n + 2)n +1
un +1 =
(n + 1)!
un (n + 1)n (n + 1)!
= ◊
un +1 n! (n + 2)n +1
(n + 1)n n + 1)(n !)
((n
=
n! [(n + 1) + 1]n+1
1
= n +1
Ê 1 ˆ
ÁË 1 + n + 1˜¯

un 1
lim = lim n +1
n Æ• un +1 nÆ• Ê 1 ˆ
ÁË 1 +
n + 1˜¯
1
<1 =
e
Hence, by D’Alembert’s ratio test, the series is divergent.

Example 8 • 2n + 1
Test the convergence of the series  n .
n =1 3 + 1
Solution
2n + 1
Let un =
3n + 1
n +1
2 n+ +1
un +1 = n +1
3n+ +1
un Ê 2 n + 1ˆ Ê 3n+
n +1
+ 1ˆ
=Á ÁË n+ ˜
un +1 Ë 3n + 1 ˜¯ 2 + 1¯
n +1
1.8 D’Alembert’s Ratio Test 1.43

Ê 1 ˆÊ 1ˆ
ÁË 1 + n ˜¯ ÁË 3 + n ˜¯
2 3
=
Ê 1 ˆÊ 1ˆ
ÁË 1 + n ˜¯ ÁË 2 + n ˜¯
3 2
Ê 1 ˆÊ 1ˆ
Á 1+ n ˜ Á3 + n ˜
u Ë 2 ¯Ë 3 ¯
lim n = llim im
n Æ• un +1 n Æ• Ê 1 ˆÊ 1ˆ
ÁË 1 + n ˜¯ ÁË 2 + n ˜¯
3 2
3
= >1
2
Hence, by D’Alembert’s ratio test, the series is convergent.

Example 9
• 5n + a
Test the convergence of the series  , a > 0, b > 0.
0
n =1 3
n
+b
Solution
5n + a
Let un =
3n + b
5n +1 + a
un +1 =
3n +1 + b
un 5n + a 3n +1 + b
lim = lim n ◊
n Æ• un +1 n Æ• 3 + b 5n +1 + a

a b
n
1+3+ n
= lim 5 ◊ 3
n Æ• b a
1+ n 5 + n
3 5
3
= <1
5
Hence, by D’Alembert’s ratio test, the series is divergent.

Example 10
2! 3! 4!
Test the convergence of the series + + + .
3 32 33
Solution
(n + 1)!
Let un =
3n
1.44 Chapter 1 Sequences and Series

(n + 2)!
un +1 =
3n +1
un (n + 1)! 3n +1
lim = lim ◊
Æ• un +1
nƕ
n n Æ• 3n (n + 2)!
Ê 3 ˆ
= lim Á ˜
n Æ• Ë n + 2 ¯

= 0 <1
Hence, by D’Alembert’s ratio test, the series is divergent.

Example 11
4 4 2 43 4 4
Test the convergence of the series 1 + + + + +.
2 ! 3! 4 ! 5!
Solution
4 n -1
Let un =
n!
4n
un +1 =
(n + 1)!
n -1
un 4 n- 1
(n + 1)!
= ◊
un +1 n! 4n
((n + 1)
=
4
un ((n + 1)
lim = lim Æ • >1
n Æ• un +1 n Æ• 4
Hence, by D’Alembert’s ratio test, the series is convergent.

Example 12
22 32 42
Test the convergence of the series 1 + + + + .
2 ! 3! 4 !
Solution
n2
Let un =
n!
(n + 1)2
un +1 =
(n + 1)!
un n2 (n + 1)!
= ◊
un +1 n ! (n + 1)2
1.8 D’Alembert’s Ratio Test 1.45

n2 (n + 1) ◊ n !
=
n !(n + 1)2
n2
=
n +1
n
=
1
1+
n
un n
lim = lim Æ • >1
n Æ• un +1 n Æ• 1
1+
n
Hence, by D’Alembert’s ratio test, the series is convergent.

Example 13
2 p 3p 4 p
Test the convergence of 1 + + + + … ,( p > 0).
2! 3! 4!
Solution
np
Let un =
n!
(n + 1) p
un +1 =
(n + 1)!
np
un n!
lim = lim
Æ•
nnƕ un +1 n ƕ ( n + 1) p

(n + 1)!
np (n + 1)!
= lim ◊
n Æ• (n + 1) p n!
(n + 1)
= lim p
Æ • >1
n Æ•
Ê 1ˆ
ÁË 1 + n ˜¯

Hence, by D’Alembert’s ratio test, the series is convergent.

Example 14
1 2 3 4
Test the convergence of the series + + + + .
1 + 2 1 + 2 1 + 2 1 + 24
2 3
1.46 Chapter 1 Sequences and Series

Solution
n
Let un =
1 + 2n
n +1
un +1 =
1 + 2 n +1
un n 1 + 2 n +1
= ◊
un +1 1 + 2n n + 1
1
+2
= 2n
Ê 1 ˆ Ê 1ˆ
ÁË n + 1˜¯ ÁË 1 + n ˜¯
2
1
+2
un 2n
lim = lim
n Æ• un +1 n Æ• Ê 1 ˆ Ê 1ˆ
ÁË n + 1˜¯ ÁË 1 + n ˜¯
2
È∵ lim 2 n Æ • ˘
=2>1 ÍÎ nÆ• ˙˚
Hence, by D’Alembert’s ratio test, the series is convergent.

Example 15
• 2 ◊ 4 ◊ 6 … 2n
Test the convergence of the series  5 ◊ 8 ◊11…(3n + 2).
n =1

Solution
2 ◊ 4 ◊ 6…
6 … 2n
Let un = [Using A.P.]
5 ◊ 8 ◊ 11… (3n + 2)
2 ◊ 4 ◊ 6…
6 … 2 n(2 n + 2)
un +1 =
5 ◊ 8 ◊ 11… (3n + 2)(3n + 5)
un 2 ◊ 4 ◊ 6…
6 … 2n 5 ◊ 8 ◊ 11… (3n + 2)(3n + 5)
= ◊
un +1 5 ◊ 8 ◊ 11… (3n + 2) 2 ◊ 4 ◊ 6…2
2nn(2 n + 2)
33n + 5
=
2n + 2
2n
5
3+
= n
2
2+
n
1.8 D’Alembert’s Ratio Test 1.47

5
un 3+
lim = lim n
n Æ• un +1 n Æ• 2
2+
n
3
= >1
2
Hence, by D’Alembert’s ratio test, the series is convergent.

Example 16
2 2 ◊ 5 2 ◊ 5 ◊ 8 2 ◊ 5 ◊ 8 ◊ 111
Test the convergence of the series + + + + .
1 1 ◊ 5 1 ◊ 5 ◊ 9 1 ◊ 5 ◊ 9 ◊ 113
Solution
2 ◊ 5 ◊ 8 ◊ 11… (3n -1
- 1)
Let un = [Using A.P.]
1 ◊ 5 ◊ 9 ◊ 13… (4 n - 3)
2 ◊ 5 ◊ 8 ◊ 11… (3n -1 - 1)(3n + 2)
un+1 =
1 ◊ 5 ◊ 9 ◊ 13… (4 n - 3)(4 n + 1)
2 ◊ 5 ◊ 8 ◊ 11…
1… (3n - 1)
un 1 ◊ 5 ◊ 9 ◊ 13…
3… (4 n - 3)
lim = lim
n Æ• un +1 n Æ• 2 ◊ 5 ◊ 8 ◊ 11…(((3n - 1))(((3n + 2)
1 ◊ 5 ◊ 9 ◊ 133… (4 n - 3)(
13… )(4 n + 1)
44nn + 1
= lim
n Æ• 33n + 2
1
4+
= lim n
n Æ• 2
3+
n
4
= >1
3
Hence, by D’Alembert’s ratio test, the series is convergent.

Example 17 2 2 2
Ê 1ˆ Ê 1◊ 2 ˆ Ê 1◊ 2 ◊ 3 ˆ
Test the convergence of the series Á ˜ + Á ˜ +Á + … •.

Ë 3¯ Ë 3 ◊ 5¯ Ë 3 ◊ 5 ◊ 7 ˜¯
Solution
2
È 1 ◊ 2 ◊ 3… n ˘
Let un = Í ˙ [Using A.P.]
Î 3 ◊ 5 ◊ 7 … (2 n + 1) ˚
2
È 1 ◊ 2 ◊ 3 … n(n + 1) ˘
un +1 = Í ˙
Î 3 ◊ 5 ◊ 7 … (2 n + 1)(2 n + 3) ˚
1.48 Chapter 1 Sequences and Series

2
È 1 ◊ 2 ◊ 3… n ˘
Í 3 ◊ 5 ◊ 7 … ((2 n + 1) ˙
lim
un
= lim Î ˚
n Æ• un +1 n Æ• 2
È 1 ◊ 2 ◊ 3…
3… nn((n + 1) ˘
Í 3 5 ◊ 7 … (2 n + 1)((2
2 n + 3) ˙˚
Î
2
2 + 3˘
È 2n
= lim Í
n Æ• Î n + 1 ˙
˚
2
È 3˘
Í2 + n ˙
= lim Í ˙
n Æ•
Í1+ 1 ˙
ÍÎ n ˙˚
= 4 >1

Hence, by D’Alembert’s ratio test, the series is convergent.

Example 18
3 4 5
Test the convergence of the series 2 + x + x 2 + x 3 + .
2 3 4
[Summer 2014]
Solution
n + 1 n -1
Let un = x
n
n+2 n
un +1 = x
n +1
un (n + 1) x n -1 n +1
= ◊
un +1 n ( n + 2) x n
1 1
1+ 1+
= n n ◊1
1 2 x
1+
n
1 1
un 1+ 1+
lim = lim n◊ n ◊1
n Æ• un +1 n Æ• 1 2 x
1+
n
1
=
x
1.8 D’Alembert’s Ratio Test 1.49

By D’Alembert’s ratio test, the series is


1
(i) convergent if > 1 or x < 1
x
1
(ii) divergent if < 1 or x > 1
x
1
The test fails if = 1, i.e., x = 1.
x
n +1 1
Then un = = 1+
n n
Ê 1ˆ
lim un = lim Á 1 + ˜
n Æ• n Æ• Ë n¯
=1 [finite and non-zero]
Sun is divergent for x = 1.
Hence, the series is convergent for x < 1 and is divergent for x ≥ 1.

Example 19
• (n + 1)
Test the convergence of the series  2
xn.
n =1 n
Solution
n +1
Let un = xn
n2
n+2
un +1 = x n +1
(n + 1)2
un (n + 1) x n (n + 1)2
= ◊
un +1 n2 (n + 2) x n +1
(n + 1)3
=
n2 ((nn + 2) x
3
Ê 1ˆ
ÁË 1 + n ˜¯ 1
= ◊
Ê 2ˆ x
ÁË 1 +
n ˜¯
3
Ê 1ˆ
un ÁË 1 + n ˜¯ 1
lim = lim ◊
n Æ• un +1 n Æ• Ê 2ˆ x
ÁË 1 + n ˜¯

1
=
x
1.50 Chapter 1 Sequences and Series

By D’Alembert’s ratio test, the series is


1
(i) convergent if > 1 or x < 1
x
1
(ii) divergent if < 1 or x > 1
x
1
The test fails if = 1, i.e., x = 1.
x
n +1
Then un =
n2
1 Ê 1ˆ
= 1+ ˜
n ÁË n¯

Let 1
vn =
n
un  1
lim
m = lim
lim 1 + 
n →∞ v n →∞  n
n

=1 [finite and non-zero]


1
and Svn = Â is divergent as p = 1.
n
By comparison test, Sun is also divergent for x = 1.
Hence, the series is convergent for x < 1 and is divergent for x ≥ 1.

Example 20
Ê xn ˆ •
Test the convergence of the series  Á 2 ˜ , for x > 0.
n =1 Ë n + 1¯
Solution
xn
Let un =
n2 + 1

x n +1
un +1 =
(n + 1)2 + 1
un x n (n + 1)2 + 1
= 2 ◊
un +1 n + 1 x n +1
2
Ê 1ˆ 1
ÁË 1 + n ˜¯ + 2 1
n
= ◊
1 x
1+ 2
n
1.8 D’Alembert’s Ratio Test 1.51

2
Ê 1ˆ 1
un ÁË 1 + n ˜¯ + 2 1
n
lim = lim ◊
n Æ• un +1 n Æ• 1 x
1+ 2
n
1
=
x
By D’Alembert’s ratio test, the series is
1
(i) convergent if > 1 or x < 1
x
1
(ii) divergent if < 1 or x > 1
x
The test fails if x = 1.
1
Then un =
n +12

1
=
Ê 1ˆ
n2 Á 1 + 2 ˜
Ë n ¯
1
Let vn = 2
n
un 1
lim = lim
n Æ• vn n Æ• Ê 1ˆ
ÁË 1 + 2 ˜¯
n
=1 [finite and non-zero]
1
and Svn = Â is convergent as p = 2 > 1.
n2
By comparison test, Sun is also convergent if x = 1.
Hence, the series is convergent for x £ 1 and is divergent for x > 1.

Example 21
2n
Test the convergence of the series  x n , x > 0.
n +1
4

Solution
2n
Let un = xn
n4 + 1
1.52 Chapter 1 Sequences and Series

2 n +1
un +1 = x n +1
(n + 1)4 + 1
un 2 n x n (n + 1)4 + 1
= ◊
un +1 n 4 + 1 2 n +1 x n +1
4
Ê 1ˆ 1
ÁË 1 + n ˜¯ + 4
n
=
Ê 1ˆ
ÁË 1 + 4 ˜¯ 2 x
n
4
Ê 1ˆ 1
un ÁË 1 + n ˜¯ + 4
n
lim = lim
n Æ• un +1 n Æ• Ê 1ˆ
ÁË 1 + 4 ˜¯ 2 x
n
1
=
2x
By D’Alembert’s ratio test, the series is

(i) convergent if 1 > 1 or x < 1


2x 2

(ii) divergent if 1 < 1 or x > 1


2x 2

The test fails if 1 = 1 or x = 1 .


2x 2
2n 1
Then un = ◊
n + 1 2n
4

1
=
n4 + 1
1
=
Ê 1ˆ
n4 Á1 + 4 ˜
Ë n ¯
1
Let vn =
n4
un 1
lim = lim
n Æ• vn nÆ• Ê 1ˆ
ÁË 1 + 4 ˜¯
n
=1 [finite and non-zero]
1
and Σvn = ∑ is convergent as p = 4 > 1.
n4
1.8 D’Alembert’s Ratio Test 1.53

1
By comparison test, Sun is also convergent if x = .
2
1 1
Hence, the series is convergent for x £ and is divergent for x > .
2 2

Example 22
n
Test the convergence of the series ∑ n +1
2
xn .

Solution
n
Let un = ◊ xn
n +1
2

(n + 1)
un +1 = ◊ x n +1
(n + 1)2 + 1
un n (n + 1)2 + 1 1
= ◊ xn ◊ n +1
un +1 n +1
2 n +1 x
n (n2 + 2 n + 2) 1
= ◊ ◊
(n + 1) (n2 + 1) x

Ê 2 2ˆ
un ÁË 1 + n + 2 ˜¯ 1
n
lim = lim ◊
Æ• un +1
nƕ
n n Æ• Ê 1ˆ Ê 1ˆ x
ÁË 1 + n ˜¯ ÁË 1 + 2 ˜¯
n
1
=
x
By D’Alembert’s ratio test, the series is
1
(i) convergent if > 1, or x < 1
x
1
(ii) divergent if < 1, orr x >1
x
The test fails for x = 1.
n
Then un =
n2 + 1
1
n2
=
1
n 1+
n2
1.54 Chapter 1 Sequences and Series

1
= 1
1
n2 1+
n2
1
Let vn = 1
n2
un 1
lim = lim
n Æ• vn n Æ• 1
1+
n2
= 1 [[finite
finite and non-zero]

1 1
and Svn = Â 1
is divergent for p = <1
2
n2
By comparison test, Sun is also divergent for x = 1.
Hence, the series is convergent for x < 1 and is divergent for x ≥ 1.

Example 23
x x2 x3 x4
Test the convergence of the series + + + + .
x +1 x + 2 x + 3 x + 4
Solution
xn
Let un =
x+n
x n +1
un +1 =
x + n +1
un xn x + n + 1
= ◊
un +1 x + n x n +1
x 1
+1+
= n n ◊1
x x
+1
n
x 1
un +1+
lim = lim n n ◊1
n Æ• un +1 n Æ• x x
+1
n
1
=
x
1.8 D’Alembert’s Ratio Test 1.55

By D’Alembert’s ratio test, the series is


1
(i) convergent if > 1 or x < 1
x
1
(ii) divergent if < 1 or x > 1
x
1
The test fails if = 1 or x = 1.
x
1
Then un =
1+ n
1
=
1 
n  + 1
n 
1
Let vn =
n
un 1
lim = lim
n Æ• vn n Æ• Ê 1 ˆ
ÁË n + 1˜¯
= 1 [finite and non-zero]
1
and Σvn = ∑ is divergent as p = 1.
n
By comparison test, Sun is also divergent for x = 1.
Hence, the series is convergent for x < 1 and is divergent for x ≥ 1.

Example 24
x x2 x3 x4
Test the convergence of the series + + + +.
1◊ 2 3 ◊ 4 5 ◊ 6 7 ◊ 8
Solution
xn
Let un =
((22 n - 1)2 n
x n +1
un +1 =
(2 n + 1)(2 n + 2)
un xn (2 n + 1)(2 n + 2)
= ◊
un +1 (2 n - 1)2 n x n +1
Ê 1ˆ
ÁË 2 + n ˜¯
Ê 1ˆ 1
= ÁË 1 + n ˜¯ ◊ x
Ê 1ˆ
ÁË 2 - n ˜¯
1.56 Chapter 1 Sequences and Series

Ê 1ˆ
ÁË 2 + n ˜¯
un Ê 1ˆ 1
lim = lim ÁË 1 + n ˜¯ ◊ x
n Æ• un +1 n Æ• Ê 1ˆ
ÁË 2 - n ˜¯

1
=
x
By D’Alembert’s ratio test, the series is
1
(i) convergent if > 1 or x < 1
x
1
(ii) divergent if < 1 or x > 1
x
1
The test fails if = 1 or x = 1.
x
1
Then un =
(2n − 1)2n
1
=
 1
2 2 2− 
2n
 n
1
Let vn =
n2
un 1
lim
m = lim
n →∞ vn n→∞  1
22 − 
 n
1
= [[finite
finite and non-zero]
4
1
and Σvn = ∑ is convergent as p = 2 > 1.
n2
By comparison test, Sun is also convergent.
Hence, the series is convergent for x £ 1 and is divergent for x > 1.

Example 25
1 x x2
Test the convergence of the series + + +.
1◊ 2 ◊ 3 4 ◊ 5 ◊ 6 7 ◊ 8 ◊ 9
[Summer 2017]
1.8 D’Alembert’s Ratio Test 1.57

Solution
xn - 1
Let un =
(3n - 2)(3n - 1) 3n
xn
un + 1 =
(3n + 1) (3n + 2)(3n + 3)
un xn - 1 (3n + 1) (3n + 2) (3n + 3)
= ◊
un + 1 (3n - 2) (3n - 1) (3n) xn
Ê 1ˆ Ê 2ˆ Ê 3ˆ
ÁË 3 + n ˜¯ ÁË 3 + n ˜¯ ÁË 3 + n ˜¯ 1
= ◊
Ê 2ˆ Ê 1ˆ x
ÁË 3 - 3 - 3
n ˜¯ ÁË n ˜¯
Ê 1ˆ Ê 2ˆ Ê 3ˆ
un ÁË 3 + n ˜¯ ÁË 3 + n ˜¯ ÁË 3 + n ˜¯ 1
lim = lim ◊
n Æ • un + 1 nÆ• Ê 2ˆ Ê 1ˆ x
ÁË 3 - 3 - 3
n ˜¯ ÁË n ˜¯
1
=
x
By D¢ Alembert’s ratio text, the series is
1
(i) convergent if > 1 or x < 1
x
1
(ii) divergent if < 1 or x > 1
x
1
The test fails if = 1 or x = 1 .
x
1
Then un =
(3n - 2)(3n - 1)(3n)
1
=
Ê 2ˆ Ê 1ˆ
n3 Á 3 - ˜ Á 3 - ˜ 3
Ë n¯ Ë n¯
1
Let vn =
n3
un 1
lim = lim
n Æ• vn n Æ• Ê 2ˆ Ê 1ˆ
ÁË 3 - n ˜¯ ÁË 3 - n ˜¯ 3

1
= [finite and non-zero]
27
1.58 Chapter 1 Sequences and Series

1
and  vn =  n3 is convergent as p = 3 > 1.

By comparison test, Â un is also convergent for x = 1.


Hence, the series is convergent for x £ 1 and is divergent for x > 1.

Example 26
3 4 (n + 1)
Test the convergence of the series 2 x + x 2 + x 3 +  + 3 x n + .
8 27 n
Solution
n +1
Let un = ◊ xn
n3
n+2
un +1 = ◊ x n +1
(n + 1)3
un (n + 1) x n (n + 1)3
= ◊
un +1 n3 (n + 2) x n +1
4
Ê 1ˆ
ÁË 1 + n ˜¯ 1
= ◊
Ê 2ˆ x
ÁË 1 + n ˜¯
4
Ê 1ˆ
un ÁË 1 + n ˜¯ 1
lim = lim ◊
n Æ• un +1 n Æ• Ê 2ˆ x
ÁË 1 +
n ˜¯
1
=
x
By D’Alembert’s ratio test, the series is
1
(i) convergent if > 1 or x < 1
x
1
(ii) divergent if < 1 or x > 1
x
1
The test fails if = 1 or x = 1 .
x
n +1
Then un =
n3
1 Ê 1ˆ
= 2 Á1 + ˜
n Ë n¯
1.8 D’Alembert’s Ratio Test 1.59

1
Let vn =
n2
un Ê 1ˆ
lim = lim 1 + ˜
n Æ• vn nÆ• ÁË n¯
=1 [finite and non-zero]
1
and Svn = Â is convergent as p = 2 > 1.
n2
By comparison test, Sun is also convergent for x = 1.
Hence, the series is convergent for x £ 1 and is divergent for x > 1.

Example 27
x2 x4 x6 1
Test the convergence of the series + + + + .
2 1 3 2 4 3 5 4
[Winter 2013]
Solution
n−2
x 2 n−
Let un =
(n + 1) n
x2n
u n +1 =
( n + 2) n + 1
n−2
un x 2 n− ( n + 2) n + 1
= ⋅
un +1 (n + 1) n x2n
( n + 2) n + 1 1
= ⋅
(n +1
+ 1) n x2
Ê 2ˆ
ÁË 1 + n ˜¯ 1 1
= 1+ ◊ 2
Ê 1ˆ n x
ÁË 1 + n ˜¯

Ê 2ˆ
un ÁË 1 + n ˜¯ 1 1
lim = lim 1+ ◊ 2
n Æ• un +1 n Æ• Ê 1ˆ n x
ÁË 1 + n ˜¯

1
=
x2
By D’Alembert’s ratio test, the series is
1
(i) convergent if 2 > 1 or x2 < 1
x
1
(ii) divergent if 2 < 1 or x > 1
2
x
1.60 Chapter 1 Sequences and Series

1
The test fails if 2
= 1 or x 2 = 1.
x
1
Then un =
(n + 1) n
1
= 3
Ê 1ˆ
n2 ÁË 1 + n ˜¯
1
Let vn = 3
n2
un 1
= lim lim
vn nÆ• Ê 1 ˆ
n Æ•
ÁË 1 + n ˜¯
= 1 [finite and non-zero]
1 3
and Svn = 3 is convergent as p = > 1.
2
n2
By comparison test, Sun is also convergent for x2 = 1.
Hence, the series is convergent for x2 £ 1 and is divergent for x2 > 1.

Example 28
3 5 7 3 9 4
Test the convergence of the series 1 + x + x2 + x + x + .
2 9 28 65
Solution
2n + 1
Let un = xn [Neglecting the first term]
n +1
3

2n + 3
un +1 = x n +1
(n + 1)3 + 1
un (2 n + 1) x n [(n + 1 1))3 + 1]
= ◊
un +1 n 3 + 1 (2 2n n + 3) x n +1

1 ˆ ÈÊ 1 ˆ 1˘
3
Ê
ÁË 2 + ˜ Í Á 1 + ˜ + 3˙
n ¯ ÍË n¯ n ˙˚
= Î
Ê 3ˆ Ê 1ˆ
ÁË 2 + n ˜¯ ÁË 1 + 3 ˜¯ x
n
Ê È
1ˆ Ê 1ˆ
3

ÁË 2 + n ˜¯ ÍÁË 1 + n ˜¯ + 3 ˙
un ÍÎ n ˙˚
lim
m = lim
n Æ• un +1 n Æ• Ê 3ˆ Ê 1ˆ
ÁË 2 + n ˜¯ ÁË 1 + 3 ˜¯ x
n
1.8 D’Alembert’s Ratio Test 1.61

1
=
x
By D’Alembert’s ratio test, the series is
1
(i) convergent if > 1 or x < 1
x
1
(ii) divergent if < 1 or x > 1
x
The test fails if x = 1.
2n + 1
Then un =
n3 + 1
1
2+
= n
2  1
n 1 + 3 
 n 
1
Let vn =
n2
1
2+
un n
m = lim
lim
n →∞ v n →∞  1 
1 + 3 
n
n
=2 [finite and non-zero]
1
and Svn = Â is convergent as p = 2 > 1.
n2
By comparison test, Sun is also convergent if x = 1.
Hence, the series is convergent for x £ 1 and is divergent for x > 1.

EXERCISE 1.3
Test the convergence of the following series:

2 2 32 4 2
1. 1 + + + + …∞
2! 3! 4!
[Ans.: Convergent]

4 ⋅ 7 ⋅ 10 …(3n + 1)
2. ∑
n =1 1⋅ 2 ⋅ 3 ⋅ 4 … n
[Ans.: Convergent]
1 2 3
3. + + + …∞
1 5 1+ 5
1+ 2
1 + 53
[Ans.: Convergent]
2! 3! 4!
4. 1 + + + + …∞ [Ans.: Convergent]
22 33 4 4
1.62 Chapter 1 Sequences and Series

2 3 4
5. + + + [Ans.: Convergent]
3! 4 ! 5!

3 32 33 34
6. 1 + + + + + [Ans.: Convergent]
2 ! 3! 4 ! 5!


n2
7. ∑3
n =1
n

[Ans.: Convergent]
∞ n −1
2
8. ∑3
n =1
n
+1
[Ans.: Convergent]


1
9. ∑ n!
n =1
[Ans.: Convergent]

n (n + 1)
2 2

10. ∑
n =1 n! [Ans.: Convergent]


3n + 4 n
11. ∑ n [Ans.: Divergent]
n =1 4 + 5
n


xn
12. ∑ n 2 , x > 0
n =1 3 ⋅ n
[Ans.: Convergent for x < 3, divergent for x > 3]

3 − 2 n −1
n

13. ∑3
n =1
n
+1
⋅ x , x>0
[Ans.: Convergent for x < 1, divergent for x > 3]

xn
14. ∑ (2 )!
n =1
n

[Ans.: Convergent]

n +1 n
15. ∑
n =1 n3 + 1
⋅x , x > 0

[Ans.: Convergent for x < 1, divergent for x > 1]

16. x + 2 x 2 + 3x 3 + 4 x 4 + … ∞
[Ans.: Convergent for x < 1, divergent for x > 1]
2 3
x x x xn
17. 1 + + + +…+ 2 + …∞
2 5 10 n +1
[Ans.: Convergent for x < 1, divergent for x > 1]
1.9 Cauchy’s Root Test 1.63

18. x x2 x3
+ + + …∞
1⋅ 3 3 ⋅ 5 5 ⋅ 7
[Ans.: Convergent for x < 1, divergent for x > 1]

3 2 8 3 15 4 n2 − 1 n
19. x + x + x + x +… + 2 x + …∞
5 10 17 n +1
[Ans.: Convergent for x < 1, divergent for x > 1]

x x2 x3
20. + + + …∞
2 3 3 4 4 5
[Ans.: Convergent for x < 1, divergent for x > 1]

1.9 CAUCHY’S ROOT TEST

1
If Sun is a positive term series and if lim (un ) n = l then
n Æ•

(i) Sun is convergent if l < 1.

(ii) Sun is divergent if l > 1.


Proof
1
Case I If lim (un ) n = l < 1.
n Æ•
1
Consider a number l < r < 1 such that (un ) n < r for all n > m

un < r n for all n > m ... (1)

The geometric series Sr n = r + r 2 + r 3 + …∞

Sn = r + r 2 + r 3 + … + r n
r (1 - r n )
=
1- r
r (1 - r n )
lim Sn = lim
n Æ• n Æ• 1 - r È∵r < 1 ˘
r Í ˙
ÍÎ\ xlim r = 0˙
n
= , which is finite
1- r Æ• ˚

Hence, the series Sr n is convergent.


From Eq. (1), un < r n for all n > m
1.64 Chapter 1 Sequences and Series

Sun < Sr n

Since Sr n is convergent, Sun is also convergent.


1
Case II: If lim ( ) 1.
n Æ•
1
(un ) n > 1 for all n > m … (2)

Neglecting the first m terms,


1 1 1 1
S( + …∞
> 1… ∞ [Using Eq. (2)]

1 1 1 1
S m n

> erms n

lim Sn > lim n Æ •


n Æ• n Æ•


The series  un is divergent. The nature of a series remains unchanged if we
n = m +1

neglect a finite number of terms in the beginning. Hence, the series  un is divergent.
n 1

1
Note: If lim( ) 1, the root test fails, i.e., no conclusion can be drawn about the
n→∞
convergence or divergence of the series.

Example 1
• 1
Test the convergence of the series  n
n 1 (log n)
Solution
1
Let un =
(log n)n
1
1
( )n
log n
1
1
lim ( ) n = lim
n Æ• n Æ• log n
= 1 [ log • Æ •]
1.9 Cauchy’s Root Test 1.65

Hence, by Cauchy’s root test, the series is convergent.

Example 2

a n +1
Test the convergence of the series ∑
n =1 n
n
.

Solution
a n +1
Let un =
nn
1
1 1+
n
( )
(a
( n) =
(u n
n
1
1
a an
lim(un ) = lim n
n →∞ n →∞ n
= 0 <1
Hence, by Cauchy’s root test, the series is convergent.

Example 3
1
Test the convergence of the series  n2
.
Ê 1ˆ
ÁË 1
1++ ˜

Solution
1
Let un =
n2
Ê 1ˆ
ÁË 1 + n ˜¯

1
1
( n )n =
(u n
Ê 1ˆ
ÁË 1
1++ ˜

1
1
lim (un ) n = lim n
n Æ• n Æ•
Ê 1ˆ
ÁË 1
1++ ˜

1
= <1
e

Hence, by Cauchy’s root test, the series is convergent.


1.66 Chapter 1 Sequences and Series

Example 4 (n - log n)n


Test the convergence of the series  .
2n ◊ nn
Solution
(n - log n)n
Let un =
2n ◊ nn
1
(n - log n) È• ˘
lim (un ) n = lim Í • form ˙
n Æ•
nÆ• n Æ• 2n Î ˚
Ê 1 log n ˆ
= lim Á -
n Æ• Ë 2 2 n ˜¯
1
1
= - lim n [Using L’Hospital’s rule]
2 nƕ 2
1
= <1
2

Hence, by Cauchy’s root test, the series is convergent.

Example 5
Test the convergence of the series
2 3 n
1 Ê 2ˆ Ê 3ˆ Ê n ˆ
+ Á ˜ + Á ˜ + + Á + .
3 Ë 5¯ Ë 7¯ Ë 2 n + 1˜¯

Solution
n
Ê n ˆ
Let un = Á
Ë2 n + 1˜¯
2n
1
n
lim (un ) n = lim
n Æ• 2nn + 1
n Æ• 2
1
= lim
n Æ• 1
2+
n
1
= <1
2

Hence, by Cauchy’s root test, the series is convergent.


1.9 Cauchy’s Root Test 1.67

Example 6
13 23 33 43
Test the convergence of the series + + + + .
3 32 33 34
Solution
n3
Let un =
3n
1
1
Ê n3 ˆ n
((un )n =Á n˜
Ë3 ¯
3
nn
=
3
3
1
nn
lim ((un )n = lim
n Æ• n Æ• 3

( )
3
1
1
= lim n n …(1)
3 nƕ
1

Let l = lim (n) n


n Æ•

1
log l = lim log n
n Æ• n

log n È• ˘
= lim Í form ˙
n Æ• n Ε ˚
1
= lim n [Applying L’Hospital’s rule]
n Æ• 1
log l = 0
l = e0
=1
1
\ lim ((nn) n = 1
n Æ•

Substituting in Eq. (1),


1
1
( n )n =
lim (u <1
n Æ• 3
Hence, by Cauchy’s root test, the series is convergent.
1.68 Chapter 1 Sequences and Series

Example 7
Test the convergence of the series
3
Ê
- - - +

Solution
-n
˘
Let un ˙
n n
-1
1 È
( )n
n n
-1
È

1 1

lim ( )
n n Æ• n n n

1)-1
1
<1
e 1
Hence, by Cauchy’s root test, the series is convergent.

Example 8 • nn x n
Test the convergence of the series  0
n 1 (n )n
Solution
nn x n
Let un
( n + )n
1
nn x n ˘ n
1
( )n =Í n
(
nx
n +1
1
x
lim ( ) n lim
n Æ• n Æ• 1
1+
n
x
1.9 Cauchy’s Root Test 1.69

Hence, by Cauchy’s root test, the series is


(i) convergent if x < 1
(ii) divergent if x > 1
The test fails if x = 1. nn
Then un =
(n + 1)n
1
lim un = lim n
n Æ• n Æ•
Ê 1ˆ
ÁË 1 + ˜¯
n
1
= π0
e
The series is divergent for x = 1.
Hence, the series is convergent if x < 1 and is divergent if x ≥ 1.

Example 9
(n + 1)n x n
Test the convergence of the series  . [Summer 2014]
n n+1
Solution
un =
( n + 1)n x n
Let
n n +1
1
(n + 1) x
( n
(u )n = n +1
n n
(n + 1) x
= 1
n.n n
1
Ê 1ˆ x
lim (un )n = lim Á 1 + ˜ 1
n Æ• n Æ• Ë n¯
nn
È 1 ˘
=x Í ∵ lim n n = 1 as solved in Ex 6 ˙
ÍÎ x Æ• ˙˚
Hence, by Cauchy’s root test, the series is convergent, if x < 1 and divergent if x > 1.
The test fails for x = 1.
(n + 1)n
Then un =
n n +1
(n + 1)n
=
n ◊ nn
1.70 Chapter 1 Sequences and Series

n
1 Ê n + 1ˆ
=
n ÁË n ˜¯
n
1 Ê 1ˆ
= 1+ ˜
n ÁË n¯
1
Let vn =
n
n
un Ê 1ˆ
lim = lim Á 1 + ˜
n Æ• vn n Æ• Ë n¯
=e [finite and non-zero]

1
Svn = Ân is divergent as p = 1.

By comparison test, Sun is also divergent for x = 1.


Hence, the series is convergent for x < 1 and divergent for x ≥ 1.

Example 10
2 3
1 2 Ê 3ˆ Ê 4ˆ
Test the convergence of the series + x + Á ˜ x 2 + Á ˜ x 3 + º• .
2 3 Ë 4¯ Ë 5¯

Solution
n -1
Ê n ˆ
un = Á x n -1
Let Ë n + 1˜¯
n -1
1 n -1
Ê n ˆ n
((u
un )n =Á x n
Ë n + 1˜¯
1
1- 1
Ê n ˆ n 1-
=Á ( )
(x n
Ë n + 1˜¯
1
1-
Ê ˆ n
1 1
Á 1 ˜ 1-
lim (un ) = lim Á
n ( x) n
n Æ• n Æ• 1˜
Á 1+ ˜
Ë n¯
=x

Hence, by Cauchy’s root test, the series is convergent, if x < 1 and is divergent if
x > 1. Root test fails for x = 1.
1.10 Cauchy’s Integral Test 1.71

EXERCISE 1.4

Test the convergence of the following series:

1 1 1
1. 1 + + + + …∞ [Ans.: Convergent]
22 33 4 4
n
 n + 1
2. ∑  3n 
 [Ans.: Convergent]
3
− n2
3. ∑ 1 + 1 
 
n
[Ans.: Convergent]
x x2 x3
4. 1 + + + + …(x > 0)
2 32 4 3
[Ans.: Convergent]
n2
 1
5. ∑ 11++ n 
[[Ans.: Divergent]
(1 + nx)n
6. ∑
nn
[Ans.: Convergent if x < 1 and divergent if x > 1]

1.10 CAUCHY’S INTEGRAL TEST


• •
If  un =  f (n) is a positive term series where f (n) decreases as n increases and let
=1
nn= n =1

Ú1 f ( x )dx = I then

(i) Sun is convergent if I is finite


(ii) Sun is divergent if I is infinite

Proof Consider the area under the curve y = f (x) from x = 1 to x = n + 1 represented
n +1
as Ú1 f ( x )dx. Plot the terms f (1), f (2), f (3), …, f (n), f (n + 1).

n +1
The area Ú1 f ( x )dx lies between the sum of the areas of smaller rectangles and sum
of the areas of larger rectangles
1.72 Chapter 1 Sequences and Series

n +1
f (2) + f (3) +  + f (n + 1) £ Ú f ( x )dx £ f (1) + f (2) + f (3) +  + f (n)
1
n +1
Sn +1 - f (1) £ Ú f ( x )dx £ Sn
1

As n Æ • first inequality reduces to



lim Sn +1 £ Ú f ( x ) dx + f (1)
n Æ• 1


This shows that if Ú1 f ( x ) dx is finite, S f (n) = Sun is convergent.

As n Æ • second inequality reduces to y


∫1
f ( x) dxx ≤ lim S n
n→∞


or m S n ≥ ∫ f ((xx)dx
lim
n→∞ 1
y = f(x)

This shows that if ∫1
f ( x)dx
dx is infinite, f(1) f(2)f(3)

f(n) f(n + 1)
Σ f (n) = Σun is divergent. x
O 1 2 3 n n+1
Fig. 1.2

Example 1 ∞
1
Test the convergence of the series ∑ n log n .
n= 2

Solution
1
Let un = = f ( n)
n log n
1
f ( x) =
x log x
• • 1
Ú2 f ( x )dx = Ú
2 x log x
dx

m 1
= lim Ú
m Æ• 2 x log x
dx

m
= lim log log x 2
m Æ• È f ¢( x ) ˘
= lim (log log m - log logg 2) Æ • Í∵ Ú dx = log f ( x )˙
m Æ• Î f ( x) ˚
Hence, by Cauchy’s integral test, the series is divergent.
1.10 Cauchy’s Integral Test 1.73

Example 2
• 3
Test the convergence of the series  n2 e - n .
n =1
Solution
3
Let un = n2 e - n = f (n)
3
f ( x) = x2 e- x
• • 3
Ú1 f ( x )dx = Ú x 2 e - x dx
1

È 1 m 3 ˘
= lim Í - Ú e - x ( -3 x 2 )dx ˙
m Æ• Î 3 1 ˚
È 1 3 m˘
= lim Í - e - x ˙ È∵ e f ( x ) f ¢( x )dx = e f ( x ) ˘
m Æ• Î ˚
Î 3 1 ˚

È 1
m Æ• Î 3
3
( ˘
= lim Í - e - m - e -1 ˙
˚
)
1 -•
= - (e - e -1 )
3
1Ê 1ˆ
= - Á0 - ˜
3Ë e¯
1
= [finite]
3e
Hence, by Cauchy’s integral test, the series is convergent.

Example 3
• 1
Test the convergence of the series  .
n=3 n log n log n - 1
2

[Winter 2015]
Solution
1
Let un =
n log n log2 n - 1
1
f ( x) =
x log x log2 x - 1
1.74 Chapter 1 Sequences and Series

• •
1
Ú f ( x ) dx = Ú dx
3 3 x log x log2 x - 1
1
Putting log x = t, dx = dt
x
When x = 3, t = log 3
When x Æ •, t = log • = •
• •
dt
Ú f ( x )dx = Ú
3 log 3 t t2 - 1
m
dt
= lim
m Æ•
Ú
log 3 t t2 - 1
m
= lim sec -1 t
m Æ• log 3

= lim ÈÎsec -1 m - sec -1 (log 3)˘˚


m Æ•

= sec -1 • - sec -1 (log 3)


p
= - sec -1 (log 3)
2
= cosec -1 (log 3) [finite ]
Hence, by Cauchy’s integral test, the series is convergent.

Example 4
• 2 tan -1 n
Test the convergence of the series  . [Winter 2014]
n =1 1 + n2
Solution
Let 2 tan -1 n
un = = f ( n)
1 + n2
2 tan -1 x
f ( x) =
1 + x2
• •
2 tan -1 x
Ú f ( x ) dx = Ú 1 + x2
dx
1 1

2 tan -1 x
m
= lim
m Æ•
Ú 1 + x2
dx
1
1.10 Cauchy’s Integral Test 1.75

dx
Putting tan–1x = t, = dt
1 + x2
p
When x = 1, t = tan–1(1) =
4
When x = m, t = tan–1 m
• tan -1 m

Ú f ( x )dx = lim
m Æ•
Ú 2 t dt
1 p
4
tan -1 m
2t 2
= lim
m Æ• 2 p
4

È p2 ˘
( )
2
= lim Í tan -1 m - ˙
m Æ• Í 16 ˙˚
Î

( ) p2
2
= tan -1 • -
16
p2 p2
= -
4 16
3p 2
= [finite]
16
Hence, by Cauchy’s integral test, the series is convergent.

Example 5
Show that the harmonic series of order p,
• 1 1 1 1
 p = p + p + p + …• is convergent if p > 1 and is divergent if p £ 1.
n=1 n 1 2 3
[Summer 2015]

Solution
Let 1
un = = f ( n)
np
1
f ( x) =
xp
• • 1
Ú1 f ( x )dx = Ú
1 xp
dx
1.76 Chapter 1 Sequences and Series

m
x - p +1
= lim
m Æ• - p + 1
1

Êm 1 ˆ
1- p
= lim Á -
m Æ• Ë 1 - p 1 - p ˜¯
1
=- , p >1
1- p
= •, p <1
If p = 1, • • 1
Ú1 dxx = Ú
f ( x )d
1 x
dx
m 1
= lim
m Æ• 1
Ú x
dx

m
= lim log x 1
m Æ•
= lim (log m - logg1
g )
m Æ•
= log • Æ •

The integral Ú1 f ( x )dx is finite if p > 1 and is infinite if p £ 1.
Hence, by Cauchy’s integral test, the series is convergent if p > 1 and is divergent if
p £ 1.

EXERCISE 1.5
Test the convergence of the following series:

1
1. ∑ n =1 n
[Ans.: Divergent]

1
2. ∑ 2
n =1 n + 1

[Ans.: Convergent]

3. ∑ ne
n =1
− n2

[Ans.: Convergent]

1
4. ∑ 2
n =1 n(log n)

[Ans.: Convergent]
1.11 Alternating Series 1.77

1.11 ALTERNATING SERIES


An infinite series with alternate positive and negative terms is called an alternating
series.
Leibnitz’s Test for Alternating Series
• •
An alternating series  (--1)
1)n -1vn = Â un is convergent if
n=
n =1 n =1
(i) each term is numerically less than its preceding term, i.e. un +1 < un or
un > unn+1

(ii) lim un = 0
n Æ•

Example 1
1 1 1
Test the convergence of the series 1 − + − +… .
2 3 4
Solution
1
Let un = ( -1)n -1
n
1
un =
n
The given series is an alternating series.
1 1
(i) un - un +1 = -
n n +1
n +1 - n
= >0 for all n ΠN
n n +1
\ un > un +1

1
(ii) lim un = lim
Æ•
nnƕ n ƕn
=0
Hence, by Leibnitz’s test, the series is convergent.

Example 2
1 1 1
Test the convergence of the series 1 - + - +
+.
2 2 3 3 4 4
Solution
( -1)n -1
Let un =
n n
1.78 Chapter 1 Sequences and Series

1
un =
n n
The given series is an alternating series.
1 1
(i) un - un +11 = -
n n (n + 1) n + 1

(n + 1) n + 1 - n n
= > 0 for all n ΠN
( n n ) ÈÎ(n + 1) ( n + 1) ˘˚
\ un > un +1

1
(ii) lim un = lim
n Æ• n nn Æ•

=0
Hence, by Leibnitz’s test, the series is convergent.

Example 3
1 1 1 1
Test the convergence of the series 2
- 2
+ 2
- + .
1 2 3 42
Solution
1
Let un = ( -1)n -1 ◊
n2
1
un =
n2
The given series is an alternating series.
1 1
(i) un - un +11 = -
n (n + 1)2
2

2n + 1
2n
= 2 > 0 for all n ΠN
n (n +1
+ 1)2
\ un > un +1
1
(ii) lim un = lim
n Æ• n Æ• n2
=0
Hence, by Leibnitz’s test, the series is convergent.

Example 4
1 1 1 1
Test the convergence of the series - + - + º.
1p 2p 3p 4p
1.11 Alternating Series 1.79

Solution
1
Let un = ( -1)n
np
1
un =
np

The given series is an alternating series.


Case I: If p > 0,
1 1
(i) un - un +1 = p -
n (n + 1) p
(n + 1) p - n p
= >0 [∵ p > 0]
n p (n + 1) p
\ un > un +1
1
(ii) lim un = lim
Æ•
nnƕ n ƕ np
=0 [∵ p > 0]
Hence, by Leibnitz’s test, the series is convergent if p > 0.
Case II: If p < 0
In this case the conditions (i) and (ii) of the Leibnitz’s test are not satisfied.
Hence, the given series is not convergent if p < 0.

Example 5
1 1 1 1
Test the convergence of the series 1 - + - + -.
2 4 8 16
Solution
1
Let un = ( -1)n -1
2 n -1
1
un = n -1
2
The given series is an alternating series.
1 1
(i) un - un +1 = nn-1 - n
2 2
1 Ê 1ˆ
= n -1 Á 1 - ˜
2 Ë 2¯
1 1
= ◊
2 n -1 2
1
= n > 0 for all n ΠN
2
1.80 Chapter 1 Sequences and Series

\ un > un +1
1
(ii) lim un = lim n -1
n Æ• n Æ• 2
=0
Hence, by Leibnitz’s test, the series is convergent.

Example 6
1 2 3 4
Test the convergence of the series - + - +.+
2 5 10 17
Solution
n
Let un = ( -1)n -1
n +1
2

n
un =
n2 + 1
The given series is an alternating series.
n n +1
(i) un - un +11 = -
n +1
2
(n + 1)2 + 1
n(n2 + 2 n + 2) - (n + 1)(
)(n2 + 1)
=
(n2 + 1)(n2 + 2 n + 2)
n2 + n - 1
= > 0 for all n ΠN
(n2 + 1)(n2 + 2 n + 2)
\ un > un +1
n
(ii) lim un = lim
n Æ• n2 + 1
n Æ•

1
= lim
n Æ• 1
n+
n
=0
Hence, by Leibnitz’s test, the series is convergent.

Example 7
• ((-1)n+1
Test the convergence of the series  . [Summer 2014]
n£1 log( n + 1)
Solution
( -1)n +1
Let un =
log(n + 1)
1.11 Alternating Series 1.81

1
un =
log(n + 1)
The given series is an alternating series.
1 1
(i) un - un +11 = -
log(n + 1) log (n + 2)
log(n + 2) - llog(
og(n + 1)
= >0
log(n + 1) ◊ llog
og (n + 2)
\ un > un +1
1
(ii) lim un = lim
n Æ• n Æ• log( n + 1)

=0
Hence, by Leibnitz’s test, the series is convergent.

Example 8
• ((-1)n -1
Test the convergence of the series  2
.
n =1 n (n + 1)
Solution
( -1)n -1
Let un =
n2 (n + 1)
1
un =
n (n + 1)
2

The given series is an alternating series.


1 1
(i) un - un +11 = -
n (n + 1)
2
(n + 1) (n + 2)
2

(n + 1)(n + 2) - n2
=
n2 (n + 1)2 ((n
n + 2)
3 +2
3n
= > 0 for all n ΠN
n2 (n + 1)2 (n + 2)
\ un > un +1

1
(ii) lim un = lim
n Æ• n Æ• n2 (n + 1)
=0
Hence, by Leibnitz’s test, the series is convergent.
1.82 Chapter 1 Sequences and Series

Example 9
x2 x3 x4
Test the convergence of the series x 1) .
Solution
1 xn
Let un 1)n
n
xn
un
n

The given series is an alternating series.


x n x n +1
(i) un n +1 = -
n n +1
xn n nx
=

xn ] [
= >0
n 1)
\ u un +1

xn
(ii) lim lim
n n Æ• n
=0
Hence, by Leibnitz’s test, the series is convergent.

Example 10
Test the convergence of the series
1 2 4
log log + log log +
3 5
Solution
1 2 3 4
log log + log log +
3 5
2 3 4 5
- log + log log + log 
1 2 4
n +1
Let un = - )n log
n
n +1
un log
n
1.11 Alternating Series 1.83

The given series is an alternating series.


n +1 n+2 È n +1 n + 2 ˘
(i) un - un +11 = logg - log > 0 Í∵ > for all n Œ N ˙
n n +1 Î n n +1 ˚
\ un > un +1

Ê n + 1ˆ
(ii) lim un = lim log Á
n Æ• n Æ• Ë n ˜¯
Ê 1ˆ
= lim log Á 1 + ˜
n Æ• Ë n¯
= log 1
=0
Hence, by Leibnitz’s test, the series is convergent.

Example 11
1 2 3 4
Test the convergence of the series - + - +.
+
2 3 4 5
Solution
n
Let un = ( −1) n −1 ⋅
n +1
n
un =
n +1
The given series is an alternating series.
n n +1
(i) un - un +11 = -
n +1 n + 2
n2 + 2n - n2 - 2n - 1
=
(n + 1)(n + 2)
1
=- <0
(n + 1)(n + 2)
Since each term of the series is not numerically less than the preceding term,
Leibnitz’s test cannot be applied.
The series can be written as

Ê 1ˆ Ê 1ˆ Ê 1ˆ Ê 1ˆ
 un = ÁË1 - 2 ˜¯ - ÁË1 - 3 ˜¯ + ÁË1 - 4 ˜¯ - ÁË1 - 5 ˜¯ + 
n=1

Ê 1 1 1 1 ˆ
= (1 - 1 + 1 - 1 + ) + Á - + - + - ˜
Ë 2 3 4 5 ¯

= Â ( -1
-1)n -1 + (log 2 - 1)
n =1
1.84 Chapter 1 Sequences and Series

As n Æ •, the sum of the above series tends to (– 1 + log 2 – 1) or (1 + log 2 – 1)


according as n is even or odd.
Hence, the given series is an oscillatory series.

EXERCISE 1.6
Test the convergence of the following series:
1 1 1 1
1. 1 − + − + − ………
2 3 4 5
[Ans.: Convergent]

n
2. ∑ (−1)
n −1

n =1 2n − 1
2
[Ans.: Oscillatory]
1 1 1 1
3. 3
− 3 (1 + 2) + 3 (1 + 2 + 3) − 3 (1 + 2 + 3 + 4) + …
2 3 4 5
[Ans.: Convergent]

4. 1 − 2 x + 3x 2 − 4 x 3 + …(x < 1)
[Ans.: Convergent]
x x2 x3 x4
5. − + − + …(0 < x < 1)
1+ x 1 + x 2 1+ x3 1+ x 4
1+ [Ans.: Convergent]

1.12 ABSOLUTE CONVERGENCE OF A SERIES



The series  un with both positive and negative terms (not necessarily alternative) is

n =1
called absolutely convergent if the corresponding series  un with all positive terms
is convergent. n =1

Conditional Convergence of a Series If the series
• •
 un is convergent
n =1
and  un is divergent, then the series  un is called conditionally convergent.
n =1 n =1

Note 1: Every absolutely convergent series is a convergent series but converse is not
true.
Note 2: Any convergent series of positive terms is also absolutely convergent.

Example 1
• ( -1)n 23n
Test the convergence of the series  .
n =1 32 n
1.12 Absolute Convergence of a Series 1.85

Solution
( -1
-1)n 23n
Let un =
32 n
3n
2
un =
32 n
n +1)
23( n+
un +1 = n +1)
32( n+
n+2
un 23n 32 n+
lim = lim ◊ n+3
n Æ• un +1 n Æ• 32 n 23 n+

9
= lim
n Æ• 8
9
= >1
8

By D’Alembert’s ratio test, Â un is convergent. Thus, the series is absolutely conver-
n=1
gent and hence is convergent.

Example 2
Test the series for absolute or conditional convergence
2 3 4
1- + 2 + 3 +.
3 3 3
Solution
n
Let un = ( -1)n -1 ◊
3n -1

2 3 4
 un = 1+ + + +…
3 32 33
n =1
n
un = n -1
3
n +1
un +1 =
3n
un n 3n
lim = lim n -1

n Æ• un +1 n Æ• 3 n +1
3
= lim
n Æ• 1
1+
n
= 3>1
1.86 Chapter 1 Sequences and Series


By D’Alembert’s ratio test, Â un is convergent and hence, the series is absolutely
convergent. n =1

Example 3
• (-1)n
Test the series for absolute or conditional convergence  .
n =1 n + n +1
[Winter 2016]
Solution
(-1)n
Let un =
n + n +1
1
un =
n + 1+n
The given series is an alternating series.
1 1
(i) un - un +1 = -
n + 1+ n n +1 + n + 2

n +1 + n + 2 - n - n +1
=
( n + 1 + n ) ( 1 + n + n + 2)

n+2 - n
= >0 for all n ΠN
( n + 1 + n ) ( 1 + n + n + 2)

\ un > un +1
1
(ii) lim un = lim =0
n Æ• n Æ• n + 1+ n
By Leibnitz’s test, Â un is convergent.

1
un =
n + 1+ n
1
=
Ê 1ˆ
n Á1 + 1 + ˜
Ë n¯
1
Let vn =
n
1.12 Absolute Convergence of a Series 1.87

un 1
lim = lim
n Æ• vn n Æ• Ê 1ˆ
Á1 + 1 + n ˜
Ë ¯
=6 [finite and non-zero]
1 1
and  vn =  1
is divergent as p = .
2
n2
By comparison test, Â un is also divergent.
The series  un is convergent and  un is divergent.
Hence, the series is conditionally convergent.

Example 4
Determine absolute or conditional convergence of the series
• n2
 ((--1)n ◊ . [Winter 2013; Summer 2017]
n =1 n3 + 1
Solution
n2
Let un = ( -1)n ◊
n3 + 1
n2
un =
n +1
3

1
=
Ê 1ˆ
n Á1 + 3 ˜
Ë n ¯
1
Let vn =
n
un 1
lim = lim
n Æ• vn n Æ• 1
1+
n3
= 1 [[finite
finite and non-zero]
1
and Svn = Â is divergent as p = 1.
n
By comparison test, Â un is also divergent.
Hence, Sun is not absolutely convergent.
To check the conditional convergence, applying Leibnitz’s test,
n2 (n +1
+ 1)2
(i) un - un +11 = -
n +1
3
(n +
+11)3 + 1
1.88 Chapter 1 Sequences and Series

n2 (n3 + 3n2 + 3n + 2) - (n3 + 1)(n2 + 2 n + 1)


=
( n3 +
+11)[(n + 1)3 + 1]
n 4 + 2 n3 + n 2 - 2
2nn -1
=
( n3 +
+1 )[(n + 1)3 +1
1)[(
1 + 1]
n 4 + n2 (2 n + 1) -
-11(2 n + 1)
=
(n3 + 1)[(n + 1)3 + 1]
n 4 + (2 n + 1)(n2 - 1)
= > 0 for all n ΠN
(n3 + 1)[[((n + 1
1))3 + 1]
un > un +1

n2
(ii) lim un = lim
n Æ• n Æ• n3 + 1
1
= lim
n Æ• Ê 1ˆ
n Á1 + 3 ˜
Ë n ¯
=0
By Leibnitz’s test, Su n is convergent.
The series Sun is convergent and  un is divergent.
Hence, the series is conditionally convergent.

Example 5
Test the series for absolute or conditional convergence
2 3 1 4 1 5 1
- ◊ + ◊ - ◊ +º.
3 4 2 5 3 6 4
Solution
Ê n +1 1ˆ
un = ( -1)n -1 Á ◊
Let Ë n + 2 n ˜¯

2 3 1 4 1 5 1
 un = + ◊ + ◊ + ◊ + ◊◊◊.
3 4 2 5 3 6 4
n=1

n +1 1
| un | = ⋅
n+2 n
1
Let vn =
n
un n +1
lim = lim
n Æ• vn n Æ• n+2
1.12 Absolute Convergence of a Series 1.89

1
1+
= lim n
n Æ• 2
1+
n
= 1 [ffinite
inite and non-zero]
1
and Σvn = ∑ is divergent as p = 1.
n
By comparison test, Σ un is also divergent.
Hence, the series is not absolutely convergent.
To check the conditional convergence, applying Leibnitz’s test,
n +1 n+2
(i) un - un +1 = -
nn((n + 2) (n + 1)(n + 3)
n 2 + 3n + 3
= >0 for all n ΠN
nn((n + 1)(n + 2)(n + 3)
un > un+
n 1

n +1
(ii) lim un = lim
Æ•
nnƕ n ƕ n( n + 2)

1
1+
= lim n
n Æ• Ê 2ˆ
+ ˜
n Á 11+
Ë n¯
=0
By Leibnitz’s test, Sun is convergent.
The series Sun is convergent and Σ un is divergent.
Hence, the series is conditionally convergent.

Example 6
x3 x5
Test the convergence of the series x − + − − , x > 0.
3 5 [Summer 2016]
Solution
n -1
x 2 n-
Let un = ( -1)n -1
2n - 1
n -1
x 2 n-
un =
2n - 1
n +1
x 2 n+
un +1 =
2n + 1
1.90 Chapter 1 Sequences and Series

n -1
un x 2 n- 2n + 1
= ◊ 2 n+
un +1 2 n - 1 x n +1
1
2+
= n◊ 1
1 x2
2-
n
Ê 1ˆ
2+
un Á n˜◊ 1
lim = lim Á
n Æ• un +1 n Æ• 1 ˜ x2
Á2- ˜
Ë n¯
1
=
x2
1
By D’Alembert’s ratio test, Â un is convergent if
x2
> 1 or x2 < 1 or x < 1 [∵ x > 0]
Thus, the given series is absolutely convergent and hence, is convergent for x < 1.
If x2 = 1 or x = 1 [∵ x > 0]
( -1)n -1
un =
2n - 1
1
un =
2n - 1
The given series is an alternating series.
1 1
(i) un - un +11 = -
2n - 1 2n + 1
2
= 2 > 0 ffor
or all n ΠN
4n - 1
1
(ii) lim un = lim
n Æ• n Æ• 2 n - 1
=0
By Leibnitz’s test, the series is convergent for x = 1
Hence, the series is convergent for x £ 1.

EXERCISE 1.7
Test the following series for absolute or conditional convergence:
1 1 1 1 1
1. 1 − + − + − +…
2 3 4 5 6
[Ans.: Conditionally convergent]
1 1 1 1 1
2. 1 + − − + + −…
2 2 32 4 2 52 6 2
[Ans.: Absolutely convergent]
1.13 Uniform Convergence of a Series 1.91

1 1 1
3. + − +…
2 3 4
[Ans.: Conditionally convergent]
sin s s x
4. −…
1
[Ans.: Absolutely convergent]

1.13 UNIFORM CONVERGENCE OF A SERIES



The series ∑
n 1
n of real-valued functions defined in the interval (a, b) is said to
converge uniformly to a function S (x) if for a given ∈> 0, there exists a number m
independent of x such that for every x ( ),
| ( ) < ∈ for all n > m
where, S )

Weierstrass’s M-Test The series ∑n =1
n is said to converge uniformly in an

interval (a, b), if there exists a convergent series ∑M


n 1
n of positive constants such that

M n for all x ( )

Proof Let ∑M
n =1
n is convergent, then for a given ∈> 0, there exists a number m such
that < ∈ for all n > m,
where … ∞ and C M Mn
then < ∈ for all n > m
( < ∈ for all n > m [∵ Mn is positive constant]

Now, | M n for all x ∈ ( )

≤ Mn +…
< Πfor all n > m
\ < ∈ for all n > m
where Sn (x) = u1(x) + u2 (x) + … + un (x)

Since m does not depend on x, the series ∑ n converges uniformly in the interval
(a, b). n 1
1.92 Chapter 1 Sequences and Series

Example 1

1
Test the series ∑n
n =1
4
+ n3 x 2
for uniform convergence.

Solution
1
Let un ( x ) =
n 4 + n3 x 2
1
| un ( x ) | = 4
n + n3 x 2
1
< for all x ∈ R
n4
1
Mn =
n4
∞ ∞
1
∑M=1
nn=
n =∑
n =1 n
4 is convergent as p = 4 > 1.

Hence, by M-test, the series is uniformly convergent for all real values of x.

Example 2

sin( x 2 + n 2 x)
Test the series ∑ for uniform convergence .
n =1 n(n 2 + 2)
n(
Solution
sin( x 2 + n 2 x)
Let un ( x ) =
n(n 2 + 2)
sin( x 2 + n 2 x)
| un ( x ) | =
n ( n 2 + 2)
| sin(( x 2 + n 2 x) |
=
n ( n 2 + 2)
1 ∵ − 1 ≤ sin q ≤ 1
≤ for all x ∈ R 
n + 2n
3
 | sin q | ≤ 1
1
< for all n ∈ N
n3
1
Mn =
n3
∞ ∞
1
∑ Mn = ∑
n=
=11 n =1 n
3 is convergent as p = 3 > 1.

Hence, by M-test, the series is uniformly convergent for all real values of x.
1.13 Uniform Convergence of a Series 1.93

Example 3
sin 2 x sin 3 x sin 4 x
Test the series sin x − + − + … . for uniform
2 2 3 3 4 4
convergence.
Solution
sin nx
Let un ( x) = ( −1) n −1
n n
sin nx
| un ( x ) | =
n n
1 ∵ −1 ≤ sin q ≤ 1
≤ for all x ∈ R
3
 | sin q | ≤ 1
n 2 
1
Mn = 3
n2
∞ ∞
1 3
∑M=1
nn=
n =∑
n =1
3 is convergent as p =
2
> 1.
2
n
Hence, by M-test, the series is uniformly convergent for all real values of x.

Example 4

Show that if 0 < r < 1, the series ∑r
n =1
n
cos n 2 x is uniformly convergent.
Solution
Let un ( x ) = r n cos n2 x

un ( x ) = r n cos n2 x

£|| r n | for all x Œ R


£ È∵- 1 £ cos
cos q £ 1˘
Í cos q £ 1 ˙
= rn, 0 < r < 1 Î ˚

Mn = r n
• •
 Mn =  r n = r + r 2 + r 3 + …
n=
n =1 n =1

which is convergent being a geometric series with 0 < r < 1.


Hence, by M-test, the series is uniformly convergent for all real values of x.
1.94 Chapter 1 Sequences and Series

EXERCISE 1.8
Test the following series for uniform convergence:

sin(x 2 + nx)
1. ∑
n =1 n(n + 2)
n(
; for all real x

∞ [[Ans.: Uniformly convergent]


1
2. ∑
n =1 n + n x
p q 2
; for all real x and p > 1

[Ans.: Uniformly convergent]


sin x sin2 x sin3x sin 4 x
3. + + + +…
12 22 32 42
[Ans.: Uniformly convergent]
4. Show that if 0 < r < 1, then the series

∑r
n =1
n
sin a n x is uniformly convergent for all real values of x.

1 1 1 1
5. Show that − + − +…
1+ x 2
2+ x 3+ x
2 2
4 + x2
converges uniformly in the interval x ≥ 0 but not absolutely.

1.14 POWER SERIES


A power series is an infinite series of the form ∑a x


n =1
n
n
= a0 + a1 ( x − c) + ..
a2 ( x − c) 2 + a3 ( x − c)3 +  where an represents the coefficient of the nth term, c is a
constant and x varies around c. When c = 0, the series becomes

∑a x
n=0
n
n
= a0 + a1 x + a2 x 2 + a3 x 3 + 

1.14.1 Interval and Radius of Convergence


A power series will converge only for certain values of x. An interval (−R, R) in which
a power series converges is called the interval of convergence. The number R is called
the radius of convergence, e.g., if a power series converges for all the values of x, then
interval of convergence will be (−•, •) and the radius of convergence will be •.

1.14.2 Test for Convergence


Since a power series may be positive, alternating or mixed series, the concept of
absolute convergence is used to test the convergence of a power series. Applying
D’Alembert’s ratio test,
1.13 Power Series 1.95

un = an x n
un +1 = an +1 x n +1
un an x n
lim = lim
n ƕ un +1 nƕ an +1 x n +1

1 a
= lim n
x nÆ
Æ•• an +1

If an
lim = l,
n Æ• an +1
un 1 l
lim = ◊l =
n Æ• un +1 x x
By D’ Alembert’s ratio test, the series is absolutely convergent and hence is convergent
l
If > 1, i.e., | x |< l, - l < x < l.
x
Here, interval of convergence of the series is (−l, l ) and the radius of convergence is l.

Example 1

xn
Obtain the range of convergence of ∑ n , x > 0.
n =1 2
Solution
xn
Let un =
2n
x n +1
un +1 =
2 n +1
un x n 2 n +1
lim = lim n ◊ n +1
n Æ• un +1 n Æ• 2 x
2
=
x
By D’Alembert’s ratio test, the series is
2
(i) convergent if > 1 or x < 2
x
2
(ii) divergent if < 1 or x > 2
x
2
The test fails if = 1, or x = 2.
x
2n
Then un = =1
2n
1.96 Chapter 1 Sequences and Series


 un = 1 + 1 + 1 +  •
n=1
which is a divergent series.
Hence, the series is convergent for 0 < x < 2 and the range of convergence is
0 < x < 2.

Example 2

2n x n
Determine the interval of convergence for the series ∑ and also,
n = 0 n!
their behaviour at each end points.
Solution
2n x n
Let un =
n!
2 n +1 x n +1
un +1 =
(n + 1)!
un 2 n x n (n + 1)!
= ◊
un +1 n ! 2 n +1 x n +1
n +1
=
2x
un n +1
lim = lim
n ƕ un +1 nƕ 2 x
= • >1
Hence, By D’Alembert’s ratio test, the series is convergent for all values of x
i.e. – • < x < • and interval of convergence is (– •, •).

Example 3

xn
Obtain the range of convergence of ∑
n =1 a + n
, x > 0, a > 0.

Solution
xn
Let un =
a+ n
x n +1
un +1 =
a + n +1
un xn a + n +1
lim = lim ◊
n Æ• un +1 n Æ• a + n x n +1
1.13 Power Series 1.97

a 1
+ 1+
n n 1
= lim ◊
n Æ• a x
+1
n
1
=
x
By D’Alembert’s ratio test, the series is
1
(i) convergent if > 1 or x < 1
x
1
(ii) divergent if < 1 or x > 1
x
The test fails if x = 1.
1
Then un =
a+ n
1  1 
=
n  a + 1
 
n 
1
Let vn =
n
u 1
lim n = lim
n Æ• vn n Æ• a
+1
n
= 1 [ffinite
inite and non-zero]
1 1
and Σvn = ∑ 1
is divergent as p = < 1.
2
2
n
By comparison test, Σun is also divergent for x = 1.
Hence, the series is convergent for 0 < x < 1 and the range of convergence is 0 < x < 1.

Example 4 ∞
( x + 1) n
Obtain the range of convergence of ∑ n .
n =1 3 ⋅ n
Solution
( x + 1)n
Let un =
3n ◊ n
( x + 1)n +1
un +1 = n +1
+1
3n+ (n + 1)
1.98 Chapter 1 Sequences and Series

n +1
+1
un ( x + 1)n 3n+ (n + 1)
= n ◊
un +1 3 ◊ n ( x + 1)n +1
3(n + 1)
=
( x + 1)n
Ê 1ˆ
3 Á1 + ˜
Ë n¯
=
x +1
 1
3 1 + 
un  n
lim
m = lim
n →∞ u
n +1
n →∞ x +1
3
=
x +1
The series is convergent if
3
>1
x +1
3 > x +1
x+
+11 < 3
−3 < ( x + 1) < 3
−4 < x < 2
At x = 2,
1
un =
n
∞ ∞
1
∴ ∑u
n =1
=1
n =∑
n =1 n
is divergent as p = 1.

( −1) n
At x = – 4, un =
n
1
un =
n

The given series is an alternating series.


1 1
(i) un − un +1 = −
n n +1
1
= >0
n(n +1
+ 1) for all n ΠN
∴ u n > u n +1

1
m un = lim
(ii) lim
n →∞ n →∞ n
=0
1.13 Power Series 1.99

By Leibnitz’s test, the series is convergent at x = – 4.


Hence, the series is convergent for – 4 £ x < 2 and the range of convergence
is – 4 £ x < 2.

Example 5 ∞
xn
Obtain the range of convergence of ∑ n+
n =1 1 + n2
.
Solution
xn
Let un =
n + 1 + n2
x n +1
u n +1 =
(n +
+11) + 1 + (n + 1) 2
un xn (n +
+11) + 1 + (n + 1) 2
= ⋅
u n +1 n + 1 + n 2 x n +1

2
 1 1  1
1 +  + + 1 + 
n n2  n 
=
 1 
1 + 2
+ 1 x

n
2
 1 1  1
1 +  + + 1 + 
un n n2  n 
lim
m = lim
n →∞ u n +1 n →∞  1 
1 + 2
+ 1 x

n
1
=
x
The series is convergent if
1
>1
x
x <1
−1 < x < 1
−1

At x = 1, 1
un =
n + 1 + n2
1
=
 1 
n 1 + + 1
 n 2

1
Let vn =
n
1.100 Chapter 1 Sequences and Series

un 1
lim
m = lim
n →∞ vn n→∞  1 
1 + 2
+ 1

n
1
= [[finite
finite and non-zero]
2
1
and Σvn = ∑ is divergent as p = 1.
n
Thus, by comparison test, Σun is also divergent if x = 1.
At x = – 1,
( −1) n
un =
n + 1 + n2
1
un =
n + 1 + n2
The given series is an alternating series.
1 1
(i) un − un +1 = −
n + 1+ n 2
(n +
+11) + 1 + (n +1
+ 1) 2

(n +
+11) + 1 + (n + 1) 2 − n − 1 + n 2
=
(n + 1 + n 2 ) ((nn + 1) + 11+
+ (n + 1) 2 

1 + 1 + ((n + 1) 2 − 1 + n 2
= >0
(n + 1 + n 2 ) (n + 1) + 11+
+ (n + 1) 2 
for all n ŒN

∴ u n > u n +1

1
m un = lim
(ii) lim
n →∞ n →∞
n + 1 + n2
=0
Thus, by Leibnitz’s test, the series is convergent if x = –1.
Hence, the series is convergent for – 1 £ x < 1 and the range of convergence is
– 1 £ x < 1.

Example 6

n -1 x2n - 1
For the series  (-1) , find the radius and interval of
n =1 2n - 1
convergence. [Winter 2016]
1.13 Power Series 1.101

Solution
(-1)n -1 x 2 n - 1
Let un =
2n - 1
(-1)n x 2 n + 2 - 1
un + 1 =
2n + 2 - 1
(-1)n x 2 n + 1
=
2n + 1
un (-1)n - 1 x 2 n -1 2n + 1
= ◊
un + 1 2n - 1 (-1)n x 2 n + 1
(2 n + 1) 1
=- ◊
(2 n - 1) x 2

un (2 n + 1) 1
lim = lim ◊
n Æ• un + 1 n Æ• (2 n - 1) x 2

1
=
x2

1 2 2
By D’Alembert’s ratio test, the series is convergent if > 1 or 1 > x or x < 1
x2
i.e., – 1 < x < 1 and divergent for x > 1.

At x = 1,
(-1)n - 1 (1)2 n - 1
un =
2n - 1
1 1 1
=1- + - +
3 5 7
(-1)n - 1
=
2n - 1
1
un =
2n - 1
The given series is an alternating series.
1 1
(i) un - un + 1 = -
2n - 1 2n + 1
2n + 1 - 2n + 1
=
(2 n - 1) (2 n + 1)
1.102 Chapter 1 Sequences and Series

2
= >0 for all n ΠN
(2 n - 1) (2 n + 1)
un > un +1

1
(ii) lim un = lim =0
n Æ• n Æ• 2n - 1

By Leibnitz’s test, Â un is convergent.

At x = –1,
(-1)n - 1 (-1)2 n - 1
un =
2n - 1

(-1)3n - 2
=
2n - 1
1
un =
2n - 1
The given series an alternating series.
Hence, by Leibnitz’s test, Â un is convergent.

Thus, for the interval –1 £ x £ 1, given series is convergent.


Since condition for convergence is | x2 | < 1, radius of convergence = 1.

Since  un is convergent, the series is absolutely convergent for –1 £ x £ 1.
n =1

Example 7
• ( -3)n x n
Find the interval of convergence of the series  .
n +1
n= 0
[Winter 2013; Summer 2016]
Solution

(-3)n x n
un =
n +1
(-3)n +1 x n +1
un +1 =
n+2
1.13 Power Series 1.103

un (-3
-3)n x n n+2
= ◊
un +1 n + 1 (-3 -3)n +1 x n +1
n+2
=
(-
-33) x n + 1
2
1+
n
=
1
(-
-33) x 1 +
n
2
1+
u n
lim n = lim
n Æ• un +1 n Æ• 1
( -3) x 1 +
n
1
=
-3 x
1
=
3x

1 1
By D’Alembert’s ratio test, the series is convergent if > 1 or 3 x < 1 or x < or
3x 3
1 1
- <x< .
3 3
Ê 1 1ˆ
The interval of convergence is Á - , ˜ .
Ë 3 3¯

1
At x = - ,
3

n
Ê 1ˆ
( -3)n Á - ˜
Ë 3¯
un =
n +1
1
=
n +1
1
=
1
n 1+
n
1.104 Chapter 1 Sequences and Series

1
Let vn =
n
un 1
lim = lim
n ƕ vn nƕ 1
1+
n
= 1 [finite and non-zero]

1 1
and Svn = Â is divergent as p = .
n 2
1
Thus, by comparison test, Sun is also divergent if x = - .
3
1
At x = ,
3
n
Ê 1ˆ
( -3) Á ˜
n
Ë 3¯
un =
n +1
( -1)n
=
n +1
1
un =
n +1
1
lim un = lim =0
n Æ• n Æ• n +1
1
By Leibnitz’s test, the series is convergent at x = .
3
Hence, the series is convergent at each end point.

Example 8 • n( x + 1)n
n(
Determine the interval of convergence for the series  ((-1)n
and also, their behaviour at each end points. n =1 2n

Solution
n( x + 1)n
Let un = ( -1)n
2n

(n + 1)( x + 1)n +1
-1)n +1
un +1 = ( -1
2 n +1
un (-
-11) n ◊ ( x + 1)n
n
2 n +1
= ◊
un +1 2n ( -11))((nn +1) (n + 11)(
)( x + 11))n +1
1.13 Power Series 1.105

n 2
= ◊
n +1 x +1
1 2
= ◊
1 x +1
1+
n

un Ê 1 ˆÊ 2 ˆ
lim = lim Á ˜
n Æ• un +1 nÆ• Á 1 ˜ Ë x + 1¯
ÁË 1 + ˜¯
n
2
=
x +1

The series is convergent if


2
>1
x +1
2 > x +1

x+
+11 < 2
−2 < ( x + 1) < 2
−3 < x < 1

The series is convergent in the interval (– 3, 1).


At x = –3,

−3 + 1) n
n((−
un = ( −1) n
2n
∞ ∞

∑u
n=1
n
n = ∑n
n =1
n=

which is a divergent series.


At x = 1,
n((11 + 1) n
un = ( −1) n
2n
= ( −1) n
n

un = n
lim un ≠ 0
n →∞

By Leibnitz’s test, the series is not convergent at x = 1.


Hence, the series is not convergent at each end point and the interval of convergence
is (– 3, 1).
1.106 Chapter 1 Sequences and Series

Example 9
• ( -1)n ( x + 2)n
For the series  n
, find the radius and interval of
n =1
convergence. For what values of x does the series converge absolutely,
conditionally? [Winter 2015]
Solution
(-
-11)n ( x + 2)n
Let un =
n
-11)nn++1 ( x + 2)n +1
(-
un +1 =
n +1
un (-
-11)n ( x + 2)n ((n + 1)
= ◊
un +1 n -11) 1 ((xx + 2)n +1
(- n+
n +

Ê 1ˆ 1
= - Á1 + ˜ ◊
Ë n ¯ ( x + 2)

un Ê 1ˆ 1
lim = lim Á 1 + ˜ ◊
n Æ• un +1 nÆ• Ë n ¯ ((x + 2)
1
=
x+2

The series is convergent if


1
>1
x+2
1> x+2
x+2 <1
-1 < ( x + 2) < 1
-3 < x < -1

At x = –3,
(-
-11)n ( -3 + 2)n 1
un = =
n n
• •
1
 un =  n
n=1
n n =1
n=

which is a divergent series.


1.13 Power Series 1.107

At x = –1,

(-
-1
1)n ( -1 + 2)n
un =
n
( -1)n
=
n
1
un =
n
1
lim un = lim = 0
n Æ• n Æ• n

By Leibnitz’s test, the series is convergent at x = –1.


Hence, interval of convergence is (–3, –1] i.e. –3 < x £ –1.
Since condition for convergence is x + 2 < 1 , radius of convergence = 1.

Since  un is convergent, the series is absolutely convergent for –3 < x £ –1.
n=1

Example 10

1
Obtain the range of convergence of ∑x
n =1
n
+ x−n
.

Solution
1
Let un =
x + x-n
n

xn
=
x2n + 1
x n +1
un +1 =
x2n+2 + 1
un xn x2n+2 + 1
= 2n ◊
un +1 x + 1 x n +1

x2n+2 + 1
=
x( x 2 n + 1)
un x2n+2 + 1
=
un +1 x( x 2 n + 1)
1.108 Chapter 1 Sequences and Series

If x > 1,
1
x2 +
u x2n
m n = lim
lim
n →∞ u n →∞  1 
n +1
x 1 + 2 n 
 x 
= x > 1 ∵ lim
lim x 2 n → ∞ 
 n→∞ 

Thus, the series is convergent for x > 1, i.e. x > 1 and x < – 1
At x = 1,
1
un =
2

1 1 1

n =1
un = + + +  ∞
2 2 2
which is a divergent series.
At x = – 1,
( −1) n
un =
2
1
un =
2
1
lim un = ≠ 0
n →∞ 2
Thus, by Leibnitz’s test, the series is not convergent at x = – 1.
Hence, the series is convergent for x > 1 and range of convergence is x > 1.

EXERCISE 1.9

Obtain the range of convergence of the following series:

1 x + 2 x 2 + 3 x 3 +  + nx n + 
1. 1+ [Ans.: −1< x < 1]
1 x x2 x3 xn
2. + + + ++ +
2 3 4 5 n+2
[Ans.: −1 < x < 1]

(−1)n x n
3. ∑ [Ans.: |x| ≤ 1]
n =1 (n + 1)


(x + 2)
4. ∑
n=0 n +1
[Ans.: −3 ≤ x ≤ −1]


xn
5. ∑ (−1)n
n=0 log(n + 1)
[Ans.: |x| < 1]
Points to Remember 1.109


 1 3
6. ∑ (−2) (n + 1))((x − 1)
n n
 Ans.: 2 < x < 2 
n=0  

7. ∑ n !(x − 1)
n =1
n
[Ans.: x = 1]


(n !!)2
8. ∑ (2n))! x
n =1
n
[Ans.: |x| < 4]


(−2)n (2 x + 1)n
9. ∑  3 1
n2
n =1
 Ans.: − 4 ≤ x ≤ − 4 
 

(−1)n x 2 n
10.

n =1
3
[Ans.: −1 ≤ x ≤ 1]
(2n) 2

POINTS TO REMEMBER

• Sequence
A sequence {un} is said to be convergent, divergent or oscillatory
according as lim un is finite, infinite or not unique respectively.
n→∞

• Series
The infinite series Sun is said to be convergent, divergent or oscilla-
tory according as lim Sn is finite, infinite or not unique respectively.
n→∞

If a positive term series Sun is convergent then lim un = 0 but


n→∞
converse is not true i.e., if lim un = 0, the series may converge or
n→∞
diverge. If lim un π 0, the series is not convergent.
n→∞

• Comparison Test
un
If Sun and Svn are series of positive terms such that nlim = l
→∞ vn
(finite and non-zero) then both series converge or diverge together.

• D’Alembert’s Ratio Test


un
If Sun is a positive term series and nlim = l then
→∞ un +1
(i) Sun is convergent if l > 1.
1.110 Chapter 1 Sequences and Series

(ii) Sun is divergent if l < 1.


(iii) The test fails if l = 1.

• Cauchy’s Root Test 1


If Sun is a positive term series and if lim(un) n = l then
n→∞
(i) Sun is convergent if l < 1.
(ii) Sun is divergent if l > 1.
This test is preferred when un contains nth powers of itself.
• Cauchy’s Integral Test
If Sun = S f (n) is a positive term series where f (n) decreases as n in-

creases and let ∫
1
f (x) dx = I then

(i) Sun is convergent if I is finite.


(ii) Sun is divergent if I is infinite.
This test is preferred when evaluation of the integral of f (x) is easy.

• Leibnitz’s Test

An alternating series ∑ (− 1)
n =1
n −1
un is convergent if

(i) each term is numerically less than its preceding term, i.e,
un +1 < un or un > un +1
(ii) lim un = 0
n→∞

• Absolute Convergence

The series ∑u n =1
n with both positive and negative terms (not neces-

sarily alternative) is called absolutely convergent if the correspond-



ing series ∑ ||u | with all positive terms is convergent.
n =1
n

• Conditional Convergence
∞ ∞
If the series ∑u
n =1
n is convergent and ∑ ||u | is n divergent then the
n =1

series ∑u
n =1
n is called conditionally convergent.
Multiple Choice Questions 1.111

• Uniform Convergence (Weierstrass’s M-Test)


The series ∑ u ( x)
n =1
n is said to converge uniformally in an interval

(a, b), if there exists a convergent series of positive constants such

that |un(x)| £ Mn for all x Œ (a, b).

MULTIPLE CHOICE QUESTIONS

Choose the correct alternative in the following questions:


1
1. The value of lim x sin is [Summer 2014]
x Æ0 x
(a) 1 (b) p (c) 0 (d) •

1
2. The sum of the series  2n
is [Winter 2015]
n =1

3
(a) 0 (b) (c) 1 (d) 2
4

( -1)n
3. The series  is [Winter 2015]
n =1 n
(a) divergent (b) absolutely convergent
(c) conditionally convergent (d) nothing can be said
1
4. The series  n(5 n - 1) is
(a) convergent (b) divergent
(c) oscillates finitely (d) oscillates infinitely

1
5. The series  n(log n)
is
n=2

(a) convergent (b) divergent


(c) oscillates finitely (d) oscillates infinitely
1
6. The series  is divergent if
np
(a) p > 1 (b) p £ 1 (c) p = 1 (d) p = 0

> 1, then  un is
un +1
7. If lim
x Æ 0 un n =1
(a) convergent (b) divergent
(c) may or may not be convergent (d) oscillatory
1.112 Chapter 1 Sequences and Series

8. The series a + ar + ar + ar +  oscillates finitely if


2 3

(a) |r| < 1 (b) r > 1 (c) r = 1 (d) r £ –1


3 9 27 81
9. The series + + + +  is
4 8 16 32
(a) convergent (b) divergent
(c) oscillates finitely (d) oscillates infinitely

1
10. The series  n 5n
is
n =1
(a) convergent (b) divergent
(c) oscillates finitely (d) oscillates infinitely
11. The series a + ar + ar + ar +  diverges if
2 3

(a) |r| < 1 (b) r ≥ 1 (c) r £ –1 (d) r = 1


1
12. The series  np converges if

(a) p > 1 (b) p < 1 (c) p = 0 (d) p = 1


1
13. The series  1
is
n4
(a) convergent (b) divergent
(c) oscillates finitely (d) oscillates infinitely

32 n
14. The series  is
n =1 4 n
(a) convergent (b) divergent
(c) oscillates finitely (d) oscillates infinitely

1
15. The series  sin n is
n =1
(a) convergent (b) divergent
(c) oscillatory (d) may or may not be convergent
un
16. lim = 1 and  vn diverges then  un is
nÆ a vn
(a) divergent (b) convergent
(c) oscillatory (d) oscillates infinitely
2 3 4
17.
p
+ p
+
+  • converges if
1 2 3p
(a) p < 2 (b) p > 2 (c) p > 1 (d) p ≥ 1
18. If Sun is a series of positive terms such that lim un = 0 then Sun is
n Æ•
(a) convergent (b) divergent
(c) may or may not be convergent (d) oscillatory
Multiple Choice Questions 1.113

1
19. Â n(log n) p is convergent

(a) for p > 1 (b) for p < 1


(c) for all real values of p (d) for no value of p

20. Â (2 x)n is divergent if
n=0

1 1 1 1
(a) -1 £ x £ 1 (b) - <x< (c) -2 £ x £ 2 (d) - ≥x≥
2 2 2 2

21. The geometric series  ar n , when r = –1 × 2 is
n=0

(a) convergent (b) divergent (c) oscillatory (d) none of these


n! un
22. If un = then lim =
nn x Æ• un +1
(a) e2 (b) e (c) e–1 (d) 1
1 1 1
23. The series 1 + + + +  is
2 4 8
(a) convergent (b) divergent (c) oscillatory (d) none of these
3n
2
24. The series  32 n is

(a) convergent (b) divergent (c) oscillatory (d) none of these



25. The power series  (3 x)n is convergent if
n =1

1 1 1 1 1
(a) x= (b) x > (c) - <x< (d) < x <1
3 3 3 3 3

n2
26. The series  n!
is
n =1
(a) oscillatory (b) divergent (c) convergent (d) none of these
2
• n
Ê 1ˆ
27. The series  ÁË1 + n ˜¯ is
n =1

(a) oscillatory (b) convergent (c) divergent (d) none of these


2 2 3 n
x x x x x
28. The series + + + ++ +  is
1◊ 2 3 ◊ 4 3 ◊ 4 5 ◊ 6 (2 n - 1) 2 n
(a) p series (b) geometric series
(c) alternating series (d) power series
1.114 Chapter 1 Sequences and Series


xn
29. The series  (-1)n+1 n
converges only if
n =1
(a) –1 < x < 1 (b) –1 £ x £ 1 (c) –1 < x £ 1 (d) –1 £ x < 1
30. Which of the following series is divergent?
Ê 1ˆ 1 1 1
(a)  ÁË1 + n ˜¯ (b)  n2 (c)  np (d)  ne
31. Which of the following series is convergent?
• • 1
1 1 1
(a) Â n0.9 (b) Â (c) Ân (d) Â n1.001
n =1 n =1 n
cos np
32. The series  1 + n2 is

(a) absolutely convergent (b) conditionally convergent


(c) convergent (d) divergent
33. The series  ( - n) is
(a) divergent (b) convergent (c) oscillatory (d) none of these
1 1 1
34. The sum of the series 1 - + 2 - 3 +  is [Summer 2016]
2 2 2
2 3 1
(a) (b) (c) (d) None of these
3 2 2

1
35. The series  (log n)n is [Summer 2016]
n =1

(a) oscillatory (b) divergent (c) convergent (d) None of these


Ê p 1ˆ
36. The sequence sin Á + ˜ converges to [Winter 2016]
Ë 6 n¯
(a) 0 (b) 1 (c) –1 (d) 0.5
• n
Ê eˆ
37. The sum of the series  ÁË p ˜¯ is [Winter 2016]
n =1

p e p e
(a) (b) (c) (d)
p -e p -e e-p p

2n
38. Â is [Winter 2016]
n = 1 3n - 1

(a) convergent and sum is 0 (b) convergent and sum is 1


(c) divergent (d) oscillating
Answers 1.115

1 1 1
39. Infinite series 1 + + + +  is [Summer 2017]
2 2 2 23
(a) divergent (b) convergent (c) oscillatory (d) None of these

Answers

1.(c) 2.(c) 3.(c) 4.(b) 5.(b) 6.(b) 7.(b) 8.(d) 9.(b) 10.(a)
11.(b) 12.(a) 13.(b) 14.(b) 15.(b) 16.(a) 17.(b) 18.(c) 19.(a) 20.(b)
21.(c) 22.(b) 23.(a) 24.(a) 25.(c) 26.(c) 27.(c) 28.(d) 29.(c) 30.(a)
31.(d) 32.(a) 33.(a) 34.(a) 35.(c) 36.(d) 37.(b) 38.(c) 39.(b)
CHAPTER
2
Taylor’s and
Maclaurin’s Series
2.1 intRoDuCtion
In this chapter we will study Taylor’s and Maclaurin’s series. A Taylor series is a
representation of a function as an infinite sum of terms that are calculated from the
values of the function’s derivatives at a single point. The concept of a Taylor series
was discovered by the Scottish mathematician James Gregory and formally introduced
by the English mathematician Brook Taylor in 1715. A Maclaurin series is a Taylor
series expansion of a function about zero. It is named after the Scottish mathematician
Colin Maclaurin, who made extensive use of this special case of Taylor series. It is
common practice to approximate a function by using a finite number of terms of its
Taylor series and Maclaurin’s series covering expansions by definition, by standard
expansion, by differentiation and integration and by substitution.

2.2 tAYloR’s sERiEs

Statement If f (x + h) is a given function of h which can be expanded into a conver-


gent series of positive ascending integral powers of h then
h2 h3 hn n
f ( x + h) = f ( x) + h f ′( x) +
f ′′( x) + f ′′′( x) + ........ + f ( x) + .......
2! 3! n!
Proof Let f (x + h) be a function of h which can be expanded into positive ascend-
ing integral powers of h, then
f (x + h) = a0 + a1 h + a2 h2 + a3 h3 + a4 h4 +…….…… ... (1)
Differentiating w.r.t. h successively,
f ¢(x + h) = a1 + a2 · 2h + a3 · 3h2 + a4 · 4h3 + ……. ……… ... (2)
2
f ¢¢(x + h) = a2 · 2 + a3 · 6h + a4 · 12h +……. ……… ... (3)
f ¢¢¢(x + h) = a3 · 6 + a4 · 24h +……. ……… ... (4)

and so on.
2.2 Chapter 2 Taylor’s and Maclaurin’s Series

Putting h = 0 in Eq. (1), (2), (3) and (4),


a0 = f (x)
a1 = f ¢ (x)
1
a2 =
f ′′ ( x)
2!
1
a3 = f ′′′ ( x) and so on
3!
Substituting a0, a1, a2 and a3 in Eq. (1),

h2 h3 hn n
f ( x + h) = f ( x ) + h f ′ ( x ) + f ′′ ( x ) + f ′′′ ( x) + ....... + f ( x) + .......
2! 3! n!
This is known as Taylor’s series.
Putting x = a and h = x – a in above series, we get Taylor’s series in powers of
(x – a) as
( x − a)2 ( x − a )3
f ( x) = f (a) + ( x − a) f ′ (a) + f ′′ ( a ) + f ′′′ (a ) + ... .......
..
2! 3!
((x − a ) n n
+ f (a ) + ......
n!
Note: To express the function in ascending powers of x, express h in terms of x.

Example 1
(m − 1) 2 2
Prove that f (mx) = f ( x) + (m − 1) x f ′′(( x) + x f ′′ ( x) + ... .
2!
Solution
f (mx) = f (mx – x + x) = f [x + (m – 1) x]
By Taylor’s series,
h2 h3
f ( x + h) = f ( x ) + h f ′ ( x ) + f ′′ ( x ) + f ′′′ ( x) + ................
2! 3!
Putting h = (m − 1) x,

(m − 1) 2 2
f [ x + (m − 1) x] = f (mx
mx) = f ( x) + (m − 1) x f ′ ( x) + x f ′′ ( x) + ..........
..
2!

Example 2
Prove that
 x2  x x2 x3
f = f ( x ) − f ′ ( x ) + f ′′ ( x ) − f ′′′ ( x) + ... .
11+ x  1+ x 2 !(1 + x) 2 3!(1 + x)3
2.2 Taylor’s Series 2.3

Solution
x2 x
= x−
1 x
1+ 1+ x
By Taylor’s series,
h2 h3
f ( x + h) = f ( x) + h f ′( x) + f ′′ ( x ) − f ′′′ ( x) + ...
2! 3!
Putting x
h=- ,
1+ x
Ê x ˆ Ê x2 ˆ
f Áx - = f Á 1+ x ˜
Ë 1 + x ˜¯ Ë ¯
Ê x2 ˆ x x2 x3
fÁ ˜ = f ( x) - f ′ (x
( )+ f ′′ ( x ) - f ′′′( x ) + ...
Ë 1+ x ¯ 1+ x 2 ! (1 + x )2 3! (1 + x )3

Example 3
Express f (x) = 2x3 + 3x2 – 8x + 7 in terms of (x – 2).
Solution
f (x) = 2x3 + 3x2 – 8x + 7
By Taylor’s series,
( x − a)2 ( x − a )3 ( x − a)n n
f ( x) = f (a ) + ( x − a ) f ′ (a ) + f ′′ (a ) + f ′′′ (a ) +
+ + f (a ) +
2! 3! n!
Putting a = 2,

( x − 2) 2 ( x − 2) 3
f ( x) = f (2) + ( x − 2) f ′ (2
( )+ ( )+
f ′′ (2 ( ) +
f ′′′ (2 ...(1)
2! 3!
f ( x) = 2 x 3 + 3 x 2 − 88xx + 7 , f (2) = 16 + 12 − 16 + 7 = 19
f ′ ( x) = 6 x + 6 x − 8,
2
f ′ (2) = 24 + 12
12 − 8 = 28
f ′′ ( x) = 12 x + 66,, f ′′ (2) = 24 + 6 = 30
f ′′′ ( x) = 12, 2) = 12
f ′′′ (2)
Substituting in Eq. (1),

( x - 2 )2 ( x - 2)3
f ( x ) = 19 + ( x - 2)28 + ◊ 30 + ◊ 12
2! 3!
= 19 + 28( x - 2) + 15( x - 2)2 + 2( x - 2)3

Example 4
Express 2 x3 + 7 x 2 + x − 6 in ascending powers of (x – 2).
2.4 Chapter 2 Taylor’s and Maclaurin’s Series

Solution
Let f ( x) = 2 x3 + 7 x 2 + x − 6
By Taylor’s series,
( x − a)2 ( x − a )3
f ( x) = f (a) + ( x − a) f ′ (a) + f ′′ (a ) + f ′′′ (a ) + 
2! 3!
Putting a = 2,
( x − 2) 2 ( x − 2) 3
f ( x) = f (2) + ( x − 2) f ′ (2
( )+ ( )+
f ′′ (2 ( ) +
f ′′′ (2 ...(1)
2! 3!
f ( x) = 2 x 3 + 7 x 2 + x − 6, f (2) = 40
f ′ ( x) = 6 x + 14 x + 1,
2
f ′ ( 2) = 5 3
f ′′ ( x) = 12 x + 14, 2) = 3 8
f ′′ (2)
′′′ ( x) = 12,
f ′′′( 2) = 1 2
f ′′′ (2)

Substituting in Eq.(1),
((xx - 2)2 ( x - 2)3
f ( x ) = 40 + ( x - 2)(
)(53) + 38) +
((38 ( )
(12
2! 3!
= 40 + 53( x - 2) + 19( x - 22))2 + 2( x - 2)3

Example 5
Expand x3 + 7 x 2 + x − 6 in powers of (x – 3).
Solution
Let f ( x) = x3 + 7 x 2 + x − 6
By Taylor’s series,
( x − a)2 ( x − a )3
f ( x) = f (a) + ( x − a) f ′ (a) + f ′′ (a ) + f ′′′ (a ) + 
2! 3!
Putting a = 3,
( x − 3) 2 ( x − 3)3
f ( x) = f (3) + ( x − 3) f ′ (3
( )+ ( )+
f ′′ (3 ( ) +
f ′′′ (3 ...(1)
2! 3!
f ( x) = x 3 + 7 x 2 + x − 6, f (3) = 87
f ′ ( x) = 3 x + 14 x + 1,
2
f ′ (3) = 70
f ′′ ( x) = 6 x + 144,, f ′′ (3) = 32
f ′′′ ( x) = 6
6,, f ′′′ (3) = 6
Substituting in Eq. (1),
( x − 3) 2 ( x − 3)3
f ( x) = 87 + ( x − 3)(
)(70) + (32) + ( 6)
2! 3!
= 87 + 70( x − 3) + 16( x − 3) 2 + ( x − 3)3
2.2 Taylor’s Series 2.5

Example 6
Expand x 4 − 3 x3 + 2 x 2 − x + 1 in powers of (x – 3).
Solution
Let f ( x) = x 4 − 3 x 3 + 2 x 2 − x + 1
By Taylor’s series,

( x − a)2 ( x − a )3 ( x − a ) 4 IV
f ( x) = f (a) + ( x − a) f ′ (a) + f ′′ (a ) + f ′′′ (a ) + f (a ) +
2! 3! 44!!
Putting a = 3,
( x − 3) 2 ( x − 3)3
f ( x) = f (3) + ( x − 3) f ′ (3
( )+ ( )+
f ′′ (3 f ′′′ (3
( )
2! 3!
( x − 3) 4 IV ...(1)
+ f (3) +
4!

f ( x) = x 4 - 3x3 + 2 x2 - x +
+11, f (3) = 16
3 2
f ¢( x ) = 4 x - 9 x + 44xx - 1, f ¢(3) = 38
2
x ) = 12 x - 18 x + 4,
f ¢¢( x) f ¢¢(3) = 58
f ¢¢¢( x ) = 24 x - 18, f ¢¢¢(3) = 54
IV
f ( x ) = 24 f IV ( x ) = 24
Substituting in Eq. (1),

( x - 3)2 ( x - 3)3 ( x - 3)4


f ( x ) = 16 + ( x - 3)(
)(38) + (58) + (54) + (24)
2! 3! 4!
38( x - 3) + 29( x - 3)2 + 9( x - 3)3 + ( x - 3)4
= 16 + 38

Example 7
Expand 49 + 69 x + 42 x 2 + 11x3 + x 4 in powers of (x + 2).
Solution
Let f ( x) = 49 + 69 x + 42 x 2 + 11x 3 + x 4
By Taylor’s series,
( x − a)2 ( x − a )3
f ( x) = f (a ) + ( x − a ) f ′ (a ) + f ′′ (a ) + f ′′′ (a )
2! 3!
…(1)
( x − a ) 4 IV
+ f (a ) +
4!
2.6 Chapter 2 Taylor’s and Maclaurin’s Series

Putting a = – 2,
( x + 2 )2 ( x + 2)3
f ( x ) = f ( -2) + ( x + 2) f ¢( -2) + f ¢¢( -2) + f ¢¢¢( -2)
2! 3!
( x + 2)4 IV
+ f ( -2) + 
4!
f ( x ) = 49 + 69 x + 42 x 2 + 11x 3 + x 4 , f (-2) = 7
2 3
f ¢( x ) = 69 + 84 x + 33 x + 4 x , f ¢(-2) = 1
2
f ¢¢( x ) = 84 + 66 x + 12 x , f ¢¢(-2) = 0
f ¢¢¢( x ) = 66 + 24 x, f ¢¢¢(-2) = 18
f IV ( x ) = 24, f IV (-2) = 24
Substituting in Eq. (1),
( x + 2 )2 ( x + 2)3 ( x + 2)4
f ( x ) = 7 + ( x + 2)(1) + (0 ) + (18) + (24)
2! 3! 4!
= 7 + ( x + 2) + 3( x + 2)3 + ( x + 2)4

Example 8
Expand f (x) = x5 – x4 + x3 – x2 + x – 1 in powers of (x – 1) and find
f (0.99).
Solution
f (x) = x5 – x4 + x3 – x2 + x – 1
By Taylor’s series,
( x − a)2 ( x − a )3
f ( x) = f (a) + ( x − a) f ′ (a) + f ′′ ( a ) + f ′′′ (a ) + ......
2! 3!
Putting a = 1,

( x − 1) 2 ( x − 1)3
f (x) = f (1) + ( x − 1) f ′ (1
( )+ f ′′(1
( )+ f ′′′ (1
()
2! 3!
( x − 1) 4 iv ( − 1)5 v
(x
+ f (1( )+ f (1 ( ) + ....... ... (1)
4! 5!

f ( x) = x 5 − x 4 + x 3 − x 2 + x − 1, f (1) = 0
4 3 2
f ¢(x) = 5x – 4x + 3x – 2x + 1, f ¢(1) = 5 – 4 + 3 – 2 + 1 = 3
f ¢¢(x) = 20x3 – 12x2 + 6x – 2, f ¢¢(1) = 20 – 12 + 6 – 2 = 12
f ¢¢¢(x) = 60x2 – 24x + 6, f ¢¢¢(1) = 60 – 24 + 6 = 42
f I V (x ) = 120x – 24, f I V (1) = 120 – 24 = 96
f V (x) = 120, f V (1) = 120
2.2 Taylor’s Series 2.7

Substituting in Eq. (1),


2 3 4
)5
x ( )+ ( )+ + (120)
2 3 4 5
2 3 4 5

Putting x = 0.99,
f (0.99) = 3 (0.99 – 1) + 6 (0.99 – 1)2 + 7 (0.99 – 1)3 + 4 (0.99 – 1)4 + (0.99 – 1)5
= 3 (– 0.01) + 6 (– 0.01)2 + 7 (– 0.01)3 + 4 (– 0.01)4 + (– 0.01)5
= – 0.02939 approx.

Example 9
Expand + in ascend-
ing powers of (x – 1).
Solution
f x x x + x x )+8
By Taylor’s series,
( x a)2 x a 3
( x a)4
f x) = a + ( x a) f a) + f a) + f a) + f ( ) +
2 3!
Putting a = 1,
(x )2 x 3
(x )4
f x) x ) f ′( ) + f ′′ ( ) + f ′′′ ( ) + f IV (1) + ...(1)
2! 3!
f x) = x + x x + = 299
3
f x) = x ) + + + 2) + 7, (1) = 286
f ( )= + + 2) + 12, f ¢¢( ) 210
f x) = + + 1) 102
f x) = =2
Substituting in Eq. (1),
2 3
(x )4
= + x ( )+ ( )+ (2 )
2 3 4
= + 286 14

Example 10
1 1 (x x + 2) 2 ( x + 2)3
Prove that + + + +
1 x 3 32 33 34
Solution
1
Let f x) =
1 x
2.8 Chapter 2 Taylor’s and Maclaurin’s Series

By Taylor’s series,

( x − a)2 ( x − a )3
f ( x) = f (a) + ( x − a) f ′ (a) + f ′′ ( a ) + f ′′′ (a ) + ... ...........
......
2! 3!
Putting a = – 2,

( x + 2) 2 ( x + 2) 3
f ( x) = f ( −2) + ( x + 2) f ′ ( −2) + f ′′ ( −2) + f ′′′ ( −2) + ...........
....... ... (1)
2! 3!
1 1
f ( x) = , f ( −2
−2) =
1− x 3
1 1
f ′′(( x) = , f ′ ( −2
−2) = 2
( − x) 2
(1 3
2 2!
f ′′ ( x) = , f ′′ ( −2
−2) = 3
((1 − x)3 3
2⋅3 3!
f ′′′ ( x) = , f ′′′ ( −
−2
2) = 4 and so on
(1 − x) 4 3
Substituting in Eq. (1),
1 ((xx + 2) ( x + 2) 2 ( x + 2)3
f ( x) = + + + + ............
3 32 33 34

Example 11
Expand log x in powers of (x – 1).
Solution
Let f ( x) = log x
By Taylor’s series,

( x − a)2 ( x − a )3
f ( x) = f (a) + ( x − a) f ′ (a) + f ′′ (a ) + f ′′′ (a ) + 
2! 3!
( x − a)n n
+ f (a ) +
n!

Putting a = 1,

( x − 1) 2 ( x − 1)3
f ( x) = f (1) + ( x − 1) f ′ (1) + f ′′ (1) + f ′′ (1) +  ...(1)
2! 3!
f ( x) = log
log x, f (1) = log
log 1 = 0
1
f ′ ( x) = , f ′ (1) = 1
x
2.2 Taylor’s Series 2.9

1
f ′′ ( x) = − , 1) = −1
f ′′ (1)
x2
2
′′ ′ ( x) =
f ′′′ , f ′′′ 1)) = 2 aand so on
′′ ′ (1
x3

Substituting in Eq. (1),

( x - 1)2 ( x - 1)3
f ( x ) = 0 + ( x - 1)(
)(1) + ( -1) + (2 ) + 
2! 3!
( x - 1)2 ( x - 1)3
g x = ( x - 1) -
log + -
2 3

Example 12
Expand log sin x in powers of (x – 2). [Summer 2014]
Solution
Let f (x) = log sin x
By Taylor’s series,
( x - a )2 ( x - a )3
f ( x ) = f ( a ) + ( x - a ) f ¢( a ) + f ¢¢(a ) + f ¢¢¢(a ) + 
2! 3!
Putting a = 2,
( x - 2 )2 ( x - 2)3
f ( x ) = f (2) + ( x - 2) f ¢(2
( )+ ( )+
f ¢¢(2 ( ) +
f ¢¢¢(2 ...(1)
2! 3!
f ( x ) = log
log sin
sin x, f (2) = log
log sin
sin (2)
cos x
f ¢( x ) = = cott x, f ¢(2) = cot
cot (2)
sin x
f ¢¢( x ) = - cosecc 2 x, cosec 2 (2)
f ¢¢(2) = - cosec
f ¢¢¢( x ) = 2 cosecc 2 x cot
cot x, f ¢¢ (2) = 2 cosecc 2 (2) cot
cot (2) and so on
Substituting in Eq. (1),

( x - 2 )2 È
f ( x ) = log sin (2) + ( x - 2) cot(2) +
log sin - cosecc 2 (2)˘˚
2! Î
( x - 2)3 È
+ 2 cot((2)˘˚ + 
2 cosec 2 (2)
3! Î
( x - 2 )2
log sin x = log sin (2) + ( x - 2) cot
log sin cot (2) - cosecc 2 (2)
2
( x - 2)3
+ cosecc 2 (2) cot(2) + 
3
2.10 Chapter 2 Taylor’s and Maclaurin’s Series

Example 13
Expand log (cos x) about
3
Solution
Let f (x) = log (cos x)
By Taylor’s series,
2
x a
f x) = a + (x a a a) + f a) + … …
2 3!

p
Putting a = ,
3
2 3
p p 1 p p
f x) f x- f x- ′′ + - f + ... (1)
3 3 3 3 3 3

1
f x) = log(cos x f log cos log = − log 2
3 2

1 p p
f x s x x tan − 3
cos x 3 3
p p
f x) = − sec 2 x f = − sec −4
3 3

p p
f x) −2 c x x ′′′ sec tan
3 3 3
= −2 ) 3
= −8 3 so on

Substituting in Eq. (1),


2
p p
f x ) = - log 2 + x x- ( )
3 2! 3

+
1
3!
x-
p
3
( )
8 3 +…

2 3
p p 4 3 p
log (co =- x 2 x x
3 3 3 3
2.2 Taylor’s Series 2.11

Example 14
Obtain tan–1 x in powers of (x – 1).
Solution
Let f (x) = tan–1 x
By Taylor’s series,

( x − a)2 ( x − a )3
f ( x) = f (a) + ( x − a) f ′ (a) + f ′′ ( a ) + f ′′′ (a ) + ……
2! 3!
Putting a = 1,

( x − 1) 2 ( x − 1)3
f ( x) = f (1) + ( x − 1) f ′ ((11) + f ′′ (1
( )+ f ′′′ (1
( ) +… ... (1)
2! 3!
p
tan −−11 x,
f ( x) = tan tann −1 1 =
f ((11) = ta
4
1 1
f ′′(( x) = , f ′ (1) =
1 + x2 2
2x 2 1
f ′′ ( x) = − , f ′′ (1) = − =−
((1 + x 2 ) 2 4 2

2 8x2 1
f ′′′ ( x) = − + , f ′′′ (1) = and so on
( +x )
(1 2 2
((1 + x 2 )3 2

Substituting in Eq. (1),

p Ê 1 ˆ ((xx - 1)2 Ê 1 ˆ ( x - 1)3 Ê 1 ˆ


f ( x) = + ( x - 1) Á ˜ + ÁË - 2 ˜¯ + + 
3! ÁË 2 ˜¯

4 Ë 2¯ 2!
p 1 1 1
tann -1 x = + ( x -1 - 1)2 +
- 1) - ( x -1 ( x - 1)3 + 
4 2 4 12

Example 15
Express 7 + ( x + 2) + 3( x + 2)3 + ( x + 2) 4 − ( x + 2)5 in ascending powers
of x.
Solution
Let f ( x + 2) = 7 + ( x + 2) + 3( x + 2)3 + ( x + 2) 4 − ( x + 2)5
f ( x) = 7 + x + 3x3 + x 4 − x5
2.12 Chapter 2 Taylor’s and Maclaurin’s Series

By Taylor’s series,
h2 h3 h 4 IV h5 V
f ( x + h ) = f ( x ) + h f ¢( x ) + f ¢¢( x ) + f ¢¢¢( x ) + f ( x) f ( x) + 
2! 3! 4! 5!
Putting h = 2,
f ( x + 2) = (7 + x + 3 x 3 + x 4 − x 5 ) + 2(1 + 9 x 2 + 4 x3 − 5 x 4 )
22 23
+ (18 x + 12 x 2 − 20 x 3 ) + (18 + 24 x − 60 x 2 )
2! 3!
4 5
2 2
+ (24 − 120 x) + ( −120)
4! 5!
= 17 − 11x − 38 x 2 − 29 x 3 − 9 x 4 − x 5

Example 16
Express (x – 1)4 + 2(x – 1)3 + 5(x – 1) + 2 in ascending powers of x.
[Summer 2016]
Solution
Let f ( x - 1) = ( x - 1)4 + 2( x - 1)3 + 5( x - 1) + 2
f ( x) = 2 + 5x + 2 x3 + x 4
By Taylor’s series,
h2 h3 h 4 iv
f ( x + h ) = f ( x ) + hf ¢( x ) + f ¢¢( x ) + f ¢¢¢( x ) + f ( x) + 
2! 3! 4!
Putting h = –1,

(-1)2 (-1)3 (-1)4 iv


f ( x - 1) = f ( x ) + (-1) f ¢( x ) + f ¢¢( x ) + f ¢¢¢ + f ( x) + 
2! 3! 4!
= (2 + 5 x + 2 x 3 + x 4 ) + (-1) (5 + 6 x 2 + 4 x 3 )
1 (-1) 1
+ (12 x + 12 x 2 ) + (12 + 24 x ) + (24) + 0
2! 3! 4!
= (2 + 5 x + 2 x 3 + x 4 ) + (-1)(5 + 6 x 2 + 4 x 3 ) + (6 x + 6 x 2 ) + (-1)(2 + 4 x ) + 1

= 2 + 5x + 2 x3 + x 4 - 5 - 6 x2 - 4 x3 + 6 x + 6 x2 - 2 - 4 x + 1

= x 4 - 2 x3 + 7 x - 4
2.2 Taylor’s Series 2.13

Example 17
Express 5 + 4((xx − 1) 2 − 3( x − 1)3 + ((xx − 1) 4 in ascending powers of x.
[Winter 2013]
Solution
Let f ( x − 1) = 5 + 4( x − 1) 2 − 3( x − 1)3 + ( x − 1) 4

f ( x) = 5 + 4 x 2 − 3x3 + x 4
By Taylor’s series,
h2 h3 h 4 IV
f ( x + h) = f ( x ) + h f ′ ( x ) + f ′′ ( x) + f ′′′ ( x) + f ( x) + 
2! 3! 4!
Putting h = – 1,
((−1) 2 ((−1)3 ((−1) 4 IV
f ( x − 1) = f ( x) + ( −1) f ′ ( x) + f ′′ ( x) + f ′′′ ( x) + f ( x) +
2! 3! 4!
( −1) 2
= ((55 + 4 x 2 − 3 x 3 + x 4 ) + ( −1))(((88 x − 9 x 2 + 4 x 3 ) + 8 − 18 x + 12 x 2 )
((8
2!
( −1)
3
( 1) 4
(−
+ ((−18 + 24 x) + ( )
(24
3! 4!
= 5 + 4 x 2 - 3 x 3 + x 4 - 8 x + 9 x 2 - 4 x 3 + 4 - 9 x + 6 x 2 + 3 - 4x
4x +1
= x 4 - 7 x 3 + 19 x 2 - 21x + 13

Example 18
 p
Find the expansion of tan  x +  in ascending powers of x up to terms
 4
in x4 and find approximately the value of tan (43°).
[Winter 2013; Summer 2016]
Solution
 π  π
Let f  x +  = tan  x + 
 4  4
f ( x) = tan x
By Taylor’s series,

h2 h3 h 4 IV
f ( x + h) = f ( x) + hhff ′ ( x) + f ′′ ( x) + f ′′′ ( x) + f ( x) + 
2! 3! 4!
2.14 Chapter 2 Taylor’s and Maclaurin’s Series

Putting x ,
4
π x2 π x3 4
f x f′ + f + f ′′′ + f + ...(1)
4 2 4 3! 4 4 4
π π
f x) = tan x f = =1
4 4
π
f x) = c2 x ′ =2
4 4
π π
f x) = ⋅ x tan x f ′′ = + tan 3 4
4 4 4
= 2 sec x t x
= 2
tan x an x
= tan an 3 x

f ′′ x) = + 2
x sec 2 x ′′′ = + n + =
4 4 4
= + tan 2 x + + 2
)
= 8 tan an 4 x
2 2
f x) = ta x x, ◊ c2
4 4 4
p
+ ◊ sec
4 4
80 and so on
Substituting in Eq. (1),

x2 x3 x4
f x x + + (16) + 80 + 
2 3! 4!
…(2)
10
tan +x
4 3 3

Now tan tan( )


π π
tan
4 180

tan − 0.0349
4
2.2 Taylor’s Series 2.15

8 10
= 1 + 2( −
−0
0.0349) + 2( −0.0349) 2 + ( −
−00.0349)3 + ( −0.0349) 4
3 3
= 0 ⋅ 9326 approx.
approx.

Example 19
Prove that
h2 h3 co
coss x
loggg[sin (x + h)]
)] = log sinn x + h cot
cot x − cosec 2 x + + 
2 3 sin 3 x
Solution
Let f (x + h) = log [sin (x + h)]
f (x) = log (sin x)
By Taylor’s series,
h2 h3
f ( x + h) = f ( x) + h f ′( x) + f ′′( x) + f ′′′( x) + ………… ...(1)
2! 3!
1
f ′ ( x) = cos x = cot x
cos
sin x
f ′′ ( x) = − cosec 2 x

2 cos x and so on
f ′′′( x) = 2 cosec
cosec 2 x cot x =
sin 3 x
Substituting in Eq. (1),

h2 h3 2 cos x
f ( x + h) = log sin x + h cot x − cosec 2 x + + ………
2! 3! sin 3 x
h2 h3 cos x
log [sin(x
sin( x + h) ] = log sin x + h cott x − cosec 2 x + + ………
2 3 sin 3 x

Example 20
 π
Expand log cos  x +  using Taylor’s theorem in ascending powers of
 4
x and hence find the value of log (cos 48°) correct up to three decimal
places.
2.16 Chapter 2 Taylor’s and Maclaurin’s Series

Solution
 π  π
Let f  x +  = log cos  x + 
 4  4
f ( x) = log cos x
By Taylor’s series,
h2 h3
f ( x + h) = f ( x) + hhff ′ ( x) + f ′′ ( x) + f ′′′ ( x) + 
2! 3!
π
Putting x = , h = x,
4
π  π π x π
2
f  + x = f   + xf xf ′   + f ′′   +  ...(1)
4   4  4  2!  4
Êpˆ p Ê 1 ˆ
f ( x ) = log cos x, f Á ˜ = log cos = log Á
log cos = - log 2
Ë 4¯ 4 Ë 2 ˜¯
1 Êpˆ p
f ¢( x ) = ( sin x ),
(- f ¢ Á ˜ = - tan = -1
cos x Ë 4¯ 4
= - tan x
Êpˆ Êpˆ
f ¢¢( x ) = - secc 2 x, f ¢¢ Á ˜ = - secc 2 Á ˜ = -2 and so on
Ë 4¯ Ë 4¯
Substituting in Eq. (1),
π  x2
f  + x = − log
g 2 + x( −1) + ( −2) + 
4  2!
π 
log cos  + x = − log
log 2 − x − x 2 +  ...(2)
4 
Now, g [ cos
log(coss 48°) = log cos((45° + 3°) ]
  π 3π  
= log cos  + 
  4 180  
 π 
= log cos  + 0.0523 
 4 

= − log
g 2 − 0.0523 − (0
( .0523) 2
= −0.402 approx.
2.2 Taylor’s Series 2.17

Example 21
sin a sin 2a
Show that tan -1 ( x + h ) = tan -1 x + (h sin a ) - (h sin a )2
1 2
sin 3a
+(h sin a )3 +
3
where a = cot–1x. [Summer 2015]
Solution
Let f (x + h) = tan–1 (x + h)
f (x) = tan–1 x
By Taylor’s series,
h2 h3
f ( x + h ) = f ( x ) + h f ′( x ) + f ′′ ( x ) + f ′′′( x ) +  ... (1)
2! 3!
1
f ′( x ) =
1 + x2
2x
f ′′( x ) = −
((1 + x 2 )2

2 2x ◊ 4x 2(3 x 2 - 1)
f ′′′( x ) = - + = and so on
((1 + x 2 )2 ((1 + x 2 )3 ((1 + x 2 )3
Putting x = cot a,
1 1
f ¢(cot a ) = 2
= 2
= sin 2 a
1 + cot a cosec a
2 cot a cos a
f ¢¢(cot a ) = - 2 2
= - 2 sin 2 a sin 2 a
(cosec a ) sin a
= - 2 sin 2 a sin a cos a = - sin 2 a sin 2a
2(3 cot 2 a - 1) 2(3 cos2 a - sin 2 a )
f ¢¢¢(cot a ) = =
(1 + cot 2 a )3 cosec6 a ◊ sin 2 a
= 2(3 - 4 sin 2 a )sin 4 a = 2(3 sin a - 4 sin 3 a )sin 3 a
= 2 sin 3a sin 3 a
Substituting in Eq. (1),

h2 h3
f ( x + h ) = tan -1 x + h sin 2 a + sin 2 a sin
( - sin sin 2a ) + (2 sinsin 3 a sin
sin 3a ) + 
2! 3!
Ê sin
sin a ˆ sinn 2a
si sin 3a
= tan -1 x + h sin a Á - (h sin a )2 + (h sin a )3 +
Ë 1 ˜¯ 2 3
2.18 Chapter 2 Taylor’s and Maclaurin’s Series

Example 22
Expand tan–1 (x + h) in powers of h and hence, find the value of
tan–1 (1.003) up to 5 places of decimal.
Solution
Let f (x + h) = tan–1 (x + h)
f (x) = tan–1 x
By Taylor’s series,
h2 h3
f ( x + h) = f ( x ) + h f ′ ( x ) + f ′′ ( x ) + f ′′′ ( x) + ………… ... (1)
2! 3!
1
f ′ ( x) =
1 + x2
2x
f ′′ ( x) = −
((1 + x 2 ) 2

2 2x ⋅ 4x 2(3 x 2 − 1)
2(
f ′′′ ( x) = − + = and so on
( + x 2 ) 2 ((1 + x 2 )3
(1 ((1 + x 2 )3
Substituting in Eq. (1),

1 h2  2x 
f ( x + h) = tan −−11 ( x + h
h)) = tan −1 x + h ⋅ +  −
1+ x 2
2 !  (1( + x 2 ) 2 
h3  2(3 x 2 − 1) 
+   +………
3!  (1 + x 2 )3 
Putting x = 1, h = 0.003,

. )2  2  ((0.003)  1
3
0.003
. (0.003
tann −−11 (1 + 0..003) = ta
tann −1 1 + +  −  +   +………
2 2! 4 3!! 2

π
n −−11 (1..003) =
tan 00015 − 2.25 × 10 −8 + 2.25 × 10 −12
+ 0..00015 [Considering first 4 terms]
4
= 0.78540 approx.

Example 23
x 7 x 2 7 x3
1+ x + 2 x 2 = 1 +
Prove that 1 + − + .
2 8 16
solution
Let f ( x) = x
f ( x + h) = x+h
2.2 Taylor’s Series 2.19

By Taylor’s series,
h2 h3
f ( x + h) = f ( x ) + h f ′ ( x ) + f ′′ ( x ) + f ′′′ ( x) + ………
2! 3!

Putting x = 1, h = x + 2x 2 ,
f ( x + h) = x + h = 1 + x + 2x2

((xx + 2 x 2 ) 2 ((xx + 2 x 2 )3
= f (1) + ( x + 2 x 2 ) f ′ ((11) + f ′′ (1
( )+ f ′′′ (1
( ) + ……… ... (1)
2! 3!

f ( x) = xx,, f (1) = 1
1 1
f ′ ( x) = , f ′ (1) =
2 x 2
1  1 1 1
f ′′ ( x) =  −  3 , f ′′ (1) = −
2  2 2 4
x
1  1  3 1 3
f ′′′ ( x) =  −   −  5 , f ′′′ (1) = and so on
2  2  2 2 8
x

Substituting in Eq. (1),

1 1 ( x 2 + 4 x 3 + 4 x 4 ) 3 ( x 3 + …)
11+ x + 2 x 2 = 1 + ( x + 2 x 2 ) − + + .........
2 4 2 8 6
x 7 x 2 7 x3
= 1+ + − +…
2 8 16

Example 24
Expand 1 + x + 2 x 2 in powers of (x – 1).
Solution

11+ x + 2 x 2 = 4 + 2 ( x −
−11) 2 + 5 ( x − 1) [Expressing in terms of (x – 1)]

Let f ( x) = x
f ( x + h) = x+h
By Taylor’s series,
h2 h3
f ( x + h) = f ( x) + h f ′( x) + f ′′( x) + f ′′′( x) + ………
2! 3!
2.20 Chapter 2 Taylor’s and Maclaurin’s Series

Putting x = 4, h = 2 (x – 1)2 + 5 (x – 1),

f x x+h x + x )

− 2
= + x− + (x ′′ ( ) + … ... (1)
2!
f x) 2
1 1
f x
2 x 4
1 1 1 1
f x) − 3
f ( )=− and so on
2 2 2
32
x
Substituting in Eq. (1),
1
4 - - + + 1)]
4
2
2 1 1
+ - + …
2! 32

1 x2 + + …………
4 64

Example 25
Show that
1 ( )3 5
[ ( x ) ( 2a x ) f ′( ) + f ( + f ( )+
2 3 5!
Solution
By Taylor’s series,
( x a)2 ( x a )3 ( x a)4
f x) = a + ( x a) f a) + f a) + f a) + f ( )
2! 3! !
( )5
+ f ( ) + ...(1)
5!
h2 3
h4 5
f x h f (x f x) + f x + f x) + x + f ( x) + …(2)
2! 3! 4! 5!

Now, x)]
Putting x = a, h = a – x in Eq.(2),
2.2 Taylor’s Series 2.21

(a − x) 2 ( a − x )3
f [a + (a − x)] = f (a ) + (a − x) f ′ (a ) + f ′′ (a ) + f ′′′ (a )
2! 3!
((a − x) 4 IV ((a − x)5 V
+ f (a) + f (a) + 
4! 5!
( x − a)2 ( x − a )3
f ( 2a − x ) = f ( a ) − ( x − a ) f ′ ( a ) + f ′′ (a ) − f ′′′ (a )
22!! 3! ...(3)
( x − a ) 4 IV ( x − a )5 V
+ f (a) − f (a) + 
44!! 5!
From Eqs (1) and (3),
1 1 ( x − a )3 ( x − a )5 V 
2
[ f ( x) − f (2a − x) ] =  22(( x − a ) f ′ (a ) + 2 ⋅
2 3!
f ′′′ (a ) +2 ⋅
5!
f (a ) + 

((x − a )3 ((x − a )5 V
= ((xx − a ) f ′ (a ) + f ′′′ (a ) + f (a) + 
3! 5!

Example 26
Using Taylor’s theorem, evaluate up to 4 places of decimals:
(i) 1.002 (ii) 25.15 (iii) 9 12
9.
(iv) 10 (v) 36.12 [Winter 2014]
Solution
Let f ( x) = x
f ( x + h) = x+h
By Taylor’s series,
h2 h3
f ( x + h) = f ( x) + h f ′( x) + f ′′( x) + f ′′′( x) + ………… ... (1)
2! 3!
(i) Putting x = 1, h = 0.02,
f ( x + h) = x + h = 1 + 0..02
02 = 1.02
(0.02) 2
= f (1) + (0.02) f ′ (1) + f ′′ ((1) + ………
… ... (2)
2!
f ( x) = xx,, f (1) = 1
1 1
f ′ ( x) = , f ′ (1) =
2 x 2
1 1
f ′′ ( x) = − 3 , f ′′ (1) = − and so on
4
4x 2
2.22 Chapter 2 Taylor’s and Maclaurin’s Series

Substituting in Eq. (2) and considering only first 3 terms,


 1  (0.02)  1 
2
02 = 1 + (0.02
1.02 02)   + − 
 2 2!  4 
= 1.0099 approx.
(ii) Putting x = 25, h = 0.15 in Eq. (1),
f ( x + h) = x + h = 25 + 0.15

(0.15) 2
= f (25) + (0.15) f ′(25) + f ′′((25
25) + … ... (3)
2!
f ( x) = x, f (25) = 5
1 1
f ′ ( x) = , f ′ (25) = = 0.1
2 x 10
1 1
f ′′ ( x) = − 3 , f ′′ (25) = − = − 0.002 and so on
500
4x 2
Substituting in Eq. (3) and considering only first 3 terms,
(0.15) 2
25.15 = 5 + (0.15) (0.1) + ( − 0.002)
2
= 5.0150 approx.
(iii) Putting x = 9, h = 0.12 in Eq. (1),
f ( x + h) = x + h = 9 + 0.12
(0.12) 2
= f (9) + (0.12) f ′(9) + f ′′((99) + … ... (4)
2!
f ( x) = xx,, f (9) = 3
1 1
f ′ ( x) = , f ′ (9) =
2 x 6
1 1
f ′′ ( x) = − 3 , f ′′ (9) = − and so on
108
4x 2
Substituting in Eq. (4) and considering only first 3 terms,
 1  (0.12)  1 
2
12 = 3 + (0.12
9.12 12)   + − 
 6 2  108 
= 3 + 0.02 − (0.12
12) (0.0
066) (0.0093)
= 3.0199 approx.

(iv) Putting x = 9, h = 1 in Eq. (1),


1
f ( x + h) = x + h = 9 + 1 = f (9) + f ′(9) + f ′′(9
( ) +… ... (5)
2!
2.2 Taylor’s Series 2.23

1 1 [Refer (iii)]
10 = 3 + −
6 216
= 3.1620 approx.

(v) Putting x = 36, h = 0.12 in Eq. (1),


f ( x + h) = x + h = 36 + 0.12 = 36 ◊ 12
(0.12)2
= f (36) + (0.12) f ′(36) + f ′′(36
6)) + ... (6)
2!
f ( x) = x, f (36) = 36 = 6
1 1 1
f ′( x ) = , f ′(36) = =
2 x 2 36 12
1 1 1
f ′′( x ) = - 3 , f ′ (36
(36) = - 3
=- and so on
864
4x 2 4(36) 2
Substituting in Eq. (6) and considering only first 3 terms,

Ê 1 ˆ (0.12)2 Ê 1 ˆ
36.12 = 6 + (0.12) Á ˜ + ÁË - 864 ˜¯ + 
Ë 12 ¯ 2!
= 6.0099 approx
x.

Example 27
Find cosh (1.505) given sinh (1.5) = 2.1293 and cosh (1.5) = 2.3524.
Solution
Let f (x) = cosh x
By Taylor’s series,
h2 h3
f ( x + h) = f ( x ) + h f ′ ( x ) + f ′′ ( x ) + f ′′′ ( x) + …………
2! 3!
Putting x = 1.5, h = 0.005,
f (x + h) = cosh (x + h) = cosh (1.5 + 0.005)

( .005) 2
(0 (0.005)3
(0
= f (1.5) + (0.005) f ′(1.5) + f ′′(1.5) + f ′′′ (1 5) + ... (1)
2! 3!
f ( x) = cosh x, f (11..5) = cosh (1.5) = 2.3524
f ¢(x) = sinh x, f ¢(1.5) = sinh (1.5) = 2.1293
f ¢¢(x) = cosh x, f ¢¢(1.5) = cosh (1.5) = 2.3524 and so on
2.24 Chapter 2 Taylor’s and Maclaurin’s Series

Substituting in Eq. (1) and considering only first 3 terms,


(0.005) 2
cosh (1.505) = cosh (1.5) + (0.005) sinh (1.5) + 1.5) + …
coosh (1.
2!
= 2.3524
2.3524
. + (0.005 ( .1293) + (12.5) (10−6 )(
. )((2 )(2.3524)
= 2.33631
631 approx.

Example 28
Find approximate value of sin (30°30¢).
Solution
Let f (x) = sin x
 π 30 π 
sin(30°30 ′ ) = sin
ssin( 30° + 30 ′ ) = sin
in(((30 sin  + ⋅
 6 60 180 
π 
= sin
sin  + 0.0087
6 
By Taylor’s series,
h2
f ( x + h) = f ( x ) + h
hff ′ ( x) +
f ′′ ( x) + 
2!
h2
sin( x + h)
h) = sin x + h cos
cos x + (−( siin n x) +
2!
π
Putting x= , h = 0.0087,
6
π  π  π  ((0.0087) 2  π
n  + 0.0087 = sin
sin sin + (0.0087
. )  cos  +  − siin
n 
6  6  6  2! 6
[Considering first 3 ter
[Considering terms
ms]
sin 30° 30¢ = 0.50752 approx.

EXERCisE 2.1
1. Expand ex in powers of (x − 1).
  (x − 1)2 (x − 1)3 
 Ans.: e  1 + ( x − 1) + + + … 
 2! 3! 
2. Expand 2x3 + 7x2 + x − 1 in powers of (x − 2).

[Ans.: 45 + 53 (x – 2) + 19 (x – 2)2 + 2 (x – 2)3]


2.2 Taylor’s Series 2.25

3. Expand x5 − 5x4 + 6x3 − 7x2 + 8x – 9 in powers of (x – 1).


 Ans.: − 6 − 3 ((x x − 1)3 + (x − 1)5 
x − 1) − 9 (x − 1)2 − 4 ((x
4. Expand x4 − 3x3 + 2x2 − x + 1 in powers of (x – 3).
 Ans.: 16 + 38 ((x
x − 3) + 29 9((x − 3)3 + (x − 3)4 
9((x − 3)2 + 9
29

5. Expand x3 − 2x2 + 3x − 5 in powers of (x – 2).


[Ans.: 11 + 7 (x – 2) + 4 (x – 2)2 + (x − 2)3]
6. Expand 2x3 + 3x2 − 8x + 7 in terms of (x – 2).
[Ans.: 19 + 28 (x – 2) + 15 (x – 2)2 + 2 (x − 2)3]
7. Expand 2x3 + 5x2 + 3x – 4 in powers of (x + 3).
[Ans.: –22 + 27(x + 3) – 13(x + 3)2 + 2(x + 3)3]
8. Expand x in powers of (x − a).
 (x − a) (x − a)3 
 Ans.: a+ − − …
 2 a 8a a 

9. Expand 1 + x + 2x 2 in powers of (x − 1).


 5 7 
 Ans.: 2 + 4 (x − 1) + 32 (x − 1) + …
2

 
10. Expand sin x in powers of (x − a).

 (x − a)2 (x − a)3 
 Ans.: sin
n a + ( x − a)c
)cos
os a − sin a − cos a + …
 2! 3! 
 p
11. Expand cos x in powers of  x −  .
 2

  p 1 p
3
1 p
5

 Ans.: −  x −  +  x −  −  x −  + …
  2  3!  2 5!  2 

 p
12. Expand tan x in powers of  x −  .
 4

  p  p
2

 Ans.: 1 + 2  x −  + 2  x −  + … 
  4  4 

p 
13. Expand sin  + x  in powers of x up to x 4.
6 

 1 3 1 x2 3 x3 1 x4 
 Ans.: + x− ⋅ − ⋅ + ⋅ +… 
 2 2 2 2! 2 3! 2 4 ! 
2.26 Chapter 2 Taylor’s and Maclaurin’s Series

p 
14. Expand tan  + x  in powers of x up to x4 and hence, find the value
4 
of tan (46° 36').

  8 3 10 4  
 Ans.:  1 + 2 x + 2 x + x + x + … , 1.0574
2

 3 3 
15. Find approximate value of cos 64°.
[Ans.: 0.4384]
16. Expand log x in powers of (x – 2).
 1 x − 2)2
1 ((x 1 (x − 2)3 
g 2 + (x − 2) − ⋅
 Ans.: log + ⋅ + …
 2 2! 4 3! 4 
p 
17. Expand log tan  + x  in powers of x.
4 
 4 3 4 5 
 Ans.: 2 x + 3 x + 3 x + … 
 
18. Expand 7 + (x + 2) + 3(x + 2)3 + (x + 2)4 in powers of x.
[Ans.: 49 + 69x + 42x2 + 11x3 + x 4]

19. Expand 17 + 6(x + 2) + 3(x + 2)3 + (x + 2)4 − (x + 2)5 in powers of x.


[Ans.: 37 − 6x − 38x2 − 29x3 − 9x 4 − x5]

20. Expand (x – 2)4 – 3(x – 2)3 + 4(x – 2)2 + 5 in powers of x.


[Ans.: 61 – 84x + 46x2 – 11x3 + x4]

21. Expand (x + 2)4 + 5(x + 2)3 + 6(x + 2)2 + 7(x + 2) + 8 in powers of (x + 1).
hint : f (x) = x 4 + 5x 3 + 6 x 2 + 77xx + 8, f [((xx + 1) + 1] = f (1)
 
 (x + 1)2

+(((x
+ x + 1)f ′(1) + f ′′ (1) + …
 2! 

 Ans.: 27
7 + 38(
38(x + 1) + 27(x + 1)2 + 9( 1)3 + (x + 1)4 
x + 1)
9((x

x2
22. Prove that sinh (x + a) = sinh a + x cosh a + sinh a + … If
2!
sinh (1.5) = 2.1293, cosh (1.5) = 2.3524, find the value of sinh (1.505).
[Ans.: 2.1411]
2.3 Maclaurin’s Series 2.27

2.3 MAClAuRin’s sERiEs


Statement If f (x) is a given function of x which can be expanded in positive ascend-
ing integral powers of x then
x2 x3 xn n
f ( x) = f (0) + x f ′(0) + f ′′ ((0) + f ′′′ (0) +
+………
……… + f (0) + ………
2! 3! n!
Proof Let f (x) be a function of x which can be expanded into positive ascending
integral powers of x.

f (x) = a0 + a1x + a2 x2 + a3 x3 + a4 x4 +……. … ... (1)

Differentiating w.r.t. x successively,


f ¢(x) = a1 + a2· 2x + a3 · 3x2 + a4 · 4x3 + ……. ……… ... (2)
2
f ¢¢(x) = a2 · 2 + a3 · 6x + a4 · 12x + ……. ……… ... (3)
f ¢¢¢(x) = a3 · 6 + a4 · 24x + ……. ……… ... (4)
and so on.
Putting x = 0 in Eq. (1), (2), (3) and (4),
a0 = f (0)

a1 = f ¢(0)

1
a2 = f ′′(0)
2!
1
a3 = f ′′′(0) and so on.
3!
Substituting a0, a1, a2 and a3 in Eq. (1),

x2 x3 xn n
f ( x) = f (0) + x f ′(0) + f ′′ ((0) + f ′′′ (0) +
+………
……… + f (0) + ………
2! 3! n!

This is known as Maclaurin’s series.


This series can also be written as,
x2 x3 xn
y = y (0) + xy1 (0) + 0) +
y2 ((0 y3 (0) +
+……
…… + yn (0) + ………
2! 3! n!

standard Expansions
Using Maclaurin’s series, expansion of some standard functions can be obtained.
These expansions can be directly used while solving the examples.
(i) Expansion of ex (Exponential series)
Proof Let y = ex, y (0) = e0 = 1
2.28 Chapter 2 Taylor’s and Maclaurin’s Series

dn
Now, yn = (e x ) = e x , yn (0) = e0 = 1
dx n
Substituting in Maclaurin’s series,
x 2 x3
ex = 1 + x + + + ………
2 ! 3!
This series is known as the exponential series.
(a) Replacing x by −x in the above series,

x 2 x3
e− x = 1 − x +
− + ………
2 ! 3!
(b) Replacing x by ax in the above series,
a 2 x 2 a3 x3
eax = 1 + ax + + +
+………
2! 3!
(ii) Expansion of sin x (Sine series)
Proof Let y = sin x, y (0) = sin 0 = 0
dn  np  y (0  np 
Now, yn = (sin x)
x ) = s in  x +  , n ( ) = sin  
dxx n  2   2 
Putting n = 1, 2, 3, 4, 5, ..….. ,

y1 (0) = 1, y2 (0) = 0, y3 (0) = – 1, y4 (0) = 0, y5 (0) = 1, and so on.


Substituting in Maclaurin’s series,
x3 x5
sin x = x − + − ………
3! 5 !
3!
This series is known as the sine series.
(iii) Expansion of cos x (Cosine series)
Proof Let y = cos x, y (0) = cos 0 = 1
dn  np   np 
Now, yn = (cos xx)) = cos  x +  ( ) = coss   ,
, yn (0
dx
dx n
 2   2 

Putting n = 1, 2, 3, 4, ….. ,
y1 (0) = 0, y2 (0) = –1, y3 (0) = 0, y4 (0) = 1, and so on.
Substituting in Maclaurin’s series,
x 2 x 4 ………
cos x = 1 −
+ −
2! 4!
2!
This series is known as the cosine series.
2.3 Maclaurin’s Series 2.29

(iv) Expansion of tan x (Tangent series)


Proof Let y = tan x, y(0) = 0
y1 = sec x = 1
2
1++ tan x = 1 + y ,
2 2
0)) = 1
y1 (0
y2 = 2 yyyy1 , y2 (0) = 2 y (0) y1 (0
0)) = 2(0)((1) = 0
y3 = 2 y + 2 yy2 ,
1
2
0)) = 2(1
y3 (0 1)) 2 + 2(0)(0) = 2
y4 = 4 y1 y2 + 2 y1 y2 + 2 yy
yy3 y4 (0) = 6( 1)(0) + 2(0)(2)
6((1
= 6 y1 y2 + 2 yy3 , =0
y5 = 6 y2 + 6 y1 y3 + 2 y1 y3 + 2 yyyy4
2
y5 (0) = 0 + 8(1)(2) + 0
= 6 y2 + 8 y1 y3 + 2 yy
2
yy4 , =1
16

Substituting in Maclaurin’s series,


x3 x5
tan x = x + (2) + (16) + ………
3! 5!
x3 2 x5
= x+ + + ………
3 15
This series is known as the tangent series.
Note: This series can also be obtained by dividing the sine and cosine series
sin x
since tan x = .
cos x
(v) Expansion of sinh x
e x − e− x
Proof sinh x =
2
Substituting exponential series ex and e–x,
 x 2 x3   x 2 x3 
 1 + x + + + … −
  1 − x + − +…
nh x = 
2 ! 3!
2!   2 ! 3!
2! 
sinnh
2
x3 x5
= x + + +………
3! 5!
(vi) Expansion of cosh x
e x + e- x
Proof cosh x =
2
Substituting exponential series ex and e–x,
 x 2 x3   x 2 x3 
 1 + x + + + … +
  1 − x + − + …
cosh x = 
2 ! 3!
2!   2 ! 3!
2! 
2
x2 x4
= 1 + + +………
2 ! 4!
2.30 Chapter 2 Taylor’s and Maclaurin’s Series

(vii) Expansion of tanh x


Proof Expansion of tanh x can be obtained by dividing the series of sinh x and
cosh x.

sinh x
nh x =
tannh
cosh x
x3 x5 x 7
x + + + +…
= 3! 5 ! 7 !
3!
x2 x4 x6
1+ + + +…
2! 4! 6!
2!
x3 2 5
= x − + x − ………
3 15
Note: This series can also be obtained by using Maclaurin’s series (refer tan-
gent series).
(viii) Expansion of log (1 + x) (Logarithmic series)
Proof Let y = log (1 + x), y (0) = log 1 = 0
dn (n − 1) !
Now, yn = [ log (1 + x)] = (−1) n −1 ⋅
dxx n
( x + 1) n
yn (0) = ( −1) n −1 ⋅ (n − 1) !

Putting n = 1, 2, 3, 4, …..
y1(0) = 1, y2 (0) = – 1, y3 (0) = 2! and so on
Substituting in Maclaurin’s series,
x 2 x3
log (1 + x) = x − + − ………
2 3
This series is known as the logarithmic series and is valid for –1 < x < 1.
Note: In the above series, replacing x by −x, we get expansion of log (1 − x).
x 2 x3 x 4 x5
log (1 − x) = − x − − − − −…
2 3 4 5
(ix) Expansion of (1 + x)m (Binomial series)
Proof Let y = (1 + x) m , y (0) = (1 + 0) m = 1
Now, yn = m (m − 1) (m − 2)……… (m − n +
+11)(1 + x) m−
m n

yn (0) = m( m − 1)(m − 2)……… ((m


m((m m − n + 1)
Putting n = 1, 2, 3, 4, …..
y1(0) = m, y2(0) = m (m − 1), y3(0) = m (m − 1) (m − 2) and so on
2.3 Maclaurin’s Series 2.31

Substituting in Maclaurin’s series,


m(m − 1) 2 m
m( m((m − 1)(
)((m − 2) 3
+ x) m = 1 + mx +
(11+ x + x + ………
2! 3!
This series is known as the binomial series and is valid for –1 < x < 1.

By Definition

Example 1
Expand 5x up to the first three non-zero terms of the series.
Solution
x
Let f (x) = 5
By Maclaurin’s series,
x2
f ( x ) = f ( 0) + x f ′ ( 0) + ( ) + ………
f ′′ (0 ...(1)
2!

x
f (x) = 5 , f (0) = 50 = 1
x
f ¢(x) = 5 log 5, f ¢(0) = 50 log 5 = log 5
x
f ¢¢(x) = 5 (log 5)2, f ¢¢(0) = 50 (log 5)2 = (log 5)2
Substituting in Eq. (1),
x2
5 x = 1 + x log 5 + (log 5) 2 + ………
2!
x
Aliter: f (x) = 5x = e log 5 = e x log 5
( x log 5) 2
= 1 + x log 5 + + ……… [Using exponential
2! series]

Example 2
 1+ x
Obtain the series log (1 + x) and find the series log 
 1 − x 
and hence,
find the value of loge   .
11
[Winter 2016]
 9
Solution
Let y = log (1 + x)
By Maclaurin’s series,
x2 x3 x4
y = y (0) + xy1 (0) + y 2 ( 0) + y3 (0) + y4 (0) +………
+ …(1)
2! 3! 4!
2.32 Chapter 2 Taylor’s and Maclaurin’s Series

y = log(1 + x), y ( 0) = 0
1
y1 = , y1 (0) = 1
1+ x
1
y2 = − , y2 (0) = −1
( + x) 2
(1
( !)
(2
y3 = , 0)) = 2 !
y3 (0
((1 + x)3
( !)
(3
y4 = − , y4 (0) = −(3!)
((1 + x) 4

Substituting in Eq. (1),


x 2 x3 x4
y =0+ x − + (2 !) − (3!) + ………
2 ! 3! 4!
x 2 x3 x 4
log (1 + x)
x) = x − + − + ………
2 3 4
Replacing x by −x,
x 2 x3 x 4
log (1 − x) = − x − − − − ………
2 3 4
1+ x
Now, logg  = log 1 + x) − log
log ((1 log (1 − x)
 1 − x 

 x3 x5 
= 2  x + + + …
 3 5 
1
Putting x = , and considering first three terms,
10
 11  1 1 1 1 1 
log e   = 2  + ⋅ 3
+ ⋅ 5 
= 0.20067
9 10 3 (10) 5 (10) 

Example 3
x 26 3
0 prove that y = 1 −
If x3 + y 3 + xy − 1 = 0, − x − .
3 81
Solution
By Maclaurin’s series,
x2 x3
y = y (0) + xy1 (0) + y2 ((00) + ( ) + 
y3 (0 ...(1)
2! 3!
x3 + y3 + xy – 1 = 0 ...(2)
2.3 Maclaurin’s Series 2.33

Putting x = 0, y (0) = 1
Differentiating Eq. (2) w.r.t. x,
3 x 2 + 3 y 2 y1 + xxyy1 + y = 0 ...(3)
1
Putting x = 0, y1 (0) = −
3
Differentiating Eq. (3) w.r.t. x,
6x + 6yy12 + 3y 2y2 + 2y1 + xy2 = 0 ...(4)

Putting x = 0,
2
 1  1
6 −  + 3
3yy2 (0) + 2  −  = 0
 3  3
y2 (0) = 0
Differentiating Eq. (4) w.r.t. x,
6 + 6y13 + 12yy1y2 + 3y 2y3 + 6yy1 y2 + 3y2 + xy3 = 0
Putting x = 0,
 1
6 + 6  −  + 0 + 3 y3 (0) = 0
 27 
− 52
y3 (0) = and so on.
27
Substituting in Eq. (1),
x x2 x 3  52 
y = 1−+ (0) +  −  + 
3 2! 3!  27 
x 26 3
= 1− − x − 
3 81

Example 4
If x3 + 2xy2 – y3 + x – 1 = 0, expand y in ascending powers of x.

Solution
By Maclaurin’s series,
x2
y = y (0) + xy1 (0) +
y2 ((00) +  ... (1)
2!
x3 + 2xy2 – y3 + x – 1 = 0 ...(2)
Putting x = 0, y(0) = – 1
Differentiating Eq. (2) w.r.t. x,
3x2 + 2y2 + 4xyy1 – 3y2y1 + 1 = 0 … (3)
2.34 Chapter 2 Taylor’s and Maclaurin’s Series

Putting x = 0,
2 – 3y1(0) + 1 = 0
y1(0) = 1
Differentiating Eq. (3) w.r.t. x,

6x + 4yy1 + 4yy1 + 4xy12 + 4xyy2 – 6yy12 – 3y2y2 = 0


Putting x = 0,
–8 + 6 – 3y2(0) = 0
2 and so on.
y 2 ( 0) = −
3
Substituting in Eq. (1),
x2  2 
y = −1+ x +  −  + 
2!  3 
x2
= −1 + x − + 
3

Example 5
If x = y (1 + y2), prove that y = x – x3 + 3x5 + … .
Solution
By Maclaurin’s series,
x2 x3 x4 x5
y = y (0) + xy1 (0) + 0) +
y2 ((0 ( )+
y3 (0 y4 ((00) + ( ) + 
y5 (0 ... (1)
2! 3! 4! 5!
x = y (1 + y2) .... (2)
Putting x = 0, y (0) = 0
Differentiating Eq. (2) w.r.t. x,
1 = y1 + 3y2y1 … (3)
Putting x = 0,
1 = y1(0)

y1(0) = 1
Differentiating Eq. (3) w.r.t. x,
0 = y2 + 6yy12 + 3y2y2 … (4)
Putting x = 0, y2(0) = 0,
Differentiating Eq. (4) w.r.t. x,

0 = y3 + 12yy1 y2 + 6y13 + 6yy1 y2 + 3y2y3


2.3 Maclaurin’s Series 2.35

0 = y3 (1 + 3y2) + 18yy1y2 + 6y13 … (5)


Putting x = 0,
0 = y3 (0) + 6
y3 (0) = −6
Differentiating Eq. (5) w.r.t. x,
0 = (1 + 3y2) y4 + 6yy1y3 + 18y12 y2 + 18yy22 + 18yy1y3 + 18y12 y2

= (1 + 3y2) y4 + 24yy1y3 + 36y12 y2 + 18yy22 … (6)

Putting x = 0, y4 (0) = 0,
Differentiating Eq. (6) w.r.t. x,

0 = (1 + 3y2) y5 + 6yy1y4 + 24y12 y3 + 24yy2y3 + 24yy1y4 + 72y1y22


+ 36y12y3 + 36yy2y3 + 18y1y22
Putting x = 0,
0 = y5 (0) + 24 (−6) + 36 (−6)
y5 (0) = 360 and so on.
Substituting in Eq (1),

x2 x3 x4 x5
y = 0 + x ⋅1 + ⋅ 0 + ( − 6) + ⋅0+ ⋅ 360 + 
2! 3! 4! 5!
= x − x 3 + 3 x 5 + 

By standard Expansion

Example 1
1 + x2
Obtain the expansion of .
1 + x4
Solution
1 + x2
1 + x 4 ) −1
= (1 + x 2 )(1+
1 + x4
= (1 + x2) (1 − x 4 + x 8 − x12 + x16 − …)
= 1 + x 2 − x 4 − x6 + x 8 + x10 − …
2.36 Chapter 2 Taylor’s and Maclaurin’s Series

Example 2
y 2 y3 y 4
If x = y − + − + , prove that
2 3 4
x 2 x3 x 4
y = x + + + +  and conversely.
2 ! 3! 4!4

Solution
x = log(1 + yy)
1 + y = ex
y = ex − 1
x 2 x3
= x+ + + 
2 ! 3!
Conversely,
y = ex − 1
ex = 1 + y
x = log(1 + y )
y 2 y3 y 4
= y− + − + 
2 3 4

Example 3
Expand 1+ sinn x .
Solution
x x
1 + sin x = sin + cos
2 2
 x 1  x 3   1 x
2
1  x
4

=  −   +  + 1 −   +   − 
 2 3!  2    2 !  2  4 2 
2 3 4
x x x x
= 1+ − − + −
2 8 48 384

Example 4
1 2
Prove that cos 2 x = 1 − x 2 + x 4 − x 6 + .
3 45
Solution
1
coss 2 x = 1 + cos
((1 cos 2 x)
2
2.3 Maclaurin’s Series 2.37

1 (2 x) 2 (2
(2 x) 4 (2 x)6 
= 1 + 1 − + − + 
2 2! 4! 6! 
1 4 2 6
= 1 − x2 + x − x +
3 45

Example 5
x4 2 6
Show that sin 2 x = x 2 − + x + .
3 45
Solution
1
n2 x =
sin (1 − cos
cos 2 x)
2
1 4 x 2 16 x 4 64 x 6 
=  1 − 1 + − + − 
2 2! 4! 6! 
x4 2 6
= x2 − + x −
3 45

Example 6
1 ∞ 32 n + 3 2 n
Prove that cos h x =
3

4 n = 0 ((2n)!
x .

Solution
1
cossshh3 x = cos h 3 x + 3 ccosh x)
(cos
((co
cos
4
1  (3 x) 2 ((33 x) 4   x2 x4 
=  1 + + +   + 3 1 + + +  

4  2! 4!   2 ! 4 ! 
1 32 + 3 2 34 + 3 4 
= ((1 + 3) + x + x + 
4 2! 4! 
1 ∞
32 n + 3 2 n
=
4

n=0 (
(2n)!
x

Example 7
8 6 32 10
Prove that sin sin h x = x 2 −
n x sin x + x − ... .
6! 10 !
Solution
 x3 x5 x7 x9  x3 x5 x7 x9 
sin sin h x =  x − + − + − ...  x + + + + + ...
n x sin
 3! 5! 7 ! 9 !
3!  3! 5! 7 ! 9!
3! 
2.38 Chapter 2 Taylor’s and Maclaurin’s Series

2 1  10  2 2 1 
= x2 + x6  − 2
+x  − + + ...
 5! (3!)  3 (5!) 2 
 9 ! 7 !3!
8 32 10
= x2 − x6 + x10 − ...
6! 10 !

Example 8
22 x 4 22 x8
Prove that coss x cos h x = 1−
ccosh
osh + − .
4! 8!
Solution
 x 2 x 4 x 6 x8   x 2 x 4 x 6 x8 
coss x cos h x = 1 − +
ccosh
osh − + −  1 + 2 ! + 4 ! + 6 ! + 8! + 
 2 ! 4 ! 6 ! 8! 
2 1  2 2 1 
= 1 + x4  − + x8  − + +
 4 ! (2 !) 2   8 ! 6 ! 2
2! ( 4 !) 2 
22 x 4 22 x8
= 1− + −
4! 8!

Example 9
Expand sin x cosh x in ascending powers of x up to x5.
Solution
 x3 x5   x2 x4 
oshh x =  x − + −  1 + + + 
sinn x cos
ccosh
 3! 5!
3!  2! 4!
2! 
 x 2 x 4  x3  x 2 x 4  x5  x 2 x 4 
= x 1 + +  − 1 + + +  + 1 + + +  + 
 2 ! 4!!  3! 
4 ! 2! 4!  5!  2! 4! 
x3 x5 x3 x5 x5
= x+ + − − + +
2 24 6 66..2 120
up to x 5 ]
[Considering the terms only up
x3 x5
= x+ − +
3 30

Example 10
2 3
 x  1 x  1 x 
Prove that logg x = log
log 2 +  − 1 −  − 1 +  − 1 + .
2  22  3 2 
Solution
 x
g x = log
log log  2 ⋅ 
 2
2.3 Maclaurin’s Series 2.39

x
= log 2 + log
2
 x 
= log 2 + log 1 +  − 1
  2 
2 3
x  1 x  1 x 
= log 2 +  − 1 −  − 1 +  − 1 − …
2  22  3 2 

Example 11
Expand log (1 + x + x2 + x3) up to x8.

Solution

log (1 + x + x2 + x3) = log [(1 + x) (1 + x2)]


= log (1 + x) + log (1 + x2)

 x 2 x 3 x 4 x 5 x 6 x 7 x8   ( x 2 ) 2 ( x 2 )3 ((x 2 ) 4 
=  x − + − + − + − + ... +  x 2 − + − + ...
 2 3 4 5 6 7 8   2 3 4 
2 3 5 6 7
x x 3 x x x 3
= x + + − x 4 + + + − x8 + ...
2 3 4 5 6 7 8

Example 12
x 2 x3 x 4
Prove that log (1 + x + x + x + x ) = x +
2 3 4
+ + − .
2 3 4
Solution
 1 − x5 
log (1 + x + x2 + x3 + x4) = log   [Using sum of G.P.]
 1− x 
= log (1 − x 5 ) − llog
og (1 − x)
 x10 x15 x 20   x 2 x3 x 4 
=  − x5 − − − −  −  − x − − − 
 2 3 4   2 3 4 
2 3 4
x x x
=x+ + + −
2 3 4

Example 13
x 2 x3 x 4
Prove that log(1 + sin
sin x) = x − + − +.
2 6 12
2.40 Chapter 2 Taylor’s and Maclaurin’s Series

Solution
n 2 x sin
sin sin 3 x sin 4 x
log(1 + sin
sin x) = sin
sin x − + − +
2 3 4
2 3 4
 x3  1 x3  1 x3  1 x3 
=  x − +  −  x − +  +  x − +  −  x − +  + 
 3!  2 3!  3 3!  4 3! 
x2  1 1  1 1
= x− + x3  − +  + x 4  −  + 
2  6 3  6 4
x 2 x3 x 4
= x− + − +
2 6 12

Example 14
x 2 2 x3
Prove that log(1 + tan
tan x) = x − + + .
2 3
Solution
n 2 x ttan
tan an 3 x
log(1 + tan
tan x) = tan
tan x − + −
2 3
2
 x3  1 x3  1
=  x + +  −  x + +  + ( x +
+ )3 − 
 3  2 3  3
x2  1 1
= x− + x3  +  + 
2  3 3
x2 2 3
= x− + x −
2 3

Example 15
 tan x  x
3
7
Prove that log   = + x4 +  .
 x  3 90
Solution
 tan x  1  x3 2 5 
log   = lo
log
g   x + + x +  
 x  x 3 15 
 x2 2 
= log
log 1 + + x 4 + 
 3 15 
2
 x2 2  1  x2 2 
=  + x 4 +  −  + x 4 +  + 
 3 15  2  3 15 
2.3 Maclaurin’s Series 2.41

x2  2 1
= + x4  −  + 
3  15 18 
x2 7 4
= + x +
3 90

Example 16
 sin h x x
2
x4
Prove that log  = − + ... .
 x  6 180
Solution
 sinh x  1 
3
x x5 
log   = log
log   x + + + ... 
 x  x 3! 5 !
3! 
 x2 x4 
= log
log 1 + + + ...
 3! 5! 
2
 x2 x4  1  x2 x4 
=  +  −  + +  + 
 3! 5 !  2  3 ! 5 ! 
 1 1
= x2 + x4  − +
 120 72 
x2 x4
= − + 
6 180

Example 17
x2 7 4
cot x) = −
Prove that log( x cot − x + .
3 90
Solution
 1 
cot x) = − log 
log( x cot 
 x cot
cot x 
 tan x 
= − log  
 x 
 x2 2 
= − log 1 + + x 4 + ... 
 3 15 
 x 2
2  1  x2 2 
2

= −  + x 4 + ....  −  + x 4 + ...  + ...
 3 15  2  3 15  
 x2  2 1 
= −  + x 4  −  + ...
3  15 18  
2
x 7
= − − x 4 + ...
3 90
2.42 Chapter 2 Taylor’s and Maclaurin’s Series

Example 18
2
Expand log(1+ ) in ascending powers of x.
Solution
log(1 ) + )]
x2 x x4 x x3 x 4
− + − + x + − +
2 3 4 2 3 4
x 2 x3 x 4 x2 x2 x3
− + − + −
2 3 4 2 2 3
x3 x2 x4
+ − + )
3 2 4
[Considering the terms onl up to x 5 ]
x x 4 x5 x3 x 4 x5 x 4 x5 x5
= 2
− + − + − + +
2 3 4 2 4 6 3 6 4
11 10
= 2
− x3 + − x 5 +
12 1

Example 19
1+ 2
x2 x4 x6
Prove that log + − + −
e 2 12 45
Solution
1 + e2 x
log )
e
log (2 co )
= + log cosh x
x2 x4 x6
= +l 1+ + + +
2! ! 6!
x2 x4 x6 1 x2 x4 1 x2
= + + + + ... − + + ... + + ... + ...
2 ! 6 2 2! ! 3 2
x2 x4 x6 x4 x6 1 x6
= + + + + − + 2⋅ + + + +
2 4! 6 2 4 48 3 8
x2 1 1
= + + − − + + ...
2 24 8 720 48 24
x2 x4 x6
= + − + ...
2 12 45
2.3 Maclaurin’s Series 2.43

Example 20
 xe x 
Expand log  x  in ascending powers of x up to the terms in x4.
 e − 1
Solution
 xe x   e x − 1
logg  x  = − log 
 e − 1  xe x 
 1 − e− x 
= − log 
 x 
 1   x 2 x3 x 4 x5   
= − log  1 − 1 − x + − + − +   
 x   2 ! 3! 4 ! 5 !   
1  x 2 x3 x 4 x5 
= − log   x − + − + +  
x 2 6 24 120 
  x x 2 x3 x 4 
= − log 1 −  − + − +  
  2 6 24 120 
2 3
  x x 2 x3 x 4  1x x  1  x x2 
2
x3
= − −  − + − +  −  − + −  −  − + 

 2 6 24 120  2  2 6 24  3  2 6 
1x 
4

−  −  −  [Considering the terms onnly up to x 4 ]

4 2  
 x x 2 x3 x 4  1  x2 x4  x   x2   x  x 
3

= − + − +  +  + +2    −  + 2     + 
 2 6 24 120  2  4 36  
2  6  
2  24  
1  x3  1 x4
2
 x  x 
2
+  + 3    −  +  + ⋅ + 
3  8  2  6   4 16
x x 2 x3 x 4 x 2 x 4 x3 x 4 x3 x 4 x 4
= − + − + + − + + − + +
2 6 24 120 8 72 12 48 24 24 64
x x2 x4
= − + +
+
2 24 2880

Example 21
x x2 x4
Prove that log(1
1++ e x ) = log
log 2 + + − + .
2 8 192
2.44 Chapter 2 Taylor’s and Maclaurin’s Series

Solution
 x 2 x3 x 4 
log (1 + e x ) = llog
og 1 + 1 + x + + + + 
 2 ! 3! 4 !
2! 
  x x 2 x3 x 4 
= log  2 1 + + + + + ... 

  2 4 12 48 
 x x 2 x3 x 4 
= log 2 + log 1 + + + + + ... 
 2 4 12 48 
2 3
 x x 2 x3 x 4  1  x x 2 x3  1  x x2 
= log 2 +  + + + + ... −  + + + ... +  + + ...
 2 4 12 48  2  2 4 12  3 2 4 
4
1x 
−  + ... + ...
42 

 x  1 1 1 1 1  1 1 1 1 1
= log 2 +   + x 2  −  + x 3  − +  + x 4  − − + − + ...
 2  4 8  12 8 24   48 32 24 16 64 
x x2  1  4
= log 2 + + +0+− x + ...
2 8  192 
x x2 x4
= log 2 + + − + ...
2 8 192

Example 22
 1
 x 5 x 2 x3 251 4
Prove that logg llog (1 + x) x  = − + − + x + ... .
  2 24 8 2880
Solution
1
1
log (1 + x) x = llog (1 + x)
x
1 x 2 x3 x 4 x5 
= x − + − + − 
x 2 3 4 5 
x x 2 x3 x 4
=1− + − + −
2 3 4 5
 x x 2 x3 x 4 
=1−  − + − + 
 2 3 4 5 
= 1− y
2.3 Maclaurin’s Series 2.45

 1

Now, 1 + x) x  = log
g log ((1
log log (1 − y )
 
y 2 y3 y 4
= −y − − − − ...
2 3 4
2
 x x 2 x3 x 4  1  x x 2 x3 
=− − + − + ...  − ⋅  − + − ... 
 2 3 4 5  2  2 3 4 
3 4
1  x x2  1  x x2 
−  − − ...  −  − + ...  − ...
32 3  42 3 
x 1 1 1 1 1  1 1 1 1 1 
= − + x 2  −  − x 3  − +  + x 4  − − + −  + ...
2 3 8  4 6 24   5 18 8 12 64 
x 5 x 2 x 3 251 4
=− + − + x + ...
2 24 8 2880

Example 23
1
 1+ ex  2
Expand   up to the term containing x2.
2e x 
Solution
1 1
 1 + ex  2  1 − x 1  2
 x 
= e + 
 2e  2 2
1
1  x2  1 2
=  1 − x + −  + 
 
2 2 !  2
1
 1 x2 2
= 1 − x + − 
 2 4 
1
  x x2  2
= 1 −  − +  

 2 4 
11 
−1
 2  2 
2
1  x x2  x x2 
= 1−  − +  +  − +  − 
22 4  2!
2! 2 4 
2 2
x x 1 x
= 1− + − ⋅ +
4 8 8 4
x 3 2
= 1− + x +
4 32
2.46 Chapter 2 Taylor’s and Maclaurin’s Series

Example 24
Expand ecos x up to x4.
Solution
y = ecos x
 x2 x4 
1− + −
 2!
2! 4! 
=e
 x2 x4 
 − + −
 2!
2! 4! 
=ee
  x2 x4  1  x2 
2

= e 1 +  − + −  +  − +  + 
  2 ! 4 !  2!  2!  
 x2 x4 x4 
= e 1 − + + + 
 2 ! 24 8 
 x2 x4 
= e 1 − + − 
 2! 6
2! 

Example 25
x3 x 4
Prove that e x cos x = 1 + x − − +.
3 6
Solution
 x 2 x3 x 4   x2 x4 
e x cos x = 1 + x + + + +  1 − + − 
 2 ! 3! 4 !  2! 4! 

 x2 x4   x2 x4  x2  x2 x4 
= 1 1 − + +  + x 1 − + +  + 1 − + + 
 2! 4!   2! 4!  2!  2 ! 44! 
x3  x 2 x 4  x4  x2 x4 
+  1 − + −  + 1 − + +  + 
3!  2! 44!!  4!  2! 4! 
x2 x4 x3 x 2 x 4 x3 x 4
= 1− + +x− + − + + +
2 24 2 2 4 6 24
y up to x 4 ]
[Considering the terms only
x3 x 4
= 1+ x − − +
3 6
2.3 Maclaurin’s Series 2.47

Example 26
x 2 x3 11 4 x5
Prove that e
x cos x
= 1+ x + − − x − +.
2 3 24 5
Solution
 x2 x4 
x 1− + −
 2! 
e x cos x
=e 2! 4!

2 3
 x3 x5  1 x3  1 x3 
= 1 +  x − + − ...  +  x − + ...  +  x − + .. 
 2! 4!  2!  2!  3!
3!  2! 
4 5
1 x3  1 x3 
+  x − − ...  +  x − − ... 
44!!  2!  5!
5 !  2 ! 
 1 1  1 1   1 1 1 
2
x
= 1 + x + + x3  − +  + x 4  − +  + x5  − +  + ...
2  2 6   2 24   24 4 120 
x 2 x 3 11 4 x 5
= 1+ x + − − x − + ...
2 3 24 5

Example 27
x4 x6
Prove that e x sin x = 1 + x 2 + + + .
3 120
Solution
 ( x sin x) 2 ( x sin
sin x)3 
e x sin x = 1 + x sin x + + + 
 2! 3! 
2 3
 x3 x5  x2  x3 x5  x3  x3 x5 
= 1 + x  x − + −  +  x − + −  +  x − + −  + 
 3! 5!  2!  3! 5!  3!  3! 5! 
 1 1   1 1 1 
= 1 + x2 + x4  − +  + x6  − + +
 6 2  120 6 6 
x4 x6
= 1 + x2 + + +
3 120

Example 28
ex  5 x3 
Prove that e = e  1 + x + x 2
+ +  .
6
2.48 Chapter 2 Taylor’s and Maclaurin’s Series

Solution
 x 2 x3 
1 + x + + + 
ex  2 ! 3! 
e =e
x2 x3
x+ + +
= ee 2! 3!
  x 2 x3  1  x2 
2
1 
= e 1 +  x + + +  +  x + +  + ((xx + … )3 + 
  2! 3!  2!  2!  3! 
  1 1  1 1 1 
= e 1 + x + x 2  +  + x 3  + +  + 
  2 2   6 2 6  
 5 3 
= e 1 + x + x + x + 
2
 6 

Example 29
Expand (1 + x)x in a series up to the term in x4.
Solution
x
1+ x )
((1 + x) x = elog(
log(1

1+ x )
= e x log(
log(1
1+

 x 2 x3 
x  x − + −
 2 3 
=e
 2 x3 x 4 
 x − + −
 2 3 
=e
2
 x3 x 4  1 x3 x 4 
= 1 +  x2 − + −  +  x 2 − + −  + 
 2 3  2!  2 3 
 x3 x 4  1
= 1 +  x2 − +
 2 3  2
( )
−  + x 4 +  + 

y up to x 4 ]
[Considering the terms only
x3 5
= 1 + x 2 − + x 4 +
2 6

Example 30
1
e 11e 2
Prove that ((1 + x) x = e − x+ x +  .
2 24
Solution
1 1
log (1+ x )
((1 + x) x = e x
2.3 Maclaurin’s Series 2.49

1  x 2 x3 
 x − + −... 
x  2 3
=e 

 x x2 
1− + −... 
 2 3
=e 

 x x 2 x3 x 4 
 − + − + −... 
 2 3 4 5
= ee 

  x x 2 x3 x 4  1  x x 2 x3 
2

= e 1 +  − + − + −  +  − + − +   + 
  2 3 4 5  2!
2!  2 3 4  
 x 1 1 
= e 1 − + x 2  +  + 
 2 3 8 
e 11e 2
=e− x+ x +
2 24

Example 31
Expand (1 + x)(1 + x) up to the term containing x3.
Solution
((1+ x )
((1 + x)((111++ x ) = elog
log (1+ x )

= e(1+ x )l))log(
log(1+ x )
log(

 x 2 x3 
((1+ x )  x − + −
 2 3 
=e
 x 2 x3 2 x3 
 x − + + x − +
 2 3 2 
=e y up to x 3 ]
[Considering the terms only
 x x 2  3
 x + − +
 2 6 
=e
2
 x 2 x3  1 x2  1
= 1+  x + − +  +  x + −  + ( x + )3 + 
 2 6  2!  2  3!
[Considerring the terms only up to x 3 ]
onsiderring

 x 2 x3  1 x2  1
= 1+  x +
 2
− +  +  x 2 + 2 ⋅ x ⋅ +  + x 3 +  + 
6  2 2  6
( )
x3
= 1 + x + x2 + +
2
2.50 Chapter 2 Taylor’s and Maclaurin’s Series

Example 32
x2 5 4
Prove that sin(e x − 1) = x + − x +.
2 24
Solution
 x 2 x3 x 4 
sin (e x − 1) = sin
sin  x + + + + 
 2
2!! 3 ! 4 ! 
3
 x 2 x3 x 4  1  x 2 x3 
=x + + + +  −  x + + +  + 
 2 ! 3! 4 !
2!  3!  2 ! 3! 
1 1  1 1
2
x
=x+ + x3  −  + x 4  −  +
2 6 6  24 4 
x2 5 4
=x+ − x +
2 24

Example 33
x2 7 4
Prove that x cosec x = 1 + + x +
6 360
Solution
x
x cosec x =
sin x
x
=
 x3
x5 
 x − 3! + 5! − 
1
=
 x 2
x4 
1 − 6 + 120 − 
−1
  x2 x4 
= 1 −  − −  
  6 120 
  x2 x4   x3 
2

= 1 +  − −  +  −  + 
  6 120   6  
[ Using (1 − x) −1
−1
= 1 + x + x 2 + ]

 x2 x4   x4 
= 1+  − −  +  −  + 
 6 120   36 
[Considering the terms only up to x 4 ]
2.3 Maclaurin’s Series 2.51

x2  1 1
= 1+ +− +  x4 + 
6  120 36 
x2 7 4
= 1+ + x
6 360

Example 34
x
Expand up to x4 and hence, prove that
e −1
x

x ex + 1 x2 x4
= 1 + − +.
2 ex − 1 12 720

Solution
x x
=
e − 1 
x
x 2
x 3
x 4 x5  
1 + x + + + + + ... − 1
 2 ! 3! 4 ! 5 !  
x
=
 x 2 x3 x 4 x5 
x+ + + + + 
 2 ! 3! 4 ! 5 ! 
−1
  x x 2 x3 x 4 
= 1 +  + + + +  

  2 ! 3! 4 ! 5 ! 
2
 x x 2 x3 x4   x x 2 x3 
=1−  + + + +  +  + + + 
 2 6 24 120   2 6 24 
3 4
 x x2  x 
− + +  +  + 
2 6  2 
x  1 1  1 1 1
=1− + x2  − +  + x3  − + − 
2  6 4  24 6 8 
 1 1 1 1 1
+ x4  − + + − +  +
 120 36 24 8 16 

x x2 x4 ...(1)
=1− + + x 3 ( 0) − +
2 12 720
x ex + 1 x  2 
= 1 + x 
2 ex − 1 2  e − 1
2.52 Chapter 2 Taylor’s and Maclaurin’s Series

x x
= +
2 ex − 1
x x x2 x4 [Using Eq. (1)]
= +1− + − +
2 2 12 720
x2 x4
=1+ − +
12 720

Example 35
Prove that
Ê x sin q ˆ x2 x3
tan -1 Á = + + sinn 3q +
+ .
Ë 1 - x cos q ˜¯
x si
sinn q sin
s in 2q si
2 3
Solution
 x sin θ 
y = tan −1 
Let  1 − x cos θ 
x sin θ
tan y =
1 − x cos θ
eiy − e − iy x sin θ
=
i (eiy + e − iy ) 1 − x cos θ
eiy − e − iy ix sin θ
− iy
=
e +e
iiyy
1 − x cos θ
Applying componendo–dividendo,
eiy 1 − x(cos θ − i sinsin θ )
=
e − iy 1 − x(cos θ + i sin
sin θ )
1 − xe − iθ
e 2iy =
1 − xeiθ
iy = log(1 − xe
2iy xe − iθ ) − log(1 − xxeeiθ )

 x 2 e −2iθ x 3 e −3iθ   x 2 e 2iθ x 3 e3iθ 


=  − xe − iθ − − − ... −  − xeiθ − − − ......
 2 3   2 3 

x 2 2iθ x3
= x(eiθ − e − iθ ) + (e − e −2iθ ) + (e3iθ − e −3iθ ) + ...
2 3

x2 x3
= x ⋅ 2i sinn θ + in 2θ + ⋅ 2i sin 3θ + ...
⋅ 2i ssin
2 3

x2 x3
y = x sin θ + sin 2θ + sin
sin 3θ + ...
2 3
2.3 Maclaurin’s Series 2.53

Example 36
((aa 2 − b 2 ) 2 a (a 2 − 3b 2 ) 3
Prove that e cos bbxx = 1 + ax
ax + x + x +
ax

2! ∞ n 3!
x
and hence, deduce e x cos α cos( x sinn α ) = ∑ cos nα .
n=0 n!

Solution
e ax cos bbxx = e ax ◊ Real Part of (eibx)
= RP of e(a+ib)x

 ((aa + ib) 2 2 ((aa + ib)3 3 


= RP of 1 + ((aa + ib) x + x + x + 
 2
2!! 3 ! 
 a 2 − b 2 + 2aib) 2 ((a
((a a 3 − ib3 + 3ia 2 b − 3ab 2 ) 3 
= RP of 1 + (a + ib) x + x + x + ...
 2 ! 3 ! 
(a − b ) 2 a (a − 3b ) 3
2 2 2 2
= 1 + ax + x + x + ...
2! 3!

Putting a = cos a and b = sin a,

(cos 2 α − sin
sin 2 α ) 2 coss3 α − 3 ccos
os α ⋅ sin 2 α 3
e x cos α cos ( x sin
n α ) = 1 + x cos α + x + x + ...
2! 3!

cos 2α 2 cos3 α − 3 cos α (1 − cos 2 α ) 3


cos α +
= 1 + x cos x + x + ...
2! 3!

x2 x3
= 1 + x cos α + cos 2α + cos 3α + ....
2! 3!

xn
=∑ cos nα
n=0 n!

Example 37
1 1 7
Prove that e x = 1 + tan x + tan 2 x − tan 3 x − tan 4 x +  .
2! 3! 4!

Solution
Let e x = a0 + a1 tan x + a2 tan 2 x + a3 tan 3 x + a4 tan 4 x +  ... (1)
2.54 Chapter 2 Taylor’s and Maclaurin’s Series

2 3 4
 x3   x3   x3   x3 
= a0 + a1  x + + ...  + a2  x + + ...  + a3  x + + ...  + a4  x + + ...  + ........
 3   3   3   3 

 x3   2x4 
= a0 + a1  x + + ...  + a2  x 2 + + ...  + a3 ( x 3 + ...) + a4 ( x 4 + ...) + ..........
 3   3 

a  2 
= a0 + a1 x + a2 x 2 +  1 + a3  x 3 +  a2 + a4  x 4 + .......... ...(2)
3  3 

x 2 x3 x 4
But ex = 1 + x + + + + ...... ...(3)
2 ! 3! 4 !
From Eqs (2) and (3),

x 2 x3 x 4 a  2 
1+ x + + + + ..... = a0 + a1 x + a2 x 2 +  1 + a3  x 3 +  a2 + a4  x 4 + .......
2 ! 3! 44!! 3  3 

Comparing coefficients of x, x2, x3 and x4 on both the sides,

1 1 a1 1 1
a0 = 1, a1 = 1, a2 = = , + a3 = =
2! 2 3 3! 6

1 a1 1 1 1 1
a3 = − = − =− =−
6 3 6 3 6 3!
2 1 1 1 2 1 7 7
a2 + a4 = = , a4 = − ⋅ =− =−
3 4 ! 24 24 3 2 24 4!
Substituting in Eq. (1),
1 1 7
e x = 1 + tan x + tan 2 x − tan 3 x − tan 4 x + ...
2! 3! 4!

Example 38
Find the values of a and b such that the expansion of
1 + ax)
xx(((1
log(1 + x) − in ascending powers of x begins with the term x4
1 + bx
x4
and prove that this term is − .
36
Solution
x(1 + a
axx)
Let f ( x) = log(
log(1 + x) −
1 + bx
2.3 Maclaurin’s Series 2.55

x 2 x3 x 4
= − + − + − ax + x 1

2 3 4
x 2 x3 x 4
= − + − + − + − + )
2 3 4
x 2 x3 x 4
= − + − + (x x2 − b x4 + − 3
+ − ab x 5 + )
2 3 4
1
− + −a x − 2
ab x − − ab 2 x 4 +
2 3 4

If the expansion begins with the term x4, the coefficients of x2 and x3 must be zero.
1 1 1
+ b − = 0, b + and − + ab 0 ... (1)
2 2 3
Substituting b in Eq. (1),
2
1 1 1
− + =0
3 2 2
1 1
− 2− + a2 =0
3 4 2
1 1 1
a= =
2 12
1 1 4 2
b + = =
6 2 6 3

1 2 1 1
Coefficient of x − ab 2 = − + =−
4 4 3 6 3 36
x4
Hence, the expansion begins with the term
36

EXERCisE 2.2
1. Expand ex sec x in powers of x using Maclaurin’s series.
[Ans.: 1 + x + x2 + ...]
x2
2. Using Maclaurin’s series, prove that e x+ +
2
x2
3. Using Maclaurin’s series, prove that a x log a + a +
2!
4
x 2
4. Prove that si
2 2
+ x +
3 45
2.56 Chapter 2 Taylor’s and Maclaurin’s Series

x 2 5x 4
5. Prove that sec x = 1 + + +
2 24
 1   x2 x4  
−1

hint : sec x = −1
= (cos x) = 1 −  − + …  
 cos x   2 ! 4 !  
 
3
x
6. Prove that e x sin2 x = 2 x + 2 x 2 − +
3
x3
7. Prove that e x cos x = 1 + x − +
3
22 x 4 2 4 x 8
8. Prove that coss x cos h x = 1−
ccosh
osh + −
4! 8!
x2 5 4
9. Prove that sin (e − 1) = x + − x +
x

2 24

x2 x4
10. Prove that cosn x = 1 − n ⋅ + n(3n − 2) ⋅ −
2! 4!
3x 2 15x 4
Hence, deduce that cos3 x = 1 − + −
2 48
(3n − 3) − [1 − (−1)n ]x n
11. Prove that sinh3 x = ∑ 8 ⋅ n!
.

x4 x6
12. Prove that e
x sin x
= 1+ x 2 + + + 
3 120
1 1
g x = (x − 1) − ((x
13. Prove that log x − 1)2 + (x − 1)3 + 
2 3
x 2 2x 3
14. Prove that log(1 − x + x ) = − x + + − ...
2

2 3
1 2 1 4 1 6
15. Prove that log cosh x = x − x + x −…
2 12 45
x 2 2x 3
16. Prove that log(1 + tan
tan x) = x − + + 
2 3
 sin x   x2 x4 x6 
17. Prove that log   = − + + + 
 x   6 180 2835 

 tan x  x 3 7 4
18. Prove that log   = + x + 
x 3 90
x2 x3
19. Prove that e x log(1 + x)
x) = x + + +
2 3
2.3 Maclaurin’s Series 2.57

π 
20. Expand log tan  + x  upto x 5 .
4 
 p   1 + tan x  
hint : log tan  + x  = log   = log(1 + tan
log(1 n x ) − log ((1 − tan x)
og
 4 1 − tan x  
 4 3 4 5 
 Ans.: 2 x + 3 x + 3 x + …
 
y2 y3 y4 x2 x3 x4
21. Prove that x = y + + + +  if y = x − + − +
2 ! 3! 4 ! 2 3 4

By Differentiation and integration

Example 1
x2 x4 x6
Prove that log(secc x) = + + +. [Summer 2017]
2 12 45

Solution
Let y = log (sec x)
dyy
d 1
= ⋅ secc x tan x
dxx sec x
d
= tan x
x3 2
= x + + x 5 +  ... (1)
3 15
Integrating Eq. (1),
x2 x4 2 x6
y = c+ + + ⋅ + ........
2 12 15 6
x2 x4 x6
log (secc x) = c + + + + .........
2 12 45
Putting x = 0,
log (secc 0) = c + 0
c = log 1, c = 0
logg1
x2 x4 x6
Hence, log(secc x) = + + + 
2 12 45

Example 2
1 x3 1◊ 3 x5 1◊ 3 ◊ 5
Prove that sinn -1 x = x + + + + .
2 3 2 ◊ 4 5 2 ◊ 4 ◊6

2.58 Chapter 2 Taylor’s and Maclaurin’s Series

Solution
Let y = sin–1 x
dyy
d 1
=
dx
dx 1 − x2
1

= (1 − x 2 ) 2

 1  3  1  3  5
− −  −   −   − 
1 2  2   2  2 2 2
= 1+ x + (− x 2 )2 + ( − x 2 )3 + …
2 2! 3!
x 2 1⋅ 3 4 1⋅ 3 ⋅ 5 6
= 1+ + x + x +…
2 2⋅ 4 2 ⋅ 4⋅
4 6
... (1)
Integrating Eq. (1),
1 x 3 1⋅ 3 x 5 1⋅ 3 ⋅ 5 x7
y = c+ x+ + + + ........
2 3 2⋅ 4 5 2⋅ 4⋅6 7
1 1 3 x 5 1⋅ 3 ⋅ 5
x 3 1⋅ x7
sin −1 x = c + x + + + + ........
2 3 2⋅ 2 4 5 2⋅ 4⋅6 7
Putting x = 0,
sin −1 0 = c
c=0
1 x 3 1⋅ 3 x 5 1⋅ 3 ⋅ 5 x 7
Hence, sin −1 x = x + + + + 
2 3 2⋅ 4 5 2⋅ 4⋅6 7

Example 3
−1 π  1 x3 1.3 x5 
Prove that cos x = −x + + +  .
2  2 3 2.4 5 
Solution
Let y = cos–1 x
ddyy 1
=−
dx
dx 1 − x2

Proceeding as in Example 2,

 1 x 3 1.3 x 5 
cos −1 x = c −  x + + + …
 2 3 2 .4 5 
2.3 Maclaurin’s Series 2.59

Putting x = 0,
cos −1 0 = c
π
c=
2

π  1 x 3 1.3 x 5 
Hence, cos −1 x = − x + + + 
2  2 3 2.4 5 

Example 4
x3 x5 x 7
Prove that tan −1 x = x − + − + .
3 5 7
Solution
Let y = tan–1 x
dy
dy 1
= = (1 + x 2 ) −1 = 1 − x 2 + x 4 − x 6 + … ... (1)
dx 1 + x 2
dx
Integrating Eq. (1),
x3 x5 x 7
y = c+ x− + − +…
3 5 7
3 5
x x x7
tan −1 x = c + x − + − + ........
3 5 7

Putting x = 0,
tan −1 0 = c
c=0
x3 x5 x 7
Hence, tan −1 x = x − + − +…
3 5 7

Example 5
x3 3x5
Prove that sinh −1 x = x − + + .
6 40
Solution
Let y = sin h −1 x = log (
log x + x 2 + 1 )
ddyy 1  2x 
= 1+
dx x + x + 1  2 x 2 + 1 
dx 2 

1
=
x2 + 1
1

= (1 + x 2 ) 2
2.60 Chapter 2 Taylor’s and Maclaurin’s Series

 1 3
− −
1 2  2   2  2 2
= 1− x + (x ) − …
2 2!
1 3 ... (1)
= 1 − x2 + x4 − …
2 8

Integrating Eq. (1),


x3 3 x5
y = c+ x− + ⋅ −…
6 8 5
x3 3x5
sinh −1 x = c + x − + −…
6 40
Putting x = 0,
sinh −1 0 = c, c = 0
x3 3x5
sinh −1 x = x − + −…
6 40

Example 6
x3 x5
Prove that tanh −1 x = x + + + .
3 5
Solution
1 ( + x) 1
(1
Let y = tanh −1 x = log = [log(1 + xx)) − log(1 − x)]
2 ( − x) 2
(1
dy 1
=
dx 1 − x 2
= (1 − x 2 ) −1
= 1 + x2 + x4 + x6 +  ...(1)
Integrating Eq. (1),
x3 x5
y = c+ x+ + +
3 5
x3 x5
tanh −1 x = c + x + + + 
3 5
Putting x = 0,
tanh −1 0 = c, c = 0
x3 x5
Hence, tanh −1 x = x + + +
3 5
2.3 Maclaurin’s Series 2.61

Example 7
y2 y4 y6
If x = 1 – + − + ........, find y in a series of x.
2! 4! 6!
Solution
y 2 y 4 y6
x =1- + - + ºº
2! 4! 6!
= cos y
y = cos –1 x
Proceeding as in Example 3,

π  x3 3x5 
y= − x + + + …
2  6 40 

Example 8
−1 1− x π 1  x3 3x5 
Show that tan = − x+ + + 
1+ x 4 2  6 40 
Solution
1− x
Let y = tan −1
1+ x
Putting x = coss 2θ ,
cos 2θ
1 − cos
y = tan −1
cos 2θ
1 + cos
2 sin 2 θ
= tan −1
2 cos 2 θ
= tann −1 tan θ

1
= cos −1 x
2
1 π  x3 3x5 
=  −x+ + +  
22  6 40 

By substitution

Example 1
Expand sinn −1 (3
(3 x − 4 x3 ) in ascending powers of x.
2.62 Chapter 2 Taylor’s and Maclaurin’s Series

Solution
Let y = sin −1 (3
(3 x − 4 x 3 )
Putting x = sin q,
y = sin −1 (3 sin
sin θ − 4 ssin
in 3 θ )
= sinn −1 (sinn 3θ )
= 3θ

= 3 sin −1 x
 1 x 3 1⋅ 3 x 5 1⋅ 3 ⋅ 5 x 7 
= 3 x + ⋅ + + + 
 2 3 2⋅ 4 5 2⋅ 4⋅6 7 

Example 2
 x3 3 5 
–1
Prove that sinh (3x + 4x ) = 3  x3− + x +  .
 6 40 

Solution
y = sinh −1 ((33 x + 4 x 3 )
Let
Putting x = sinh q,
y = sinh −1 (3 sin
inhhq
h q + 4 ssin
inh 3 q )
inh
= sinh −1 (sinh
h 3q )
= 3q
= 3 sinh −1 x
 x3 3x5 
= 3 x − + − …
 6 40 

Example 3
−1  2 x   x3 x5 x 7 
Prove that sinn  2
= 2  x − + − +  .
1 + x   3 5 7 

Solution
 2x 
y = sin −1 
 1 + x 2 
Let
2.3 Maclaurin’s Series 2.63

Putting n θ,
x = tan

 2 tan q 
y = sin −1  
 1 + tan q 
2

= sinn −1 (sinn 2q )
= 2q
= 2 tan −1 x
 x3 x5 x7 
= 2  x − + − +…
 3 5 7 

Example 4
−1  1 
Expand sec  .
1 − 2 x2 
Solution
Ê 1 ˆ
y = sec -1 Á
Let Ë 1 - 2 x 2 ˜¯
Putting n θ,
x = sin
Ê 1 ˆ
y = sec -1 Á
Ë 1 - 2 sin q ˜¯
2

Ê 1 ˆ
= sec -1 Á
Ë cos 2q ˜¯
= secc -1 (secc 2q )
= 2q
= 2 ssiin -1 x
Ê x3 3x5 ˆ
= 2Á x + + + …˜
Ë 6 40 ¯

Example 5
−1
−1  x − x   x3 x5 x 7 
Prove that coss  −1 
= π − 2  nπ + x − + − + … .
 x+ x   3 5 7 

Solution
 x − x −1  −1  x − 1
2
y = cos −−11  = 
 x 2 + 1
Let cos
 x + x −1 
2.64 Chapter 2 Taylor’s and Maclaurin’s Series

Putting x = tan q,
 tan 2 θ − 1
y = cos −1 
 tan 2 θ + 1
= cos −1 ( − coss 2θ )
= cos −1[ − cos (22nnπ + 2θ )]
Considering general value of cos 2θ ]
[[Considering
−1
= cos
cos [co s{π − (2nπ + 2θ )}]
[[cos{
cos{
= π − 2 (nπ + θ )
= π − 2 ((nnπ + tan −1 x)
 x3 x5 x 7 
= π − 2  nπ + x − + − + …
 3 5 7 

Example 6
−1  x3 x5 
Prove that cos [tanh (log x)] = p − 2  x − + − … .
 3 5 

Solution
Let y = cos −1[tanh (log x)]
 elogg x − e − llog
og x

= cos −1  logg x − log x 
e +e 
−1
 x−x 
= cos −1  
 x + x −1 
 x2 − 1 
= cos −1  2 
 x +1
Putting x = tan q,
 tan 2 q − 1
y = cos −1 
 tan 2 q + 1
= cos −1 ( − coss 2q )
= cos −1 [cos (p − 2q )]
= p − 2q
= p − 2 tan −1 x
 x3 x5 
= p − 2  x − + − …
 3 5 
2.3 Maclaurin’s Series 2.65

Example 7
 1 + x 2 − 1 1 
−1 x3 x5 
Prove that tan   =  x − + − ... . [Winter 2013]
 x  2 3 5
Solution
 1++ x 2 − 1
Let y = tan −1  1 
 x
Putting x = tan q,
Ê 1 + tan 2 q - 1ˆ
y = tan -1 Á 1+ ˜
Ë tan q ¯
Ê sec q - 1ˆ
= tan -1 Á
Ë tan q ˜¯
Ê 1 - cos q ˆ
= tan -1 Á
Ë sin q ˜¯
Ê q ˆ
2 sin 2
Á 2 ˜
= tan -1 Á
q q˜
Á 2 sin
n cos ˜
Ë 2 2¯
Ê qˆ
= tann -1 Á tan ˜
Ë 2¯
q
=
2
1
= tan -1 x
2
1Ê x3 x5 ˆ
= Áx- + - º˜
2Ë 3 5 ¯

Example 8
 p − qx
qx  −1 p  x3 x5 x 7 
Prove that tan −1   = tan −  x − + − + … .
 q + px
px  q 3 5 7 
Solution
 p 
−x
 q 
Let y = tan −1  
1+ p x
 q 
p
Putting x = tan
nqq, = tan A
q
2.66 Chapter 2 Taylor’s and Maclaurin’s Series

 tann A − tan θ 
y = tan −1 
 1 + tan A ⋅ tan θ 
= tan −1[tan ( A − θ )]
= A −θ
p
= tan −1 − tan −1 x
q
p  x3 x5 x7 
= tan −1 −  x − + − + …
q  3 5 7 

EXERCisE 2.3

tan−1x 4 23 5
1. Prove that = x − x3 + x − …. .
1+ x 2 3 15
 2x   x3 x5 
2. Prove that tan−1  = 2 x − + − … .
 1 − x 2   3 5 

 3x − x 3   x3 x5 
3. Prove that tan−1  2  = 3  x − + − … .
 1 − 3x   3 5 
 3x − x 3  p  x3 x5 
4. Prove that cott−1  2 
= − 3  x − + − … .
 1 − 3x  2  3 5 

 x  1 3 5
5. Prove that tan−1  = x + x3 + x + …. .
 1 − x 2  6 40

 1− x  p  x3 x5 x7 
6. Prove that tan−1   = −  x − + − + … .
 1+ x  4  3 5 7 

 1− x  p 1  x 3 3 x5 
7. Prove that tan−1   = −  x + + + … .
 1+ x  4 2 6 40 

p −  x − x + x − … .
3 5

8. Prove that cot −1 x =  


2  3 5

p  x3 3 5 
9. Prove that cos–1(4x3 – 3x) = 3  −  x + + x + …  .
2  6 40 

( 1 + x ) = x − x3 + x5 − …...
3 5
10. Prove that secc −1 2
Points to Remember 2.67

 2 − 3x  −1 2  x3 x5 
11. Prove that tan−1   = ta n −  x − + − … .
 3 + 2x  3  3 5 

points to REMEMBER

• Taylor’s Series
h2 h3
(i) f (x + h) = f (x) + h f ¢(x) + f ≤(x) + f ≤¢(x) + …
2! 3!
hn n
+ f (x) + …
n!
(x − a)2 (x − a)3
(x
(ii) f (x) = f (a) + (x – a) f ¢(a) + f ≤(a) + f ≤¢(a)
2! 3!
x − a)n n
((x
+…+ f (a) + …
n!
• Maclaurin’s Series
x2 x3 xn n
f (x) = f (0) + xf ¢(0) +f ≤(0) + f ≤¢ (0) + … + f (0) + …
2! 3! n!
• List of Expansion of Some Standard Functions
x x2 x3
(i) ex = 1 + + + +…
1! 2! 3!
x3 x5 x7
(ii) sin x = x – + – +…
3! 5! 7!
x2 x4 x6
(iii) cos x = 1 – + – +…
2! 4! 6!
x3 2x 5
(iv) tan x = x + + +…
3 15
x3 x5 x7
(v) sinh x = x + + + +…
3! 5! 7!
x2 x4 x6
(vi) cosh x = 1 + + + +…
2! 4! 6!
x3 2x 5
(vii) tanh x = x – + –…
3 15
x2 x3 x4
(viii) log (1 + x) = x – + – …
2 3 4
m (m − 1) 2
(ix) (1 + x)m = 1 + mx + x +…
2!
2.68 Chapter 2 Taylor’s and Maclaurin’s Series

MultiplE ChoiCE QuEstions


Choose the correct alternative in the following questions:
1. The expansion of log (1 + x) in Maclaurin series is
x2 x3 x 4 x2 x3 x 4
(a) x - + - + (b) x + + + +
2 3 4 2 3 4
x2 x3 x 4 x2 x3 x 4
(c) x+ + + + (d) x - + - +
2 3! 4! 2 3! 4!
2. The expansion of tan x in Maclaurin series is
x3 x5 x3 2 5
(a) x + + + (b) x - + x -
2! 3! 3 15
x3 x5 x3 2 5
(c) x- + - (d) x + + x +
3! 5! 3 15
3. The Maclaurin series of sin x is
x3 x5 5 x2 x4
(a) x- + x - (b) 1 + + +
3! 5! 2! 4!
x2 x4 x3 x5
(c) 1 - + + (d) x - + +
2! 4! 3! 5!
4. The nth term in Maclaurin series expansion is
f n ( x) f n (0) f ( x) f (0)
(a) (b) (c) (d)
n! n! n! n!
1
5. Taylor’s series expansion of y = about x = 1 is
x
(a) 1 - ( x - 1) + ( x - 1)2 - ( x - 1)3 + 
(b) 1 + ( x - 1) + ( x - 1)2 + ( x - 1)3 + 
2 3
(c) 1 - ( x - 1) + ( x - 1) - ( x - 1) + 
2! 3!
( x - 1)2 ( x - 1)3
(d) 1 + ( x - 1) + + +
2! 3!
p
6. Taylor’s series expansion of y = sin x about x = is
2
2 4
Ê pˆ Ê pˆ
(a) 1 - Á x - ˜ + Á x - ˜ - 
Ë 2¯ Ë 2¯
2 4
1Ê pˆ 1Ê pˆ
(b) 1 - Á x - ˜ + Á x - ˜ -
2! Ë 2 ¯ 4! Ë 2¯
Multiple Choice Questions 2.69

3 5
Ê pˆ 1 Ê pˆ 1Ê pˆ
(c) Á x - ˜ - Á x - ˜ + Á x - ˜ - 
Ë 2 ¯ 3! Ë 2¯ 5! Ë 2¯
3 5
Ê pˆ Ê pˆ Ê pˆ
(d) Á x - ˜ - Á x - ˜ + Á x - ˜ - 
Ë 2¯ Ë 2¯ Ë 2¯
7. Maclaurin’s series of f(x) is
x x2 xn n
(a) f ( x ) + f ¢( x ) + f ¢¢( x ) +  + f ( x) + 
1! 2! n!
x x2 xn n
(b) f (0) + f ¢(0) + f ¢¢(0) +  + f (0) + 
1! 2! n!
x x2 xn n
(c) 1 + f ¢(1) + f ¢¢(1) +  + f (1) + 
1! 2! n!
x x2 xn n
(d) 1 + f ¢( x ) + f ¢¢( x ) +  + f ( x) + 
1! 2! n!
8. The Taylor’s series expansion of log x in (x – 1) is
1 1
(a) ( x - 1) + ( x - 1)2 + ( x - 1)3 + 
2 3
1 1
(b) ( x - 1) - ( x - 1)2 + ( x - 1)3 -
2 3
1 1
(c) 1 + ( x - 1) + ( x - 1)2 + ( x - 1)3 + 
2 3
1 1
(d) 1 - ( x - 1) +( x - 1)2 - ( x - 1)3 + 
2 3
9. Which of the following is the coefficient of x¢¢ in the expansion of ex?
1 1 1
(a) (b) 11! (c) - (d)
11 11 11!
10. The coefficient of x5 in the expansion of cos x is
1 1 1
(a) 0 (b) (c) - (d)
5! 5! 5
11. The coefficient of x100 in the expansion of log (1 – x)2 is
1 1 1 1
(a) (b) - (c) - (d)
100 100 50 50
12. The constant term in the expansion of 7 + ( x + 2) + 3( x + 2)3 + ( x + 2)4 is
(a) 40 (b) 48 (c) 49 (d) 50
2.70 Chapter 2 Taylor’s and Maclaurin’s Series

13. The coefficient of x2 in the expansion of ex cos x is


1 1
(a) (b) –1 (c) 0 (d) -
2 2
14. The coefficients of x4 and x5, respectively, in the expansion of
( x - 2)4 - 3( x - 2)3 + 4( x - 2)2 + 5( x - 1) - 100 are
(a) (1, 1) (b) (0, 0) (c) (0, 1) (d) (1, 0)
4 3 2
15. The expansion of x - 3 x + 2 x - x + 1 about 3 is
(a) 16 + 38( x + 3) + 29( x + 3)2 + 9( x + 3)3 + ( x + 3)4
(b) 16 + 38 x + 29 x 2 + 9 x 3 + x 4
(c) 16 + 38( x - 3) + 29( x - 3)2 + 9( x - 3)3 + ( x - 3)4
(d) 16 - 38( x + 3) + 29( x + 3)2 - 9( x + 3)3 + ( x + 3)4
16. The Maclaurin series of sin x is [Summer 2015]

x 2 n +1

( -1)n x 2 n +1
(a) Â (2n + 1)! (b) Â
n=0 n = 0 (2 n + 1)!

• •
x2n x2n
(c) Â (2n)! (d) Â (-1)n (2n)!
n=0 n=0
–x
17. The Maclaurin’s series of e is [Winter 2015]
• • •

xn ( -1)n x n xn ( -1)n x n
(a) Â (b) Â n!
(c) Â n! (d) Â n!
n = 0 n! n=0 n =1 n =1

3 5
x x
18. The series x + + +  represent expansion of [Summer 2016]
3! 5!
(a) sin x (b) cos x (c) sinh x (d) cosh x
2 3 4
x x x
19. The series x - + - +  represent expansion of [Summer 2017]
2 3 4
(a) ex (b) log (1 + x) (c) sin x (d) cos x
5 x
20. The coefficient of x in the expansion of e is [Winter 2016]
1 1 1
(a) (b) (c) (d) 5
5 4! 5!

Answers

1.(a) 2.(d) 3.(a) 4.(b) 5.(a) 6.(b) 7.(b) 8.(b) 9.(d) 10.(a)
11.(c) 12.(c) 13.(c) 14.(d) 15.(c) 16.(b) 17.(b) 18. (c) 19.(b) 20.(c)
Unit 2
Curve Sketching
CHAPTER
Curve
3
Sketching
3.1 inTroDucTion

Curve sketching is a procedure to obtain an approximate shape of the curve without


plotting a large number of points on it. The graph of a given function is helpful in
giving a visual presentation of the behaviour of the function involving a study of
symmetries, asymptotes, the interval of rising up or falling down and of the concavity
upwards and downwards, etc. Before drawing curves, we will study monotonic
functions, concavity and convexity of a curve and points of inflection.

3.2 MonoTonic funcTions


A function f (x) is called monotonic increasing if y
(i) f (x) is continuous in the closed interval [a, b],
(ii) f (x) is differentiable in the open interval (a, b) and
(iii) f ¢(x) ≥ 0 throughout the interval (a, b).
O a b x
If f ¢(x) > 0, the function is called strictly increasing.
fig. 3.1
A function f (x) is called monotonic decreasing if
y
(i) f (x) is continuous in the closed interval [a, b],
(ii) f (x) is differentiable in the open interval (a, b) and
(iii) f ¢(x) £ 0 throughout the interval (a, b).
If f ¢(x) < 0, the function is called strictly decreasing. O a b x
fig. 3.2
A function is called monotonic in the interval [a, b], if it is increasing or decreasing in
the interval [a, b].

3.3 concaviTY, convEXiTY anD PoinTs of


infLEcTion of a curvE
A function f (x) is concave upwards (convex downwards) if
(i) f (x) is continuous in the closed interval [a, b],
(ii) f (x) is differentiable in the open interval (a, b), and
(iii) f ≤(x) > 0 throughout the interval (a, b).
3.2 Chapter 3 Curve Sketching

O x
fig. 3.3
A function f (x) is concave downwards (convex upwards) if
(i) f (x) is continuous in the closed interval [a, b],
(ii) f (x) is differentiable in the open interval (a, b), and
(iii) f ≤(x) < 0 throughout the interval (a, b).
y

O x

fig. 3.4
A point at which f ≤(x) = 0 is known as point of inflection where the function changes
the direction of concavity from downwards to upwards or vice versa.
y

O x

fig. 3.5

3.4 MaXiMa anD MiniMa


Maxima and minima are largest and smallest value respectively that a function takes
either within a given neighbourhood or on the entire function domain.
First Derivative Test for Local Extremum
(a) f (x) has a local maximum at x = a, if f ¢(a) = 0 and f ¢(x) changes sign from
positive to negative at x = a.
3.4 Maxima and Minima 3.3

(b) f (x) has a local minimum at x = a, if f ¢(a) = 0 and f ¢(x) changes sign from
negative to positive at x = a.
Second Derivative Test for Local Extremum
(a) f (x) has a local maximum at x = a, if f ¢(a) = 0 and f ≤(a) < 0.
(b) f (x) has a local minimum at x = a, if f ¢(a) = 0 and f ≤(a) > 0.
(c) The test fails if f ¢(a) = 0 and f ≤(a) = 0. f (x) may have a local maximum, a local
minimum or neither.

Example 1
Discuss the function f (x) = x4 – 4x3 with respect to increasing or decreasing
nature, concavity, points of inflection and local maxima and minima.
Solution
f (x) = x4 – 4x3
f ¢(x) = 4x3 – 12x2 = 4x2 (x – 3)
f ≤(x) = 12x2 – 24x = 12x (x – 2)
(i) Increasing or decreasing function
(a) f (x) is an increasing function if f ¢(x) > 0 i.e. 4x2(x − 3) > 0, i.e. x2(x − 3) >
0, i.e. x − 3 > 0 since x2 > 0 i.e. x > 3.
Hence, f (x) is increasing in the interval (3, •).
(b) f (x) is a decreasing function if f ¢(x) < 0, i.e. 4x2(x − 3) < 0, i.e. x2(x − 3) <
0, i.e. x − 3 < 0 since x2 > 0, i.e. x < 3.
Hence, f (x) is decreasing in the interval (– •, 3).
(ii) Concavity
(a) f (x) is concave upwards if f ≤(x) > 0, i.e. 12x(x − 2) > 0, i.e. x(x − 2) > 0.
Now, x(x − 2) > 0 if
Case (I) x > 0 and x − 2 > 0, i.e. x > 0 and x > 2, i.e. x > 2.
Case (II) x < 0 and x − 2 < 0, i.e. x < 0 and x < 2, i.e. x < 0
Hence, f ≤(x) > 0 in the intervals (− •, 0) and (2, •) and the function f (x) is
concave upwards in these intervals.
(b) f (x) is concave downwards if f ≤(x) < 0, i.e. 12x(x − 2) < 0, i.e. x(x − 2) < 0
i.e. x(x – 2) < 0
Now, x(x − 2) < 0 if
Case (I) x > 0 and x−2 < 0, i.e. x > 0 and x < 2, i.e. 0 < x < 2.
Case (II) x < 0 and x−2 > 0, i.e. x < 0 and x > 2 but this is not possible.
Hence, f ≤(x) < 0 in the interval (0, 2) and the function f (x) is concave down-
wards in this interval.
(iii) Points of inflection
At the points of inflection, f ≤(x) = 0, i.e. 12x(x − 2) = 0, i.e. x = 0, x = 2.
At x = 0, f (x) = 0
3.4 Chapter 3 Curve Sketching

At x = 2, f (x) = (2)4 − 4(2)3 = −16


Hence, the points of inflection are (0, 0) and (2, −16).
(iv) Local maxima and minima
At stationary point, f ¢(x) = 0, i.e. 4x2(x − 3) = 0, i.e. x = 0 or x = 3.
f ≤(0) = 0
f ≤(3) = 12(3)(3−2) = 36 > 0
Hence, f (x) has a local minimum at x = 3.

3.5 Tracing of carTEsian curvEs


The points to be taken into consideration while tracing a Cartesian curve f (x, y) = 0
are as follows:
(i) Symmetry
(a) The curve is symmetric about x-axis if the powers of y occurring in the
equation are all even, i.e. f (x, –y) = f (x, y).
(b) The curve is symmetric about y-axis if the powers of x occurring in the
equation are all even, i.e. f (–x, y) = f (x, y).
(c) The curve is symmetric about the line y = x, if on interchanging x and y, the
equation remains unchanged, i.e. f (y, x) = f (x, y).
(d) The curve is symmetric about the line y = –x if on replacing x by –y and y
by –x, the equation remains unchanged, i.e. f (–y, –x) = f (x, y)
(e) The curve is symmetric in opposite quadrants or about origin if on replac-
ing x by –x and y by –y, the equation remains unchanged, i.e. f (–x, –y) =
f (x, y).

(ii) Origin
The curve passes through the origin if there is no constant term in the equation.
(a) If the curve passes through the origin, the tangents at the origin are
obtained by equating the lowest degree term in x and y to zero.
(b) If there are two or more tangents at the origin, it is called a multiple point.
The multiple point is called a node, a cusp or an isolated point if the
tangents at this point are real and distinct, real and coincident or imaginary
respectively.
(iii) Points of Intersection
(a) The points of intersection of the curve with x and y axis are obtained by
putting y = 0 and x = 0 respectively in the equation of the curve.
(b) Tangent at the point of intersection is obtained by shifting the origin to this
point and then equating the lowest degree term to zero.

(iv) Region of Existence


This region is obtained by expressing one variable in terms of other, i.e.,
y = f (x) [or x = f (y)] and then finding the values of x (or y) at which y(or x)
3.5 Tracing of Cartesian Curves 3.5

becomes imaginary. The curve does not exist in the region which lies between
these values of x (or y).
(v) Asymptotes
(a) Asymptotes parallel to x-axis are obtained by equating the coefficient of
highest degree term of x in the equation to zero.
(b) Asymptotes parallel to y-axis are obtained by equating the coefficient of
highest degree term of y in the equation to zero.
(c) Oblique asymptotes are obtained by the following method:
Let y = mx + c be the asymptote to the curve and f2(x, y), f3(x, y) are the
second and third degree terms in the equation.
Putting x = 1 and y = m in f2(x, y) and f3(x, y)
f2(x, y) = f2(1, m) or f2(m)
f3 (x, y) = f3(1, m) or f3(m)
f2 (m)
Find c= − .
f 3′ (m)
Solve f3(m) = 0
m = m1, m2, …
Calculate c at m1, m2, …
Substituting the values of m and c in y = mx + c, we get oblique asymptotes
to the curve.
(vi) Interval of Increasing-decreasing Function
dy
dy
(a) The curve increases strictly in the interval in which > 0.
ddxx
dy
dy
(b) The curve decreases strictly in the interval in which < 0.
ddxx

(c) The curve attains its maximum and minimum values at the points where
dy
dy
= 0.
dx
dx

Example 1
Trace the cissoid y2 (a – x) = x3, a > 0. [Winter 2015]
Solution
(i) Symmetry: The curve is symmetric about x-axis.
(ii) Origin: The curve passes through the origin.
Equating the lowest degree term, i.e. ay2 to zero, we get y = 0. Thus, x-axis is
the tangent at the origin.
(iii) Points of Intersection: Putting y = 0, we get x = 0. Thus, the curve meets the
coordinate axes only at the origin.
3.6 Chapter 3 Curve Sketching

x
(iv) Region of Existence: From the equation of the curve, y = ± x which
a−x
becomes imaginary when x < 0 or x > a. Hence, the curve does not exist in the
region – • < x < 0 and a < x < •. Thus, the curve lies in the region 0 < x < a.
(v) Asymptotes:
(a) Since coefficient of highest degree term of
x is constant, there is no asymptote parallel
to x-axis.
(b) Equating the coefficient of highest degree
term of y to zero, we get a – x = 0. Thus,
x = a is the asymptote parallel to y-axis.
(vi) Interval of Increasing-decreasing Function:

dyy x 2 (3a − 2 x)
d
=
dx 2 y (a − x) 2
dx

Since the curve is symmetric about x-axis, con-


sidering the part of the curve above fig. 3.6
x-axis (y > 0),
dy
dy 3
> 0, when x < a, i.e. 0 < x < a [In the region of existence]
dxx
d 2
Thus, curve is strictly increasing in this interval.

Example 2
Trace the curve y2(2a – x) = x3. [Summer 2017]
Solution
(i) Symmetry: The curve is symmetric about x-axis.
(ii) Origin: The curve passes through the origin.
Equating the lowest degree term, i.e. 2ay2 to zero, we get y = 0. Thus, x-axis is
the tangent at the origin.
(iii) Points of intersection: Putting y = 0, we get x = 0. Thus, the curve meets the
coordinate axes only at the origin.
x
(iv) Region of Existence: From the equation of the curve, y = ± x
2a - x
which becomes imaginary when x < 0 or x > 2a. Hence, the curve does not
exist in the region – • < x < 0 and 2a < x < •. Thus, the curve lies in the region
0 < x < 2a.
3.5 Tracing of Cartesian Curves 3.7

(v) Asymptotes:
(a) Since coefficient of highest degree term of x is constant, there is no
asymptote parallel to x-axis.
(b) Equating the coefficient of highest degree term of y to zero, we get
2a – x = 0.
Thus, x = 2a is the asymptote parallel to
y-axis.
(vi) Interval of Increasing-decreasing Function:

dy x 2 (6 a - 2 x )
=
dx 2 y (2 a - x )2 2a

Since the curve is symmetric about x-axis,


considering the part of the curve above x-
dy
axis (y > 0), > 0 , when x < 3a [In the
dx
regions of existence]
Thus, the curve is strictly increasing in the
region 0 < x < 2a.
fig. 3.7

Example 3
Trace the witch of agnesi xy2 = 4a2 (a – x). [Winter 2013]
Solution
(i) Symmetry: The curve is symmetric about x-axis.
(ii) Origin: The curve does not pass through the origin.
(iii) Points of Intersection: Putting y = 0, we get x = a. Thus, the curve meets
x-axis at A(a, 0).
Shifting the origin to A(a, 0) by putting x = X + a and y = Y + 0 in the equation
of the curve, (X + a)Y 2 = 4a2 (–X), (X + a)Y 2 + 4a2 X = 0.
Equating the lowest degree term i.e. 4a2 X to zero, we get X = 0, x – a = 0.
Thus, x = a is the tangent at A(a, 0).
a−x
(iv) Region of Existence: From the equation of the curve, y = ± 2a which
x
becomes imaginary when x < 0 or x > a. Hence, the curve does not exist
in the region – • < x < 0 and a < x < •. Thus, the curve lies in the region
0 < x < a.
3.8 Chapter 3 Curve Sketching

(v) Asymptotes: y
(a) Equating the coefficient of highest
degree term of x to zero, we get
y2 + 4a2 = 0 which gives imaginary
values. Thus, there is no asymptote
parallel to x-axis. x=a
(b) Equating the coefficient of highest
degree term of y to zero, we get x = 0.
Thus, y-axis is the asymptote. O A(a, 0) x
(vi) Interval of Increasing-decreasing Func-
tion:
dy
dy 2a 3
=− 2
dx
dx x y
Since the curve is symmetric about
x-axis, considering the part of the curve
dy
dy fig. 3.8
above x-axis (y > 0), < 0 for all values of x.
dx
dx
Thus, the curve is strictly decreasing in the 0 < x < a.

Example 4
Trace the curve y2(a + x) = x2(a – x) a > 0. [Winter 2016]
Solution
(i) Symmetry: The curve is symmetric about x-axis
(ii) Origin: The curve passes through the origin.
Equating the lowest degree term, i.e. ay2 – ax2 to zero, we get y = ± x. Thus,
y = ± x are the two tangents at origin.
(iii) Points of intersection: Putting y = 0, we get x = 0, x = a. Thus, the curve meets
x-axis at O(0, 0) and A(a, 0).
Now, shifting the origin to A(a, 0) by putting x = X + a and y = Y + 0 in the
equation of the curve,
Y2[a + X + a] = [X + a]2 [a – X – a]
Y2[X + 2a] = [X + a]2 [–X]
Y2[X + 2a] + X[X2 + 2aX + a2] = 0
Equating the lowest degree term is a2X to zero, we get X = 0, x – a = 0. Thus,
x = a is the tangent at A(a, 0).
(iv) Region of Existence: From the equation of the curve, y = ± x a - x which
becomes imaginary when x < –a or x > a. a+x
Hence, the curve does not exist in the region – • < x < –a and a < x < •.
Thus, the curve lies in the region –a < x < a.
3.5 Tracing of Cartesian Curves 3.9

(v) Asymptotes:
(a) Since the coefficient of highest degree term of x is constant, there is no
asymptote parallel to x-axis.
(b) Equating the coefficient of
highest degree term of y to
zero, we get x + a = 0. Thus,
x = –a is the asymptote parallel
to y-axis.
Since the curve meets the x-axis
at two points O(0, 0) and A(0, 0) a
loop exists in the region 0 < x < a.
Also, y = ± x are the tangents
at the origin. Hence, after pass-
ing through the origin, the curve
extends towards the asymptotes fig. 3.9
parallel to y-axis, i.e. x = –a in the
region –a < x < 0.

Example 5
Trace the strophoid y2 (a + x) = x2 (b – x).
Solution
(i) Symmetry: The curve is symmetric about x-axis.
(ii) Origin: The curve passes through the origin.
b
Equating the lowest degree term i.e. ay2 – bx2 to zero, we get y = ± x. Thus,
a
b
y=± are the two tangents at origin.
a
(iii) Points of Intersection: Putting y = 0, we get x = 0, b. Thus, the curve meets
x-axis at O(0, 0) and A(b, 0). Shifting the origin to A(b, 0) by putting x = X + b and
y = Y + 0 in the equation of the curve,
Y 2(a + X + b) = (X + b)2 (–X)
Y 2(a + b + X ) + X (X 2 + 2bX + b2) = 0
Equating the lowest degree term, i.e. b2X to zero, we get X = 0, x – b = 0. Thus,
x = b is the tangent at A(b, 0).
b−x
(iv) Region of Existence: From the equation of the curve, y = ± x which
a+ x
becomes imaginary when x < – a or x > b. Hence, the curve does not exist in the
region – • < x < – a and b < x < •. Thus, the curve lies in the region – a < x < b.
3.10 Chapter 3 Curve Sketching

(v) Asymptotes:
(a) Since the coefficient of highest degree term of x is constant, there is no
asymptote parallel to x-axis.
(b) Equating the coefficient of highest degree term of y to zero, we get
x + a = 0. Thus, x = – a is the
asymptote parallel to y-axis.
Since the curve meets x-axis at
two points O(0, 0) and A(b, 0), a
loop exists in the region 0 < x < b.
b
Also, y = ± x are the tangents
a
at the origin. Hence, after passing
through the origin, the curve
extends towards the asymptote
x = – a in the region – a < x < 0.
fig. 3.10

Example 6
Trace the Folium of Descartes x3 + y3 = 3axy. [Summer 2014]
Solution
(i) Symmetry: The curve is not symmetric about the coordinate axes but is sym-
metric about the line y = x, since after interchanging y and x, equation of the
curve remains unchanged.
(ii) Origin: The curve passes through the origin.
Equating the lowest degree term, i.e. xy to zero, we get x = 0 and y = 0. Thus,
x = 0 and y = 0 are the tangents at the origin.
(iii) Points of Intersection: y

(a) Putting y = 0, we get x = 0. Thus,


the curve meets the coordinate y=
x

axes only at the origin. A


F 3a , 3a I
H 2 2K
(b) Putting y = x, we get 2x3 = 3ax2, x + y = 3a

3a 3a O
x = 0, and y = 0,
2 2
Thus, the curve meets the line
 3a 3a 
y = x at O(0, 0) and A  ,  .
 2 2 x+y+a=0

fig. 3.11
 3a 3a 
Tangent at A  ,  is given by,
2 2
Ê 3a ˆ Ê dy ˆ Ê 3a ˆ
ÁË y - 2 ˜¯ = ÁË dx ˜¯ Ê 3a 3a ˆ ÁË x - 2 ˜¯
ÁË 2 ,
2 ˜¯
3.5 Tracing of Cartesian Curves 3.11

È ay - x 2 ˘ Ê 3a ˆ
=Í 2 ˙ ÁË x - 2 ˜¯
ÍÎ y - ax ˙˚ Ê 3a , 3a ˆ
Á Ë 2 2¯˜

Ê 3a ˆ
= -1 Á x - ˜
Ë 2¯
 3a 3a 
Thus, x + y = 3a is the tangent at A  2 , 2  .

(iv) Region of Existence: In the equation of the curve, x and y cannot be nega-
tive simultaneously, otherwise equation of the curve will not be satisfied.
Thus, the curve does not exist in the region where x < 0 and y < 0, i.e. third
quadrant.
(v) Asymptotes:
(a) Since coefficients of highest degree term of x and y are constant, the
curve does not have any asymptotes parallel to coordinate axes.
(b) Oblique Asymptotes: Let y = mx + c be the asymptote of the curve.
Putting x = 1 and y = m in the third and second degree terms of the equa-
tion separately
f3(m) = 1 + m3, f2(m) = –3am
Solving f3(m) = 0, 1 + m3 = 0, m = –1
f2 (m) −3am a
c=− =− = = −a
f 3′ (m) 3m 2 m

Thus, y = –x – a, i.e. x + y + a = 0 is the asymptote of the curve.


Since no part of the curve lies in the third quadrant and coordinate axes
are the tangents at the origin, after passing through the origin, the curve
extends towards the asymptote x + y + a = 0 in the second and fourth
quadrants.

Example 7
x
Trace the catenary y = c cosh c .

Solution
Rewriting the equation
c  cx − 
x
y= e + e c
2  

(i) Symmetry: The curve is symmetric about y-axis, since on replacing x by –x,
equation remains unchanged.
(ii) Origin: The curve does not pass through the origin.
(iii) Points of Intersection: Putting x = 0, we get y = c.
3.12 Chapter 3 Curve Sketching

From the equation of the curve,


dy c Ê 1 c 1 - c ˆ
x x
dy
= Á e - e ˜
dx 2 Ë c
dx c ¯
dyy
d
=0
dx (0,
dx 0 ,c )

Thus, the tangent is parallel to x-axis at A(0, c). The curve does not meet
x-axis.
(iv) Region of Existence: Since
x
1 £ cosh < • for – • < x < •, the curve
c
lies in the region c £ y < •,
–• < x < •.
(v) There is no asymptote to the curve.
(vi) Interval of Increasing-decreasing Function:
dyy 1 Ê c - ˆ
x x
d
= Áe - e c ˜
dx 2 Ë
dx ¯
x y
= sinh
c
dy
dy
(a) > 0, when x > 0
dxx
d
Thus, the curve is strictly A(0, c)
increasing in 0 < x < •.
dy
dy O x
(b) < 0, when x < 0
dx
dx
Thus, the curve is strictly
decreasing in – • < x < 0. fig. 3.12

Example 8
Trace the curve y (x2 + a2) = a3.
Solution
(i) Symmetry: The curve is symmetric about y-axis.
(ii) Origin: The curve does not pass through the origin.
(iii) Points of Intersection:
(a) Putting x = 0, we get y = a. Thus, the curve meets the y-axis at A(0, a).
Shifting the origin to A(0, a) by putting x = X + 0 and y = Y + a in the equa-
tion of the curve,
3.5 Tracing of Cartesian Curves 3.13

(Y + a)(X 2 + a2) = a3
(Y + a) X 2 + a2Y = 0
Equating the lowest degree term, i.e. a2Y to zero, we get Y = 0, y – a = 0.
Thus, y = a is the tangent at A(0, a).
(b) The curve does not meet x-axis.
a− y
(iv) Region of Existence: From the equation of the curve, x = ± a which
y
becomes imaginary when a – y < 0, i.e. y > a and y < 0. Thus, the curve does
not exist in the region a < y < • and – • < y < 0. Hence, the curve lies in the
region 0 < y < a.
(v) Asymptotes:
(a) Equating the coefficient of highest degree term of x to zero, we get y = 0.
Thus, x-axis is the asymptote parallel to x-axis.
(b) Since coefficient of highest degree term of x is constant, there is no
asymptote parallel to y-axis.
(vi) Interval of Increasing-decreasing Function:

ddyy 2 xa 3
=−
( )
2
dx
dx x2 + a2
dy
dy
< 0 when x > 0
dxx
d
Thus, y is strictly decreasing in the
interval 0 < x < •.
dyy
d fig. 3.13
> 0 when x < 0
dx
dx
Thus, y is strictly increasing in the interval – • < x < 0.

Example 9
Trace the curve (x2 – a2)(y2 – b2) = a2 b2.
Solution
(i) Symmetry: The curve is symmetric about both x and y axes.
(ii) Origin: The curve passes through the origin.
Equating the lowest degree terms, i.e. – b2 x2 – a2 y2 to zero, we get imaginary
values. Thus, tangents at the origin are imaginary. Hence, the origin is an
isolated point.
(iii) Points of Intersection: The curve does not meet x and y axes.
bx
(iv) Region of Existence: From equation of the curve, y = ± ,
x - a2
2
3.14 Chapter 3 Curve Sketching

ay y
x=±
y 2 − b2
y is imaginary when x2 – a2 < 0,
i.e., – a < x < a and x is
imaginary when y2 – b2 < 0, i.e.
– b < y < b.
x = −a x=a y=b
Hence, the curve does not
exist in the region where – a O y = −b x
< x < a and –b < y < b. Thus,
the curve lies in the region
– • < x < – a, a < x < • and
– • < y < – b, b < y < •.
(v) Asymptotes:
(a) Equating the coefficient
of highest degree term of x fig. 3.14
2 2
to zero, we get y – b = 0. Thus, y = ± b are the asymptotes parallel to
x-axis.
(b) Equating the coefficient of highest degree term of y to zero, we get
x2 – a2 = 0. Thus, x = ± a are the asymptotes parallel to y-axis.
(vi) Interval of Increasing-decreasing Function:
ddyy ba 2
=− 3
dx
dx
(x 2
− a2 )2

dyy
d
< 0 for all values of x in the region of existence.
dx
dx
Thus, y is strictly decreasing.

Example 10
Trace the curve y2 = (x – 1) (x – 2) (x – 3).
Solution
(i) Symmetry: The curve is symmetric about x-axis.
(ii) Origin: The curve does not pass through the origin.
(iii) Points of Intersection: Putting y = 0, we get x = 1, 2, 3. Thus, the curve meets the
x-axis at A(1, 0), B(2, 0) and C(3, 0). Shifting the origin to A(1, 0), B(2, 0) and
C (3, 0) by putting
(a) x = X + 1, y = Y + 0 in the equation of the curve, Y 2 = X (X – 1)(X – 2)
Equating the lowest degree term, i.e. 2X to zero, we get X = 0, x – 1 = 0.
Thus, x = 1 is the tangent at A(1, 0).
3.5 Tracing of Cartesian Curves 3.15

(b) x = X + 2, y = Y + 0 in the equation of the curve, Y 2 = (X + 1) X (X – 1)


Equating the lowest degree term, i.e. –X to zero, we get x – 2 = 0. Thus,
x = 2 is the tangent at B(2, 0).
(c) x = X + 3, y = Y + 0 in the equation of the curve, Y 2 = (X + 2) (X + 1) X
Equating the lowest degree term, i.e. 2X to zero, we get X = 0, x – 3 = 0.
Thus, x = 3 is the tangent at C(3, 0).
(iv) Region of Existence: From the equation of the curve, y = ± ( xx− −1
−1))(( x − 2))(((x − 3)
which becomes imaginary when x < 1, 2 < x < 3. Hence, the curve does not
exists in the region – • < x < 1 and 2 < x < 3. Thus, the curve lies in the region
1 < x < 2 and x > 3.
(v) Asymptotes: Since the coef- y
ficients of highest degree of x
and y are constants, there are
no asymptotes to the curve.
(vi) Interval of Increasing-de-
creasing Function: x=1 x=2 x=3
dyy
d 3 x 2 − 12 x + 11

dx
dx 2 ( x − 1)( x − 2)( x − 3)
x
3( x − 1.42)( x − 2.5) O A B C
=± (1, 0) (2,0) (3,0)
2 ( x − 1)( x − 22)(
)( x − 3)

dy
dy
(a) > 0 when x > 2.5, i.e. 3 <
dxx
d
x < • [in region of existence]
and when fig. 3.15
x < 1.42, i.e., 1 < x < 1.42
Thus, y is strictly increasing in both the intervals.
dy
dy
(b) < 0 when 1.42 < x < 2.5, i.e. 1.42 < x < 2 [in region of existence]
dx
dx
Thus, y is strictly decreasing in this interval.

Example 11
Trace the curve y2 (x + a) = x2 (3a – x).
Solution
(i) Symmetry: The curve is symmetric about x-axis.
(ii) Origin: The curve passes through the origin.
Equating the lowest degree term, i.e. ay2 – 3ax2 to zero, we get y = ± x 3
Thus, y = ± x 3 are two tangents at the origin.
(iii) Points of Intersection: Putting y = 0, we get x = 0, 3a. Thus, the curve meets
the x-axis at A(3a, 0) and O(0, 0). Shifting the origin to A(3a, 0) by putting
3.16 Chapter 3 Curve Sketching

x = X + 3a and y = Y + 0 in the equation of the curve, Y 2 (X + 3a + a)


= (X + 3a)2 (–X).
Equating the lowest degree term, i.e. –9a2X to zero, we get X = 0, x – 3a = 0.
Thus, x = 3a is the tangent at A(3a, 0).
3a − x
(iv) Region of Existence: From the equation of the curve, y = ± x which
x+a
becomes imaginary when x > 3a or x < – a. Hence, the curve does not exist in
the region where 3a < x < • and – • < x < – a. Thus, the curve lies in the region
– a < x < 3a.
(v) Asymptotes:
(a) Since coefficient of highest degree
term of x is constant, there is no
asymptote parallel to x-axis.
(b) Equating the coefficient of highest
degree term of y to zero, we get
x + a = 0. Thus, x = – a is the
asymptote parallel to x-axis.
Since the curve meets the x-axis at two
points, due to symmetry a loop exists
between O(0, 0) and A(3a, 0). Also,
y = ± x 3 are the tangents at the fig. 3.16
origin, after passing through the origin
the curve extends towards the asymptote.

Example 12
x2 + 2x
Trace the curve y = .
x2 + 4
Solution
(i) Symmetry: The curve is not symmetric.
(ii) Origin: The curve passes through the origin.
Equating the lowest degree term, i.e. 4y – 2x to zero, we get x = 2y. Thus,
x = 2y is the tangent at the origin.
(iii) Points of Intersection: Putting y = 0, we get x = 0, – 2. Thus, the curve meets
the x-axis at A(–2, 0) and O(0, 0). Shifting the origin to P(–2, 0) by putting
x = X – 2, y = Y + 0 in the equation of the curve,
Y (X – 2)2 + 4 = (X – 2)2 + 2(X – 2)
Y (X 2 – 4X + 8) = X 2 + 6X
Equating the lowest degree term, i.e. 8Y – 6X to zero, we get 4y – 3(x + 2) = 0.
Thus, 4y – 3x – 6 = 0 is the tangent at (–2, 0).
(iv) Region of existence: y is defined for all values of x. Thus, the curve lies in the
region – • < x < •.
3.5 Tracing of Cartesian Curves 3.17

(v) Asymptotes:
(a) Equating the coefficient of highest degree of x to zero, we get y – 1 = 0.
Thus, y = 1 is the asymptote parallel to x-axis.
(b) Equating the coefficient of highest degree of y to zero, we get x2 + 4 = 0
which gives imaginary values. Thus, there is no asymptote parallel to
y-axis.
When y = 1, x = 2. This shows that the curve meets y = 1 at B(2, 1).
Thus, the curve approaches the asymptote y = 1 from above when x Æ + •
and from below when x Æ – •.
(vi) Interval of Increasing-decreasing Function:

dyy −
d −2
2(( x 2 − 4 x + 4) −
2 −2
2( x + 0.83)( x − 4.83
83)
= =
dx
dx ( x 2 + 4) 2 ( x 2 + 4) 2
dyy
d
(a) > 0 when – 0.83 < x < 4.83
dx
dx
Thus, the curve is strictly increasing in
this interval.
dy
dy
(b) < 0, when – • < x < – 0.83 and
dx
dx
4.83 < x < •

Thus, the curve is strictly decreasing fig. 3.17


in both the intervals.

Example 13
Trace the curve x3 + y3 = 3ax2 (a > 0).
Solution
(i) Symmetry: The curve is neither symmetric about the coordinate axes nor about
the line y = x.
(ii) Origin: The curve passes through the origin.
Equating the lowest degree term, i.e. 3ax2 to zero, we get x = 0. Thus x = 0,
i.e. y-axis is the tangent at origin.
(iii) Points of Intersection: Putting y = 0, we get x = 0, 3a. Thus, the curve meets x-
axis at O(0, 0) and A(3a, 0). Shifting the origin to A(3a, 0) by putting x = X + 3a,
y = Y + 0 in the equation of the curve,
(X + 3a)3 + Y 3 = 3a (X + 3a)2
X + 9a X + 6aX 2 + Y 3 = 0.
3 2

Equating the lowest degree term i.e. 9a2 X to zero, we get X = 0, x – 3a = 0.


Thus, x = 3a is the tangent at A(3a, 0).
3.18 Chapter 3 Curve Sketching

(iv) Region of Existence: x and y cannot be negative simultaneously, but can take
opposite signs. Thus, the curve does not exist in the region where x < 0 and
y < 0, i.e. third quadrant.
(v) Asymptotes:
(a) Since coefficients of highest degree terms in x and y are constant, the
curve does not have any asymptotes parallel to x and y-axis.
(b) Oblique Asymptote: Let y = mx + c be the asymptote of the curve.
Putting x = 1, y = m in the third and second degree terms of the equation
separately.
f3 (m) = 1 + m3, f2 (m) = –3a
Solving f3 (m) = 0, 1 + m3 = 0, m = –1,
f 2 (m) -3a
c=- =- =a
f ¢3 (m) 3m 2

Thus, y = –x + a or y + x = a is the asymptote of the curve and curve meets


 a 2a 
the asymptote at  , 
3 3 
(vi) Interval of Increasing-decreasing Function:
dyy x(2a − x)
d
=
dx
dx y2

dyy
d
(a) < 0, when x < 0 and x > 2a fig. 3.18
dx
dx
Thus, the curve is strictly decreasing in – • < x < 0 and 2a < x < •.
dyy
d
(b) > 0, when 0 < x < 2a
dx
dx
Thus, the curve is strictly increasing in 0 < x < 2a.

Example 14
Trace the curve y3 = a2x – x3.
Solution
(i) Symmetry: The curve is symmetric in opposite quadrants, since on replacing x
by –x and y by –y, equation remains unchanged.
(ii) Origin: The curve passes through the origin.
Equating the lowest degree term, i.e. a2 x to zero, we get x = 0. Thus, x = 0, i.e.,
y-axis is the tangent at origin.
3.5 Tracing of Cartesian Curves 3.19

(iii) Points of Intersection: Putting y = 0, we get x = 0, ± a. Thus, the curve meets


the x-axis at O(0, 0), A(a, 0) and B(– a, 0). Shifting the origin to A(a, 0) and
B(– a, 0) by putting
(a) x = X + a, y = Y + 0 in the equation of the curve,
Y 3 = a2 (X + a) – (X + a)3
Y 3 + X 3 + 3aX 2 + 2a2 X = 0
Equating the lowest degree term, i.e. 2a2 X to zero, we get X = 0,
x – a = 0. Thus, x = a is the tangent at A(a, 0).
(b) x = X – a, y = Y + 0 in the equation of the curve,
y3 = a2 (X – a) – (X – a)3
Y 3 + X 3 – 3aX 2 + 2a2 X = 0
Equating the lowest degree term, i.e. 2a2 X to zero, we get X = 0, x + a = 0.
Thus, x = – a is the tangent at B(– a, 0).
(iv) Region of Existence: The curve exists everywhere in the region – • < x < •.
(v) Asymptotes:
(a) Since coefficients of highest degree term of x and y are constant, the curve
does not have any asymptotes parallel to coordinate axes.
(b) Oblique Asymptotes: Let y = mx + c be the asymptote of the curve.
Putting x = 1 and y = m in the third and second degree terms of the equa-
tion separately
f3 (m) = m3 + 1, f2 (m) = 0 y
3
Solving f3 (m) = 0, m + 1 = 0,
m = –1,
f2 ( m)
c=− =0 B(−a, 0) O A(a, 0)
f 3′ (m) x
Thus, y = –x is the asymptote of the y=
curve. −x
(vi) Interval of Increasing-decreasing Func-
tion: fig. 3.19
dyy a − 3 x
d 2 2
=
dx
dx 3y2
dyy
d −a a
(a) < 0, when x < and x >
dx
dx 3 3
−a
Thus, the curve is strictly decreasing in the region – • < x < and
3
a
<x<•
3
3.20 Chapter 3 Curve Sketching

dyy
d −a a
(b) > 0, when <x<
dx
dx 3 3
−a a
Thus, the curve is strictly increasing in the region <x< .
3 3

Example 15
Trace the curve 9ay2 = x(x – 3a)2. [Summer 2015]
Solution
(i) Symmetry: The curve is symmetric about the x-axis.
(ii) Origin: The curve passes through the origin.
Equating the lowest degree term, i.e., 9a2 x = 0 to zero, we get x = 0. Thus,
y-axis is the tangent at origin.
(iii) Points of Intersection: Putting y = 0, we get x = 0 and x = 3 a. Thus, the curve
meets the x-axis at O(0, 0) and A(3a, 0). Shifting the origin to A(3a, 0) by
putting x = X + 3a and y = Y + 0 in the equation of the curve,
9aY2 = (X + 3a)X2
Equating the lowest degree term, i.e., 9aY2 –3aX2 to zero, we get
X x - 3a
Y= ± =±
3 3
x - 3a
y=± are two real and distinct tangents at A(3a, 0). Hence, point
3
A(3a, 0) is a node.
(iv) Region of Existence: y
From the equation of the curve,

x( x - 3a )2 x – 3a
y=± y=–
3
9a
y is imaginary when x < 0.
Hence, the curve does not exist
O a 2a (3a, 0) x
in the region x < 0.
(v) Asymptotes: Since coefficients
of highest degree term of x and y x – 3a
y=
are constant, the curve does not 3
have any asymptotes parallel to fig. 3.20
the co-ordinate axes.
3.5 Tracing of Cartesian Curves 3.21

EXErcisE 3.1
Trace the following curves:
1. a 2 x 2 = y 3 (2a − y )
ans.: y
(0, 2a)

O x

2. 4 ay 2 = x(x − 2a)2
ans.: y

(2a, 0)
O x

3. ay 2 = x 3
ans.: y

O x

4. ay2 – x(y + x2) ans.:


y

O x
3.22 Chapter 3 Curve Sketching

5. a 2 y 2 = x 2 (x − a)(2a − x)
ans.:
y

O (a, 0) (2a, 0) x

6. y = (x 2 − x − 6)(x − 7)
ans.:
y
(0, 42)

(−2, 0) (3, 0)
O (7, 0) x

7. y = x 3 − 12 x − 16
ans.: y

(−2, 0) (4, 0)
O x

(0, −16)

(2, −32)

x 2 − 3x
8. y = ans.: y
x −1
x=1
2
y=
x−

O (3, 0) x
3.5 Tracing of Cartesian Curves 3.23

9. y x = x − a
2 2 2 2

ans.: y

y=1
x = −a x=a

O x

y = −1

10. y 2 (x − a) = x 2 (x + a), a > 0


ans.: y
a
y =−
x = −a x−

x=a

O x

x+
y=
−a

11. y (a − x ) = a x
2 2 2 3
ans.: y

x = −aa x=a

O x

x2 + 1
12. y = ans.:
x2 − 1 y

x = −1 x=1
y=1

O x
(−1, 0)
3.24 Chapter 3 Curve Sketching

3.6 Tracing of ParaMETric curvEs


The points to be taken into consideration while tracing a parametric curve x = f1 (t),
y = f2 (t), where t is a parameter are as follows:
(i) Symmetry
(a) The curve is symmetric about x-axis if x is an even function and y is an
odd function of t.
(b) The curve is symmetric about y-axis if x is an odd function and y is an
even function of t.
(c) The curve is symmetric about y-axis if after replacing t by p – t, x be-
comes negative and y remains positive.
(ii) Origin
The curve passes through the origin if there exists at least one real value of t
at which x = 0 and y = 0.
(iii) Points of Intersection
(a) Points of intersection with x-axis: Find the value of t at which y = 0 and
then find x for this value of t.
(b) Points of intersection with y-axis: Find the value of t at which x = 0 and
then find y for this value of t.
(iv) Tangents
ddyy
(a) Tangent is parallel to x-axis at the point where = 0.
dx
dx
dy
dy
(b) Tangent is parallel to y-axis at the point where Æ •.
ddxx
(v) Maximum and Minimum Values
Determine the maximum and minimum values of x and y if exists.
(vi) Region
Determine the region where x and y are real. The curve does not exist in the
region, where x or y is imaginary.
(vii) Variation of x and y
Determine the values of x and y for some suitable values of t.
Note: If x and y are periodic functions of t having the same period, then the curve is
traced for one period only.

Example 1
Trace the hyplocycloid x = a cos3 t, y = b sin3 t.
Solution
x and y are periodic functions of t with period 2p. Hence, the curve is traced between
0 to 2p.
3.6 Tracing of Parametric Curves 3.25

(i) Symmetry: The curve is symmetric about x-axis since x is an even function of t
and y is an odd function of t. Also the curve is symmetric about y-axis since after
replacing t by p – t, x becomes negative but y remains positive.
(ii) Origin: The curve does not pass through the origin.
(iii) Points of Intersection:
(a) At t = 0, y = 0 and x = a.
π
(b) At t = , x = 0 and y = b.
2
Thus, the curve meets the x-axis at A(a, 0) and y-axis at B(0, b).

ddxx dyy
d
(iv) Tangents: = –3a cos2 t sin t, = 3b sin2 t cos t
dt dt

dyy dy/dt
d 3b sin 2 t cos t b
= = = − tan t
dx dx/dt −3a cos 2 t sin t
dx a
dyy
d
= 0, when t = 0
dx
dx
Thus, the tangent is x-axis at t = 0 i.e., at A(a, 0).
dy
dy p
Æ • when t = .
dxx
d 2
p
Thus, the tangent is y-axis at t = , i.e., at B(0, b).
2
(v) Maximum and Minimum Values: Maximum values of x and y are a and b re-
spectively since maximum value of cos t and sin t is 1. Minimum values of x and
y are – a and – b respectively since minimum value of cos t and sin t is –1.
(vi) Region: The curve lies in the region – a < x < a and – b < y < b.
(vii) Asymptotes: There is no asymptote of the curve since x and y are finite for all
values of t.
(viii) Variation of x and y:
y
B(0, b)
p p p p
t 0
6 4 3 2
3 3a a a
x a 0
8 2 2 8
C O A(a, 0) x
b b 3 3b (−a, 0)
y 0 b
8 2 2 8

D(0, − b)

fig. 3.21
3.26 Chapter 3 Curve Sketching

Example 2
 t 
Trace the tractrix x = a cos t + log tan    , y = a sin t.
  2 
Solution
(i) Symmetry: The curve is symmetric about x-axis since x is an even function of t
and y is an odd function of t.
Replacing t by p – t,
 π t  
x = a cos(π − t ) + log tan  −  
  2 2 
 t 
= a  − cos t + log cot 
 2 
 t 
= a  − cos t − log tan 
 2 
= –x
y = a sin (p – t)
= a sin t
=y
Thus, the curve is symmetric about y-axis.
(ii) Origin: The curve does not pass through the origin.
(iii) Points of Intersection:
(a) At t = 0, y = 0 and x Æ – • [∵ log 0 Æ – •]
p
(b) At t = , x = 0 and y = a
2
Thus, the curve meets the y-axis at A(0, a) and does not meet y-axis.

Tangents: x = a cos t + log tan 2 


1 t
(iv)
 2 2
 
ddxx  1 1 t 2 t 1

= a  − sin t + ⋅ ⋅ 2 tann sec ⋅ 
dt  2 t 2 2 2
tan 2
 2 
 1 
= a  − sin t +
 siinn t 
cos 2 t
a cos
=
sin t
dyy
= a cos
cos t
dt
3.6 Tracing of Parametric Curves 3.27

dy
dy dt a cos t ⋅ sin t
= = = tan t
dx dx a cos 2 t
dt
ddyy p
At point A(0, a): = tan → ∞
dx
dx 2
Thus, the tangent is y-axis.
(v) Maximum and Minimum Values: Maximum and minimum values of y are a
and – a respectively since maximum and minimum values of sin t are 1 and –1
respectively.
x lies between – • to •.
(vi) Region: The curve lies in the region – a < y < a and – • < x < •.
(vii) Asymptotes: lim x = - • and lim x = •
Æ0
tÆ0
ttÆ t Æp
At t = 0 and t = p, y = 0
Thus, y = 0 i.e., x-axis is the asymptote of the curve.
(viii) Variation of x and y:

A(0, a)
p p p p
t 0
6 4 3 2
x - • - 0.45 a - 0.17 a - 0.04 a 0
t=0 t=p
y 0 0.5 a 0.71 a 0.87 a a
O x

fig. 3.22

Example 3
Trace the cycloid x = a (t + sin t), y = a (1 – cos t).
Solution
(i) Symmetry: The curve is symmetric about y-axis since x is an odd function of
t and y is and even function of t.
(ii) Origin: At t = 0, x = 0 and y = 0. Thus, the curve passes through the
origin.
3.28 Chapter 3 Curve Sketching

(iii) Points of Intersection:


(a) x = 0 only at t = 0
Thus, the curve meets the y-axis only at origin.
(b) If y = 0, cos t = 1, t = 0, ± 2p, ± 4p,
Then x = 0, ± 2ap, ± 4ap, …
Thus, the curve meets the x-axis at (0, 0), (± 2ap, 0), (± 4ap, 0) …
(iv) Tangents:
ddxx dy
dy
= a (1 + cos t ) , = a sin t
dt dt
dy
dy t t
2 sin cos
dy dt
dy a sin t 2 2 t
= = = = tan
dx dx
dx dx a a((1 + cos t ) 2 t 2
2 cos
dt 2
dyy
d
(a) = 0, at t = 0, ± 2p, ± 4p, …
dx
dx
Thus, tangent is x-axis at (0, 0), (± 2ap, 0), (± 4ap, 0), …
dyy
d
(b) Æ •, at t = ± p, ± 3p, ± 5p, …
dx
dx
Thus, tangent is parallel to y-axis at (± ap, 2a), (± 3ap, 2a), (± 5ap, 2a), …
(v) Maximum and Minimum Values: Maximum and minimum values of y are 2a
and 0 since minimum and maximum values of cos t are –1 and 1.
(vi) Region: Curve lies in the region 0 < y < 2a and – • < x < •.
(vii) Asymptotes: There is no asymptote of the curve since x and y are finite for all
finite values of t.
(viii) Since sin t is a periodic function of period 2p, y is the periodic function of pe-
riod 2p. Thus, curve repeat itself in the intervals [0, ± 2ap], [ ± 2ap, ± 4ap], …
(ix) Variation of x and y:

p 3p
t 0 p 2p
2 2

Êp ˆ Ê 3p ˆ
x 0 a Á + 1˜ ap aÁ + 1˜ 2ap
Ë2 ¯ Ë 2 ¯

y 0 a 2a a 0
3.6 Tracing of Parametric Curves 3.29

(–3a
app,, 2a)
p (–a p, 2a)
ap,
ap ((a
ap, 2a) (3aap
p,, 2a)
p

(–2ap,
p, 0) O (0, 0) (2ap,
p, 0) x

fig. 3.23

Example 4
Trace the curve x = a sin 2t (1 + cos 2t), y = a cos 2t (1 – cos 2t).
Solution
x and y are periodic functions of t with period p, hence, we will discuss the curve only
in the interval 0 £ t < p
(i) Symmetry: Curve is symmetric about y-axis since x is an odd function of t and y is
an even function of t.
(ii) Origin: At t = 0, x = 0 and y = 0. Thus, origin lies on the curve.
p
(iii) Points of Intersection: (a) x = 0 at t = 0, ± 2 , then y = 0, – 2a.
Thus, the curve meets the y-axis at (0, 0), (0, – 2a).
p
(b) y = 0 at t = 0, ± 4
then x = 0, ± a
Thus, the curve meets the x-axis at (0, 0), (a, 0), (– a, 0).
dxx
d
(iv) Tangents: = 2a cos 2t (1 + cos 2t) + a sin 2t (– 2 sin 2t)
dt
= 2a cos 2t + 2a cos 4t
= 2a ◊ 2 cos 3t cos t
dy
dy
= – 2a sin 2t (1 – cos 2t) + a cos 2t (2 sin 2t)
dt
= 4a sin 2t cos 2t – 2a sin 2t
= 2a (sin 4t – sin 2t)
= 2a ◊ 2 cos 3t sin t
dy
dy
dyy
d 4a cos 3t sin t
= dt = = tan t
dx
dx dxx
d 4a cos 3t cos t
dt
3.30 Chapter 3 Curve Sketching

dy
dy y
(a) dx
dx = 0, at t = 0
–3a 3 a 3a 3 a
B , A ,
Thus, the tangent is x-axis at (0, 4 4 4 4
0).
dy
dy p
dx Æ • at t = 2
(b) dx O x
Thus, the tangent is y-axis at (0,
– 2a).
(v) Asymptotes: There is no asymptote
of the curve since x and y are finite C, (0, –2a)
C
for all values of t.
(vi) Variation of x and y fig. 3.24

t 0 p p p 2p 5p p
6 3 2 3 6

x 0 33a 3 a 3 0 −a 3 −33 3 0
4 4 4 4
a −3a −3a a
y 0 –2a 0
4 4 4 4

EXErcisE 3.2
Trace the following curves:
1. Astroid x = a cos3 t, y = b sin3 t
ans.:
y

(0, a)

( , 0)
(–a O (a, 0) x

(0, –a)
3.7 Tracing of Polar Curves 3.31

2. Cycloid x = a(t + sin t), y = a(1 + cos t)


ans.:
y

(–22ap, 2a) (0, 2a) (2ap


p,, 2a)

(–33ap, 0) (–ap,
p, 0) O (ap, 0) (3ap,
p, 0) x

3. x = a cos t, y = b sin t
ans.:
y
(0, b)

x
O ( , 0)
(a

4. Cycloid x = a(t – sin t), y = a(1 + cos t)


ans.:
y

(–2ap,
p, 0) (0, 2a) (2ap
p,, 2a)

x
(–ap,
p, 0) O (ap, 0)

5. Parabola x = 1 + sin t, y = 2 cos 2t


ans.:
y

2–1 2+1
,0 ,0
2 2
x
O

(–2, 0)

3.7 Tracing of PoLar curvEs

relation between Polar and cartesian coordinates


The polar coordinates r and q can be converted to the Cartesian coordinates x and y by
using the trignometric sine and cosine functions,
x = r cos q, y = r sin q
3.32 Chapter 3 Curve Sketching

The Cartesian coordinates x and y can be converted to polar coordinates r and q with
r ≥ 0 and q in the interval (-p, p) by relation,
Ê yˆ
r = x 2 + y 2 , q = tan -1 Á ˜
Ë x¯
The polar curves are given by polar coordinates (r, q ) and is written as r = f (q ) or
q = f (r).
The points to be taken into consideration while tracing a polar curve r = f (q ) are as
follows:
(i) Symmetry
(a) A curve is symmetric about the initial line q = 0 (x-axis), if the equation
remains unchanged after replacing q by –q.
p
(b) A curve is symmetric about the line q = (line through pole perpendicular to
2
the initial line), if the equation remains unchanged after replacing q by p – q.
(c) A curve is symmetric about the pole (opposite quadrant), if the equation
remains unchanged when q is replaced by p + q (or r is replaced by – r)
p
(d) A curve is symmetric about the line q = , if the equation remains un-
p 4
changed after replacing q by – q.
2
(ii) Pole
The pole lines on the curve, if for r = 0, there exists at least one real value of q.
(iii) Points of Intersection
p
Determine the points where the curve meets the initial line q = 0, q = 2 and
q = p.
(iv) Direction of Tangent
Determine f, i.e. angle between the radius vector and the tangent at the points
dq
of intersection using tan f = r .
dr
The angle f gives the direction of the tangent at the point of intersection.
(v) Region
(a) Determine the maximum and minimum value of r if exists. If minimum
value of r is a then no part of the curve lies inside the circle with radius a
and centre at pole. If maximum value of r is b then the whole curve lies
within the circle of radius b and centre at the pole.
(b) Determine the range of q in which r2 < 0, i.e. r is imaginary, then curve
does not exists in this range.
(vi) Asymptotes
If r Æ • for some q = q1 then the asymptote of the curve may exist and is
given by
r sin (q – q1) = f ¢(q1)
1
where q1 is the solution of = 0.
(q )
f (q
3.7 Tracing of Polar Curves 3.33

(vii) Variation of r
Trace the variation of r for some suitable values of q.
dr
If dq > 0 then r increases as q increases.
dr
and if dq < 0 then r decreases as q increases.
If the curve meets the line of symmetry at two points, then a loop exists
between these two points.
Note: Curve of the type r = a sin nq or r = a cos nq consists of (i) n similar loops, if n
is odd, and (ii) 2n similar loops, if n is even.
If n = 1 then the curve becomes a circle.

Example 1
Trace the cardioid r = a (1 – cos q ).
Solution
(i) Symmetry: The curve is symmetric about the initial line q = 0, since when q is
replaced by – q, equation of the curve remains unchanged.
(ii) Pole: Pole lies on the curve since when q = 0, r = 0. Tangent at the pole is the
initial line q = 0.
(iii) Points of Intersection: Putting q = p , p we get r = a, 2a respectively.
2

Thus, the curve meets the line q = p and q = p at A a


2
a,,
p
2 ( ) and B(2a, p)

respectively.
(iv) Direction of Tangent: r = a (1 – cos q )
dr
dq = a sin q

a (1 − ccos q ) 2 sin 2 q q
dq 2
tan f = r = = = tan
dr a sin q q
2 sinn cos q 2
2 2
f= q
2

At point A a,
a,( )
p
2
: f = p , thus, the tangent makes an angle p with the line
4 4
q= p .
2
At point B(2a, p): f = p , thus, the tangent is perpendicular to the line q = p.
2
3.34 Chapter 3 Curve Sketching

(v) Region: Since minimum value of cos q is –1, the maximum value of r is 2a.
Thus, the whole curve lies within a
q p
circle with centre at the pole and 2
radius 2a. p
(vi) Asymptote: There is no asymptote A a,
2
of the curve since r is finite for all
values of q. B
q=0
(vii) Variation of r: (2a,, p) O

p p 2p
q 0 p
3 2 3

r 0 a a
3
2a
2 2
fig. 3.25

Example 2
Trace the limacon of pascal r = a + b cos q, where a > 0, b > 0.
Solution
(i) Symmetry: The curve is symmetric about the line q = 0, since when q is replaced
by –q, equation of the curve remains unchanged. Three different cases arise:
Case I a > b
(ii) Pole: It does not lie on the curve. If

r = 0, cos q = – a < –1 which is not possible. Thus, r π 0 for any value of q.


b
(iii) Points of Intersection: Putting q = 0, p , p , we get r = (a + b), a, (a – b)
2
respectively. Thus, curve meets the line q = 0, q = p and q = p at A (a + b, 0),

( 2)
2
p and C (a – b, p) respectively.
B a,
a,
p
q=
(iv) Direction of Tangent: r = a + b cos q 2 p
B a,
2
dr
= –b sin q
dq
A (a + b, 0)
( + b cos q )
(a
tan f = r dq = C O q=0
dr − b sin q (a – b, p)
p)

At point A(a + b, 0), tan f Æ •,


f= p
2 fig. 3.26
Thus, the tangent is perpendicu-
lar to the initial line q = 0
3.7 Tracing of Polar Curves 3.35

( p2 )
At point B a,
a,
a  −a 
: tan f = – b , f = tan–1 
b
–1 a
 = p – tan b

Thus, the tangent makes an angle p – tan–1 a with the line q = p .


b 2

At point C (a – b, p ): tan f Æ •, f = p
2
Thus, the tangent is perpendicular to the line q = p.
(v) Region: Minimum value of r = a – b, since minimum value of cos q = –1. Thus
r is always positive.
(vi) Asymptote: There is no asymptote of the curve since r is finite for all values of q.
(vii) Variation of r:
p p 2p
q 0 3 p
2 3
b b
r a+b a+2 a a– 2 a–b

Case II a < b
(ii) Pole: It lies on the curve.
−a
If r = 0, cos q = > –1
b
 −a   −a 
Thus at q = cos–1   , r = 0. Hence, q = cos–1 
 b  is the tangent at
b 
origin.

(iii) Points of Intersection: The curve meets the line q = 0, q = p and q = p at


A (a + b, 0), B a, ( 2)
a,
p
and p q=
2

2
C (a – b, p) respectively.
(iv) Direction of Tangent: Same as a, p B
2
case I
(v) Region: Minimum value of r = a – A (a + b, 0)
b < 0, thus r is negative for some O C {– (b – a), 0} q=0
values of q.
(vi) Asymptote: Same as case I
(vii) Variation of r: Same as case I.
But, here a – b < 0. Hence, for
some values of q, r is negative. fig. 3.27
Thus, a smaller loop exists between the points 0 and C.
Case III a = b
The curve r = a (1 + cos q ) is a cardioid. [Winter 2014]
(ii) Pole: It lies on the curve, since at q = p, r = 0. Tangent at the pole is the line
q = p.
3.36 Chapter 3 Curve Sketching

(iii) Points of Intersection: Curve meets the line q = 0, q = p at A(2a, 0) and B

( 2)
2
p respectively.
a,
a,
(iv) Direction of Tangent: From Case I
a + b cos q
tan f =
− b sin q q=p
2
1 + cos q
= a, p B
− sin q 2

2 cos 2 q A (2a, 0)
= 2
O
n cos q
q q=0
− 2 sin
2 2
= – cot q
2
= tan (p q
2 2
+ ) fig. 3.28

f= p + q
2 2
At point A(2a, 0), f = p . Thus, the tangent is perpendicular to the initial line
q = 0. 2

( p2 )
At point B a,
a, , f = p + p = 3p . Thus, the tangent makes an angle
2 4 4
3p with the line q = p .
4 2
(v) Region: The maximum value of r is 2a. Thus, the whole curve lies within a
circle with centre at the pole and radius 2a.
(vi) Asymptotes: Same as case I
(vii) Variation of r:
p p 2p
q 0 p
3 2 3

r 2a 3a a a 0
2 2

Example 3
Trace the lemniscate of Bernoulli r2 = a2 cos 2q. [Summer 2015]
Solution
(i) Symmetry: The curve is symmetric about the initial line q = 0 and the line
p
q= , since when q is replaced by –q and by p – q respectively, equation of
2
the curve remains unchanged.
3.7 Tracing of Polar Curves 3.37

The curve is also symmetric about the pole since power of r is even.
(ii) Pole: It lies on the curve since r = 0, at q = ± p . Tangents at the pole are the
4
lines q = ± p .
4
(iii) Points of Intersection: The curve meets the initial line q = 0 at A(a, 0) and
B(– a, 0).
(iv) Direction of Tangent: r2 = a2 cos 2q
dr
2r = – 2a2 sin 2q
dq
dr 2
= – a sin 2q
dq r
tan f = r d r = 2 − r
dq 2

a sin 2q
a 2 cos 2q
=– 2
a sin 2q
= – cot 2q = tan
p
2 (
+ 2q )
f = p + 2q
2
At point A(a, 0), f = p . Thus, the tangent is perpendicular to the initial line
q = 0. 2

Due to symmetry, the curve is discussed only between q = 0 to q = p .


2
(v) Region:
(a) Since maximum value of cos 2q is 1, the maximum value of r is a. Thus,
the whole curve lies within a circle with centre at the pole and radius a.
 Due to symmetry
tr 
try
p 
p considering q 
(b) cos 2q < 0, if < 2q < p, i.e., p < q < .
2 4 2  p
between 0 and
 2 
p p
Thus r2 < 0, when <q< .
4 2
p p
Hence, the curve does not exists in the region <q< .
4 2
(vi) Asymptote: There is no asymp-
tote of the curve since r is finite
for all values of q.
(vii) Variation of r: Since the curve
meets the initial line at two
( )
points O 0,
p
4
and A(a, 0)

and is symmetric about the


initial line, a loop exists between
the points O and A.
fig. 3.29
3.38 Chapter 3 Curve Sketching

p p p
q 0 8 6 4

a a
r a 1
4 2 0
( 2)

Example 4
Trace the lemniscate r2 = a2 sin 2q. [Summer 2016]
Solution
(i) Symmetry: The curve is symmetric at about
(a) the pole since power of r is even
(b) the line q = p , since on replacing q by p – q, equation remains un-
4 2
changed.
(ii) Pole: It lies on the curve since r = 0 at q = 0 and q = p . Tangents at the pole
2
are the lines q = 0 and q = p .

2 2
2
(iii) Points of Intersection: The curve meets the line q = p at A a
4
a,, ( )
p
4
.
(iv) Direction of Tangent: r = a sin 2q
dr
2r
dq
= 2a2 cos 2q

dr a 2 cos 2q
=
dq r

dq r2 2
tan f = r = 2 = a 2 sin 2q = tan 2q
dr a cos 2q a cos 2q
f = 2q

( p4 )
At point A a,
a, , f = p . Thus, the tangent is perpendicular to the line
2
q= p .
4
(v) Region:
(a) Since maximum value of sin 2q is 1, the maximum value of r is a. Thus,
the whole curve lies within a circle of radius a and centre at the pole.
p
(b) sin 2q < 0, p < 2q < 2p, i.e. <q<p
2
Thus, r2 < 0, when p < q < p. Hence, the curve does not exists in the
2
p
region < q < p, i.e., second quadrant and due to symmetry in the
2
fourth quadrant too.
3.7 Tracing of Polar Curves 3.39

(vi) Asymptote: There is no asymptote of the curve since r is finite for all values of q.
(vii) Variation of r: Since, the curve p
q
p
meets the line q = p
2 q
at two 4
4
points O(0, 0) and A a,
a, ( p4 ) A a,
p
4

and is symmetric about this line,


a loop exists between the points O q=0
O and A.
p p 3p p
q 0 8 4 2
8
a a
r 0 1 a 1 0 fig. 3.30
4
( )
(2 ( )4
(2

Example 5
Trace the three-leaved rose r = a sin 3q. [Summer 2014, 2015]
Solution
Here n = 3 (odd). The curve consists of three similar loops.
(i) Symmetry: The curve is symmetric about the line q = p , since on replacing
2
q by p – q, equation of the curve remains unchanged.
(ii) Pole: It lies on the curve since r = 0 at q = 0, p , 2p , p. Tangents at the pole
3 3
are the lines q = 0, q = p , q = 2p , q = p.
3 3
(iii) Points of Intersection: The curve meets the line q = p at A − a,
p
.
2
( 2 )
(iv) Direction of Tangent: r = a sin 3q
dr
dq = 3a cos 3q
a sin 3q
tan f = r dq = = 1 tan 3q
dr 3a cos
cos 3q 3

At point A − a,( p
2 )
, tan f = 1 tan 3p Æ •, f = p . Thus, the tangent is
3 2 2

perpendicular to the line q = p .


2
(v) Region: Since, the maximum value of sin 3q is 1, the maximum value of r
is a. Thus, the whole curve lies within a circle of radius a and centre at the
pole.
3.40 Chapter 3 Curve Sketching

(vi) Asymptote: There is no asymptote of the curve since r is finite for all values
of q.
(vii) Variation of r: The curve is symmetric p
q=
above the line q = ± p and meets this 2
2
Ê pˆ
line at A Á a, - ˜ and also passes a, p
Ë 2¯ 6
through the pole O. Hence, a loop
exists between the points O and A.
This curve consists of three similar O q=0
loops. Hence, two more similar loops,
exists in the first and second quadrant
due to symmetry.
p
A a, –
p p p 2
q 0
6 3 2
fig. 3.31
r 0 a 0 –a

Example 6
Trace the four-leaved rose r = a cos 2q, a > 0.
Solution
Here n = 2 (even). The curve consists of 2n, i.e. 4 similar loops.
(i) Symmetry: The curve is symmetric about the
(a) initial line q = 0, since on replacing q by – q, equation of the curve remains
unchanged.
(b) line q = p , since on replacing q by (p – q ), equation of the curve remains
2
unchanged.
(ii) Pole: It lies on the curve, since r = 0 at q = p . The tangent at the pole is the
4
line q = p .
4
(iii) Points of Intersection: The curve meets the initial line q = 0 at A(a, 0).
(iv) Direction of Tangent: r = a cos 2q
dr
= –2a sin 2q
dq
a cos 2q
tan f = r dq = = – 1 cot 2q
dr − 2a sin
sin 2q 2
3.7 Tracing of Polar Curves 3.41

1
At point A(a, 0), tan f = – cot 0 Æ •, f = p .
2 2
Thus, the tangent is perpendicular to the initial line.
(v) Region: Since maximum value of cos 2q is 1, the maximum value of r is a.
Thus, the whole curve lies within a circle of radius a and centre at the pole.
(vi) Asymptote: There is no asymptote of the curve since r is finite for all values of
q.
(vii) Variation of r: The curve is symmetrical about the initial line q = 0 and
meet this line at A(a, 0) and also
passes through origin. Hence, a q=
p
loop exists between the points O 2
and A. This curve consists of 4 q=
p
similar loops. Hence, three more 4
similar loops can be drawn using the
symmetry about the line q = 0 and
A (a , 0)
q= p . O q=0
2

p p 3p p
q 0 8 4 2
8

a a
r a 0 – –a
2 2 fig. 3.32

EXErcisE 3.3
Trace the following curves:
1. r = a (1 + sin q )
ans.:
q p
2

B(0, 2a)
B

O A((a, 0)
A
q=0
3.42 Chapter 3 Curve Sketching

2. r2 cos 2q = a2
ans.:
q p
2 p
q
4

q=0
O

q p
4

3. r = 2a cos q
ans.:

q=p
2

q=0
O (2a, 0)

4. r = a cos 3q
ans.:
q p
2
q p
6
O A((a, 0)
A
q=0

5. r = a sin 2q
ans.:

q=p
2
q=p
4

q=0
O

q = –p
4
Multiple Choice Questions 3.43

6. r = 2 (1 – 2 sin q )

ans.:
q p
2
q 5p q p
6 6
O q=0
A(2, 0)

MuLTiPLE cHoicE QuEsTions


Choose the correct alternative in the following questions:
1. The curve x2 = 4ay is symmetric about
(a) x-axis (b) y-axis
(c) both x and y axes (d) origin
2. If only even powers of x occur in the equation of a curve, the curve is symmetric
about [Summer 2016]
(a) x-axis (b) y-axis (c) both the axes (d) y = x
3. If the equation of the curve contains only even powers of y, the curve is symmetric
about
(a) x-axis (b) y-axis (c) both the axes (d) y = x
4. On interchanging x and y, if the equation remains the same then the curve is
symmetric about
(a) x-axis (b) y-axis (c) y = x (d) y = –x
5. On replacing x and y simultaneously by –x and –y respectively, if the equation
remains unchanged, the curve is symmetric about
(a) origin (b) y = x (c) y = –x (d) x-axis
6. If there is no constant term in the equation of the curve, the curve
(a) is parallel to x-axis (b) is parallel to y-axis
(c) passes through the origin (d) is symmetrical about the origin
7. If there are two or more tangents at the origin then it is called
(a) multiple point (b) double point
(c) triple point (d) node
8. A double point is called a node if the tangents at that point are
(a) real and distinct (b) real and coincident
(c) imaginary (d) none of these
9. A double point is called a cusp if the tangents to the curve at that point are
(a) real and distinct (b) real and coincident
(c) imaginary (d) none of these
3.44 Chapter 3 Curve Sketching

10. The curve x3 + y3 = 3axy is symmetric about [Winter 2015]


(a) x-axis (b) y-axis (c) y = x (d) y = –x
11. The curve a2y = x3 is symmetric about
(a) y = x (b) y = –x (c) origin (d) both the axes
12. Which of the following curves passes through the origin?
(a) xy2 = 4a2 (2a – x) (b) y2 (a – x) = a + x
2 2
(c) y (a – x) = x (x + a) (d) none of these
13. For the curve y2 (2a – x) = x3, the origin is
(a) node (b) cusp (c) isolated point (d) none of these
14. Asymptote parallel to the y-axis of the curve (2 – x)y2 = x3 is given by
(a) x = –2 (b) x = 2 (c) y = 2 (d) y = –2
15. If the equation of the curve is unchanged when q is replaced by – q, the curve is
symmetric about
(a) pole (b) initial line q = 0
p p
(c) q = (d) q =
4 2
16. The curve r = a (1 + sin q) is symmetric about the line
p p
(a) q = (b) q = (c) q = p (d) p = 0
4 2
17. The curve r2 = a2 cos2q is symmetric about
3p p
(a) pole (b) initial line (c) q = (d) q =
2 2
18. If x = f(t) is even and y = g(t) is odd then the curve is symmetric about
(a) x-axis (b) y-axis (c) both the axes (d) origin
19. The curve y = x3 – 3x2 – 9x + 9 has a point of inflection at
(a) x = 1 (b) x = –1 (c) x = 2 (d) x = –2
20. Asymptotes parallel to the x-axis of the curve x2y2 – x2y – xy2 + x + y + 1 = 0 are
given by
(a) y = 0, y = 1 (b) y = 0, y – 1 (c) x = 0, x = 1 (d) x = 0, x = –1
21. The polar equation r = a, a > 0 represents______ [Winter 2015]
(a) line (b) rectangle (c) circle (d) parabola
22. The values of x for which the graphs of y = x and y2 = 4x intersect are
[Winter 2013]
(a) 4, 4 (b) – 4, 4 (c) 0, 4 (d) 0, – 4
23. The tangent to the curve x = t2 – 1, y = t2 – t parallel to the x-axis is at t = ____
[Summer 2014]
1 1 1
(a) 0 (b) (c) (d) -
3 2 3
2 x2
24. The vertical asymptote of the curve y - is [Summer 2014]
x2 - 1
(a) x = 1 (b) x = –1 (c) x = ±1 (d) y = 2
Multiple Choice Questions 3.45

25. The equation (a – x)y2 = x2(a + x) is symmetric about [Winter 2014]


(a) x-axis (b) y-axis (c) both x and y (d) y = x
26. The tangent at the origin to the curve y2(a + x ) = x2(3a – x) is [Winter 2014]
(a) ± 3x (b) 1 (c) ± 2x (d) ± 2x
2 2 2
27. The parametric equation of x 3 + y 3 = a 3 is [Winter 2014]
(a) x = a cosq, y = a sinq (b) x = a3cos q, y = a3 sinq
(c) x = a cos3q, y = a sin3q (d) x = 1, y = 0
28. The slope of the tangent to the curve y = x ex at (0,0) is [Winter 2014]
(a) 0 (b) 1 (c) –1 (d) 4
x3 x2
29. The point of inflection of - - 2 x + 14 is [Winter 2014]
3 2
1 1
(a) x = (b) x = - (c) x = –1 (d) x = –2
2 2
1
30. A curve which passes through the origin and has the slope - is given by
3 [Winter 2014]

(a) x + 3y = 1 (b) x + 3y = 0 (c) x – 3y = 0 (d) x – 3y = 1


31. The curve y = x2 + 3 is symmetric w.r.t. [Winter 2014]
(a) x-axis (b) y-axis (c) origin (d) both the axes
32. The curve r = a cos q is a [Winter 2014]
(a) line (b) lemniscate (c) circle (d) cardiod
33. The curve z = x2 + y2 is a [Winter 2014]
(a) cone (b) paraboloid (c) sphere (d) circle
34. The number of leaves in the curve r = sin 5q is [Summer 2015]
(a) 6 (b) 2 (c) 3 (d) 5
35. The values of x at which the curve y = x and y = 4x – x2 intersect each other are
[Summer 2015]
(a) 0, 4 (b) – 4, 4 (c) 0, 3 (d) – 3, 3
36. The slope of the tangent line of the curve x + y = xy at the point (2, 2) is
[Summer 2015]
(a) –1 (b) –2 (c) –3 (d) –4
37. The number of places the curve x = cos 3t, y = sin t cross is the x-axis is
[Summer 2015]
(a) 2 (b) 1 (c) 3 (d) 0
38. The parameter in the equation x = t2, y = t4 is eliminated by the equation
[Summer 2015]
(a) y = x2 for x ≥ 0 (b) y = x for x ≥ 0
(c) y = 2x2 for x ≥ 0 (d) none of these
39 The curve r = cos 2q has _____ petals. [Summer 2016]
(a) 2 (b) 4 (c) 3 (d) none of these
3.46 Chapter 3 Curve Sketching

x2
40. Asymptotes parallel to y-axis of the curve y = is the line [Winter 2016]
x-3
(a) x = 0 (b) y = 3 (c) x = 3 (d) does not exist

41. The curve x3y + y3x = 3 is symmetric about the [Winter 2016]
(a) x-axis (b) y-axis (c) origin (d) line y = x
3x2
42. Asymptotes parallel to y-axis of the curve y = is the line
x-2
[Summer 2017]

(a) x = 0 (b) y = 0 (c) x = 2 (d) y = 2

43. Curve y2(a + x) = x2(b – x) is symmetric about [Summer 2017]


(a) x-axis (b) y-axis (c) line x = b (d) line x = a

dy
44. The curve increases strictly in the interval in which  [Summer 2017]
dx
(a) < 0 (b) > 0 (c) = 0 (d) none of these

answers

1.(b) 2.(b) 3.(a) 4.(c) 5.(a) 6.(c) 7.(a) 8.(a) 9.(b) 10.(c)
11.(c) 12.(c) 13.(b) 14.(b) 15.(b) 16.(b) 17.(a) 18.(a) 19.(a) 20.(a)
21.(c) 22.(c) 23.(c) 24.(c) 25.(a) 26.(a) 27.(c) 28.(b) 29.(a) 30.(b)
31.(b) 32.(c) 33.(b) 34. (d) 35.(c) 36.(a) 37.(a) 38.(a) 39.(a) 40.(c)
41.(d) 42.(c) 43.(a) 44.(b)
Unit 3
Indeterminate Forms
&
Improper Integrals
CHAPTER
4
Indeterminate
Forms
4.1 INTRODUCTION
We have studied certain rules to evaluate the limits. But some limits cannot be evalu-
ated by using these rules. These limits are known as indeterminate forms. There are
seven types of indeterminate forms:

0 ∞
(i) (ii) ∞ (iii) 0 × ∞ (iv) ∞ − ∞
0
(v) 1∞ (vi) 0° (vii) ∞°

These limits can be evaluated by using L’Hospital’s rule.

4.2 L’HOSPITAL’S RULE

Statement If f (x) and g (x) are two functions of x which can be expanded by Taylor’s
series in the neighbourhood of x = a and if lim f ( x) = f (a ) = 0, lim
lim gg(( x) = gg((a ) = 0,
x→a x→a
then

f ( x) f ′ ( x)
lim = lim
x→a g ( x) x → a g ′ ( x)

Proof Let x=a+h

f ( x) f ( a + h)
lim = lim
x→a g ( x) h → 0 g ( a + h)
4.2 Chapter 4 Indeterminate Forms

h2
f (a ) + hf ′ (a ) + f ″ (a ) + .......
= lim 2! [By Taylor’s theorem]
h→ 0 2
h
g (a ) + hg ′ (a ) + g ″ (a ) + ..............
2!
h2
hf ′ ((a ) + f ″ (a ) + .......
= lim 2! [∵ f (a ) = 00,, g (a ) = 0]
h→ 0 2
h
hg ′ ((a ) + g ″ (a ) + .......
2!
h
f ′ (a ) + f ″ (a ) + .......
= lim 2!
h→ 0 h
g ′ (a ) + g ″ (a ) + ........
......
2!
f ′ (a)
=
g ′(a)
f ′ ( x)
= lim , provided g ′ (a ) ≠ 0.
x → a g ′ ( x)

Standard Limits
The following standard limits can be used to solve the problems:

sin x tan x
(i) lim =1 (ii) lim =1
x→0 x x→0 x

ax −1 ex − 1
(iii) lim
m = llog a (iv) lim =1
x→0 x x→0 x

1 x
 1
m(1 + x) x = e
(v) lim( (vi) lim 1 +  = e
x→0 x →∞  x

sin −1 x sin h x
(vii) lim =1 (vii) lim =1
x→0 x→0 x
x

tan −1 x
(ix) lim =1
x→0 x

0
4.3 TYPE 1 : FORM
0
Problems under this type are solved by using L’Hospital’s rule considering the fact
that
0
4.3 Type 1: Form 4.3
0

f ( x) f ′ ( x)
lim = lim
lim lim f ( x) = 0 and lim g ( x) = 0.
if lim
x→ a g ( x ) x→ a g ′ ( x )
g( x→ a x→ a

Example 1
((11 + x) n − 1
Prove that lim = n.
x→0 x
Solution
(1 + x) n − 1 0 
Let l = lim  0 form 
x→0 x
n(((1 + x) n −1
n
= lim [[A
Applying L’Hospital’s rule]
x→0 1
=n

Example 2
xe x - log
log (1 + x )
Evaluate lim .
x Æ0 x2
Solution
xe x − log (1 + x) 0 
Let l = lim  0 form 
x→0 x2
1
e x + xe x −
1 + x 0 
= lim  0 form 
x→0 2x
[Applying L’Hospital’s rule]
1
e x + e x + xe x +
( + x) 2
(1
= lim [Applying L’Hospital’s rule]
x→0 2
3
=
2

Example 3
log x − (x
x log ( x − 1)
Evaluate lim .
x →1 ( x − 1) log
log x
4.4 Chapter 4 Indeterminate Forms

Solution
x log x - ( x - 1) È0 ˘
Let l = lim Í 0 form ˙
x Æ1 ( x - 1) log x Î ˚

1
x◊+ log x - 1
= lim x Applying L’Hospital’s rule]
[[Applying
x Æ1 1
logg x + ( x - 1) ◊
x
log x È0 ˘
= llim
im Í 0 form ˙
x Æ1 1 Î ˚
log x + 1 -
x

1
= lim x Applying L’Hospital’s rule]
[[Applying
x Æ1 1 1
+
x x2
1
=
2

Example 4
e x - e - x - 2 log(1 + x )
Evaluate lim .
x Æ0 x sin
sin x
Solution
e x - e - x - 2 log(1 + x ) È0 ˘
Let l = lim Í 0 form ˙
x Æ0 x sin
sin x Î ˚
1
e x + e- x - 2 ◊
1+ x È0 ˘
= lim Í 0 form ˙ L’Hospiita
[ Applying L’ tal’
l s rule]
x Æ0 n x + x cos x
sin Î ˚
2
e x - e- x +
( + x )2
(1
= lim [ Applying L’Hospita
ospitall’s rule]
Hospital
H
s x + cos
x Æ 0 cos cos x - x sin
sin x

2
=
2
=1
0
4.3 Type 1: Form 4.5
0

Example 5
e x + e- x - x2 - 2
Evaluate lim . [Winter 2016; Summer 2014]
x Æ0 sin 2 x - x 2
Solution
e x + e- x - x2 - 2 È0 ˘
Let l = lim 2 2 Í 0 form ˙
x Æ0 sin x - x Î ˚
e - e- x - 2 x
x
= lim [ Applying L’Hospital’s rule]
x Æ 0 2 sin x cos x - 2 x

e x + e- x - 2
= lim [ Applying L’Hospital’s rule]
x Æ0 2( - sin x )sin x + 2 cos2 x - 2
e x + e- x - 2
= lim
x Æ0 -2 sin 2 x + 2 cos2 x - 2
e x + e- x - 2
= lim
x Æ 0 2 cos 2 x - 2

e x - e- x
= lim [ Applying L’Hospital’s rule]
x Æ 0 ( -2)2 sin 2 x

e x - e- x
= lim
x Æ 0 - 4 sin 2 x

e x + e- x
= lim [Applying L’Hospital’ss rule]
x Æ 0 -8 cos 2 x
2
=
-8
1
=-
4

Example 6
2x - 1
Evaluate lim 1
.
x Æ0
(1 + x) 2 -1
Solution
2x - 1 È0 ˘
Let l = lim 1 Í 0 form ˙
x Æ0 Î ˚
(1 + x) 2 -1
4.6 Chapter 4 Indeterminate Forms

2 x log
log 2
= lim 1
[ Applying L’Hospital’s rule]
x Æ0 -
1
(1 + x ) 2
2
= 2 log
log 2

Example 7

Evaluate lim
(
log 1 + kx 2 ).
x→0 1 − cos x
Solution

l = lim
(
log 1 + kx 2 ) 0 
Let
x→0 1 − cos x  0 form 
1
⋅ 2kx
= lim 1 + kx
2
[[A
Applying L’Hospital’s rule]
x→0 sin x
1
= 2k lim
( )
sin x
x→0
1 + kx 2 ⋅
x
 sin x 
=22kk ∵ lix →m0 x = 1

Example 8
log (1 + x 3 )
Evaluate lim .
x Æ0 sin 3 x
Solution
log (1 + x 3 ) È0 ˘
Let l = lim 3 Í 0 form ˙
x Æ0 sin x Î ˚
1
⋅ 3x 2 [Applying L’Hospital’s rule]
= lim 1 + x 3

n 2 x ccos
x → 0 3 sin os x
2
Ê x ˆ 1
= lim Á ˜
x Æ 0 Ë sin x ¯ (1 + x ) cos
3
cos x
È x ˘
=1 Í xliÆm0 sin x = 1˙
Î ˚
0
4.3 Type 1: Form 4.7
0

Example 9
x − xx
Evaluate lim
x →1 1 + log
.
log x − x
Solution
x − xx 0 
Let l = lim  0 form 
x →1 1 + log x − x

x − e x log
log x
= lim
x →1 1 + log x − x

1 − e x lologg x (1 + log x)
= lim [Appl
A ying
ngg L’’Hospita
Hospitall’s rule]
x →1 1
−1
x

og x  1 
−e x log x (1 + llog
og x) 2 − e x llog  
x
= lim [Appl
A ying L’Hospiita
tal’s rule]
x →1 1
− 2
x
=2

Example 10
xy - yx
Evaluate lim .
xÆ y xx - yy
Solution
xy − yx 0 
Let l = lim  0 form 
x→ y x x − y y

yx y −1 − y x log
log y
= lim [Applying L’Hospital’s rule]
x→ y x (1 + llog
x
og x) − 0
y y − y y log y
=
y y (1 + log yy)
1 − log y
=
1 + log y

Example 11
sin ( x cos
cos x )
Evaluate lim .
p cos( x sin
sin x )

2
4.8 Chapter 4 Indeterminate Forms

Solution
sin ( x cos x) 0 
Let l = lim  0 form 
x→
p cos ( x sin x)
2

cos ( x cos x)(cos x − x sinn x)


= limp [Applying L’Hospital’s rule]
x→ − sin ( x sin x)(sin x + x coss x)
2

p
=
2

Example 12
cos 2 π x
Evaluate lim . [Winter 2015]
x→
1 e 2 x − 2 xe
2

Solution
cos 2 p x 0 
Let l = lim1  0 form 
x→
2
e 2 x − 2 xxe

2 coss p x ( −p sin
sin p x)
= lim [Applying L’Hospital’s rule]
x→
1 2e − 2e
2x
2

−p sin
sin 2p x 0 
= lim1  0 form 
x→ 2( e 2 x − e )
2

−2p 2 cos
cos 2p x
= lim [Applying L’Hospital’s rule]
x→
1 2 ⋅ 2e 2 x
2

p2
=
2e

Example 13
cos (θ − α ) 1 2
1 − cos
Prove that θlim = secc α .
→α (sin θ − sin
sin α ) 2 2

Solution
1 - cos(q - a ) È0 ˘
Let l = lim 2 Í 0 form ˙
q Æa (sin q - sin
sin a ) Î ˚
0
4.3 Type 1: Form 4.9
0

sin (q − a )
= lim [Applying L’Hospital’s rule]
q →a 2 (sin q − sin
sin a ) cos q
sin (q − a ) 0 
= lim  0 form 
q →a q − 2 sin
(sin 22q sin a cos q )
cos (q − a )
= lim [Applying L’Hospital’s rule]
q →acos 2q + 2 ssin
2 cos in a sin q
cos 0
=
cos 2a + 2 ssin
2 cos in a sin a
1
=
2(1 − 2 sin 2 a ) + 2 sin 2 a
1
=
2 − 2 sin 2 a
1
=
2 cos 2 a
1
= sec 2 a
2

Example 14
sin x − 7 ssin
5 sin in 2 x + 3 sin 3 x
Evaluate lim .
x→0 tan x − x
Solution
sin x − 7 sin 2 x + 3 sin 3 x
5 sin 0 
Let l = lim  0 form 
x→0 tan x − x
cos x − 14 cos 2 x + 9 cos
5 cos cos 3 x
= lim [Appl
A ying L’Hospita
Hospital
Hospitall’s rule]
x→0 sec x − 1
2

−5 sin
sin x + 28 sin
sin 2 x − 27 sin 3 x 0 
= lim  0 form  [Appllyying L’’Hospita
Hospitall’s rule]
x→0 2 secc 2 x tan
tan x
sin x
sin sin 2 x sin 3 x
−5
− 5 + 56 ⋅ − 81
= lim x 2 x 3x
x→0 tan x
2 sec x ⋅
2

x
−5 + 56 − 81  sin nx tan x 
= ∵ lim = 1 and lim = 1
2 →0

xx→ 0 nx x → 0 x 
= −15
4.10 Chapter 4 Indeterminate Forms

Example 15
3
x2
2 x − 2e + 2 cos ( x ) + sin
2
sin 3 x 2
Evaluate lim .
x→0 x2

Solution
3
2
2 x 2 − 2e x + 2 cos
cos x 2 + ssin
in 3 x 0 
Let l = lim  0 form 
x→0 x2
2
3
3 1
4 x − 2e x (2 x) − 2 sin
sin x 2  x 2  + 3 sin 2 x cos x
2  0 
= lim  0 form 
x→0 2x
[Applying L’Hospital’s rule]
2 2  3
3 1 1 3

4 − 4 ((ee x + xe x ⋅ 2 x) − 3  x cos
cos x 2 ⋅ x 2 + sin x 2  + 6 sin x cos
cos 2 x − 3 sin
sin 3 x
 2 2 x 
= lim
x→0 2
[Applying L’Hospital’s rule]
3
2
sin x x
4 − 4 − lim ⋅
x→0 2 x x
=
2
3
1 1 sin x 2  3

= − ⋅ lim ⋅x sin x 2
2 2 x→0 3
2
∵lim = 1
x  x→0 3 
=0  x2 

Example 16
A  sin kx k 
Evaluate lim − .
x→0 x 2  sin lx l 
Solution
A  sin kx k 
Let l = lim −
x→0 x 2  sin lx l 
l sin kkxx − k sin lx 0 
= A lim  0 form 
x→0 lx 2 sin lx
A l sin kkxx − k ssiin lx
= lim
l x → 0 sin lx
x2 ⋅ lx
lx
0
4.3 Type 1: Form 4.11
0

A l sin kkxx − k sin lx 0   sin lx 


= lim
2 xx→  0 form  ∵ lim = 1
l →0
→ 0 x3 x→0 lx 
A lk coss kx − kl cos lx 0 
= lim
2 x→0  0 form  [Appl
A ing L’Hospittaal’
l s rule]
l 3x 2
A −lk
− lk 2 sin kx + kl 2 sin lx
= lim [[A
Applying L’Hospital’s rule]
l 2 x→0 6x
A  2 sin kx sin lx 
= 2 lim  −lk ⋅ ⋅ k + kl
kl 2 ⋅ ⋅l
6l x → 0
 kx lx 
 sin lx 
=
A
6l 2
(
−lk 3 + kl 3 ) ∵ lim
→00

xx→
sin kx
kx
= 1 = lim
x→0 lx 

=
Ak 2
6l
(
l − k2 )

Example 17
1
x 2 tan
tan x
Evaluate lim 3
.
x→0
(e − 1) x 2

Solution
x tan x È0 ˘
Let l = lim 3 Í 0 form ˙
x Æ0
x Î ˚
(e - 1) 2

x x tan x
= lim 3

x→0 x
(e x − 1) 2

x x  tan x 
= lim ⋅ lim  
x→
→00 3 x→0  x 
(e x − 1) 2

3
 x 2  tan x 
= lim  x  ∵ lim = 1
x → 0  e − 1 x→0 x 
x 1
Now, lim
m = lim [Applying L’Hospital’s rule]
x→
→00 e x − 1 x→0 e x
=1
3
3
\  x 2
lim
m  = (1
x → 0  e x − 1
1)) 2 = 1

Hence, l=1
4.12 Chapter 4 Indeterminate Forms

Example 18
x
logg sec x ccos
Evaluate lim 2.
x →0 log
g x ccos x
sec
2

Solution
x
logg sec x cos
Let l = lim 2
x → 0 log
g x cos x
sec
2

x x
log cos logg ssec
= lim 2⋅ 2
x → 0 log sec x log cos x

x  − log cos x 
log cos  
= lim 2 ⋅ 2
x → 0 ( − log c
coos x) log cos x
2
 x
log cos
 2  0 
= lim   0 form 
x→0 log cos x 
 

1  1 x
⋅ − sin 
x x  2 2 
log cos cos
lim 2 = lim 2
Now, → 0 log 1 [Applying L’Hospital’s rule]
xx→ cos x x→0
log cos ((− sin x)
cos x
x
tan
= lim 2
x → 0 2 tan x

 x
tan
1 2 ⋅  x 
= lim 
x→0 4 x   tan x 
 
2 
1
=
4
0
4.3 Type 1: Form 4.13
0

2
 x
log cos 2
 2  =  1 = 1  tan x 
∴ lim    ∵ lim = 1
x→0 log cos x  4 16 x→0 x 
 

1
Hence, l=
16

Example 19
1
(1 + x) x − e e
Prove that lim =− .
x→0 x 2

Solution
1
(1 + x) x − e 0 
Let l = lim  0 form 
x→0 x
1
log(1+ x )
ex −e
= lim [Applying L’Hospital’s rule]
x→0 x
1
log(1+ x )  1 1 
ex  − x 2 log (1 + x) + x ((1 + x) 
  0 
= lim  0 form 
x→0 1
[Applying L’Hospital’s rule]

= lim (1 + x) x llim
1
[ − log (1 + x) ⋅ (1 + x) + x ] 0 
x→0 x→0 x 2 (1 + x)  0 form 

 − log (1 + x)
x) − 1 + 1 0  [Applying L’Hospital’s rule]
= e lim    0 form 
x→0
 2 x + 3x 2

 1 
  − 1 + x  
= e lim  
x→0
 2 + 6x 
 
e
=−
2
4.14 Chapter 4 Indeterminate Forms

Example 20

( )
2n
1− x −1
Prove that lim = 2− n .
x→0
(1 − cos
cos x )
n

Solution

( ) ⋅( )
2n 2n
1− x −1 1− x +1
Let l = lim
cos x ) (
(1 − cos 1 − x + 1)
x→0 n 2n

((11 − x − 1) 2 n
= lim n
 x
( )
x→0 2n
 2 sin
2
 1− x +1
2
( − x) 2 n 2n
= lim 2n

2n
 x
( )
x→0 2n
2n  sin  1− x +1
 2
2n
 x 
  2n
= lim  2  ∵ ( − { } = x2n 
n
−xx) 2 n = ( − x) 2
( )  
2n
x→0 x 1− x +1
 sin 
2
1
=
2n

Example 21
x 2 + 2 cos
cos x − 2
Evaluate lim .
x→0 x sin
sin x
Solution
x 2 + 2 cos x − 2 0 
Let l = lim  0 form 
x→0 x sin x
2 x − 2 sin
sin x 0 
= lim  0 form  [Appl
A ying L’Hospita
Hospital
Hospitall’s rrule
ule]
x→0 n x + x cos x
sin
2 − 2 cos x
= lim [Appl
A ying L’Hospita
Hospital
Hospitall’s rullee]
x → 0 coss x + cos
cos x − x sin
sin x
=0
0
4.3 Type 1: Form 4.15
0

Example 22
e x sin x − x − x 2
Evaluate lim .
x→0 x 2 + x log(
log(1 − x)
Solution
e x sin x − x − x 2 0 
Let l = lim  0 form 
x → 0 x 2 + x log(1 − x )

sin x + e x cos x − 1 − 2 x
e x sin 0 
= lim  0 form  [Appl
A yyiing L’’Hospita
Hospitall’s rule]
x→0 x
2 x + llog(
og(1 − x) −
1− x
e (sin x + cos
x
cos x) + e x (cos x − sin
sin x) − 2
= lim [[A
Applying L’Hospital’s rule]
x→0 1 1 x
2− − −
1 − x 1 − x (1 − x) 2
cos x − 1)
22(((ee x cos 0 
= lim  0 form 
x→0  1  x
2 1 − −
 1 − x  (1 ( − x) 2
cos x − e x sin
2(e x cos sin x)
= lim [Appl
A ying L’Hospita
Hospital
Hospitall’s rule]
x→0  1  1 2x
2 − 2− −
 (1 − x)  (1 − x) (1 − x)3
2

2
=−
3

Example 23
sinn 2 x + p ssin x
If lim is finite, find the value of p and hence, the limit.
x→0 x3
Solution

n 2 x + p sin x
sin
Let l = lim , where l is finite
x→0 x3
n 2 x + p sin x
sin 0 
l = lim  0 form 
x→0 x3
cos 2 x + p cos x
2 cos
= lim [[A
Applying L’Hospital’s rule]
x→0 3x 2
2+ p
=
0
4.16 Chapter 4 Indeterminate Forms

∴ 2 + p = 0, p = −2
2 cos 2 x − 2 cos x 0 
Thus, l = lim  0 form 
x→0 3x 2
−4 si
sinn 2 x + 2 sin x 0 
= lim  0 form  [Applying L’Hospital’s rule]
x→0 6x
−88 co
− coss 2 x + 2 cos x
= lim [Applying L’Hospital’s rule]
x→0 6
= −1
Hence, p = −2 and l = −1

Example 24
m [ x −−33 sin
Find the values of a and b, if lim sin x + ax −2 + b] = 0 .
x→0

Solution
sin x + a
axx + b
bxx 3 0 
0 = lim 3  0 form 
x→0 x
cos x + a + 3bx 2
= lim [Appl
A ying L’Hospital
Hospital’s rule]
x→0 3x 2
1+ a
=
0
1+ a = 0
a = −1
cos x − 1 + 3b
bxx 2 0 
Thus, 0 = lim  0 form 
x→0 3x 2
− sin x + 6b
bxx 0 
= lim  0 form  [Applying L’Hospital’s rule]
x→0 6x
− cos x + 6b
= lim [Appl
A yinngg L’Hospital
Hospital’s rule]
x→0 6
−1 + 6b
=
6
1
b=
6

1
Hence, a = −1, b =
6
0
4.3 Type 1: Form 4.17
0

Example 25
sin 3 x + aaxx + bbxx3
Find the values of a and b, such that lim =0
x→0 x3
Solution
sin 3 x + ax + bx 3 0 
0 = lim  0 form 
x→0 x3
cos 3 x + a + 3bx 2
3 cos
= lim [Applying L’Hospital’s rule]
x→0 3x 2
3+ a
=
0
\ a + 3 = 0, a = −3

cos 3 x − 3 + 3bx
33co bx 2 0 
Thus, 0 = lim 2  0 form 
x→0 3x
−999si
− sin 3 x + 6bx
bx
= lim [Ap
Applying L’Hospita
Appl Hospital
Hospitall’s rule]
x→0 6x
 9  sin 3 x  
= lim
lim  −   + b 
x→0
 2 3x  

9  sin 3 x 
= − +b ∵ lix →m0 3 x = 1
2
9
b=
2
9
Hence, a = −3, b =
2

Example 26
x(1 + a ccos
os x) − b ssin x
Find the values of a and b, if lim = 1.
x→0 x3
Solution
x(1 + a cos x) − b sin x 0 
1 = lim  0 form 
x→0 x3
1 + a cos x) + x( − a sin x) − b cos x
(1+
= lim [Appyling L’Hospital’s rule]
x→0 3x 2
4.18 Chapter 4 Indeterminate Forms

1 + (a − b) coss x − ax ssin
in x
= lim
x→0 3x 2
1+ a − b
=
0
\ 1+ a − b = 0
a − b = −1 ...(1)

1 − cos x − ax sinsin x 0 
Thus, 1 = lim 2  0 form 
x→0 3x
sinn x − a (sin x + x cos x)
= lim [[A
Applying L’Hospital’s rule]
x→0 6x
 sin
n x a sin sin x a 
= lim
lim  − ⋅ − cos x 
x→0
 6x 6 x 6 
1 a a  sin x 
= − − ∵ lix →m0 x = 1
6 6 6
1 − 2a
=
6
5
a=−
2
Substituting in Eq. (1),
3
b=−
2
5 3
Hence, a = − ,b= −
2 2

Example 27
a sinh x + b sin
sin x 5
Find the values of a and b, if lim 3
= .
x→0 x 3
Solution
5 a sinh x + b sin
sin x 0 
= lim 3  0 form 
3 x → 0 x
a cosh x + b cos
cos x
= lim [[A
Applying L’Hospital’s rule]
x→0 3x 2
a+b
=
0
0
4.3 Type 1: Form 4.19
0

\ a+b = 0
b = −a ...(1)
5 a cosh x − a cos
cos x 0 
Thus, = lim  0 form 
3 x→0 3x 2
a sinh x + a sin
sin x
= lim [Applying L’’Hospita
Hospitall’s rule]
x→0 6x
a  sinh x sin x 
= lim  + 
x→0 6  x x 
a  sinh x sin x 
= ∵ lixx→
m = 1 and lim = 1
3 →00
→ x x→0 x 
a
=
3
a=5
b = −5 [Using Eq. (1)]

Example 28
sin 2 x − b tan
a sin tan x
Find the values of a and b, such that lim 3
= 1.
x→0 x
Solution
sin 2 x − b tan x
a sin 0 
1 = lim  0 form 
x→0 x3
cos 2 x − b sec
2a cos sec 2 x
= lim 2
[[A
Applying L’Hospital’s rule]
x→0 3x
2a − b
=
0
\ a −b = 0
22a
b = 2a

coss 2 x − 2a sec
2a co sec 2 x 0 
Thus, 1 = lim 2  0 form 
x→0 3x
− 4a si
sinn 2 x − 4a sec x ⋅ ssec 0
ec x tan x 
= lim  0 form  [Applying L’Hospital’s rule]
x→0 6x
 4a  sin 2 x  4a  tan x  
= lim −    + sec 2 x 
x→0
3  2x 6  x  
4.20 Chapter 4 Indeterminate Forms

 4a 2a 
= − + 
 3 3
6a
=−
3
= −2a

1
Hence, a = − , b = −1
2

Example 29
sin 2 x + b log
a sin log cos
cos x 1
Find the values of a and b such that lim 4
=− .
x→0 x 2
Solution
1 sin 2 x + b log
a sin log cos
cos x 0 
− = lim  0 form 
2 x→0 x4
1
a ⋅ 2 sin x cos x + b ⋅ ( − sin x)
= lim cos x [Applying L’Hospital’s rule]
x→0 4 x3
sin 2 x − b ttan
a sin an x 0 
= lim  0 form 
x→0 4 x3
cos 2 x − b ssec
2a cos ec 2 x
= lim [Applying L’Hospital’s rule]
x→0 12 x 2
2a − b
=
0
∴ 22aa − b = 0
b = 2a ...(1)

cos 2 x − 2a sec
sec 2 x 0 
Thus, − 1 = lim 2a cos
2 x→0 12 x 2  0 form 
−44a si
− sinn 2 x − 4a
a sec 2 x tan x
= lim
x→0 24 x
 − a sin 2 x a tan x 
= lim  − secsec 2 x ⋅ 
x→0  3 2x 6 x 
0
4.3 Type 1: Form 4.21
0

a a  sin 2 x tan x 
=− − ∵ lixx→
m = 1 and lim = 1
3 6 →00
→ 2x x→0 x 
a
=−
2
a =1
a=2

Substituting in Eq. (1),


b=2
Hence, a = 1, b = 2

Example 30
sin x − sin 2 x
a sin
Find a and b, if lim = b.
x→0 tan 3 x
Solution
a sin x − sin 2 x
b = lim
x→0 tan 3 x
a sin x − sin 2 x
= lim 3
x→0
 tan x 
3
x 
 x 
sin x − sin 2 x
a sin 0   tan x 
= 1
= lim  0 form  ∵ lim
x→0 x3 x→0 x 
cos x − 2 cos
a cos cos 2 x
= lim 2
[Applying L’Hospital’s rule]
x→0 3x
a−2
=
0
\ a−2= 0
a=2

Thus, cos x − 2 cos 2 x


2 cos 0 
b = lim  0 form 
x→0 3x 2
−2 si
sinn x + 4 sin 2 x
= lim [Applying L’Hospita
Hospital
Hospitall’s rule]
x→0 6x
 2  sin n x  4  ssin 2 x  
= lim
lim  −  +  
x→0
 6  x  3  2x  
4.22 Chapter 4 Indeterminate Forms

2 4
=− +
6 3
 sin x 
=1 ∵ lix →m0 x = 1

Hence, a = 2, b =1

Example 31
cos x + ce − x
ae x − b cos
Find a, b, c, if lim = 2.
x→0 x sin
sin x
Solution
coss x + ce − x
ae x − b co
2 = lim
x→0 x sin
sin x
coss x + ce − x
ae x − b co
= lim
x→0  sin x 
x⋅ x
 x 
coss x + ce − x
ae x − b co  sin x 
= lim ∵ lim = 1
x→0 x2 x→0 x 
a −b+c
=
0

∴ a −b+c = 0 ... (1)

coss x + ce − x
ae x − b co 0 
Thus, 2 = lim  0 form 
x→0 x2
ae x + b sin x − ce − x
= lim [Applying L’Hospital’s rule]
x→0 2x
a−c
= [Applying L’Hospital’s rule]
0
∴ a − c = 0, a = c ... (2)

ae x + b sin x − ae − x 0 
Thus, 2 = lim  0 form 
x→0 2x
coss x + ae − x
ae x + b co
= lim [Applying L’Hospital’s rule]
x→0 2
a+b+a
=
2
0
4.3 Type 1: Form 4.23
0

∴ 2a + b = 4 ... (3)
From Eqs (1) and (2),
2a − b = 0
2a ... (4)
Solving Eqs (3) and (4),
a = 1, b = 2, c =1

EXERCISE 4.1
log a − a 2 log
x 2 log log x 1
1. Prove that lim = log
log a − .
x →a x −a
2 2
2
log(1 − x 2 )
2. Prove that lim = 2.
x → 0 log cos x

1 − tan x
3. Prove that limπ = 2.
x→
4
1 − 2 sin x

e x − 1 + 2x
4. Prove that lim = 1.
x → 0 log(1 + x 2 )

tan−1 a 2 − x 2
a + x tan
5. Evaluate lim .
x →a
a−x
 tan−1 a 2 − x 2 
Hint : lim(x + a) as x → a, a − x → 0 
 x →a
a2 − x 2 
[Ans.: 2a]
cos x − a + b
a cos bx 2 1
6. Find a and b, if lim 4
= .
x →0 x 12
Ans.: a = 2, b = 1
x (a + b cos
cos x) − c ssin
in x
7. Find the values of a, b and c such that lim = 1.
x →0 x3
[[Ans.: a = 0, b = −3, c = −3]

x ((1 − a cos x)) + b sin x 1


cos x
8. Find the values of a and b such that lim = .
x →0 x3 3
 1 1
 Ans.: a = 2 , b = − 2 
 
ae x − be − x − cx
9. Find the values of a, b and c such that lim = 4.
x →0 x − sin
sin x
[[Ans.: a = 2, b = 2, c = 4]
4.24 Chapter 4 Indeterminate Forms

1− x
e x + loge
10. Evaluate lim e .
x →0 tan x − x  1
 Ans.: − 2 
 
1 − x + log
log x
11. Evaluate lim .
x →1
1−
1 − 2x − x 2
[Ans.: –1]
xe − log(1 + x) 3
x

12. Prove that lim = .


x →0 x2 2

e x − 1 + 2x
13. Prove that lim = 1.
x → 0 log(1 + x 2 )

1 − tan x
14. Prove that lim = 2.
x→
π
4
1 − 2 sin x

3 x − 12 − x 8
15. Prove that lim
m = .
x →3
2 x − 3 19 − 5x 69
alog
log x
−x a
16. Prove that lim = log
g .
x →1 log x e

x − a + x −a 1
17. Prove that lim
m = .
x →a
x −a 2 2
2a
tan x − x 1
18. Prove that lim = .
x →0 x3 3


4.4 TYPE 2: FORM

Problems under this type are also solved by using L’Hospital’s rule considering the
fact that
f ( x) f ′ ( x)
lim = lim
lim lim f ( x) = ∞ and lim g ( x) = ∞.
if lim
x →a g ( x) x →a g ′ ( x) x →a x →a

Example 1
log x
Prove that lim = 0, (n
(n > 0).
x →∞ x n

4.4 Type 2: Form 4.25

Solution
log x ∞ 
Let l = lim  ∞ form 
x →∞ xn
1
m x
= lim [Appl
[Appl
A ying L’Hospita
Hospital
Hospitall’s rule]
x →∞ nx n −1

1 1
= li lim n
n x →∞ x
=0

Example 2
log sin x
Evaluate lim
x →0
.
cot x
Solution
log sin x ∞ 
Let l = lim  ∞ form 
x→0 cot x
1
⋅ cos x
= lim sin x [[A
Applying L’Hospital’s rule]
x → 0 − cosec 2 x

= lim − ( cos x sin x )


x→0

=0

Example 3
 π
log  x − 
 2
Prove that lim = 0.
π tan x
x→
2

Solution
Ê pˆ
log Á x - ˜
Ë 2¯ È• ˘
Let l = lim Í • form ˙

p tan x Î ˚
2

Ê 1 ˆ
Á p˜
ÁË x - ˜¯
2
= lim [Applying L’Hospital’s rule]

p sec 2 x
2
4.26 Chapter 4 Indeterminate Forms

cos2 x È0 ˘
= lim Í 0 form ˙
p p Î ˚

2 x-
2
2 coss x ( - sin
sin x )
= lim [Applying L’Hospital’s rule]

p 1
2
=0

Example 4
log ( x – a )
Prove that lim = 1.
x → a log ( a x – a a )
x→

Solution

log ( x - a ) È• ˘
Let l = lim Í • form ˙
x a
xÆa log (a - a ) Î ˚
1
((xx - a )
= lim [Applying L’Hospital’s rule]
xÆa 1 . x
a log
log a
a - aa
x

1  a x − aa  0 
= lim x . li
lim   0 form 
x → a a log a
x→ x→a  x − a  
x
1 . lim a loglog a
= a
[[A
Applying L’Hospital’s rule to second term]
a log
log a x
x→→ a 1
1
= a ⋅ a a log
log a
a lloog a
=1

Example 5
n x = 1.
Prove that lim log x tan
x→0

Solution

Let l = lim log x tan x


x→0

log tan x ∞ 
= lim  ∞ form 
[Change of base property]
x→0 log x

4.4 Type 2: Form 4.27

1 . 2
sec x
= lim tan x [Applying L’Hospital’s rule]
x→0 1
x

= lim
x .
limsec 2 x  tan x 
∵ lim = 1
→0

xx→ 0 tan x x → 0 x→0 x 
=1

Example 6
Evaluate lim log tan x tan 2 x .
x →0

Solution
Let l = lim log tatann x tan 2 x
x→0

log tan 2 x ∞ 
= lim  ∞ form 
x→0
log tan x
1
⋅ 2 sec
sec 2 2 x
= lim tan 2 x [[A
Applying L’Hospital’s rule]
x→0 1
⋅ sec 2 x
tan x
tan x
⋅ sec 2 2 x
= lim x
x → 0 tan 2 x
⋅ sec 2 x
2x
 tan x
tan 
=1 ∵ lix →m0 x = 1

Example 7
sin h−1 x
Prove that lim = 1.
x →∞ cos h −1 x

Solution
sin h −1 x
Let l = lim
x →∞ cos h −1 x

= lim
(
log x + x 2 + 1 ) ∞ 
 ∞ form 
x →∞
log ( x + x2 − 1)
4.28 Chapter 4 Indeterminate Forms

1  1 
⋅ 1 + ⋅ 2 x

= lim
( )
x + x2 + 1  2 x + 1
2

[Applying L’Hospital’s rule]
x →∞
1  1 
⋅ 1 + ⋅ 2 x
(x + x2 − 1 )  2 x2 − 1 

x2 +
+11+ x

= lim
(x + x2 + 1 ) x2 + 1
x →∞
x2 −
−11+ x
(x + x2 − 1 ) x2 − 1

x2 − 1
= lim
x →∞
x2 + 1
1
1−
x2
= lim
x →∞
1
1+ 2
x
=1

Example 8
1 2 3 x
e + e + e + ... + e
x x x x
Prove that lim = e − 1.
x →∞ x
Solution
1 2 3 x
e x + e x + e x + ... + e x
Let l = lim
x →∞ x
  1x
1
e 1 −  e x  
x

    1
= lim 
[Sum of GP]
1

x →∞ x
1− ex
1
e x (e − 1) 1
= lim 1
x →∞ x
ex −1
Putting 1
= y, when x → ∞, y → 0
x
(e −1
− 1)e y y 0 
l = lim  0 form 
y→0 ey −1

4.4 Type 2: Form 4.29

(e −
−11) ( ye y + e y )
= lim [Applying L’Hospital’s rule]
y →0 ey
= e −1

Example 9
xn
Prove that lim kx = 0 .
x →∞ e

Solution
xn ∞ 
Let l = lim
m  ∞ ffor
orm 
x →∞ e kx

nx n −1 ∞ 
= lim  ∞ form  [Applying L’Hospital’s rule]
x →∞ ke kx

n (n − 1))xx n − 2 ∞ 
= lim  ∞ form  [Applying L’Hospital’s rule]
x →∞ k 2 e kx
Applying L’Hospital’s rule (n – 2) times in the above expression,
n (n −
−11) ((nn − 2)...2 ⋅1
l = lim
x →∞ k n e kx
kx

n!
= lim n kx
x →∞ k e kx

=0 ∵ lim e kx = ∞]
[[∵
x →∞

Example 10
12 + 22 + 32 + ... + x 2 1
Prove that lim = .
x →∞ x3 3
Solution
12 + 22 + 32 + ... + x 2
Let l = lim
x →∞ x3
x (x +
+11) (2 x + 1)  n (n + 1) ((22n + 1) 
= lim ∵∑ n =
2

x →∞ 6 x3 6
2 x + 3x 2 + x
3
= lim
x →∞ 6 x3
3 1
2+ + 2
= lim x x
x →∞ 6
4.30 Chapter 4 Indeterminate Forms

2
=
6
1
=
3

Example 11
1
ex
Prove that lim x
= e2 .
x →∞
 1  x 
1 +  
 x 
Solution
ex È• ˘
Let l = lim x Í • form ˙
x Æ•
ÈÊ 1ˆ
x˘ Î ˚
ÍÁ 1 + ˜ ˙
ÍÎ Ë x¯ ˙
˚
ex
= lim
x Æ• x2
Ê 1ˆ
ÁË 1 + x ˜¯
Taking logarithm on both sides,
È x2 ˘
Ê 1 ˆ
log l = lim Í log
log e - log Á 1 + ˜ ˙
x
x Æ• Í Ë x¯ ˙
Î ˚
È Ê 1ˆ˘
= lim Í x - x 2 log Á 1 + ˜ ˙
x Æ• Î Ë x¯˚
È1 Ê 1ˆ˘
= lim x 2 Í - log Á 1 + ˜ ˙
x Æ• Î x Ë x¯˚

1 Ê 1ˆ
- log Á 1 + ˜
x Ë x¯ È0 ˘
= lim Í 0 form ˙
x Æ• 1 Î ˚
x2
1 1 Ê 1ˆ
- 2 - -
x 1 ÁË x 2 ˜¯
1+
= lim x [ Applying L’Hospital’s rule]
x Æ• 2
- 3
x
4.5 Type 3 : 0 × ∞ Form 4.31

1
1−
1
1+
= lim x
x →∞ 2
x
1 1
= lim
2 x →∞ 1
1+
x
1
=
2
1
Hence, l = e2

EXERCISE 4.2
log x
1. Prove that lim = 0.
cot x
x →0

log(1 − x)
2. Prove that lim = 0.
x →1 cot π x

log
gsin x ccos x
3. Prove that lim = 4.
x →0 x
log
g x ccos
sin
2
2
n 2 x = 1.
4. Prove that limlogsin x sin
x →0

log(1 + e 3 x )
5. Prove that lim = 3.
x →∞ x
log x 2
6. Prove that lim = 0. [[Hint: Put x 2 = y ]
x → 0 cot x 2

xm
m x = 0 (m
7. Prove that lim (m > 0) .
x →∞ e

2 3 n
 1  1  1  1
  +   +   +  +  
8. Prove that lim
m e e e e = 0.
n →∞ n
1
9. Prove that limlog x sin 2 x = .
x →0 2

4.5 TYPE 3 : 0 × ∞ FORM


To solve the problems of the type
lim [ f ( x) ⋅ g ( x)], when lim
lim f ( x) = 0, lim
lim g ( x) = ∞ (i.e. 0 × ∞ fo
orrm)
x→ a x→ a x→ a
x→
4.32 Chapter 4 Indeterminate Forms

f ( x) g ( x)
m [ f ( x) ⋅ g ( x) ] = lim
We write lim lim or li
limm ⋅
x→a x→ a 1 x→a 1
g ( x) f ( x)
0 ∞
These new forms are of the type or respectively, which can be solved using
0 ∞
L’Hospital’s rule.

Example 1
Prove that lim log x = 0.
m x log
x→0

Solution
Let l = lim x log x [0 × ∞ for
orm
m]
x→0

log x ∞ 
= lim  ∞ form 
x→0 1
x
1
l = lim x [Applying L’Hospital’s rule]
x→0 1
− 2
x
= lim( − x)
x→0

=0

Example 2
log x = 0 .
Prove that lim sin x log
x→0

Solution
Let l = lim sin x log x [0 × ∞ ffor
orm]
x→0

log x ∞ 
= lim  ∞ form 
x→0 cosec x
1
= lim x [Applying L’Hospital’s rule]
x → 0 − cosec x cot
cot x
tan x
= − lim sin x ⋅
x→0 x
tan x
= − lim sin x ⋅ lim
→0

xx→ 0 x→0 x  tan x 
∵lim = 1
=0
x→0 x 
4.5 Type 3 : 0 × ∞ Form 4.33

Example 3
 a
m 2 x ⋅ ssin  x  = a.
Prove that lim
x →∞ 2 
Solution
 a
Let l = lim 2 x ⋅ sin  x 
x →∞ 2 

1 1
Putting 2x = , t = x ,
t 2
when x → ∞, 2 x → ∞, t→0
sin at
l = lim
t →0 t
a sin at
= lim
t →0 at
 sin x
sin 
= a ⋅1 ∵ lim
lim = 1
x→0 x 
=a

Example 4
 x
Evaluate lim log  2 −  cot( x − a ).
x→ a
x→  a
Solution
 x
Let l = lim log  2 −  cot( x − a ) [0 × ∞ form]
x→a  a
Ê xˆ
log Á 2 - ˜
Ë a¯ È0 ˘
= lim Í 0 form ˙
x Æ a tan( x - a ) Î ˚

1 Ê 1ˆ
-
Ê x ˆ ÁË a ˜¯
ÁË 2 -
a ˜¯
= lim [ Applying L’Hospita
ospitall’s ruulle]
Hospital
H
xÆa secc 2 ( x - a )
1
=-
a
4.34 Chapter 4 Indeterminate Forms

Example 5
 1x 
m a − 1 x = llog a .
Prove that lim [Winter 2013]
x →∞ 
 
Solution
Let l = lim (a − 1) ⋅ x
x →∞
1
x [0 × ∞ form]
orm

= lim
(a − 1)
1
x 0 
x →∞ 1  0 form 
x
1
Putting = t , when x → ∞, t → 0
x
at − 1 0 
l = lim  0 form 
t →0 t
a t log
log a [Applying L’Hospital’s rule]
= lim
t →0 1
= log a

Example 6
Ê p xˆ
Prove that lim tann 2 Á ˜ (1 + sec
sec p x ) = - 2.
x Æ1 Ë 2 ¯
Solution
Ê pxˆ
Let l = lim tan 2 Á ˜ ( 11+ + sec p x ) [• ¥ 0 form ]
x Æ1 Ë 2 ¯
1 + sec p x È0 ˘
= lim Í 0 form ˙
x Æ1 Ê pxˆ Î ˚
cot 2 Á ˜
Ë 2 ¯
p sec
sec p x tan
tan p x
= lim Applying L’Hospital’s rule]
[[Applying
x Æ1 Ê pxˆ Ê pxˆ p
2 ccoot Á ˜ Á - cosec 2
Ë 2 ¯Ë 2 ˜¯ 2
Ê ˆÊ ˆ
Á sec p x ˜ Á tan p x ˜
= - Á lim lim
x Æ1 p x ˜ Á xÆ
Æ1 px ˜
Á cosec 2 ˜Á cot ˜
Ë 2 ¯Ë 2 ¯
Ê ˆ
Á sec p ˜ tan p x È0 ˘
= -Á ˜ lim Í 0 form ˙
p x Æ1 px Î ˚
Á cosec 2 ˜ cot
Ë 2 ¯ 2
4.5 Type 3 : 0 × ∞ Form 4.35

π sec
sec 2 π x
= −( −1) lim [Applying L’Hospital’s rule]
2 π x π
x →1 
 −cosec 
22
sec 2 π
= −2
π
cosec 2
2
= −2

Example 7
a+ x
Evaluate lim
m tan −1 a 2 − x 2 .
⋅ tan
x→ a
x→ a−x

Solution
a+ x
Let l = lim
m ◊ tan -1 a 2 - x 2 [• ¥ 0 form ]
xÆa
xÆ a-x
a+ x a+ x
= lim ◊ tan -1 a 2 - x 2
xÆa a-x a+ x
tan -1 a 2 - x 2
= lim ((aa + x )
xÆa
a2 - x2

tan -1 a 2 - x 2
= lim (a + x ) ◊ lim
xÆa xÆa
a2 - x2
tan -1 a 2 - x 2
= 2 a lim
xÆa
a2 - x2

Let a2 − x2 = α
When a, α → 0
x → a,
tan -1 a
\ l = 2 a ◊ lim
a Æ0 a
È tan -1 a ˘
=2
2aa Í ∵ li m = 1˙
ÎÍ a Æ0 a ˚˙

Example 8
a−x 1
Evaluate lim sin −1 cosec a 2 − x 2 =
a+ x
.
x→ a
x→ 2a
4.36 Chapter 4 Indeterminate Forms

Solution
a−x
Let l = lim sin −1 cosec a 2 − x 2 [[00 × ∞ for
orm
m]
x→a
x→ a+ x
a−x
sin −1
= lim a+ x
x→ a
sin a − x 2
2

Here, applying L’Hospital’s rule will make the expression complicated, so we


rearrange the terms to apply the limits directly.
a−x
Let = α , a2 − x2 = β
a+ x
When x → a, α → 0 and
and β → 0
1
\ l = lim sin −1 α ⋅ lim
α →0 β →0 sin β
  sin −1 α     β  1 
=  lim   ⋅ α  lim
β →0  ⋅ 
  α     sin β  β 
α →0

  sin −1 α  
∵ αlim   =1
1 →0  α 
= lim α ⋅ lim  
α →0 β →0 β   sin β  
and lim  = 1
β →0  β  
 
a−x 1
= lim ⋅ [Resubstituting a and b ]
x→ a a + x a − x2
2

a−x 1
= lim ⋅
x→a a+ x a+ x a−x
1
= lim
x→a a + x

1
=
2a

Example 9
m n
Evaluate lim x (log x) , where m and n are positive integers.
x →0

Solution
Let l = lim x m (log x) n [0 × • form]
x→0

(log x) n ∞ 
= lim  ∞ form 
x→0 1
xm
4.5 Type 3 : 0 × ∞ Form 4.37

1
n (log x) n −1
= lim x [Applying L’Hospital’s rule]
x→0 − m ( x) − m −1
( −1)1 n (log x) n −1 ∞ 
= lim  ∞ form 
x→0 m ( x) − m
1
( − 1)1 n (n − 1) (log x) n − 2 ⋅
= lim x [Appl
A ying L’H
Hospiita
tal’s rule]
x→0 m ( − m)1 ( x) − m −1
( −1) 2 n (n − 1) (log x) n − 2 ∞ 
= lim  ∞ form 
x→0 m 2 ( x) − m
Applying L’Hospital’s rule (n – 2) times in the above expression,
( −1) n nn!(
!(log x)0
l = lim
x→0 m n ( x) − m
( −1) n n ! m
= lim ⋅x
x→0 mn
=0

EXERCISE 4.3
log x = 0.
1. Prove that lim x log
x →0

2. Prove that lim x 2 e − x = 0.


x →∞

m x2 1− e
3. Prove that lim
x →∞
( −
2 gy
x2 ) = 2gy.
log x = 0.
n x log
4. Prove that limtan
x →0

 πx 4
5. Prove that lim(x − 1)tan   = − .
2
x →1  2  π
πx
6. Prove that lim(1 sec π x))ta
lim( + sec )tan
tann = 0.
x →1 2
 πx
( − x) cott   = 0.
7. Prove that limlog((1
x →1  2 
 1+ x 
8. Prove that limlog
x →0
 1 − x  cott x = 2.

2+x
9. Prove that lim
m tan−1 4 − x 2 = 4.
x →2 2−x
4.38 Chapter 4 Indeterminate Forms

4.6 TYPE 4 : ∞ − ∞ FORM


m f ( x) = ∞ and,
m[ f ( x) − g ( x)] , when lim
To evaluate the limits of the type lim[
x→a x→a
x→

0 ∞
m g ( x) = ∞ [i.e., (• – •) form], we reduce the expression in the form of
lim or by
x→a 0 ∞
taking LCM or by rearranging the terms and then L’Hospital’s rule is applied.

Example 1
Prove that lim (cosh −1 x − log
log x) = log 2 .
x →∞

Solution
Let l = lim (cosh −1 x − log
g x) [∞ − ∞ form]
x →∞

( )
= lim log x + x 2 − 1 − log x 
x →∞  
 x + x2 − 1 
= lim log  
x →∞
 x 
 1
= lim log 1 + 1 − 2 
x→∞
 x 
= log 2

Example 2
1 1 
m − 2 log(1 + x)  .
Evaluate lim
x→0 
x x 

Solution
1 1 
Let l = lim  − 2 log(1 + x)  [∞ − ∞ ffor
orm]

x→0 x x 

 x − log(1 + x)  0 
= lim    0 form 
x→0
 x2
 1 
1− 0 
= lim  1 + x   0 form  [Applying L’Hospital’s rule]
x→0  
 2x 
4.6 Type 4 : ∞ − ∞ Form 4.39

 1  2 
m   1 + x  
= lim [Appl
A ying L’’Hospita
Hospitall’s rule]
x→0  
 2 
1
=
2

Example 3
Ê x 1 ˆ
Evaluate lim Á - . [Winter 2016]
x Æ1 Ë x - 1 log x ˜¯
Solution
Ê x 1 ˆ
Let l = lim Á - [• – • form]
x Æ1 Ë x - 1 log x ˜¯
È x log x - x + 1 ˘ È0 ˘
= lim Í ˙ Í 0 form ˙
x Æ1 Î log x ( x - 1) ˚ Î ˚
È ˘
Í 1 + log x - 1 ˙
= lim Í ˙ [Applying L’Hospital’s rule]
x Æ1 x - 1
Í + log x ˙
ÍÎ x ˙˚
log x È0 ˘
= lim Í 0 form ˙
x -1
x Æ1 Î ˚
+ log x
x
1
= lim x
[Applying L’Hospital’s rule]
x Æ1 1 1
2
+
x x
1
=
2

Example 4
 1 1 
Evaluate lim  − .
x→ 2 x − 2
 log( x − 1) 
log(

Solution
 1 1 
Let l = lim  − [∞ − ∞ form]
x→2 x − 2
 log( x − 1) 
4.40 Chapter 4 Indeterminate Forms

log( x − 1) − ( x − 2) 0 
= lim  0 form 
( x − 2) llog(
x→ 2 og( x − 1)
1
−1
= lim x −1 [Applying L’Hospital’s rule]
x→ 2 x − 2
+ log( x − 1)
x −1
− ((xx − 1)
11− 0 
= lim  0 form 
( x − 2) + ( x − 1) log( x − 1)
x→2

−1
= lim [[A
Applying L’Hospital’s rule]
x→2 1
1+ (x −−11) ⋅ + log( x − 1)
( −1
(x − 1)
1
=−
2

Example 5
a x
m  − cot  = 0 .
Prove that lim
x→0  x a
Solution
a x
Let l = lim  − cot  [∞ − ∞ form]
x→0  x a
x
Putting = yy,, when x → 0, y → 0
a

1 
l = lim  − cot y 
y→0  y 
1 1 
= lim  − [∞ − ∞ form]
y→0  y tan y 
 ttaan y − y  0 
= lim   0 form 
y → 0  y tantan y 
 tan y − y  1
= lim  ⋅ lim
→0

yy→ 0 y 2  y → 0  tan y 
 y 

tan y − y 0   tan y 
= lim ⋅1  0 form  ∵ lim = 1
→0

yy→ 0 y2  y→0 y 
sec 2 y − 1 0 
= lim  0 form 
[Applying L’Hospital’s rule]
y→0 2y
4.6 Type 4 : ∞ − ∞ Form 4.41

2 secc y ⋅ ssec
ec y tan y
= lim [Applying L’Hospital’s rule]
y→0 2
=0

Example 6
Èp p ˘ p
Prove that lim Í - p x ˙= .
x Æ0 Î 4 x 2 xx((e + 1) ˚ 4
Solution
p È 1 1 ˘
Let l= lim Í - p x ˙ [• - • form ]
2 x Æ 0
Î 2 x x(e + 1) ˚
p ep x + 1 - 2 È0 ˘
= lim Í 0 form ˙
2 x Æ0 2 x(ep x + 1) Î ˚
p p ep x
= lim [ Applying L’Hospital’s rule]
2 x Æ0 2 È(ep x + 1) + x(p ep x )˘
Î ˚
p2 e0
= ◊ 0
4 (e + 1)
p2
=
8

Example 7
 1   1  1
m  x +  llog
Prove that lim og  x +  − log
g x = .
x →∞  
2   
2  2
Solution
 1   1 
Let l = lim  x +  log  x +  − log x 
x →∞  2   2
[∞ − ∞ form ]

 1
 1 x+
= lim  x +  log  2
x →∞  2  
x 
  1 1  1 
= lim  x log 1 +  + llo
og 1 +  
x →∞
  2x  2  2x  
2x
1  1 1  1
= lim og 1 + 
lim llog + lim log 1 + 
x →∞ 2  2x  2 x →∞  2x 
1 1   1
ax

= log e + log1  x→∞ 
∵ li m 1 +  = e
2 2   ax  
1
=
2
4.42 Chapter 4 Indeterminate Forms

Example 8
Ê 1 1 ˆ
Evaluate lim Á 2 - 2 ˜ . [Winter 2014; Summer 2015]
x Æ0 Ë x sin
si n x¯
Solution
Ê 1 1 ˆ
Let l = lim Á 2 - 2 ˜ [• - • form ]
x Æ0 Ë x sin
si n x¯
Ê sin 2 x - x 2 ˆ È0 ˘
= lim Á 2 2 ˜ Í 0 form ˙
x Æ 0 Ë x sinsin x ¯ Î ˚
Ê sin 2 x - x 2 ˆ È sin 2 x ˘
= lim Á ˜ Í∵ lim = 1˙
x Æ0 Ë x4 ¯ ÎÍ x Æ 0 x 2
˚˙
Ê 2 sin cos x - 2 x ˆ
sin x cos
= lim Á ˜¯ Applying L’Hospital’s rule]
[[Applying
x Æ0 Ë 4 x3
Ê sin 2 x - 2 x ˆ
= lim Á ˜¯
x Æ0 Ë 4 x3
Ê 2 cos 2 x - 2 ˆ
= lim Á ˜¯ [ Applying L’Hospital’s rule]
x Æ0 Ë 12 x 2
Ê - 4 sin 2 x ˆ
= lim Á [ Applying L’Hospital’s rule]
x Æ0 Ë 24 x ˜¯
Ê -8 cos 2 x ˆ
= lim Á [ Applying L’Hospital’s rule]
x Æ0 Ë 24 ˜¯
-8
=
24
1
=-
3

Example 9
 1 
m  2 − ccot 2 x .
Evaluate lim
x→0  x 

Solution
Ê 1 ˆ
Let l = lim Á 2 - cot 2 x˜ [• - • form ]
x Æ0 Ë x ¯
4.6 Type 4 : ∞ − ∞ Form 4.43

Ê 1 1 ˆ
= lim Á 2 - ˜
x Æ0 Ë x tann 2
ta x¯
tan 2 x - x 2 È0 ˘
= lim 2 2 Í 0 form ˙
x Æ0 x tan
tan x Î ˚
tan 2 x - x 2
= llim
im
x Æ0 tan 2 x
x4 ◊
x2
Ê tan 2 x - x 2 ˆ Ê x2 ˆ
= lim Á ◊ lim
˜ x Æ0 Á 2 ˜
Æ0
Æ
xxÆ 0Ë x4 ¯ Ë tan x ¯
Ê tan 2 x - x 2 ˆ È tan x ˘
= lim Á ˜ ◊◊11 Í∵ xlim = 1˙
x Æ0 Ë x4 ¯ Î Æ 0 x ˚
tan 2 x - x 2 È0 ˘
= lim 4 Í 0 form ˙
x Æ0 x Î ˚
ec 2 x - 2 x
tan x ◊ ssec
2 tan
= lim Applying L’Hospital’s rule]
[[Applying
x Æ0 4 x3

= lim
(
2 tan x 1 + tan 2 x - 2 x )
x Æ0 3
4x
1 tan tan 3 x - x
n x + tan
= lim
2 x Æ0 x3
1 Ê tan x - x ˆ 1 tan 3 x
= lim Á ˜ + lim
2 x Æ0 Ë x 3 ¯ 2 x Æ0 x 3
1 Ê tan x - x ˆ 1 È tan x ˘
= lim Á ˜+ Í∵ xlim = 1˙
Æ0 Ë
2 xxÆ x3 ¯ 2 Î Æ0 x ˚

1 1 Ê sec 2 x - 1ˆ È0 ˘
= + lim Á Í 0 form ˙ L’Hospitaall’s rule]
[ Applying L’
2 2 x Æ0 Ë 3 x 2 ˜¯ Î ˚

1 1 2 secc 2 x ◊ tan x
= + lim [ Applying L’Hospital’s rule]
2 2 x Æ0 6x
1 1 tan x
= + lim sec 2 x ◊ lim
2 6 x Æ0 x Æ0 x
1 1
= +
2 6
2
=
3
4.44 Chapter 4 Indeterminate Forms

Example 10
 a cot
cot x b  1
If lim  + 2  = , find a and b.
x→0  x x  3
Solution
1  a cot
cot x b 
= lim  + 2
3 x → 0  x x 
 a b
= lim  + 
x → 0  x tan
tan x x 2 
 ax + b tan
tan x  0 
= lim  2   0 form 
x→0  tan x 
x tan
((ax + b tan
tan x)
= lim
x→0  tan x 
((xx ⋅ x) 
2
 x 

ax + b tan
tan x  x 
= lim ⋅ lim  
→00

xx→ x3 x → 0  tan x 

ax + b tan
tan x 0   x 
= lim 3
⋅1  0 form
rm  ∵ lim = 1
x→0 x  x → 0 tan x

 a + b sec 2 x 
= lim   [Applying L’Hospital’s rule]
x→0  3x 2
a + b sec 0
=
0
a+b
=
0
a + b = 0, a = –b ... (1)
1 −b + b sec 2 x 0 
Thus, = lim  0 form 
3 x→0 3x 2
b ⋅ 2 sec
sec x sec x tan
tan x
= lim [Applying L’Hospital’s rule]
x→0 6x
 b   tan x 
=  lim
m sec 2 x  lim 
 x→
x →0
→ 0 3  x→0 x 
b  tan x
tan 
= ∵ xli→m0 x = 1
3
b =1
From Eq. (1), a = –1
Hence, a = –1, b = 1
4.6 Type 4 : ∞ − ∞ Form 4.45

Example 11
 f ′ ( x) 1 
Evaluate lim  − .
x→ a
x→
 f ( x) − f (a) x − a 
Solution
 f ′ ( x) 1 
Let l = lim  −  [∞ − ∞ form]
x→a
 f ( x) − f (a) x − a 
′ ( x) − [ f ( x) − f (a)]
((xx − a ) f ′( 0 
= lim  0 form 
x→a [ ) − f (a)] ((xx − a)
f ( x
f ′ ( x) + ((xx − a ) f ′′ ( x) − f ′ ( x) 0 
= lliim  0 form  [[A
Applying L’Hospital’s rule]
x→a
x→ [ f ( x) − f (a)] + ((xx − a) f ′( x)
′′ ( x) + ((xx − a ) f ′′′(
f ′′( ′′′ ( x)
= lim [Appl
A ying L’Hospita
Hospital
Hospitall’s rule]
′ ( x) + f ′(
x → a f ′( ′ ( x) + ((xx − a ) f ′′(
′′ ( x)
f ′′ (a )
=
2 f ′(a)

EXERCISE 4.4
 1
m  ccot x −  = 0.
1. Prove that lim
x →0  x
 1 
2. Prove that lim
m − cot x − a) = 0 .
cot(((x
x →a x − a
 

3. Prove that lim (secc x − tan


tan x) = 0.
π
x→
2

 sec x  2
2 x sec
m  ttan x −
4. Prove that lim  = π .
π π
x→
2

 1 1  1
5. Prove that lim  − =− .
x →0 x − a log(x + 1 − a)  2

 1 1  1
6. Prove that lim  − =− .
x →3 x − 3 log(x − 2) 
log( 2

4.46 Chapter 4 Indeterminate Forms

  1 
m x − x 2 llog  1 +
7. Evaluate lim 
x →∞   x

 1
Hint : Put x = y 
 
 1 
 Ans.: 2 
1 1  1
8. Prove that lim  − x  = .
x →0  x e − 1 2

4.7 TYPE 5 : 1∞, ∞ 0, 0 0 FORMS

To evaluate the limits of the type lim[ f ( x)]g ( x ) which takes any one of the above
x →a
x→
form for f (x) > 0, we proceed as follows:
Let l = lim [ f ( x)]g ( x ) where f (x) > 0
x →a
x→

logg l = lim
lim [ g ( xx)) ⋅ log
log f (x
( )]
x→a
x→

which takes the form • × 0, i.e., type 3 form.

Example 1
1
m (a + x) = ae.
Prove that lim
x x
[Summer 2014]
x→0

Solution
1
Let l = lim (a x + x ) x [1• form]
x Æ0

1
log l = lim ◊ log (a x + x ) [• × 0 form ]
x Æ0 x
log (a x + x ) È0 ˘
= lim Í 0 form ˙
x Æ0 x Î ˚
1
x
(a x log
log a + 1)
= lim a + x Applying L’Hospital’s rule]
[[Applying
x Æ0 1
0
a lloog a + 1
=
a0 + 0
logge a + llog
oge e
=
1
4.7 Type 5: 1•, •0, 00 Forms 4.47

= log ae
Hence, l=a
ae

Example 2
1
m(e − 5 x) = e −2 .
Prove that lim( 3x x
x→0

Solution
1
Let l = lim(e3 x − 5 x) x [1• form]
x→0
1
g l = lim
log lim llog( og(ee3 x − 5 x)
x→0 x

log(e3 x − 5 x) 0 
= lim  0 form 
x→0 x
1
⋅ ((33e3 x − 5)
((ee3 x − 5 x)
= lim [[A
Applying L’Hospital’s rule]
x→0 1
3e0 − 5
=
e0
= −2
Hence, l = e -2

Example 3
1
1 − log x
Evaluate lim(logg x) .
x→e

Solution
1
Let l = lim(log x) 1− log x
[1• form]
x→e

1 0 
g l = lim
log ⋅ log(logg x)  0 form 
1 − log x
x→e
x→

1 1

log x x
= lim [[A
Applying L’Hospital’s rule]
x→e
x→ 1

x
= −1
1
Hence, l = e −1 =
e
4.48 Chapter 4 Indeterminate Forms

Example 4
1
1
Ê a x + bx + c x ˆ 3x
˜ = ((abc) .
Prove that lim
mÁ 9 [Summer 2015]
x Æ0 Ë 3 ¯
Solution
1
Ê a x + bx + c x ˆ 3x
Let l = lim
mÁ ˜ [1• form]
x Æ0 Ë 3 ¯

1 Ê ax + bx + cx ˆ
log l = lim log
gÁ ˜
x Æ0 3 x Ë 3 ¯
Ê ax + bx + cx ˆ
log Á ˜
Ë 3 ¯ È0 ˘
= lim Í 0 form ˙
x Æ0 3xx Î ˚
Ê 3 ˆ (a x log
log a + b x log b + c x log
log c)
ÁË x ˜ ◊
a + bx + cx ¯ 3
= lim
x Æ0 3
[Applying L’Hospital’s rule]
Ê 1 ˆ (a 0 log a + b0 log b + c 0 log c)
=Á 0
Ë a + b0 + c 0 ˜¯ 3
1
= log abc
9
1
= log (abc) 9
1
Hence, l = (abc) 9

Example 5
1
Ê 1x + 2 x + 3 x ˆ x
Evaluate lim Á ˜ [Summer 2017]
x Æ 0Ë 3 ¯
Solution 1
Ê 1x + 2 x + 3 x ˆ x
Let l = lim Á ˜ [1• form]
xÆ0 Ë 3 ¯

1 Ê 1x + 2 x + 3 x ˆ
log l = lim log Á ˜
xÆ 0 x Ë 3 ¯
4.7 Type 5: 1•, •0, 00 Forms 4.49

Ê 1x + 2 x + 3 x ˆ
log Á ˜
Ë 3 ¯ È0 ˘
= lim Í 0 form ˙
xÆ0 x Î ˚
3 (1x log 1 + 2 x log 2 + 3 x log 3)
= lim ◊
xÆ 0 1x + 2 x + 3 x 3
[Applying L’Hospital’s rule]
1
= (log 2 + log 3)
3
1
= log(6)
3
1
= log(6) 3
1
Hence, l= (6) 3

Example 6
x
 1x 1 1 1
 1
Prove that lim
m a + b x
+ c x
+ d x
 = (abcd ) .
4
x →∞
 4 
Solution
x
 1 1 1 1

Let l = lim  a x + b x + c x + d x 
x →∞
 4 
1
Putting = y, when x Æ •, y Æ 0
x
1
Ê ay + by + cy + d y ˆ y
l = lim Á ˜ [1• form]
yÆ0 Ë 4 ¯
1  ay + by + cy + d y 
logg l = lim log  
y→0 y  4
 ay + by + cy + d y 
log  
 4 0 
= lim  0 form 
y→0 y
   a log a + b log b + c log c + d log d 
y y y y
4
= lim  y 
y→0  a + b + c + d  
y y y
 4 

[Applying L’Hospital’s rule]


4.50 Chapter 4 Indeterminate Forms

g a + llog
log og b + log
log c + log d
=
4
1
= log (abcd )
4
1
= log(abcd ) 4
1
Hence, l = ((abcd ) 4

Example 7
x
 ax + 1
2

 =e .
a
Prove that lim 
x →∞  ax − 1

Solution
x
 ax + 1
Let l = lim  
x →∞  ax − 1

x
 1
1+
 ax 
= lim
m [[1∞ form]
x →∞  1
1− 
 ax 
 1 
 1 + ax 
logg l = lim
lim x log  
 1− 1 
x →∞

 ax 
ax   1   1 
= lim
lim og 1 +  − log 1 −  
llog
x →∞ a   ax a   ax 
− ax
1  1 
ax
1 
= lim log 1 +  + log 1 −  
  a axx   aaxx  
x →∞ a

1   1
ax

= (log e + log e) ∵ li m 1 +  = e
a  x →∞  ax  
1
= (1 + 1)
a
2
=
a
2

Hence, l = ea
4.7 Type 5: 1•, •0, 00 Forms 4.51

Example 8
1
1  a x   x−x a
m 
Evaluate lim +  .
  x
x→ a 2
x→ a  
Solution
1
1  a x   x−a
Let l = lim   +  [1∞ form]
  x
x→ a 2
x→ a  

1 È1 Ê a x ˆ˘
log l = lim log Í Á
lo + ˙
xÆa x - a ÍÎ 2 Ë x a ˜¯ ˙˚
a+ x
log
2 ax
= lim
xÆa x-a
log(a + x) − log 2 ax
= lim
x→a x−a
1
log(a + x) − log 2 a − lo
log
gx
2 0 
= lim  0 fform
orm 
x→a x−a 
1 1

= lim
m a + x 2 x [Appl
A ying L’’Hospita
Hospitall’s rule]
x→a 1
1 1
= -
2a 2a
=0
Hence, l = e0
=1

Example 9
 3
 2 
Prove that lim(cos
m( s 2 x) x
= e −6 .
x→0

Solution
 3
 2 
Let l = lim(cos 2 x) x [1• form]
x→0
4.52 Chapter 4 Indeterminate Forms

3
log l = lim ◊ log(coss 2 x )
x Æ0 x2
3 log(cos
log(cos 2 x ) È0 ˘
= lim 2 Í 0 form ˙
x Æ0 x Î ˚
3 ((-2 sin
sin 2 x )
= llim
im ◊ Applying L’Hospital’s rule]
[[Applying
x Æ 0 cos 2 x 2x
Ê tan 2 x ˆ
= lim - 6 Á
x Æ0 Ë 2 x ˜¯
= -6

Hence, l = e -6

Example 10
1
1
Ê tan x ˆ x 2
˜ =e .
Prove that lim Á 3
x Æ0 Ë x ¯

Solution
1
 tan x  x2  tan x
tan 
Let l = lim   [1∞ for
form] ∵ lim
lim = 1
x→0  x  x→0 x 
1  tan x 
g l = lim
log log 
x→0 x 2
 x 
 tan x 
log 
 x  0 
= lim  0 form 
x→0 x2
x  x sec
sec 2 x − tan x  1
= lim  ⋅ [Applying L’Hospital’s rule]
x → 0 tan x
 x2  2x
sec 2 x − tan x
x sec 0   x 
= lim  0 form  ∵ lim = 1
x→0 2 x3 x → 0 tan x

secc 2 x + x ⋅ 2 sec 2 x tan
tan x − sec 2 x
= lim
x→0 6x2
2
secc x tan tan x
= lim ⋅ [Applying L’Hospital’s rule]
x→0 3 x
1
=
3
1
Hence, l = e3
4.7 Type 5: 1•, •0, 00 Forms 4.53

Example 11
px
tan 2
Ê xˆ 2a
Prove that lim Á 2 - ˜ = e p .
xÆa Ë
xÆ a¯
Solution
px
tan
Ê xˆ 2a
Let l = lim Á 2 - ˜ [1• form]
xÆa Ë a¯

Ê p xˆ Ê xˆ
log l = lim tan
n Á ˜ log Á 2 - ˜
xÆa
xÆ Ë 2a ¯ Ë a¯
Ê xˆ
log Á 2 - ˜
Ë a¯ È0 ˘
= lim Í 0 form ˙
xÆa
xÆ Ê ˆ
p x Î ˚
co
ott Á ˜
Ë 2a ¯
1 Ê 1ˆ 1
= lim ÁË - a ˜¯ [Applying L’Hospital’s rule]
xÆa Ê xˆ Ê 2 p xˆ Ê p ˆ
ÁË 2 - a ˜¯ ÁË - cosec 2 a ˜¯ ÁË 2 a ˜¯

2
=
p
2
Hence, l = ep

Example 12
Evaluate lim(coss x )cot x . [Winter 2013]
x Æ0

Solution
Let l = lim(cos x )cot x [1• fform]
orm]
x Æ0
log l = lim cot
cot x ◊ llog(coss x )
x Æ0
log(coss x ) È0 ˘
= lim Í 0 form ˙
x Æ0 tan x Î ˚
1
( - ssin x )
im cos x 2
= llim [ Applying L’Hospital’s rule]
x Æ0 sec x
=0
4.54 Chapter 4 Indeterminate Forms

Hence, l = e0 = 1

Example 13
2
Evaluate lim(cosec x) tan x .
π
x→
2

Solution
2
Let l = lim (cosec x )tan x
[1• form]
p

2

tann 2 x ◊ log(cosec x )
log l = lim ta
p

2
log(cosec x ) È0 ˘
= lim 2 Í 0 form


p cot x Î ˚
2
1
( -cosec
cosec x cot
cot x )
cosec x
= lim [ Applyinngg L’Hospita
ospitall’s rule]
Hospital
H

p cosec 2 x )
2 cott x( -cosec
2

sin 2 x
= lim

p 2
2
1
=
2
1
Hence, l = e2

Example 14
1
1
Ê sin h x ˆ x 2
˜ =e .
Prove that lim Á 6
x Æ0 Ë x ¯
Solution
1
 sinh x  x2  sinh x 
Let l = lim   [1• fform] ∵ lim = 1
x→0  x 
x→0 x 

1 Ê sinh x ˆ
log l = lim log Á
x Æ0 x2 Ë x ˜¯
4.7 Type 5: 1•, •0, 00 Forms 4.55

Ê sinh x ˆ
log Á
Ë x ˜¯ È0 ˘
= lim Í 0 form ˙
x Æ0 x2 Î ˚
x Ê x cosh x - sin
nhh xˆ 1
= lim Á ˜¯ ◊ 2 x [ Applying L’Hospital’s rule]
x Æ 0 sinh x Ë x 2

x cosh x - sinh x È0 ˘È x ˘
= lim Í 0 form ˙ Í∵ xlim = 1˙
x Æ0 2 x3 Î ˚Î Æ0
Æ sinh x ˚
x sinh x + cosh x - cosh x
= lim [ Applying L’Hospital’s rule]
x Æ0 6 x2
1 sinh x
= lim ◊
x Æ0 6 x
1 È sinh x ˘
= Í∵ xlim = 1˙
6 Î Æ0 x ˚
1
Hence, l = e6

Example 15
p
sec2
Ê p ˆ 2 - bx
m Á sin 2
Evaluate lim ˜ .
x Æ0 Ë 2 - ax ¯
Solution
p
sec2
Ê p ˆ 2 - bx
m Á sin 2
Let l = lim [[1• form]
form]
x Æ0 Ë 2 - ax ˜¯
p Ê 2 p ˆ
log l = lim secc 2 ◊ llog
og Á si
sinn
x Æ0 2 - bx Ë 2 - ax ˜¯
Ê p ˆ
logg Á ssin 2
Ë 2 - ax ˜¯
= lim
x Æ0 p
cos2
2 - bx
1 Ê p ˆ Ê p ˆ È p ˘
◊ 2 sin
nÁ ˜ ◊ cos Á ˜ ◊ Í- ((- a )˙
p Ë 2 - ax ¯ Ë 2 - ax ¯ Î (2 ( - ax ) 2
˚
sin 2
= lim 2 - ax
x Æ0 Ê p ˆÈ Ê p ˆ˘È p ˘
2 coss Á ˜ Í - sin Á ˜ ˙ Í - ( - b)˙
Ë 2 - bx ¯ Î Ë 2 - bx ¯ ˚ Î (2( - bx ) 2
˚
Applying L’Hospital’s rule]
[[Applying
4.56 Chapter 4 Indeterminate Forms

Ê p ˆ
cot Á
Ë 2 - ax ˜¯ 2 a ( 2 - bx ) 2 È0 ˘
= lim ◊ lim Í 0 form ˙
x Æ0 Ê 2p ˆ x Æ0 b(2 - a axx )2 Î ˚
- sin Á ˜
Ë 2 - bx ¯
Ê p ˆ È p ˘
-cosec 2 Á ◊ - ( - a )˙
2a Ë 2 - ax ˜¯ ÍÎ (2 - ax )2 ˚
=- lim [ Applying L’H pital’s rule]
Hosspita
b x Æ0 Ê 2p ˆ È 2p ˘
◊ - ( - b)˙
Ë 2 - bx ˜¯ ÍÎ (2 - bx )2
cos Á
˚
Ê pa ˆ
-
2a Á 4 ˜
=- ◊Á
b -2p b ˜
Á ˜
Ë 4 ¯
a2
=-
b2
a2

Hence, l = e b2

Example 16
n2
Ê qˆ
Evaluate lim Á coss ˜ .
nÆ• Ë n¯
Solution
n2
Ê qˆ
Let l = lim
m Á cos ˜ [[1• form]
form]
n Æ• Ë n¯
Ê qˆ
log l = lim n2 log
g Á cos ˜
n Æ• Ë n¯
1
Putting = x,
n
when n Æ •, x Æ 0
sq x)
log(cossq È0 ˘
log l = lim Í 0 form ˙
x Æ0 x2 Î ˚
1
((-q sin
sin q x )
= lim cos qx Applying L’Hospital’s rule]
[[Applying
x Æ0 2x
q 1 q sin
sin q x
= - lim ◊
2 x Æ0 cos q x qx
4.7 Type 5: 1•, •0, 00 Forms 4.57

q2 È sin q x ˘
=- Í∵ xlim = 1˙
2 Î Æ 0 qx ˚
q2
-
Hence, l=e 2

Example 17
x2
Ê 1ˆ
Evaluate lim Á x sin
n ˜ .
x Æ• Ë x¯
Solution
x2
Ê 1ˆ
Let l = lim Á x sin ˜ [[1• form]
form]
x Æ• Ë x¯

 1
g l = lim
log lim x 2 logg  x sin 
x →∞  x
 1
logg  x ssin 
 x
= lim
x →∞ 1
x2
1
Let = y,
x
when x → ∞, y → 0

1 
logg  sin y 
y  0 
logg l = lim  0 form 
y→0 y2
y  cos y ssiin y 
− 2 
sin y  y y 
= lim [[A
Applying L’Hospital’s rule]
y→0 2y
cot y − 1
y cot
= lim
y→0 2 y2
y − tan
tan y 0 
= lim 2  0 form 
y→0 2 y tan
tan y
1 − sseec 2 y
= lim [[A
Applying L’Hospital’s rule]
tan y + 2 y 2 sec
y → 0 4 y tan sec 2 y
− taan
n2 y
= lim
tan y + 2 y 2 sec
y → 0 4 y tan sec 2 y
4.58 Chapter 4 Indeterminate Forms

tan 2 y
1 y2
= − lim
2 y → 0 2 tan y
+ sec 2 y
y
1 1  tan y 
=− ⋅ ∵ lim = 1
2 2 +1  y→0 y 
1
=−
6

Example 18
m( t x)sin x .
Evaluate lim(cot
x →0
0

Solution

Let l = lim(cot x)sin x [∞ 0 for


form]
x→0

g l = lim
log sin x ⋅ llog(cott x)
lim sin [0 × ∞ for
orm
m]
x→0

log(cott x) ∞ 
= lim  ∞ fform
orm 
x→0 cosec x 
1
( −cosec
cosec 2 x)
= lim cot x [Appl
A ying L’Hosspita
pital’s rule]
x → 0 − cosec x cot
cot x
cosec x
= lim
x→0 cot 2 x
1 sin 2 x
= lim ⋅
x → 0 sin x cos 2 x

sin x
= lim
x→0 cos 2 x
=0

Hence, l = e0 = 1
4.7 Type 5: 1•, •0, 00 Forms 4.59

Example 19
tan x
Ê 1ˆ
Evaluate: lim Á ˜ . [Summer 2016]
x Æ0 Ë x ¯

Solution
tan x
Ê 1ˆ
Let l = lim Á ˜
xÆ0 Ë x¯
[•∞ form ]
Ê 1ˆ
log l = lim tan x log Á ˜
xÆ0 Ë x¯
log x È• ˘
= lim - Í • form ˙
xÆ0 cot x Î ˚
Ê 1ˆ
ÁË - x ˜¯
= lim [Applying L’Hospital’s rule ]
xÆ0 - cosec 2 x
1
= lim x
xÆ0 1
sin 2 x
sin 2 x
= lim
xÆ0 x
sin 2 x
= lim ◊x
xÆ0 x2
2
Ê sin x ˆ
= lim Á
xÆ0 Ë x ¯
˜ ◊ xlim
Æ0
x

= 1.0
=0
Hence, l = e0 = 1

Example 20
1− cos x
 1
Prove that lim   = 1.
x→0  x 
4.60 Chapter 4 Indeterminate Forms

Solution
1− cos x
 1
Let l = lim
m  [∞ 0 form]
x→0  x 

 1
logg l = llim 1 − ccos
im(((1
im os x) log  
x→0  x
 x
= lim
m  2 sin 2  (−
( log x)
x→0  2
2 2
 x  x
2  sin   
 2  2
= lliim 2
( log x)
(−
x→0
 x
 
2

  x 
 sin 
x 2 ( − llog
og x)  2 
= lim ∵ lim   = 1
→0

xx→ 0 2  x→0  x  
  2  
1 (−
( log x) ∞ 
= lim  ∞ form 
x→0 2  1
 2 
x
 1

1  
= lim  x  [Applying L’Hospital’s rule]
2 x→0 2
− 3
 x 
1  x2 
= lim  
2 x→0  2 
=0
Hence, l = e0 = 1

Example 21
sinh −1 x
cosh −1 x
Prove that lim e = e.
x →∞

Solution
sinh −1 x
cosh −1 x
Let l = lim e
x →∞

= lim
m esinh
x →∞
( −1

)
x cosh −1 x
[∞ 0 form]
4.7 Type 5: 1•, •0, 00 Forms 4.61

sinh −1 x
g l = lim
log lim ⋅ log e
x →∞ cosh −1 x

log ( x + x 2 + 1) ∞ 
= lim  ∞ form 
x →∞
log ( x + x − 1)) 2

1   1
1 +
⋅ 2 x

x + x +1 2
2 x +1 2
= lim
m [Appl
plying L’Hospital
Hospital’s rule]
x →∞
1  1 
 1+ ⋅ 2 x
x + x2 − 1  2 x2 − 1 
x2 − 1
= lim
x →∞
x2 + 1
1
1− 2
= lim x
x →∞ 1
1+ 2
x
=1
Hence, l = e1 = e

Example 22
ex
Prove that lim x
= 1.
x→0
 1  x 
1 +  
 x 
Solution
x
 1  x 
Let l = lim 1 +  
x→0  x  

x2
 1
= lim 1 +  [∞ 0 form]
x→0  x
 1
g l = lim x 2 log 1 + 
log
x→0  x
 1
log 1 + 
 x ∞ 
= lim  ∞ form 
x→0 1
x2
4.62 Chapter 4 Indeterminate Forms

1  1

1  x 2 
1+
= lim
m x [Appl
[Appl
A ying L’Hospital
Hospital’s rule]
x→0 2
− 3
x
x
= lim
x→0  1
2 1 + 
 x
x2
= lim
x → 0 2 ( x + 1)

=0
\ l = e0
=1
ex e0
Hence, lim x
= =1
x Æ0 1
ÈÊ 1ˆ ˘
x
ÍÁ 1 + ˜ ˙
ÍÎË x¯ ˙
˚

Example 23
1
Ê 1ˆ x
Prove that lim Á ˜ = 1 .
x Æ• Ë x ¯

Solution
1
 1 x [00 form]
Let l = lim  
x →∞  x 

1  1
g l = llim lo
log log  
x →∞ x  x
− log
log x ∞ 
= lim  ∞ form 
x →∞ x
1
= − lim x [Applying L’Hospital’s rule]
x →∞ 1

=0
Hence, l = e0 = 1
4.7 Type 5: 1•, •0, 00 Forms 4.63

Example 24
1
Prove that lim(1 - x ) 2 lo g(1- x )
log(
= e. [Winter 2013]
x Æ1
Solution 1
2 lo g(1- x )
Let l = lim(1 - x ) log(
[00 form]
x Æ1

1 È• ˘
log l = lim log (1 - x 2 ) Í • ffor
orm ˙
x Æ1 log(1 - x ) Î ˚
2x
-
(1 - x 2 )
= lim [Applying L’Hospital’s rule]
x Æ1 1
( -1)
(1 - x )
2 xx((1 − x)
= lim
x →1(1
1−− x))((1 + x)
2x
= lim
x →1 1 + x

=1
Hence, l=e

Example 25
p
-x
Evaluate lim (cos x) 2 . [Summer 2016]
p

2
Solution
p
-x
Let l = lim (cos x ) 2 [0∞ form]
p

2
Êp ˆ
log l = lim Á - x ˜ log cos x [0 ¥ • form]
p Ë 2 ¯

2
log cos x È• ˘
= lim Í • form ˙

p 1 Î ˚
2
Êp ˆ
ÁË 2 - x ˜¯
sin x
-
= lim cos x
p
xÆ -
1
2 2
(-1)
Êp ˆ
ÁË 2 - x ˜¯
4.64 Chapter 4 Indeterminate Forms

- tan x
= lim

p 1
2 2
Êp ˆ
ÁË 2 - x ˜¯
2
Êp ˆ
= lim - Á - x ˜ tan x

p Ë 2 ¯
2
2
Êp ˆ
ÁË 2 - x ˜¯
= lim -

p cot x
2

Êp ˆ
-2 Á - x ˜ (-1)
Ë2 ¯
= lim

p -cosec 2 x
2

Êp ˆ
= lim 2 Á - x ˜ sin 2 x
p Ë 2 ¯

2
=0
Hence, l = e0 = 1

Example 26
1 − xx
Evaluate lim .
x→0 x log
log x
Solution
1 − xx
Let l = lim
x→0 x log x
1 − e x log
log x
0 
= lim  0 form 
x → 0 x log
log x
log x  1 
−ee x log
−  x ⋅ + log x
x
= lim [[A
Applying L’Hospital’s rule]
x→0 1
x ⋅ + log x
x
= lim ( −e x log x )
x→0

= lim( − x x )
x→0

= − lim x x
x→0

Let L = lim x x 00 ffor


orm 
x→0
4.7 Type 5: 1•, •0, 00 Forms 4.65

g L = llim
log log( x x )
im log(
x→0

= lim
m x log
log x
x→0

log x ∞ 
= lim  ∞ form 
x→0 1
x
1
m x
= lim [Appl
A ying L’Hospita
Hospital
Hospitall’s rule]
x→0 1
− 2
x
= lim( − x)
x→0

=0

L = e0 = 1
Hence, l = −1

Example 27
1 − x sin x
Prove that lim = −1 .
x → 0 x log x

Solution
Let l1 = lim x sin x [00 form]
x→0

g l1 = lim sin x ⋅ log x


log
x→0

log x ∞ 
= lim  ∞ form 
x→0 cosec x
1
= lim x [Appl
A ying L’Hospita
Hospital
Hospitall’s rullee]
x → 0 − cosecc x cot
cot x
 sin x 
2
= lim  − 
x→0 
cos x 
x cos
2
 sin x  x
= − lim   ⋅
x→0  x cos x
 sin x 
=0 ∵ lxiim
m = 1
→0 x 
log l1 = 0
l1 = e0 = 1
∴ lim
lim x sin
sin x
=1 ...(1)
x→0
4.66 Chapter 4 Indeterminate Forms

Let l2 = lim x log x [0 × ∞ form]


x→0

log x ∞ 
= lim  ∞ form 
x→0 1
x
1
m x
= lim [Appl
A ying L’Hospita
Hospital
Hospitall’s rul
rule]
x→0 1
− 2
x
= lim(
lim( − x)
x→0

=0 ...(2)

1 − x sin x 0 
Let l = lim
x → 0 x log
log x  0 form  [Using Eqs (1) and (2)]

1 - esinn x log
log x
0 
= lim  0 form 
x Æ0 x log
log x
log x Ê s
siin x ˆ
- esinn x log Á + cos
cos x ◊ log
log x ˜
1- x sin x
Ë x ¯
lim = lim [Applying L’Hospital’s rule]

Æ00 x lo
logg x x Æ0 1 + log x

 sin x  1 
− x sin x   ⋅ + cos x 
  x lo
log
g x 
= lim ...(3)
x→0 1
+1
log x
[Dividing numerator and denominator by log x]
 1 
1 + cos 0
 −∞   sin x 
=−  Using Eq
q. (1
1)) and lim = 1
1  → x 
+1 x 0

−∞
= −1

EXERCISE 4.5
1
 a x + a2x + ... + anx  x 1
1. Prove that lim  1  = (a1a 2 ...a n ) n
.
x →0  n 
4x
 x1 1 1 1

 1 + 2x + 3x + 4 x 
2. Prove that lim = 24 .
x →∞  4 
 
4.7 Type 5: 1•, •0, 00 Forms 4.67

1 1
3. Prove that lim(1 + sin
sin x) 2 x = e 2 .
x →0
1
1
4. Prove that lim(x)1− x = .
x →1 e
2  1 
5. Prove that lim(sin x)tan x
= .
x→
π  e 
2
1
 sin x  x 2
1

6. Prove that lim   = e 6.
x →0  x 
x
 a
7. Prove that lim  1 +  = e .
a
x →∞  x
x
 x + 1
 =e.
2
8. Prove that lim 
x →∞  x − 1

x
 2 x + 1
9. Prove that lim   = e.
x →∞  2 x − 1

lim( + sin
10. Prove that lim(1 sin x)cose
cosec
cosec x
sec
= e.
x →0

lim( + sin
11. Prove that lim(1 sin x)cot
cot x
= e.
x →0

lim( + tan
12. Prove that lim(1 tan x)cot
cot x
= e.
x →0

1
1
13. Prove that lim(1 − tan
tan x) x = .
x →0 e
1
1
lim(coss x) x =
2
14. Prove that lim(co .
x →0
e
1
 sin x  x
15. Prove that lim  x  = 0 .
x →0 

a2
2 −
lim(coss ax)cosec
16. Prove that lim(co bx
=e 2 b2
.
x →0

2 1
lim(coss x)cot
17. Prove that lim(co x
= .
x →0
e
18. Prove that lim(cosec x)sin x = 1.
x →0

x
 2
19. Prove that lim  1 +  = e .
2
x →∞  x
4.68 Chapter 4 Indeterminate Forms

cot( x − a )
 x −
1
20. Prove that lim  2 −  = e a.
x →a  a
21. Prove that lim(tan x)cos 2x
2x
= 1.
π
x→
4

22. Prove that lim(sec x)cot x = 1 .


π
x→
2

2 sin x
 1
23. Prove that lim   = 1.
x →0  x 

24. Prove that lim(sin x)tan x = 1.


lim(sin
x →0
1

25. Prove that lim(1 − x n )log(1− x ) = e .


x →1

 πx
tan  
 2 
26. Prove that lim x = e.
x →0
1
27. Prove that lim(e 3 x − 5x) x = e −2.
x →0

 3
 2 
lim(coss 2 x) x
28. Prove that lim(co = e −6.
x →0

2
29. Prove that lim(coss x)cos x
= 1.
π
x→
2

4.8 SOLVING INDETERMINATE FORMS USING EXPANSION


In some cases, it is difficult to differentiate the numerator or denominator, or in some
cases, power of x in the denominator is very large. In such cases, expansion of function
is used to evaluate the limit.

Example 1
(1 + x) n − 1
Evaluate lim .
x →0 x
Solution
(1 + x) n − 1
Let l = lim
x→0 x
 n(n − 1) 2 n(n − 1)(
)(n − 2) 3 
1 + nx + 2 ! x + 3!
x +  −1

= lim
x→0 x
4.8 Solving Indeterminate Forms Using Expansion 4.69

 n(n − 1)
n( n( n −
n( −1 n − 2) 2
1)((n 
= lim  n + x+ x + 
x→0
 2! 3! 
=n

Example 2
sin -1 x - x 1
Prove that lim 3
= .
x Æ0 x 6
Solution
sin −1 x − x
Let l = lim
x→0 x3
 x3 3 5 
 x + 6 + 40 x + ... − x
= lim
x→0 x3
1 3 2 
= lim  + x + ...
x→0  6 40 
1
=
6

Example 3
an −1 x
sinn x − ttan
Evaluate lim .
x→0 log (1 + x)
x 2 log
Solution
tan −1 x
sinn x − tan
Let l = lim
log (1 + x)
x → 0 x 2 log

 x3 x5   x3 x5 
 x − + − ... −  x − + − …
3! 5!
3!   3 5 
= lim
x→0  x 2 x3 x 4 
x 2  x − + − + ...
 2 3 4 
 1 23 2 
x3  − x + ...
 6 120 
= lim
x→0  x x 2 x3 
x3  x − + − + ...
 2 3 4 
1
=
6
4.70 Chapter 4 Indeterminate Forms

Example 4
sin sin −1 x − x 2
n x sin
Evaluate lim .
x→0 x6
Solution
 x3 x5  x3 3 5 
 x − + − ...  x + + x + ... − x 2
3! 5 !  6 40 
Let l = lim
x →0 x6
 2 x 4 3 6 x 4 x 6 x8 x 6 x8 
x + + x − − − + + + ...  − x 2
= lim 
6 40 6 36 80 120 720 
x →0 x6
x6
+ Higher powers of x
= lim 18
x→0 x6
1
=
18

Example 5
tan tan −1 x − x 2
n x tan
Evaluate lim .
x→0 x6
Solution
Let  x3 2 5  x3 x5 x 7 
 x + + x + ...  x − + − + ... − x 2
3 15  3 5 7 
l = lim 6
x→0 x
 2 x 4 x 6 x8 x 4 x 6 x8 2 6 2 8 
 x − 3 + 5 − 7 + 3 − 9 + 15 + 15 x − 45 x + ... − x
2

= lliim
x→0 x6
1 1 2 
x 6  − +  + Higher powers of x more than 6
 5 9 15 
= lim
x→0 x6
2
=
9

Example 6
tanh x − 2 sin
sin x + x
Evaluate lim
x→0 5
.
x
4.8 Solving Indeterminate Forms Using Expansion 4.71

Solution
tanh x − 2 sin x + x
Let l = lim
x→0 x5
 1 3 2 5 17 7   x3 x5 x 7 
 x − x + x − x +  − 2  x − − − −  + x
3 15 315  3! 5 ! 7 !
3! 
= lim 5
x→0 x
5 7 138 2 
x  − x + 
 60 5040 
= lim
x→0 x5
7
=
60

Example 7
1− x
e x + log 
 e 
Evaluate lim .
x→0 tan x − x
Solution
1− x
e x + log 
 e 
Let l = lim
x→0 tan x − x
e x + log(1 − x) − log e
= lim
x→0 tan x − x
 x 2 x3 x 4   x 2 x3 x 4 
1 + x + 2! + + +  +  − x − − − −  − 1
2 ! 3! 4 !   2 3 4 
= lim
x→0  x 3
2 5 
 x + 3 + 15 x +  − x

x3 5 4
− − x −
= lim 63 24
x→0 x 2
+ x5 + 
3 15
1 5
− − x −
= lim 6 24
x→0 1 2
+ x2 + 
3 15
1
=−
2
4.72 Chapter 4 Indeterminate Forms

Example 8
e 2 x − (1 + x) 2
Evaluate lim .
log(1 + x)
x → 0 x log(

Solution
e 2 x − (1 + x) 2
Let l = lim
x → 0 x log(1 + x x)
 ( x) 2 ((2 x)3 (2
(2 ( x) 4 
1 + 2 x + 2 ! + 3! + 4 ! +  − ((1 1 + 2x + x2 )
= lim  
x→0  x 2 x3 
x  x − + − 
 2 3 
8 3 16 4
x2 +x + x +
= lim 6 24
x→0  x x 2

x 2 1 − + − 
 2 3 
8 16 2
1+
x+ x +
= lim 6 24
x→0 2
x x
1− + −
2 3
=1

Example 9
e x sin x − x − x 2 2
Prove that lim =− .
x→0 x + x log
2
log (1 − x) 3
Solution
e x sin x − x − x 2
Let l = lim
x →0 x 2 + x log
log (1 − x)
 x 2 x3  x3 
1 + x + 2! + + ...  x − + ... − x − x 2
2 ! 3!  3! 
= lim
x→0  x 2 x3 
x 2 + x  − x − − − ...
 2 3 

 x3 x 4 x3 x5 x4 x6 
 x − + x 2
− + − + − + ... − x − x 2
3!
3! 3! 2! 2 ! 2 !3!
3! 3! 3! 3!3! 
= lim 3 4
x→0 x x
x 2 − x 2 − − − ...
2 3
4.8 Solving Indeterminate Forms Using Expansion 4.73

x3 x5
− + ...
= lim 3 3 124
x →0 x x
− − − ...
2 3
1 x2
− + ...
= lim 3 12
x→0 1 x
− − − ...
2 3
1
= 3
1

2
2
=−
3

Example 10
1
( + x) x − e
(1 e
Prove that lim =− .
x→0 x 2
Solution
1
( + x) x − e
(1
Let l = lim
x→0 x
1
log (1+ x )
ex −e
= lim
x→0 x
1  x 2 x3 
 x − + − ...
x 2 3 
e −e
= lim
x→0 x
 x x2 
1− + − ...
 2 3 
e −e
= lim
x→0 x
 x x2 
 − + − ...
 2 3 
ee −e
= lim
x→0 x
  x x2  1 x 
2

e 1 +  − + − ... +  − + ... + ... − e
  2 3  2 2  
= lim 
x→0 x
4.74 Chapter 4 Indeterminate Forms

 1 11x 
= lim e  − + − ...
x→0  2 24 
e
=−
2

Example 11
1
ex
( + x) − e +
(1 x

Prove that lim 2 = 11e .


x→0 x2 24
Solution
1
ex
( + x) x − e +
(1
Let l = lim 2
x→0 x2
1
log (1+ x ) ex
e x
−e+
= lim 2
x→0 x2
1  x x3 x 4
2 
 x − + − + ... ex
x 2 3 4 
e −e+
= lim 2
x→0 x2
 x x x  2 3
1− + − + ... ex
e 
2 3 4
−e+
= lim 2
x→0 x2
 x x 2 x3 
 − + − + ... ex
 2 3 4 
ee −e+
= lim 2
x→0 x2
  x x 2 x3  1  x x2 
2
 ex
e 1 +  − + − + ... +  − + −  +   − e +
  2 3 4  22!!  2 3   2
= lim 2
x→0 x
 ex ex 2
ex 2 3
ex ex 3
 ex
ex
 e − 2 + 3 + 8 − 4 − 6 + ... − e + 2
= lim
x→0 x2
 11e 5e 
= lim  − x + ...
x → 0  24 12 
11e
=
24
4.8 Solving Indeterminate Forms Using Expansion 4.75

Example 12
1
  cosh x − 1  x2 1

  = e .
12
Prove that lim  2 
x→0
  x2 
Solution
1
  cosh x − 1  x2
Let l = lim  2   
x→0
  x2 
1   cosh x − 1 
g l = lim
log logg  2 
lo  
 
2
x→0 x x2 
   x2 x4 x6  
1   1 + + + + ... − 1 
= lim
m log  2   2!! 4 ! 6 !  
x→0 x 2
  x 2 

  x2 x4 x6 
= lim
1
⋅ log  2  + + + ...  

x→0 x2   2 24 2 720  
  x

1   x2 x4 
= lim 2
⋅ log 1 +  + + ... 

x →0 x   12 360 

1  x 2 x 4 
2
 1  x2 x4 
= lim 2 ⋅  + + ...  −  + + ...  + .. 
x →0 x
 12 360  2  12 360  
1 x2 x2 
= lim  + − + ...
x → 0  12 360 288 
1
=
12
1
Hence, l = e12

Example 13
e x − esin x
Prove that lim = 1.
x → 0 x − sin x

Solution
l = lim e − e
x sin x
sin
Let
x → 0 x − sin
sin x
4.76 Chapter 4 Indeterminate Forms

 x3 x5 
 x − + − ...
 3! 5 ! 
3!
e −e
x sin x
= e −e
x

 x3 x5 
 − + − ...
x  3!
3! 5 ! 
= e −e e
x

x3 x5
= ex − exez where, z = − + − ...
33!! 5!
  z2 
= e x 1 − 1 + z + + ... 
  2 ! 
 z2 
= e x  − z − − ...
 2! 
 x3 x5 
x − sin x = x −  x − + − ...
 3! 5!
3! 
 x3 x5 
=  − + ...
 3!
3! 5! 
= −z
 z2 
−ee x  z + + ...

 2! 
∴ l = lim
x→0 −z
 z 
= lim e x 1 + + ...
x→0  2! 
= e0

=1 ∵ lim z = 0
 x → 0 

Example 14
sin(sin x) − sin 2 x 1
x sin(sin
Prove that lim = .
x→0 x6 18
Solution
Let l = lim x sin sinn x) − sin 2 x
sin (si
((sin
x→0 x6
sin 3 x sin
sin 5 x
n x ) = sin
sin (sin sin x - + - ...
3! 5!
3 5
Ê x3 x5 ˆ 1Ê x3 x5 ˆ 1Ê x3 x5 ˆ
= Áx- + - ...˜ - Á x - + - ...˜ + Á x - + - ...˜ - ...
Ë 3! 5! ¯ 3! Ë 3! 5! ¯ 5! Ë 3! 5! ¯
4.8 Solving Indeterminate Forms Using Expansion 4.77

 x3 x5  1 x3  1
= x− + − ... −  x 3 − 3 x 2 ⋅ + ... + ( x 5 − ...)
 6 120  6 6  120
x3 x5
= x− + − ...
3 10
x4 x6
x sin (sin x) = x 2 − + − ...
3 10
2
 x3 x5 
sin x =  x − + − ...
2

 3! 5!
3! 
x6 x3 x5
= x2 + − 2x + 2x ⋅ + ...
36 6 120
x4 2 6
= x2 − + x − ...
3 45
x6 2 6
x sin (sin x) − sin 2 x = − x − ...
10 45
1
= x 6 − ...
18
1 6
x − Higher powers of x
18 1
Hence, l = lim 6
=
x→0 x 18

Example 15
x (a + b cos
cos x) − c ssin
in x
Find a, b, c, if lim 5
= 1.
x→0 x
Solution
x (a + b cos
cos x) − c sin
sin x
1 = lim
x→0 x5
  x2 x4 x6   x3 x5 x7 
x  a + b 1 − + − + ...  − c  x − + − + ...
 2
2!! 4 ! 6!
6 !   3! 5! 7 ! 
= lim  5

x→0 x
 b c  b c  b c
((a + b − c) x + x 3  − +  + x 5  −  + x 7  − +  + ...
 2 6  4! 5!  6! 7!
= lim 5
x→0 x
 b c   b c  b c
(a + b − c) + x 2  − +  + x 4  −  + x 6  − +  + ...
 2 6  4! 5!  6! 7!
= lim 4
x→0 x
4.78 Chapter 4 Indeterminate Forms

But limit is finite, therefore, numerator must be zero.


b c b c
a + b − c = 0, − + = 0, − = 1,
2 6 24 120
a + b − c = 0, − 3b + c = 0, 5b − c = 120 ⋅
Solving all the equations, we get a = 120, b = 60, c = 180

EXERCISE 4.6
x − sin
sin x 1
1. Prove that lim = .
x →0 tan3 x 6
tan x − x 1
2. Prove that lim = .
x →0 x 2 tan
tan x 3
tan2 x − x 2 2
3. Prove that lim = .
x → 0 x 2 tan
tan2 x 3
x3
sin x − x +
6 = 1
4. Prove that lim .
x →0 x5 120

sinh x − 2 x 1
2 sinh
5. Prove that lim = .
x →0 x 2 sin
sin x 3
log(1 + x 3 )
6. Prove that lim = 1.
x →0 sin3 x
e x − e x cos x
7. Prove that lim = 3.
x → 0 x − sin
sin x
an−1 x 1
n x − ttan
sin
8. Prove that lim = .
log(1 + x)
x → 0 x 2 log( 3

9. Prove that lim


e x sin
sin x
− cosh
cosh x 2 ( ) = 1.
4
x →0 x 6

10. Prove that lim


2
2 x 2 − 2e x + 2 coss (x ) + sin x = −1.
3
2
3

x →0 x4
sinh x − x 1
11. Prove that lim = .
x →0 n x − x ccos x 2
sin
sin x − 6 x + x 3
6 sin 3
12. Prove that lim = .
x → 0 log(1 + x) − 2 x 3 + x 4 40
Multiple Choice Questions 4.79

POINTS TO REMEMBER

• L’Hospital’s Rule
f ( x) f ′(x)
If lim f (x) = 0, lim g (x) = 0, then lim = lim
x →a x →a x →a g(x) x → a g ′( x)

MULTIPLE CHOICE QUESTIONS

Choose the correct alternative in the following questions:


log x
1. The value of lim n , n > 0 is
x Æ• x

(a) • (b) –• (c) 1 (d) 0


tan x
2. The value of lim is
x Æ0 x
(a) 0 (b) 1 (c) p (d) •
x
(1 - a )e
3. If lim exists and is finite then it is equal to
x Æ0 x
1
(a) 1 (b) 0 (c) –1 (d)
2
px + sin x
4. The integer p for which lim is finite is
x Æ0 x2
(a) 0 (b) –1 (c) 1 (d) 2
-1
5. The value of lim(cosh x - log x ) is
x Æ0
(a) log 1 (b) log 2 (c) log 3 (d) 0
6. The value of lim x 2 e - x is
x Æ•
(a) 1 (b) e1 (c) e2 (d) 0
7. The value of lim (cos x )cos x is
p

2
(a) 0 (b) 1 (c) –1 (d) 2
8. The value of lim log tan x tan 2 x is
x Æ0

(a) 0 (b) 1 (c) –1 (d) 2


x
9. The value of lim(1 - x ) is
x Æ0
1
(a) 0 (b) 1 (c) –1 (d)
2
4.80 Chapter 4 Indeterminate Forms

1 + 2 + 3 + ... + x
10. The value of lim is
x Æ• x2
1
(a) 0 (b) 1 (c) –1 (d)
2
1
Ê 1ˆ x
11. The value of lim Á ˜ is
x Æ• Ë x ¯

(a) 0 (b) 1 (c) 2 (d) 3


12. The value of lim sin x log x is
x Æ0
(a) –1 (b) 0 (c) 1 (d) 5
13. The value of lim(cos x )cot x is [Summer 2016]
x Æ0
(a) 0 (b) –1 (c) 1 (d) 2
1
14. The value of lim(e3 x - 5 x ) x is
x Æ0
(a) 0 (b) 1 (c) e–2 (d) e2
sin 2 x + p sin x
15. If lim is finite then value of p is
x Æ0 x3
(a) –2 (b) 1 (c) –1 (d) 2
tan x
16. The value of lim is [Winter 2013]
x Æ0 x
(a) 0 (b) 1 (c) p (d) •
x
17. The value of lim is [Winter 2013]
x Æ0 x
(a) 0 (b) ±1 (c) p (d) •
2
x - x-2
18. The value of lim is [Winter 2014]
x Æ0 x2 - 4
3 3
(a) 4 (b) (c) 0 (d) –
4 4

19. The value of lim x x is [Winter 2015]


x Æ0
1
(a) 1 (b) –1 (c) e (d)
e
1 - n2
20. The value of lim is [Summer 2014]
n Æ• Sn

(a) –2 (b) 1 (c) 2 (d) 0


Answers 4.81

1
21. The value of lim xx is [Summer 2015]
x Æ•

(a) • (b) – • (c) 1 (d) 0


n
Ê 1ˆ
22. The value of lim Á 1 + ˜ is [Winter 2015]
n Æ• Ë n¯
1
(a) 1 (b) –1 (c) e (d)
e
sin x
23. The value of lim is [Summer 2016]
x Æ• x

(a) 1 (b) –1 (c) 0 (d) none of these


sin( x - 6)
24. The value of lim is [Winter 2016]
x Æ6 x-6
(a) 0 (b) 1 (c) –1 (d) 0.5
sin 2 x
25. The value of lim is [Summer 2017]
x Æ0 x
1
(a) 2 (b) 1 (c) –1 (d)
2
cos x
26. The value of lim is [Summer 2017]
p p

2 x-
2
p
(a) 0 (b) 1 (c) –1 (d)
2

Answers
1.(d) 2.(b) 3.(b) 4.(a) 5.(b) 6.(d) 7.(a) 8.(b) 9.(a) 10.(d)
11.(b) 12.(b) 13.(c) 14.(c) 15.(a) 16.(b) 17.(b) 18.(b) 19.(a) 20.(a)
21.(c) 22.(c) 23.(c) 24.(b) 25.(a) 26.(c)
CHAPTER
Improper
5
Integrals
5.1 INTRODUCTION
b
The definition of a definite integral Ú f ( x) dx requires the interval [a, b] be finite. The
a
fundamental theorem of calculus requires that f (x) be continuous on [a, b] or at least
bounded. In this chapter, we will study a method of evaluating integrals that fail these
requirements either because their limits of integration are infinite, or because a finite
number of discontinuities exist on the interval [a, b]. Integrals that fail either of these
requirements are known as improper integrals. Improper integrals cannot be computed
using a normal Riemann integral.

5.2 IMPROPER INTEGRALS


b
The integral Ú f ( x ) dx is called an improper integral if
a
(i) one or both limits of integration are infinite
(ii) function f (x) becomes infinite at a point within or at the end points of the inter-
val of integration.
Improper integrals are classified into three kinds.

5.3 IMPROPER INTEGRALS OF THE FIRST KIND



It is a definite integral in which one or both limits of integration are infinite, e.g. Ú e - x dx
0
is an improper integral of the first kind since the upper limit of integration is infinite.
These integrals are evaluated as follows:
(i) If f (x) is continuous on [ a, •) then
• b

Ú dxx = lim Ú f ( x ) dx
f ( x) d
b Æ•
… (1)
a a

(ii) If f (x) is continuous on ( -•, b] then


b b

Ú f ( x ) ddxx = lim
a Æ-•
Æ- •
Ú f ( x)
x ) dx … (2)
-• a
5.2 Chapter 5 Improper Integrals

( , •) then
(iii) If f (x) is continuous on (-•
• b

Ú f ( x ) ddxx = lim
a Æ-•
Æ- •
Ú f ( x)
x ) dx
-• b Æ• a
0 b
= lim
lim
a Æ-•

Ú f ( x) dx + blim
Æ•
Ú f ( x) dx … (3)
a 0

The improper integral is said to converge (or exist) when the limit in RHS of (1), (2)
and (3) exist (or finite). Otherwise, it is said to diverge.

Example 1

1
Evaluate Ú dx .
1 x
Solution
• b
1 1
Ú x
dx = lim Ú
b Æ• x
dx
1 1
b
= lim 2 x
b Æ• 1

= lim (2 b - 2)
b Æ•
=•

Example 2

1
Evaluate Ú dx .
1 x2

Solution
∞ b
1 1
∫1
x2
dx = lim ∫ 2 d
b →∞
1
x
dxx

b
1
= lim −
b →∞ x1
 1 
= lim  − + 1
b →∞  b 
=1
5.3 Improper Integrals of the First Kind 5.3

Example 3

d
dx
Evaluate Ú 2
. [Winter 2014]
0 x +1
Solution
• b
dxx
d dx
dx
Ú x2 + 1 = lim Ú
b Æ• 2
x +1
0 0
b
m tan -1 x
= lim
b Æ• 0

= lim ÈÎ tan -1 (b) - tann -1 0 ˘˚


b Æ•

n -1 (b)
= lim tan
b Æ•

tan -1 (•)
= tan
p
=
2

Example 4
0
Evaluate Ú x sin x dx .
-•

Solution
0 0

Ú dxx = lim
x sin x d
a Æ-•
Æ- •
Ú x sin
sin x dx
-• a
0
= lim - x cos x + sin x a
a Æ-•

= lim (a cos
cos a - sin
n a)
a Æ-•

= -•
• sin a and cos a oscillate between ± 1]
[[∵ sin

Example 5
0

∫e
2x
Evaluate dx.
−∞
5.4 Chapter 5 Improper Integrals

Solution
0 0

Ú e2 x dx = lim Úe
2x
dx
a Æ-•
Æ- •
-• a
0
e2 x
= lim
a Æ-• 2
a

Ê1 1 ˆ
= lim Á - e2 a ˜
a Æ-• Ë 2 2 ¯
1
= -0
2
1
=
2

Example 6

1
Evaluate Ú dx .
-• 1 + x2
Solution
• 0 b
1 1 1
Ú 2
dx = lim Ú
• 1 + x2
dx + lim Ú
x2
dx
-• 1 + 0 1+
x a Æ-
Æ-• bÆ •
a
0 b
= lim tan -1 x + lim tan -1 x
a Æ-• a b Æ• 0
-1 -1
= lim (0 - tan a ) + lim (tan b - 0)
Æ- •
aÆ-
a b Æ•

p p
= +
2 2
=p

Example 7

Evaluate Ú e x dx .
-•

Solution
• 0 b

Ú e x dx = lim Ú e dx + lim Ú e dx
x x
a Æ-
Æ-•• bÆ•
-• a 0
5.3 Improper Integrals of the First Kind 5.5

0 b
= lim e x + lim e x
a Æ-• a b Æ• 0

= lim (1 - e ) + lim (eb - 1)


a
a Æ-• b Æ•
b
= (1 - 0) + lim (e - 1)
b Æ•
=•

Example 8

1
Evaluate Ú -x
x
dx .
-• e + e

Solution
∞ ∞
1 ex
∫ e x + e− x
−∞
dx = ∫ e2 x + 1 dx
−∞
dx

0 b
ex ex
a →− ∞ ∫ e 2 x + 1 b→ ∞ ∫ e2 x + 1
= lim
m d
dxx + lim dx
dx
a 0

Putting u = e x , du = e x dxx ,

ex du -1
-1
Ú e2 x + 1 dx = Ú u2 + 1 = tan u = tan -1 e x

1 0 b
Ú x -x
dx = lim tann -1 e x + lim tan -1 e x
tan
-• e +e a Æ-• a b Æ• 0

Êp ˆ Ê pˆ
= lim Á - tan -1 e a ˜ + lim Á tan -1 eb - ˜
a Æ-• Ë 4 ¯ bÆ• Ë 4¯
Êp ˆ Êp pˆ
= Á - 0˜ + Á - ˜
Ë4 ¯ Ë 2 4¯
p
=
2

Example 9

x
Evaluate Ú dx .
-• (1 + x 2 )2
5.6 Chapter 5 Improper Integrals

Solution
• 0 b
x x
Ú x = lim dx + lim Ú
Æ• x 2
dx
-• a 0
0 b
- x - 2x
= li ( +x x d
a
a
2
0 b
1 1
= - + -
a Æ-• 2 1 x2 a
x )0
È n [ ]n +1 ˘
∵ f( )
Î Ú
f x ) dx
n +1 ˚
1 1 È 1 1
= - + - +
a -• ) 2
1
=- +
2 2
=0

Example 10

dv
Evaluate Ú 1
. [Winter 2016]
0 ( v v)
Solution

1
• b 2
lim Ú 1 + v 1 dv
v
Ú 1
0 v v 0 1 + tan v
b f ¢( ) ˘
lim log (1 tan - ) ∵Ú dv log | f ( ) |
b Æ• 0 f( )

+ tan -1 b log1˘
b Æ•

+ tan - • - 0
p
+
2
5.3 Improper Integrals of the First Kind 5.7

Example 11

x+3
Evaluate Ú 2
dx .
2 (x -
-11)( x + 1)
Solution
• b
x+3 È 2 (-
-22 x - 1) ˘
Ú ( x --11)( x 2 + 1) dx = blim Ú Î x -1 +
Æ• Í ˙ dx [ Usin expansion ]
Usingg partial fraction expansion
2 2 x2 + 1 ˚

Èb 2 b
2x
b
1 ˘
= lim Í Ú ddxx - Ú 2 dxdx - Ú 2 dx ˙
b Æ• Í x - 1
Î2 2 x +1 2 x +1 ˙˚

È b˘
= lim Í 2 log( x - 1) - log( x 2 + 1) - tan -1 x ˙
b Æ• Î 2˚

È f ¢( x ) ˘
Í∵ Ú dx = log f ( x ) ˙
Î f ( x ) ˚
È b˘
( x - 1)2
= lim Í log 2 - tan -1 x ˙
b Æ• Í x +1 ˙
Î 2˚

È (b - 1)2 1 ˘
= lim Í log 2 - tan --11 b - log + tan -1 2 ˙
b Æ• Í
Î b +1 5 ˚˙
È Ê 1ˆ
2 ˘
Í 1 -
ÁË b ˜¯ ˙
Í
= lim logg -1
-1
- tan b + lo -1 ˙
logg 5 + tan 2
b Æ• Í 1 ˙
Í 1+ 2 ˙
ÍÎ b ˙˚

= log 1 - tan --11 • + lo


logg 5 + tan -1 2
p
= 0- + log 5 + tan -1 2
2
p
=- + log 5 + tan -1 2
2

Example 12

1
Prove that p-integral Ú dx converges when p > 1 and diverges when
p £ 1. 1 xp
5.8 Chapter 5 Improper Integrals

Solution
• b
1 1
Ú xp dx = lim Ú
b Æ• xp
dx
1 1
b
x - p +1
= lim , p π1
b Æ• -p +1
1

È b - ( p -1) - 1 ˘
= - lim Í ˙, p π 1
Î p - 1 ˚˙
b Æ• Í

Case (I) When p > 1


È 1 ˘

1 Í b p -1 - 1 ˙
Ú xp
dx = - lim Í
b Æ• Í p - 1 ˙
˙
1
ÍÎ ˙˚
È1 ˘
Í • - 1˙
= -Í ˙
Í p -1 ˙
ÍÎ ˙˚
1
= [finite
f
finite]
p -1
Case (II) When p < 1

1 È b1- p - 1 ˘
Ú xp
dx = - lim Í
b Æ• Í p - 1 ˙
˙
1 Î ˚
È • - 1˘
= -Í ˙
Î p -1˚
= -•
Case (III) When p = 1

1 •
Ú x dx = log x 1 = •
1

1
Hence, p integral Ú xp dx converges when p > 1 and diverges when p £ 1.
1

EXERCISE 5.1
Evaluate the following improper integrals:

1
1. ∫ x dx
1
[Ans.: •]
5.4 Improper Integrals of the Second Kind 5.9


1  π
2. ∫ 1+ x
0
2
dx  Ans.: 2 
 

2
−1  1
3. ∫ dx  Ans.: − 2 
−∞
x5  

1  1 
4. ∫ x log
2
2
x
 Ans.: log 2 
 

 1
∫e
−x
5. sin xdx  Ans.: 2 
0  

∫x e
2 −x
6. dx [Ans.: 2]
0

∫ | x |e
− x2
7. dx [Ans.: 1]
−∞

1  π
8. ∫x 1 x −12
dx  Ans.: 2 
 

1  π
9.
−∞
x∫+ 2 x +5
dx
2  Ans.: 2 
 


e− x
 2
10. ∫1 x
dx  Ans.: e 
 

5.4 IMPROPER INTEGRALS OF THE SECOND KIND


It is a definite integral in which integrand become infinite (or unbounded or
discontinuous) at one or more points within or at the end points of the interval of
integration, e.g.
1
1 1
(i) Ú x dx is an improper integral of the second kind as
x
is not continuous at x = 0.
0
2
1 1
(ii) Ú x -1 2
dx is an improper integral of the second kind because 2
x -1
is not
-2
continuous at x = –1 and x = 1.
These integrals are evaluated as follows:
(i) If f (x) is unbounded at x = a then
b b

Ú f ( x )dx = lim Ú f ( x )dx


cÆ a
… (1)
a c
5.10 Chapter 5 Improper Integrals

(ii) If f (x) is unbounded at x = b then


b c

Ú f ( x )dx = lim Ú f ( x )dx


cÆb
… (2)
a a

(iii) If f (x) is unbounded at x = a and x = b then


b 0 c2

Ú f ( x ) ddxx = lim
c1 Æ a
Ú x )dx + lim
f ( x)
c2 Æ b
Ú f ( x )dx … (3)
a c1 0

The improper integral is said to converge (or exist) when the limit in RHS of (1), (2)
and (3) exist (or finite). Otherwise, it is said to diverge.

Example 1
3
1
Evaluate Ú dx . [Summer 2014, 2017]
0 3- x
Solution
1
The integrand is unbounded at x = 3.
3- x

3 c
1 1
Ú 3- x
dx = lim Ú
c Æ3 3- x
dx
0 0
c
= lim -2 3 - x
c Æ3 0

= lim( -2 3 - c + 2 3 )
c Æ3

=2 3

Example 2
5
1
Check the convergence of Ú x 2 dx. [Summer 2016]
0

Solution
1
The integrand is unbounded at x = 0.
x2
5 5
1 1
Ú x2 dx = lim
cÆ0
Ú x 2 dx
0 c
5.4 Improper Integrals of the Second Kind 5.11

5
1
= lim -
cÆ0 x c

È 1 Ê 1ˆ ˘
= lim Í- - Á - ˜ ˙
cÆ0 Î 5 Ë c ¯ ˚

=•

Hence, the integrand is divergent.

Example 3
1 dx
Check the convergence of Ú0 1 - x . If convergent, then evaluate the
same. [Winter 2016]
Solution
1
The integrand is unbounded at x = 1.
1- x
1 dx c 1
Ú0 1 - x = clim Ú
Æ1 0 1 - x
dx

c
= lim - log(1 - x ) 0
c Æ1

= lim [ - log(1 - c) + log1]


c Æ1

= –log (1 – 1)
=•
Hence, the integrand is divergent.

Example 4
3
d
dx
Check the convergence of Ú . [Summer 2015]
0 9 - x2
Solution
1
The integrand is unbounded at x = 3.
9 - x2
3 c
dxx
d dx
dx
Ú 2
= lim Ú
c Æ3
0 9- x 0 9 - x2
5.12 Chapter 5 Improper Integrals

c
xˆ -1 Ê
= lim sin Á ˜
c Æ3 Ë 3¯ 0

È Ê cˆ ˘
= lim Ísin -1 Á ˜ - sin -1 0 ˙
c Æ3 Î Ë 3¯ ˚
= sin -1 (1) - 0
p
= [finite]
2
Hence, the integrand is convergent.

Example 5
p
2
Evaluate Ú sec xd
xdx .
0
Solution
p
The integrand sec x is not continuous at x = .
2
p
2 c

Ú sec cÆ
im Ú sec
sec x dx = llim
p
sec x dx
0 0
2
c
= lim sec x + tan
lim log sec tan x
p 0

2
= lim lo
ogg secc c + tan
tan c
p

2
p p
= log
log sec - tan
2 2
=•

Example 6
3
d
dx
Evaluate Ú 2
. [Summer 2015]
0
( x - 1) 3
Solution
1
The integrand 2
is unbounded at x = 1.
(x - 1) 3
5.4 Improper Integrals of the Second Kind 5.13

3 3
dx dx
Ú = lim
1
+ lim
Æ1
Ú 2
0 c2
(x ) x )3 (x )3
c1 3
1 1
(x x - 1) 3
= lim + lim
1Æ c Æ1 1
3 0 3 c2
1 1˘ 1 1˘
= 1 3 1 ( )3 ˙
1 1 c2 Æ1

È 1˘ È 1 ˘
-( )3 + - 0˙

È 1 ˘
= +1

Example 7
1
1
Evaluate Ú 2
dx .
1 3
x
Solution
1
The integrand 2 is unbounded at x = 0.
x3
1 c 1
1
Ú 2
x x Ú 2
d
1 3 -
x x3 2
x3
c 1
1 1 1
= lim 3 x + lim 3x 3
1 Æ0 c2 Æ 0
-1 c2
1 1 1˘
= 3( -1) 3 + lim 3 3
2
1 2 Æ0

1
= 3( 3 - 0]
=6
5.14 Chapter 5 Improper Integrals

Example 8
5
1
Evaluate ∫ ((x − 2)
0
2
dx .
dx

Solution
1
The integrand is unbounded at x = 2.
( x − 2) 2

5 c1 5
1 1 1
Ú (x
( - 2 )2
dx = lim
c1 Æ
Æ22
Ú ((x - 2)2 dx + lim
c2 Æ 2
Ú ((x - 2)2 dx
0 0 c2

c1 5
1 1
= lim - + lim -
c1 Æ 2 x-2 0
c2 Æ 2 x-2 c2

Ê 1 1ˆ Ê 1 1 ˆ
= lim Á - - ˜ + lim Á - +
c1 Æ 2 Ë c1 - 2 2 ¯ 2 Ë 3 c2 - 2 ˜¯
c Æ 2

= -• + •, indeterminate form

Hence, no conclusion can be made about the value of the integral.

Example 9
a
1
Evaluate ∫
−a a2 − x2
dx .
dx

Solution
1
The integrand is unbounded at x = ± a.
a
a - x2
2

a 0 c2
1 1 1
Ú 2 2
dx = lim
c1 Æ- a
Ú 2 2
dx + lim
c2 Æ a
Ú dx
-a a -x c1 a -x 0 a - x2
2

0 c2
-1 x -1 x
= lim sin + lim sin
c1 Æ- a a c2 Æ a a
c1 0
5.4 Improper Integrals of the Second Kind 5.15

È c ˘ È c ˘
= lim Ísin -1 0 - sin -1 1 ˙ + lim Ísin -1 2 - sin -1 0 ˙
c1 Æ- a Î a ˚ c2 Æ a Î a ˚
Ê aˆ Ê aˆ
= - sin -1 Á - ˜ + sin -1 Á ˜
Ë a¯ Ë a¯

= sin -1 1 + sin -1 1

= 2 sin -1 1
p
= 2◊
2
=p

Example 10
π
2
sin x
Evaluate ∫
0 1 − cos x
dx.
dx

Solution
The integrand is unbounded at x = 0.
p p
2 2
sin x
sin sin x
sin
Ú 11- cos
cos x
dx = lim Ú
cÆ0 1 - cos x
dx
0 c
p
2 1
-
= lim Ú (1
(1 - cos x ) 2 siinn x dx
cÆ0
c
p
1 2
È∵ Ú[ f ( x )]n f ¢( x )dx ˘
( - cos x ) 2
(1 Í ˙
= lim Í [ f ( x )]n +1 ˙
cÆ0 1
Í = n +1 ˙
2 c
Î ˚
p
1 2
= lim 2(1 - cos x ) 2
cÆ0
c
5.16 Chapter 5 Improper Integrals

È 1˘
= lim 2 Í1 - (1 - cos c) 2 ˙
cÆ0 Í ˙˚
Î
=2

5.5 IMPROPER INTEGRAL OF THE THIRD KIND

It is a definite integral in which one or both limits of integration are infinite, and the
integrand become infinite at one or more points within or at the end points of the
interval of integration. Thus it is a combination of the first kind and the second kind.

1
For example, Ú 2 dx is an improper integral of the third kind as the upper limit of
0 x
1
integration is infinite and integrand 2 is infinite at x = 0.
x

Example 1

1
Evaluate the improper integral Ú dx.
0 x2
Solution
• 1 c2
1 1 1
Ú x2 dx = lim
c1 Æ 0
Ú x2 dxx + lim
d
c2 Æ •
Ú x 2 dx
0 c1 1

1 c
1 1 2
= lim - + lim
im -
c1 Æ 0 x c2 Æ• x1
c1

Ê 1ˆ Ê 1 ˆ
= lim Á -1 + ˜ + lim Á - + 1˜
c1 Æ 0 Ë c1 ¯ 2 Ë c2 ¯
c Æ •

= • +1

=•
5.6 Convergence and Divergence of Improper Integrals 5.17

EXERCISE 5.2
Evaluate the following improper integrals:
1
1
1. ∫ x dx
0

[Ans.: •]
1
1
2. ∫
0 x
dx

[Ans.: 2]
1
1
3. ∫x
0
2
dx

[Ans.: •]
1

4. ∫ log xdx
0

[Ans.: –1]
9
1
5. ∫ 2
dx
0
(x − 1) 3

[Ans.: 9]
1
1
6. ∫ 1− x
0
4
dx

[Ans.: •]
2
1
7. ∫
0 x(2 − x)
dx

[Ans.: p ]

5.6 CONVERGENCE AND DIVERGENCE OF


IMPROPER INTEGRALS

(i) Direct Comparison Test


(a) If f (x) and g(x) are two continuous functions for x ≥ a such that 0 £ f ( x ) £ g( x ),
then
∞ ∞ ∞ ∞

∫ f ( x)dx
dx converges, if ∫ g ( x)dx
dx converges and ∫
a
f ( x)ddxx ≤ ∫ g ( x)dx .
a
a a

(b) If f (x) and g(x) are two continuous functions for x ≥ a such that f ( x) ≥ g ( x),
∞ ∞

then ∫ f ( x)dx
a
dx diverges, if ∫ g ( x)dx
a
dx diverges.
5.18 Chapter 5 Improper Integrals

(ii) Limit Comparison Test


If f (x) and g (x) are two continuous functions for x ≥ a such that f ( x) > 00,, g ( x) > 0
∞ ∞
f ( x)
and lim
x →∞ g ( x )
= ll,, 0 < l < ∞ then ∫
a
f ( x) dx
dx and ∫ g ( x) dx
a
dx both converge or diverge

simultaneously.
Note: Convergence of improper integral of second kind can be tested by direct
comparison test and limit comparison test similarly.

Example 1

cos x
Test the convergence of the improper integral Ú dx .
1 x2
Solution
cos x 1
f ( x) = and let g ( x) = 2
x2 x
cos x 1
≤ 2 for x ≥1 cos x ≤ 1]
[[∵ cos
x2 x
∞ ∞ b
1 1
∫ g ( x ) dx = ∫
1 1
x2 b →∞ ∫ x 2
dxx = li
d lim
1
dxx
b
1
= lim
lim −
b →∞ x1
 1 
= lim
lim  − + 1
b →∞  b 
=1

1
Thus, ∫x
1
2
dx is convergent.
dx

cos x
By direct comparison test, ∫
1
x2
is convergent.

Example 2

- x2
Test the convergence of the improper integral Úe dx .
1
Solution
2
f ( x) = e − x and let g ( x) = e − x
2 ∵ x 2 ≥ x 
e− x ≤ e− x for x ≥1  2 
− x ≤ − x
5.6 Convergence and Divergence of Improper Integrals 5.19

• •
-x
Ú g( x) dx = Ú e dx
1 1
b
= lim Ú e - x dx
b Æ•
1

= lim (e -1 - e - b )
b Æ•

= e -1
1
=

e
-x
Thus, Úe dx is convergent.
1

- x2
By direct comparison text, Úe dx is convergent.
1

Example 3

33x + 5
Test the convergence of the improper integral Ú dx .
4 x4 + 7
Solution
3x + 5 1
f ( x) = and let g ( x) = 3
x +7
4
x
3x + 5
4
= lim x + 7
f ( x)
lim
x Æ• g( x )
xƕ x ƕ 1
x3
3x 4 + 5x3
= lim
x Æ•x4 + 7
5
3+
= lim x
x Æ• 7
1+ 4
x
=3
• •
1
Ú g ( x ) dx = Ú x 3 dx
4 4
b
1
= lim Ú dx
b Æ•
4 x3
5.20 Chapter 5 Improper Integrals

b
1
= lim -
b Æ• 2 x2 4
1
= 0+
2( 4 2 )
1
=

32
1
Thus, ∫x
4
3
dxx is convergent.

3x + 5
By Limit comparison test, ∫x
4
4
+7
is convergent.

Example 4
1
1
Test the convergence of the improper integral Ú 2
dx.
0 x + x
Solution
1 1
f ( x) = and let g ( x) =
x + x
2
x
1
<
1
0 < x ≤1 [∵ x 2
+ x > x]
x2 + x x
1 1 1
1 1
∫ g ( x) ddxx = ∫
0 0 x
dx lim ∫
dx = li
c→0
c x
dx
dx

1
= lim 2 x c
c→0

lim ( 2 − 2 c )
= lim
c→0

=2
1
1
Thus, ∫
0 x
dx is convergent.
dx

1
1
By direct comparison test, ∫x
0
2
+ x
dx is convergent.
dx

Example 5
1 − e− x
1

Test the convergence of the improper integral ∫0 x3 dx.


5.6 Convergence and Divergence of Improper Integrals 5.21

Solution
1 − e− x 1
f ( x) = and let g ( x) = 3
x3 x
1 − e− x
lim (1 − e − x ) = 1
f ( x) 3
lim = limm x = lim
x →∞ g ( x )
x→∞ x →∞ 1 x →∞

x3
1 1
1
∫0 g ( x) ddxx = ∫0 x3 ddxx
1
1
= lim ∫ dxx
c→0
c
x3
1
1
= lim
lim − 2
c→0 2x c
 1 1 
= lim
lim  − + 2 
c→0  2 2c 
=∞
1
1
Thus ∫x 0
3
dxx is divergent.
1 − e− x
1

By limit comparison test, ∫0 x3 dx dx is divergent.

EXERCISE 5.3
Test the convergence of the following improper integrals.

sin2 x
1. ∫1 x 2 dx
[Ans.: convergent]
 1
∞ sin2  
2.  x
∫ x
dx
1 [Ans.: convergent]

x −12

3. ∫ x 6 + 16
dx
2 [Ans.: divergent]

x
4. ∫ 3x
1
4
+ 5x 2 + 1
dx
[Ans.: convergent]

log x dx
5. ∫ −x
1 x +e [Ans.: divergent]
5.22 Chapter 5 Improper Integrals


2 + sin x
6. ∫
−∞ x + 1
2
dx
[Ans.: convergent]
1
1
7. ∫ ( x + 1)
0 1− x2
dx

π [Ans.: convergent]
2
8. ∫ log sin x dx
0 [Ans.: convergent]
π
2
e − x cos x
9. ∫0 x dx
[Ans.: divergent]
π
2
tan x
10. ∫
1
3
dx
x2 [Ans.: divergent]

MULTIPLE CHOICE QUESTIONS


Choose the correct alternative in the following questions:
0
1
1. The value of Ú 3- x
dx is
-•
(a) 0 (b) • (c) 3 (d) 1

1
2. The value of Ú x 2 dx is
0
(a) 0 (b) 1 (c) • (d) 2

-x
3. The value of Ú (1 + 2 x)e dx is
0
(a) 0 (b) 1 (c) 3 (d) •
1
1
4. The value of Ú 10 + 2 z dz is
-5
(a) 0 (b) 1 (c) 3 (d) •
1
5. The value of Ú 6 - y dy is
-•
(a) 0 (b) 1 (c) 2 (d) •
Multiple Choice Questions 5.23


9
6. The value of Ú (1 - 3z)4 dz is
2

3 1
(a) 0 (b) (c) (d) •
125 125

6 x3
7. The value of Ú ( x 4 + 1)2
dx is
-•

(a) 0 (b) 1 (c) 6 (d) •



1
8. The value of Ú x 2 + x - 6 dx is
1
(a) 0 (b) 1 (c) 6 (d) •
1
0
ex
9. The value of Ú x2
dx is
-•

(a) 0 (b) 1 (c) 2 (d) •


p
10. The value of Ú sec 2 x dx is
1

(a) 0 (b) 1 (c) p (d) •



dx
11. The value of Ú x2 + 4 is
0

p p
(a) 0 (b) 1 (c) (d)
2 4
3
dx
12. The value of Ú is
0 9 - x2
p
(a) 0 (b) 1 (c) (d) •
2

2 -x
13. The value of Úx e dx is
0

(a) 0 (b) 2 (c) –2 (d) •


0

Ú2
14. The value of 5x
dx is
-•
1 1 2
(a) 0 (b) (c) (d)
5 log 2 log 2 5 log 2
5.24 Chapter 5 Improper Integrals


dx
15. The value of Ú 25 + 4 x 2
is
-•
p p p
(a) (b) (c) (d) p
10 5 2

1
16. The value of Ú x 2 dx is [Winter 2015]
1
(a) 1 (b) 0 (c) –1 (d) does not exist
• 1
17. The value of Ú0 1 + x 2 dx is [Winter 2016]

p
(a) p (b) (c) 0 (d) 1
2
• -x
18. The value of Ú0 e cos 2 x dx is [Summer 2017]

1 1 2
(a) 0 (b) - (c) (d)
5 5 5

Answers
1.(b) 2.(c) 3.(c) 4.(d) 5.(d) 6.(c) 7.(a) 8.(d) 9.(b) 10.(d)
11.(d) 12.(c) 13.(b) 14.(b) 15.(a) 16.(a) 17.(b) 18.(c)
Unit 4
Applications of
Integration
CHAPTER
Applications
6
of Integration
6.1 INTRODUCTION
In this chapter we will explore few applications of definite integrals. Applications of
definite integrals in finding volumes—by slicing, by disk method, by washer method
and by cylindrical shells are discussed in detail.

6.2 VOLUME BY SLICING


The volume of a solid of known integrable cross-section area A(x) formed by a plane
perpendicular to the x-axis at any point between x = a to x = b is
b
V = ∫ A( x)dx
a

Note: If the cross-section A(y) is perpendicular to the y-axis at any point between
y = c to y = d, then the volume of the solid is
d
V = ∫ A( y )dy
c

Example 1
Find the volume of the solid generated by rotating the plane region
1
bounded by y = , x = 1 and x = 3 about the x-axis.
x
Solution
The volume is generated by rotating the dotted region about the x-axis.
The cross section PQ of the generated volume is a circle of radius y perpendicular to
x-axis.
6.2 Chapter 6 Applications of Integrations

Area of cross section, A = p y2 y

1
= ◊ 1
x2 y
x
For the region shown, x varies from
1 to 3. P (x, y)
3 p x 1
V Ú1 x2
dx
B
x 3
x
O A
3
1
Q
x1
1
+1
3
2
p
3

Fig. 6.1

Example 2
Find the volume of the solid that lies between planes perpendicular to
the x-axis at x = 0 and x = 4. The cross-sections perpendicular to the
x-axis between these planes run from one side of the parabola x = y 2 to
the other. The cross-sections are squares with bases in the xy-plane.
Solution
The cross-section of the solid is a square PQRS with side runs from y x to
y x
Length PQ

Area of cross-section, x y

For the region shown, x varies S


from 0 to 4. P(x,y
4
V xd
0
4 x
O
x2 S P y) x 4
4
2
0 Q
R Q
= 32
Fig. 6.2
6.2 Volume by Slicing 6.3

Example 3
Find the volume of a right circular cone of base radius r and height h.
Solution y

The cone is generated by rotating the line B(0,h)


AB about the y-axis. Equation of the line
AB passing through the points A(r, 0) and
B (0, h) is
h-0 x
y-0 = (x - r) P Q(x,y)
0-r
Ê yˆ
x = r Á1 - ˜
Ë h¯
O x A(r,0) x

The cross-section PQ of the cone is a circle


of radius x perpendicular to the y-axis.
Fig. 6.3
Area of cross-section, A = p x 2
2
Ê yˆ
= p r 2 Á1 - ˜
Ë h¯
For the region shown, y varies from 0 to h.
2
h Ê yˆ
V = Ú p r 2 Á 1 - ˜ dy
o Ë h¯
h
3
Ê yˆ
ÁË 1 - h ˜¯
= p r2
Ê 1ˆ
3Á - ˜
Ë h¯
o

p r2h
=-
3
(0 - 1)
1
= p r2h
3

Example 4
Find the volume of the solid with a circular base of radius 5 and whose
cross sections perpendicular to the base and parallel to the x-axis are
equilateral triangles.
6.4 Chapter 6 Applications of Integrations

Solution
Equation of the circular base of radius 5 is,
x 2 + y 2 = 25 …(1)
The cross-section of the solid is an equilateral triangle PQR with base 2x.

1
Area of cross section, A= ¥ (base
(base) ¥ (height
(height )
2
1
= (2 x )(h )
2
1
= (22xx )( x tan 60°)
2
= x2 3
= (25 - y 2 ) 3 [from Eq. (1)]
For the region shown, y varies from −5 to 5.
y
Q

Q
h
R P (x
( , y) 60°
x R P
O 2x

Fig. 6.4
5
V =Ú 3 (25 - y 2 ) dy
-5
5
= 2Ú 3 (25 - y 2 ) dy
0
5
y3
= 2 3 25 y -
3
0
Ê 125 ˆ
= 2 3 Á 125 -
Ë 3 ˜¯
500 3
=
3

Example 5
Use the method of slicing to find the volume of solid with semicircular
È p p˘
base defined by y = 5 cos x on the interval Í - , ˙ . The cross-sections
Î 2 2˚
6.2 Volume by Slicing 6.5

of the solid are squares perpendicular to the x-axis with base running
from x-axis to the curve. [Winter 2015]
Solution
The cross-section of the solid is a square with side runs from y = 0 (x-axis) to
y = 5 cos x .

Length of one side of the square = 5 cos x

( )
2
Area of cross-section, A = 5 cos x

= 25 cos2 x
È p p˘ p p
In the given interval Í - , ˙ , x varies from - to .
Î 2 2 ˚ 2 2
p
2
V = Ú 25 cos2 x dx
p
-
2
p

Ê 1 + cos 2 x ˆ
2
= 25 Ú Á ˜¯ dx
p
Ë 2
-
2
p
25 sin 2 x 2
= x+
2 2 -
p
2

25 ÈÊ p p ˆ ˘
= ÍÁË + ˜¯ + 0 ˙
2 Î 2 2
[∵sin p = 0]
˚
25
= p
2

EXERCISE 6.1
1. Find the volume of the solid that lies between planes perpendicular to
the y-axis at y = 0 and y = 2. The cross sections perpendicular to the y-axis
are circular disks with diameters running from the y-axis to the parabola
x = 5y 2 .

[Ans.: 8p ]
2. Find the volume of the solid that lies between the planes perpendicular
to the x-axis at x = −1 and x = 1. The cross-sections perpendicular to the
6.6 Chapter 6 Applications of Integrations

x-axis are circular disks whose diameters run from the parabola y = x2 to
the parabola y = 2 − x2.
 16 
 Ans.: 15 π 
 

4 3
3. Show that the volume of a sphere of radius r is πr .
3
4. Find the volume of the solid whose base is the region bounded between
the curves y = x and y = x2, and whose cross-sections perpendicular to the
x-axis are squares.
 1
 Ans.: 30 
 

5. Find the volume of the solid whose base is a triangle with vertices (0, 0),
(2, 0) and (0, 2) and whose cross-sections perpendicular to the base and
parallel to the y-axis are semicircles.
 π
 Ans.: 3 
 

6.3 VOLUME OF SOLID OF REVOLUTION

A solid generated by revolving a plane area about a line in the plane is called a solid
of revolution.

Volume of Solids of Revolution by Disk Method


A solid generated by revolving a plane area about a line in the plane is called a solid of
revolution and the method is known as disk method.

Volume of Solids of Revolution by Washer Method


The method to find volume of a solid generated by revolving the region bounded
between two curves is known as washer method.

6.3.1 Volume of Solid of Revolution in Cartesian Form


Let y = f (x) be a curve and the area bounded by the curve, the x-axis and the two lines
x = a and x = b be revolved about the x-axis. An elementary strip of width dx at point
P(x, y) of the curve, generates elementary solid of volume p y2dx, when revolved about
the x-axis.
Summing up the volumes of revolution of all such strips from x = a to x = b, the
volume of solid of revolution is given by,
b
V = Ú p y 2 dx
a
6.3 Volume of Solid of Revolution 6.7

Similarly, if the area bounded by the curve x = f (y), y P(x y)


the y-axis and the two lines, y = c and y = d is revolved y = f (x)
about the y-axis, then the volume of solid of revolution
is given by,

d O x a x b
V x dy x
Fig. 6.5
The volume of solid of revolution about any axis can y
be obtained by calculating the length of the x f ( y)
y d
perpendicular from point P(x, y) on the axis of
revolution. If the area bounded by the curve y = f (x) P (x y)
is revolved about the line AB, then the volume of the c
solid of revolution is given by,
O x
M)
Fig. 6.6
with proper limits of integration.

6.3.2 Volume of Solid of Revolution in Parametric Form


When the equation of the curve is given in parametric form y B
x = f1 (t), y = f2 (t) with t1 £ t £ t2, the volume of the solid of
P(x y)
revolution about the x-axis is given by,
M
t d
V y dt
dt A
y f (x)
Similarly, the volume of the solid of revolution about the
y-axis is given by, O x
t dy Fig. 6.7
V x 2 dt
dt

6.3.3 Volume of Solid of Revolution in Polar Form


For the curve r = f (q ), bounded between the radii vectors q = q1 and q = q2, the
volume of the solid of revolution about the initial line q = 0 is given by,
2
V n d
3
2 q=p B
=∫ pr n d
3
P (r, q )
Similarly, the volume of the solid of revolution
about the line through the pole and perpendicular to
A
the initial line is given by,
q 2 q
V r cos dq q1 q2
q1 3
O q=0
q 2 2p 3
Úq1 3 r cos dq Fig. 6.8
6.8 Chapter 6 Applications of Integrations

Example 1
x2 y2
Find the volume generated by revolving the ellipse 2 + 2 = 1 about
the x-axis. a b
Solution y
The volume is generated by revolving the upper-
half of the ellipse about the x-axis. For the upper (0, b)
half of the ellipse, x varies from –a to a. Due to
symmetry about y-axis, considering the region in
B A
the first quadrant where x varies from 0 to a,
(� a, 0) O (a, 0) x
a
V = 2∫ p y 2 dxx
0
(0, � b)
 x2 
a
= 2p b ∫ 1 − 2  dx
2
dx
0 a 
a
x3 Fig. 6.9
= 2p b x − 2
2

3a 0

 a3 
= 2p b 2  a − 2 
 3a 
4
= p ab 2
3

Example 2
The region between the curve y = x , 0 £ x £ 4 and the x-axis is revolved
about the x-axis to generate a solid. Find its volume.
[Summer 2014, 2015]
Solution
The volume of solid is generated by revolving the region about the x-axis. For the
region shown, x varies from 0 to 4.
4
V = Ú p y 2 dx
0
y y= x
4
= p Ú x dx
0
4
x2
=p O (4, 0)
x
2
0

Ê 16 ˆ x=4
=pÁ ˜
Ë 2¯
= 8p Fig. 6.10
6.3 Volume of Solid of Revolution 6.9

Example 3
The graph of y = x2 between x = 1 and x = 2 is rotated around the x-axis.
Find the volume of the solid so generated. [Winter 2016]
Solution
The volume of solid is generated by revolving the region about the x-axis. For the
region shown, x varies from x = 1 to x = 2. y
2 (2, 4) y = x2
V = Ú p y 2 dx
1
2
= p Ú x 4 dx (1, 1)
1
x
p 5 2 O
= x
5 1

p
= [32 - 1]
5
Fig. 6.11
31p
=
5

Example 4
Find the volume generated by revolving the area bounded by the
parabola y2 = 8x and its latus rectum about (i) x-axis, (ii) latus rectum,
and (iii) y-axis.
y
Solution y2 = 8 x
(i) The volume is generated by revolving the
region about the x-axis. For the region above
the x-axis, x varies from 0 to 2. A
2 O (2, 0) x
V = Ú p y 2 dx
0
2 x=2
=pÚ 8
8xx ◊ dx
0
2
x2
= 8p
2 Fig. 6.12
0
= 16p
(ii) The volume is generated by revolving the region about the latus rectum. If
P(x, y) is any point on the curve, its distance from the latus rectum is 2 – x.
For the region shown, y varies from – 4 to 4. Due to symmetry about the x-axis,
considering the region in the first quadrant where y varies from 0 to 4,
6.10 Chapter 6 Applications of Integrations

4 y
V ) dy B y2 8x
2
(2, 4)
2
4
y
2 4 A
4
= dy O (2, 0) x

4 x 2
y3 y5
= − (2, 4)
6 320 0 C
256
= p Fig. 6.13
15

(iii) The volume is generated by revolving the region about the y-axis. For the region
shown, y varies from – 4 to 4. Due to symmetry about the x-axis, considering the
region in the first quadrant where y varies from 0 to 4,
4
V 2 d

y2 
4
4
y x=
64 8
5 4
p y
32 5 0

32
p
5

Example 5
Find the volume of the solid generated by revolving the region bounded
by the curve y = log x and x = 2 about the x-axis.
Solution
The volume of the solid is generated by revolving the y
y = log x
region about the x-axis. For the region shown, x varies
from 1 to 2.
x=2
V y dx
2 O (2, 0) x
= p Ú (log x
1
2 2 1 ˘
= ) x
Fig. 6.14
= 2(log 2) 2
2
log ˘
6.3 Volume of Solid of Revolution 6.11

È Ê 2 2 1 ˆ˘
= 2p Í(log 2)2 - Á log x ◊ x 1 - Ú ◊ x dx ˜ ˙
Î Ë 1 x ¯˚

= 2p ÈÍ(log 2)2 - 2 log 2 + x 1 ˘˙


2
Î ˚
= 2p [(log 2)2 - 2 log 2 + 1]
= 2p (1 - log 2)2

Example 6
Find the volume of the solid formed by the revolution of the curve
xy2 = a2 (a – x) through four right angles about the y-axis.
Solution
The volume of the solid is formed by revolving the region about the y-axis. For the
region shown, y varies from – • to •. Due to symmetry about the x-axis, considering
the region in the first quadrant, where y varies from 0 to •,

V 2 dy
y
∞ a6
=2 y
0 (
a3 
∵x= 2 2
y a
Putting y = a tan q , A
dy = a sec2 q dq O (a, 0) x
When y = 0, q=0
p
When yƕ q=
2
p
sec 2 q
V = 2p a 7 Ú 2 dq Fig. 6.15
(a )
0 2
2
tan q + a
2 2

p
2p a 7
=
a 4 Ú 0
2 cos2q dq
p
= p a 3 Ú 2 (1 + cos 2q ) dq
0
p
sin 2q 2
= pa q +3
2 0
6.12 Chapter 6 Applications of Integrations

Êpˆ
= p a3 Á ˜
Ë 2¯
p2 3
= a
2

Example 7
Find the volume of the solid of revolution of the loop of the curve
x 2 (a + x)
y =
2
about x-axis.
a−x y
Solution
The volume of the solid of revolution is
generated by revolving the upper half of the
loop about the x-axis. For the loop, x varies B A
from – a to 0. (- a, 0) (a, 0) x
O
0
V = ∫ p y dx 2 x=a
−a

0 x 2 (a + x)
=p∫ dx
dx
−a a−x
Fig. 6.16
Putting a – x = t,
dx = –dt
When x = –a, t = 2a
When x = 0, t=a
(a − t ) 2 (2a − t )
a
V = −p ∫ dt
2a t
2a 1
= p ∫ [2a 3 − t (a 2 + 4a 2 ) + t 2 (2a + 2a ) − t 3 ]dt
a t

2a  2a 3 2
=p∫  t − 5a + 4at − t  dt
2
a

2a
t3
= p 2a 3 log t − 5a 2 t + 2at 2 −
3 a

 8 1
= p a 3  2 log 2 − 10 + 5 + 8 − 2 − + 
 3 3
 2
= 2p a 3  log 2 − 
 3
6.3 Volume of Solid of Revolution 6.13

Example 8 x3
Find the volume of the solid generated by revolving the curve y 2 =
about its asymptote. 2a − x

Solution
The volume of the solid is generated by revolving the region about its asymptote.
The asymptote is x = 2a. If P(x, y) is any point on the curve, its distance from the
asymptote is 2a – x. For the region shown, y varies from – • to •. Due to symmetry
about the x-axis, considering the region in the first quadrant where y varies from 0 to •,

V = 2Ú p (2 a - x )2 dy
d y
0
3
x2
But, y=
2a − x
(33aa − x ) x 2a − x A
dy = ddxx O (2a, O) x
( 2a − x ) 2
When y = 0, x=0 x = 2a
When y Æ •, x = 2a
2a
V = 2π ∫ (3a − x ) x 2a − x dxx
0
Fig. 6.17
Putting x = 2a sin2q ,
dx = 4a sinq cosq dq
When x = 0, q = 0
p
When x = 2a, q =
2
π
V = 2π ∫ 2 (3a − 2a sin 2 θ )2a cos θ sin θ ⋅ 4 a sin θ cos θ dθ
0
π
= 16π a3 ∫ 2 (3 − 2 sin 2 θ ) sin 2 θ cos 2 θ dθ
0

 π π

= 16π a3  3∫ 2 sin 2 θ cos 2 θ dθ − 2∫ 2 sin 4 θ cos 2 θ dθ 
 0 0

 1⋅1 π 3 ⋅1⋅1⋅ π 
= 16π a3  3 ⋅ ⋅ − 2⋅ ⋅ 
 4⋅2 2 6⋅4⋅2 2
= 2π 2 a3

Example 9
Find the volume of the solid obtained by rotating the region enclosed by
the curves y = x and y = x2 about the x-axis. [Winter 2013]
6.14 Chapter 6 Applications of Integrations

Solution
The points of intersection of the curves y = x and y = x2 are obtained as,
x = x2
x2 – x = 0
x (x – 1) = 0
x = 0, 1 and y = 0, 1
Hence, O: (0, 0) and A: (1, 1) are the points of
intersection.
The volume is generated by rotating the region Fig. 6.18
about the x-axis. For the region shown, x varies from 0 to 1.
1
V = Ú p ( y12 - y22 ) dx where y1 = x and y2 = x 2
0
1
= p Ú ((xx 2 - x 4 ) dx
0
1
x3 x5
=p -
3 5
0
Ê 1 1ˆ
=pÁ - ˜
Ë 3 5¯
2p
=
15

Example 10
Find the volume generated by revolving the area cut off from the parabola
9y = 4 (9 - x2) by the line 4x + 3y = 12 about x-axis.
Solution y

The points of intersection of the parabola


9y = 4(9 – x2) and the line 4x + 3y = 12 are obtained B (0, 4)
as,
3(12 – 4x) = 36 – 4x2
4x2 – 12x = 0
4x (x – 3) = 0 A
C
x = 0, 3 and y = 4, 0 (−3, 0) O (3, 0) x
Hence, A: (3, 0) and B: (0, 4) are the points of
intersection.
The volume is generated by revolving the region Fig. 6.19
about the x-axis. For the region shown, x varies from 0 to 3.
6.3 Volume of Solid of Revolution 6.15

3 12 4 x
V
3


x

3 È16
x

16p 3
x
81
3
16p x 5
- x2
81 5
0
48
p
5

Example 11
Find the volume of the solid that results when
y
the region enclosed by the curves y = x2 and
x = y2 is revolved about y-axis. [Winter 2015] A(1, 1)

Solution O
x
2 2
The points of intersection of curves y = x and x = y are
obtained as,
x = (x2)2 Fig. 6.20
x4 – x = 0
x (x3 – 1) = 0
x = 0, 1 and y = 0, 1
Hence, O: (0, 0) and A: (1, 1) are the points of intersection.
The volume is generated by revolving the region about the y-axis. For the region
shown, y varies from 0 to 1.
1
V y
1
y
1
y5 y2
-
5 2
0
1 1
-
6.16 Chapter 6 Applications of Integrations

Example 12
Find the volume of the solid formed by revolving the area enclosed
between the curve 27ay2 = 4(x – 2a)3 and the parabola y2 = 4ax about
x-axis.
Solution
The points of intersection of parabola y2 = 4ax y
and the curve 27ay2 = 4(x – 2a)3 are obtained as,
27a (4ax) = 4(x – 2a)3
x – 6ax – 15a2x – 8a3 = 0
3 2

(x + a)2 (x – 8a) = 0 A B
x = –a, 8a O (2a, 0) (8a, 0) x
But x = –a does not lie on the curve. Hence,
x = 8a. Hence, A: (2a, 0) and B: (8a, 0) are the
points of intersection.
The volume is generated by revolving the region
about the x-axis. For the region shown, x varies Fig. 6.21
from 0 to 8a for y1 and 2a to 8a for y2.
4( x − 2a)3
where y1 = 4 aaxx and y2 =
2 2

27a
8a 8a
V = Ú p y12 dx - Ú p y22 dx
0 2a

8a 8a 4( x - 2 a )3
= pÚ 4 axdx - p Ú dx
0 2a 27a
8a 8a
x2 4p ( x - 2a ) 4
= 4ap -
2 27 a 4
0 2a

= 128p a 3 - 48p a 3
= 80p a 3

Example 13
2 2 2
Find the volume of the solid generated by revolving the curve x 3 + y 3 = a 3
about the x-axis. [Summer 2016]
Solution
The volume of the solid is generated by revolving the upper half of the astroid about
the x-axis. For the upper half of the astroid, x varies from –a to a.
6.3 Volume of Solid of Revolution 6.17

Due to symmetry about y-axis, considering the region in the first quadrant where x
varies from 0 to a, a
V = 2Ú p y dx
2

0
Using parametric equations, y
x = a cos3q, y = a sin3q
B
p
(0, b)
2
V = -2p Ú a sin q .3a cos q (- sin q ) dq
2 6 2

0 (a, 0)
x
p (–a, 0) C O A
2
= 6p a 3 Ú sin 7 q cos2 q dq
0 (0, –b)
p D
2
= 6p a 3 Ú sin 7 q (1 - sin 2 q ) dq Fig. 6.22
0
p
2
= 6p a 3 Ú [sin 7 q - sin 9 q ] dq
0

Èp p ˘
Í 2 2 ˙
= 6p a Í Ú sin q dq - Ú sin q dq ˙
3 7 9

Í0 0 ˙
ÍÎ ˙˚
È6 4 2 8 6 4 2 ˘
= 6p a 3 Í ◊ ◊ - ◊ ◊ ◊ ˙
Î7 5 3 9 7 5 3˚
6 4 2 È 8˘
= 6p a 3 ◊ ◊ ◊ Í1 - ˙
7 5 3 Î 9˚
6 4 2 1
= 6p a 3 ◊ ◊ ◊ ◊
7 5 3 9
32p a 3
=
105

EXERCISE 6.2
1. Find the volume of the solid of revolution generated by revolving the
plane area bounded by the curves y = x3, y = 0, and x = 2 about x-axis.
 128p 
 Ans.: 7 
 
6.18 Chapter 6 Applications of Integrations

2. Find the volume of the solid of revolution generated by revolving the


region bounded by the curve ex sin x and x-axis about the x-axis.
 p 2p 
 Ans.: 8 (e − 1)
 
3. Show that the volume generated by revolving the loop of the curve
y2(a + x) = x2(3a – x) about the x-axis is p a3(8 log 2 – 3).
4. Find the volume generated by revolving the curve x (y2 + a2) = a3 about
its asymptote.
 1 2 3
 Ans.: 2 p a 
 
5. The area bounded by the curve y = x(x – 1) (2 – x) and the x-axis between
x = 1 and x = 2 is revolved about the x-axis. Prove that the volume
8p
generated is .
105
6. The area enclosed by the parabolas x2 = 4ay and x2 = 4a(2a – y) revolves
about the line y = 2a. Find the volume of the solid so generated.
 32 3 
 Ans.: 3 p a 
 
7. The curve included between the curves y2 = 4ax and x2 = 4ay revolves
about the x-axis. Find the volume of the solid of revolution.
 96 3
 Ans.: 5 p a 
 

8. Find the volume generated by revolving the area between the curve
y +8
= x − 2 and the x-axis about the line x + 5 = 0.
x
Ans.: 432p 

9. Show that the volume of the spindle formed by the revolution of a


parabolic arc about the line joining the vertex to one extremity of the
2p a 3
latus rectum is , 4a being the latus rectum of the parabola.
15 5
x2 y2 a
10. The ellipse 2 + 2 = 1 is divided into two parts by the line x = and
a b 2
the smaller part is rotated through four right angles about this line. Find
the volume generated.
 3 3 p 
 Ans.: p a 2 b  − 
  4 3  
6.3 Volume of Solid of Revolution 6.19

x2 y2
11. The first quadrant of the ellipse + = 1 revolves about the line
a 2 b2
joining its extremities. Show that the volume of the solid generated is
p a 2 b2  5 p 
 − .
a 2 + b2  3 2 
12. Find the volume of the solid obtained by revolving the area between the
curves y2 = x3 and x2 = y3 about the x-axis.
 5 
 Ans.: 28 p 
 
13. The parabola y2 = 8ax divides the circle x2 + y2 = 9a2 into two arcs the
smaller of which is rotated about the x-axis. Show that the volume of
28
solid generated is p a2 .
3
14. Show that the volume obtained by revolving the area enclosed between the
a p a3
curves xy2 = a2(a – x) and (a – x) y2 = a2x about x = is (4 − p ).
2 4
15. The loop of the curve 2ay2 = x(x – a)2 revolves about the straight line y = a.
Find the volume of the solid generated.
 8 2 
 Ans.: p a3 
 15 
16. The area bounded by the hyperbola xy = 4 and the line x + y = 5 is
revolved about x-axis. Find the volume of the solid thus formed.
[Ans.: 9p ]

Parametric Form

Example 1
Find the volume of a solid generated when the region between the graph
1
f(x) = + x2 and g(x) = x over the interval [0, 2] is revolved about the
2
x-axis. [Summer 2015]

Solution
1
From given equations, y– = x2
2
and y =x
6.20 Chapter 6 Applications of Integrations

The volume is obtained by revolving the region about the x-axis. For the region shown
x varies from 0 to 2.
2
1
V = Ú p ÈÎ y12 - y22 ˘˚ dx where y1 = x 2 + and y2 = x
0
2
1
2 ÈÊ 1ˆ
2 ˘ y = – + x2
= Ú p ÍÁ x 2 + ˜ - x 2 ˙ dx
y 2

0 ÎË 2¯ ˚
2 x
È 1 1 ˘ y=
= p Ú Í x 4 + 2 x 2 + - x 2 ˙ dx
0
Î 2 4 ˚
x
2 x=2
Ê 4 1 2ˆ
O
= pÚ ÁË x + x + - x ˜¯ dx
2

0
4
2 Fig. 6.23
Ê 1ˆ
= p Ú Á x4 + ˜ dx
Ë 4¯
0
2
x5 x
=p +
5 4
0

È3 1 ˘
2
= p Í + - 0˙
Î5 2 ˚
È 64 + 5 ˘
= pÍ
Î 10 ˙˚
Ê 69 ˆ
= pÁ ˜
Ë 10 ¯

Example 2
For the cycloid, x = a (q + sinq ), y = a (1 - cosq ), find the volume of the
solid generated by the revolution of one arch about (i) the tangent at the
vertex (i.e., x-axis), (ii) y-axis, and (iii) the base.
Solution
(i) x = a (q + sinq ) y
dx
= a( 1 + cos q )
a((1 A (2a, 0) B
dq
The volume of the solid is generated by
revolving one arch about the x-axis. For the
region shown, x varies from –ap to ap, P((xx, y)
hence q varies from –p to p. Due to −ap O ap x
Fig. 6.24
6.3 Volume of Solid of Revolution 6.21

symmetry about the y-axis, considering the region in the first quadrant where q varies
from 0 to p,
p dx
V y2 dq
dq
p
= a(1 + co
q q
=2 a n4 2 cos2 dq
2 2
4 q q
16p a 3 n cos dq
2 2
q
Putting =t
2
dq = 2 dt
When q = 0, t = 0
p
When q = p, t =
2
π
V = 32π a3 ∫ 2 sin 4 t cos 2 t dt
0

3 ⋅1⋅1 π
= 32π a3 ⋅ ⋅
6⋅4⋅2 2
= π 2 a3
(ii) y = a (1 – cosq )
y
= a sin q
dq
The volume of the solid is generated by revolving one arch about the y-axis. For the
region shown, y varies from 0 to 2a, hence q varies from 0 to p.
p dy
V x2 dq
dq
p
= a a q dq
0

= a + q n s 3
q) q
1
=p n (1 sin sin ) d
4

1 q2 1 ˘
= cos q q) + - sin 2q - + cos 2q
2 4
p
1 1
+ -3 cos cos 3q
4 3 0
6.22 Chapter 6 Applications of Integrations

 π2 1 3 1   1 3 1 
= π a3   π 2 − 2 + π 2 − − + −  −2− − + 
 2 4 4 12   4 4 12  
3 8
= π a3  π 2 − 
2 3
(iii) The volume of the solid is generated by revolving one arch about the base which
is the line joining the cusps. If P(x, y) is any point on the curve, its distance from
the base = 2a – y = 2a – a (1 – cosq ) = a(1 + cosq ).
For the region shown, x varies from –ap to ap, hence q varies from –p to p. Due to
symmetry about the y-axis, considering the region in the first quadrant where q varies
from 0 to p,
p dx
V = 2 Ú p ( 2 a - y )2 dq
0 dq
p
p Ú a 2 (1 + cos q )2 a(1 + coss q ) dq
= 22p
0
p
= 2p a 3 Ú (1 + cos q )3 dq
0
3
pÊ qˆ
p a 3 Ú Á 2 cos
= 22p cos2 ˜ dq
0 Ë 2¯
p q
= 16p a 3 Ú co
coss6 dq
0 2
q
Putting = t,
2
dq = 2 dt
When q = 0, t=0
p
When q = p, t =
2
p
V = 16p a 3 Ú 2 2 cos6 t dt
0
p
= 32p a 3 Ú 2 cos6 t dt
0
5 ◊ 3 ◊1 p
= 32p a 3

6◊4◊2 2
= 5p 2 a 3

Example 3
Find the volume of the solid generated by the revolution of the loop of
1
the curve x = t2, y = t − t 3 about the x-axis.
3
6.3 Volume of Solid of Revolution 6.23

Solution
x = t2
dxx
d y
= 2t
dt
The volume of the solid is generated by revolving
the upper half of the loop of the curve about the
A
wn, x varies from 0 to
x-axis. For the region shown,
O (3, 0) x
3, hence t varies from 0 to 3 .
3 dxx
d
V = ∫ π y2 dt
0 dt
2
3 1  Fig. 6.25
= π ∫  t − t 3  2t dt
0  3 
3 2 1 
= 2π ∫  t 2 − t 4 + t 6  t dt
0  3 9 
3 2 1 
= 2π ∫  t 3 − t 5 + t 7  dt
0  3 9 
3
t 4 2 t6 1 t8
= 2π − ⋅ + ⋅
4 3 6 9 8 0

9 9
= 2π  − 3 + 
4 8
3
= π
4

Example 4
Find the volume of the solid of revolution generated by revolving the
3 3
curve x = 2t + 3, y = 4t2 – 9 about the x-axis for t = − to t = .
2 2
Solution
x = 2t + 3
dxx
d
=2
dt
The volume of solid is generated by revolving the curve about the x-axis. For the
3 3
required region, t varies from − to .
2 2
3
dx
V = Ú 23 p y 2 dt
- dt
2
3
= pÚ 2
3 (4t 2 - 9)2 (2) dt
-
2
6.24 Chapter 6 Applications of Integrations

3
= 4p Ú 2 (16t 4 - 72t 2 + 81) dt [∵ (4t 2 – 9)2 is an even function]
0
3
t5 t3 2
= 44p
p 16 − 72 + 81t
5 3 0

= 1296p

Example 5
Prove that the volume of solid generated by revolving the cissoid
sin 3 t
x = 2a sin t, y = 2a
2
about its asymptote is 2p 2a3.
cos t
Solution
sin 3 t
y = 2a
cos t
dyy  3 sinn 2 t cos 2 t + sinn 3 t sin t 
= 2a  
dt  cos 2 t
 3 coss 2 t + sin 2 t 
= 2a sin 2 t  
 cos 2 t
3 cos 2 t + 1 − cos 2 t 
3co
= 2a sin 2 t  
 cos 2 t
 2 cos 2 t + 1
= 2a sin 2 t 
 cos 2 t 

The volume of solid is generated by revolving the region about its asymptote, i.e. the
line x = 2a.
If P(x, y) is any point on the curve, its distance from the asymptote is 2a – x =
2a – 2a sin2 t = 2a cos2 t. For the region shown, y varies from –• to •, hence t varies from
π π
− to . Due to symmetry about the x-axis, considering the region in the first
2 2
y
quadrant where t varies from 0 to π ,
2
p ( , y))
(x
P(x
dy
V = 2Ú 2 p (2 a - x ) 2
dt A
0 dt
O (2a, 0) x
p
Ê 2 cos2 t + 1ˆ
= 2p Ú 2 4 a cos t ◊ 2 a sin t Á
2 4 2
dt
0 Ë cos2 t ˜¯
p x = 2a
= 16p a 3 Ú 2 (2 cos4 t sin 2 t + sin 2 t coss2 t ) dt
0 Fig. 6.26
6.3 Volume of Solid of Revolution 6.25

È 3 ◊1 ◊1 p 1 1 p ˘
= 16p a 3 Í2 ◊ ◊ + ◊ ◊ ˙
Î 6◊4◊2 2 4 2 2 ˚
Ê 6ˆ
= 16p 2 a 3 Á ˜
Ë 48 ¯
= 2p 2 a 3

EXERCISE 6.3
1. For the cycloid x = a (q – sinq ), y = a (1 – cosq ), find the volume of the
solid generated by the revolution of an arch about (i) x-axis, (ii) y-axis,
and (iii) the tangent at the vertex.
[Ans.: 5p 2a 3, 6p 3a 3, p 2a 3]
2. Find the volume formed by revolving one arch of the cycloid x = a(q + sinq ),
y = a(1 + cosq ) about x-axis.
[Ans.: 5p 2a3]
3. Find the volume of the solid formed by revolving the tractrix x = a cos t
t
+ a log tan , y = a sin t about its asymptote.
2  2p a 3 
 Ans.: 
 3 

4. If the ellipse x = a cosq, y = b sinq is revolved about the line x = 2a, show
that the volume of the solid generated is 4p 2a2b.
p
5. The area of the curve x = a cos3q, y = a sin3q lying between q = − and
2
p
q = rotates about the x-axis. Find the volume of solid so generated.
2
 16 3
 Ans.: 105 p a 
 

Polar Form
Example 1
Find the volume of the solid generated by the revolution about the initial
line of the cardioid r = a (1 - cosq ).
Solution
The volume of the solid is generated by revolving the upper half of the cardioid about
the initial line q = 0. For the region above the initial line, q varies from 0 to p.
6.26 Chapter 6 Applications of Integrations

p 2 3
V =∫ p r sin q dq
0 3
2p p 3
3 Ú0
= a (1 - cos q )3 sin q dq p
q=
2
Putting 1 – cosq = t,
sinq dq = dt
When q = 0, t = 0
A
When q = p, t = 2 (2a, 0) O q =0

2p 3 2 3
V= a Ú t dt
3 0
2
2p 3 t 4
= a Fig. 6.27
3 4
0
8
= p a3
3

Example 2
Find the volume of the solid generated by revolving the cardioid
p = a(1 + cos q) about the initial line. [Summer 2017]
Solution
The volume of the solid is generated by revolving the upper half of the cardioid about
the initial line q = 0. For the region above the initial line, q varies from 0 to p.
p 2 3
V=Ú p r sin q d q q=
p
0 3 2
2p p 3
3 Ú0
= a (1 + cos q )3 sin q dq

Putting 1 + cosq = t, (2a, 0)


– sinq dq = dt O A q =0
When q = 0, t = 2
When q = p, t = 0
2p 3 0 3
V= a Ú -t dt Fig. 6.28
3 2
2
2p 3 t 4
= a
3 4
0
6.3 Volume of Solid of Revolution 6.27

2p 3 16
= a ◊
3 4
8
= p a3
3

Example 3
π
Find the volume of revolution of a loop about the line θ = of the curve
2
r2 = a2 cos 2q.
Solution
p
The volume of solid is generated by revolving a loop of the curve about the line q = .
2
p p
For the loop of the curve, q varies from − to . Due to symmetry about the line
4 4
p
q = 0, considering the loop above the initial line where q varies from 0 to ,
p 4
2 3
V = 2Ú 4 p r cos q dq
0 3

q= p
p 3 2
4p q= p
=
3 Ú 0
4 a 3 (cos 2q ) 2 cos q dq 4
p 3
4 3 4
= p a Ú (1 - 2 sin 2 q ) 2 cos q dq
3 0
A
q=0
(a, 0)
Putting 2 sinq = sin t, O
2 cosq dq = cos t dt
When q = 0, t=0
q=− p
p p 4
When q= , t= Fig. 6.29
4 2
p
4 3 2 1
V= p a Ú cos3 t ◊ cos t dt
3 0 2
p
4
= p a 3 Ú 2 cos4 t dt
3 2 0

4 3 ◊1 p
= p a3 ◊ ◊
3 2 4◊2 2
p 2 a3
=
4 2
6.28 Chapter 6 Applications of Integrations

Example 4 q p
2
Find the volume of the solid generated by
revolving the curve r = a + b cos q, (a > b)
about the initial line.
Solution O q=0
The volume of solid is generated by revolving the
upper half of the curve about the initial line. For the
region above the initial line, q varies from 0 to p.
p 2 Fig. 6.30
V r q dq
3
2p p
= a b cos q q dq
3
2p 1 p
- Ú (a b co sin q ) dq
3 b
p
- p cos q )4 [ (q )]n +1 ˘
= ∵Ú [ (q )dq
3 4 n +1 ˚
0

p
[(
6b
p
[{ ]
6b
p
3

Example 5
π
The arch of the cardioid r = a (1 + cos q) included between =− and
2
= is rotated about the line = . Find the volume of the solid of
2 2
revolution.
q= p
2
Solution
The volume of solid is generated by revolving
O q=0
p
the curve about the line q = . For the region
2
p p
shown, q varies from − to . Due to
2 2 Fig. 6.31
6.3 Volume of Solid of Revolution 6.29

symmetry about the initial line, considering the region above the initial line where q
p
varies from 0 to ,
2
p
2
V a cos q q
3
p
4
= +3 + co q q
3
4 31 p 2 1 p
= pa ⋅ +3 ⋅ +1
3 4 2 2 3 2 2
pa
= + 16)
4

Example 6
For the curve r2 = a2 cos 2q, prove that the volume of revolution of a loop
2
a3
about the tangent at the pole is .
8
Solution
The volume is generated by revolving the loop about the tangent at the pole, i.e., the

p p
line q . If P (r, q ) is any point on the curve, its distance from the line q is
4 4
p 1
r sin q , i.e., r(cos q sin )
4 2
For the region shown, q varies from to .
4 4
p
2 1
V ⋅ r q sinq q
4 3 2
p
2p a 4
p − n ) q
3 2 4

2  p p

q sin q dq
3 −
4

4

2  p

−0
3
p 3
2 2
p 2q ) 2 co q q
3 0

p
2 2 2
s q ) 2 cos q q
3 0
6.30 Chapter 6 Applications of Integrations

Putting 2 sinq = sin t,


q p
2 cosq dq = cos t dt 2 p
q
When q = 0, t=0 4
p p
When q = , t=
4 2 P (r, )
p A
2 2 1 q=0
V a co dt O (a, o)
3 0
2
p
2
= pa t dt
3 0
q=− p
2 3 3 1 p
4
3 4 2 2 Fig. 6.32
2
p 3
a
8

Example 7
A solid is formed by rotating the area between two loops of the curve
r = a (1 + 2cosq ) through four right angles. Find the volume generated.
Solution
The volume of solid is generated by rotating the area between two loops of the curve
2p
through four right angles. For the curve ACOBA, q varies from 0 to . For the curve
3
4p q p
BEOB, q varies from p to . 2
3 C
At the pole, r=0
E
1 + 2 cos q = 0
1 O B A q=0
cos q = − F
2
2p D
q =
3
Fig. 6.33
Ê Volume obtained by revolvingˆ Ê Volume obtained by revolvingˆ
V =Á ˜¯ - ÁË ˜¯
Ë the area ACOBA the area BEOB
2p 4p
2 3 2
p a (1 + 2 cos q ) siin q dq - Ú 3 p a 3 (1 + 2 cos q ) sin q dq
3 3
=Ú 3
0 3 p 3
6.3 Volume of Solid of Revolution 6.31

2p
p a3
Ú (1 + 2 cos q ) (-2 sin q ) dq
3
= (-) 3
3 0
4p
p a3
Ú (1 + 2 cos q ) (-2 sin q ) dq
3
+ 3
3 p
2p 4p
p a 3 (1 + 2 cos q )4 3 p a 3 (1 + 2 cos q )4 3
=- +
3 4 3 4
0 p
3 3
p a Ê 81ˆ p a Ê 1 ˆ
=- - + -
3 ÁË 4 ˜¯ 3 ÁË 4 ˜¯
20 3
= pa
3

Example 8
Show that if the area lying within the cardioid r = 2a (1 + cosq ) and
outside the parabola r (1 + cosq ) = 2a revolves about the initial line,
the volume generated is 18p a3.
Solution
The points of intersection of the cardioid q= p
2
r = 2a (1 + cos q) and parabola r (1 + cos q) = 2a
are obtained as, P r = 2a(1 + cos q )
2a
2a
2 a(1 + cos
cos q ) =
1 + cos q
1 + 2 cosq + cos2q = 1 O B A q=0
cosq = 0, cosq = –2 (does not exist)
p
q =± Q
2
r (1 + cos q ) = 2a
p Fig. 6.34
Hence, at P, q =
2
The volume is generated by revolving the region about the initial line. For the regions
p
OBAPO and OBPO, q varies from 0 to .
2
Ê Volume obtained by revolvingˆ Ê Volume obtained by revolvingˆ
V =Á ˜¯ - ÁË ˜¯
Ë the area OBAPO the area OBPO
p p 3
2 2 Ê 2a ˆ
p ÈÎ2 a (1 + cos q )˘˚ sin q dq - Ú 2 p Á
3
=Ú2 sin q dq
0 3 0 3 Ë 1 + cos q ˜¯
6.32 Chapter 6 Applications of Integrations

p p
16 3 2 16
p a Ú (1 + cos q ) (- sin q ) dq + p a 3 Ú 2 (1 + cos q ) (- sin q ) dq
3 -3
=-
3 0 3 0
p p
16 (1 + cos q )4 2 16 (1 + cos q )-2 2
= - p a3 + p a3
3 4 3 -2
0 0

È
Í∵ Ú [ f (q )]n f ¢(q ) dq =
[ f (q )]n+1 ˘˙
Í n +1 ˙
Î ˚
4 8 Ê 3ˆ
= - p a 3 ( -15) - p a 3 Á ˜
3 3 Ë 4¯
= 18p a 3

EXERCISE 6.4
1. Find the volume of solid formed by revolving the curve r = a(1 + cosq )
about the initial line.
È 8 3˘
Í Ans.: 3 p a ˙
Î ˚
2 2
2. Find the volume of solid formed by revolving the curve r = a cos 2q
about (i) the initial line, and (ii) the tangent at the pole.
È ˘
Í Ans.: (i)
p a3 È
6 2 Î
3 (
log 2 + 1 - 2 ˘ ˙
3lo
˚ )
Í ˙
Í ˘˙
Í
ÎÍ
(ii)
p a3 È 3
12 ÍÎ 2
(
log 2 + 1 - 1˙ ˙
˚ ˚˙
)
3. Prove that the volume generated by revolving the loop of the curve
p p
r = a cos 3q lying between q = - to q = about the initial line.
6 6
È 19p a 3 ˘
Í Ans.: ˙
Î 960 ˚
4. Find the volume generated by revolving the curve r = 2a cos q about the
initial line.
È 4p a 3 ˘
Í Ans.: ˙
Î 3 ˚
5. Show that the volume of the solid generated by the revolution of the
2 Ê2 1 ˆ
curve r = a + b sec q about its asymptote is 2p a Á a + bp ˜ .
Ë3 2 ¯
6.4 Volume by Cylindrical Shells 6.33

6.4 VOLUME BY CYLINDRICAL SHELLS


In some cases, it is difficult to find the volume by the previous (Sec 6.3) method. In
such cases, the method of cylindrical shell is applied. If h is the height and r1, r2 are the
inner and outer radii of the shell respectively then
Volume of cylindrical shell = Volume of outer shell − Volume of inner shell
= π r22 h − π r12 h
= π h( r2 + r1 )( r2 − r1 )
r +r 
= 2π  2 1  h( r2 − r1 )
 2 
Let dr = r2 − r1 (thickness of the shell)
r +r
r= 2 1 ((Average radius of shell)
2
Volume = 2π rh dr …(1)
(i) Let the area bounded by the curve y = f (x), the y-axis and the lines y = c and y = d
be rotated about x-axis. An elementary strip PQ parallel to x-axis and of width dy
generates a cylindrical shell of radius y, height PQ and thickness dy, when rotated
about the x-axis.
Volume of cylindrical shell = (2p y) (PQ) dy [Using Eq.(1)]
Summing up the volumes of all cylindrical shells generated by rotation of such strips,
the volume of the solid of revolution is given by,
d
V = ∫ ((22π y )( PQ)dyy
c

y
y

P Q
y=d
x = f((y
(y )

P Q O x
y

O x
y=c

Fig. 6.35

(ii) Let the area bounded by the curve y = f (x), the x-axis and the lines x = a and x = b be
rotated about the y-axis. An elementary strip PQ parallel to y-axis and of width dx
generates a cylindrical shell of radius x, height PQ and thickness dx, when rotated
about the y-axis.
Volume of cylindrical shell = (2p x)(PQ)dx [Using Eq. (1)]
6.34 Chapter 6 Applications of Integrations

Summing up the volumes of all cylindrical shells generated by rotation of such strips,
the volume of solid of revolution is given by,
b
V = Ú (2p x )( PQ)dx [Using Eq. (1)]
a

y
y

Q Q

x=b
x=a
O P x

O P x
x

Fig. 6.36

(iii) Let the area bounded by the curves be rotated about any line. An elementary strip
parallel to the line generates a cylindrical shell of radius r, height h and thickness
dr, when rotated about the y-axis.
Volume of cylindrical shell = (2π r )()(h)dr [using Eq. (1)]
Summing up the volumes of all cylindrical shells generated by rotation of such strips,
the volume of the solid of revolution is given by,
r2
V = Ú ((2
2p r )(h )dr
r1

Example 1
Using the method of cylindrical shells, find the volume of the solid
generated by rotating about the y-axis the region bounded by the curves
1
y = , y = 0, x = 1, x = 2.
x
Solution
The volume is generated by rotating the region about the y-axis.
The height of the shell = PQ
1
=
x
For the region shown, x varies from 1 to 2.
2 1
V = Ú 2p x ◊ dx
1 x
2
= 2p Ú dx
1
6.4 Volume by Cylindrical Shells 6.35

2
= 2p x 1
= 2p (2 - 1)
= 2p
y
y

1
y= Q
x
1
( ,
(x
Q(x )
x
P x
x
O A(1,0) P B(2,0) x
( ,0)
(x
Fig. 6.37

Example 2
Using the method of cylindrical shells, find the volume of the solid
generated by rotating about the y-axis the region bounded by the curves
y = x and y = x2.
Solution
The volume is generated by rotating the region about the y-axis.
The height of the shell = PQ
= x – x2
The points of intersection of y = x and y = x2 are obtained as

x2 = x
x = 0, 1
\ y = 0, 1

The points of intersection are O(0, 0) and A(1, 1) For the region shown, x varies from
0 to 1.
1
V = Ú 2p x( x - x 2 )dx
0
1
= 2p Ú ( x 2 - x 3 )dx
0
1
x3 x 4
= 2p -
3 4
0
6.36 Chapter 6 Applications of Integrations

Ê 1 1ˆ
= 2p Á - ˜
Ë 3 4¯
p
=
6

y
y

Q
A(1, 1)
( , x)
(x
Q y = x2
y=x P

( , x 2)
(x
P(x O x

O x

Fig. 6.38

Example 3
Using the method of cylindrical shells, find the volume of the solid
obtained by rotating about the x-axis the region under the curve y = x
from 0 to 1.
Solution
The volume is generated by rotating the region about the x-axis.
The height of the shell = PQ
=1 − y2
For the region shown, y varies from 0 to 1.
1
V = Ú 2p y(1 - y 2 )dy
0
1
= 2p Ú ( y - y3 )dy
0
1
y2 y 4
= 2p -
2 4
0

Ê 1 1ˆ
= 2p Á - ˜
Ë 2 4¯
p
=
2
6.4 Volume by Cylindrical Shells 6.37

x = y2
P Q
B (1, 1)
( 2, y)
(y
P Q (1, y)
O x
x=1

O A (1, 0) x

Fig. 6.39

Example 4
Using the method of cylindrical shells, find the volume of the solid
generated by rotating about the x-axis the region bounded by the curves
x = 1 + y 2 , x = 0, y = 1, y = 2 .
Solution
The volume is generated by rotating the region about the x-axis.
The height of the shell = PQ
= (1 + y2) − 0
= 1 + y2
For the region shown, y varies from 1 to 2.
2
V = Ú 2p y(1 + y 2 ) dy
1
2
= 2p Ú ( y + y3 ) dy
1
2
y2 y 4
= 2p +
2 4
1

È Ê 1 1ˆ ˘
= 2p Í(2 + 4) - Á + ˜ ˙
Î Ë 2 4¯ ˚
Ê 21ˆ
= 2p Á ˜
Ë 4¯
21
= p
2
6.38 Chapter 6 Applications of Integrations

y=2 P Q
(0, 2) B
C
(0, y) P Q (1 + y2, y)

(0, 1) A D O
y=1 x
O E (1, 0) x

Fig. 6.40

Example 5
Using the method of cylindrical shells f ind the volume of the solid
generated by rotating about the line x = 4 the region bounded by the
curves y = x2, y = 0, x = 1, x = 2.
Solution
The volume is generated by rotating the region about the line x = 4
The height of the shell = PQ
= x2
The radius of the shell = PC
= OC − OP
=4−x
For the region shown, x varies from 1 to 2.
2
V = ∫ 2π (4 − x) ⋅ x 2 dx
1
2
= 2π ∫ (4 x 2 − x 3 )dx
dx
1
2
x3 x 4
= 2π 4 ⋅ −
3 4 1

 32   4 1  
= 2π  − 4 −  −  
 3   3 4 
67
= π
6
6.4 Volume by Cylindrical Shells 6.39

y
x=4

y = x2 Q
x=4
( x2)Q
(x, Q
O x
P
O A P B C x
(1, 0) (x,
( 0) (2, 0) (4, 0)

Fig. 6.41

Example 6
Using the method of cylindrical shells, find the volume of the solid
obtained by rotating about the line y = −1 the region bounded by the
curves y = x2, x = y2.
Solution
The volume is generated by rotating the region about the line y = −1
The height of the shell = PQ
= y − y2
The radius of the shell = QC
= QB + BC
= y + ( −1)
= y −1
The points of intersection of y = x2 and x = y2 are obtained as

x = x4
x(1 − x 3 ) = 0
x = 0, 1
∴ y = 0, 1

The points of intersection are O(0, 0) and A(1, 1).


For the region shown, y varies from 0 to 1.
1
V = Ú 2p ( y - 1) ◊
0
( )
y - y 2 dy

1Ê ˆ
3 1
= 2p Ú Ë y 2 - y3 - y 2 + y 2 ¯ dy
0
6.40 Chapter 6 Applications of Integrations

1
5 3
4 3
2y 2 y y2y 2
= 2p - - +
5 4 3 3 0

Ê 2 1 2 1ˆ
= 2p Á - - + ˜
Ë 5 4 3 3¯
11
=-
60

y
y

x = y2

( 2,y
(y , ) A (1,1)
P
Q (¥ y ,y
, ) Q(¥yy ,y
, )
P
X = ¥y
O x O B x
C y = -1

y = -1

Fig. 6.42

EXERCISE 6.5
1. Using the method of cylindrical shells, find the volume of the solid
generated by rotating about the x-axis the region bounded by the curves
y = 4 x 2 and 2 x + y = 6.
6
 250 
 Ans.: 3 π 
 
2. Using the method of cylindrical shells, find the volume of the solid
generated by rotating about the x-axis the region bounded by y = x3, y = 8
and x = 0.
 768 
 Ans.: 7 π 
 
Multiple Choice Questions 6.41

3. Using the method of cylindrical shells, find the volume of the solid
generated by rotating about the y-axis the region bounded by the curves
y = 2x2 − x3 and y = 0.
 16 
 Ans.: 5 π 
 
4. Using the method of cylindrical shells, find the volume of the solid
generated by rotating about the y-axis the region bounded by the curves
y = 4(x − 2)2 and y = x2 − 4x + 7.
Ans.: 16 π 

5. Using the method of cylindrical shells, find the volume of the solid
generated by rotating about the line x = 1 the region bounded by the
curves y = x 2 , y = 0, x = 1 and x = 2.
 17 
 Ans.: 6 π 
 
6. Using the method of cylindrical shells, find the volume of the solid
generated by rotating about the line y = 3. The region bounded by the
curves y = x − 1, y = 0 and x = 5.
[ Ans.: 24 π ]

MULTIPLE CHOICE QUESTIONS

Choose the correct alternative in the following questions:


1. The volume of the solid obtained by revolving the region bounded by the curves
y = x3, the y-axis and the line y =1 about y-axis is
2p 3p p 4p
(a) (b) (c) (d)
3 5 5 3
2. The volume of the solid formed by the rotation of the area bounded by the curve
p
y = a cos x and the lines x = 0, x = about the x-axis is
2
p 2 a2 3p 2 a 2 p a2 p 2a
(a) (b) (c) (d)
4 4 4 4
3. The volume of the solid generated by revolving the curve y = sin x, 0 £ x £ p
about the x-axis is
p2 p2 p2
(a) p2 (b) (c) (d)
2 4 6
x 2 y2
4. The volume of the solid formed by revolving the ellipse 2 + 2 = 1 about the
major axis is a b
6.42 Chapter 6 Applications of Integrations

2 2 2 4 2 4
(a) pa b (b) p a b2 (c) pa b (d) p a b2
3 3 3 3
x 2 y2
5. If the ellipse + = 1 revolves about the line y = 5, the volume is
9 16
(a) 60p2 (b) 90p2 (c) 120p2 (d) 240p2
6. The volume generated by revolving the cardioid r = a(1 + cos q), 0 £ q £ p is
p 3 2p 3 4p 3 8p 3
(a) a (b) a (c) a (d) a
3 3 3 3
7. The volume generated by the revolution of r = 2a cos q between q = 0 and
p
q= is
2
p a3 2p a 3 4p a 3 4p a 3
(a) (b) (c) (d)
3 3 3 3
8. The curve y = f(x) is rotated about the straight line x = a between y = c and y = d.
The volume so formed is
d d d d
(b) p Ú y dy (c) p Ú (a - x ) dy (d) p Ú (a - y ) dx
2
(a) p Ú y dx 2 2 2

c c c c

9. The volume of the solid generated by revolving the curve x = a cos t, y = b sin t
about the line x = 2a is
2 2 4p a 2 b2
(a) 2pa2b2 (b) 4p a b (c) (d) 4p a 2 b2
3 5
10. The volume of the solid obtained by revolving the area under y = e–2x about the
x-axis is

p p
(a) (b) (c) 2p (d) p
2 4
11. The volume of the solid generated by revolving the area enclosed by the curve x
= a cos3t, y = a sin3 t about the x-axis is
32p a 3 23p a 3 32p 3 a 3 23p 3 a 3
(a) (b) (c) (d)
105 105 105 105
12. The volume of the solid generated by revolving the area enclosed by y = x, y = 0
and x = a about the x-axis is
2 3 p 3
(a) pa3 (b) 2pa3 (c) pa (d) a
3 3
13. The volume of the solid generated by revolving the region between the y-axis
and the curve x = 2 y , 0 £ y £ 4 about y-axis is [Summer 2016]
(a) 2p (b) 32p (c) 16p (d) none of these
Answers 6.43

a a
14. The integral Ú- a f ( x)dx = 2 Ú0 f ( x ) dx if [Summer 2016]
(a) f is an odd function (b) f is neither even nor odd function
(c) f is an even function (d) none of these

Answers

1.(b) 2.(a) 3.(b) 4.(d) 5.(c) 6.(b) 7.(c) 8.(c) 9.(d) 10.(b)
11.(a) 12.(d) 13. (b) 14.(c)
Unit 5
Partial Derivatives
CHAPTER
Partial
7
Derivatives
7.1 INTRODUCTION

We often come across functions which depend on two or more variables. For example,
area of a triangle depends on its base and height, hence we can say that area is the
function of two variables, i.e., its base and height. u is called a function of two variables
x and y, if u has a definite value for every pair of x and y. It is written as u = f (x, y). The
variables x and y are independent variables while u is dependent variable. The set of
all the pairs (x, y) for which u is defined is called the domain of the function. Similarly,
we can define function of more than two variables.

7.2 FUNCTIONS OF TWO OR MORE VARIABLES

The function f (x, y) is called a real-valued function of two or more variables if


there are two or more independent variables, e.g., total surface area of a rectangular
parallelepiped is 2(xy + yz + zx) which is a function of three variables.

Graphs and Level Curves


The level curves of the function z = f (x, y) are two dimensional curves obtained by
substituting, z = k, where k is any number. Hence, the equations of the level curves are
f (x, y) = k.
consider a function. f ( x, y ) == x2 + y2 .
Let z = x 2 + y2 .
Squaring both the sides,
z2 = x2 + y2
The level curves (or contour curves) for this function are found by substituting z = k.
Hence, k2 = x2 + y2, where k is any number. In this case, the level curves are circles
of radius k with centre at the origin. We can graph these in one of two ways. We can
either graph them on the surface itself or we can graph them in a two dimensional axis
system. Hence, graphs for some values of k are shown in Fig. 7.1.
7.2 Chapter 7 Partial Derivatives

5
k=5

4 k=4

3 k=3

2 k=2

1 k=1

–5 –4 –3 –2 –1 0 1 2 3 4 5

–1

–2

–3

–4

–5

Fig. 7.1

7.3 LIMIT AND CONTINUITY OF FUNCTIONS OF


SEVERAL VARIABLES

7.3.1 Limits
If f (x, y) is a function of two variables x, y then
m f ( x, y ) = l
lim
( x , y )→
) (a, b)

if and only if for any chosen number e > 0 however small, there exists a number d > 0
such that
f ( x, y ) − l < e

|
for all values of (x, y) for which, x - a < d and y - b < d| | |
7.3.2 Working Rule for Evaluation of Limit
1. Evaluate limits
(i) lim
m lim { }
lim f ( x, y ) and (ii) lim
x→a y →b
m lim {
lim f ( x, y )
y →b x→a }
If both the limit values are equal, then lim
m f ( x, y ) exists.
( x , y )→
) (a, b)
7.3 Limit and Continuity of Functions of Several Variables 7.3

2. If a = 0, b = 0, evaluate limit along different paths say y = mx or y = mx n, etc.


If all limit values are equal, then lim m f ( x, y ) exists.
( x , y )→
) (a, b)

7.3.3 Theorems on Limit


If lim
m f ( x, y ) = l and lim
m g ( x, y ) = m,
m
( x , y )→ ( a , b ) ( x , y )→ ( a , b )

(i) lim
m [ f ( x, y ) + g ( xx,, y )]
)] = llim f ( x, y ) + llim g ( x, y )
( x , y ) → ( aa,, b ) ( x , y )→ ( a , b ) ( x , y )→
) (a, b)

=l+m
(ii) lim
m [ f ( x, y ) − g ( xx,, y )]
)] = llim f ( x, y ) − llim g ( x, y )
( x , y ) → ( aa,, b ) ( x , y )→ ( a , b ) ( x , y )→
) (a, b)

= l−m
(iii) lim
m [ f ( x, y ) ⋅ g ( xx,, y )]
)] = llim f ( x, y ) ⋅ llim g ( x, y )
( x , y ) → ( aa,, b ) ( x , y )→ ( a , b ) ( x , y )→
) (a, b)

= lm
 f ( x, y )  lim
m f ( x, y ) l
( x , y )→
) (a, b)
(iv) lim  g ( x, y )  = = , provided m ≠ 0
( x , y )→
) (a, b)
  lim
( x y ))→ ( a , b )
g ( x, y ) m

7.3.4 Continuity
Let f (x, y) be a function of x and y defined at (a, b) as well as in the neighbourhood of it.
The function f (x, y) is continuous at (a, b) if the following three conditions are satisfied:
(i) f (a, b) exists, i.e., f (x, y) is defined at (a, b).
(ii) lim m f ( x, y ) exists.
( x , y )→
) (a, b)

(iii) lim
m f ( x, y ) = f ( a , b ) .
( x , y )→
) (a, b)

A function f (x, y) is continuous in a domain if it is continuous at each point of that


domain.
Note:
f
If f (x, y) and g (x, y) are continuous at (a, b), then f + g , f − g , f g , (provided g π 0)
are continuous at (a, b). g

Example 1
x2 + y
Find lim
m .
( x , y )Æ 1, 2 ) 3 x +
) (1, y2
Solution
x2 + y 12 + 2
lim =
) (1, 2 ) 3 x +
( x , y )Æ y2 3(1) + 22
3
=
7
7.4 Chapter 7 Partial Derivatives

Example 2
x− y
Find lim
m .
( x , y )→ ( 0 , 0 ) x+ y
Solution
 x − y
lim
m  llim = lim
lim 1 = 1
x→0  y→0 x + y 
 x→0
 x − y
lim
m llim = lim( −1) = −1
y→0 
 x → 0 x + y  y → 0
Since both the limits are different, the limit does not exist.

Example 3
By considering different paths of approach, show that the function
x4 − y 2
f ( x, y ) = 4 has no limit as ( x, y ) → (0,
0, 0). [Winter 2015]
x + y2
Solution Ê x 4 - y2 ˆ x4
lim Á lim 4 ˜ = lim 4 = lim 1 = 1
Æ00 Ë y Æ 0 x + y ¯
Æ
xxÆ 2 Æ00 x
Æ
xxÆ x Æ0

Ê x 4 - y2 ˆ Ê - y2 ˆ
lim Á lim 4
Æ00 x + y 2 ¯
y Æ 0 Ë xxÆ
˜ yÆ0 Á 2 ˜ = yliÆm(
= lim -1)
1) = -1
Æ Ë y ¯ 0

Since both the limits are different, f (x, y) has no limit as (x, y) Æ (0, 0).

Example 4
xy
Find lim .
( x , y )Æ( 0, 0 ) y - x2
2

Solution
 xy 
lim
m llim 2 = lim
lim 0 = 0
x→0 
 y → 0 y − x 2  x → 0
 xy 
lim
m llim 2 = lim 0 = 0
y→0 
 x → 0 y − x 2  y → 0

Putting y = mx and taking limit x Æ 0,

x(mx)
x( m m
lim = lim =
→0

xx→ 0 mx) 2 − x 2 x → 0 m 2 − 1 m 2 − 1
((mx
Since the last limit depends on m and m is not fixed, the limit does not exist.
7.3 Limit and Continuity of Functions of Several Variables 7.5

Example 5
x2 y
Find lim .
) ( x y
Solution x2 y 
lim lim
x y24 x→0

x2 y
lim lim
x y24 y 0

Putting y = mx and taking limit x Æ 0,


x2 ) mx
lim 4 2
= lim =0
x x→0 x2 m2
Putting y = mx2 and taking limit x Æ 0,
x2 ) m m
lim lim
x 4 2 x→0 1+ m m2
Since the last limit depends on m and m is not fixed, the limit does not exist.

Example 6
Show that f (x, y) = x2 + 2y is continuous at (1, 2).
Solution
lim ( x, m x 2) 5
( , y) (1

f ( 2) + 2(2 5
lim ( x, ) = f (1 2)
( , y )→ (1

Hence, f (x, y) is continuous at (1, 2).

Example 7
Show that , )
2)
is discontinuous at (1, 2).

Solution
lim ( x, m x +2= 4
( , y) (1

f ( 2) 0
lim ( x, ) ≠ f (1 2)
( , y) (1 2 )

Hence, y) is discontinuous at (1, 2).


7.6 Chapter 7 Partial Derivatives

Example 8 2 2

Discuss the continuity of y) , ) (


x y
0)
at (0, 0).
Solution
x2 y2
lim lim f ( , y ) lim lim
x →0 x → x2 y2
2
x
lim 2
= =1
x →0

x2 y2
lim lim f x, y ) lim lim
y →0 y →0 x y2
2
− = −1 1
y →0

Since both the limits are not equal, y) is discontinuous at (0, 0).

Example 9
Determine the set of points at which the given function is continuous:
Ï 3x2 y
Ô , ( x, y) π (0, 0)
f ( x, y ) = Ì x 2 + y 2
Ô 0, ( x, y) = (0, 0)
Ó [Winter 2013]
Solution
3x2 y
For (x, y) π (0, 0), the function f ( x, y) = is a rational function and hence, it
x 2 + y2
is continuous.

For (x, y) = (0, 0), f(x, y) = 0


˘ 3x2 y
lim lim f ( , y) = lim lim 2
x Æ0 x Æ0 x y2
= lim 0 0

Ê 3x2 y
lim È lim f ( , y)˘ = lim lim 2
yÆ yÆ 0 x y2
= 0=0
yÆ0
7.3 Limit and Continuity of Functions of Several Variables 7.7

Putting y = mx and taking limit x Æ 0,


3 x 2 (mx ) 3x3 m 3mx
lim = lim = lim =0
x Æ0 x +m x
2 2 2 x Æ0 x (1 + m )
2 2 x Æ0 1 + m 2

Hence, the limit exists at (0, 0).


lim f ( x, y) = f (0, 0)
( x , y ) Æ ( 0 ,0 )

Hence, f(x, y) is continuous at (0, 0).

Example 10
2 xy
Show that f ( x, y ) = y ) ≠ (0, 0)
, ( x, y)
x + y2 2

= 0, ( x, y ) = (0, 0)
is continuous at every point except at the origin. [Summer 2017]
Solution
 2 xy 
m lim f ( x, y )  = lim  lim 2
lim
→0

xx→  y→0
0  xx→
→0
→ 0  y→0 x + y 2 

= lim 0 = 0
x→0

 2 xy 
m lim f ( x, y )  = lim  lim 2
lim
y→
→00  x→0  y→ →00  x→0 x + y 2 

= lim 0 = 0
y→0

Putting y = mx and taking limit x Æ 0,


2 x(mx) 2m 2m
lim = li
lim =
x→0 x 2 + (mx) 2 x → 0 1 + m 2 1 + m 2
Since the last limit depends on m and m is not fixed, the limit does not exist.
Hence, f ( x) is discontinuous at the origin, i.e., (0, 0).
Let ( x, y) = (aa,, b) π (0, 0) be an arbitrary point in xy-plane, where a and b are real
numbers.
2 xy
limm f ( x, y ) = lim
( x , y )→ ( a
a,, b ) ( x , y )→ ( a , b ) x 2 + y 2

2ab
= 2
a + b2
2ab
f ( a, b) = 2
a + b2
which is finite for real values of a and b.
m f ( x, y ) = f ( a , b )
lim
( x , y )→
) (a, b)

This shows that f ( x, y ) is continuous at (a, b).


Hence, f ( x, y ) is continuous at every point except at the origin.
7.8 Chapter 7 Partial Derivatives

Example 11
Ï 2 x2 y
Ô ( x, y ) π 0
Show that f ( x, y) = Ì x 3 + y3
Ô 0 ( x, y ) = 0
Ó
is not continuous at the origin. [Winter 2016]
Solution
È ˘ Ê 2 x2 y ˆ
lim Í lim f ( x, y)˙ = lim Á lim 3 ˜
xÆ0 ÎyÆ0 ˚ x Æ 0 Ë y Æ 0 x + y3 ¯
= lim (0) = 0
xÆ0

È ˘ Ê 2 x2 y ˆ
lim Í lim f ( x, y)˙ = lim Á lim 3 ˜
yÆ0 ÎxÆ0 ˚ y Æ 0 Ë x Æ 0 x + y3 ¯
= lim (0) = 0
yÆ0

Putting y = mx and taking limit x Æ 0,


2 x 2 (mx ) 2 mx 3
lim = lim
xÆ0 x 3 + (mx )3 x Æ 0 x 3 (1 + m3 )
2m
=
1 + m3
Since the last limit depends on m and m is not fixed, the limit does not exist.
Hence, f(x) is discontinuous at the origin.

Example 12
Determine the continuity of the function
Ï 2 Ê 1 ˆ
Ô( x + y )sin Á 2 ( x, y) π (0,0)
2
,
f ( x, y ) = Ì Ë x + y 2 ˜¯
Ô
Ó0, ( x, y) = (0,0)
at origin. [Summer 2016]
Solution

È ˘ È Ê 1 ˆ˘
lim Í lim f ( x, y)˙ = lim Í lim ( x 2 + y 2 )sin Á 2 ˜˙
x Æ0 Î yÆ0 ˚ x Æ0 ÎÍ yÆ0 Ë x + y 2 ¯ ˚˙
7.3 Limit and Continuity of Functions of Several Variables 7.9

È Ê 1 ˆ˘
= lim Í x 2 sin Á 2 ˜ ˙
x Æ0 Î Ë x ¯˚
È Ê 1 ˆ˘
Í sin ÁË 2 ˜¯ ˙
x ˙
= lim Í
x Æ0 Í 1 ˙
Í x2 ˙
Î ˚
=1
È Ê 1 ˆ˘
lim È lim f ( x, y)˘ = lim Í lim ( x 2 + y 2 )sin Á 2 ˜˙
Í x Æ0
yÆ0 Î ˚˙ yÆ0 ÎÍ x Æ0 Ë x + y 2 ¯ ˚˙
È Ê 1 ˆ˘
= lim Í y 2 sin Á 2 ˜ ˙
yÆ0 Í
Î Ë y ¯ ˚˙
È Ê 1 ˆ˘
Í sin Á 2 ˜ ˙
Í Ëy ¯˙
= lim Í ˙
yÆ0 1
Í 2
˙
Í y ˙
Î ˚
=1
Putting y = mx and taking limit x Æ 0,
Ê 1 ˆ Ê 1 ˆ
lim ( x 2 + m 2 x 2 ) sin Á 2 ˜ = (1 + m 2 ) sin Á
x Æ0 Ë x +m x ¯
2 2 Ë 1 + m 2 ˜¯

Since the last limit depends on m and m is not fixed, the limit does not exist. Hence,
f (x) is discontinuous at origin (0, 0).
Let (x, y) = (a, b) be an arbitrary point in xy-plane, where a and b are real numbers.
Ê 1 ˆ
lim f ( x, y ) = lim ( x 2 + y 2 )sin Á 2 ˜
( x , y )Æ ( a , b ) ( x , y )Æ ( a , b ) Ë x + y2 ¯
Ê 1 ˆ
= (a 2 + b2 )sin Á 2
Ë a + b2 ˜¯
Ê 1 ˆ
f (a, b) = (a 2 + b2 )sin Á 2
Ë a + b2 ˜¯

which is finite for real values of a and b.


lim f ( x, y) = f (a, b)
( x , y )Æ ( a , b )

This shows that f (x, y) is continuous at (a, b).


Hence, f(x, y) is continuous at every point except at the origin.
7.10 Chapter 7 Partial Derivatives

Example 13
xy
Show that f ( x, y) = , ( x, y) π (0, 0)
x 2 + y2
= 0, ( x, y) = (0, 0)
is continuous at the origin. [Summer 2014]
Solution
xy
For (x, y) π (0, 0), the function f ( x, y) = is a rational function and hence,
x + y2
2
it is continuous.
For (x, y) = (0, 0), f(x, y) = 0
È ˘ Ê xy ˆ
lim lim f ( x, y)˙ = lim lim
Æ0
Æ 0Í
Î yÆ0 ˚ Æ0
Æ Á Æ ˜
xxÆ xÆ
x 0
Ë
y 0
x +y ¯
2 2

= lim 0 = 0
x Æ0

Ê ˆ
lim È lim f ( x, y)˘ = lim lim
xy

Æ0 Í x Æ0
0Î ˚˙ yÆÆ00 Á x Æ0 2 ˜
Ë x +y ¯
2

= lim 0 = 0
yÆ0

Putting y = mx and taking limit x Æ 0,

x(mx ) mx 2Ê m ˆ
lim = lim = lim Á ˜x=0
x Æ0 x Æ0
x 2 + m2 x 2 x 1 + m 2 x Æ0 Ë 1 + m 2 ¯
The limit exists at the origin.
Hence, f(x, y) is continuous at the origin.

EXERCISE 7.1
1. Evaluate the following limits:
3x 2 y xy + 4 x+y
(i) lim (ii) lim (iii) lim
( x , y )→(1, 2) x + y 2 + 5
2
( x , y )→( ∞, 2) x 2 + 2y 2 ( x , y )→( 0, 0) x + 2y
1

x 2 ( y −1)2 2x − y
(iv) lim e (v) lim
( x , y )→( 0, 1) ( x , y )→( 0, 0) x2 + y2
 3 
 Ans. : ((i)
i)
5
(ii) 0 (iii) does not exist 
 
 (iv) 0 (v) does not exist 
7.5 Higher-Order Partial Derivatives 7.11

2x − y
2. Show that for f (x, y ) = m llim f (x, y ) ≠ llim
, lim im lim x, y )
lim f ((x
2 x + y x →0  y →0  y →0  x →0 

3. Check the continuity of the following functions:


x
(i) f (x, y ) = at (0, 0)
3 x + 5y
xy
(ii) f (x, y ) = y ) ≠ (0, 0)
, (x, y)
x + y2
2

= 0, (x, y ) = (0, 0)
at origin.
x 2y 2
(iii) f (x, y ) = at (0, 0)).
x4 + y4
 Ans.: (i) Discontinuous (ii) Discontinuous
 
 (iii) Discontinuous 

7.4 PARTIAL DERIVATIVES


A partial derivative of a function of several variables is the ordinary derivative w.r.t.
one of the variables, when all the remaining variables are kept constant. consider a
function u = f (x, y). Here, u is the dependent variable and x and y are independent
variables. The partial derivative of u = f (x, y) w.r.t. x is the ordinary derivative of u
∂u ∂f
∂f
w.r.t. x, keeping y constant. It is denoted by or or ux or fx and is known as
∂x ∂x
first-order partial derivative of u w.r.t. x.
∂u  f ( x + ∆ x, y ) − f ( x, y ) 
= lim  
∂x ∆ x → 0
 ∆x 
Similarly, the partial derivative of u = f (x, y) w.r.t. y is the ordinary derivative of u w.r.t.
∂u ∂f
∂f
y treating x as constant. It is denoted by or or uy or fy and is known as first-
∂y
∂y ∂y
∂y
order partial derivative of u w.r.t. y.

∂u  f ( x, y + ∆y ) − f ( x, y ) 
= lim  
∂y
∂y ∆y
y → 0
 ∆y 

7.5 HIGHER-ORDER PARTIAL DERIVATIVES


Partial derivatives of higher order, of a function u = f (x, y), are obtained by partial dif-
ferentiation of first-order partial derivative. Thus, if u = f (x, y) then
7.12 Chapter 7 Partial Derivatives

∂ 2 u ∂  ∂u 
=  
∂x 2 ∂x  ∂x 
∂2u ∂  ∂u 
=  
∂y ∂x ∂∂yy  ∂x 

∂ 2 u ∂  ∂u 
=
∂y 2 ∂∂yy  ∂y
∂y ∂y 
∂2u ∂  ∂u 
=
∂x ∂y ∂x  ∂y
∂y 

are called second-order partial derivatives. Similarly, other higher-order derivatives


can also be obtained.
Clairaut's Theorem
∂2u ∂2u
If u = f (x, y) possesses continuous second-order partial derivatives and
∂x ∂y ∂y ∂x
∂2u ∂2u
then = . This is also called commutative property.
∂x ∂y ∂y ∂x
Note: Standard rules for derivatives of sum, difference, product and quotient are also
applicable for partial derivatives.

Example 1
If f((x, y) = x2y + xy2 then find fx(1, 2) and fy(1, 2) by definition.
[Summer 2017]

Solution
f((x, y) = x2y + xy2
∂f È f ( x + Dx, yy)) - f ( x, y) ˘
fx = = lim
∂x Dxx Æ 0 ÍÎ Dx ˙
˚
[( x + Dx )2 y + ( x + Dx ) y 2 ] - x 2 y - xy 2
= lim
Dx Æ 0
Dx Dx
Dx
x 2 y + 2 x Dxxyy + ( Dx )2 y + xxyy 2 + Dxxyy 2 - x 2 y - xxyy 2
= lim
Dx Æ 0
Dx Dx
Dx
2 x Dxxyy + ( Dx )2 y + Dxxyy 2
= lim
Dx Æ 0
Dx Dx
Dx
= lim (2 xy + y 2 ) + Dxxyy
Dx Æ 0
Dx
7.5 Higher-Order Partial Derivatives 7.13

∂f
fx = = 2 xy + y 2
∂x
f x (1, 2) = 2(1)(2) + (2)2 = 4 + 4 = 8
∂f È f ( x, y + Dy) - f ( x, y) ˘
fy = = lim Í ˙
∂y Dy Æ 0 Î Dy ˚
[ x 2 ( y + Dy) + x( y + Dy)2 ] - x 2 y - xy 2
= lim
Dy Æ 0 Dy
x 2 y + x 2 Dy + xy 2 + 2 xy Dy + x( Dy)2 - x 2 y - xy 2
= lim
Dy Æ 0 Dy
2 xy( Dy) + x 2 ( Dy) + x( Dy)2
= lim
Dy Æ 0 Dy
= lim (2 xy + x 2 ) + x( Dy)
Dy Æ 0

∂f
fy = = 2 xy + x 2
∂y
fy(1, 2) = 2(1)(2) + (1)2 = 4 +1 = 5

Example 2
1
− ∂u ∂u
If u = (1 − 2 xxyy + y ) −y = u3 y2.
2 2
then show that x
∂x ∂y
Solution
1

u = ((11 − 2 xxyy + y 2 ) 2

Differentiating u partially w.r.t. x and y,

∂u 1 −
3
= − (1 − 2 xy + y 2 ) 2 ( −2
−2 y )
∂x 2
∂u 1 −
3
= − (1 − 2 xy + y 2 ) 2 ( −
−22 x + 2 y )
∂y
∂y 2

∂u ∂u −
3
Hence, x −y = ((1
1 − 2 xy + y 2 ) 2 ( xxyy − xxyy + y 2 )
∂x ∂y
3
 − 
1
= ((1
1 − 2 xy + y 2 ) 2  y 2
= u3 y2
7.14 Chapter 7 Partial Derivatives

Example 3
If u = log (tan x + tan y + tan z) then show that
∂u ∂u ∂u
sinn 2 x + ssin
in 2 y + sinn 2 z = 2.
∂x ∂y ∂z

Solution
u = log (tan x + tan y + tan z)
Differentiating u partially w.r.t. x, y and z,
∂u 1
= ⋅ sec2 x
∂x tan x + tan y + tan z
∂u 1
= ⋅ sec2 y
∂∂yy tan x + tan y + tan z
∂u 1
= ⋅ sec2 z
∂z ttaan x + tan
tan y + tan z

∂u ∂u ∂u
Hence, sinn 2 x + ssin
in 2 y + sin 2 z
∂x ∂y ∂z
2si
2 sin
n x cos sec 2 x + 2 ssin
cos x sec in y cos sec 2 y + 2 sin z cos z sec 2 z
cos y sec
=
n x + tan y + tan z
tan
2(tan x + tan y + tan z )
=
n x + tan y + tan z
tan
=2

Example 4
e x+ y+ z ∂u ∂u ∂u
If u = , show that + + = 2u. [Summer 2017]
e +e +e
x y z
∂x ∂∂yy ∂z
Solution
ex+ y+ z
u=
ex + e y + ez
Differentiating u partially w.r.t. x,

∂u ex+ y+ z ex+ y+ z
= x − ⋅ ex
∂x e + e y + e z ( e x + e y + e z ) 2
ex+ y+ z  ex 
= 1 −
 e x + e y + e z  …(1)
ex + e y + ez
7.5 Higher-Order Partial Derivatives 7.15

∂u ex+ y+ z  ey 
Similarly, = x 1 −
 e x + e y + e z  …(2)
∂y e + e y + e z
∂y

∂u ex+ y+ z  ez 
= x 1 −
 e x + e y + e z  …(3)
∂z e + e y + e z

Adding Eqs (1), (2) and (3),

∂u ∂u ∂u ex+ y+ z  ex + e y + ez 
+ + = x  3 − e x + e y + e z 
∂x ∂y
∂y ∂z e + e y + e z
ex+ y+ z
= 3 − 1)
((3
ex + e y + ez
= 2u

Example 5
2
x2 + y 2  ∂u ∂u   ∂u ∂u 
If u = , show that  −  = 4 1 − − .
x+ y  ∂x ∂y
∂y   ∂x ∂y
∂y 
Solution
x2 + y 2
u=
x+ y
u( x + y) = x2 + y 2 …(1)
Differentiating Eq. (1) partially w.r.t. x,
∂u
u + ( x + y) = 2x
∂x
∂u 2 x − u
=
∂x x+ y
Differentiating Eq. (1) partially w.r.t. y,
∂u
u + ( x + y) = 2y
∂y
∂u 2 y − u
=
∂y x+ y
2 2
 ∂u ∂u   2x − u 2 y − u 
LHS =  −  =  −
 ∂x ∂∂yy   x+ y x + y 
2
 2( x − y ) 
=  …(2)
 ( x + y) 
7.16 Chapter 7 Partial Derivatives

 ∂u ∂u   2x − u 2 y − u 
RHS = 4 1 − − = 4 1 − −
 ∂x ∂∂yy   x+ y x + y 
 2x − u + 2 y − u 
= 4 1 − 
 x+ y
 2( x + y ) 2u 
= 4 1 − +
 ( x + y ) ( x + y ) 
  x2 + y 2 
= 4 1 − 2 + 2  2 
  ( x + y) 
 −( x + y )2 + 2 x 2 + 2 y 2 
= 4 
 ( x + y)2 
44(((xx + y − 2 xy )
2 2
=
((xx + y ) 2
2
 2((xx − y ) 
=  …(3)
 ((xx + y ) 
From Eqs (1) and (2),
2
 ∂u ∂u   ∂u ∂u 
 ∂x − ∂∂yy  = 4 1 − ∂x − ∂∂yy 

Example 6  2 
1 − x 
−  na 2t 
Find the value of n for which u = kktt e 2
satisfies the equation
∂u ∂ 2u
= a2 2 .
∂t ∂x
Solution
 x2 

1 − 2 
 na t 
u = ktt e 2

Differentiating u partially w.r.t. t,


 x2   x2 
∂u 1 − 3 − 2  2   x2 
1 −

= − kt 2 e  na t  + k t 2 e  na t   2 2 
∂t 2  na t 
1 x2
= − t −1u + u 2 2
2 na t
 x2 1
= u 2 2 −  ...(1)
 na t 2t 
7.5 Higher-Order Partial Derivatives 7.17

Differentiating u partially w.r.t. x,


 x2 
∂u −
1 −
2   2x 
= kt 2 e  na t   − 2 
∂x  na t 

∂u
Differentiating partially w.r.t. x,
∂x
1
kt 2  
−  x2   x2 
∂2u − 2 
 na t   2x  − 2 
= − 2 2x ⋅ e  − 2  + e
 na t 
( )
(2
∂x 2
na t  na t 
 
2  2x 2

= − 2  − 2 u + u
na t  na t 

 4x2 2 
= u 2 4 2 − 2 
n a t a t
na
n
∂2u  4x2 2
a2 = u  2 2 2 −  …(2)
∂x 2
nat nt
From Eqs (1) and (2),
n=4

Example 7
Aee − gx sin(nt
Find the value of n for which v = A nt − gx
g ) satisfies the partial

differential equation ∂v = ∂ v2 where g, A are constants.


2

∂t ∂x
Solution
Aee − gx sin(n
v= A ntt − g
gx)

Differentiating v partially w.r.t. t,

∂v
= Ae − gx [cos(nt − gx)] ⋅ n
∂t
Differentiating v partially w.r.t. x,

∂v
= − Agee − gx sin(nntt − ggxx) + [ Ae − gx cos( nntt − ggxx)]( − g )
∂x
= − Agee − gx [sin(nt
nt − ggxx) + cos(nt nt − ggxx)]
7.18 Chapter 7 Partial Derivatives

∂v
Differentiating partially w.r.t. x,
∂x

∂2v
= Ag 2 e − gx [sin(nt − ggxx) + cos( nt − gx
gx)]
∂x 2
− Age − gx [ − g cos((nt − gx) + g sin(nt − gx
g )]
= Ag 2 e − gx ⋅ 2 cos( ntt − gx
g )
∂v ∂ 2 v
Also, =
∂t ∂x 2
n ⋅ A ⋅ e − gx cos( ntt − g
gxxx)) = 2 Ag 2 e − gx cos((nt − gx)
Ag
∴ n = 2g 2

Example 8
y ∂ 2u
If u = e , find
x
.
∂y ∂x
Solution
y
u = ex
Differentiating u partially w.r.t. x,
∂u y ∂ y
= ex (x )
∂x ∂x
y
yx y −1
= e x ⋅ yx
∂u
Differentiating partially w.r.t. y,
∂x
∂  ∂u  xy ∂ y y ∂
  = e ( x y ) ⋅ yyxx y −−11 + e x x y −1 + e x y ( x y −1 )
∂y ∂x
∂y ∂y
∂y ∂y
∂y
∂2u y y y
= e x x y log x ⋅ yyxx y −−11 + e x x y −1 + e x yx y −1 log x
∂y ∂x
∂y
y
= e x x y −1 ( yx y log x + 1 + y log x)

Example 9
∂2 z
If z 3 − zzxx − y = 0, find .
∂x ∂y
Solution
z 3 − zzxx − y = 0 ...(1)
7.5 Higher-Order Partial Derivatives 7.19

Differentiating Eq. (1) partially w.r.t. y,


∂z ∂z
3z 2
3 − x −1 = 0
∂y ∂y
∂z 1
= 2
∂y 3 z − x
Differentiating Eq. (1) partially w.r.t. x,
∂z ∂z
3z 2 −x −z=0
∂x ∂x
∂z z
=
∂x 3 z 2 − x
∂z
Differentiating partially w.r.t. x,
∂∂yy
∂2 z 1  ∂z 
=− 6z − 1
 6
∂x ∂y 3z 2 − x) 2
(3z ∂x 
1  6z2 
=− 2 
− 1
3z − x)  3z − x 
(3z 2 2

3z 2 + x
=−
(3 z 2 − x)3

Example 10
 xy  ∂ 2u 1
 , show that ∂x ∂y =
−1
If u = tann  3
.
 1+ x + y 
2 2
(1 + x + y ) 2
2 2

Solution
 xy 
u = tan −1  
 1+ x + y 
2 2

Differentiating u partially w.r.t. y,

∂u 1 ∂  xy 
=  
∂x x2 y 2 ∂x  1 + x 2 + y 2 
1+
1 + x2 + y 2
 1 
1 + x 2 + y 2 y − xy
xy ⋅ 2x
1+ x + y 2  2
2 1+ x + y
2 2 
=  
1+ x + y + x y 
2 2 2 2
1+ x + y
2 2

7.20 Chapter 7 Partial Derivatives

1 + x2 + y 2  (1 + x 2 + y 2 − x 2 ) y 
=  
+ x 2 + y 2 (1 + x 2 )  (11+ 
 + x + y ) 1+ x + y
11+ 2 2 2 2

((1 + y 2 ) y
=
+ x 2 ))((1 + y 2 ) 1 + x 2 + y 2
((111+
y
=
(1 + x ) 2
1 + x2 + y 2

Differentiating ∂u partially w.r.t. y,


∂x
∂ 2u ∂  y 
=  
∂x ∂y ∂∂yy  (1 + x ) 1 + x + y 
2 2 2
 1+

1 ∂  y 
=  
1 + x ∂y
2
∂y  1 + x + y 
2 2

 2y 
 1
1++ x 2 + y 2 (1) − y
2 
1  2 1+ x + y 
2

=
1 + x 2  1 + x2 + y 2 

1 È 1 + x 2 + y2 - y2 ˘
= Í ˙
1 + x 2 Í (1 + x 2 + y2 ) 1 + x 2 + y2 ˙
Î 1+ ˚
1
= 3
(1 + x + 2 2 2
y )

Example 11
∂3u
xyz
If u = e , show that = (1 + 3 xyz + x 2 y 2 z 2 )e xyz .
∂x ∂y ∂z
Solution
u = e xyz
Differentiating u partially w.r.t. z,
∂u
= e xyz ⋅ xy
xy
∂z
∂u
Differentiating partially w.r.t. y,
∂z
∂  ∂u  ∂2u
  = = xe xyz + x 2 yze
y xyz
∂y ∂z
∂y ∂y ∂z
7.5 Higher-Order Partial Derivatives 7.21

∂2u
Differentiating partially w.r.t. x,
∂y ∂z
∂  ∂2u 
= e xyz + xyze xyz + 2 xyze xyz + x 2 y 2 z 2 e xyz
∂x  ∂y ∂z 
∂ 3u
= ((1
1 + 3 xyz + x 2 y 2 z 2 )e xyz
∂x ∂y ∂z

Example 12
∂ 3u
If u = e3xyz, show that = (3 + 27 xyz + 27 x 2 y 2 z 2 )e3 xyz .
∂x∂y∂z
[Summer 2014]

Solution
u = e3xyz
Differentiating u partially w.r.t. z,
∂u
= 3 xy e3 xyz
∂z
∂u
Differentiating partially w.r.t. y,
∂z
∂2 u
∂ y∂ z
(
= 3 x e3 xyz + ye3 xy ◊ 3 xz )
= e3 xyz (3 x + 9 x 2 yz )

∂2 u
Differentiating partially w.r.t. x,
∂ y∂ z
∂3u
= e3 xyz (3 + 18 xyz ) + (3 x + 9 x 2 yz )e3 xyz ◊ 3 yz
∂ x ∂ y∂ z
= e3 xyz (3 + 18 xyz + 9 xyz + 27 x 2 y 2 z 2 )
= e3 xyz (3 + 27 xyz + 27 x 2 y 2 z 2 )

Example 13
∂ 2u ∂ 2u
If u = log( x 2 + y 2 ), verify that = .
∂x ∂y ∂y ∂x
7.22 Chapter 7 Partial Derivatives

Solution
u = log( x 2 + y 2 )
Differentiating u partially w.r.t. x,
∂u 1 2x
= 2 (2 x) = 2
∂x x + y 2
x + y2
Differentiating u partially w.r.t. y,
∂u 1 2y
= (2 y ) = 2
∂y x 2 + y 2
∂y x + y2
∂u
Differentiating partially w.r.t. y,
∂x

∂2u  1  4 xy
= 2x − 2 2 2
2y = − 2 …(1)
∂y ∂x  (x + y )  ( x + y 2 )2
∂u
Differentiating partially w.r.t. x,
∂y
∂y

∂2u  1  4 xy
= 2 y − 2 2 2
2x = − 2 …(2)
∂x ∂y  (x + y )  ( x + y 2 )2
From Eqs (1) and (2),
∂2u ∂ 2u
=
∂x ∂y ∂y ∂x

Example 14
∂ 2u ∂ 2u
=
2
If u = x3 y + e xy , prove that .
∂x ∂y ∂y ∂x
Solution
2
u = x 3 y + e xy
Differentiating u partially w.r.t. y,
∂u 2
= x 3 + e xy ⋅ 2 xy
∂y
∂y
∂u
Differentiating partially w.r.t. x,
∂y
∂y
∂  ∂u  2 2

  = 3 x 2 + 2 yye xy + 2 xye xy ⋅ y 2
∂x  ∂y
∂y 
∂2u 2
= 3 x 2 + 2 yyee xy (1 + xy
xy 2 ) …(1)
∂x ∂y
7.5 Higher-Order Partial Derivatives 7.23

Differentiating u partially w.r.t. x,


∂u 2
= 3 x 2 y + e xy ⋅ y 2
∂x
∂u
Differentiating partially w.r.t. y,
∂x
∂  ∂u  xy 2 2 xy 2
  = 3 x + 2 yyee + y e ⋅ 2 xy
2
xy
∂∂yy  ∂x 
∂2u 2
= 3 x 2 + 2 ye xy (1 + xxyy 2 ) ...(2)
∂y ∂x
From Eqs (1) and (2),
∂2u ∂ 2u
=
∂y ∂x ∂x ∂y

Example 15
If z = x + y x, prove that ∂ z = ∂ z .
2 2
[Winter 2013]
∂x ∂y ∂y ∂x
Solution
z = x + yx
Differentiating z partially w.r.t. x,
∂z
= 1 + y x log y
∂x
Differentiating z partially w.r.t. y,
∂z
= xy x -1
∂y
∂z
Differentiating partially w.r.t. y,
∂x
∂2 z 1
xy x -1
= y x ◊ + log y ◊ xy
∂y ∂x y
= y x --11 + x log y( y x -1 )
= y x -1 (1 + x log y) ...(1)
∂z
Differentiating partially w.r.t. x,
∂y
∂2 z
= y x --11 ◊1 + xxyy x -1 log y
∂x ∂y
= y x -1 (1 + x log y) ...(2)
7.24 Chapter 7 Partial Derivatives

From Eqs (1) and (2),

∂2 z 2
z
=
∂ x ∂ y

Example 16
∂2 z 2
z
If z = xy + y x, prove that =
∂ y x
Solution
x
z x
x
z e
=e x y

Differentiating z partially w.r.t. y,


∂z x
x
⋅ ⋅
y y
∂z
Differentiating partially w.r.t. x,
y

∂ ∂z 1 x
= y x
⋅ ⋅ + ⋅ y⋅
∂x y x x y y
∂2 z ey x x y
= +1 + +x y) …(1)
∂ y x y
Differentiating z partially w.r.t. x,
∂z
= y x
⋅ ⋅ y
∂x x
∂z
Differentiating partially w.r.t. y,
∂x
∂ ∂z 1 y x
y x) + ⋅ ⋅
y ∂x x y y
∂ z
2
e y x x y
1) …(2)
x x y
From Eqs (1) and (2),
∂2 z z 2

x ∂ y
7.5 Higher-Order Partial Derivatives 7.25

Example 17
3
∂ 2u ∂ 2u
If u = (3 xxyy − y ) − ( y − 2 x) , show that ∂x ∂y = ∂y ∂x .
3 2 2

Solution
3
u = (3 xy
xy − y 3 ) − ( y 2 − 2 x) 2
Differentiating u partially w.r.t. y,
∂u 3 1
= 3x − 3 y 2 − ( y 2 − 2 x) 2 (2 y )
∂y
∂y 2
∂u
Differentiating partially w.r.t. x,
∂y
∂y
∂  ∂u  3y −
1

  = 3 − ( y 2 − 2 x) 2 ( −2)
∂x  ∂y
∂y  2
∂2u 3y
= 3+ …(1)
∂x ∂y y − 2x
2

Differentiating u partially w.r.t. x,


∂u 3 1
= 3 y − ( y 2 − 2 x) 2 ( −2)
∂x 2
1
= 3 y + 3( y 2 − 2 x) 2
∂u
Differentiating partially w.r.t. y,
∂x
∂  ∂u  3 2 −
1

  = 3 + ( y − 2 x ) 2
(2 y )
∂y  ∂x 
∂y 2
∂2u 3y
= 3+ …(2)
∂y ∂x y − 2x
2

From Eqs (1) and (2),


∂2u ∂2u
=
∂x ∂y ∂y ∂x

Example 18
 y  x
If z = x 2 tan −1   − y 2 tan −1   , prove that ∂ z = ∂ z = x 2 − y 2 .
2 2 2 2

 x  y ∂x ∂y ∂y ∂x x + y
7.26 Chapter 7 Partial Derivatives

Solution
 y  x
z = x 2 tan −1   − y 2 tan −1  
 x  y
Differentiating z partially w.r.t. y,
∂z 1  1 1  x −1  x 
= x2 2  
− y2  − y 2  − 2 y tan  y 
∂y y  x x2
1+ 2 1+ 2
x y
x3 xy 2  x
= + − 2 y tan −1  
x +y
2 2
y +x
2 2
 y
 x
= x − 2 y tan −1  
 y
∂z
Differentiating partially w.r.t. x,
∂y
∂y
∂  ∂z  1  1
  = 1− 2y
∂x  ∂y
∂y  x 2  y 
1+ 2
y
∂2 z 2 y2 y 2 + x2 − 2 y 2
= 1− 2 =
∂x ∂y y + x2 y 2 + x2
x2 − y 2
= …(1)
x2 + y 2
Differentiating z partially w.r.t. x,
∂z  y 1  y y2  1 
= 2 x tan −1   + x 2 ⋅  −  −
∂y  x y2  x2  x 2  y 
1+ 2 1+ 2
x y
 y x2 y y3
= 2 x tan −1   − 2 −
 x  x + y2 x2 + y2
 y
= 2 x tan −1   − y
 x
∂z
Differentiating partially w.r.t. y,
∂x
∂  ∂z  1  1
  = 2x ⋅   −1
∂y  ∂x 
∂y y2  x 
1+ 2
x
∂2 z 2x2
= −1
∂y ∂x x 2 + y 2
2x2 − x2 − y 2
=
x2 + y 2
x2 − y 2
= …(2)
x2 + y 2
7.5 Higher-Order Partial Derivatives 7.27

From Eqs (1) and (2),


∂2 z ∂2 z x2 − y 2
= = 2
∂x ∂y ∂y ∂x x + y 2

Example 19
∂2u ∂2u ∂2u
If u = log (x2 + y2 + z2), prove that x =y =z .
∂y ∂z ∂z ∂x ∂x ∂y
Solution
u = log (x2 + y2 + z2)
Differentiating u partially w.r.t. y,
∂u 1
= ⋅2y
∂y x 2 + y 2 + z 2
∂y
∂u
Differentiating partially w.r.t. z,
∂y
∂y
∂  ∂u  2y
  =− 2 ⋅ 2z
∂z  ∂y
∂y  ( x + y 2 + z 2 )2
∂2u 4 xyz
x =− 2
∂z ∂y ( x + y 2 + z 2 )2
∂2u 4 xyz
or x =− 2 …(1)
∂y ∂z ( x + y 2 + z 2 )2

Differentiating u partially w.r.t. x,


∂u 1
= ⋅ 2x
∂x x 2 + y 2 + z 2
∂u
Differentiating partially w.r.t. z,
∂x
∂  ∂u  2x
  = − 2 ⋅ 2z
∂z ∂x ( x + y 2 + z 2 )2
∂2u 4 xyz
y =− 2 …(2)
∂z ∂x ( x + y 2 + z 2 )2
∂u
Differentiating partially w.r.t. y,
∂x
∂  ∂u  2x
  = − 2 ⋅2y
∂y ∂x
∂y ( x + y 2 + z 2 )2
∂2u 4 xyz
z =− 2 …(3)
∂x ∂y ( x + y 2 + z 2 )2
From Eqs (1), (2) and (3),
∂2u ∂2u ∂2u
x =y =z
∂y ∂z ∂z ∂x ∂x ∂y
7.28 Chapter 7 Partial Derivatives

Example 20
3
∂2 z 2 ∂ z
2
If z = tan( y + ax) + ( y − a
ax) 2 , find the value of − a .
∂x 2 ∂y 2
∂y
Solution
3
z = tan( y + ax) + ( y − a
ax) 2
Differentiating z partially w.r.t. x,
∂z 3 1
= a sec 2 ( y + ax) − a ( y − ax) 2
∂x 2
∂z
Differentiating partially w.r.t. x,
∂x
∂2 z 3 −
1
= 2a 2 sec 2 ( y + ax) tan(( y + aaxx) + a 2 ( y − ax
ax ) 2 …(1)
∂x 2
4
Differentiating z partially w.r.t. y,
∂z 3 1
= sec 2 ( y + a
axx) + ( y − ax) 2
∂y
∂y 2

∂z
Differentiating partially w.r.t. y,
∂y
∂y

∂2 z 3 −
1
= 2 sec 2 ( y + a
axxx)) tan( y + ax) + ( y − ax
ax ) 2 …(2)
∂y
∂y 2
4
From Eqs (1) and (2),

∂2 z ∂2 z
− a2 2 = 0
∂x 2
∂y
∂y

Example 21
1 ∂2 z 1 ∂2 z
If a2 x2 + b2 y2 = c2 z2, evaluate + .
a 2 ∂x 2 b 2 ∂y 2
Solution
a2 x2 + b2 y2 = c2 z2
Differentiating partially w.r.t. x,
∂z
2a 2 x = 2c
2c 2 z ⋅
∂x
∂z a 2 x
=
∂x c 2 z
7.5 Higher-Order Partial Derivatives 7.29

∂z
Differentiating partially w.r.t. x,
∂x
∂2 z a2  1 x ∂z 
=  − 2 ⋅ 
∂x 2 c 2 z z ∂x
a2  x a2 x 
= 1− ⋅ 2 
c 2 z  z c z
1 ∂2 z 1  a2 x2 
= 2 1 − 2 2 
a ∂x
2 2
c z c z 
1 ∂2 z 1  b2 y 2 
Similarly, = 1−
b 2 ∂y 2 c 2 z  c 2 z 2 
1 ∂2 z 1 ∂2 z 1  a 2 x2 + b2 y 2 
Hence, + = 2 −
a 2 ∂x 2 b 2 ∂y 2 c 2 z  c2 z 2 

1  c2 z 2 
= 2 
2− 2 2
c z c z 
1
= (2 − 1)
c2 z
1
= 2
c z

Example 22
∂ 2u ∂ 2u ∂ 2u 4
If u = log (x3 + y3 – x2y – xy2), prove that + 2 + 2 =− .
∂x 2
∂x ∂y ∂∂yy ( x + y)2
Solution
u = log (x3 + y3 – x2y – xy2)
= log [(x + y)(x2 – xy + y2) – xy(x + y)]
= log [(x + y)(x2 – xy + y2 – xy)]
= log [(x + y)(x – y)2]
= log (x + y) + 2log (x – y)
Differentiating u partially w.r.t. x,
∂u 1 2
= +
∂x x + y x − y
∂u
Differentiating partially w.r.t. x,
∂x
∂2u 1 2
=− −
∂x 2
( x + y) 2
( x − y)2
7.30 Chapter 7 Partial Derivatives

Differentiating u partially w.r.t. y,


∂u 1 2
= −
∂∂yy x + y x − y
∂u
Differentiating partially w.r.t. y,
∂y
∂y
∂2u 1 2
=− −
∂∂yy 2
( x + y) 2
( x − y)2
∂u
Differentiating partially w.r.t. x,
∂y
∂y
∂ 2u 1 2
=− +
∂x ∂y ( x + y)2 ( x − y)2
∂2u ∂ 2u ∂2u 4
+ 2 + 2 =−
∂x 2
∂x ∂y ∂∂yy ( x + y)2

Example 23
2
 ∂ ∂ ∂ 9
If u = log (x3 + y3 + z3 – 3xyz), prove that  + +  u = − .
 ∂x ∂y ∂z  ( x + y + z )2

Solution
2
 ∂ ∂ ∂  ∂ ∂ ∂   ∂u ∂u ∂u 
 ∂x + ∂y + ∂z  u =  ∂x + ∂y + ∂z   ∂x + ∂y +
∂y ∂z 
 ∂ ∂ ∂
= + + v
 ∂x ∂y ∂z 

∂u ∂u ∂u
where v= + +
∂x ∂y ∂z

u = log( x 3 + y 3 + z 3 − 3 xyz )

Differentiating u partially w.r.t. x, y, and z,


∂u 3 x 2 − 3 yz
y
= 3
∂x x + y + z − 3 xyz
3 3

∂u 3 y 2 − 3 xz
x
= 3
∂y x + y 3 + z 3 − 3 xyz
∂y
7.5 Higher-Order Partial Derivatives 7.31

∂u 3 z 2 − 3 xxy
= 3
∂z x + y 3 + z 3 − 3 xyz
∂u ∂u ∂u
v= + +
∂x ∂y∂y ∂z
3( x 2 + y 2 + z 2 ) − 3(
3( xxy + yz + zx)
=
x + y + z 3 − 3 xyz
3 3

3( x 2 + y 2 + z 2 − xxyy − yyzz − zzxx) ( x + y + z )


= ⋅
x 3 + y 3 + z 3 − 3 xyz ( x + y + z)

3( x 3 + y 3 + z 3 − 3xyz
3 xyz )
=
( x 3 + y 3 + z 3 − 3 xyz )( x + y + z )
3
=
x+ y+ z
2
 ∂ ∂ ∂  ∂ ∂ ∂ 3 
Hence,  ∂x + ∂y + ∂z  u =  ∂x + ∂y + ∂z   x + y + z 
3 3 3
=− − −
( x + y + z) 2
( x + y + z) 2
( x + y + z )2
9
=−
((xx + y + z ) 2

Example 24
If u = 3 (ax + by + cz)2 – (x2 + y2 + z2) and a2 + b2 + c2 = 1, show that
∂ 2u ∂ 2u ∂ 2u
+ 2 + 2 = 0.
∂x 2 ∂y
∂y ∂z
Solution
u = 3 (ax + by + cz)2 – (x2 + y2 + z2)
Differentiating u partially w.r.t. x,
∂u
= 6(ax + by + cz )a − 2 x
∂x
∂u
Differentiating partially w.r.t. x,
∂x
∂2u
= 66aa ⋅ a − 2
∂x 2
= 66aa 2 − 2
7.32 Chapter 7 Partial Derivatives

Differentiating u partially w.r.t. y,


∂u
= 6(ax + by + cz )b − 2 y
∂y
∂y
∂u
Differentiating partially w.r.t. y,
∂y
∂y
∂2u
= 6b ⋅ b − 2
∂y 2
∂y
= 6b 2 − 2
Differentiating u partially w.r.t. z,

∂u
= 6(ax + by + cz )c − 2 z
∂z
∂u
Differentiating partially w.r.t. z,
∂z
∂2u
= 6c ⋅ c − 2
∂z 2
= 6c 2 − 2

∂ 2u ∂2u ∂ 2u
Hence, + 2 + 2 =66((a 2 + b 2 + c 2 ) − 6
∂x 2 ∂y
∂y ∂z

= 6(1) − 6 [∵ a 2 + b 2 + c 2 = 1]
=0

Example 25
, find the value of ∂ u2 + ∂ u2 + ∂ u2 .
1 2 2 2
If u =
x2 + y 2 + z 2 ∂x ∂y ∂y ∂z
Solution
1
u=
x + y2 + z2
2

Differentiating u partially w.r.t. x,


∂u 1
=− ⋅ 2x
∂x 3
2( x + y + z )
2 2 2 2

x
=− 3
( x2 + y 2 + z 2 ) 2
7.5 Higher-Order Partial Derivatives 7.33

∂u
Differentiating partially w.r.t. x,
∂x
 
∂2u  1 3x ⋅ 2 x 
=− −
∂x 2  2 3 5 

 ( x + y + z )
2 2 2
2( x + y + z ) 
2 2 2 2

1
=− 5
((xx 2 + y 2 + z 2 − 3 x 2 )
((xx + y + z )
2 2 2 2

−22 x 2 + y 2 + z 2 )

((−
=− 5
((xx 2 + y 2 + z 2 ) 2
∂2u ( x2 − 2 y 2 + z 2 )
Similarly, =−
∂y
∂y 2 5
( x2 + y 2 + z 2 ) 2
∂2u ( x2 + y 2 − 2z 2 )
and =−
∂z 2 5
( x2 + y 2 + z 2 ) 2
∂2u ∂ 2u ∂ 2u ( −2 x 2 + 2 y 2 + 2 z 2 + 2 x 2 − 2 y 2 − 2 z 2 )
Hence, + + = −
∂x 2 ∂y
∂y 2 ∂z 2 5
( x2 + y 2 + z 2 ) 2
=0

Example 26
 x
If u = z tann −1   , find the value of ∂ u + ∂ u + ∂ u .
2 2 2

 y ∂x 2 ∂y
∂y 2 ∂z 2
Solution
 x
u = z tan −1  
 y
Differentiating u partially w.r.t. x,
∂u 1 1
=z ⋅
∂x 2
x y
1+ 2
y
zy
=
y + x2
2

∂u
Differentiating partially w.r.t. x,
∂x
∂2u yz ⋅ 2 x
=− 2
∂x 2 ( x + y 2 )2
2 xyz
=− 2
( x + y 2 )2
7.34 Chapter 7 Partial Derivatives

Differentiating u partially w.r.t. y,


∂u z  x
= −
∂y
∂y x 2  y 2 
1+ 2
y
xz
=− 2
y + x2
∂u
Differentiating partially w.r.t. y,
∂y
∂y
∂2u xz ⋅ 2 y
=
∂y 2 ( x 2 + y 2 ) 2
∂y
2 xyz
= 2
( x + y 2 )2
Differentiating u partially w.r.t. z,
∂u  x
= tan −1  
∂z  y
∂u
Differentiating partially w.r.t. z,
∂z
∂2u
=0
∂z 2
∂ 2u ∂ 2u ∂ 2u 2 xyz
2xyz 2 xyz
Hence, + 2+ 2 =− 2 +
∂x 2 ∂y
∂y ∂z ( x + y 2 )2 ( x 2 + y 2 )2
=0

Example 27

1
∂  2 ∂v  ∂  2 ∂v 
If v = ((11 − 2 xxyy + y 2 ) 2 , find the value of (1 − x ) + y .
∂x  ∂x  ∂y  ∂y
∂y 
Solution 1

v = ((11 − 2 xxyy + y 2 ) 2

Differentiating v partially w.r.t. x,


∂v 1 −
3
= − (1 − 2 xy + y 2 ) 2 ( −2
−2 y )
∂x 2
∂∂vv −
3
(1 − x 2 ) = yy((1 − x 2 )(1 − 2 xy + y 2 ) 2
∂x
∂  ∂ v  ∂  − 
3
(1 − x 2
) = y 
 (1 − x 2
)(1
(1 − 2 xy
xy + y 2
) 2

∂x  ∂x  ∂x
 −
3
3 −
5

= y ( −2 x)(1 − 2 xy + y 2 ) 2 − (1 − x 2 )(1 − 2 xy + y 2 ) 2 ( −2 y ) 
 2 
7.5 Higher-Order Partial Derivatives 7.35

5

− − x − x 2 )]
5

− xy −2 x − xy 2 + − x 2 y)
5

− x −2 − y y) …(1)
Differentiating v partially w.r.t. y,
∂v 1 −
3
= − (1− +y + 2 y)
y 2
3

y2 = − y2 +y − +y ) 2
y
∂ −
3

3
y2 − − 2
− − xy y ) 2
y y
5
3y2
+ ( )(1 2 )
5

1− + 2
y xy y( x y) ]
5
= − y+ − x2 y − y+ − y3 y
+ − y xy 2 )
5
= − y) …(2)
Adding Eqs (1) and (2),
∂ ∂v
( x 0.
∂x ∂x y y

Example 28
∂ 2u 1 ∂ 1 ∂ 2u
If u = (ar + br )(cos nq + sin nq ), show that 2 +
n –n 0
∂r r r r2 ∂ 2
Solution
u = (arn + br–n)(cos nq + sin nq )
Differentiating u partially w.r.t. r,

∂u −n 1
= n−
− )(cos n n )
∂r
∂u
Differentiating partially w.r.t. r,
∂r
7.36 Chapter 7 Partial Derivatives

∂2u
= − r 2
n + sin n )
∂r 2
Differentiating u partially w.r.t. q,
∂u
= n
)( +n n )

∂u
Differentiating partially w.r.t. q,

∂2u n
)( −n n )
∂ 2
− n 2 ar + si )
∂u2
1∂ 1 ∂u 2
= − + r 2
n + si )
∂r 2 r r r2 ∂ 2
n2
+n ar r n 2
n sin − ar + n + si )
r2
n n )r n −2 )
0

Example 29 2 2
∂r ∂r
If x r sin , show that + = 1.
∂x y
Solution
x r sin
2
r …(1)
Differentiating Eq. (1) partially w.r.t. x,
∂r
2 r
∂x
∂r x
=
∂x r
Differentiating Eq. (1) partially w.r.t. y,
∂r
2 2r
y
∂r y
=
y r
2 2
∂r ∂r x2 y 2 x2 y 2 r 2
Hence, + = + 2 = = 2 =1
∂x y r r r2 r
7.5 Higher-Order Partial Derivatives 7.37

Example 30
If x = cos q - r sin q , y = sin q + r cos
cos q , show that
∂r x ∂θ cos θ
(i) = (ii) =− .
∂x r ∂x r
Solution
(i) x = cos θ − r sin θ …(1)
y = sin θ + r cos θ …(2)
2
x = cos θ − 2r ccos
2
os θ sin
sin θ + r sin θ 2 2

y 2 = sin 2 θ + 2r sin θ coss θ + r 2 cos


cos 2 θ
x2 + y 2 = 1 + r 2 …(3)
Differentiating Eq. (3) partially w.r.t. x,
∂r
22xx = 2r
∂x
∂r x
=
∂x r
(ii) Multiplying Eq. (1) by cos q and Eq. (2) by sin q,
x cos θ = cos 2 θ − r sin
n θ cos
cos θ …(4)
y sin θ = sin 2 θ + r coss θ sin
sin θ …(5)
Adding Eqs (4) and (5),
x cos θ + y sin θ = 1
x cot θ + y = cosec θ …(6)
Differentiating Eq. (6) partially w.r.t. x,
∂θ ∂θ
cott θ − x cosec 2θ ot θ cosecθ
= − ccot
∂x ∂x
∂θ ∂ θ
cott θ + cot
cot θ coseccθ = x cosec 2θ
∂x ∂x
∂q
= (cos q - r sin
sin q )cosec 2q
∂x
∂q ∂q
= cosssq
q cosec 2q - r ssin
in q cosec 2q
∂x ∂x
∂q ∂q
= cco
ot q cosecq - r cosecq
∂x ∂x
∂q
cot q = -r cosec q
∂x
7.38 Chapter 7 Partial Derivatives

∂q q co q
=- =-
∂x r cosec q r

Example 31
∂x 1 y y 1 ∂x
Show that = and =− and hence, show that
∂r r ∂ ∂r r∂
∂2 x 1 1 ∂2 x
+ + = 0 if x = e rcos q cos (r sin q ) and y = e rcos q sin (r sin q ).
∂r 2 r ∂r r 2 ∂ 2
Solution
x = e rcosq cos (r sin q )
Differentiating x partially w.r.t. r,
∂x
= r ⋅ rs − r sin ) sin
∂r
= e r co [ cos s s sin
r sin ) …(1)
r s
Now, y e sin(r sin )
Differentiating y partially w.r.t. r,
y cos
e r sin sin
∂r
e r cos r n ) …(2)
Differentiating x partially w.r.t. q,
∂x
= r − s − sin(r ⋅ r )

= +r n ) …(3)
Differentiating y partially w.r.t. q,

y r
− s ⋅r

+r n ) …(4)

From Eqs (1) and (4),


∂x 1 y
...(5)
∂r r∂
From Eqs (2) and (3),
y 1 ∂x
=−
∂r r∂
∂x y
= −r ...(6)
∂ ∂r
7.5 Higher-Order Partial Derivatives 7.39

Differentiating Eq. (5) partially w.r.t. r,


∂ 2 x ∂  1 ∂y 
=  
∂r 2 ∂r  r r∂θ 
−1 ∂y 1 ∂ 2 y
= +
r 2 ∂θ r ∂r∂θ
Differentiating Eq. (6) partially w.r.t. q,
∂2 x ∂  ∂y ∂y 
=  − r 
∂θ 2
∂θ ∂r
∂2 y
= −r
∂θ ∂r
∂2 y
= −r
∂ ∂θ
∂r

Hence, ∂ x + 1 ∂x + 1 ∂ x = −1 ∂y + 1 ∂ y + 1 ∂y − 1 ∂ y
2 2 2 2

∂r 2 r ∂r r 2 ∂θ 2 r 2 ∂θ r ∂r∂θ r 2 ∂θ r ∂r∂θ
=0

Example 32
−r2
1 ∂  2 ∂θ  ∂θ
If θ = t e
n 4t
then find n so that r = .
r 2 ∂r  ∂r  ∂t
[Winter 2016; Summer 2016]
Solution
−r2
θ=t e n 4t

Differentiating q partially w.r.t. t,


2 2
−r −r
∂θ  r2 
= nt n −1e 4t + t n e 4t  2 
∂t  4t 
2
−r
 1 
= e 4t  nt nn−−−11 + r 2 t n − 2 
 4 
Differentiating q partially w.r.t. r,
2
−r
∂θ  −2r 
= t n e 4t 
∂r  4t 
∂  2 ∂θ  ∂  t n −1 3 −4rt 
2

r  = − re 
∂r  ∂r  ∂r  2 
 2 −r  −2r  
2 2
−r
t n −1
=− 3r e 4t + r e 4t 
3

2   4t  

7.40 Chapter 7 Partial Derivatives

2
1 3 r2 n
r = t + t 2

r 2 ∂r ∂r 2 4
1 ∂ 2∂ ∂
Given r = ,
r ∂r
2
∂r t
r2 r2
3 r
e t e nt r tn 2

2 4 4
3 1
t nt
2
3
n −
2

Example 33
Find the value of n so that v = r n (3 cos2 q – 1) satisfies the equation
∂ 2 ∂v 1 ∂ ∂v
r + n
∂r ∂r sin ∂ ∂
Solution
v = rn (3 cos2 q – 1)
Differentiating v partially w.r.t. r,
∂v
= n 1
− 1)
∂r
∂ 2
r nr n (3 − 1)]
∂r ∂r r
=n r cos − 1) …(1)
Differentiating v partially w.r.t. q,
∂v
= n ⋅ 6 cos )

= −3r n 2
∂ ∂v ∂
sin = ( 3r n si sin

= −3 2 )
= −3 ⋅ +2 2 cos − 1)]
1 ∂ ∂v
sin = −3 2 co 2
2)
sin
= −6 3 s )
7.5 Higher-Order Partial Derivatives 7.41

∂  2 ∂v  1 ∂  ∂v 
Given r + n θ  = 0,
 sin
∂r  ∂r  sin θ ∂θ  ∂θ 
n(n + 1) r n (3 cos2 q – 1) – 6r n (3 cos2 q – 1) = 0
n(n + 1) – 6 = 0
n2 + n – 6 = 0
(n + 3)(n – 2) = 0
n = –3, 2

Example 34
If xx yy zz = c, show that at x = y = z,
∂2 z ∂2 z ∂2 z ∂2 z 2( x 2 − 2)
(i) = −( x log eex) −1 (ii) 2 − 2 xy + = .
∂x ∂y ∂x ∂x ∂y ∂y
∂y 2 x(1 + log x)
x

Solution
(i) xx yy zz = c
Taking logarithm on both the sides,
g x x + log
log log y y + log
g z z = log
log c
log x + y log
x log log y + z log
log z = log c …(1)

Differentiating Eq. (1) partially w.r.t. x,


1 ∂z 1 ∂z
x ⋅ + log x + ⋅ log z + z ⋅ =0 ∵ z = f ( x, y )]
[[∵
x ∂x z ∂x
∂z 1 + log x
=−
∂x 1 + log z
∂z
Differentiating partially w.r.t. y,
∂x

∂  ∂z   1 1 ∂z 
  = − (1
( + log x)  − ⋅
∂∂yy  ∂x   (
(1 + log z
z)) 2
z ∂x 
∂2 z (1 + lloog x)  1 + log x 
= −
log z ) 2  1 + log z 
∂y ∂x z ((1 + log
∂2 z ((1 + log
log x) 2
=−
∂x ∂y z ((1 + loogg z )3
7.42 Chapter 7 Partial Derivatives

At x = y = z,
∂2 z (1 + log x) 2
=−
∂x ∂y xx((1 + log x)3
1
=−
xx((1 + log x)
= −[ x(log e + log x)]−1 [ log e = 1]
[∵
−1
= −( x log
log ex
ex) .

∂z
(ii) Differentiating partially w.r.t. x,
∂x
∂ 2 z ∂  1 + log x 
= −
∂x 2 ∂x  1 + log z 
(1 + log x) 1 ∂z 1
= ⋅ −
(1 + log z ) z ∂x x(1
2
1 + logg z)
1 + log x) (1+
(1+ 1 + log x) 1
=− ⋅ −
z (1
1++ log z ) (1+
2
1 + log z)
z ) x(1 + log
g z)
At x = y = z,
∂2 z 2
=−
∂x 2
xx((1 + log x)

∂2 z − −((1 + log y ) 2 1
Similarly, = −
∂y
∂y 2
zz((1 + log z )3 yy((1 + log z )
At x = y = z,
∂2 z 2
=−
∂y 2
∂y xx((1 + log x)

∂2 z ∂2 z ∂2 z 2  1   2 
Hence, − 2 xy + =− − 2 xy  −  + − 
∂x 2
∂x ∂y ∂y∂y 2
xx((1 + log x)  x(1 + log x)   xx((1 + log x) 
( xy − 2)
22((xy
=
x(1 + llog
og x)
( x 2 − 2)
2(x
2(
= [∵ x = y = z ]
x(1 + llog
og x)

Example 35
x2 y2 z2
If 2 + + = 1, prove that
a + u b2 + u c2 + u
2 2 2
 ∂u   ∂u   ∂u   ∂u ∂u ∂u 
  +   +   = 2  x + y +z .
∂x  ∂∂yy  ∂z  ∂x ∂y
∂y ∂z 
7.5 Higher-Order Partial Derivatives 7.43

Solution
x2 y2 z2
+ + =1 ...(1)
a 2 + u b2 + u c2 + u
Differentiating Eq. (1) partially w.r.t. x,
2x x2 ∂u y2 ∂u z2 ∂u
− − − =0
a 2 + u (a 2 + u ) 2 ∂x (b 2 + u ) 2 ∂x (c 2 + u ) 2 ∂x
∂u  x 2 y2 z2  2x
 2 + + 2
= 2
∂x  ( a + u ) 2
(b + u )
2 2
(c + u )  a + u
2

∂u 2x
⋅p= 2
∂x (a + u )
x2 y2 z2
where p= + 2 + 2
(a + u )
2 2
(b + u ) 2
(c + u ) 2
∂u 2x
=
∂x ( a 2 + u ) p
∂u 2y
Similarly, =
∂y (b 2 + u ) p
∂y
∂u 2z
= 2
∂z ( c + u ) p
2 2 2
 ∂u   ∂u   ∂u  4  x2 y2 z2 
  +   +   = 2  2 + 2 + 2 
∂x  ∂y
∂y  ∂z p  (a + u ) 2
(b + u ) 2
(c + u ) 2 
4
= 2 ( p)
p
4
= …(2)
p
∂u ∂u ∂u 2  x 2 y2 z2 
x +y +z =  2 + 2 + 2
∂x ∂y ∂z p  a + u b + u c + u 
2
= (1)
p
2
= …(3)
p
From Eqs (2) and (3),
2 2 2
 ∂u   ∂u   ∂u   ∂u ∂u ∂u 
  +   +   = 2  x + y +z 
∂x  ∂∂yy  ∂z  ∂x ∂y
∂y ∂z 
7.44 Chapter 7 Partial Derivatives

Example 36
∂z ∂z
If z = e ax + by f (aaxx − b
byy ), prove that b +a = 2abz.
∂x ∂y
Solution
z = e ax + by f (a
axx − b
byy )
Differentiating z partially w.r.t. x,
∂z
= ae ax + by f (a byy ) + ae ax ++by
axx − b by
f ′ (ax − by )
∂x
= az + ae ax + by f ′ (a
axx − bbyy )
Differentiating z partially w.r.t. y,
∂z
= be ax + by f (a byy ) − be ax ++by
axx − b by
f ′ (ax − by )
∂y
∂y
= bz − be ax + by f ′ (a
axx − b
byy )
∂z ∂z
Hence, b +a = abz + abe ax + by f ′ (a by ) + abz − abe ax + by f ′ (a
axx − by axx − by
by )
∂x ∂y
= 2abz

Example 37
∂ 2u 2 ∂ u
2
If u = f (x + ky) + y (x – ky), show that = k .
∂y 2
∂y ∂x 2

Solution
u = f (x + ky) + y (x – ky)
Differentiating u partially w.r.t. x,
∂u
= φ' ( x + ky ) ⋅1 + ψ' ( x − ky ) ⋅1
∂x

∂u
Differentiating partially w.r.t. x,
∂x
∂2u
= φ'' ( x + kkyy ) + ψ'' ( x − ky ) …(1)
∂x 2
Differentiating u partially w.r.t. y,
∂u
= φ' ( x + ky ) ⋅ k + ψ' ( x − kkyy ) ⋅ ( − k )
∂y
∂y
7.5 Higher-Order Partial Derivatives 7.45

∂u
Differentiating partially w.r.t. y,
∂y
∂y
∂2u
= φ'' ( x + kkyy ) ⋅ k 2 + ψ'' ( x − kkyy )( − k ) 2
∂y 2
∂y
= k 2 [φ'' ( x + ky ) + ψ'' ( x − ky
ky )] …(2)
From Eqs (1) and (2),
∂ 2u ∂2u
= k2 2
∂y
∂y 2
∂x

Example 38
∂ 2u ∂ 2u ∂ 2u
If u = xf (x + y) + yf (x + y), show that − 2 + 2 = 0.
∂x 2 ∂x ∂y ∂y
∂y
Solution
u = xf (x + y) + y f (x + y),
Differentiating u partially w.r.t. x,
∂u
f' ( x + y ) + yφ' ( x + y )
= f ( x + y ) + xxf'
∂x
∂u
Differentiating partially w.r.t. x,
∂x
∂ 2u
= f' ( x + y ) + f' ( x + y ) + xf '' ( x + y ) + yφ'' ( x + y )
∂x 2
= 2 f' ( x + y ) + xf'' ( x + y ) + yφ'' ( x + y )
∂u
Differentiating partially w.r.t. y,
∂x
∂ 2u
= f ' ( x + y ) + xf '' ( x + y ) + yφ'' ( x + y ) + φ' ( x + y )
∂x ∂y
Differentiating u partially w.r.t. y,
∂u
= xf ' ( x + y ) + φ ( x + y ) + yφ' ( x + y )
∂y
∂y
∂u
Differentiating partially w.r.t. y,
∂y
∂y
∂ 2u
= xf '' ( x + y ) + φ' ( x + y ) + φ' ( x + y ) + yφ'' ( x + y )
∂y 2
∂y
= xf '' ( x + y ) + 2φ''(( x + y ) + yφ'' ( x + y )
∂2u ∂2u ∂2u
Hence, −2 + 2
∂x 2
∂x ∂y ∂y
∂y
7.46 Chapter 7 Partial Derivatives

= 2 f' ( x + y ) + xf '' ( x + y ) + yφ'' ( x + y ) − 2 f ' ( x + y ) − 2 xxf'


f'' ( x + y )
− 2 yφ'' ( x + y ) − 2φ' ( x + y ) + xf''
xf '' ( x + y ) + 2φ' ( x + y ) + yφ'' ( x + y )
=0

Example 39
If u = f ( )
x 2 + y 2 , prove that

∂ 2u ∂ 2u
+ 2 =
∂x 2 ∂y
∂y
1
f′ ( x 2 + y 2 + f ′′ ) ( x2 + y 2 . )
x +y
2 2

Solution
Let x2 + y 2 = r
u = f(r)
Differentiating u partially w.r.t. x,
∂u ∂
= f (r )
∂x ∂x
∂f ∂r
∂f
= ⋅
∂r ∂x

= f ′(r ) ⋅ x2 + y 2
∂x
1
= f ′(r ) ⋅ ⋅ 2x
2 x + y2
2

∂u
Differentiating partially w.r.t. x,
∂x
∂2 u ∂r x f ¢(r ) 1
= f ¢¢(r ) ◊ + - xf ¢(r ) ◊ 2x
∂x 2 ∂x x 2 + y 2 x 2 + y2
3
2( x +
2
y2 ) 2

x x f ¢(r ) x 2 f ¢(r )
= f ¢¢(r ) ◊ + - 3
x 2 + y2 x 2 + y2 x 2 + y2
(x +
2 2 2
y )
x 2
f ¢(r ) x f ¢(r )
2
= f ¢¢(r ) + -
x +y
2 2
x +y
2 2 3
(x + 2 2 2
y )

∂2 u y2 f ¢(r ) y 2 f ¢(r )
= f ¢¢(r ) + -
Similarly, ∂y 2 x 2 + y2 x 2 + y2
3
( x 2 + y2 ) 2
7.5 Higher-Order Partial Derivatives 7.47

∂2 u ∂2 u ( x 2 + y2 ) 2 f ¢(r ) ( x 2 + y 2 ) f ¢(r )
Hence, + = f ¢¢(r ) + -
∂x 2 ∂y 2 x 2 + y2 x 2 + y2
3
(xx 2 + y 2 ) 2
2 f ¢(r ) f ¢(r )
= f ¢¢(r ) + -
x +y
2 2
x 2 + y2
f ¢(r )
= f ¢¢(r ) +
x 2 + y2

= f ¢¢ ( x 2 + y2 + ) f¢ ( x 2 + y2 )
x +y
2 2

Example 40
 x2  ∂ 2u ∂ 2u ∂ 2u
If u = f   , show that x 2 2 + 33xy
xy + 2 y 2 2 = 0.
 y ∂x ∂x ∂y ∂y
Solution
 x2 
u= f 
 y
Differentiating u partially w.r.t. x,
∂u  x2  ∂  x2 
= f'   
∂x  y  ∂x  y 
 x2   2x 
= f'   
 y y 

∂u
Differentiating partially w.r.t. x,
∂x
∂2u 2  x 2   x2  ∂  x2   2x 
= f ′   + f ′′   ⋅   ⋅  
∂x 2
y  y  y  ∂x  y   y 
2
2  x2   x2   2x 
= f ′   + f ′′   ⋅  
y  y  y  y
Differentiating u partially w.r.t. y,

∂u  x2  ∂  x2 
= f ′  ⋅  
∂y
∂y  y  ∂y  y 
 x2   x2 
= f ′  ⋅ − 2 
 y  y 
7.48 Chapter 7 Partial Derivatives

∂u
Differentiating partially w.r.t. y,
∂y
∂y

∂2u 2 x 2  x 2   x 2   x2  ∂  x2 
= f ′ + −
 y   y 2  f ′′  y  ⋅ ∂y  y 
∂y 2
∂y y3
2
2x2  x2   x2   x2 
= 3 f ′   +  2  f ′′  
y  y y   y

∂u
Differentiating partially w.r.t. y,
∂x
∂2u 2xx  x 2  2 x
2  x2   x2 
= − 2 f ′  + f ′′   ⋅  − 2 
∂x ∂y y  y y  y  y 

∂2u ∂2u ∂2u 22xx 2  x 2  4 x 4  x2  6x2  x2 


x2 + 33xy
xy + 2 y2 2 = f ′   + 2 f ′′   − f ′ 
∂x 2
∂x ∂y ∂y y  y y  y y  y
6x 4
6  x2  4x2  x2  2x4  x2 
− f ′′   + f ′   + 2 f ′′  
y 2
 y y  y y  y
=0

Example 41
 xy  ∂u ∂u ∂u ∂u
If u = e xyz f   , prove that x + z = 2 xyzu and y +z = 2 xyzu
 z ∂x ∂z ∂y ∂z
∂ 2u ∂ 2u
and hence, show that x =y .
∂z ∂x ∂z ∂y
Solution
 xy 
u = e xyz f  
 z

Differentiating u partially w.r.t. x, y and z,

∂u  xy    xy    y 
= e xyz yyzz ⋅ f   + e xyz  f ′  z    z 
∂x  z  
∂u  xy    xy    x 
= e xxyzyz xz ⋅ f   + e xyz
 f ′  z    z 
∂y
∂y  z  
∂u  xy    xy    − xy 
= e xyz xxyy f   + e xyz  f ′     2 
∂z  z    z  z 
7.5 Higher-Order Partial Derivatives 7.49

∂u ∂u
(i) x +z
∂x ∂z
xy xy xy xy xy
xyz
⋅f + e f′ + ⋅f − e f′
z z z z z z
xy
f
z
2 u

∂u ∂u
(ii) y +z
y ∂z
xy xy xy xy xy
xyz
⋅f + e f′ + ⋅f e f′
z z z z z z
xy
2 xyze f
z
2 xyzu

∂u
(iii) Differentiating w.r.t. x,
∂z
∂2u xy xy xy  y
⋅ xy f + ⋅f + e xy f ′
∂ x z z z z
xy  xy  xy  y xy
+ e yz f ′ +e f ′′
z z2 z z z2
 xy  y
+ e xyz f ′
z z2
∂2u xy xy x2 y 2 xy xy xy 
x e xyz x 2 y 2 z f + ⋅f − 3 f ′′ f′ …(1)
∂z x z z z z z 2
z
∂u
Differentiating w.r.t. y,
∂z
∂ 2u xy xy xy ˘ x
= ◊ y f + ◊f +e y f¢
∂ y z z z z
È xy ˘ xy xy ˘ x xy
+e x f 2
+e f ¢¢
z z z z z2
È xy ˘ x
+e f¢
z z2
∂2u  xy xy x2 y 2 xy xy xy 
y = 2 2
⋅f + ⋅f f ′′ f′ …(2)
∂z y z z z 3
z z 2
z
7.50 Chapter 7 Partial Derivatives

From Eqs (1) and (2),


∂2u ∂2u
x =y
∂z ∂x ∂z ∂y

Example 42
∂ 2u ∂ 2u ∂ 2u
If u = r , r = x + y + z , show that + 1)r m − 2 .
m 2 2 2
+ 2 + 2 = m(m +1
∂x ∂y
2
∂y ∂z
Solution
u = rm
Differentiating u partially w.r.t. x,
∂u ∂r
= mr m −1 ...(1)
∂x ∂x
But r= x2 + y 2 + z 2
r 2 = x2 + y 2 + z 2
Differentiating r 2 partially w.r.t. x,
∂r
2r = 2x
∂x
∂r x
=
∂x r
Substituting in Eq. (1),
∂u x
= mr m −1
∂x r
= mr m − 2 x
∂u
Differentiating partially w.r.t. x,
∂x
∂2u  ∂r 
= m  r m −−22 + (m − 2)r m − 3 x
∂x 2  ∂x 
 x 
= m  r mm−−−22 + (m − 2)r m − 3 x 
 r 
−22
− m−4 2
= m[r m−
m
+ ( m − 2) r x ] ...(2)
∂u2
Similarly, = m[r m −−22 + (m − 2)r m − 4 y 2 ] …(3)
∂∂yy 2
∂2u
= m[r m −−22 + (m − 2)r m − 4 z 2 ] …(4)
∂z 2
Adding Eqs (2), (3) and (4),
∂ 2u ∂ 2u ∂ 2u
+ 2 + 2 =3 mr m −−22 + m(m − 2)r m − 4 ( x 2 + y 2 + z 2 )
3mr
∂x 2 ∂y
∂y ∂z
7.5 Higher-Order Partial Derivatives 7.51

=3mr m −−22 + m(m − 2)r m − 4 ⋅ r 2


3mr
= r m − 2 (3m + m 2 − 2m)
= r m − 2 (m + m 2 )
+ 1)r m − 2
= m(m +1

Example 43
If u = f (r) and r 2 = x2 + y2 + z2, prove that
∂ 2u ∂ 2u ∂ 2u 2
+ 2 + 2 = f ′′ (r ) + f ′ (r ). [Summer 2015]
∂x ∂y
2
∂y ∂z r
Solution
u = f (r)
Differentiating u partially w.r.t. x,
∂u ∂
= f (r )
∂x ∂x
∂ ∂r
= f (r ) ⋅
∂r ∂x
∂r
= f ′(r ) ⋅ ...(1)
∂x
But r 2 = x2 + y2 + z2
Differentiating r 2 partially w.r.t. x,
∂r
2r = 2x
∂x
∂r x
=
∂x r
Subsituting in Eq. (1),
∂u x
= f ′(r ) ⋅
∂x r
∂u
Differentiating partially w.r.t. x,
∂x
∂2u ∂  x
=  f ′(r ) 
∂x 2
∂x  r
∂r x f ′ ( r )  1  ∂r
= f ′′ (r ) ⋅ + + xf ′ (r )  − 2  ⋅
∂x r r  r  ∂x
x x f ′(r ) x x
= f ′′ (r ) + − 2 f ′(r ) ⋅
r r r r r
x 2 f ′(r ) x 2
= f ′′ (r ) 2 + − 3 f ′(r ) …(2)
r r r
7.52 Chapter 7 Partial Derivatives

∂2u y 2 f ′(r ) y 2
Similarly, = f ′′ (r ) 2 + − 3 f ′(r ) …(3)
∂y
∂y 2
r r r
∂2u z 2 f ′(r ) z 2
and = f ′′ (r ) 2 + − 3 f ′(r ) …(4)
∂z 2
r r r
Adding Eqs (2), (3) and (4),
∂ 2 u ∂ 2 u ∂ 2 u f ′′ (r ) 2 3 f ′(r ) ( x 2 + y 2 + z 2 )
+ + = ( x + y 2
+ z 2
) + − f ′(r )
∂x 2 ∂y∂y 2 ∂z 2 r2 r r3
f ′′ (r ) 2 3 f ′ (r ) r 2
= ⋅r + − 3 f ′(r )
r2 r r
2 f ′(r )
= f ′′ (r ) +
r

Example 44
If u = f (r 2) where r 2 = x2 + y 2 + z2, prove that
∂ 2u ∂ 2u ∂ 2u
+ 2 + 2 = 44rr 2 f ′′ (r 2 ) + 6 f ′ (r 2 ).
∂x ∂y
2
∂y ∂z
Solution
u = f (r 2)
Differentiating u partially w.r.t. x,
∂u ∂
= f (r 2 )
∂x ∂x

= f (l ), where r 2 = l
∂x
∂ ∂l
= f (l ) ⋅
∂l ∂x
∂l
= f ′ (l )
∂x
∂r 2
= f ′(r 2 )
∂x
∂u ∂r
∴ = f ′ ( r 2 ) ⋅ 2r ...(1)
∂x ∂x
But r 2 = x2 + y2 + z2
Differentiating r 2 partially w.r.t. x,
∂r
2r = 2x
∂x
∂r x
=
∂x r
7.5 Higher-Order Partial Derivatives 7.53

Substituting in Eq. (1),

∂u x
= f ′ ( r 2 ) ⋅ 2r
∂x r
= 2 x f ′(r 2 )

∂u
Differentiating partially w.r.t. x,
∂x

∂2u ∂∂ff ′ (r 2 )
= 2 f ′ ( r 2
) + 2 x
∂x 2 ∂x
∂r
= 2 f ′ (r 2 ) + 2 x f ′′ (r 2 ) ⋅ 2r
∂x
x
= 2 f ′ (r 2 ) + 2 x f ′′ (r 2 ) ⋅ 2r
r …(2)
= 2 f ′ (r 2 ) + 4 x 2 f ′′ (r 2 )

∂2u
Similarly, = 2 f ′ (r 2 ) + 4 y 2 f ′′ (r 2 ) …(3)
∂y 2
∂y

∂ 2u
and = 2 f ′ (r 2 ) + 4 z 2 f ′′ (r 2 ) …(4)
∂z 2
Adding Eqs (2), (3) and (4),

∂2u ∂2u ∂2u


+ 2 + 2 = 6 f ′ (r 2 ) + 4( x 2 + y 2 + z 2 ) f ′′ (r 2 )
∂x 2 ∂y
∂y ∂z
= 6 f ′ (r 2 ) + 4r 2 f ′′ (r 2 )

Example 45
∂ 2v ∂ 2v 1
If v = x log(x + r) – r where r = x + y , prove that 2 + 2 =
2 2 2
.
∂x ∂∂yy x+r

Solution
v = x log(x + r) – r
Differentiating v partially w.r.t. x,
∂v x  ∂r  ∂r
= log( x + r ) + 1 +  −
∂x x + r  ∂x  ∂x
But r 2 = x2 + y 2
7.54 Chapter 7 Partial Derivatives

Differentiating r 2 partially w.r.t. x,


∂r
2r = 2x
∂x
∂r x
=
∂x r
∂r y
Similarly, =
∂∂yy r
∂v x  x x
∴ = log( x + r ) + 1 +  −
∂x x+r  r r
x (r + x) x
= log( x + r ) + ⋅ −
(x + r) r r
x x
= log( x + r ) + −
r r
= log( x + r )

∂v
Differentiating partially w.r.t. x,
∂x
∂2v 1  ∂r 
= 1 + 
∂x 2 x + r  ∂x 
1  x
= 1 + 
x+r  r
1
=
r
Differentiating v partially w.r.t. y,
∂v x ∂r ∂r
= ⋅ −
∂y x + r ∂y
∂y ∂y ∂y
∂y
x y y
= ⋅ −
x+r r r
y  x − x − r
=  
r  x+r 
y
=−
x+r
∂v
Differentiating partially w.r.t. y,
∂y
∂y
∂2v 1 y ∂r
=− + ⋅
∂y 2
∂y x + r ( x + r ) 2 ∂y
∂y
1  y y
=− 1 − ⋅ 
x+r x+r r
7.6 Variables to be Treated as Constants 7.55

1  rx + r 2 − y 2 
=−  
x + r  r(x + r) 
1  rx + x 2 
=−  
x + r  r(x + r) 
x
=−
r(x + r)

∂2v ∂2v 1  x 
Hence, + 2 = 1 − 
∂x 2
∂y
∂y r  x + r
1  x + r − x
=  
r  x+r 
1
=
x+r

7.6 VARIABLES TO BE TREATED AS CONSTANTS


In some problems, it is difficult to identify which variable is to be treated as constant. In
such cases, the variable to be treated as constant is written as the suffix of the bracket.
 ∂r 
Thus,   means that r is first to be expressed as a function of x and y and then
 ∂x  y
 ∂x 
differentiated w.r.t. x keeping y constant. Similarly,   means that x is first to be
 ∂r  θ
expressed as a function of r and q and then differentiated w.r.t. r, keeping q constant.

Example 1
 ∂u   ∂v  1  ∂v   ∂y∂y 
If x2 = au + bv, y2 = au – bv, prove that     = =     ,
 ∂x  y  ∂u  v 2  ∂y
∂y  x  ∂v  u
where a, b are constants.

Solution
x2 = au + bv …(1)
Differentiating Eq. (1) partially w.r.t. u keeping v constant,
 ∂x 
2x   = a
 ∂u  v
 ∂x  a
  =
∂u v 2 x
7.56 Chapter 7 Partial Derivatives

y2 = au – bv …(2)
Differentiating Eq. (2) partially w.r.t. v keeping u constant,

 ∂y 
2 y   = −b
 ∂v  u
 ∂y  b
  = −
∂v u 2y

Now, x2 = au + bv, y2 = au – bv
x2 + y2 = 2au

x2 + y 2
u= …(3)
2a
Differentiating Eq. (3) partially w.r.t. x keeping y constant,

 ∂u  x
  =
∂x y a

Also, x2 – y2 = 2bv
x2 − y 2
v= …(4)
2b
Differentiating Eq. (4) partially w.r.t. y keeping x constant,
 ∂v  y
 ∂y =−
∂y  x b

 ∂u   ∂x  x a
Hence,     = ⋅
∂x y ∂u v a 2 x
1
=
2
 ∂v   ∂y ∂y   y  b 
and
 ∂y   = −  −
∂y  x  ∂v  u  b   2 y 
1
=
2

Example 2
r θ r Ê ∂x ˆ Ê ∂r ˆ
If x = (e + e −θ ), y = (eθ − e −θ ) then show that ÁË ˜¯ = ÁË ˜¯ .
2 2 ∂r q ∂x y
7.6 Variables to be Treated as Constants 7.57

Solution
r q r q
x= (e + e -q ), y= ( e - e -q )
2 2
= r cosh q ...(1) = r sinh q ...(2)
Differentiating Eq. (1) partially w.r.t. r keeping q constant,
 ∂x 
  = cosh θ
∂r θ
...(3)
Squaring and subtracting Eq. (1) from Eq. (2),
Also, x2 – y2 = r2
2
Differentiating r partially w.r.t. x keeping y constant,
 ∂r 
22xx = 2r  
 ∂x  y
 ∂r  x
  = = cosh θ ...(4)
∂x y r
From Eqs (3) and (4),

 ∂x   ∂r 
  =  
∂r θ ∂x y

Example 3
If x = r cos q, y = r sin q, prove that
 ∂r   ∂x   ∂θ  1  ∂x 
(i)   =   (ii) r   =  
∂x y ∂r θ ∂x y r ∂θ r

∂ 2 r ∂ 2 r 1   ∂r   ∂r  
2 2
∂ 2θ ∂ 2θ
(iii) + 2 =   +    (iv) + =0
∂x 2 ∂y
∂y r  ∂x   ∂y
∂y   ∂x 2 ∂y 2

Solution
(i) x = r cos q, y = r sin q,
x2 + y2 = r 2
2
Differentiating r partially w.r.t. x keeping y constant,
 ∂r 
22xx = 2r  
 ∂x  y
 ∂r  x
  = …(1)
∂x y r
Again, x = r cos q
7.58 Chapter 7 Partial Derivatives

Differentiating x partially w.r.t. r keeping q constant,


 ∂x  x
  = cos θ = …(2)
∂r θ r
From Eqs (1) and (2),
 ∂r   ∂x 
  =  
∂x y ∂r θ
(ii) x = r cos q, y = r sin q
Differentiating x partially w.r.t. q keeping r constant,
 ∂x 
  = − r sin θ
∂θ r
1  ∂x 
  = − sin θ …(3)
r  ∂θ  r
y
Now, tan θ =
x
y
θ = tan −1
x
Differentiating q partially w.r.t. x keeping y constant,
Ê ∂q ˆ 1 Ê yˆ
ÁË ∂x ˜¯ = Á- ˜
y2 Ë x 2 ¯
y
1+ 2
x
y
=- 2
x + y2
r sin q
=-
r2
sin q
=-
r
Ê ∂q ˆ 1 Ê ∂x ˆ
r Á ˜ = - sinq = Á ˜ [From Eq. (3)]
Ë ∂x ¯ y r Ë ∂q ¯ r
∂r x
(iii) =
∂x r
∂r
Differentiating partially w.r.t. x,
∂x
∂  ∂r  1 x ∂r
 = −
∂x  ∂x  r r 2 ∂x
∂2 r 1 x2  ∂r x 
= − ∵ ∂x = r 
∂x 2 r r 3
r 2 − x2
=
r3
2
y
= 3
r
7.6 Variables to be Treated as Constants 7.59

∂2r x2  ∂r y 
Similarly, = ∵ ∂y = 
∂y 2 r 3
∂y  ∂y r 
∂2 r ∂2 r y 2 x2
+ = +
∂x 2 ∂∂yy 2 r 3 r 3
1  x2 y 2 
= +
r  r 2 r 2 

1   ∂r   ∂r  
2 2

=   + .
r  ∂x   ∂y  
y
(iv) tan θ =
x
y
θ = tan −1
x
Differentiating q partially w.r.t. x,
∂θ 1  y
= − 
∂x y 2  x2 
1+ 2
x
y
=− 2
x + y2
∂θ
Differentiating partially w.r.t. x,
∂x
∂  ∂θ  y
 = ⋅ 2x
∂x  ∂x  ( x 2 + y 2 ) 2
2 xy
= 2
( x + y 2 )2
∂ 2θ 2 xy
=
∂x 2 ( x 2 + y 2 ) 2
Differentiating q partially w.r.t y,
∂θ 1 1
= ⋅
∂y
∂y 2
y x
1+ 2
x
x
= 2
x + y2
∂  ∂θ  x⋅2y
=− 2
∂y  ∂y 
∂y ( x + y 2 )2
∂ 2θ 2 xy
xy
=− 2
∂y 2
∂y ((xx + y 2 ) 2
7.60 Chapter 7 Partial Derivatives

∂ 2θ ∂ 2θ
Hence, + =0
∂x 2 ∂y 2

Example 4
If x 2 = a u + b v , y 2 = a u − b v , where a and b are constants, find
 ∂u   ∂x 
    .
∂x y ∂u v
Solution
x2 = a u + b v ...(1)
y = a u −b v
2
...(2)
x 2 + y 2 = 2a u
x2 + y 2
u= ...(3)
2a
Differentiating Eq. (3) partially w.r.t x keeping y constant,

1  ∂u  1
  = (2x
2 x)
2 u ∂x y 2 a
x
=
a
 ∂u  2x u
  =
∂x y a
x  x2 + y 2 
=
a  a 
x((xx 2 + y 2 )
=
a2

Differentiating Eq. (1) partially w.r.t. u keeping v constant,


 ∂x  a
2x   =
 ∂u  v 2 u
 ∂x  1  a 
  =
∂u v 2 x  2 u 
a a
= ⋅ 2
2x x + y2
a2
=
2 x(xx 2 + y 2 ) ...(4)
7.6 Variables to be Treated as Constants 7.61

Hence,
 ∂u   ∂x 
   
∂x y ∂u v
x( x 2 + y 2 ) a2
= ⋅
a2 2 x( x 2 + y 2 )
1
=
2

Example 5

cos θ sin
si nθ  ∂x   ∂u   ∂y ∂y   ∂u 
If x = , y= , evaluate  ∂u   ∂x  +  ∂u  ⋅  ∂y
∂y  x
.
u u θ y θ

Solution
cos θ
x=
u
Differentiating x partially w.r.t. u keeping q constant,

 ∂x  cos θ
  = − 2
∂u θ u
sin θ
Also y=
u

Differentiating y partially w.r.t. u keeping q constant,

 ∂y
∂y  sin θ
  = − 2
∂u θ u

coss 2 θ sin
sin 2 θ 1
Now, x2 + y 2 = + = 2
u2 u2 u
1
u2 =
x + y2
2

Differentiating u2 partially w.r.t. x keeping y constant,

 ∂u  1
2u   = − 2 (2x
2 x)
 ∂x  y (x + y2 )
 ∂u  x
  = −
∂x y u ( x 2 + y 2 )2
7.62 Chapter 7 Partial Derivatives

Differentiating u2 partially w.r.t. y keeping x constant,


 ∂u  1
2u   = − 2 (2 y )
 ∂y  x (x + y2 )
 ∂u  y
 ∂y  = − u ( x 2 + y 2 ) 2
x

Hence,
 ∂x   ∂u   ∂y   ∂u   cos θ   x   sin θ   y 
  ⋅   +   ⋅   =  − 2   − 2 2
− 2  −
  2 2
∂u θ ∂x y ∂u θ  ∂y  x u  u ( x 2
+ y )  u  u ( x 2
+ y ) 
x cos θ + y sin θ
=
u 3 ( x 2 + y 2 )2
cos 2 θ sin 2 θ
+
= u u
3 1
u 4
u
=1

Example 6
If u = ax + by, v = bx – ay, find the value of
 ∂u   ∂x   ∂y∂y   ∂v 
  ⋅   ⋅   ⋅   .
∂x y ∂u v ∂v x  ∂y ∂y  u
Solution
u = ax + by ... (1)
Differentiating u partially w.r.t. x keeping y constant,
 ∂u 
  = a
∂x y
Also, v = bbxx − ay
ay ... (2)
b v
∴ y = x−
a a
Differentiating y partially w.r.t. v keeping x constant,
 ∂y
∂y  1
  = −
∂v x a
Eliminating y from Eqs. (1) and (2),
u−a ax bx − v
=
b a
au − a 2 x = b 2 x − bv
au + bv
x= 2
a + b2
7.6 Variables to be Treated as Constants 7.63

Differentiating x partially w.r.t u keeping v constant,


 ∂x  a
  = 2
∂u v a + b 2
Eliminating x from Eqs. (1) and (2),
u − by
b v + ay
a
=
a b
bu − b 2 y = av + a 2 y
bu − (a 2 + b 2 ) y
v=
a
Differentiating v partially w.r.t. y keeping u constant,
 ∂v   a 2 + b2 
 ∂y  = −
∂y  u  a 

 ∂u   ∂x   ∂y ∂y   ∂v   a   1  a + b 
2 2
Hence,   ⋅   ⋅   ⋅   = a  2 2 
−  −
 ∂x  y  ∂u  v  ∂v  x  ∂y
∂y  u  a + b   a  a 
=1

Example 7
 ∂u   ∂v   ∂u   ∂v 
If x = u tan v, y = u sec v, prove that   ⋅   =   ⋅   .
 ∂x  y  ∂x  y  ∂∂yy  x  ∂y
∂y  x
Solution
x = u tan v ...(1)
y = u sec v ...(2)
Squaring and subtracting Eq. (1) from Eq. (2),
y2 – x2 = u2 ...(3)
Differentiating Eq. (3) partially w.r.t. x keeping y constant,

 ∂u 
−2
−2 x = 2u  
 ∂x  y
 ∂u  x
  = −
∂x y u

Differentiating Eq. (3) partially w.r.t. y keeping x constant,


 ∂u 
2 y = 2u  
 ∂y  x
 ∂u  y
 ∂y  = u
x
7.64 Chapter 7 Partial Derivatives

Dividing Eq. (1) by Eq. (2),


x
= sin v ...(4)
y
Differentiating Eq. (4) partially w.r.t. x keeping y constant,
1  ∂v 
= cos v ⋅  
y  ∂x  y
 ∂v  1
  =
∂x y y cos v
Differentiating Eq. (4) partially w.r.t. y keeping x constant,
x  ∂v 
− = cos v  
y 2
 ∂y
∂y  x
 ∂v  x
 ∂y  =− 2
∂y  x y cos v
 ∂u   ∂v   x  1 
    =  −  
∂x y ∂x y u  y cos v 
x
=− ...(5)
uy cos v
 ∂u   ∂v   y  x 
 ∂y  ⋅  =    − 2
∂y  x  u   y cos v 
∂y  x  ∂y
x
=− ...(6)
uy cos v
From Eqs (5) and (6),
 ∂u   ∂v   ∂u   ∂v 
    =    
∂x y ∂x y  ∂∂yy  x  ∂y
∂y  x

Example 8
u v  ∂u   ∂v  x2 + y 2
If ux + vy = 0 and + = 1 then prove that   −   = 2 .
x y  ∂x  y  ∂∂yy  x y − x 2

Solution
ux + vy = 0 …(1)
u v
and + = 1, …(2)
x y

vy
From Eq. (1), u=−
x
7.6 Variables to be Treated as Constants 7.65

Substituting in Eq. (2),

vy v
− + =1
x2 y
v( − y 2 + x 2 ) = x 2 y
x2 y
v=
x − y2
2

Differentiating v partially w.r.t. y keeping x constant,

 ∂v   1 y 
 ∂y  = x2  2 − 2 ( −2 y ) 
∂y  x x − y
2
(x − y )
2 2

 x2 − y 2 + 2 y 2 
= x2  
 (x − y ) 
2 2 2

x2 ( x2 + y 2 )
=
((xx 2 − y 2 ) 2

ux
From Eq. (1), v=−
y
Substituting in Eq. (2),

u ux
− =1
x y2
u ( y 2 − x 2 ) = xxy 2
xy 2
u=
y 2 − x2

Differentiating u w.r.t. x keeping y constant,

 ∂u  2  1 x 
  = y  2 − 2 ( −2 x) 
∂x y  y − x 2
( y − x 2 2
) 
 y 2 ( x2 + y 2 ) 
= 2 2 2 
 (y − x ) 

 ∂u   ∂v  ( x 2 + y 2 )( y 2 − x 2 )
Hence,   − =
∂x y  ∂∂yy  x ( y 2 − x 2 )2
( x2 + y 2 )
=
y 2 − x2
7.66 Chapter 7 Partial Derivatives

EXERCISE 7.2
∂u ∂u 1
1. If y prove that x +y sin .
∂x ∂y 2

∂2u ∂2u
2. If u ax by y ) and k, find the value of + .
∂x 2 ∂y 2
[Ans. : 0]
∂ ∂u
3. If an y , show that si in y 2.
∂x ∂y
4. If 0, show that
∂z ∂z  ∂2 z ∂z
2
 ∂2 z
(i) z (ii) z +
∂x ∂y ∂x ∂y 2

∂2 z ∂2 z
5. If 0, prove that + = .
∂x 2 ∂y + 3

y ∂2u ∂2u
6. If y ) + tan 1
, show that + =0
x ∂x 2 ∂y 2

1 ∂2u ∂2u ∂2u


7. If , find the value of + + .
z 2
∂x 2 ∂y 2 ∂z 2
2 
Ans.: 2
(

8. If x e r co cos(r si and y e r cos sin(r si , prove that


∂x 1 ∂ 1 ∂x
=
∂r r ∂ ∂r r∂
∂2 x 1∂ 1 ∂2 x
Hence, deduce that 0
∂r 2 r2 ∂
∂2v ∂2v
9. If ), prove that + ′′ y).
∂x 2 ∂y 2
10. If ax nd , show that
∂ ∂v ∂ ∂v
u u
∂y ∂x x ∂y
r r ∂x ∂r
11. If x ), prove that .
∂r ∂x y

: , sin r2
7.6 Variables to be Treated as Constants 7.67

∂ 2θ θ(x 2 + ry 2 )
ge θ = r − x, r 2 = x 2 + y 2 , show that
12. If log = .
∂y 2 r3
 r − x ∂r y
Hint : θ = e , ∂y = r 
 
∂2u ∂2u
13. If u = eax sin by, prove that = .
∂x ∂y ∂y ∂x

 xy  ∂2u 1
n−1 
14. If u = tan  , prove that = .
 1+ x + y 
2 2 ∂ x ∂ y 3
(1 + x + y )
2 2 2

( x − a )2
1 − ∂2u ∂2u
15. If u = e 4y
, prove that = .
y ∂x ∂y ∂y ∂x
3
∂2u ∂2u
16. If u = tan(y + ax) − (y − a
ax) 2 , prove that = .
∂x ∂y ∂y ∂x
xy ∂3u ∂3u
17. If u = , prove that = .
2x + y ∂y ∂z 2 ∂z 2 ∂y
∂3u ∂3u
18. If u = xmyn, prove that = .
∂x ∂y ∂z ∂y ∂x 2
∂u ∂u
19. Find and for the following functions:
∂x ∂y
3
(i) x + y − 1 (ii) 1 − x 2 − y 2 (iii) yx (iv) log ( + by ) (v) (y − ax) 2
g10 (ax

 1 1 −x −y 
 Ans. : (i)) , (ii) , 
 x + y −1 x + y −1 1− x − y
1 2 2
1− x − y
2 2

 x −1 a b 
 x
(iii) y log y , xy (iv) , 
 (loge 10)(ax + by ) (loge 10)(ax + by ) 
 3a 1
3 1 
 (v) − (y − ax ax) 2 , ((yy − ax) 2 
 2 2 

∂z ∂z
∂z
20. If x 4 − x
xyy 2 + yyzz 2 − z 4 = 6, find and
d .
∂x ∂y
 y 2 − 4 x 3 2 xy − z 2 
 Ans. : , 
 2yz − 4 z 3 2yz − 4 z 3 

∂z ∂z
21. If z 3 + x
xyy − y 2 z = 6, find and at (0, 1, 2).
∂x ∂y
 1 4
 Ans. : − 11, 11
 
7.68 Chapter 7 Partial Derivatives

r2

22. Find the value of for which u = t n e 4 kt satisfies the partial differential
∂u  ∂ 2 u 2 ∂u 
equation = k 2 + .
∂t  ∂r r ∂r   3
 Ans. : n = − 2 
 
∂r ∂θ
sin φ, z = r coss θ,fi
23. If x = r sin θ cos φ, y = r sin θ sin , nd , in terms of r, q, f.
∂x ∂x
 y x2 + y2 
Hint : r 2 = x 2 + y 2 + z 2 , φ = tan−−11 , θ = tan−1 
 x z 
 coss θ cos φ − sin φ 
n θ cos φ,
 Ans. : sin ,
 r r sin θ 
∂u ∂u ∂u
24. If u = x2 (y – z) + y2 (z – x) + z2 (x – y), prove that + + = 0.
∂x ∂y ∂z

∂2u ∂2u
25. If u = e x (x cos y − y sin y ), prove that + = 0.
∂x 2 ∂y 2

y ∂2 f ∂2 f ∂2 f
26. For the function f (x, y , z) = z tan−1 , prove that 2 + 2 + 2 = 0 .
x ∂x ∂y ∂z
∂2 z ∂2 z ∂z
27. If z(x + y ) = x 2 + y 2 , prove that x + y =2 .
∂x 2
∂x ∂y ∂x
2 2
x2 y2  ∂u   ∂u   ∂u ∂u 
28. If + = 1, prove that   +   = 2  x +y 
a+u b+u  ∂x   ∂y   ∂x ∂y 
∂3u ∂3u
29. If u = xy, prove that =
∂x 2 ∂y ∂x ∂y ∂x

x2 y2 z2
30. If + + = 1, prove that
2+u 4+u 6+u
2 2 2
 ∂u   ∂u   ∂u   ∂u ∂u ∂u 
 ∂x  +  ∂y  +  ∂z  = 2  x ∂x + y ∂y + z ∂z  .

31. If u = (x2 – y2) f(r), where r = xy, show that


∂2u
= (x 2 − y 2 )[3f
3 f ′(r) + rf ′′(r)].
∂x ∂y
∂z ∂z
32. If z = f (x2, y), prove that x = 2y .
∂x ∂y
7.6 Variables to be Treated as Constants 7.69

1
33. Prove that z = [ f (cctt + r) + φ(ct − r)] satisfies the partial differential
r
∂ 2 u c 2 ∂  2 ∂u 
equation = r  where is constant.
∂t 2 r 2 ∂r  ∂r 

∂2u ∂2u ∂2v ∂2v


34. If u + iv
iv = f(
f (x + iiyy ), prove that + = 0, + = 0.
∂x 2 ∂y 2 ∂x 2 ∂y 2

Hint : u + iv
iv = f (x + iyiy ), u − iv = f (x − iy ) 
 
 1 1
u = [ f (x + iiyy ) + f (x − iiyy )], v = [ f (x + iy ) − f (x − iy )]
 2 2i 

35. If u, v, w are function of x, y, z given as x = u + v + w, y = u2 + v2 + w2,


∂u vw((w − v )
z = u 3 + v 3 + w 3 , prove that = .
∂x (u − v )(v − w )(w − u)
[Hint : Differentiate x, y, z w.r.t. x and solve the equations using Cramer’s rule.]
n
36. If u = (x 2 + y 2 + z 2 ) 2 , find the value of which satisfies the equation
∂2u ∂2u ∂2u
+ + = 0.
∂x 2 ∂y 2 ∂z 2
[Ans. : 0, –1]
2
 ∂2u   ∂2u   ∂2u 
37. If u = log(e x + e y ), show that  2   2  −  = 0.
 ∂x   ∂y   ∂x ∂y 

 ∂z   ∂z  xy
38. If z = yyff (x 2 − y 2 ), show that y   + x   = .
 ∂x   ∂y  y

39. If x 2 = a u + b v and y 2 = a u − b v , where are constant, prove that


 ∂u   ∂x  1  ∂v   ∂y 
 ∂x   ∂u  = 2 =  ∂y   ∂v  .
y v x u

 ∂u   ∂x   ∂y   ∂v 
40. If u = ax
ax + b
byy , v = bx − ay , find the value of      ∂v   ∂y  .
 ∂x  y  ∂u  v x u

[Ans. : 1]
∂ z
3
x
41. If z = x log (x + r) – r, where r2 = x2 + y2, prove that = − 3 
∂x 3 r 
7.70 Chapter 7 Partial Derivatives

7.7 TOTAL DERIVATIVES


7.7.1 Chain Rule
If z = f (u), where u is again a function of two variables x and y, i.e., z
u = f (x, y) then
∂z dzdz ∂ u df ∂ u
df ∂u
= ⋅ or ⋅ or f ′ (u ) u
∂x du ∂x du ∂x ∂x
∂z dzz ∂u dff ∂u ∂u x y
= ⋅ or ⋅ or f ′ (u ) Fig. 7.2
∂y du ∂∂yy
∂y du ∂ y ∂y

7.7.2 Composite Function of One Variable


u
If u = f (x, y), where x = f (t), y = y (t) then u is a
function of t and is called the composite function of a
single variable t and x y
du ∂u dx
dx ∂u dy
dy u
= ⋅ + ⋅
dt ∂x dt ∂y dt
t
is called the total derivative of u. x y z
If u = f (x, y, z) and x = f (t), y = y (t), z = x (t) then Fig. 7.3
total derivative of u is given as
du ∂u dx dx ∂u dy dy ∂u dz dz t
= ⋅ + ⋅ + ⋅ .
dt ∂ x dt ∂ y dt ∂ z dt Fig. 7.4

Example 1
du
If u = y 2 – 4ax, x = at 2, y = 2at, find .
dt u
Solution
u = y 2 – 4ax, x = at 2, y = 2at
du ∂u dx dx ∂u dy dy x y
= ⋅ + ⋅
dt ∂x dt ∂y dt
= ( − 4a ) 2a
att + (2 y )2a t
du
Substituting y, = −8a 2 t + 2(2at )(2a ) Fig. 7.5
dt
= −8a 2 t + 8a 2 t
=0

Example 2
 x du
.
If u = sin   where x = et, y = t2, find
 y dt
7.7 Total Derivatives 7.71

Solution u
 x
u = sin
n   , x = et , y = t 2
 y
du ∂u dx
dx ∂u dy
dy x y
= ⋅ + ⋅
dt ∂x dt ∂y dt
1  x  x  x
= coss   ⋅ et +  − 2  cos   ⋅ 2t t
y  y  y   y
Substituting x and y, Fig. 7.6
du 1 e  t
e e 
t t
= 2 cos  2  ⋅ et − 2 2 cos  2  ⋅ 2t
dt t t  (t ) t 
1 t  et   2 
= e cos  2  1 − 
t 2
t  t

Example 3
du u
If u = x2y3, x = log t, y = e t, find .
dt
Solution
u = x2y3, x = log t, y = e t x y
du ∂u dx dx ∂u dy dy
= ⋅ + ⋅
dt ∂x dt ∂y dt
t
1
= ((2
2 xy 3 ) + ((3 x 2 y 2 )et
t Fig. 7.7
Substituting x and y,
du 1
= 2(log t )e3t ⋅ + 3(log t ) 2 e 2t ⋅ et
dt t
2
= log t e3t + 3(log t ) 2 e3t
t

Example 4
1 du
If u = xy + yz + zx where x = , y = et , z = e − t , find .
t dt
Solution u
1
u = xy + yz + zx, x = , y = et , z = e − t
t
du ∂u dx
dx ∂u dy
dy ∂u dz
dz x y z
= ⋅ + ⋅ + ⋅
dt ∂x dt ∂y dt ∂z dt
 1
= ( y + z )  − 2  + ( x + z ) et + ( y + x)( −e − t ) t
 t  Fig. 7.8
7.72 Chapter 7 Partial Derivatives

Substituting x, y and z,
du 1 1
− (e e + +e t
et − e e t

dt t2 t t
1
− 2 (e t e− e e t)
t

Example 5
2 dz
If t , find .
dt
Solution z

z x2 y 2
at
dz ∂ z dx ∂ z y x y
= +
dt ∂x dt y dt
= at x 2a
t
Substituting x, y and z, Fig. 7.9

dz
a + at + at + a ) 2a
dt
= a + t at a 2a
t t)
5t )

Example 6
− dz 3
If y x prove that .
dt 1 t2
[Summer 2015]
Solution
z −y x =4 3

z
dz ∂z dx ∂z y
⋅ + ⋅
dt ∂ x dt y dt
1 1 1) x y
= + 12t 2
1 − (x y) 2
− (x y) 2

2
3 t
= t
1 2 xy Fig. 7.10
7.7 Total Derivatives 7.73

Substituting x and y,
dzz
d 3(1 − 4t 2 )
=
dt 1 − 9t 2 − 16t 6 + 24t 4
3(1 − 4t 2 )
=
1 − 8t 2 − t 2 − 16t 6 + 16t 4 + 8t 4
3(1 − 4t 2 )
=
1 − 8t 2 + 16t 4 − t 2 − 16t 6 + 8t 4
3(1 − 4t 2 )
=
(1 − 4t 2 ) 2 − t 2 (1 + 16t 4 − 8t 2 )
3(1 − 4t 2 )
=
(1 − 4t 2 ) 2 − t 2 (1 − 4t 2 ) 2
3(1 − 4t 2 )
=
(1 − 4t 2 ) 1 − t 2
3
=
1− t2

Example 7
 y du
If u = tann −1   , x = et − e − t , y = et + e − t , find .
 x dt
Solution
−1  y 
u = tann −1 −t
  , x = e − e , y = e + e
t t −t

x
du ∂u dx
dx ∂u dy
dy
= . + .
dt ∂ x dt ∂ y dt
u
1  y t 1  1 t
= 2 
− 2  (e + e − t ) + −t
  (e − e )
y  x  y2  x 
1+ 2 1+ 2
x x
x y
y x
=− ⋅y+ 2 ⋅x
x +y
2 2
x + y2
x2 − y 2
=
x2 + y 2 t
−t 2
(e − e ) − (e + e )
t t −t 2 Fig. 7.11
=
(e t − e − t ) 2 + (e t + e − t ) 2
7.74 Chapter 7 Partial Derivatives

4
=−
2(e 2t + e −2t )
2
= − 2t
e + e −2t

Example 8
Ê xˆ ∂z ∂z
For z = tann -1 Á ˜ , x = u cos v, y = u sin v, evaluate and at the
Ë y¯ ∂u ∂v
Ê pˆ
point Á 1.3, ˜ . [Winter 2015]
Ë 6¯
Solution z
Ê xˆ
n -1 Á ˜ , x = u cos v, y = u sin v
z = tan
Ë y¯
∂z ∂z ∂x ∂z ∂y
= ◊ + ◊ x y
∂u ∂x ∂u ∂y ∂u
1 Ê 1ˆ 1 Ê xˆ
= ÁË y ˜¯ ◊◊co
2
cos v + Á - ˜ ◊ sin v
x x 2 Ë y2 ¯
1+ 2 1+ 2 u
y y Fig. 7.12
y cos
cos v x sin
sin v
= 2 -
y + x 2 y2 + x 2
sin v cos v - u coss v sin
u sin sin v
=
x 2 + y2
=0
z
∂z ∂z ∂x ∂z ∂y
= ◊ + ◊
∂v ∂x ∂v ∂y ∂v
1Ê 1ˆ 1 Ê xˆ
= Á ˜ ( -u sin v) + Á - 2 ˜ (u cos v)
x Ë y¯ 2
x2 Ë y ¯
x y
1+ 2 1+ 2
y y
yu sin v xu cos v
=- 2 -
y + x 2 y2 + x 2 v
Fig. 7.13
( y2 + x 2 )
=-
y2 + x 2
= -1
7.7 Total Derivatives 7.75

Example 9
du
If u = x 2 + y 2 + z 2 , where x = et , y = et sin t , z = et cos t,
t find .
dt
Solution u

u = x 2 + y 2 + z 2 , x = et , y = et sin t , z = et cos t

du ∂u dx
dx ∂u dy
dy ∂u dz
dz
= ⋅ + ⋅ + ⋅ x z y
dt ∂x dt ∂y dt ∂z dt

= 2 xet + 2 y (et sin t + et cos t ) + 2 z (et cos t − et sin t )


= 22eet ⋅ et + 2et sin t ⋅ et (sin t + cos t ) + 2et cos t ⋅ et (cos t − sin t ) t

= 22ee 2t (1 + sin 2 t + sin


sin t cos t + ccoos 2 t − cos
cos t sinn t ) Fig. 7.14
= 4e 2t

Example 10
π
t find d
If z = e xy , x = t cos t , y = t sin t, dzz at
t= . z
dt 2
Solution
z = e xy , x = t cos t , y = t sin t x y

ddzz ∂z dx
dx ∂z dy
dy
= . + .
dt ∂x dt ∂y dt
= e xy y (cos t − t sinn t ) + e xy xx(sin
(sin t + t cos t ) t

π π Fig. 7.15
At t = , x = 0, y =
2 2
dzz
d π  π 
Hence, = e0   0 −  + 0
dt t = π
2  2  
2

p2
=-
4

Example 11
dz
If z = x2y + 3xy4 where x = sin 2t and y = cos t, find when t = 0.
dt
[Winter 2013]
7.76 Chapter 7 Partial Derivatives

Solution z
z = x2y + 3xy4, x = sin 2t, y = cos t
dz ∂z d ∂ z dy
= ◊ +
dt ∂x dt ∂y dt
x y
y3 )( n )

At t = 0, x = 0, y = 1
dz t
Hence, )]
dt t 0 Fig. 7.16

Example 12
y dz
If u x z2 − = 1 show that + + =0
x y 1 z2
Solution
u x y2 z −2 =1

∂u ∂u ∂u
d dx + dz = 0
∂x y ∂z

0
…(1)

We have, x2 + z2 − 1
x − 2
y2 z2
x2 z − y2 z y2 z2

Similarly, z

and z ⋅ y2
Substituting in Eq. (1),
1 − + 1 − + 1 − 2
=0
 dx dy dz 
1 − y2 1 z + + 0
 1 x z2 
2
7.7 Total Derivatives 7.77

dx dy dz
Hence, + + =0
1− x 2
1− y 2
1 − z2

7.7.3 Composite Function of Two Variables


z
If z = f (x, y), where x = f (u, v), y = y (u, v) then
z
z is a function of u, v and is called the composite
function of two variables u and v.
x y
∂z ∂z ∂x ∂z ∂y
∂y
= ⋅ + ⋅ x y
∂u ∂x ∂u ∂y
∂y ∂u
∂z ∂z ∂x ∂z ∂y
∂y
= ⋅ + ⋅ u
∂v ∂x ∂v ∂y
∂y ∂v v
Fig. 7.17
Fig. 7.18

Example 1
y
If z = f (u , v),
v), u = log( x 2 + y 2 ), v = , show that
x
∂z ∂z ∂z
x − y = ((11 + v 2 ) .
∂y ∂x ∂v
z
Solution
y
z = f (u , v), u = log( x 2 + y 2 ), v = ,
x u v

∂z ∂z ∂u ∂z ∂v
= ⋅ + ⋅
∂x ∂u ∂x ∂v ∂x
∂z ∂z  y 
x
1
= ⋅ 2 ⋅ 2x +  − 2 
∂u x + y 2
∂v  x  Fig. 7.19
∂z 2 xy ∂z y ∂z 2
y = ⋅ − ⋅ ...(1)
∂x x 2 + y 2 ∂u x 2 ∂v
z

∂z ∂z ∂u ∂z ∂v
and = ⋅ + ⋅
∂∂yy ∂u ∂y∂y ∂v ∂y ∂y
u v
∂z 2y ∂z 1
= ⋅ 2 + ⋅
∂u x + y 2
∂v x
∂z 2 xy ∂z ∂z y
x = ⋅ + ...(2)
∂y x 2 + y 2 ∂u ∂v
∂y
Fig. 7.20
7.78 Chapter 7 Partial Derivatives

Subtracting Eq. (1) from Eq. (2),


∂z ∂z ∂z y 2 ∂z
Hence, x −y = +
∂y ∂x ∂v x 2 ∂v
∂z
= (1 + v 2 )
∂v

Example 2
∂w
If w = φ (u , vv),
), u = x 2 − y 2 − 2 xy, v = y, prove that = 0 is equivalent
∂w ∂w ∂ v
to ((xx + y ) + ((xx − y ) = 0.
∂x ∂y
w
Solution
w = φ (u , vv),
), u = x 2 − y 2 − 2 xy, v = y
∂w ∂w ∂u ∂w ∂v
= ⋅ + ⋅
u v
∂x ∂u ∂x ∂v ∂x
∂w ∂w
= 2 x − 2 y) +
((2 ⋅0
∂u ∂v x
∂w ∂w Fig. 7.21
= (2 x − 2 y )
∂x ∂u w
∂w ∂w ∂u ∂w ∂v
and = ⋅ + ⋅
∂y ∂u ∂y
∂y ∂y ∂v ∂y ∂y
v
∂w ∂w
u
= −2

((−2 y − 2 x) + ⋅1
∂u ∂v
∂w ∂w ∂w y
= −2( x + y ) +
∂y
∂y ∂u ∂v Fig. 7.22
∂w ∂w ∂w ∂w ∂w
((xx + y ) + ((xx − y ) = ((xx + y )22(((xx − y ) − ((xx − y )2((xx + y ) + ((xx − y )
∂x ∂y ∂u ∂u ∂v
∂w
= ( x − y)
∂v
∂w ∂w ∂w
If = 0 then ((xx + y ) + ((xx − y ) = 0.
∂v ∂ x ∂y

∂w ∂w ∂w
Hence, = 0 is equivalent to ((xx + y ) + ((xx − y ) = 0.
∂v ∂ x ∂y
7.7 Total Derivatives 7.79

Example 3
y and x = eu + e − v and y = e − u − ev , show that
If z = f ( x, y)
∂z ∂z ∂z ∂z
− =x −y . [Summer 2014]
∂u ∂v ∂x ∂∂yy
Solution
z
z = f ((xx, y ), x = eu + e − v , y = e − u − ev
∂z ∂z ∂x ∂z ∂y∂y
= ⋅ + ⋅
∂u ∂x ∂u ∂y∂y ∂u x y
∂z u ∂z
∂z
= −e − u )
e + ( −e ...(1)
∂x ∂y
∂y
∂z ∂z ∂x ∂z ∂y ∂y u
and = ⋅ + ⋅
∂v ∂x ∂v ∂y ∂y ∂v Fig. 7.23
∂z ∂z
∂z
= −ee − v ) + ( −e
(− −ev ) ...(2)
∂x ∂y
∂y z

Subtracting Eq. (2) from Eq. (1),

∂z ∂z ∂z u ∂z x y
Hence, − = (e + e − v ) − (e − u − e v )
∂u ∂v ∂x ∂y
∂y
∂z ∂z
=x −y v
∂x ∂y
∂y
Fig. 7.24

Example 4
If z = f (u , v) and u = x cos θ − y sin θ , v = x sin θ + y cos
cos θ , show that
∂z ∂z ∂z ∂z
x +y =u +v .
∂x ∂y ∂u ∂v
Solution
z = f (u , v), u = x coss θ − y sin θ , v = x sin θ + y cos
cos θ z

∂z ∂z ∂u ∂z ∂v
= +
∂x ∂u ∂x ∂v ∂x
∂z ∂z
∂z u v
= coss θ + sin θ
∂u ∂v
∂z ∂z ∂z
x = x coss θ + x sin θ ...(1) x
∂x ∂u ∂v
Fig. 7.25
7.80 Chapter 7 Partial Derivatives

∂z ∂z ∂u ∂z ∂v
= +
∂∂yy ∂u ∂y
∂y ∂v ∂y ∂y
∂z ∂z
∂z z
= ( − sin θ ) + cos θ
∂u ∂v
∂z ∂z ∂z
y = ( − y sin θ ) + ( y cos
cos θ ) ...(2) u v
∂y ∂u ∂v
Adding Eqs (1) and (2),
y
∂z ∂z ∂z ∂z
x +y = ( x cos θ − y si
sinn θ ) + ( x sin θ + y cos
cos θ ) Fig. 7.26
∂x ∂y ∂u ∂v
∂z ∂z
=u +v
∂u ∂v

Example 5
u+v ∂z ∂z ∂z
If z = f (x, y), x = uv, y = , prove that u + v = 2x .
u−v ∂u ∂v ∂x
Solution
u+v
z = f(x, y), x = uv, y = ,
u−v
∂z ∂z ∂x ∂z ∂y
∂y
= ⋅ + ⋅
∂u ∂x ∂u ∂y
∂y ∂u z

∂z ∂z  (u − v) − (u + v) 
= v+  
∂x ∂y 
∂y (u − v) 2 
x y
∂z 2v ∂ z
= v−
∂x (u − v) 2 ∂y
∂y
u
∂z ∂z 2uv ∂z
u = uv − ...(1) Fig. 7.27
∂u ∂x (u − v) 2 ∂y
∂z ∂z ∂x ∂z ∂y
∂y
and = ⋅ + ⋅ z
∂v ∂x ∂v ∂y
∂y ∂v
∂z ∂z  (u − v) − (u + v)( −1) 
= u+  
∂x ∂y 
∂y (u − v) 2  x y
∂z ∂z 2u ∂z
= u+ ⋅
∂v ∂x (u − v) ∂y
2
∂y
v
∂z ∂z 2uv ∂z
v = uv + ⋅ ...(2) Fig. 7.28
∂v ∂x (u − v) 2 ∂y∂y
7.7 Total Derivatives 7.81

Adding Eqs (1) and (2),


∂z ∂z ∂z
u + v = 2uv
∂u ∂v ∂x
∂z
= 2x ⋅
∂x

Example 6
If z = f ( x, y),
y ), x = u cosh v, y = u sin h v, prove that
2 2 2 2
 ∂z  1  ∂z   ∂z   ∂z 
  − 2   =   −   .
∂u u ∂v ∂x  ∂∂yy 
z
Solution
z = f ( x, y ), x = u cosh v, y = u sinh v
x y
∂z ∂z ∂x ∂z ∂y ∂y
= ⋅ + ⋅
∂u ∂x ∂u ∂y ∂y ∂u
∂z ∂z u
= cosh v + sinh v ...(1)
∂x ∂y
∂y
Fig. 7.29
∂z ∂z ∂x ∂z ∂y∂y z
and = ⋅ + ⋅
∂v ∂x ∂v ∂y∂y ∂v
∂z ∂z
= u sinh v + u cosh v x y
∂x ∂y
∂y
1 ∂z ∂z ∂z
⋅ = sinh v + cosh v ...(2)
u ∂v ∂x ∂y v

Squaring and subtracting Eq. (2) from Eq. (1), Fig. 7.30
2 2 2 2
 ∂z  1  ∂z   ∂z   ∂z  ∂z ∂z
  − 2   =   cosh v +   sinh
nh 2 v + 2
2
cosh v sinh v
∂u u ∂v ∂x  ∂y
∂y  ∂x ∂y
2 2
 ∂z   ∂z  ∂z ∂z
−   sinh 2 v −   cosh 2 v − 2 cosh
h v sin
ssinh
inh
hv
 ∂x   ∂y  ∂x ∂y
2 2
 ∂z   ∂z 
=   (coshh 2 v − sinh 2 v) −   (coshh 2 v − sinh 2 v)
 ∂x   ∂y
∂y 
2 2
 ∂z   ∂z 
=   − 
 ∂x   ∂y 
7.82 Chapter 7 Partial Derivatives

Example 7
inh θ , show that ((xx − y )(
If x = r cosh θ , y = r sinh )((z x − z y ) = r zr − zθ .
Solution u
z = f ((xx, y ), x = r cosh θ , inh θ
y = r sinh
∂z ∂z ∂x ∂z ∂y ∂y
= ⋅ + ⋅
∂r ∂x ∂r ∂y∂y ∂r x y
∂z ∂z
= ⋅ cosh θ + ⋅ sinh θ
∂x ∂y
∂y
∂z ∂z ∂x ∂z ∂y ∂y
and = ⋅ + ⋅ r
∂θ ∂x ∂θ ∂y ∂y ∂θ
Fig. 7.31
∂z ∂z
=⋅ r sinh θ + ⋅ r cosh θ
∂x ∂y
∂y z

∂z ∂z ∂z ∂z ∂z ∂z
r − = ⋅ r cosh θ + ⋅ r sinh θ − ⋅ r sinh θ − ⋅ r cosh θ
∂r ∂θ ∂x ∂y
∂y ∂x ∂y
∂y
x y
∂z ∂z
= (r cosh θ − r sin
inhh θ ) + (r sinh θ − r cosh θ )
∂x ∂y
∂y
∂z ∂z q
= ( x − y) − ( x − y)
∂x ∂y Fig. 7.32
 ∂z ∂z 
= ((xx − y )  − 
 ∂x ∂y 
Hence, ( x − y )(
)( z x − z y ) = r zr − zθ

Example 8
If z = f (x, y) where x2 = au + bv, y2 = au – bv then prove that
∂z ∂z 1  ∂z ∂z
u +v = x +y 
∂u ∂v 2  ∂ x ∂ y
Solution z

z = f ((xx, y ), x 2 = auu + bbvvv, y 2 = au − bv


∂z ∂z ∂x ∂z ∂y ∂y y
= + x
∂u ∂x ∂u ∂y ∂y ∂u
∂z a ∂z a
= ⋅ + ⋅
∂x 2 x ∂y∂y 2 y u
Fig. 7.33
∂z ∂z au ∂z au
u = ⋅ + ⋅ ...(1)
∂u ∂x 2
2xx ∂y 2 y
7.7 Total Derivatives 7.83

∂z ∂z ∂x ∂z ∂y
∂y z
= +
∂v ∂x ∂v ∂y
∂y ∂v
∂z b ∂z  b 
= ⋅ + −  x y
∂x 2 x ∂y
∂y  2 y 
∂z ∂z bv ∂z bv
v = ⋅ − ⋅ ...(2) v
∂v ∂x 2 x ∂y 2 y
Adding Eqs (1) and (2), Fig. 7.34

∂z ∂z ∂z au ∂z au ∂z bv ∂z bv
u +v = + + −
∂u ∂v ∂x 2 x ∂y 2 y ∂x 2 x ∂y 2 y
∂z  au + bv  ∂z  au − bv 
=  +
∂x  2xx  ∂y  2 y 
∂z  x 2  ∂z  y 2 
= +  
∂x  2 x  ∂y
∂y  2 y 
1  ∂z ∂z 
=  x +y 
2  ∂x ∂y 
∂y

Example 9
If u = f (x, y) where x = r cos q and y = r sin q, prove that
2 2 2 2
 ∂u  1  ∂u   ∂u   ∂u 
  + 2   =   +   .
∂r r ∂θ ∂x  ∂y
∂y 

Solution
u = f (x, y), x = r cos q, y = r sin q u
∂u ∂u ∂x ∂u ∂y
∂y
= ⋅ + ⋅
∂r ∂x ∂r ∂y
∂y ∂r
∂u ∂u
∂u x y
= coss θ + sin θ …(1)
∂x ∂y
∂y
∂u ∂u ∂x ∂u ∂y∂y
and = ⋅ + ⋅ r
∂θ ∂x ∂θ ∂∂yy ∂θ
∂u ∂u
∂u Fig. 7.35
= −rr sin θ ) +
(− r cos θ
∂x ∂∂yy
1 ∂u ∂u ∂u
= − sin θ + cos θ …(2)
r ∂θ ∂x ∂y
∂y
7.84 Chapter 7 Partial Derivatives

Squaring and adding Eqs (1) and (2), u

2 2 2 2
 ∂u  1  ∂u   ∂u   ∂u  ∂u ∂u
  + 2   = coss θ   + sin θ   + 2 sinn θ cos
cos θ
2 2

∂r r ∂θ ∂x  ∂y
∂y  ∂x ∂y
∂y x y
2 2
 ∂u   ∂u  ∂u ∂u
+ sinn 2 θ   + cos 2 θ   − 2 siinn θ cos
cos θ
 ∂x   ∂y
∂y  ∂x ∂y
∂y q
2 2
 ∂u   ∂u 
=   +  Fig. 7.36
 ∂x   ∂y
∂y 

Example 10
If z = f (u , v) where u = x 2 + y 2 , v = 2 xy, show that
∂z ∂z  ∂z 
x −y = 2 u 2 − v2   .
∂x ∂y  ∂u 
Solution
z = f (u , v),
v), u = x 2 + y 2 , v = 2 xy
∂z ∂z ∂u ∂z ∂v
= ⋅ + ⋅
∂x ∂u ∂x ∂v ∂x
∂z ∂z z
= ⋅ 2x + ⋅ 2 y
∂u ∂v
∂z ∂z ∂z
x = 2x2 + 2 xy ...(1)
∂x ∂u ∂v u v

∂z ∂z ∂u ∂z ∂v
and = ⋅ + ⋅
∂y ∂u ∂y
∂y ∂y ∂v ∂y
∂y
x
∂z ∂z Fig. 7.37
= ⋅ 2 y + ⋅ 2x
∂u ∂v z
∂z ∂z ∂z
y = 2 y2 + 2 xy
∂y ∂u ∂v ...(2)
u v
Substracting Eq. (2) from Eq. (1),
∂z ∂z ∂z
x −y = 2( x 2 − y 2 ) y
∂x ∂y ∂u
 ∂z  Fig. 7.38
= 2 ((xx 2 + y 2 ) 2 − 4 x 2 y 2  
 ∂u 
 ∂z 
= 2 ((u
u 2 − v2 )  
 ∂u 
7.7 Total Derivatives 7.85

Example 11
If z = f (u , v),
v and u = x 2 − y 2 , v = 2 xy, show that
1 
 ∂z  
2 2 2 2
 ∂z   ∂z  2 2  ∂z 
  +   = 4(u + v )   +    .
2

∂x  ∂∂yy   ∂u ∂v 
Solution z
z = f (u , v),
v and u = x 2 − y 2 , v = 2 xy
∂z ∂z ∂u ∂z ∂v
= ⋅ + ⋅
∂x ∂u ∂x ∂v ∂x u v
∂z ∂z
= ⋅ 2x + ⋅ 2 y
∂u ∂v
 ∂z ∂z 
= 2 x + y  ...(1) x
 ∂u ∂v 
Fig. 7.39
∂z ∂z ∂u ∂z ∂v
and = ⋅ + ⋅ u
∂y ∂u ∂y
∂y ∂y ∂v ∂y
∂y
∂z ∂z
∂z
= ( −2 y ) + (2 x)
∂u ∂v
x z y
 ∂ z ∂z 
= 2 −y +x  ...(2)
 ∂u ∂v 
Squaring and adding Eq. (1) and Eq. (2),
2
t
2 2 2
 ∂z   ∂z   ∂z ∂z   ∂z ∂z 
  +   = 4  x + y  + 4  − y +x 
∂x  ∂∂yy  ∂u ∂v ∂u ∂v  Fig. 7.40

  ∂z  2  ∂z 
2
∂z ∂z
= 4  x 2   + y 2   + 2 xy ⋅
  ∂ u   ∂ v  ∂ u ∂v

∂z ∂z 
2 2
 ∂z   ∂z 
+ y 2   + x 2   − 2 xy ⋅ 
 ∂u   ∂v  ∂u ∂v 

  ∂z  2  ∂z  2 
= 4((xx 2 + y 2 )   +   
 ∂u   ∂v  
1 
 ∂z  
2 2
2 2 2  ∂z 
 
= 4 ((xx + y )    +   
2

  ∂u   ∂v  
1 
 ∂z   ∂z  
2 2

= 4 ( x 2 − y 2 ) 2 + 4 x 2 y 2  2   +   
  ∂u   ∂v  
1 
 ∂z   ∂z  
2 2

= 4(u 2 + v 2 ) 2   +   
  ∂u   ∂v  
7.86 Chapter 7 Partial Derivatives

Example 12
If x u
= eu v then show that
2 2 2 2
∂z ∂z −2 u ∂z ∂z
e − sin v 2

∂x ∂u ∂v
z
y
Solution
y
z eu cosec v y eu v x

∂z ∂z ∂x ∂z y
= +
∂u ∂x ∂u y ∂u u
∂z u ∂z
= e v e v Fig. 7.41
∂x y z

∂z ∂z ∂x ∂z y
and ⋅ +
∂v ∂x ∂v y ∂v
x y
∂z ∂z
= 2
)
∂x y
v

2 2 2 2
∂z z ∂z ∂
e 2u
− n v e u 2
e cosec v e v
∂u ∂v ∂x y Fig. 7.42

∂z ∂z 2 u
+2 e v cot v
∂x y
2
∂z
+ in 2 v) (e 2 2
v cot )
x
2
∂z
in 2 v) e cosec v
y
u 
in v e v ot v
x y
2 2
∂z ∂z
) (co
∂x y
2 2
∂z ∂z
∂ y

Example 13
u u
If co prove that

2 2 2 2
∂z ∂z ∂z ∂z ∂z −2 u ∂z ∂z
(i) x +y = e 2u (ii) + e +
∂v ∂u y ∂x y ∂u ∂v
7.7 Total Derivatives 7.87

Solution
z = f ( x, y ), x = eu cos v, y = eu sin v

∂z ∂z ∂x ∂z ∂y ∂y
(i) = ⋅ + ⋅
∂u ∂x ∂u ∂y∂y ∂u z
∂z u ∂z
∂z
= ⋅ e cos v + eu sin v
∂x ∂y
∂y
∂z ∂z y
=x +y x
∂x ∂y
∂y
∂z ∂z ∂z
y = xy + y 2 ...(1) u
∂u ∂x ∂y
Fig. 7.43
∂z ∂z ∂x ∂z ∂y
∂y
and = ⋅ + ⋅
∂v ∂x ∂v ∂y
∂y ∂v
∂z ∂z
∂z
= (−
−eeu sin v) + eu cos v
∂x ∂∂yy z
∂z ∂z
= −y + x
∂x ∂y
∂y
x y
∂z ∂z ∂z
x = − xy + x 2 ...(2)
∂v ∂x ∂y
Adding Eqs (1) and (2), v

∂z ∂z ∂z Fig. 7.44
x +y = ( y 2 + x2 )
∂v ∂u ∂y
∂ z
= e 2u
∂y


−2 u  ∂z 
−2
2
 ∂z  
2
−2 u
 2  ∂z  2 2  ∂z 
2
∂z ∂z 
(ii) e   +    = e  x   + y   + 2 xy ⋅ 
 ∂u   ∂v     ∂x   ∂y  ∂x ∂y
∂y 
  ∂z  2  ∂z 
2
∂z ∂z 
+  y 2   + x 2   − 2 xy ⋅ 
  ∂x   ∂y
∂y  ∂x ∂y
∂y 

  ∂z 
2
 ∂z  
2

= e −2u ( x 2 + y 2 )   + ( x 2 + y 2 )   
  ∂x   ∂∂yy  
  ∂z  2  ∂z  2   ∂z  2  ∂z 
2
−2 u 2 u
=e e   +    =   +  
 ∂x   ∂y ∂y    ∂x   ∂y ∂y 
7.88 Chapter 7 Partial Derivatives

Example 14
 y − x z − x ∂u ∂u ∂u
If u = f  ,  , show that x 2 + y2 + z2 = 0.
 xy xz  ∂x ∂y ∂z
[Summer 2016]
Solution
y−x 1 1 z−x 1 1
Let l= = − ,m= = −
xy x y xz x z
∂l 1 ∂m 1
=− 2, =− 2,
∂x x ∂x x
∂l 1 ∂m
= 2, = 0,
∂y
∂y y ∂y
∂y
∂l ∂m 1
= 0, =
∂z ∂z z 2
 y − x z − x
x
u= f , = f (l , m) u
 xy xz 
∂u ∂u ∂l ∂u ∂m
= ◊ + ◊
∂x ∂l ∂x ∂m ∂x
l m
∂u Ê 1 ˆ ∂u Ê 1 ˆ
= - + -
∂l ÁË x 2 ˜¯ ∂m ÁË x 2 ˜¯

∂u  ∂u ∂u  x
x2 = − +
 ∂l ∂m 
…(1)
∂x
Fig. 7.45
∂u ∂u ∂l ∂u ∂m
Also, = ⋅ + ⋅ u
∂y ∂l ∂y
∂y ∂y ∂m ∂y
∂y
∂u  1  ∂u
= + ⋅0
∂l  y 2  ∂m l m

∂u ∂u
y2 = …(2)
∂y ∂l y

∂u ∂u ∂l ∂u ∂m Fig. 7.46
and = ⋅ + ⋅
∂z ∂l ∂z ∂m ∂z u
∂u ∂u  1 
= ⋅0 +  
∂l ∂m  z 2 
l m
∂u ∂u
z2
= …(3)
∂z ∂m
z
Fig. 7.47
7.7 Total Derivatives 7.89

Adding Eqs (1), (2) and (3),

∂u ∂u ∂u  ∂u ∂u  ∂u ∂u
x2 + y2 + z2 = − + + + =0
∂x ∂y ∂z  ∂l ∂m  ∂l ∂m

Example 15
∂u ∂u ∂u
If u = f ( x − y, y − z , z − x) then show that + + = 0.
∂x ∂y
∂y ∂z
Solution
Let x − y = l, y − z = m, z− x = n

∂l ∂m ∂n
= 1, = 0, = −1,
∂x ∂x ∂x
∂l ∂m ∂n
= −1, = 1, = 0,
∂y
∂y ∂∂yy ∂y
∂y
∂l ∂m ∂n
=0 = −1 =1
∂z ∂z ∂z
u
u = f ( x − y, y − z , z − x) = f (l , m, n)
∂u ∂u ∂l ∂u ∂m ∂u ∂n
= + +
∂x ∂l ∂x ∂m ∂x ∂n ∂x l m n
∂u ∂u ∂u
= ⋅1 + ⋅0 + ( −1
−1)
∂l ∂m ∂n
x
∂u ∂u
= −
∂l ∂n ... (1) Fig. 7.48
u
∂u ∂u ∂l ∂u ∂m ∂u ∂n
also, = + +
∂y ∂l ∂y
∂y ∂y ∂m ∂y ∂y ∂n ∂y ∂y
∂u ∂u
∂u ∂u l m n
= (−
−11) + (1) + ⋅ 0
∂l ∂m ∂n
∂u ∂u
=− + ... (2)
y
∂l ∂m
Fig. 7.49
∂u ∂u ∂l ∂u ∂m ∂u ∂n
and = + + u
∂z ∂l ∂z ∂m ∂z ∂n ∂z

∂u ∂u
∂u ∂u
= ⋅0 + ( −1) + (1) l m n
∂l ∂m ∂n
∂u ∂u
=− + ... (3)
∂m ∂n z
Fig. 7.50
7.90 Chapter 7 Partial Derivatives

Adding Eqs (1), (2) and (3),


∂u ∂u ∂u
+ + =0
∂x ∂y ∂z

Example 16
∂w ∂w r
Find , in terms of r and s if w = x + 2y + z2, where x = ,
∂r ∂s s
y = r2 + log s, z = 2r. [Winter 2014]
Solution
r
x= , y = r 2 + log s, z = 2r
s
∂x 1 ∂y ∂z
= , = 2r , =2
∂r s ∂r ∂r
∂x r ∂y 1 ∂z
=- 2, = , =0
∂s s ∂s s ∂s w
2
w = x + 2y + z
∂w ∂w ∂x ∂w ∂y ∂w ∂z
= ◊ + ◊ + ◊ x y z
∂r ∂x ∂r ∂y ∂r ∂z ∂r
Ê 1ˆ
= (1) Á ˜ + (2)(2r ) + (2 z)(
z )(2)
Ë s¯ r

1 Fig. 7.51
= + 4r + 4 z w
s
1
= + 4r + 44((2r )
s
x y z
1
= + 12 s
s
∂w ∂w ∂x ∂w ∂y ∂w ∂z s
= ◊ + ◊ + ◊
∂s ∂x ∂s ∂y ∂s ∂z ∂s
Fig. 7.52
Ê rˆ Ê 1ˆ
= (1) Á - 2 ˜ + (2) Á ˜ + (2 z ))((0)
Ë s ¯ Ë s¯
r 2
=- 2
+
s s
7.7 Total Derivatives 7.91

Example 17
 x y z ∂u ∂u ∂u
If u = f  , ,  , prove that x + y +z = 0. [Winter 2013]
 y z x ∂x ∂y ∂z

Solution
x y z
Let = l, = m, =n
y z x
∂l 1 ∂m ∂n z
= , = 0, =- 2,
∂x y ∂x ∂x x
∂l - x ∂m 1 ∂n
= , = = 0,
∂y y 2 ∂y z ∂y
∂l ∂m - y ∂n 1
=0 = 2 , = u
∂z ∂z z ∂z x

∂u ∂u ∂l ∂u ∂m ∂u ∂n
= ⋅ + ⋅ + ⋅ l m n
∂x ∂l ∂x ∂m ∂x ∂n ∂x
∂u 1 ∂u ∂u  − z 
= ⋅ + ⋅0 +  
∂l y ∂m ∂n  x 2  x
∂u x ∂u z ∂u
x = ⋅ − ⋅ ...(1) Fig. 7.53
∂x y ∂l x ∂n
u
∂u ∂u ∂l ∂u ∂m ∂u ∂n
Also, = ⋅ + ⋅ + ⋅
∂y ∂l ∂y
∂y ∂y ∂m ∂y
∂y ∂n ∂y
∂y
m n
∂u  − x  ∂u 1 ∂u l
= + ⋅ + ⋅0
∂l  y 2  ∂m z ∂n
∂u x ∂u y ∂u
y =− ⋅ + ⋅ ...(2) y
∂y
∂y y ∂l z ∂m Fig. 7.54
u
∂u ∂u ∂l ∂u ∂m ∂u ∂n
and = ⋅ + ⋅ + ⋅
∂z ∂l ∂z ∂m ∂z ∂n ∂z
∂u ∂u  − y  ∂u 1 l m n
= ⋅0 +  + ⋅
∂l ∂m  z 2  ∂n x
∂u − y ∂u z ∂u
z = + ⋅ z
∂z z ∂m x ∂n ...(3)
Fig. 7.55
Adding Eqs (1), (2) and (3),
∂u ∂u ∂u
Hence, x +y +z =0
∂x ∂y ∂z
7.92 Chapter 7 Partial Derivatives

Example 18
1 ∂u 1 ∂u 1 ∂u
If u = f ( x 2 − y 2 , y 2 − z 2 , z 2 − x 2 ) , prove that + + = 0.
x ∂x y ∂y z ∂z
Solution u
Let l = x2 − y 2 , m = y 2 − z 2 , n = z 2 − x2
∂l ∂m ∂n
= 2 x, = 0, = −2 x
∂x ∂x ∂x l m n
∂l ∂m ∂n
= −2 y, = 2 y, =0
∂y
∂y ∂y
∂y ∂y
∂y
∂l ∂m ∂n x
= 0, = −2 z , = 2z
∂z ∂z ∂z Fig. 7.56
u = f (x2 – y2, y2 – z2, z2 – x2) = f (l, m, n)
∂u ∂u ∂l ∂u ∂m ∂u ∂n
= ⋅ + ⋅ + ⋅
∂x ∂l ∂x ∂m ∂x ∂n ∂x
∂u ∂u ∂u
= ⋅ 2x + ⋅ 0 + ⋅ ( −2 x)
∂l ∂m ∂n
1 ∂u ∂u ∂u u
= 2 −2 ...(1)
x ∂x ∂l ∂n

∂u ∂u ∂l ∂u ∂m ∂u ∂n l m n
Also, = ⋅ + ⋅ + ⋅
∂y ∂l ∂y
∂y ∂y ∂m ∂y
∂y ∂n ∂y
∂y
∂u ∂u
∂u ∂u
= ( −2 y ) + ( 2 y ) + ( 0) y
∂l ∂m ∂n
1 ∂u ∂u ∂u Fig. 7.57
= −2 + 2 ...(2)
y ∂y
∂y ∂l ∂m
∂u ∂u ∂l ∂u ∂m ∂u ∂n u
and = ⋅ + ⋅ + ⋅
∂z ∂l ∂z ∂m ∂z ∂n ∂z
∂u ∂u
∂u ∂u
=
⋅0 + ( −2 z ) + (2 z ) l m n
∂l ∂m ∂n
1 ∂u ∂u ∂u
= −2 +2 ...(3)
z ∂z ∂m ∂n z
Adding Eqs (1), (2) and (3), Fig. 7.58
1 ∂u 1 ∂u 1 ∂u
+ + =0
x ∂x y ∂y z ∂z
7.7 Total Derivatives 7.93

Example 19
If u = f ( x 2 + 2 yyzz , y 2 + 2 zx), prove that
∂u ∂u ∂u
( y 2 − zx ) + ((xx 2 − yz ) + ((zz 2 − xy ) = 0.
∂x ∂y ∂z
Solution
Let x 2 + 2 yyzz = l , y 2 + 2 zzxx = m u

∂l ∂l ∂l
= 2 x, = 2 z, = 2y
∂x ∂∂yy ∂z
m
∂m ∂m ∂m l
= 2 z, = 2 y, = 2x
∂x ∂∂yy ∂z
∂u ∂u ∂l ∂u ∂m
= ⋅ + ⋅ x
∂x ∂l ∂x ∂m ∂x
Fig. 7.59
∂u ∂u
= ⋅ 2x + ⋅ 2z
∂l ∂m
∂u
∂u ∂u ∂u
( y 2 − zx
zx ) = (2 xy 2 − 2
2xx 2 z ) + (2 y 2 z − 2 z 2 x) ... (1)
∂x ∂l ∂m
∂u ∂u ∂l ∂u ∂m u
Also, = ⋅ + ⋅
∂y ∂l ∂y ∂m ∂y
∂u ∂u
= ⋅ 2z + ⋅2y
∂l ∂m l m

∂u ∂u ∂u
( x 2 − yz ) = (2 x 2 z − 2 yz 2 ) + (2 x 2 y − 2 y 2 z ) ... (2)
∂y ∂l ∂m y
∂u ∂u ∂l ∂u ∂m
and = ⋅ + ⋅ Fig. 7.60
∂z ∂l ∂z ∂m ∂z
∂u ∂u
= ⋅2y + ⋅ 2x u
∂l ∂m
∂u
∂u ∂u ∂u
( z 2 − xy
xy ) = (2 yz 2 − 2 xy 2 ) + (2
2xy 2zz 2 x − 2 x 2 y ) ... (3)
∂z ∂l ∂m
l m
Adding Eqs (1), (2) and (3),

∂u ∂u ∂u
Hence, ( y 2 − zx) + ((xx 2 − yz ) + ((zz 2 − xy ) =0
∂x ∂y ∂z z
Fig. 7.61
7.94 Chapter 7 Partial Derivatives

Example 20
∂u ∂u ∂u
If u = f (e y − z , e z − x , e x−
x y
), show that + + = 0.
∂x ∂y
∂y ∂z

Solution
Let l = ey−z , m = ez − x , n = ex− y

∂l ∂m ∂n
= 0, = −e z − x = − m, = ex− y = n
∂x ∂x ∂x
∂l ∂m ∂n
= e y − z = l, = 0, = −e x − y = − n
∂y
∂y ∂y
∂y ∂y
∂y
∂l ∂m ∂n
= − e y − z = −l , = e z − x = m, =0
∂z ∂z ∂z
u
u = f (e y − z , e z − x , e x − y ) = f (l , m, n).
∂u ∂u ∂l ∂u ∂m ∂u ∂n
= ⋅ + ⋅ +
∂x ∂l ∂x ∂m ∂x ∂n ∂x
l m n
∂u ∂u ∂u
= ⋅0 + ⋅ ( − m) + ⋅ n
∂l ∂m ∂n
∂u ∂u
= −m +n ...(1) x
∂m ∂n
Fig. 7.62

∂u ∂u ∂l ∂u ∂m ∂u ∂n
= ⋅ + ⋅ + ⋅ u
∂y ∂l ∂y
∂y ∂y ∂m ∂y ∂y ∂n ∂y ∂y
∂u ∂u ∂u
= ⋅l + ⋅ 0 + ⋅ ( − n)
∂l ∂m ∂n
l m n
∂u ∂u
=l −n ...(2)
∂l ∂n

Fig. 7.63
7.7 Total Derivatives 7.95

∂u ∂u ∂l ∂u ∂m ∂u ∂n u
and = ⋅ + ⋅ + ⋅
∂z ∂l ∂z ∂m ∂z ∂n ∂z
∂u ∂u ∂u
= ( −l
−l ) + ⋅m + ⋅0
∂l ∂m ∂n l m n
∂u ∂u
= −l +m ...(3)
∂l ∂m
Adding Eqs (1), (2) and (3), z
Fig. 7.64
∂u ∂u ∂u
+ + =0
∂x ∂y
∂y ∂z

Example 21
If x = vw , y = wu , z = uv and f is a function of x, y and z, prove
∂φ ∂φ ∂φ ∂φ ∂φ ∂φ
that x +y +z =u +v +w .
∂x ∂y ∂z ∂u ∂v ∂w
Solution
x = vw , y = wu , z = uuv
∂x ∂∂yy 1 w ∂z 1 v
= 0, = , =
∂u ∂u 2 u ∂u 2 u
∂x 1 w ∂y
∂y ∂z 1 u
= , = 0, =
∂v 2 v ∂v ∂v 2 v
f
∂x 1 v ∂y 1 u
∂y ∂z
= , = , =0
∂w 2 w ∂w 2 w ∂w
∂φ ∂φ ∂x ∂φ ∂∂yy ∂φ ∂z
= ⋅ + ⋅ + ⋅ x y z
∂u ∂x ∂u ∂y ∂u ∂z ∂u
∂φ ∂φ 1 w ∂φ 1 v
= ⋅0 + ⋅ + ⋅
∂x ∂y 2 u ∂z 2 u u

∂φ 1  ∂φ ∂φ  Fig. 7.65
u =  uw + uv 
∂u 2  ∂y ∂z 
1  ∂φ ∂φ 
= y +z  ...(1)
2  ∂y ∂z 

∂φ ∂φ ∂x ∂φ ∂∂yy ∂φ ∂z
Also, = ⋅ + ⋅ + ⋅
∂v ∂x ∂v ∂y ∂v ∂z ∂v
7.96 Chapter 7 Partial Derivatives

f
∂φ 1 w ∂φ ∂φ 1 u
= ⋅ + ⋅0 +
∂x 2 v ∂y ∂z 2 v
∂φ 1  ∂φ ∂φ 
v =  vw + uv  x y z
∂v 2  ∂x ∂z 
1  ∂φ ∂φ 
=  x +z  ...(2) v
2 ∂x ∂z 
Fig. 7.66
∂φ ∂φ ∂x ∂φ ∂∂yy ∂φ ∂z
and = ⋅ + ⋅ + ⋅
∂w ∂x ∂w ∂y ∂w ∂z ∂w f

∂φ 1 v ∂φ 1 u ∂φ
= ⋅ + + ⋅0
∂x 2 w ∂y 2 w ∂z x y z
∂φ 1  ∂φ ∂φ 
w = vw + uw 
∂w 2  ∂x ∂y
∂y 
w
1  ∂φ ∂φ 
= x +y  ...(3) Fig. 7.67
2  ∂x ∂y 
∂y

Adding Eqs (1), (2) and (3),

∂φ ∂φ ∂φ ∂φ ∂φ ∂φ
u +v +w =x +y +z
∂u ∂v ∂w ∂x ∂y ∂z

Example 22
∂z ∂z
If f ( xxyy 2 , z − 2 x)
x) = 0, show that 2
2x −y = 4 x.
∂x ∂y
Solution
Let l = xxyy 2 , m = z − 2 x, f

f (l , m) = 0

∂∂ff ∂f
∂f ∂l ∂f ∂f ∂m l m
= . + .
∂x ∂l ∂x ∂m ∂x
∂f 2
∂f ∂f  ∂z
∂f 
0= (y ) +  − 2 ∵ f ( xy x) = 0
xy 2 , z − 2 x) x y x z
∂l ∂m  ∂x

∂∂ff ∂z
2− x y
∂l = ∂x …(1)
∂f
∂f y2 Fig. 7.68
∂m
7.7 Total Derivatives 7.97

∂∂ff ∂f
∂f ∂l ∂f
∂f ∂m
and = ⋅ + ⋅ =0
∂y ∂l ∂y
∂y ∂y ∂m ∂y∂y
∂∂ff ∂f  ∂z 
∂f
(2 xy ) + =0
∂l ∂m  ∂y
∂y 
∂∂ff ∂z
∂l = − ∂y
∂y
…(2)
∂f
∂f 2 xy
∂m
From Eqs (1) and (2),
∂z ∂z
2−

∂y
∂x = − y
2
y 2 xy
∂z ∂z
4x − 2x = − y
∂x ∂y
∂y

∂z ∂z
Hence, 2x −y = 4x
∂x ∂y

Example 23
 z y ∂z ∂z
If f  3 ,
x  = 0, prove that x + y = 3 z.
x ∂x ∂y
f
Solution
 z y
Let l =  3,m = l m
x  x
f (l , m) = 0.
z x x y

∂f ∂f ∂l ∂f ∂m x y
= ◊ + ◊
∂x ∂l ∂x ∂m ∂x
Fig. 7.69
∂f Ê -3 z 1 ∂z ˆ ∂f Ê y ˆ È Ê z yˆ ˘
0= Á + 3 ˜+ Á- ˜ Í∵ f ÁË 3 , ˜¯ = 0˙
∂l Ë x 4 x ∂x ¯ ∂m Ë x 2 ¯ Î x x ˚
∂∂ff  3 z 1 ∂z  ∂f ∂f  y 
 − 4 + 3  =  
∂l x x ∂x ∂m  x 2 
∂f 
∂f ∂f 2
∂z  ∂f
 −3
−3 z + x  = ( x y) …(1)
∂l  ∂x  ∂m
7.98 Chapter 7 Partial Derivatives

∂∂ff ∂f
∂f ∂l ∂f
∂f ∂m
= . + .
∂y ∂l ∂y
∂y ∂y ∂m ∂y∂y
∂∂ff  1 ∂z  ∂f ∂f  1 
and 0=   +  
∂l  x ∂y
3
∂y  ∂m  x 
∂∂ff  1 ∂z  ∂f  1 
∂f
=−  
∂l  x 3 ∂y
∂y  ∂m  x 
∂∂ff  ∂z  ∂f 2
∂f
 ∂y  =− (x ) ...(2)
∂l ∂y  ∂m
Dividing Eq. (1) by Eq. (2),
∂z
−3 z + x 2
∂x = x y
∂z − x2
∂y
∂y
∂z ∂z
−3 z + x = − y
∂x ∂y
∂y
∂z ∂z
Hence, x +y = 3z
∂x ∂y

Example 24
If f (llxx + m
myy + nnzz , x 2 + y 2 + z 2 ) = 0, prove that
∂z ∂z
ly − mx) + ((ny
((ly ny − mz ) + (lz (lz − nx) = 0.
∂x ∂y
f
Solution
Let u = llxx + m
myy + nz
nz , v = x 2 + y 2 + z 2
f (u , v) = 0 u v

∂∂ff ∂f∂f ∂u ∂f ∂f ∂v
= ⋅ + ⋅
∂x ∂u ∂x ∂v ∂x x,, z

∂∂ff  ∂z  ∂∂ff  ∂z 
0=  l + n  +  2 x + 2 z 
x y
∂u  ∂x ∂v ∂x
Fig. 7.70
∂f  ∂z ∂f  ∂z
 l + n  = −  2 x + 2 z 
∂u ∂x ∂v ∂x …(1)
∂∂ff ∂f
∂f ∂u ∂f
∂f ∂v
and = ⋅ + ⋅
∂y ∂u ∂y
∂y ∂y ∂v ∂y
∂y

∂∂ff  ∂z  ∂∂ff  ∂z 
0=  m + n  +  2 y + 2z 
∂u  ∂y  ∂v 
∂y ∂y 
∂y
7.7 Total Derivatives 7.99

∂∂ff  ∂z  ∂f 
∂f ∂z  f
m + n  = −  2 y + 2z  …(2)
∂u  ∂y 
∂y ∂v  ∂y 
∂y

Dividing Eq. (1) by Eq. (2),


u v
∂z ∂z
l+n
x+ z
∂x = ∂x
∂z ∂z
m+n y+z
∂y ∂y y,, z
∂z ∂z ∂z ∂z ∂z ∂z ∂z ∂z
ly + lz + ny + nz . = mx + nx + mz + nz ⋅ x y
∂y ∂x ∂x ∂y ∂y ∂x ∂x ∂y
Fig. 7.71
∂z ∂z
ly − mx) + ((ny
Hence, ((ly ny − mz ) + (lz
(lz − nx) = 0
∂x ∂y

Example 25
If u is a function of x and y and x and y are functions of r and q given by
x = er cos q, y = er sin q, show that
  ∂u  
2 2 2 2
 ∂u   ∂u  −2 r  ∂u 
  +   = e   +   
∂x  ∂∂yy   ∂r ∂θ 
u

Solution
u = f (x, y), x = er cos q, y = er sin q x y

∂u ∂u ∂x ∂u ∂y ∂y
= ⋅ + ⋅
∂r ∂x ∂r ∂y ∂y ∂r
r
∂u r ∂u r
∂u
= e cos θ + e sin θ
∂x ∂y
∂y Fig. 7.72
∂u ∂u
=x +y …(1) u
∂x ∂y
∂y

∂u ∂u ∂x ∂u ∂y∂y
Again, = ⋅ + ⋅ x y
∂θ ∂x ∂θ ∂∂yy ∂θ
∂u ∂u r
∂u
= −ee r sin θ ) +
(− e cos θ q
∂x ∂∂yy
∂u ∂u Fig. 7.73
= −y +x …(2)
∂x ∂y
∂y
7.100 Chapter 7 Partial Derivatives

Squaring and adding Eqs (1) and (2),

2 2 2 2 2
 ∂u   ∂u  2  ∂u  ∂u ∂u  ∂u   ∂u 
  +   = x   + 2 xy + y2   + y2  
 ∂x 
∂r ∂θ ∂x ∂x ∂y
∂y  ∂∂yy 
2
∂u ∂u  ∂u 
− 2 xy + x2  
∂x ∂y
∂y  ∂y
∂y 
  ∂u  2    ∂u  2 
= ( x + y )    + ((xx + y )   
2 2 2 2

  ∂x    ∂y
∂y  

  ∂u  2  ∂u  2 
= ((xx 2 + y 2 )   +   
 ∂x   ∂y ∂y  
  ∂u  2  ∂u  2

= e2 r   +   
 ∂x   ∂y ∂y  
  ∂u  2  ∂u  2    ∂u  2  ∂u  2 
  +    = e −2 r   +   
 ∂x   ∂y     ∂r   ∂θ  

Example 26
If x + y = 2eθ cos φ and x − y = 2ieθ sin
n φ , prove that
∂v ∂v ∂v
+i = 2y
∂θ ∂φ ∂y
∂y v

Solution
x + y = 22eeθ cos φ , x − y = 2iieeθ sin φ y
x
2 x = 2eθ (cos φ + i sin φ )
x = eθ eiφ = eθ + iφ
q
∂x
= eθ + iφ = x
∂θ Fig. 7.74
∂x
= ieθ + iφ = ix v
∂φ
And 2 y = 2eq ( cos q - i sin f )
y = eq e - i f = eq - i f x y

∂y
∂y
= eθ − iφ = y
∂θ f
∂y
∂y Fig. 7.75
= −ieθ − iφ = −iy
∂φ
7.7 Total Derivatives 7.101

Let v = f (x, y)
∂v ∂v ∂x ∂v ∂y∂y
= ⋅ + ⋅
∂θ ∂x ∂θ ∂∂yy ∂θ
∂v ∂v
= x+ y …(1)
∂x ∂y
∂y
∂v ∂v ∂x ∂v ∂y ∂y
= ⋅ + ⋅
∂φ ∂x ∂φ ∂y ∂y ∂φ
∂v ∂v
= (ix) + ( −iy
−iy )
∂x ∂y
∂y
 ∂v ∂v 
= ix − y  …(2)
 ∂x ∂y 
∂y
∂v ∂v ∂v ∂v  ∂v ∂v 
+i = x + y + i2  x − y 
∂θ ∂φ ∂x ∂y
∂y  ∂x ∂y 
∂y
∂v ∂v ∂v ∂v
=x +y −x +y
∂x ∂y
∂y ∂x ∂y
∂y
∂v
= 2y
∂y
∂y

Example 27
∂2 z ∂2 z
If z = f ( x, y)
y where x = log u , y = log v, show that = uv .
∂x ∂y ∂u ∂v
Solution
z
z = f ((xx, y ), x = log u , y = log v
∂z ∂z ∂x ∂z ∂y ∂y
= ⋅ + ⋅
∂u ∂x ∂u ∂y ∂y ∂u x y
∂z 1 ∂z
= ⋅ + ⋅0
∂x u ∂y
∂y
1 ∂z u
=
u ∂x
Fig. 7.76
∂z ∂z ∂x ∂z ∂y ∂y
= ⋅ + ⋅
∂v ∂x ∂v ∂y ∂y ∂v
z
∂z ∂z 1
= ⋅0 +
∂x ∂∂yy v
1 ∂z x y
=
v ∂y
∂y
∂ 1 ∂
≡ v
∂v v ∂y
∂y
Fig. 7.77
7.102 Chapter 7 Partial Derivatives

∂  ∂z   1 ∂   1 ∂z 
 =  
∂v  ∂u   v ∂∂yy   u ∂x 
Now, ∂2 z 1 ∂2 z
=
∂v∂u uv ∂y∂x
∂2 z ∂2 z
= uv
∂ x∂ y ∂u ∂v
EXERCISE 7.3
 x dz 2
n−1   ,where
1. If z = tan , x = 2t, y = 1 – t2, prove that = .
y dt 1 + t 2
du
2. If u = x3 + y3, where x = a cos t, y = b sin t, find .
dt
[Ans. : −3a 3 cos2 t sin t + 3b 2 sin2 t cos t ]

3. If u =xeyz, where y = a 2 − x 2 , z = sin3 x, find du .


dx
 y  x2 
 Ans. : e z  1
1−− + 3x cot x  
  y 
r −x
4. If u = e l
, where r2 = x2 + y2 and l is a constant, show that
∂ 2 u ∂ 2 u 2 ∂u u
+ + ⋅ = .
∂x 2 ∂y 2 l ∂x lr
∂2u ∂2u
5. If u = log r and r = (x − a)2 + (y − b)2 , prove that + = 0 if a, b
∂x 2 ∂y 2
are constants.
∂2u ∂2u ∂2u
6. If u2 (x2 + y2 + z2) = 1, prove that + + = 0.
∂x 2 ∂y 2 ∂z 2
 1
Hint : Let x + y + z = r , u = r 
2 2 2 2

 
∂2u ∂2u ∂2u
7. If u = rm, where r = x 2 + y 2 + z 2 , find the value of + + .
∂x 2 ∂y 2 ∂z 2
[Ans. : m (m + 1) rm– 2]
8. If u = f (r), where is given by the relation x = r cos q, y = r sin q, prove
∂ 2 u ∂ 2 u d 2 f 1 df
df
that + 2 = 2 + .
∂x 2
∂y dr r dr
9. If z = f (u,v ), where u = x2 + y2, y = 2 then show that
∂z ∂z  ∂z 
x −y = 2 u2 + v 2   .
∂x ∂y  ∂u 
7.7 Total Derivatives 7.103

10. If z = f (u,v ), where u = x2 + y2, v = x2 – y2 then show that


∂z ∂z ∂z
(i) y +x = 4 xy .
∂x ∂y ∂u
 ∂z 
2
 ∂z 
2
  ∂z  2  ∂z  2  ∂z ∂z
(ii)   +   = 4u   +    + 8v ⋅
 ∂x   ∂y    ∂ u   ∂ v   ∂ u ∂v

y
n−1   , where x = 3u2 + 2v, y = 4u – 2v3, z = 2u2 – 3v2, find
11. If w = z sin
 x
∂w ∂w
∂w
and
d .
∂u ∂v
12. If w = (x2 +y –2)4 + (x – y +2)3, where x = u – 2v + 1 and y = 2u + v – 2,
∂w
find at u = 0, v = 0.
∂v
[Ans. : – 882]
u
13. If w = x + 2y + z 2 , x = , y = u2 + ev, z = 2u, show that
v
∂w ∂w
u +v = 12u 2 + 2vve v .
∂u ∂v
14. If F is a function of x, y, z then show that
∂F ∂F ∂F ∂F ∂F ∂F
u +v +w =x + 2y + 3z , where x = u + v + w, y = uv
∂u ∂v dw ∂x ∂y dz
+ vw + wu, z = uvw.
u
15. If z = f (x,y ), x = uv, y =
, prove that
v
∂z 1 ∂z 1 ∂z ∂z v ∂z v 2 ∂z
= + and
d = − .
∂x 2v ∂u 2u ∂u ∂y 2 ∂u 2u ∂v
16. If x = u + v, y = uv and F is a function of x, y, prove that
∂2 F ∂2 F ∂2 F ∂2 y ∂v
− 2 + = ( x 2
− 4 y ) −2 .
∂u 2 ∂u ∂v ∂v 2 ∂y 2 ∂y
  ∂ ∂   ∂F ∂F  
Hint: LHS =  −  − 
 ∂ u ∂ v   ∂u ∂v  
17. If u = f(xn – yn, yn – zn, zn – xn), prove that
1 ∂u 1 ∂u 1 ∂u
n −1
+ n −1 + n −1 = 0.
x ∂x y ∂y z ∂z

18. If z = f( y where x = u − av , y = u + av , prove that


f (x, y),
∂2 z ∂2 z 2 ∂ z
2
a2 − = 4 a .
∂u 2 ∂v 2 ∂x ∂y
7.104 Chapter 7 Partial Derivatives

19. If z = f( v where u = lx
f (u, v), lx + m
myy , v = ly − mx, prove that
∂2 z ∂2 z  ∂2 z ∂2 z 
+ 2 = (l 2 + m2 )  2 + 2  .
∂x 2
∂y  ∂u ∂v 
20. If x = u + av and y = u + bv , transform the equation
∂2 z ∂2 z ∂2 z ∂2 z
2 − 5 + 3 = 0 into the equation = 0, find the values
∂x 2 ∂x ∂y ∂y 2 ∂u ∂v
of a and b.  2
 Ans. : a = 1, b = 3 
 

∂2 z ∂2 z
21. If z = f (x, y),
y ), y = e x , v = e y , prove that = uv .
∂x ∂y ∂u ∂v
cos u sin u
22. If z = f (x, y),
y ), x = ,y = , prove that
v v
∂z ∂z ∂z ∂z
v − = (y − x ) − (y + x ) .
∂v ∂u ∂x ∂y
∂u ∂u ∂u
23. If u = f (2 x − 3y , 3y − 4 z, 4 z − 2 x), prove that 6 +4 +3 = 0.
∂x ∂y ∂z

24. If u = f(x2 + 2yz, y2 + 2zx), prove that


∂u ∂u ∂u
(y 2 − zx) + (x 2 − yz) + (z 2 − xy ) = 0.
∂x ∂y ∂z
25. If u = f(ax2 + 2hxy + by2) v = f (ax2 + 2hxy + by2) show that
∂  ∂v  ∂  ∂v 
 u  = u .
∂y  ∂x  ∂x  ∂y 
∂v ∂v ∂v
26. If x + y = 2eq cos f and x – y = 2ieq sin f, prove that +i = 2y .
∂θ ∂φ ∂y
27. Find the values of the constants and such that u = x + y and v = x + by
∂2 f ∂2 f ∂2 f ∂2 f
transform the equation 9 − 9 + 2 = 0 into = 0, where
∂x 2
∂x ∂y ∂y 2
∂u ∂v
f is a function of x and y.
 3 
 Ans. : a = 2 , b = 3
 
28. If x = y cosh q, y = r sinh q and z = f(x,y ), prove that
 ∂z ∂z  ∂z ∂z
(i) (x − y )  −  =r −
 ∂x ∂y  ∂r ∂θ

2  ∂ z ∂2 z  2 ∂ z ∂z ∂ 2 z
2 2

(ii) (x − y )  2 − 2  = r +r −
2

 ∂x ∂y  ∂r 2
∂r ∂θ 2
7.8 Implicit Differentiation 7.105

29. If x = e v sec u, y = e v tan u and z = f(


f (x, y),
y prove that
 ∂2 z ∂z   ∂2 z ∂2 z  ∂2 z
cosssu  −  = xy  2 + 2  + (x(x 2 + y 2 ) .
 ∂u ∂v ∂u   ∂x ∂y  ∂x ∂y

∂z ∂z
30. If f (x 2 y 3, z − 3x)
x) = 0, prove that 3x − 2y = 9 x.
∂x ∂y
∂z ∂z
31. If f (y + z, x 2 + y 2 + z 2 ) = 0, prove that (y − z) −x = x.
∂x ∂y
∂z ∂z
x − az
32. If f (ccx az, ccyy − b
bz) = 0, prove that a −b = c.
∂x ∂y

cos θ nθ
sin
si  ∂x   ∂u 
33. If x = ,y = , prove that     = coss2 θ.
u u  ∂u  θ  ∂x  y

34. If x 2 = au
au + b
bvv , y 2 = au − bv , prove that
 ∂u   ∂x  1  ∂v   ∂y 
 ∂x   ∂u  = 2 =  ∂y   ∂v  .
y v x u

35. If u = ax
ax + b
byy , v = bx − ay , prove that
 ∂y   ∂v  a 2 + b2  ∂u   ∂x  a2
(i)     = (ii)     = 2
 ∂v  x  ∂y  u a2  ∂x  y  ∂u  v a + b 2

7.8 IMPLICIT DIFFERENTIATION


Any function of the type f (x, y) = c is called an implicit function, where y is a function
of x and c is a constant. ∂f
dy ∂x
If f (x, y) = c then =−
dx ∂f
∂y
Proof If f (x, y) is a function of x and y, where y is a function of x then total differential
coefficient of f w.r.t. x is given by
d f ∂f dx dx ∂f dy
dy
= ⋅ + ⋅ f
dx ∂x dx
dx dx ∂y dx
dx
But y) = c
f ( x, y)
dff
d x y
=0
dx
dx
∂f dx
∂f dx ∂f
∂f dy
dy x
⋅ + ⋅ =0
∂x dx
dx ∂y
∂y dx
dx
Fig. 7.78
7.106 Chapter 7 Partial Derivatives

∂f
dy ∂x
=−
dx ∂f
∂y

Example 1
ddyy
If y log(cos x) = x log(sin y ), find .
ddxx
Solution
Let f ( x, y ) = y log(cos
og(cos x) − x log(sinn y )
∂f
= − ∂x
dy
dy
dx
dx ∂f
∂y
1
y ( − sin xx)) − log(sin y )
=− cos x
x
log cos x − ⋅ cos y
sin y
y tan x + log sin y
=
log cos x − x cot y

Example 2
dyy
d
If x3 + y 3 = 3axy
ax , find .
dx
dx
Solution
Let f (x, y) = x 3 + y 3 − 3axy
ax

∂f
= − ∂x
dyy
d
dx ∂f
∂y
3 x 2 − 3ay
a
=−
3 y 2 − 3ax
a
x 2 − ay
a
=−
y − ax
2
a
ay − x 2
=
y 2 − ax
a
7.8 Implicit Differentiation 7.107

Example 3
If y sin x = x cos y,
y find
ddyy
.
dx
dx
Solution
Let f (x, y) = y sin x – x cos y
∂f
= − ∂x
dyy
d
dx
dx ∂f
∂y
y cos x − cos y
=−
sinn x + x sin y
coss y − y cos x
=
n x + x sin y
sin

Example 4
If x y + y x = c, find d
dyy
.
dx
dx
Solution
Let f (x, y) = x y + y x − c

∂f
= − ∂x
dyy
d
dx
dx ∂f
∂y
yx y −1 + y x log y
=−
x y log x + xy x −1

Example 5
ddyy
If (cos xx)) y = ((si
sin y ) x , find
sin . [Winter 2014]
dx
dx
Solution
(cos xx)) y = ((si
sin y ) x
sin
Taking logarithm of both the sides,
y log cos x = x log sin y
Let f ( x, y ) = y log cos
cos x − x log sin y
7.108 Chapter 7 Partial Derivatives

∂f
= − ∂x
ddyy
dx
dx ∂f
∂y
y
( − sin xx)) − log sin y
= − cos x
x
log cos x − (cos y )
sin y
an x + log
y tan log sin
sin y
=
log cos x − x cot y

Example 6
dyy
d
If ax 2 + 2hx
hxyy + bbyy 2 + 2 g
gxx + 2 ffyy + c = 0, find .
dx
dx
Solution
Let f ( x, y ) = a
axx 2 + 2hxy + by 2 + 2 gx + 2 fy + c
hxy
∂f
= − ∂x
dyy
d
dx
dx ∂f
∂y
2(ax + hy + g )
=−
2(hx + by + f )
ax + hy + g
=−
hx + by + f

Example 7
du
If u = sin( x 2 + y 2 ) and a 2 x 2 + b 2 y 2 = c 2 , find .
dx
dx
Solution
u = sin( x 2 + y 2 ) and a 2 x 2 + b 2 y 2 = c 2
∂u
= cos( x 2 + y 2 ) ⋅ 2 x
∂x
∂u
= cos( x 2 + y 2 ) ⋅ 2 y
∂y
∂y
Let f ( x, yy)) = a 2 x 2 + b 2 y 2 − c 2
∂f
= − ∂x
dyy
d
dx ∂f
∂y
7.8 Implicit Differentiation 7.109

2a 2 x
=−
2b 2 y
a2 x
=−
b2 y
du ∂u d x ∂u dy
dy
= ⋅ + ⋅
dxx ∂x d x ∂y d x
d
 a2 x 
= 2 x cos( x 2 + y 2 ) + 2 y cos( x 2 + y 2 )  − 2 
 b y
 a2 
= 2 x cos( x 2 + y 2 ) ⋅ 1 − 2 
 b 

Example 8
du
If u = x log( xy ) where x3 + y 3 + 3 xy = 1,
1 find .
dx
dx
Solution
u = x log (xy)
= x(log x + logg y)
∂u 1
= x ⋅ + (log x + log y )
∂x x
= 1 + log
g x + log y
∂u 1 x
=x =
∂y
∂y y y
Let f (x, y ) = x 3 + y 3 + 3 xy − 1
∂f
∂f
= − ∂x
dy
dy
dx ∂f
∂f
∂y
∂y
3x 2 + 3 y
=− 2
3 y + 3x
x2 + y
=−
y2 + x

du ∂u d x ∂u dy
dy
= ⋅ + ⋅
d x ∂x d x ∂y d x
x  x2 + y 
= 1 + log
g x + log y + −
y  y 2 + x 
x  x2 + y 
= 1 + log(
og( xy ) −
y  y 2 + x 
7.110 Chapter 7 Partial Derivatives

Example 9
 x du
If u = tan −1   where x 2 + y 2 = a 2 , find .
 y dx
dx
Solution
 x
u = tan −1  
 y
∂u 1 1 y
= ⋅ =
∂x x2 y x2 + y2
1+ 2
y
∂u 1  x x
= 2 
− 2 = − 2
∂y x  y  x + y2
1+ 2
y

Let f (x, y) = x 2 + y 2 – a2
∂f
= − ∂x
dy
dy
dx
dx ∂f
∂y
2x x
=− =−
2y y
du ∂u dx dx ∂u dy
= . + .
dx ∂x dx
dx dx ∂y dx
y x  x
= − 2  − y 
x +y
2 2
x + y2
y x2
= +
x +y
2 2
y( x + y 2 )
2

y 2 + x2
=
y( x2 + y 2 )
1
=
y

Example 10
φ f − φy fx
If u = φ ( x, y ) and f ( x, y ) = 0, prove that du = x y .
dx fy
Solution
f ( x, y ) = 0
7.8 Implicit Differentiation 7.111

∂f
f
= − ∂x = − x
ddyy
dx
dx ∂f fy
∂y
u = φ ( x, y )
du ∂u dx dx ∂u dy dy
= ⋅ + ⋅
d x ∂x d x ∂y d x
dy
dy
= φx + φ y
dx
 fx 
= φx + φ y  − 
 fy 
φx f y − φ y f x
=
fy

Example 11
∂φ ∂∂ff dy ∂∂ff ∂φ
If f (x, y) = 0, f (x, z) = 0, show that ⋅ ⋅ = ⋅ .
∂x ∂y
∂y dz
dz ∂x ∂z
Solution
f ( x, yy)) = 0 and φ ( x, z ) = 0
∂f ∂φ
∂x
= − ∂x
dy
dy dz
dz
=− and
dx ∂f dx ∂φ
∂y ∂z
∂f ˆ
Ê
Á
∂x ˜
Á- ˜
dy
dy ∂f ˜
Á
dx ÁË
∂y ˜¯
=
dzz Ê ∂f ˆ
d
d x Á ∂x ˜
Á- ˜
Á ∂f ˜
ÁË ∂z ˜¯

Ê ∂f ˆ
Á ∂x ˜
Á ˜
Á ∂f ˜
ddyy ÁË ∂y ˜¯
=
dz
dz Ê ∂f ˆ
Á ∂x ˜
Á ˜
Á ∂f ˜
ÁË ∂z ˜¯
7.112 Chapter 7 Partial Derivatives

∂f ∂∂ff dy ∂∂ff ∂f
Hence, ◊ ◊ = ◊
∂x ∂y
∂y dz
dz ∂ x ∂ z

Example 12
 ∂z   ∂x   ∂y ∂y 
If φ ( x, y, z ) = 0, prove that       = −1.
 ∂∂yy  x  ∂z  y  ∂z  z
Solution
φ ( x, y , z ) = 0
∂φ
 dz
dz   ∂z  ∂y
 dy = =− ...(1)
dy  x  ∂y  x ∂φ
∂z
∂φ
 dxdx   ∂x  ∂z
  =   = − ∂φ ...(2)
dz y
dz ∂z y
∂x
∂φ
 dy
dy   ∂y  ∂x
  =   = − ∂φ ...(3)
dx z
dx ∂x z
∂y
From Eqs (1), (2) and (3),
 ∂φ   ∂φ   ∂φ 
 ∂z   ∂x   ∂y
∂y   ∂y∂y   ∂z   ∂x 
 ∂y      =  − ∂φ   − ∂φ   − ∂φ 
∂y  x ∂z y ∂x z    
 ∂z   ∂x   ∂y ∂y 
= −1

EXERCISE 7.4
dy
1. If x3 + y3 – 3axy = 0, find .
dx  ay − x 2 
 Ans. : 2 
 y − ax 
dy
2. If x 3 + 3x 2 + 6 xy 2 + y 3 = 1, find .
dx
 x 2 + 2 x + 2y 2 ) 
((x
 Ans. : − 
 ( xy + y 2 ) 
(4
dy y(y − x log y)
y
3. If xy = yx, prove that = .
dx x(x − y log x)
x
7.8 Implicit Differentiation 7.113

dy
4. If f (x, y) = x sin (x – y) – (x + y) = 0, find .
dx
 [sin(x − y )](1 + x) − 1
 Ans.: x cos(
cos(x − y ) + 1 

y dy
5. If y x = sin x, find .
dx
 ∂z ∂z 
Hint: f = x log y − log sin x, let x = z, log z = y log
y y
g x find ,
∂x ∂y 
 
 ∂f ∂f 
 and then en and 
 ∂ x ∂ y 
 −((yx log y − cot x) 
− y −1

 Ans.: y 
 x log x log y + x y y −1 
d2 y
6. If x 5 + y 5 = 5a 3 x 2 , find 2 .  6a 3 x 2 (a 3 + x 3 ) 
dx
 Ans. : 
 y9 
7. If xy3 – yx3 = 6 is the equation of curve, find the slope of the tangent at
the point (1, 2).
 dy 
Hint : Find dx at (1, 2)
 2
 Ans. : − 11

d2 y
8. Find , if x 1 − y 2 + y 1−
1 − x 2 = a.  a 
dx 2  Ans. : 3 
 ((1 − x 2 ) 2 
du
9. If u = x log xy and x 3 + y 3 + 3xy − 1 = 0, find .
dx
 ∂f 
 du ∂u d x ∂u dy dy ∂ 
Hint : = ⋅ + ⋅ =− x
 dx ∂x d x ∂y dx d x ∂f 
 ∂y 
 x  x + ay  
2

 Ans. : 1 + log xy −  2 
 y  y + ax  
m−2
d2 y m x
10. If xm + ym = bm, show that = −(m − 1)b .
dx 2 y 2 m −1
du
11. If u = x2y and x 2 + x
xyy + y 2 = 1, find .
dx
d2 y  2a 2 x 2 
12. If x + y = 3ax , find −
3 3 2
.  Ans.: 
dx 2  y5 
7.114 Chapter 7 Partial Derivatives

7.9 EULER’S THEOREM FOR HOMOGENEOUS FUNCTIONS

A function u = f (x, y) is said to be a homogeneous function of degree n, if


 y
u = xn f  
 x
where n is a real number.
Note: Degree of a homogeneous function u = f (x, y) can be obtained by replacing x
by xt and y by yt and if
f (xt, yt) = tn f (x, y) = tnu
then u is a homogeneous function of degree n. Same method can be extended for a
function of more than two variables.

7.9.1 Euler’s Theorem for a Function of Two Variables


Theorem If u is a homogeneous function of two variables x and y of degree n then
∂u ∂u
x +y = nu
∂x ∂y
Let u = f (x, y) be a homogeneous function of degree n.
u = f ( X , Y ) = t n f ( x, y )
where X = xt and Y = yt,
Differentiating u = f (X, Y ) w.r.t. t using composite function,
∂u ∂u ∂∂XX ∂u ∂Y ∂u ∂u
= ⋅ + ⋅ =x +y
∂t ∂∂X
X ∂t ∂Y ∂t ∂∂X
X ∂Y
At t = 1, X = x and Y = y
∂u ∂u ∂u u
=x +y …(1)
∂t ∂x ∂y
∂y
n
Differentiating u = t f (x, y) w.r.t. t,
∂u
= nt n −1 f ( x, y ) X Y
∂t
At t = 1,
∂u
= nf ( x, y ) = nu …(2)
∂t t
From Eqs (1) and (2),
Fig. 7.79
∂u ∂u
x +y = nu
∂x ∂y

7.9.2 Euler’s Theorem for a Function of Three Variables


Theorem If u is a homogeneous function of three variables x, y, z of degree n then
∂u ∂u ∂u
x +y +z = nu
∂x ∂y ∂z
7.9 Euler’s Theorem for Homogeneous Functions 7.115

Let u = f (x, y, z) be a homogeneous function of degree n. u


u = f ( X , Y , Z ) = t f ( x, y , z )
n

where X = xt, Y = yt, Z = zt


Differentiating u = f (X, Y, Z) w.r.t. t using composite function, x y z

∂u ∂u ∂∂XX ∂u ∂Y ∂u ∂Z ∂u ∂u ∂u
= ⋅ + ⋅ + ⋅ =x +y +z
∂t ∂∂X
X ∂t ∂Y ∂t ∂Z ∂t ∂∂X
X ∂Y ∂Z t

At t = 1, X = x, Y = y and Z = z Fig. 7.80


∂u ∂u ∂u ∂u
=x +y +z …(3)
∂t ∂X
∂X ∂Y ∂Z
Differentiating u = t nf (x, y, z) w.r.t. t,
∂u
= nt n −1 f ( x, y, z )
∂t
At t = 1,
∂u
= nf ( x, y, z ) = nu …(4)
∂t
From Eqs (3) and (4),
∂u ∂u ∂u
x +y +z = nu
∂x ∂y ∂z

7.9.3 Deductions from Euler’s Theorem


Corollary 1 If u is a homogeneous function of two variables x, y of degree n then

∂ 2u ∂ 2u 2 ∂ u
2
x2 + 2 xy + y =n
n((n − 1)u.
∂x 2 ∂x ∂y ∂y 2

Let u be a homogeneous function of two variables x and y of degree n.


By Euler’s theorem,
∂u ∂u
x +y = nu …(5)
∂x ∂y
Differentiating Eq. (5) partially w.r.t x,

∂ 2 u ∂u ∂ 2u ∂u
x + + y =n
∂x 2
∂x ∂x ∂y ∂x
∂ 2u ∂ 2u ∂u
x +y = (n − 1) …(6)
∂x 2
∂x ∂y ∂x
Differentiating Eq. (5) partially w.r.t. y,

∂ 2u ∂ 2 u ∂u ∂u
x +y 2+ =n
∂x ∂y ∂y ∂y ∂y
7.116 Chapter 7 Partial Derivatives

∂ 2u ∂ 2u ∂u
x + y 2 = (n − 1) …(7)
∂x ∂y ∂y ∂y

Multiplying Eq. (6) by x and Eq. (7) by y and adding,


∂2u ∂ 2u ∂2u  ∂u ∂u 
x2 + 2 xy + y 2 2 = (n − 1)  x + y  = (n − 1)nu [Using Eq. (5)]
∂x 2
∂x ∂y ∂y  ∂ x ∂y 
∂2u ∂2u ∂2u
x2 + 2 xy + y 2 2 = nn((n − 1)u
∂x 2
∂x ∂y ∂y

Example 1
Verify Euler’s theorem for
(i) u = x yz − 4 y z + 2 xz u= x+ y+ z
2 2 2 3
(ii)
−1 y x2 + y 2
(iii) u = x y sin
4 2
(iv) u=
x x+ y
x+ y+ z
(v) u = aaxx 2 + 2hxy + by 2 (vi) u =
x+ y+ z
Solution
(i) u = x 2 yz − 4 y 2 z 2 + 2 xz 3

Replacing x by xt, y by yt and z by zt,


u = t 4 ( x 2 yyzz − 4 y 2 z 2 + 2 xz
xz 3 )
Hence, u is a homogeneous function of degree 4.
By Euler’s theorem,
∂u ∂u ∂u
x +y +z = nu = 4u …(1)
∂x ∂y ∂z
Differentiating u partially w.r.t. x, y and z,
∂u
= 2 xyz + 2 z 3 ,
∂x
∂u
= x 2 z − 8 yz 2 ,
∂y
∂y
∂u
= x 2 y − 8 y 2 z + 6 xz 2
∂z
∂u ∂u ∂u
x +y +z = 2 x 2 yyzz + 2 xxzz 3 + x 2 yz − 8 y 2 z 2 + x 2 yyzz − 8 y 2 z 2 + 6 xz
x 3
∂x ∂y
∂y ∂z
=4
4xx 2 yyzz − 16 y 2 z 2 + 8 xz
16 xz 3
=44(( x 2 yyzz − 4 y 2 z 2 + 2 xz
xz 3 )
= 4u …(2)
7.9 Euler’s Theorem for Homogeneous Functions 7.117

Hence, from Eqs (1) and (2), theorem is verified.


(ii) u= x+ y+ z
Replacing x by xt, y by yt and z by zt,
u = t11// 2 ( x + y + z )
1
Hence, u is a homogeneous function of degree .
By Euler’s theorem, 2
∂u ∂u ∂u 1
x +y +z = nu = u ... (3)
∂x ∂y ∂z 2
Differentiating u partially w.r.t. x, y and z,
∂u 1 ∂u 1 ∂u 1
= , = , =
∂x 2 x ∂∂yy 2 y ∂z 2 z
... (4)
∂u ∂u ∂u 1
x
∂x
+y
∂y
∂y
+z =
∂z 2
( x+ y+ z = ) 1
2
u

Hence, from Eqs (3) and (4), theorem is verified.


y
(iii) u = x 4 y 2 sin −1
x
Replacing x by xt and y by yt,
 y
u = t 6  x 4 y 2 sin −1 
 x
Hence, u is a homogeneous function of degree 6.
By Euler’s theorem,
∂u ∂u
x +y = nu = 6u …(5)
∂x ∂y
Differentiating u partially w.r.t. x,
 3 −1 y x4  y 
∂u 2  4 x sin +  − 2  
=y x y2 x 
∂x  1− 2
 x 
 3 −1 y yx 3 
= y 2  4 x sin − 
 x x2 − y 2 
Differentiating u partially w.r.t. y,
∂u  y 1 1
= x 4  2 y sin −1 + y 2 ⋅ 
∂y
∂y x y x
2
 1− 2
 x 
y x4 y 2
= 2 x 4 y sin −1 +
x x2 − y 2
7.118 Chapter 7 Partial Derivatives

∂u ∂u y x4 y3 y x4 y3
x +y =44xx 4 y 2 sin −−11 − + 2 x 4 y 2 sin −1 +
∂x ∂y x x −y
2 2 x x2 − y 2
y
= 6 x 4 y 2 sin −1
x ...(6)
= 6u
Hence, from Eqs (5) and (6), theorem is verified.
x2 + y 2
(iv) u=
x+ y
Replacing x by xt and y by yt,
 x2 + y 2 
u = t
 x + y 
Hence, u is a homogeneous function of degree 1.
By Euler’s theorem,
∂u ∂u
x +y = nu = u …(7)
∂x ∂y
Differentiating u partially w.r.t. x,
∂u 2x ( x2 + y 2 )
= −
∂x x + y ( x + y ) 2
2 x 2 + 2 xxyy − x 2 − y 2
=
( x + y)2
x 2 − y 2 + 2 xy
=
( x + y)2
Differentiating u partially w.r.t. y,
∂u 2y x2 + y 2
= −
∂y x + y ( x + y ) 2
∂y
2 yx + 2 y 2 − x 2 − y 2
=
( x + y)2
y 2 − x 2 + 2 xy
=
( x + y)2
∂u ∂u x 3 − xxyy 2 + 2 x 2 y + y 3 − x 2 y + 2 xy 2
x +y =
∂x ∂y ( x + y)2
x 3 + y 3 + xy 2 + x 2 y
=
( x + y)2
( x + y )( x 2 − xxyy + y 2 ) + xy ( y + x)
=
( x + y)2
( x + y )( x 2 − xy + y 2 + xxyy )
=
( x + y)2
7.9 Euler’s Theorem for Homogeneous Functions 7.119

∂u ∂u x 2 + y 2
x +y =
∂x ∂y x+ y
=u …(8)
Hence, from Eqs (7) and (8), theorem is verified.
(v) u=aaxx 2 + 2hxy + by 2
Replacing x by xt and y by yt,
u = t 2 (ax 2 + 2hx
hxyy + b
byy 2 )
Hence, u is a homogeneous function of degree 2.
By Euler’s theorem,
∂u ∂u
x +y = 2u ... (9)
∂x ∂y
Differentiating u partially w.r.t. x and y,
∂u
= 2ax + 2hy
∂x
∂u
= 2hx + 2by
∂y
∂y
∂u ∂u
x +y = 2ax 2 + 2hxhxyy + 2hxy + 2by 2
∂x ∂y
∂y
= 22((ax 2 + by 2 + 2hxy
hxy )
= 2u. ... (10)
Hence, from Eqs (9) and (10), theorem is verified.
x+ y+ z
(vi) u= .
x+ y+ z
Replacing x by xt and y by yt,
1 
x+ y+ z 
u = t2  
 x + y + z
1
Hence, u is a homogeneous function of degree .
By Euler’s theorem, 2
∂u ∂u ∂u 1
x +y +z = nu = u …(11)
∂x ∂y ∂z 2
Differentiating u partially w.r.t. x, y, and z,
∂u 1 x+ y+ z 1
= − ⋅
∂x
( )
2
x+ y+ z x+ y+ z 2 x
∂u 1 x+ y+ z 1
= − ⋅
∂y
∂y
( )
2
x+ y+ z x+ y+ z 2 y

∂u 1 x+ y+ z 1
= − ⋅
∂z
( )
2
x+ y+ z x+ y+ z 2 z
7.120 Chapter 7 Partial Derivatives

x
∂u
+y
∂u
+z
∂u
=
x+ y+ z

( x + y + z) ( x+ y+ z )
∂x ∂y ∂z
( )
2
x+ y+ z 2 x+ y+ z
1 x+ y+ z
=
2 x+ y+ z
1 …(12)
= u
2
Hence, from Eqs (11) and (12), theorem is verified.

Example 2
x
 x y
 x ∂u ∂u
If u = e sinn   + e cos   , prove that x + y
y x
= 0.
 y  y ∂x ∂y
Solution x
 x y
 x
u = e y sin
n   + e x cos   ,
 y  y
Replacing x by xt and y by yt,
 x  x y
 x
u = t 0 e y sin
n   + e x cos   
  y  y  
Hence, u is a homogeneous function of degree 0.
By Euler’s theorem,
∂u ∂u
x +y =0
∂x ∂y

Example 3
∂u ∂u
Find x +y where u = (8x2 + y2) (log x – log y).
∂x ∂y
Solution
u = (8 x 2 + y 2 ) (log x − logg y )
 x
= (8 x 2 + y 2 ) log  
 y
Replacing x by xt and y by yt,
 x
u = t 2 (8 x 2 + y 2 ) log  
 y
Hence, u is a homogeneous function of degree 2.
By Euler’s theorem,
∂u ∂u
x +y = 2u
∂x ∂y
=2
2((8 x 2 + y 2 )(log x − log y )
7.9 Euler’s Theorem for Homogeneous Functions 7.121

Example 4
x2  y  y 2  x 
If u = f + g , prove that
y  x  x  y 
 ∂u  y   ∂u  x
x2  y − xf    + y 2  x − yg    = 0.
 ∂x  x  ∂y  y 
Solution 2
x  y  y2  x 
u= f + g
y  x  x  y 
Replacing x by xt and y by yt,
 x2  y  y 2  x  
u =t f   + g 
 y  x x  y 
Hence, u is a homogeneous function of degree 1.
By Euler’s theorem,
∂u ∂u
x +y = 1⋅ u
∂x ∂y
x2  y  y 2  x 
= f + g
y  x  x  y 
∂u ∂u  y  x
x2 y + xy 2 = x3 f   + y 3 g  
∂x ∂y  x   y
 ∂u  y   ∂u  x
x2  y − xf    + y 2  x − yg    = 0
 ∂x  x  ∂y  y 

Example 5
1 g x − log y
1 log ∂u ∂u
If u ( x, y ) = + + , prove that x + y + 2u ( x, y ) = 0.
x 2
x
xy x 2
∂x ∂y
[Summer 2014]

Solution
1 g x − log y
1 log
u ( x, y ) = + +
x 2 xy x2
1 1 1  x
= + + 2 log  
x 2
xy x  y
Replacing x by xt and y by yt,
1 1 1  x
u ( x, y ) = t −2  2 + + 2 log   
x xy x  y 
7.122 Chapter 7 Partial Derivatives

Hence, u is a homogeneous function of degree –2.


By Euler’s theorem,
∂u ∂u
x +y = −2
2u ( x, y )
∂x ∂y
∂u ∂u
Hence, x +y + 22uuu(( x, y ) = 0
∂x ∂y

Example 6
x2 + y 2 ∂z ∂z
If z = log( x 2 + y 2 ) + − 2 log( x + y ), find the value of x + y .
x+ y ∂x ∂y
Solution
x2 + y 2
z = log( x 2 + y 2 ) + − 2 log( x + y )
x+ y
x2 + y 2
= log( x 2 + y 2 ) + − log( x + y ) 2
x+ y
( x2 + y 2 ) x2 + y 2
= log +
( x + y)2 x+ y
= u+v
x2 + y 2 x2 + y 2
where u = log , v=
( x + y) 2
x+ y
Replacing x by xt and y by yt in u and v,
x2 + y 2  x2 + y 2 
u = t 0 log , v = t
( x + y) 2
 x + y 
Hence, u is a homogeneous function of degree 0 and v is a homogeneous function of
degree 1.
By Euler’s theorem,
∂u ∂u
x +y = 0⋅u = 0 …(1)
∂x ∂y
∂v ∂v
and x +y = 1⋅ v …(2)
∂x ∂y
Adding Eqs (1) and (2),

 ∂u ∂v   ∂u ∂v 
x +  + y +  = 0+v
 ∂x ∂x   ∂y ∂y 
∂z ∂z x 2 + y 2
x +y =
∂x ∂y x+ y
7.9 Euler’s Theorem for Homogeneous Functions 7.123

Example 7
 y ∂u ∂u
If u = f   + x 2 + y 2 , prove that x + y = x2 + y 2 .
 x ∂x ∂y
Solution
Let u=v+w
 y
where v = f   ,w = x2 + y 2
 x
Replacing x by xt and y by yt in v and w,
 y
v = t 0 f   , w = t x2 + y 2
 x
Hence, v is a homogeneous function of degree 0 and w is a homogeneous function of
degree 1.
By Euler’s theorem,
∂v ∂v
x +y = 0⋅v = 0 …(1)
∂x ∂y
∂w ∂w
and x +y = 1⋅ w = w …(2)
∂x ∂y
Adding Eqs (1) and (2),

 ∂v ∂w   ∂v ∂w 
x +  + y +  = w
 ∂x ∂x   ∂y ∂y 
∂u ∂u
Hence, x +y = x2 + y 2
∂x ∂y

Example 8
 xy  x2 y ∂T ∂T
If T = sinn  2 2
+ x 2
+ y 2
+ , find the value of x +y .
x +y  x+ y ∂x ∂y
Solution
Let T = u+v+w
 xy 
where u = sin  2
 x + y 2 
Replacing x by xt and y by yt,
  xy  
u = t 0 sin  2 2
  x + y 
Hence, u is a homogeneous function of degree 0.
v= x2 + y 2
7.124 Chapter 7 Partial Derivatives

Replacing x by xt and y by yt,


v = t x2 + y 2
Hence, v is a homogeneous function of degree 1.
x2 y
w=
x+ y
Replacing x by xt and y by yt,
 x2 y 
w = t2 
 x + y 
Hence, w is a homogeneous function of degree 2.
By Euler’s theorem,
∂u ∂u
x +y =0 ...(1)
1
∂x ∂y
∂v ∂v
x +y = 1⋅ v = v ...(2)
2
∂x ∂y
∂w ∂w
x +y = 2 ⋅ w = 2w ...(3)
3
∂x ∂y
Adding Eqs (1), (2) and (3),

 ∂u ∂v ∂w   ∂u ∂v ∂w 
x + +  + y  + +  = v + 2w
 ∂x ∂x ∂x   ∂y ∂y ∂y 
∂ ∂ x2 y
x (u + v + w) + y (u + v + w) = x2 + y 2 + 2
∂x ∂∂yy x+ y
∂T ∂T 2x2 y
x +y = x2 + y 2 +
∂x ∂y
∂y x+ y

Example 9
x3 y 3 z 3 ∂u ∂u ∂u
If u = 3 then prove that x +y +z = 6u.
x +y +z
3 3
∂x ∂y ∂z
Solution
x3 y 3 z 3
u=
x + y3 + z3
3

Replacing x by xt, y by yt and z by zt,


 x3 y 3 z 3 
u = t6  3
 x + y 3 + z 3 
Hence, u is a homogeneous function of degree 6.
By Euler’s theorem,
∂u ∂u ∂u
x +y +z = 6u
∂x ∂y ∂z
7.9 Euler’s Theorem for Homogeneous Functions 7.125

Example 10
x2 y 2 z 2  xy + yz 
If u = + cos  2 then show that
x +y +z
2 2 2
 x + y 2 + z 2 
∂u ∂u ∂u 4x2 y 2 z 2
x +y +z = 2 .
∂x ∂y ∂z x + y 2 + z 2
Solution
Let u=v+w
x2 y 2 z 2  xy + yz 
where v= 2 , w = cos  2
x + y2 + z2  x + y 2 + z 2 
Replacing x by xt, y by yt and z by zt,
 x2 y 2 z 2   xy + yz 
v = t4  2 , w = t 0 cos  2
 x + y 2
+ z 2
  x + y 2 + z 2 
Hence, v is a homogeneous function of degree 4 and w is a homogeneous function of
degree 0.
By Euler’s theorem,
∂v ∂v ∂v
x +y +z = 4v …(1)
∂x ∂y ∂z
∂w ∂w ∂w
and x +y +z = 0⋅ w = 0 …(2)
∂x ∂y ∂z
Adding Eqs (1) and (2),

 ∂v ∂w   ∂v ∂w   ∂v ∂w 
x  +  + y  +  + z  +  = 4v
 ∂x ∂x   ∂y ∂y   ∂z ∂z 
∂(v + w)
∂( ∂(v + w) ∂(v + w)
x +y +z = 4v
∂x ∂y ∂z

∂u ∂u ∂u 4x2 y 2 z 2
Hence, x +y +z = 2
∂x ∂y ∂z x + y 2 + z 2

Example 11
1 ∂v ∂v
If v = f (θ ) where x = r cos q, y = r sin q, show that x + y + v = 0.
r ∂x ∂y

Solution
x = r cos q, y = r sin q
7.126 Chapter 7 Partial Derivatives

 y
∴ r= x 2 + y 2 and θ = tan −1  
 x
1
Now, v= f (θ )
r
1   y 
= f  tan −1   
x +y
2
 2  x

Replacing x by xt and y by yt,



1  y 
v= f  tan −1   
t x +y 
2 2 x

 1   y  
= t −−11  f  tan −1    
 x 2 + y 2   x  

Hence, v is a homogeneous function of degree –1.
By Euler’s theorem,
∂v ∂v
x +y = −1⋅ v
∂x ∂y
∂v ∂v
x + y +v = 0
∂x ∂y

Example 12
∂z ∂z
If x = eu tan v, y = eu sec v and z = e–2u f (v), prove that x + y + 2 z = 0.
∂x ∂y
Solution
x = eu tan v, y = eu sec v
∴ y 2 − x 2 = e 2u (sec 2 v − tan
n 2 v)
= e 2u

1
e −2u =
y 2 − x2
x tan v
Also, =
y sec v
= sin v

 x
v = sin −1  
 y
−2 u
Now, z = e f (v )
1  −1 x
= 2 f  sin
y − x2 y 
7.9 Euler’s Theorem for Homogeneous Functions 7.127

Replacing x by xt and y by yt,

1  x
Now, z= f  sin −1 
t (y − x ) 
2 2 2
y
 1  x
= t −2  2 f  sin −1  
(y − x )
2
 y 

Hence, z is a homogeneous function of degree –2.


By Euler’s theorem,
∂z ∂z
+y
x = −2 z
∂x ∂y
∂z ∂z
x + y + 2z = 0
∂x ∂y

Example 13
If u = f (v) where v is a homogeneous function of x, y of degree n, prove that
∂u ∂u ∂u ∂u
x +y = nv
nvff ′ (v). Hence, deduce that if u = log v, x + y = n.
∂x ∂y ∂x ∂y
Solution
v is a homogeneous function of degree n.
By Euler’s theorem,
∂v ∂v
x +y = nv ...(1)
∂x ∂y
Now, u = f (v)
∂u ∂v
= f ′ (v ) ,
∂x ∂x
∂u ∂v
= f ′ (v )
∂y
∂y ∂y
∂y
∂u ∂u ∂v ∂v
x +y = xf ′ (v) + yf ′ (v)
∂x ∂y ∂x ∂y
 ∂v ∂v 
= f ′ (v )  x + y  .
 ∂x ∂y 
= f (v) ⋅ nv [From Eq. (1)] …(2)

u = log v
If f (v) = log v
1
f ′ (v ) =
v
7.128 Chapter 7 Partial Derivatives

Substituting in Eq. (2),


∂u ∂u 1
x +y = ⋅ nv
∂x ∂y v
=n

Example 14
n  y  y
If z = f (x, y), u = x φ   , v = x nψ   , show that
 x  x
∂z ∂z  ∂z ∂z 
x +y = nu +v .
∂x ∂y  ∂u ∂v 
Solution
u and v are homogeneous functions of x, y of degree n.
By Euler’s theorem, z

∂u ∂u
x +y = nu
∂x ∂y
u v
∂v ∂v
x +y = nv
∂x ∂y
We know that
∂z ∂z ∂u ∂z ∂v
x
= ⋅ + ⋅ Fig. 7.81
∂x ∂u ∂x ∂v ∂x
∂z ∂z ∂u ∂z ∂v z
and = ⋅ + ⋅
∂y ∂u ∂y
∂y ∂y ∂v ∂y
∂y
∂z ∂z ∂z  ∂u ∂u  ∂z  ∂v ∂v 
Hence, x +y =  x + y  + x + y 
∂x ∂y ∂u  ∂x ∂y  ∂v  ∂x ∂y  u v

∂z ∂z
= (nu ) + (nv)
∂u ∂v
 ∂z ∂z  y
= nu +v 
 ∂u ∂v  Fig. 7.82

Example 15
∂u ∂u
If u is a homogeneous function of x, y of degree n and X = ,Y =
∂x ∂y
∂f ∂f  n 
and u = f (X , Y ), prove that X +Y =  u.
∂X ∂Y  n − 1
Solution
∂u ∂u
If u is a homogeneous function of x, y of degree n then X = and Y = are also
∂x ∂y
homogeneous functions of degree (n – 1).
7.9 Euler’s Theorem for Homogeneous Functions 7.129

u
∂u ∂u
x +y = nu
∂x ∂y
∂X ∂X
x +y = (n − 1) X X Y
∂x ∂y
∂Y ∂Y
x +y = (n − 1)Y
∂x ∂y
x
Now, u = f (X , Y)
Fig. 7.83
∂u ∂u ∂X ∂X ∂u ∂Y
= ⋅ + ⋅
∂x ∂X ∂X ∂x ∂Y ∂x u

∂u ∂u ∂∂X X ∂u ∂Y
and = ⋅ + ⋅
∂y ∂∂X
∂y X ∂y∂y ∂Y ∂y
∂y
X Y
∂u ∂u ∂u  ∂X ∂X  ∂u  ∂Y ∂Y 
x +y = x +y + x +y 
∂x ∂y ∂X  ∂x ∂y  ∂Y  ∂x ∂y 
∂u ∂u
∂u y
nu = [(n − 1) X ] + [(n − 1)Y ]
∂X
∂X ∂Y Fig. 7.84
∂u ∂u  n 
X +Y = u
∂X
∂X ∂Y  n − 1

Example 16
y
 x x 2 ∂ u
2
∂ 2u 2 ∂ u
2

If u =   , prove that x + 2xy


2 xy + y = 0.
 y ∂x 2 ∂x ∂y ∂y 2
Solution
y
 x x
u= 
 y
Replacing x by xt and y by yt,
y
 x x
u=t   0

 y
Hence, u is a homogeneous function of degree 0.
By cor. 1,
∂2u ∂2u ∂2u
x 2 2 + 22xy
xy + y 2 2 = 0(0 − 1)u
∂x ∂x ∂y ∂y
=0
7.130 Chapter 7 Partial Derivatives

Example 17
y
 x
If u = y 2 e x + x 2 tann −1   , show that
 y
xu x + yu y = 2u and x 2 u xx + 2 xy u xy + y 2 u yy = 2u.
2u
Solution
y
 x
u = y 2 e x + x 2 tan −1  
 y
Replacing x by xt and y by yt,
 y
 x
u = t 2  y 2 e x + x 2 tan −1   
  y 
u is a homogeneous function of degree 2.
By Euler’s theorem,
∂u ∂u
x +y = nu
∂x ∂y
= 2u
∂u
2
∂u
2
∂u
2
x 2 2 + 2 xy + y 2 2 = n(n − 1)u
∂x ∂x ∂y ∂y
= 2(1)u
= 2u

Example 18
 x2 + y 2  2 ∂ u
2
∂ 2u 2 ∂ u
2
If u = logg   , fi nd the value of x + 2 xy + y .
 x+ y  ∂x 2 ∂x ∂y ∂y 2
Solution
 x2 + y 2 
u = log  
 x+ y 
Replacing x by xt and y by yt,
 x2 + y 2 
u = t 0 log  
 x+ y 
Hence, u is a homogeneous function of degree 0.
By cor. 1,
∂2u ∂2u ∂2u
x2 2 + 2
2xy
xy + y 2 2 = 0(0 − 1)u
∂x ∂ x∂ y ∂y
=0
7.9 Euler’s Theorem for Homogeneous Functions 7.131

Example 19
 y  y ∂ 2u ∂ 2u ∂ 2u
If u = xf
x   + g   , show that x 2 2 + 2 xy + y 2 2 = 0.
0
 x  x ∂x ∂x ∂y ∂y
Solution
Let u = v + w,
 y  y
where v = xf
x  , w= g 
 x  x
Replacing x by xt and y by yt in v and w,
 y  y
v = tx f   , w = t 0 g  
 x  x
Hence, v is a homogeneous function of degree 1 and w is a homogeneous function of
degree 0.
By cor. 1,
∂2v ∂2v ∂2v
x 2 2 + 22xy
xy + y 2 2 = 1(1 − 1)v = 0 …(1)
∂x ∂x ∂y ∂y
∂2 w ∂2 w ∂2 w
x2 + 2 xy + y 2 2 = 0(0 − 1) w = 0 …(2)
∂x 2
∂x ∂y ∂y
Adding Eqs (1) and (2),
 ∂2v ∂2 w  ∂2v ∂2 w  2  ∂ v
2
∂2 w
x 2  2 + 2  + 2 xy  + + y  ∂y 2 ∂y 2  = 0
+
 ∂x ∂x   ∂x ∂y ∂x ∂y 
∂2u ∂2u 2 ∂ u
2
Hence, x2 + 2 xy + y =0
∂x 2 ∂x ∂y ∂y 2

Example 20
( x 2 + y 2 )n  y  x
If z = + xf   + φ   , show that
2n
2 n(2n − 1)
n(  x  y
∂ z
2
∂ z
2
∂ z
2
x 2 2 + 2 xy + y 2 2 = ( x 2 + y 2 )n .
∂x ∂x ∂y ∂y
Solution
Let z=u+v+w
(x + y )
2 2 n
 y  x
where u= xf   , w = φ  
, v = xf
2n
2 n(2n − 1)
n(  x  y
Replacing x by xt and y by yt in u, v and w,
t 2n ( x 2 + y 2 )n  y  x
u= txf   , w = t 0φ  
, v = txf
2n
2 n((2n − 1)
n  x  y
Hence, u, v and w are homogeneous functions of degree 2n, 1 and 0 respectively.
7.132 Chapter 7 Partial Derivatives

By cor. 1,
∂2u ∂2u ∂2u
x2 +2
2xy
xy + y 2 2 = 2nn((2n − 1)u …(1)
∂x 2
∂x ∂y ∂y
∂2v ∂2v 2 ∂ v
2
x2 + 2xy
2 xy + y = 1(1 − 1)v = 0 …(2)
∂x 2 ∂x ∂y ∂y 2
∂2 w ∂2 w 2 ∂ w
2
and x2 + 2 xy + y = 0(0 − 1) w = 0 …(3)
∂x 2 ∂x ∂y ∂y 2
Adding Eqs (1), (2) and (3),
 ∂2u ∂2 v ∂2 w   ∂2u ∂2v ∂2 w 
x 2  2 + 2 + 2  + 2 xy  + +
 ∂x ∂x ∂x   ∂x ∂y ∂x ∂y ∂x ∂y 
 ∂2u ∂2 v ∂2 w 
+ y 2  2 + 2 + 2  = 2nn((2n − 1)u
 ∂∂yy ∂y
∂y ∂∂yy 
∂2 z ∂2 z 2 ∂ z
2
Hence, x2 + 2 xy + y = ( x 2 + y 2 )n
∂x 2 ∂x ∂y ∂y 2

Example 21
 y −n  x
If z = x f   + y f   , prove that
n

x  y
∂2 z ∂2 z 2 ∂ z
2
∂z ∂z
x2 + 2 xy + y +x +y = n 2 z.
∂x 2
∂x ∂y ∂y 2
∂x ∂y
Solution
Let z=u+v
 y  x
where u = xn f   , v = y −n f  
 x  y
Replacing x by xt and y by yt in u and v,
 y  x
u = t n xn f   , v = t −n y −n f  
 x  y
Hence, u is a homogeneous function of degree n and v is a homogeneous function of
degree –n.
By Euler’s theorem and cor. 1,

∂2u ∂ 2u ∂ 2u ∂u ∂u
x2 + 2 xy + y2 2 + x + y = n(n − 1)u + nu
∂x 2
∂x ∂y ∂y ∂x ∂y
= n2u …(1)
∂2v ∂2v 2 ∂ v
2
∂v ∂v
and x2 + 2 xy + y +x +y = − n( − n − 1)v − nv
∂x 2
∂x ∂y ∂y 2
∂x ∂y
7.9 Euler’s Theorem for Homogeneous Functions 7.133

= n2v …(2)
Adding Eqs (1) and (2),
 ∂2u ∂2 v   ∂2u ∂2v   ∂2u ∂2 v   ∂u ∂v 
x 2  2 + 2  + 2 xy  +  + y2  2 + 2  + x  + 
 ∂x ∂x   ∂x ∂y ∂x ∂y   ∂y ∂y   ∂x ∂x 
 ∂u ∂v 
+ y  +  = n 2 (u + v)
 ∂y ∂y 

∂2 z ∂2 z ∂2 z ∂z ∂z
Hence, x2 + 2 xy + y2 2 + x + y = n2 z
∂x 2
∂x ∂y ∂y ∂x ∂y

Example 22
x3 + y 3 1  x2 + y 2 
If u = + 7 sinn −1  2 , find value of
y x x x 
 x + 2 xy
∂ 2u ∂ 2u ∂ 2u ∂u ∂u
x2 2 + 2 2xy
xy + y2 2 + x + y at x = 1, y = 2.
∂x ∂x ∂y ∂y ∂x ∂y
Solution
Let u=v+w
 x3 + y 3  1 −1  x + y 
2 2

where v=  , w =  x 7 sin  x 2 + 2 xy  
 y x   x 
Replacing x by xt and y by yt in v and w,
3
 x3 + y 3  −77

1 −1  x + y 
2 2

v = t2   , w = t  s in   
 y x  x
7
 x + 2 xy
2
x 
3
Hence, v is a homogeneous function of degree and w is a homogeneous function
of degree –7. 2
By Euler’s theorem and cor. 1,
∂2v ∂2v 2 ∂ v
2
∂v ∂v 3  3  3 9
x2 + 2 xy + y +x +y =  − 1 v + v = v …(1)
∂x 2
∂x ∂y ∂y 2
∂x ∂y 2  2  2 4
∂2 w ∂2 w 2 ∂ w
2
∂w ∂w
x2 + 2xy
2 xy + y +x +y = −7( −
−77 − 1) w − 7 w = 49 w …(2)
∂x 2
∂x ∂y ∂y 2
∂x ∂y
Adding Eqs (1) and (2),
 ∂2v ∂2 w  ∂2v ∂2 w  2  ∂ v
2
∂2 w  ∂v ∂w 
x 2  2 + 2  + 2 xy  +  + y  + 2 
+ x + 
 ∂x ∂x   ∂x ∂y ∂x ∂y   ∂y 2
∂y   ∂x ∂x 
 ∂v ∂w  9v
+ y +  = + 49 w
 ∂y ∂y  4
7.134 Chapter 7 Partial Derivatives

∂2u ∂2u 2 ∂ u
2
∂u ∂u 9v
x2 + 2 xy + y +x +y = + 49 w
∂x 2
∂x ∂y ∂y 2
∂x ∂y 4
At x = 1, y = 2,
1+ 8 9
v= = .
2 1 2
1  1+ 4
w = 7 sin −1 
(1)  1 + 4 
π
and n −1
sin −1
(1) =
2

∂2u ∂2u 2 ∂ u
2
∂u ∂u 81 49π
Hence, x2 + 2 xy + y +x +y = +
∂x 2
∂x ∂y ∂y 2
∂x ∂y 8 2

EXERCISE 7.5
1. Verify Euler’s theorem for
 x2 + y2   xy + yz + zx   x2 + y2 
(i) u = tan−1   (ii) u = log  2 (iii) u = log  2
 y   x + y 2 + z 2   x − y 2 
x 2 + y 2 + z2 x y z
(iv) u = 3x 2 yz + 5xy 2 z + 4 z 4 (v) u = (vi) u = + + .
x+y+z y z x

xy + yz + zx ∂u ∂u ∂u
2. If u = coss , find x +y +z .
x +y +z
2 2 2
∂x ∂y ∂z
[Ans. : 0]
 
 xy + yz  x + y + z ∂u ∂u ∂u
3. If u = coss  2 + sin  , evaluate x +y +z .
 x + y 2 + z 2   1  ∂ x ∂ y ∂z
 (xy ) 4 
[Ans. : 0]
 x − y ∂u y ∂u
n−1 
4. If u = sin  , show that =− .
 x + y  ∂ x x ∂y
x
− ∂2 z ∂2 z 2 ∂ z
2
5. If z = x 3 e y , find the value of x 2 + 2 xy + y .
∂x 2 ∂x ∂y ∂y 2
[Ans. : 6]
∂ u ∂ u ∂ u
6. If u = x 2 yz − 4 y 2 z 2 + 2 xz 3 , prove that x +y +z = −4
4u.
∂x ∂y ∂z
x2 y2 z2
7. If + + = 1, where u is a homogeneous function in x, y,
a 2 + u b2 + u c 2 + u
2 2 2
 ∂u   ∂u   ∂u 
z of degree n, prove that   +   +   = 2nu.
 ∂x   ∂y   ∂z 
7.9 Euler’s Theorem for Homogeneous Functions 7.135

x 3y 3 ∂u ∂u
8. If u = , prove that x +y = 3u.
x +y
3 3
∂x ∂y

x2 + y2 ∂u ∂u 3
9. If u = , prove that x +y = u.
x+y ∂x ∂y 2

xy ∂2 z ∂2 z ∂2 z
10. If u = , find the value of x 2 2 + 2 xy + y2 2 .
x+y ∂x ∂x ∂y ∂y
[Ans. : 0]
3
x y z3 3
 xy + yz + zx 
11. If u = + log  2 , prove that
x 3 + y 3 + z3  x + y 2 + z 2 
∂u ∂u ∂u
x +y +z = 6u.
∂x ∂y ∂z

y y ∂2u ∂2u ∂2u


12. If u = xφ   + ψ   , prove that x 2 2 + 2 xy + y 2 2 = 0.
0
 x  x ∂x ∂x ∂y ∂y

−1  y  −1  y 
13. If u = 3x cot   + 16 y cos   , prove that
4 4

 x  x
∂2u ∂2u 2 ∂ u
2
x2 + 2 xy + y = 12u.
∂x 2 ∂x ∂y ∂y 2

y
n−1   , prove that
14. If u = x 3 y 2 sin
 x
∂2u ∂2u ∂2u ∂u ∂u
x2 + 2 xy + y2 2 + x +y = 25u.
∂x 2
∂x ∂y ∂y ∂x ∂y

3 y − 3 x ∂u ∂u 3 y − 3 x
15. If u = x 2 log
g  , prove that x + y = 2 x 2
log
g 3 .
3 y + x
3
∂x ∂y  y + x
3

 x 2 − y 2 y 2 − z2 z2 − x 2 
16. If u = f  , , , prove that
 z2 x2 y 2 
∂2u ∂2u 2 ∂ u
2
x2 + 2 xy + y = 0.
0
∂x 2 ∂x ∂y ∂y 2
n
 x y z ∂u ∂u ∂u
17. If u =  + +  , show that x +y +z = 0.
 y z x ∂x ∂y ∂z

y x ∂2u ∂2u ∂2u


18. If u = x 2 sin−1 − y 2 cos −1 , prove that x 2 2 + 2 xy + y 2 2 = 2u.
x y ∂x ∂x ∂y ∂y
7.136 Chapter 7 Partial Derivatives

y y ∂2u ∂2u ∂2u


19. If u = x sin−1 + tan−1 find the value of x 2 2 + 2 xy + y2 2 .
x x ∂x ∂x ∂y ∂y
[Ans. : 0]
∂2u ∂2u ∂2u
20. If y = x cos u, prove that x 2 2 + 2 xy + y 2 2 = 0.
0
∂x ∂x ∂y ∂y
 y −1 y 
Hint: cosssu = x , u = cos x 
 
 y y − 
y
  x x x
21. If u = x 1 tan−1   + e x  + y −3 sin−1   + log  , prove that
  x  x    y  y y 
∂u
2
∂u2
∂u
2
∂u ∂u
x 2 2 + 2 xy + y2 2 + x +y = 9u.
∂x ∂x ∂y ∂y ∂x ∂y
22. If z = f(x, y) and u, v are homogeneous functions of degree n in x, y,
∂z ∂z  ∂z ∂z 
then show that x +y = nu +v .
∂x ∂y  ∂u ∂v 
2
∂2u ∂2u 2 ∂ u
2
4
23. If u = (x 2 + y 2 ) 3 , prove that x 2 + 2 xy + y = u.
∂x 2 ∂x ∂y ∂y 2 9
Corollary 2: If z = f (u) is a homogeneous function of degree n in variables x and y,
∂u ∂u f (u )
then x + y =n .
∂x ∂y f ′ (u )
By Euler’s theorem,
∂z ∂z
x + y = nz = nf (u )
∂x ∂y
∂ ∂
x f (u ) + y f (u ) = nf (u )
∂x ∂y
∂u ∂u
x f ′ (u ) + y f ′ (uu ) = nf (u )
∂x ∂y
∂u ∂u f (u )
x +y =n
∂x ∂y f ′ (u )

Note: If v = f (u) is a homogeneous function of degree n in variables x, y and z then


∂u ∂u ∂u f (u )
x +y +z =n .
∂x ∂y ∂z f ′ (u )
Corollary 3: If z = f (u) is a homogeneous function of degree n in variables x and y
then
∂ 2u ∂ 2u ∂ 2u
x 2 2 + 2 xy + y 2 2 = g (u )[ g′
g ′ (u ) − 1]
∂x ∂x ∂y ∂y
f (u )
where g (u ) = n .
f ′ (u )
7.9 Euler’s Theorem for Homogeneous Functions 7.137

By cor. 2,
∂u ∂u f (u )
x +y =n = g (u ) ... (1)
∂x ∂y f ′ (u )

Differentiating Eq. (1) partially w.r.t. x,

∂ 2 u ∂u ∂ 2u ∂u
x + +y = g ′ (u )
∂x 2
∂x ∂x ∂y ∂x
∂ 2u ∂ 2u ∂u
x + y = [ g ′ (u ) −1
− 1] ... (2)
∂x 2
∂x ∂y ∂x
Differentiating Eq. (1) partially w.r.t. y,
∂2u ∂ 2 u ∂u ∂u
x +y 2+ = g ′ (u )
∂y ∂x ∂y ∂y ∂y
∂ 2u ∂ 2u ∂u
x + y 2 = [ g′
g ′ (u ) −1
− 1] ... (3)
∂x ∂y ∂y ∂y
Multiplying Eq. (2) by x and Eq. (3) by y and adding,
∂ 2u ∂ 2u 2 ∂ u
2
 ∂u ∂u 
x2 2
+ 2 xy + y = [ g ′ (u ) − 1]  x + y 
∂x ∂x ∂y ∂y 2
 ∂x ∂y 

∂ 2u ∂ 2u ∂ 2u
x2 + 2 xy + y 2 2 = [ g ′ (u ) − 1]g (u ) [Using Eq. (1)]
∂x 2
∂x ∂y ∂y
f (u )
where g (u ) = n .
f ′ (u )

Example 1
 x3 + y 3  ∂u ∂u
If u = secc −1   , prove that x + y = 2cot u.
 x+ y  ∂x ∂y
Solution
 x3 + y 3 
u = sec −1 
 x + y 

Replacing x by xt and y by yt,


  x3 + y 3  
u = sec −1 t 2  
  x+ y 
x3 + y 3
u is a nonhomogeneous function. But secu = is a homogeneous function of
x+ y
degree 2.
7.138 Chapter 7 Partial Derivatives

Let f (u) = sec u


By cor. 2,
∂u ∂u f (u ) sec u
x +y =n =2 = 2cot u
∂x ∂y f ′ (u ) secc u tan u

Example 2
∂u ∂u ∂u
If u = sin–1 (xyz), prove that x +y +z = 3tan u.
∂x ∂y ∂z
Solution
u = sin–1 (xyz)
Replacing x by xt, y by yt and z by zt,
u = sin −1 t 3 ( xyz ) 

u is a nonhomogeneous function. But sin u = xyz is a homogeneous function of


degree 3.
Let f (u) = sin u
By cor. 2,
∂u ∂u ∂u f (u) sin u
x +y +z =n =3 = 3tan u
∂x ∂y ∂z f ′( u ) cos u

Example 3
∂u ∂u
If u = log x + log y, prove that x +y = 2.
∂x ∂y
Solution
u = log x + log y = log xy
Replacing x by xt and y by yt,
u = log t 2 ( xy ) 
u is a nonhomogeneous function. But eu = xy is a homogeneous function of
degree 2.
Let f (u) = eu
By cor. 2,
∂u ∂u f (u ) eu
x +y =n =2 u =2
∂x ∂y f ′ (u ) e

Example 4 ∂u ∂u ∂u
If u = log (x2 + y2 + z2), prove that x +y +z = 2.
∂x ∂y ∂z
7.9 Euler’s Theorem for Homogeneous Functions 7.139

Solution
u = log (x2 + y2 + z2)
Replacing x by xt, y by yt, and z by zt,
u = log t 2 ( x 2 + y 2 + z 2 ) 
u is a nonhomogeneous function. But eu = x2 + y2 + z2 is a homogeneous function of
degree 2.
Let f (u) = eu
By cor. 2,
∂u ∂u ∂u f (u ) eu
x +y +z =n =2 u =2
∂x ∂y ∂z f ′ (u ) e

Example 5
Ê x 7 + y7 + z7 ˆ ∂u ∂u ∂u
If u = log
g Á x + y + z ˜ , show that x ∂x + y ∂y + z ∂z = 6 .
Ë ¯
[Summer 2014]
Solution
Ê x 7 + y7 + z7 ˆ
u = log Á ˜
Ë x+ y+z ¯
Replacing x by xt, y by yt and z by zt,
È t 7 ( x 7 + y7 + z7 ) ˘
u = log Í ˙
ÍÎ t ( x + y + z ) ˙˚
È Ê x 7 + y7 + z7 ˆ ˘
= log Ít 6 Á ˜˙
ÍÎ Ë x + y + z ¯ ˙˚

x 7 + y7 + z7
u is a nonhomogeneous function. But eu = is a homogeneous function
x+ y+z
of degree 6.
Let f(u) = eu
By cor. 2,
∂u ∂u ∂u f (u) eu
x +y +z =n =6 u =6
∂x ∂y ∂z f ¢(u) e

Example 6
 y
x2 f  
 x ∂u ∂u
If u = e , prove that x +y = 2u log u .
∂x ∂y
7.140 Chapter 7 Partial Derivatives

Solution
 y
x2 f  
 x
u=e
Replacing x by xt and y by yt,
 y
t 2 x2 f  
 x
u=e

 y
u is a nonhomogeneous function. But log u = x 2 f   is a homogeneous function of
 x
degree 2.
Let f (u) = log u
By cor. 2,
∂u ∂u f (u ) log u
x +y =n =2 = 2u log u
∂x ∂y f ′ (u ) 1
u

Example 7
 xy + yz 
If u = coss  2
 x + y 2 + z 2 
+ sin
sin ( x + y + z , show that )
∂u ∂u ∂u 1
x
∂x
+y
∂y
+z =
∂z 2
( x + y + z cos ) ( x+ y+ z . )
Solution
Let u=v+w

 xy + yz 
where v = cos  2
 x + y 2 + z 2 
and w = sin ( x+ y+ z )
Replacing x by xt, y by yt, and z by zt,

 xy + yz 
v = t 0 coss  2 ,
 x + y 2 + z 2 
 1 
w = sin t 2 ( x + y + z  )
v is a homogeneous function of degree 0.
By Euler’s theorem,

∂v ∂v ∂v
x + y + z = 0⋅v = 0 ... (1)
∂x ∂y ∂z
7.9 Euler’s Theorem for Homogeneous Functions 7.141

w is a nonhomogeneous function. But sin −1 w = ( x+ y+ z ) is a homogeneous


1
function of x, y, z of degree .
2
Let f ( w) = sin −1 w
By cor. 2,

∂w ∂w ∂w f ( w) 1 sin −1 w
x +y +z =n =
∂x ∂y ∂z f ′ ( w) 2 1
1 − w2

=
1
2
(
x + y + z 1 − sin 2 ) ( x+ y+ z )
=
( ) (
x + y + z cos x+ y+ z ) ... (2)
2
Adding Eqs (1) and (2),

 ∂v ∂w   ∂v ∂w   ∂v ∂w 
x +  + y +  + z +  =
( x + y + z cco) (
os x+ y+ z )
 ∂x ∂x   ∂y ∂y   ∂z ∂z  2

Hence, x
∂u
+y
∂u
+z
∂u
=
( x + y + z cos) ( x+ y+ z )
∂x ∂y ∂z 2

Example 8
 ∂u ∂u   ∂v ∂v 
If x = eu tan v, y = eu sec v, prove that  x + y   x + y  = 0.
 ∂x ∂y   ∂x ∂y 
Solution
x = eu tan v, y = eu sec v
y − x = e 2u (sec 2 v − tan 2 v) = e 2u
2 2

x
and = sin v
y
 x
v = sin −1  
 y
v is homogeneous function of degree 0.
By Euler’s theorem,
∂v ∂v
x +y =0
∂∂xx ∂y
 ∂ u ∂ u  ∂ v ∂ v
Hence,  x ∂x + y ∂y   x ∂x + y ∂y  = 0
7.142 Chapter 7 Partial Derivatives

Example 9
Ê 1 1ˆ
x + y4 ˜ ∂u ∂u
If u = sin -1 ÁÁ 1
4

then find the values of x +y and
∂x ∂y
ÁË x 5 + y 5 ˜¯

∂2 y ∂2 u ∂2 u
x2 + 2 xy + y2 2 . [Summer 2017]
∂x 2 ∂x ∂y ∂y
Solution
Ê 1 1 ˆ
4 + y4
Á x ˜
u = sin -1 Á 1 1 ˜
ÁË x 5 + y 5 ˜¯
Replacing x by xt and y by yt,
È 1 Ê 1 1 ˆ˘

-1 Í 20
4 + y4 ˙
Á x ˜
u = sin Ít Á 1 1 ˜˙
Í ÁË x 5 + y 5 ˜¯ ˙
Î ˚
Ê 1 1 ˆ
Á x4 y4 ˜
u is a nonhomogeneous function. But sin u = Á 1 + 1 ˜ is a homogenous function
1 ÁË x 5 y 5 ˜¯
of degree .
20
Let f(u) = sin u
(i) By cor. 2,
∂u ∂u f (u) 1 sin u 1
x +y =n = = tan u
∂y ∂y f ¢(u) 20 cos u 20
(ii) By cor. 3,
∂2 u ∂2 u ∂2 u
x 2 2 + 2 xy + y2 = g(u) [ g ¢(u) - 1]
∂x ∂x ∂y ∂y 2
f (u) 1
where g(u) = n = tan u
f ¢(u) 20
1
g ¢(u) =sec 2 u
20
∂2 u ∂2 u 2 ∂ u
2
1 Ê 1 ˆ
Hence, x 2 + 2 xy + y = tan u Á sec 2 u - 1˜
∂x 2 ∂x ∂y ∂y 2 20 Ë 20 ¯
1 È1 ˘
= tan u Í (1 + tan 2 u) - 1˙
20 Î 20 ˚
7.9 Euler’s Theorem for Homogeneous Functions 7.143

1
= tan u(1 + tan 2 u - 20)
400
tan u
= (tan 2 u - 19)
400

Example 10
1
1 1
2
x
If u sin , prove that
x
1
(i) an u
x y
2
2
(ii) x an 13)
x x y

Solution 1
 1 1
3
2
1 x y
u sin 1 1
x y2
Replacing x by xt and y by yt,
1
2

1 
u= n 12 


1
1 1 2

u is a nonhomogeneous function. But sin u 1 1


is a homogeneous function

1
with degree .
12
Let f (u) = sin u
(i) By cor. 2,
∂u u f( ) 1 sin 1
x n an u
x y f ( ) cos
(ii) By cor. 3,
∂ 2u ∂ 2u ∂ 2u
x2 +2 u u 1]
∂x 2 ∂x y y
7.144 Chapter 7 Partial Derivatives

1
where g an u
f ( )
1
g′ sec 2 u
2

Hence, x an
y
1
= tan u
12
tan u
= tan 13)
144

Example 11
3 3
1
If u = log , find the value of
3
2
2 u
(i) (ii) x
x y x x y2
Solution
3 3
1
u = log 2 2
3
Replacing x by xt and y by yt,
3 3
1
u= l
3
x3 y3
u is a nonhomogeneous function. But e3u is a homogeneous function of
degree 1. x2 y2

Let f (u) = e3u


(i) By cor. 2,
∂u u f (u ) e 1
x n = ⋅ =
x y f ( ) 3
(ii) By cor. 3,

x ( ) 1]
∂ y
1
where
′ 3
′ ) 0.
2
u 1 1
Hence, x 2 2 0 1 =−
∂x x y 3 3
7.9 Euler’s Theorem for Homogeneous Functions 7.145

Example 12
2 2
If u tan , prove that
2
2 u
x = –2 sin3 u cos u.
x x y2
Solution 2 2
u = tan 1

Replacing x by xt and y by yt,


2 2

u = tan 

x2 y2
u is a nonhomogeneous function. But tan u is a homogeneous function of
degree 1. x y

Let f (u) = tan u


By cor. 3,
2
u
x2 1]
y
sin 2u
where =
( ) sec 2
′ cos u
2
u n u
Hence, x 2 cos 1)
∂ y
− sin 2 u )
−2 sin 3 cos u

Example 13
3 3
If u sinh , prove that
2
u
x2 2
= –tanh3 u.
x x y
Solution
3 3
u sinh −1 2 2
7.146 Chapter 7 Partial Derivatives

Replacing x by xt and y by yt,


3 3
u s nh −

x3 y3
u is a nonhomogeneous function. But sinh u is a homogeneous function of
degree 1. x2 y2

Let f (u) = sinh u


By cor. 3,
∂ 2u ∂ 2u 2
u
x2 +2 u u 1]
∂x 2 ∂x y y
sinh u
where g a u
cos u
g′ sec .
2
2
Hence, x a sec 1)
y
= tanh u(– tanh2 u)
= –tanh3 u

Example 14
x y x y
If u = log + sin −1 , prove that
x y x y
2
u s n cos w
x2 , where
x x cos w
x
w= n .

Solution
Let u=v+w
x x y
where v log , w sin −1
x y x y
Replacing x by xt and y by yt,
x y
1

v 0
t log , w = sin −
x y
v is a homogeneous function of degree 0.
7.9 Euler’s Theorem for Homogeneous Functions 7.147

By cor. 1,
2
v
x2 0 ... (1)
y
x y
and w is a nonhomogeneous function. But sin w = is a homogeneous
1 x y
function of degree
2
Let f (w) = sin w
By cor. 3,
∂2 w ∂2 w ∂2 w
x2 2 + 2 w w 1]
∂x ∂x y y
1 sin 1
where g an w
f ( ) cos
1
g′ sec .
2
Hence, x2 an
y
1 (1 2cos s n cos ... (2)
2 2cos cos w
Adding Eqs (1) and (2),
2
n cos w
x2 =−
cos3 w
2
u s n cos w x y
Hence, x 2 , where w = sin 1

cos w

Example 15
If u = log (x3 + y3 + z3 – 3xyz), show that
3 ∂u u u
(i) + + = (ii) x z =3
∂x z x z x z
∂u u u u u 2
u 9
(iii) + +2 2 2 =−
∂x z x z z x 2

Solution
u = log (x3 + y3 + z3 – 3xyz)
(i) Differentiating u w.r.t. x, y and z,
∂u 1
)
∂ + + − xyz
7.148 Chapter 7 Partial Derivatives

∂u 1
= (3 y 2 − 3 xz )
∂y x 3 + y 3 + z 3 − 3 xyz
∂u 1
= (3 z 2 − 3 xy )
∂z x 3 + y 3 + z 3 − 3 xyz
∂u ∂u ∂u 3( x 2 + y 2 + z 2 − xy − yz − zx)
+ + =
∂x ∂y ∂z x 3 + y 3 + z 3 − 3 xyz
3( x 2 + y 2 + z 2 − xy − yz − zx)
=
( x + y + z )( x 2 + y 2 + z 2 − xy − yz − zx)
3
=
x+ y+ z
(ii) Replacing x by xt, y by yt and z by zt,
u = log t 3 ( x 3 + y 3 + z 3 − 3 xyz )
u is a nonhomogeneous function. But eu = x3 + y3 + z3 – 3xyz is a homogeneous function
of degree 3.
Let f (u) = eu
By cor. 2,
∂u ∂u ∂u f (u ) eu
x +y +z =n = 3 u = 3.
∂x ∂y ∂z f ′ (u ) e

∂ 2u ∂ 2u ∂ 2u ∂ 2u ∂ 2u ∂ 2u  ∂ ∂ ∂   ∂u ∂u ∂u 
(iii) + + + 2 + 2 + 2 = + +  + +
∂x 2
∂y 2
∂z 2
∂x ∂y ∂y ∂z ∂z ∂x  ∂x ∂y ∂z   ∂x ∂y ∂z 
 ∂ ∂ ∂ 3 
= + + 
 ∂x ∂y ∂z   x + y + z 
3
=− (1 + 1 + 1)
( x + y + z )2
9
=−
( x + y + z )2

Example 16
∂ 2u ∂ 2u 2 ∂ u
2

If u = log r and r = x + y , prove that x


2 2 2
2
+ 2xy
2 xy + y + 1 = 0.
∂x 2 ∂x ∂y ∂y 2
Solution
u = log r = log x 2 + y 2
7.9 Euler’s Theorem for Homogeneous Functions 7.149

Replacing x by xt and y by yt,


u = log t x 2 + y 2

u is a nonhomogeneous function of x and y. But e = x + y is a homogeneous


u 2 2

function of degree 1.
Let f (u) = eu
By cor. 3,
∂ 2u ∂ 2u ∂ 2u
x 2 2 + 2 xy + y 2 2 = g (u ) [ g ′ (u ) − 1]
∂x ∂x ∂y ∂y
f (u ) eu
where g (u ) = n = =1
f ′ (u ) eu
g ′ (u ) = 0.
∂u
2
∂u 2
2 ∂ u
2
x2 + 2 xy + y = 1(0 − 1) = −1
∂x 2 ∂x ∂y ∂y 2
∂2u ∂ 2u ∂ 2u
Hence, x2 + 2 xy + y2 2 + 1 = 0
∂x 2
∂x ∂y ∂y

Example 17 2
 ∂ ∂ 4
If u = log r, r = x + y – x y – xy , show that  +  u = −
3 3 2 2
 ∂x ∂y  ( x + y)2
∂u ∂u
and x + y = 3.
∂x ∂y
Solution
2
 ∂ ∂  ∂ ∂   ∂u ∂u 
 ∂x + ∂y  u =  ∂x + ∂y   ∂x + ∂y
∂y 
u = log( x 3 + y 3 − x 2 y − xy
xy 2 )
= log ( x + y )( x 2 + y 2 − xy ) − xy ( x + y ) 
= log( x + y )( x 2 + y 2 − 2 xy ) = llog(
og( x + y ) + 2 log( x − y )
∂u 1 2
= +
∂x x+ y x− y
∂u 1 2
= −
∂y
∂y x+ y x− y
∂u ∂u 2
+ =
∂x ∂∂yy x + y
2
 ∂ ∂  ∂ ∂  2 
 ∂x + ∂y  u =  ∂x + ∂y   x + y 

2 2 4
=− − =−
(xx + y ) 2 ( x + y ) 2 ( x + y)2
7.150 Chapter 7 Partial Derivatives

Replacing x by xt and y by yt in u,
u = log t3 (x3 + y3 – x2y – xy2)
u is a nonhomogeneous function. But eu = x3 + y3 – x2y – xy2 is a homogeneous function
of degree 3.
Let f (u) = eu
By cor. 2,
∂u ∂u f (u) eu
x +y =n =3 u =3
∂x ∂y f ′( u ) e

Example 18
−1 ∂ 2u ∂ 2u 2 ∂ u
2
If u = sin x + y , prove that x
2 2
+ 2 xy 2
+y = tann 3 u.
∂x 2
∂x ∂y ∂y 2

Solution
u = sin −1 x 2 + y 2
Replacing x by xt and y by yt,
u = sin −1 t x 2 + y 2 
 
u is a nonhomogeneous function. But sin u = x 2 + y 2 is a homogeneous function of
degree 1.
Let f (u ) = sin u
∂u ∂u f (u ) sin u
x +y =n =1 = tan u
∂x ∂y f ′ (u ) cos u
∂2u ∂2u ∂2u
x2 + 2 xy + y 2 2 = g (u )[ g ′ (u ) − 1]
∂x 2
∂x ∂y ∂∂yy
f (u )
where g (u ) = n = tan u
f ′ (u )
g ′ (u ) = sec 2 u

∂2u ∂2u 2 ∂ u
2
Hence, x2 + 2 xy + y = tan u (sec 2 u − 1)
∂x 2 ∂x ∂y ∂y 2
= tann u (ta
tann 2 u )
((tan
= tan 3 u

Example 19
∂ 2u ∂ 2u 2 ∂ u
2
Find x 2 + 2 xy + y if u = log( x3 + y 3 − x 2 y − xxy 2 ) .
∂x 2
∂x ∂y ∂y 2
7.9 Euler’s Theorem for Homogeneous Functions 7.151

Solution
u = log( x 3 + y 3 − x 2 y − xy
x 2)
Replacing x by xt and y by yt,
u = log[t 3 ( x 3 + y 2 − x 2 y − xy 2 )]
u is a nonhomogeneous function. But eu = x 3 + y 3 − x 2 y − xy
x 2 is a homogeneous
function of degree 3.
Let f (u ) = eu
∂u ∂u f (u ) eu
x +y =n =3 u =3
∂x ∂y f ′ (u ) e
∂2u ∂2u ∂2u
x2 + 2 xy + y 2 2 = g (u )[ g ′ (u ) − 1]
∂x 2
∂x ∂y ∂y
f (u )
where g (u ) = n =3
f ′ (u )
g ′ (u ) = 0

∂u 2
∂u
2
∂u 2
Hence, x2 + 22xy
xy + y 2 2 = 3(0 − 1) = −3
∂x 2 ∂x ∂y ∂y

Example 20
x2 y 2 z 2 −1
 x+ y+ z 
If u = + cos   then find the value of
x2 + y 2 + z 2  x + y + z
∂u ∂u ∂u
x +y +z .
∂x ∂y ∂z
Solution
Let u=v+w
x2 y 2 z 2  x+ y+ z 
v= 2 , w = cos −1
where x + y2 + z2  x + y + z 
Replacing x by xt and y by yt,
 x2 y 2 z 2   1  x+ y + z 
v = t4  2 w = cos −1 t 2 
, 
 x + y 2 + z 2    x + y + z  
Hence, v is a homogeneous function of degree 4.
By Euler’s theorem,
∂v ∂v ∂v
x +y +z = nv = 4v ...(1)
∂x ∂y ∂z
and w is a nonhomogeneous function. But cos w is a homogeneous function of degree
1
.
2
7.152 Chapter 7 Partial Derivatives

Let f ( w) = cos w
By cor. 2,
∂w ∂w ∂w f ( w) 1 cos w 1
x +y +z =n = = − cot w ...(2)
∂x ∂y ∂z f ′ ( w) 2 ( − sin w) 2
Adding Eqs (1) and (2),

 ∂v ∂w   ∂v ∂w   ∂v ∂w  1
x +  + y  +  + z  +  = 4v − cott w
 ∂x ∂x   ∂y ∂y   ∂z ∂z  2

∂u ∂u ∂u  x2 y 2 z 2  1   x+ y + z 
Hence, x +y +z = 4 2 2
− cott cos −1  
∂x ∂y ∂z x +y +z  2
2
  x + y + z  

Example 21
If u = sin ( )
x + y , prove that
∂u ∂u 1
x
∂x
+y =
∂y 2
( x + y coss ) ( x+ y . )
Solution
u = sin ( x+ y )
Replacing x by xt and y by yt,
 1 
u = sin t 2

( x+ y 

)
u is a nonhomogeneous function. But sin −1 u = x + y is a homogeneous function
1
of degree .
2
Let f (u ) = sin −1 u
By cor. 2,
∂u ∂u f (u )
x +y =n
∂x ∂y f ′ (u )
1 sin −1 u
=
2 1
1 − u2
1
= sin −1 u
1 − u 2 sin
2
7.9 Euler’s Theorem for Homogeneous Functions 7.153

=
1
2
( )(
1 − sin 2 x + y ⋅ x + y )
=
1
2
( ) (
x + y cos x + y )

Example 22
Ê x 2 + y2 ˆ
If u = tan -1 Á ˜ , prove that
Ë x +y ¯
∂2 u ∂2 u ∂2 u
x2 + 2 xy + y 2 2 = - 2 sin3 u cos u . [Winter 2016]
∂x 2 ∂x ∂y ∂y
Solution
Ê x 2 + y2 ˆ
u = tan -1 Á ˜
Ë x+y ¯
Replacing x by xt and y by yt,
È Ê x 2 + y2 ˆ ˘
u = tan -1 Ít Á ˜˙
ÍÎ Ë x + y ¯ ˙˚

x 2 + y2
u is a nonhomogenous function. But tan u = is a homogeneous function of
degree 1. x+y

Let f(u) = tan u

By cor. 3,
∂2 u ∂2 u ∂2 u
x2 + 2 xy + y 2 2 = g(u) [ g ¢(u) - 1]
∂x 2 ∂x ∂y ∂y
nf (u) tan u 1
where g(u) = = 1◊ = sin u cos u = sin 2u
f ¢(u) 2
sec u 2
g¢(u) = cos 2u

∂2 u ∂2 u ∂2 u sin 2u
Hence, x2 + 2 xy + y2 2 = (cos 2u - 1)
∂x 2 ∂x ∂y ∂y 2

= sin u cos u [ -2 sin 2 u]


= –2 sin3u cos u
7.154 Chapter 7 Partial Derivatives

Example 23
x2 y 2 ∂u ∂u
If sinn u = , find x + y .
x+ y ∂x ∂y
Solution
x2 y 2
Let f (u ) = sin
sin u =
x+ y
Replacing x by xt and y by yt
 x2 y 2 
f (u ) = sin
sin u = t 3 
 x + y 
f(u) is a homogeneous function of degree 3.
By cor. 2,
∂u ∂u f (u )
x +y =n
∂x ∂y f ′ (u )
sin u
=3
cos u
= 3 tan u

Example 24
 x3 + y 3 
If u = tann −1  , prove that
 x − y 
∂u ∂u
(i) x +y = sinn 2u.
∂x ∂y

∂ 2u ∂ 2u 2 ∂ u
2

(ii) x
2
+ 2xy
2 xy + y = 2 sin
n u cos 3u [Summer 2015]
∂x 2 ∂x ∂y ∂y 2

Solution
 x3 + y 3 
u = tan −1 
 x − y 

Replacing x by xt and y by yt,


  x3 + y 3  
u = tan −1 t 2  
  x − y 
 
7.9 Euler’s Theorem for Homogeneous Functions 7.155

x3 + y 3
u is a nonhomogeneous function. But tan u = is a homogeneous function of
degree 2. x− y

Let f (u) = tan u


By cor. 2,
∂u ∂u f (u ) tan u
tan
x +y =n = 2 2 = 2 sinn u cos u = sin 2u
∂x ∂y f ′ (u ) sec u
By cor. 3,
∂2u ∂2u 2 ∂ u
2
x2 + 2 xy + y = g (u )[ g ′ (u ) − 1]
∂x 2 ∂x ∂y ∂y 2

f (u )
where g (u ) = n = sin 2u
f ′ (u )
g ′ (u ) = 2 cos
cos 2u

∂2u ∂2u 2 ∂ u
2
Hence, x2 + 2 xy + y = sin 2u (2 cos 2u − 1)
∂x 2 ∂x ∂y ∂y 2
= 2 sin 2u cos 2u - sin 2u
= sin 44u
u - sin 2u
= 2 sin u cos 3u

Example 25
If u = tan–1 (x2 + 2y2), show that x2uxx + 2xy uxy + y2uyy = 2 sin u cos 3u.
[Summer 2016]
Solution
u = tan–1(x2 + 2y2)
Replacing x by xt and y by yt,
u = tan–1[t2(x2 + 2y2)]
u is a nonhomogeneous function. But tan u = x2 + 2y2 is a homogeneous function of
degree 2.
Let f (u) = tan u
By cor. 3,
∂2 u ∂2 u ∂2 u
x2 + 2 xy + y 2 2 = g(u)[ g ¢(u) - 1]
∂x 2 ∂x ∂y ∂y

f (u) tan u
where g(u) = n =2 = 2 sin u cos u = sin 2u
f ¢(u) sec 2 u
7.156 Chapter 7 Partial Derivatives

g ¢(u) = 2 cos 2u

∂2 u ∂2 u ∂2 u
Hence, x2 + 2 xy + y 2 2 = sin 2u [2 cos 2u - 1]
∂x 2 ∂x ∂y ∂y

x 2 uxx + 2 xy uxy + y 2 uyy = 2 sin 2u cos 2u - sin 2u


= sin 4u - sin 2u
= 2 sin u cos3u

Example 26
If u = tan–1(x2 + xy + 2015y2) then show that
∂u ∂u
(i) x +y = sin 2u
∂x ∂y

2 ∂ 2u ∂ 2u ∂ 2u
(ii) x + 22xy
xy + y 2 2 = 2 sin
n u cos 3u [Winter 2015]
∂x 2 ∂x ∂y ∂y

Solution
u = tan–1(x2 + xy + 2015y2)
Replacing x by xt and y by yt,
u = tan–1[t2(x2 + xy + 2015y2)]

u is a nonhomogeneous function. But tan u = x2 + xy + 2015y2 is a homogeneous


function of degree 2.

Let f (u) = tan u


By cor. 2,
∂u ∂u f (u ) tan u
tan
x +y =n = 2 2 = 2 sinn u cos u = sin 2u
∂x ∂y f ¢ (u ) sec u
By cor. 3,
∂2 u ∂2 u ∂2 u
x2 + 2 xy + y 2 2 = g(u)[[g
g ¢(u) - 1]
∂x 2 ∂x ∂y ∂y

f (u)
where g(u) = n = sin 2u
f ¢(u)
g ¢(u) = 2 cos
cos 2u
7.9 Euler’s Theorem for Homogeneous Functions 7.157

∂2 u ∂2 u ∂2 u
Hence, x2 + 2 xy + y 2 2 = sin 2u(2 cos 2u - 1)
∂x 2 ∂x ∂y ∂y
= 2 sin 2u cos 2u - siin 2u
= sin
sin 4u - ssin
in 2u
= 2 sin
sin u cos
cos 3u

Example 27
Ê 1 1 ˆ
Á x + y2
2 ˜
If u = cosec -1 Á 1 1 ˜ , prove that
Á x3 + y3 ˜
Ë ¯
∂ 2u ∂ 2u 2 ∂ u
2
tan u
x2 + 2 xy + y = (13 + tan 2 u ). [Winter 2014]
∂x 2
∂x ∂y ∂y 2
144

Solution
Ê
1 1 ˆ

-1 Á
x2 + y2 ˜
u = cosec Á 1 1 ˜
Á x3 + y3 ˜
Ë ¯
Replacing x by xt and y by yt,
Ê 1 1 ˆ
1
-1 Á 12 x2 + y2 ˜
u = cosec Á t 1 1 ˜
Á x 3 + y 3 ˜¯
Ë
1 1
x2 + y2
u is a nonhomogeneous function. But cosec u = 1 1
is a homogeneous function
x3 + y3
1
of degree .
12
Let f (u) = cosec u
By cor. 3,
∂2u ∂2u ∂2u
x2 + 2 xy + y 2 2 = g (u )[ g ′ (u ) − 1]
∂x 2
∂x ∂y ∂y
7.158 Chapter 7 Partial Derivatives

f (u ) 1 cosec u 1
where g (u ) = n = = − tan u
f ′ (u ) 12 ((−cosec u cot u ) 12

1 1
g ′ (u ) = − secc 2 u = − (1 + tan
tan 2 u )
12 12

∂2 u ∂2 u ∂2 u 1 È 1 ˘
Hence, x2 + 2 xy + y 2 2 = - tann u Í- (1 + tan 2 u) - 1˙
∂x 2 ∂x ∂y ∂y 12 Î 12 ˚

1 Ê -1 - tan 2 u - 12 ˆ
=- tan u Á ˜
12 Ë 12 ¯
tan u
= (13 + tan 2 u)
144

Example 28
-1
Ê x+y ˆ
If u = sinn Á ˜ , prove that
Ë x + y¯
∂u ∂u
(i) 22x + 2y = tan u
∂x ∂y

∂ 2u ∂ 2u 2 ∂ u
2
(ii) x 2
∂x 2
+ 2 xy
∂x ∂y
+ y
∂y 2
1
= tan 3 u − tan u
4
( ) [Winter 2013]

Solution
Ê x+y ˆ
u = sin -1 Á ˜
Ë x + y¯

Replacing x by xt and y by yt,


È Ê x + y ˆ˘
u = sin -1 Ít1/ 2 Á ˜˙
ÍÎ Ë x + y ¯ ˙˚

x+y
u is a nonhomogeneous function. But sin u = is a homogeneous function of
1 x+ y
degree .
2
7.9 Euler’s Theorem for Homogeneous Functions 7.159

Let f (u) = sin u


By cor. 2,
∂u ∂u f (u )
x +y =n
∂x ∂y f ′ (u )
1 sin u
=
2 cos u
1
= tan u
2
∂u ∂u
2
2x + 2y = tan u
∂x ∂y
By cor. 3,
∂2u ∂2u 2 ∂ u
2
x2 + 2 xy + y = g (u )[ g ′ (u ) − 1]
∂x 2 ∂x ∂y ∂y 2

f (u ) 1 sin u 1
where g (u ) = n = ⋅ = ⋅ tan u
f ′ (u ) 2 cos u 2
1 2 1
g ′ (u ) = sec u = ((ta
tan 2 u + 1)
tan
2 2

∂2u ∂ 2u ∂ 2u 1 1 
Hence, x2
∂x 2
+ 2 xy
∂x ∂y ∂y 2  2
(
+ y 2 2 = tan u  tan 2 u + 1 − 1

)
1 1 1
= tan u  tan 2 u − 
2 2 2
1
= (tan 3 u − tan
tan u )
4

Example 29
−1 x2 + y 2
If u = sinn , show that
x+ y
∂ 2u ∂ 2u 2 ∂ u
2
1
x2 + 2 xy + y = tan u (tan 2 u − 1).
∂x 2
∂x ∂y ∂y 2
4
Solution
x2 + y 2
u = sin −1
x+ y
Replacing x by xt and y by yt,
 x2 + y 2 
u = sin −1 t1/ 2 
 x + y 
7.160 Chapter 7 Partial Derivatives

x2 + y 2
u is a nonhomogeneous function. But sin u = is a homogeneous function
x+ y
1
of degree .
2
Let f(u) = sin u
By cor. 3,
∂2u ∂2u ∂2u
x 2 2 + 2 xy + y 2 2 = g (u )[ g ′ (u ) − 1]
∂x ∂x ∂y ∂y
f (u ) 1 sin u 1
where g (u ) = n = = tan u
f ′ (u ) 2 cos u 2
1 2
g ′ (u ) = secc u.
2
∂2u ∂2u 2 ∂ u
2
1 1 
Hence, x2 + 2 xy + y = tan u  sec 2 u − 1
∂x 2 ∂x ∂y ∂y 2 2 2 

1 1 + tan 2 u − 2 
1+
= tan u  
2  2
1
= tan u (ta n 2 u −1
tan − 1)
4

Example 30
1
-1
If u = sin ( x + 2 2 5
y ) , show that
∂ 2u ∂ 2u 2 ∂ u
2
2
x2 + 2 xy + y = tan u (2 tan 2 u − 3).
∂x 2
∂x ∂y ∂y 2
25
Solution 1
u = sin −1 ( x 2 + y 2 ) 5
Replacing x by xt and y by yt,
 2 1

u = sin −1 t 5 ( x 2 + y 2 ) 5 
1
u is a nonhomogeneous function. But sinn u = ( x 2 + y 2 ) 5 is a homogeneous function
2
of degree .
5
Let f(u) = sin u
By cor. 3,

∂2u ∂2u 2 ∂ u
2
x2 + 2 xy + y = g (u )[ g ′ (u ) − 1]
∂x 2 ∂x ∂y ∂y 2
7.9 Euler’s Theorem for Homogeneous Functions 7.161

f (u ) 2 sin u 2
where g (u ) = n = = tan u
f ′ (u ) 5 cos u 5
2 2
g ′ (u ) = sec u
5
∂2u ∂2u ∂2u 2 2 
Hence, x2 + 2 xy + y 2 2 = tan u  sec 2 u − 1
∂x 2
∂x ∂y ∂y 5 5 

2  2 + 2 tan 2 u − 5 
= tan u  
5  5
2
= tan u (2 tan 2 u − 3)
25

EXERCISE 7.6

 x+y  ∂u ∂u 1
1. If u = coss −1 , prove that x +y = − cot u.
 x + y  ∂x ∂y 2

 x 2y 2  ∂u ∂u
n−1 
2. If u = sin  , prove that x +y = 3 tan u.
 x + y ∂x ∂y

∂u ∂u
3. If u = log (x2 + xy + y2), prove that x +y = 2.
∂x ∂y

∂u ∂u
4. If (x – y) tan u = x3 + y3, prove that x +y = sin2u.
∂x ∂y

∂u ∂u
5. If u = log (x3 + y3 – x2y – xy2), prove that x +y = 3.
∂x ∂y

 x 3 + y 3 + z3  ∂u ∂u ∂u
6. If u = sin–1   , prove that x +y +z = 2 tan u.
 ax + by + cz  ∂x ∂y ∂z

 x3 + y3  ∂u ∂u
n−1 
7. If u = tan  , prove that x +y = sin2u.
 x−y  ∂x ∂y

 41 1

−1  x + y 4
 , prove that
8. If u = sin
n
 61 1

 x + y6 
∂ 2u ∂ 2u 2 ∂ u
2
1
x2 + 2 xy + y = tan u(tan2 u − 1).
∂x 2
∂x ∂y ∂y 2
144
Points to Remember 7.162

1 1
9. If x + y sin2 u = x 3 + y 3 , prove that
∂ 2u ∂ 2u 2 ∂ u
2
n u  13 tan2 u 
tan
x2 + 2 xy + y =  + .
∂x 2 ∂x ∂y ∂y 2 12  12 12 

 x 5 − 2y 5 + 6 z 5  ∂u ∂u ∂u 7
10. If u = coss −1  , show that x +y +z = − cot .
3  ∂ x ∂ y ∂ z 2
 ax + by + cz 
3 3

∂u ∂u
11. If x + y cot u − x − y = 0, prove that 4 x + 4y + sin2u = 0.
∂x ∂y
 x+y 
Hint : cot u = 
 x+ y

 ax + by + cz  ∂u ∂u ∂u
n−1
12. If u = sin , prove that x +y +z = 2 tan u.
 x n + y n + zn  ∂x ∂y ∂z
 

 1

π (2 x 2
+ y 2
+ xz ) 2
13. If v = log sin   , prove that
at
 1 
 2(x + x
2
xyy + 2yyzz + z ) 
2 3

∂v ∂v ∂v 1 −−vv
x +y +z = e 1 − e 2v ⋅ sin−1(e v )
∂x ∂y ∂z 3

POINTS TO REMEMBER
• Chain Rule

∂z dz ∂u df ∂u ∂u
= or or f ′(u)
∂x du ∂x du ∂x ∂x
∂z dz ∂u df ∂u ∂u
= or or f ′(u)
∂y du ∂y du ∂y ∂y

• Composite Function of One Variable

du ∂u dx ∂u dy
= +
dt ∂x dt ∂y dt

du ∂u d x ∂u dy ∂u dz
= + + .
dt ∂x dt ∂y dt ∂z dt
Multiple Choice Questions 7.163

• Composite Function of Two Variables


∂z ∂z ∂x ∂z ∂y
= +
∂u ∂x ∂u ∂y ∂u
∂z ∂z ∂x ∂z ∂y
= +
∂v ∂x ∂v ∂y ∂v

• Implicit Differentiation

∂f
dy ∂ f
=− x =− x.
dx ∂f fy
∂y

• Euler’s Theorem for Homogeneous Functions


∂u ∂u
x +y = nu
∂x ∂y
∂u ∂u ∂u
x +y +z = nu
∂x ∂y ∂z

∂2u ∂2u ∂2u


x2 +2
2xy
xy + y2 2 = n
n((n − 1)u.
∂x 2
∂x ∂y ∂y

MULTIPLE CHOICE QUESTIONS

Choose the correct alternative in the following questions:


x-y
1. The value of lim is
( x , y )Æ(0,0) x+y
(a) limit does not exist (b) 0
(c) 1 (d) –1
x+ y
2. The value of lim , x π 0, y π 0 is
( x , y )Æ(0,0)
x2 + y
(a) limit does not exist (b) 0
(c) 1 (d) –1
x2 y
3. The value of lim is
( x , y )Æ(0,0) x 4 + y2
1
(a) 0 (b) (c) 1 (d) does not exist
2
7.164 Chapter 7 Partial Derivatives

xy
4. The value of lim is
( x , y )Æ(0,0) y - x2
2

(a) 0 (b) 1 (c) –1 (d) does not exist


2
8x y
5. The value of lim is
x Æ1 x + y2 + 5
2
y Æ1

3 8 8 3
(a) (b) (c) (d)
7 5 7 5
4 xy
6. The value of lim is
x Æ1 6 x 2 + y2
yÆ2

4 2 3 2
(a) (b) (c) (d)
5 3 10 5
2 x2 + y
7. The value of lim is
x Æ1 4x - y
yÆ0

3 1 5
(a) (b) (c) 1 (d)
2 2 2
2 x2 + y
8. The value of lim is
xÆ0 4 x2 - y
yÆ0

1
(a) –1 (b) (c) 1 (d) does not exist
2
x 2 - xy
9. The value of lim is [Winter 2015]
( x , y )Æ(0,0) x- y
(a) 1 (b) 0 (c) –1 (d) does not exist
x - yx 2
10. The value of lim is [Winter 2014]
( x , y )Æ(0,0) x + y

(a) 2 (b) 1 (c) 0 (d) –1


∂r ∂r
11. If x = r cosq, y = r sinq then = — and =—
∂x ∂y
x x y x
(a) , tan q (b) , (c) tan q, sin q (d) ,sin q
r r r r
∂u
12. If u = sin (ax + by + cz) then =
∂x
(a) a cos (ax + by + cz) (b) a sin (ax + by + cz)
(c) b cos (ax + by + cz) (d) b sin (ax + by + cz)
Multiple Choice Questions 7.165

∂u ∂x ∂u
13. If u = x 2 y + y 2 z + z 2 x then + + =
∂x ∂y ∂z
1 1
(a) x + y + z (b) (x + y + z)2 (c) (d)
x+ y+z ( x + y + z )2
∂2 f
14. If f = x2 + y2 then =
∂x ∂y
(a) 1 (b) 0 (c) –1 (d) 2
∂u
15. If u = log (x2 + y2) then =
∂x
2y 2 2x y
(a) (b) (c) (b)
x +y
2 2
x +y
2 2
x +y
2 2
x + y2
2

∂u
16. If u = sin (x + y) then =
∂y
(a) sin x (b) cos(x + y) (c) tan(x + y) (d) cos x
∂ u ∂2 u 2
17. If u = ex (x cos y – y sin y) then + =
∂x 2 ∂y 2
(a) 1 (b) –1 (c) 0 (d) 2
∂u
18. If u = xy then is
∂x
(a) 0 (b) yxy –1 (c) xy log x (d) yxy
∂u ∂u
19. If u = yx then = —— and = ——
∂x ∂y
(a) log y,yx–1 (b) yxlog x, x (c) yx, xyx–1 (d) yx log y, xyx–1
∂2 u
20. If u = x3 + y3 then =
∂x ∂y
(a) –3 (b) 3 (c) 0 (d) 1
∂ u 2
21. If u = f (x + ay) + g (x – ay) then is
∂y 2
∂2 u ∂2 u ∂2 u ∂2 u
(a) (b) a (c) a 2 (d)
∂x 2
∂x 2
∂x 2 ∂x ∂y
∂u ∂u ∂u
22. If u = ( x - y) + ( y - z ) + ( z - x ) then + +
4 4 4
is
∂x ∂y ∂z
(a) 1 (b) u (c) 4u (d) 0
23. If u = tan–1(x + y) then ux – uy is
(a) 0 (b) 1 (c) –1 (d) sin x cos y
7.166 Chapter 7 Partial Derivatives

24. If f = log (x tan –1 y) then fxy is equal to


1 1 1
(a) - 2 (b) 0 (c) 2 (d)
x x x
∂ z
25. If z = x2 – y2 then = [Winter 2014]
∂x
(a) 2y (b) 0 (c) 2z (d) 2x
3 cos ( xw) sin y 5
∂f
26 If f(x, y, z, w) = + 5 xzw then at (1,2,3, 4) is
(1 + y 2 ) ∂z
ey +
xy w [Winter 2015]
(a) 20 (b) 200 (c) 0 (d) 1
Ê xˆ Ê yˆ
27. If u = sin -1 Á ˜ + tan -1 Á ˜ then the value of xux + yuy is [Summer 2015]
Ë y¯ Ë x¯
(a) u (b) –u (c) 0 (d) 1
1
28. For the function u = ( x 2 + y2 ) 3 , the value of x 2 uxx + 2 xy uxy + y 2 uyy is
[Summer 2015]
2u u 2u
(a) –2u (b) - (c) (d) -
3 9 9
y
x Ê yˆ ∂f ∂f
29. If f(x,y) = + sin Á ˜ e x then the value of x +y is [Winter 2015]
y Ë x¯ ∂x ∂y
(a) 1 (b) 0 (c) –1 (d) f
x+ y
30. The degree of the homogeneous function z = is
x+y
1 1
(a) (b) - (c) 1 (d) 2
2 2
Ê yˆ ∂u ∂u
31. If u = tan–1 Á ˜ then x +y =
Ë x¯ ∂x ∂y
(a) 0 (b) sin 2u (c) cos 2u (d) 1
Ê x ˆ ∂u ∂u
32. If u = xy 2f Á ˜ then x +y =
Ë y¯ ∂x ∂y
(a) u (b) 2u (c) 3u (d) 4u
xy ∂ u ∂ u
33. If u = then x +y =
x+y ∂x ∂y
(a) 0 (b) 1 (c) u (d) 2u
∂u ∂u ∂u
34. If u = log ( x 3 + y3 + z 3 - 3 xyz ) then x +y +z =
∂x ∂y ∂z
(a) 4 (b) 3 (c) 1 (d) 0
Multiple Choice Questions 7.167

Ê x+y ˆ ∂u ∂u
35. If u = sin–1 Á ˜ and x +y = m tan u then m =
Ë x + y¯ ∂ x ∂y
1 1
(a) (b) – (c) 1 (d) 2
2 2
x

36. If u = e then x ux + y uy =
y

(a) 0 (b) 1 (c) 2 (d) –1


Ê yˆ
37. If u = f Á ˜ then xux + yuy =
Ë x¯
(a) 1 (b) –xy (c) 0 (d) xy
38. If z is a homogeneous function of degree n then x z xx + 2 xy z xy + y 2 z yy =2

(a) nz (b) 0 (c) n(n–1)z (d) 1


39. If u = 3 x 2 yz + 5 x 2 y 2 + 4 z 4 then xux + yuy + zuz =
(a) 4u (b) 0 (c) 8u (d) 2u
x 2
∂u ∂u
40. If u = log then x +y is
y ∂x ∂y
(a) 2u (b) 3u (c) u (d) 1
41. If z = f(x,y), dz is equal to
Ê ∂f ˆ Ê ∂f ˆ Ê ∂f ˆ Ê ∂f ˆ
(a) Á ˜ dx + Á ˜ dy (b) Á ˜ dx + Á ˜ dy
Ë ∂x ¯ Ë ∂y ¯ Ë ∂y ¯ Ë ∂x ¯

Ê ∂f ˆ Ê ∂f ˆ Ê ∂f ˆ Ê ∂f ˆ
(c) Á ˜ dx - Á ˜ dy (d) Á ˜ dx - Á ˜ dy
Ë ∂x ¯ Ë ∂y ¯ Ë ∂y ¯ Ë ∂x ¯
x y
Ê xˆ Ê xˆ ∂u ∂u
42. If u = e sin Á ˜ + e x cos Á ˜ then x
y
+y is
Ë y¯ Ë y¯ ∂x ∂y
(a) 2u (b) u (c) 0 (d) 3u
Ê x 2 + y2 ˆ ∂z ∂z
43. If z = sin -1 Á ˜ then x +y is
ÁË x + y ˜¯ ∂x ∂y

1
(a) 0 (b) (c) 1 (d) 2
2
1 1 1 ∂f ∂f
44. If f ( x, y) = + + then x +y is
x 3
y 3
x +y3 3 ∂x ∂y
(a) 0 (b) 3f (c) 9 (d) –3f
7.168 Chapter 7 Partial Derivatives

x- y
45. is a homogeneous function of degree ——
x+ y
1
(a) 0 (b) 1 (c) –1 (d)
2
dy
46. For an implicit function f(x,y) = c, the value of is [Summer 2017, 2015]
dx
fx fy fx fy
(a) (b) (c) - (d) -
fy fx fy fx
dy
47. If x 3 + y3 + 3 xy = 0 then =
dx
x2 - y x2 + y x2 + y x2 + y
(a) (b) - (c) (d)
y2 - x y2 + x x2 - y x-y
du
48. If u = sin ( xy 2 ), x = log t , y = et then =
dt
Ê1 ˆ
(a) y 2 Á + 2 x ˜ cos xy 2 (b) 0
Ët ¯
(c) 1 (d) –1
∂f
49. If f is a function of u, v, w and u, v, w are functions of x, y, z then is
∂y
∂f ∂u ∂f ∂v ∂f ∂w ∂f ∂x ∂f ∂y ∂f ∂z
(a) ◊ + ◊ + ◊ (b) ◊ + ◊ + ◊
∂u ∂z ∂v ∂z ∂w ∂z ∂x ∂v ∂y ∂u ∂z ∂u
∂f ∂u ∂f ∂v ∂f ∂w ∂f ∂x ∂f ∂y ∂f ∂z
(c) ◊ + ◊ + ◊ (d) ◊ + ◊ + ◊
∂u ∂y ∂v ∂y ∂w ∂y ∂x ∂v ∂y ∂v ∂z ∂v
∂x
50. If u = x 2 - y 2 , v = xy then is
∂u
x y y x
(a) (b) (c) (d)
2( x + y )
2 2
2( x + y )
2 2
x +y
2 2 x 2 + y2
2
51. If f ( x, y) = e , the total differential of the function at the point (1,2) is
xy

(a) e(dx + dy) (b) e4 (dx + dy) (c) e4 (4dx + dy) (d) 4e4 (dx + dy)

Ê yˆ ∂u ∂u
52. The u = x 3 cos Á ˜ then x +y = [Summer 2016]
Ë x¯ ∂x ∂y
(a) u (b) 2u (c) 3u (d) None of these
53. Let f(x, y) = y sin (xy). The value of fx(p, 1) is
(a) 0 (b) 1 (c) –1 (d) 2.5
Multiple Choice Questions 7.169

x y
Ê xˆ Ê yˆ ∂f ∂f
54. If f = e tan Á ˜ + e x cot Á ˜ then x
y
+y is [Winter 2016]
Ë y¯ Ë x¯ ∂x ∂y
(a) 0 (b) f (c) –f (c) 2f
Ê yˆ
55. The function f ( x, y) = x y f Á ˜ is homogeneous of degree n =
2
Ë x¯
[Winter 2016]

(a) 0 (b) 1 (c) 2 (d) 3

Answers

1.(a) 2.(c) 3.(d) 4.(d) 5.(c) 6.(a) 7.(b) 8.(d) 9.(d) 10.(c)
11.(b) 12.(a) 13.(b) 14.(b) 15.(c) 16.(b) 17.(c) 18.(b) 19.(d) 20.(c)
21.(c) 22.(d) 23.(a) 24.(b) 25.(d) 26.(a) 27.(c) 28.(d) 29.(b) 30.(b)
31.(a) 32.(c) 33.(c) 34.(b) 35.(a) 36.(a) 37.(c) 38.(c) 39.(a) 40.(d)
41.(a) 42.(c) 43.(a) 44.(d) 45.(d) 46.(c) 47.(b) 48.(a) 49.(c) 50.(a)
51.(d) 52.(c) 53.(c) 54.(a) 55.(d)
CHAPTER
Applications
8
of Partial
Derivatives
8.1 INTRODUCTION
In this chapter, we will study various applications of partial derivatives. Most of the
applications are extensions of applications of ordinary derivatives. Partial derivatives
are useful in finding equations of tangent planes and normal lines for surfaces. They are
applied in approximating any function and error calculation. An important application
of partial derivatives is finding the maximum and minimum values of function of
several variables. Lagrange’s multiplier method is useful in finding extreme values
of constrained functions. Any function of several variables can be expanded in power
series using Taylor’s theorem with the help of partial derivatives.

8.2 TANGENT PLANE AND NORMAL TO A SURFACE


Let P be any point on the surface y
f (x, y, z) = 0 or z = f (x, y) and let Q be
any other point on it. Any curve is taken
on the surface joining Q to P. As Q Tangent line
T
tends to P along this curve, the chord
PQ tends, in general, to a definite P Part of surface
Q
straight line. This straight line is called
a tangent line to the surface at P. Since
different curves can be obtained by
joining Q to P, different tangent lines O x
are obtained at P. All these tangent
lines lie in a plane called the tangent
z
plane to the surface at P.
The equation of the tangent plane at Fig. 8.1
P(x0, y0, z0) to the surface f (x, y, z) = 0 is
(x − x0) fx(x0, y0, z0) + (y − y0) fy (x0, y0, z0) + (z − z0) fz (x0, y0, z0) = 0
8.2 Chapter 8 Applications of Partial Derivatives

∂f
where f x ( x0 , y0 , z0 ) =
∂x ( x0 , y0 , z0 )
∂f
f y ( x0 , y0 , z0 ) =
∂y ( x0 , y0 , z0 )

∂f
f z ( x0 , y0 , z0 ) =
∂z ( x0 , y0 , z0 )

The line through P, perpendicular to the tangent plane is called the normal to the
surface at P. The equations of the normal line to the surface at P(x0, y0, z0) are

x − x0 y − y0 z − z0
= =
f x ( x0 , y0 , z0 ) f y ( x0 , y0 , z0 ) f z ( x0 , y0 , z0 )

Example 1
Find the equations of the tangent plane and normal line to the surface
xyz = 6 at (1, 2, 3). [Winter 2014]
Solution
Let f (xyz) = xyz − 6
f x ( x, y, z ) = yz,
yz, f x (1, 2, 3) = 6
f y ( x, y, z ) = xz,
xz, f y (1, 2, 3) = 3
fz ( x, y, z ) = xxyy, fz (1, 2, 3) = 2
Hence, the equation of the tangent plane at (1, 2, 3) is
6( x - 1) + 3( y - 2) + 2( z - 3) = 0
6 x - 6 + 3y - 6 + 2z - 6 = 0
6 x + 3 y + 2 z = 118
The set of equations of the normal line is
x -1 y - 2 z - 3
= =
6 3 2

Example 2
Find the equations of the tangent plane and normal line to the surface
x2 + y2 + z – 9 = 0 at the point (1, 2, 4). [Winter 2016]
Solution
Let f(x, y, z) = x2 + y2 + z – 9
8.2 Tangent Plane and Normal to a Surface 8.3

fx(x, y, z) = 2x, fx(1, 2, 4) = 2


fy(x, y, z) = 2y, fy(1, 2, 4) = 4
fz(x, y, z) = 1, fz(1, 2, 4) = 1
Hence, the equation of the tangent plane at (1, 2, 4) is
2(x – 1) + 4(y – 2) + 1(z – 4) = 0
2x – 2 + 4y – 8 + z – 4 = 0
2x + 4y + z – 14 = 0
2x + 4y + z = 14
The set of equations of the normal line is
x -1 y - 2 z - 4
= =
2 4 1

Example 3
Find the equations of the tangent plane and normal line to the surface
x2 + y2 + z2 = 3 at the point (1, 1, 1).
Solution
Let f (x, y, z) = x2 + y2 + z2 − 3
f x ( x, y, z ) = 2 x, f x (1, 1, 1) = 2
f y ( x, y, z ) = 2 y, f y (1, 1, 1) = 2
fz ( x, y, z ) = 2 z, 1 =2
fz (1, 1, 1)
Hence, the equation of the tangent plane at (1, 1, 1) is
2( x − 1) + 2( y − 1) + 2( z − 1) = 0
2x − 2 + 2 y − 2 + 2z − 2 = 0
2x + 2 y + 2z = 6
x+ y+ z = 3
The set of equations of the normal line is
x −1 y −1 z −1
= =
2 2 2
x −1 = y −1 = z −1

Example 4
Find the equations of the tangent plane and normal line to the surface
2x2 + y + 2z = 3 at (2, 1, – 3). [Summer 2017]
Solution
Let f(x, y, z) = 2x2 + y2 + 2z – 3
8.4 Chapter 8 Applications of Partial Derivatives

fx(x, y, z) = 4x, fx(2, 1, –3) = 8


fy(x, y, z) = 2y, fy(2, 1, –3) = 2
fz(x, y, z) = 2, fz(2, 1, –3) = 2
Hence, the equation of the tangent plane at (2, 1, –3) is
8(x – 2) + 2(y – 1) + 2(z + 3) = 0
8x – 16 + 2y – 2 + 2z + 6 = 0
8x + 2y + 2z – 12 = 0
8x + 2y + 2z = 12
4x + y + z = 6
The set of equations of the normal line is
x - 2 y -1 z + 3
= =
8 2 2
x-2
= y -1= z + 3
4

Example 5
Find the equations of the tangent plane and normal line at the point
x2 z2
(−2, 1, −3) to the ellipsoid + y + = 3.
2

4 9
[Winter 2013; Summer 2016]
Solution
x2 z2
Let f ( x, y , z ) = + y2 + − 3
4 9
x
f x ( x, y , z ) = , f x (-2, 1, - 3) = -1
2
f y ( x, y, z ) = 2 yy,, f y (-2, 1, - 3) = 2
2z 2
f z ( x, y , z ) = , f z (-2, 1, - 3) = -
9 3
Hence, the equation of the tangent plane at (−2, 1, −3) is
2
−1
−1( x + 2) + 2( y − 1) − ( z + 3) = 0
3
3( x + 2) − 6
6(( y − 1) + 2
2(( z + 3) = 0
3x + 6 − 6 y + 6 + 2 z + 6 = 0
3 x − 6 y + 2z
2 = −18
The set of equations of the normal line is
x + 2 y −1 z + 3
= =
−1 2 −2
3
8.2 Tangent Plane and Normal to a Surface 8.5

Example 6
Find the equations of the tangent plane and normal line to the surface
z = 2 x 2 + y 2 at the point (1, 1, 3).
Solution
Let f ( x, y , z ) = 2 x 2 + y 2 − z
f x ( x, y, z ) = 4 xx,, f x (1, 1, 3) = 4
f y ( x, y, z ) = 2 yy,, f y (1, 1, 3) = 2
f z ( x, y , z ) = -
-1,
-1
11,, f z (1, 1, 3) = -1
Hence, the equation of the tangent plane at (1, 1, 3) is
4( x − 1) + 2( y − 1) − 1( z − 3) = 0
4x − 4 + 2 y − 2 − z + 3 = 0
4x + 2 y − z = 3

The set of equations of the normal line is


x −1 y −1 z − 3
= =
4 2 −1

Example 7
Find the equations of the tangent plane and normal line to the surface
x 2 yyzz + 3 y 2 = 2 xz 2 − 8 z at the point (1, 2, −1).

Solution
Let f ( x, y, z ) = x 2 yyzz + 3 y 2 − 2 xz 2 + 8 z

f x ( x, y, z ) = 2 xyz − 2 z 2 , f x (1, 2, − 1) = 2(1


(1)(2)( −1) − 2( −1) 2 = −6
f y ( x, y , z ) = x 2 z + 6 y , f y (1, 2, − 1) = (1) 2 ( −1) + 6(2) = 11
f z ( x, y, z ) = x 2 y − 4 xz + 8, f z (1, 2, − 1) = (1) 2 (2) − 4(1)(
)( −1) + 8 = 114
Hence, the equation of the tangent plane at (1, 2, −1) is
−6( x − 1) + 11( y − 2) + 14( z + 1) = 0
−6 x + 6 + 11 y − 22 + 14 z + +14 = 0
6x − 1
111 y − 14 z + 2 = 0

The set of equations of the normal line is


x −1 y − 2 z +1
= =
−6 11 14
8.6 Chapter 8 Applications of Partial Derivatives

Example 8
Find the equations of the tangent plane and normal line to the surface
2xz2 – 3xy – 4x = 7 at the point (1, –1, 2). [Summer 2015]
Solution
Let f(x, y, z) = 2xz2 – 3xy – 4x = 7
f x ( x, y, z ) = 2 z 2 - 3 y - 4, 2))2 - 3( -1) - 4 = 7
f x (1, - 1, 2) = (2
f y ( x, y, z ) = -3 x, f y (1, - 1, 2) = -3(1) = - 3
fz ( x, y, z ) = 4 xz,
xz, fz (1, - 1, 2) = 4(1)(2
2)) = 8

Hence, the equation of the tangent plane at (1, –1, 2) is


7( x - 1) + ( -3)( y + 1) + 8( z - 2) = 0
7 x - 7 - 3 y - 3 + 8 z - 16
16 = 0
7 x - 3 y + 8 z = 226

The set of equations of the normal line is


x -1 y +1 z - 2
= =
7 -3 8

Example 9
Find the equation of the tangent plane and normal line to the surface
z + 8 = xxee y cos z at the point (8, 0, 0).
Solution
Let f ( x, y, z ) = xe y cos
cos z − z − 8
f x ( x, y, z ) = e y cos z f x (8, 0, 0) = 1
f y ( x, y, z ) = xe y cos
cos z f y (8, 0, 0) = 8
f z ( x, y, zz)) = sin z − 1 0, 0) = −1
f z (8, 0,
Hence, the equation of the tangent plane at (8,0,0) is
1( x − 8) + 8( y − 0) − 1( z − 0) = 0
x − 8 + 8y − z = 0
x + 8y − z − 8 = 0
The set of equations of the normal line is
x −8 y −0 z −0
= =
1 8 −1
y
x − 8 = = −z
8
8.2 Tangent Plane and Normal to a Surface 8.7

Example 10
Find the equations of the tangent plane and normal line to the surface
cos px – x2y + exz + yz = 4 at the point P(0, 1, 2). [Winter 2015]
Solution
Let f(x, y, z) = cos px – x2y + exz + yz = 4
sin p x - 2 xxyy + zzee xz ,
f x ( x, y, z ) = -p sin f x (0, 1, 2) = 2
2
f y ( x, y, z ) = - x + z, f y (0, 1, 2) = 2
fz ( x, y, z ) = xe xz + y, fz (0, 1, 2) = 1
Hence, the equation of the tangent plane at P(0, 1, 2) is
2( x - 0) + 2( y - 1) + 1( z - 2) = 0
2x + 2y - 2 + z - 2 = 0
2x + 2y + z = - 4
The set of equations of the normal line is
x y -1 z - 2
= =
2 2 1

Example 11
Show that the plane tangent to the surface z = x 2 + y 2 at the point (a, b, c)
intersects the z-axis at a point where z = −c.
Solution
Let f ( x, y , z ) = x 2 + y 2 − z
f x ( x, y , z ) = 2 x, f x (a, b, cc)) = 2a
f y ( x, y , z ) = 2 y , f y (a, b, cc)) = 2b
f z ( x, y, z ) = -1, f z (a, b, c) = -1
Hence, the equation of the tangent plane at (a, b, c) is
2a ( x − a ) + 2b( y − b) − 1( z − c) = 0
2ax − 2a
2a 2 + 2b
byy − 2b 2 − z + c = 0
2ax + 2by
2by − z − 2a 2 − 2b 2 + c = 0 …(1)
The point of intersection of tangent plane with the z-axis is obtained by putting x = 0,
y = 0 in Eq. (1),
− z − 2a 2 − 2b 2 + c = 0
z = c − 2a 2 − 2b 2
= c − 2( a 2 + b 2 ) …(2)
8.8 Chapter 8 Applications of Partial Derivatives

Since the point (a, b, c) lies on the surface x + y = z ,


2 2
…(3)
a 2 + b2 = c
Substituting Eq. (3) in Eq. (2),
z = c − 2c = − c

Example 12
Show that the plane 4 x − 6 y − z + 14 = 0 touches the surface
x 2 + 3 y 2 + 2 z = 0 and find the point of contact.
Solution
Let f ( x, y , z ) = x 2 + 3 y 2 + 2 z
f x ( x, y , z ) = 2 x
f y ( x, y , z ) = 6 y
f z ( x, y , z ) = 2
Let P( x0 , y0 , z0 ) be a point on the surface
f x ( x0 , y0 , z0 ) = 2 x0
f y ( x0 , y0 , z0 ) = 6 y0
f z ( x0 , y0 , z0 ) = 2
Hence, the equation of the tangent plane at (x0, y0, z0) is
2 x0 ( x − x0 ) + 6 y0 ( y − y0 ) + 2( z − z0 ) = 0
2 xx0 − 2
2xx02 + 6 yyyy0 − 6 y02 + 2 z − 2 z0 = 0
xx0 + 3 yyyy0 + z + z0 = 0
xx
Comparing with the plane equation 4x − 6y − z + 14 = 0

x0 3 y0 1 z
= = = 0
4 −6 −1 14
\ x0 = − 4, y0 = 2, z0 = −14
The point (− 4, 2, −14) satisfies the equation of the surface and the tangent plane.
Hence, the point of contact is (−4, 2, −14).

Example 13
Show that the surfaces z = xy − 2 and x 2 + y 2 + z 2 = 3 have the same
tangent plane at (1, 1, −1). [Summer 2014]
Solution
For the surface f (x, y, z) = xy – z −2,
8.2 Tangent Plane and Normal to a Surface 8.9

f x ( x, y, z ) = yy,, f x (1, 1, - 1) = 1
f y ( x, y, z ) = xx,, f y (1, 1, - 1) = 1
f z ( x, y , z ) = -
-1, f z (1, 1, - 1) = -1
Hence, the equation of the tangent plane is
1( x − 1) + 1( y − 1) − 1( z + 1) = 0
x −1+ y −1− z −1 = 0
x+ y − z −3= 0

For the surface f ( x, y, z ) = x 2 + y 2 + z 2 - 3,


f x ( x, y , z ) = 2
2xx 1,, - 1) = 2
f x (1, 1
f y ( x, y , z ) = 2 y 1,, - 1) = 2
f y (1, 1
f z ( x, y , z ) = 2
2zz f z (1, 1, - 1) = -2
Hence, the equation of the tangent plane is
2( x − 1) + 2( y − 1) − 2( z + 1) = 0
2x − 2 + 2 y − 2 − 2z − 2 = 0
x+ y − z −3= 0

Hence, the surfaces z = xxyy − 2 and x 2 + y 2 + z 2 = 3 have the same tangent plane at
(1, 1, −1).

Example 14
Find equations of the normal line of the sphere x2 + y2 + z2 = 6 at the
point ( a, b, c). Show that the normal line passes through the origin.
Solution
Let f(x, y, z) = x2 + y2 + z2 − 6
f x ( x, y , z ) = 2
2xxx,, f x (a, b, c) = 2a
f y ( x, y, z ) = 2 yy,, f y (a, b, c) = 2b
y z) = 2
f z ( x, y, 2zz , f z (a, b, c) = 2c
Hence, the set of equations of the normal line at (a, b, c) is
x−a y −b z −c
= =
2a 2b 2c
At the origin, x = 0, y = 0, z = 0.
0−a 0−b 0−c 1
= = =−
2a 2b 2c 2
The point (0,0,0) satisfies the equation of the normal line.
Hence, the normal line passes through the origin (0, 0, 0).
8.10 Chapter 8 Applications of Partial Derivatives

8.3 LINEAR APPROXIMATION OR LINEARIZATION


The tangent plane to a surface at a point stays close to the surface near the point.
If the function f (x, y) is differentiable at the point (x0, y0) the tangent plane to the
surface z = f (x, y) at the point (x0, y0) provides a good approximation to f (x, y) near
(x0, y0).
The equation of the tangent plane to the surface z = f (x, y) at the point (x0, y0) is
z = (x )( y y0 )
z z 0 ,y f (x 0 )( y y0 )
x f x x x ,y − 0 )
[∵ z y ]
The surface is close to the tangent plane near the point (x0, y0).
f x, y f (x )+ x x−x )
This is called as linear approximation or linearization of f (x, y) near the point (x0, y0).
This concept can be extended to a function of more than two variables.

x, y z )  L x, y z ) = x z ,y )( x x )
+f x , 0 − f (x ,z z 0 )
where f (x, y, z) is differentiable at the point (x0, y0, z0).

Example 1
x2 y2
Find the linearization of f x, y ) + at (−4, 3).
16 9
Solution
2 2
y
16 9
2x x 1

2 2
=
3
Hence, the linearization of f (x, y) is
1 2
4 3)

Example 2
Find the linearization of f(x, y) = xexy at the point (1, 0) and use it to
approximate f (1.1, –0.1).
8.3 Linear Approximation or Linearization 8.11

Solution
f ( x, y ) = xxee xy f (1, 0) = 1.e0 = 1
f x ( x, y ) = xye xy + e xy f x (1, 0) = (1)(
)(0)e0 + e0 = 1
f y ( x, y ) = x 2 e xy f y (1, 0) = (1) 2 e0 = 1

Hence, the linearization of f (x, y) is


L(x, y) = 1 + (1)(x −1) + (1)(y) = x + y
Using this linearization to approximate f (1.1, –0.1)
f (1.1, –0.1) = 1.1 – 0.1
=1

Example 3
Find the linearization of f(x, y, z) = x2 –xy + 3 sin z at the point (2, 1, 0).
Solution
f ( x, y, z ) = x 2 − xxyy + 3 sin
sin z f (2, 1, 0) = (2) 2 − (2)(11)) + 3 sin 0
= 4−2+0
=2
f x ( x y, z ) = 2 x − y 1, 0) = 22((2) − 1
f x (2, 1,
=3
f y ( x, yy.. z ) = − x f y (2, 1, 0) = −2
f z ( x, y, z ) = 3 cos
cos z f z (2, 1, 0) = 3 cos
cos 0° = 3

Hence, the linearization of f (x, y, z) is


L(x, y, z) = 2 + 3(x −2) + (−2)(y −1) + 3(z −0)
= 2 + 3x −6 −2y + 2 + 3z
= 3x −2y + 3z −2

Example 4
Find the linearization of f(x, y, z) = xy + yz + xz at (1, 0, 0).
Solution
f ( x, y, z ) = xy + yz + xz f (1, 0, 0) = 0
f x ( x, y , z ) = y + z f x (1, 0, 0) = 0
f y ( x, y , z ) = x + z f y (1, 0, 0) = 1
f z ( x, y , z ) = y + x f z (1, 0, 0) = 1
8.12 Chapter 8 Applications of Partial Derivatives

Hence, the linearization of f (x, y, z) is


L(x, y, z) = 0 + 0(x − 1) + 1(y − 0) + 1(z − 0)
=y+z
EXERCISE 8.1
1. Find the equations of the tangent plane and the normal line to the
following surfaces at indicated points.
(a) z = 4 − x 2 − 2y 2 , (1, − 1, 1)
 2 2 1
(b) z = 1 − x − y
2 2
,  3 , 3 , 3 

 1
(c) z = 1 −
10
(
1 2
x + 4y 2 ) ,  1, 1, 2 

(d) x 2 y 2 + xz − 2y 3 = 10 , (2,
2, 1, 4)
4

 x −1 y +1 z −1 
 Ans. : (a) x − 2y + z − 4 = 0 ,
1
=
−2
=
1 
 
 3x − 2 3y − 2 3z − 1
(b) 2 x + 2y + z − 3 = 0 , = =
 2 2 1 
 
 1 
z−
 x −1 y −1 2 
(c) 2 x + 8y + 10 z − 15 = 0 , = =
 2 8 10 
 
 x − 2 y −1 z − 4 
(d) 4 x + y + z = 1 3=0
13 , = =
 4 1 1 

2. Show that the tangent plane to the surface x2 = y(x + z) at any point
passes through the origin.
3. Show that the plane 3x + 12y – 6z – 17 = 0 touches the surface 3x2 – 6y2 +
9z2 + 17 = 0. Find also the point of contact.
  2 
 Ans. :  −1, 2,  
 3 
4. Show that the plane ax + by + cz + d = 0 touches the surface
a 2 b2
px 2 + 9y 2 + 2 z = 0, if + + 2cd = 0
p q
Ê pˆ
5. Find the linearization of f(x, y) = excosy at Á 0, ˜
Ë 2¯
È p˘
ÍÎ Ans. : - y + 3 ˙˚

6. Find the linearization of f (x, y , z) = x 2 + y 2 + z 2 at (1, 0, 0).


[Ans.: x]
8.4 Errors and Approximations 8.13

8.4 ERRORS AND APPROXIMATIONS

Let u = f ((x, y) be a continuous function of x and y. If d x and d y are small increments


in x and y respectively and d u is corresponding increment in u then
u + d u = f (x + d x, y + d y)
d u = f ((x + d x, y + d y) − f (x, y)
∂∂ff ∂f
∂f
= δ x + δ y [Expanding by Taylor’s theorem and ignoring higher powers
∂x ∂y
∂y
and products of d x and d y.]
∂u ∂u
or δu = δx+ δy
∂x ∂y
For a function u = f ((x, y, z) of three variables,
∂u ∂u ∂u
δu = δx+ δ y + δz
∂x ∂y ∂z
If d x is the error in x then
(i) d x is known as absolute error in x.
δx
(ii) is known as relative error in x.
x
δx
(iii) × 100 is known as percentage error in x.
x

Example 1
Find the percentage error in calculating the area of a rectangle when an
error of 3% is made in measuring each of its sides.
Solution
Let a and b be the sides of the rectangle and A be its area.
A = ab
Taking logarithm on both sides,
log A = log a + log b
1 1 1
δ A = δ a + δb
A a b
δA δa δb
× 100 = ×100
× 100 + × 100
A a b
δa δb
Putting × 100 = 3, × 100 = 3,
a b
δA
× 100 = 3 + 3
A
=6
Hence, percentage error in calculating area = 6%.
8.14 Chapter 8 Applications of Partial Derivatives

Example 2
Find the percentage error in the area of an ellipse when errors of 2%
and 3% are made in measuring its major and minor axes respectively.

Solution
Let 2a and 2b be the major and minor axes of the ellipse and A be its area.
A = p ab
Taking logarithm on both sides,
log π + log
logg A = log log a + log b
1 1 1
δ A = 0 + δ a + δb
A a b
δA δa δb
× 100 =
×100 × 100 +
×100 × 100
A a b

δa δb
Putting × 100 = 2,
×100 × 100 = 3,
a b
δA
× 100 = 2 + 3
×100
A
=5
Hence, percentage error in area of ellipse = 5%.

Example 3
2 1 1
The focal length of mirror is found from the formula = − . Find
f v u
the percentage error in f if u and v are both in error by 2% each.
Solution
2 1 1
= −
f v u
2 1 1
− 2
δ f = − 2 δv + 2 δu
f v u
2δf 1 δv 1 δu
− × 100 = − ×100
× 100 + × 100
f f v v u u

du dv
Putting ¥ 100 = 2, ¥ 100 = 2,
u v
8.4 Errors and Approximations 8.15

2δf 1 1
− × 100 = − (2) + (2)
f f v u
 1 1  2
= −2  −  = −2 
 v u  f 
δf
× 100 = 2
f
Hence, percentage error in f = 2%.

Example 4
Find the possible percentage error in computing the parallel resistance
R of two resistances R1 and R2 if R1, R2 are each in error by 2%.
Solution
1 1 1
= +
R R1 R2
1 1 1
− δ R = − 2 δ R1 − 2 δ R2
R2 R1 R2
1 δR 1 δ R1 1 δ R2
× 100 =
×100 × 100 +
×100 × 100
R R R1 R1 R2 R2
δ R1 δR
Putting × 100 = 2, 2 × 100 = 2,
R1 R2
1 δR 1 1
× 100 = (2) + ( 2)
R R R1 R2

 1 1
= 2 + 
 R1 R2 
 1
= 2 
 R
δR
∴ × 100 = 2
R
Hence, percentage error in R = 2%

Example 5
The time of swing t of a pendulum of length l under certain conditions is
2
l  r 
given by t = 2π , where g ′ = g   ⋅ Find the percentage error
g′ r + h
in t due to error p% in h and q% in l where r is a constant.
8.16 Chapter 8 Applications of Partial Derivatives

Solution
l
t = 2π
g′

l
t 2 = 4π 2
g′
( r + h) 2   r  
2

= 4π 2 l ∵ g ′ = g  
gr 2   r + h  
4π 2
= ll(r + h) 2
gr 2
Taking logarithm on both sides,
 4π 2 
2 logg t = log  2  + log
log l + 2 llog(
og(r + h)
 gr 
1 1 2h 1
2 δt = 0 + δl + δh
t l r+h h
δt δl  δh  h
2 × 100 = × 100 + 2  × 100
t l  h  r+h
δl δh
× 100 = q,
×100 × 100 = p
×100
l h
δt ph
Putting 2 ×100
× 100 = q + 2 ,
t r+h
δt q ph
× 100 = +
×100
t 2 r+h
q ph
Hence, percentage error in t = +
2 r+h

Example 6
1
The area of DABC is calculated using the formula ∆ = ab sinn C. Errors
2
of 2%, 3%, 4% are made in measuring a, b, C respectively. If the correct
value of C is 30°, find percentage error in the calculated value of D.
Solution
1
∆= ab sin C
2
8.4 Errors and Approximations 8.17

Taking logarithm on both sides,


1
logg ∆ = llog + log
log a + llog
og b + log sin C
2
1 1 1 cos C
cos
δ∆ = 0 + δ a + δ b + δC
∆ a b sin C
δ∆ δa δb
× 100 = × 100 + × 100 + cot C δ C × 100
∆ a b
δa δb δC
Putting × 100 = 2, × 100 = 3, × 100 = 4δ C × 100 = 4C ,
a b C
δ∆
× 100 = 2 + 3 + cott C (4C )

 π π
= 5 +  cot   4 × 
 6   6
 2π 
= 5 +173
+  
3
= 8.6
62
Hence, percentage error in the calculated value of D = 8.62%

Example 7
In calculating the volume of a right circular cone, errors of 2% and 1%
are made in height and radius of base respectively. Find the percentage
error in calculating the volume.
Solution
Let r and h be the radius of base and height of the right circular cone and V be its
volume.
1
V = π r 2h
3
Taking logarithm on both sides,
π
logg V = log
log + 2 log log r + log h
3
1 2 1
δV = 0 + δ r + δ h
V r h
δV δr δh
× 100 = 2 ×100
×100 × 100 + × 1100
00
V r h
δr δh
Putting × 100 = 1,
×100 × 100 = 2,
×100
r h
δV
× 100 = 2(1) + 2
×100
V
=4
Hence, percentage error in volume = 4%
8.18 Chapter 8 Applications of Partial Derivatives

Example 8
In calculating the volume of a right circular cylinder, errors of 2% and
1% are found in measuring height and base radius respectively. Find the
percentage error in calculated volume of the cylinder.
Solution
Let r and h be the base radius and height of the right circular cylinder and V be its
volume.
V = p r2 h
Taking logarithm on both sides,
log π + 2 log
logg V = log log r + log h
1 2 1
δV = 0 + δ r + δ h
V r h
δV δr δh
× 100 = 2 ×100
×100 × 100 + × 10
00
V r h
δr δh
Putting × 100 = 1,
×100 × 100 = 2,
×100
r h
δV
× 100 = 2(1) + 2
×100
V
=4
Hence, percentage error in volume = 4%.

Example 9
The diameter and height of a right circular cylinder are measured to be 5
inches and 8 inches respectively. If each of these dimensions were in error
by 0.1 inch, find the percentage error in the volume of the cylinder.
Solution
Let d and h are diameter and height of the cylinder respectively and V be its volume.
2
d 1
V = π   h = π d 2h
 2 4
Taking logarithm on both sides,
π
logg V = log
log + 2 log log d + log h
4
1 2 1
δV = 0 + δ d + δ h
V d h
1 2 1
δV = δ d + δ h
V d h
δV δd δh
× 100 = 2 × 100 + × 100
V d h
8.4 Errors and Approximations 8.19

Putting d = 5 in., h = 8 in., d d = 0.1in, d h = 0.1 in.,


δV 0.1 0.1
× 100 = 2 × × 100 + × 100
V 5 8
= 5.2
25
Hence, percentage error in volume = 5.25%.

Example 10 1

Find the approximate value of [(0.98)2 + (2.01)2 + (1.94)2] 2 .


Solution
Let u = x2 + y 2 + z 2
u2 = x2 + y2 + z2
2ud u = 2xd x + 2yd y + 2zd z
ud u = xd x + yd y + zd z
Putting x = 1, y = 2, z = 2,
d x = 0.98 − 1 = − 0.02,
d y = 2.01 − 2 = 0.01,
d z = 1.94 − 2 = − 0.06
u = 12 + 22 + 22
=3
and uδ u = 1( −0.02) + 2(0.01) + 2( −0.06)
= − 0.1
12
δ u = − 0.004

Approximate value = u + d u
= 3 − 0.04
= 2.96

EXERCISE 8.2
1. In calculating the volume of right circular cone, errors of 2.75% and
1.25% are made in height and radius of the base. Find the percentage
error in volume.
[Ans.: 5.25%]
2. The height of a cone is H = 30 cm, the radius of base R = 10 cm. How
will the volume of the cone change if H is increasing by 3 mm while R is
decreasing by 1 mm?
[Hint: d h = 3 mm = 0.3 cm, d r = −1 mm = −0.1 cm]
[Ans.: decreased by 10p cm3]
8.20 Chapter 8 Applications of Partial Derivatives

3. Find the percentage error in calculating the area of a rectangle when an


error of 2% is made in measuring each of its sides.
[Ans.: 4%]
4. If R1 and R2 are two resistances in parallel, their resistance R is given by
1 1 1
= +
R R1 R2
If there is an error of 2% in both R1 and R2, find percentage error in R.
[Ans.: 2%]

E
5. The resistance R of circuit was found by using the formula I = . If there
R
is an error of 0.1 ampere in reading I and 0.5 volts in reading E, find the
corresponding percentage error in R when I = 15 amperes and E = 100 volts.
[Ans.: − 0.167%]
6. The voltage V across a resistor is measured with error h, and the resistance

R is measured with an error R. Show that the error in calculating the power
V2 V
W= is (2Rh − VR). If V can be measured to an accuracy of 0.5% and to
R R2
an accuracy of 1%, what is the approximate possible percentage error in W?
[Ans.: 0%]
7. At a distance of 20 metres from the foot of a tower, the elevation of its
top is 60°. If the possible error in measuring distance and elevation are 1
cm and 1 minute, find the approximate error in calculating height.
[Ans.: 0.040]
8. The diameter and the altitude of a right circular cylinder are measured
as 24 cm and 30 cm respectively. There is an error of 0.1 cm in each
measurement. Find the possible error in the volume of the cylinder.

1
[[Ans.: 50.4p cm]
9. If f = (160 − x − y ) , find the approximate value of f (2.1, 2.9) − f (2, 3)
3 3 3

[Ans.: 0.016]
xyz
10. If f = e , find the approximate value of f when x = 0.01, y = 1.01, z = 2.01.
1
[Ans.:
[ 1.02]
11. Find [(2.92)3 + (5.87
.87)3 ]5 approximately by using the theory of
approximation.
[Ans.: 2.96]
8.5 Maximum and Minimum Values by Second Derivative Test 8.21

8.5 MAXIMUM AND MINIMUM VALUES BY


SECOND DERIVATIVE TEST
Let u = f (x, y) be a continuous function of x and y. u will be maximum at x = a,
y = b, if f (a, b) > f (a + h, b + k) and will be minimum at x = a, y = b,
if f (a, b) < f (a + h, b + k) for small positive or negative values of h and k.
The point at which function f (x, y) is either maximum or minimum is known as
stationary point. The value of the function at stationary point is known as extreme
(maximum or minimum) value of the function f (x, y).
Working Rule to determine the maxima and minima (extreme values)
of a function f (x, y)
∂f
∂f ∂f
∂f
Step I Solve = 0 and = 0 simultaneously for x and y.
∂x ∂y
∂y
∂2 f ∂2 f ∂2 f
Step II Obtain the values of r = ,s= ,t = 2 .
∂x 2
∂x ∂y ∂y
Step III (i) If rt – s2 > 0 and r < 0 (or t < 0) at (a, b) then f (x, y) is maximum at (a, b)
and the maximum value of the function is f (a, b).
(ii) If rt – s2 > 0 and r > 0 (or t > 0) at (a, b) then f (x, y) is minimum at (a, b) and
the minimum value of the function is f (a, b).
(iii) If rt – s2 < 0 at (a, b) then f (x, y) is neither maximum nor minimum at
(a, b). Such a point is known as saddle point.
(iv) If rt – s2 = 0 at (a, b) then no conclusion can be made about the extreme
values of f (x, y) and further investigation is required.

Example 1
Discuss the maxima and minima of the function x 2 + y 2 + 6 x + 12.
Solution
Let f (x, y) = x2 + y2 + 6x + 12
Step I For extreme values,
∂f
∂f
=0
∂x
2x + 6 = 0
2( x + 3) = 0
x+3= 0
x = −3
∂∂ff
and =0
∂y
∂y
2y = 0
y=0
8.22 Chapter 8 Applications of Partial Derivatives

Stationary point is (– 3, 0).


∂2 f
Step II r= =2
∂x 2
∂2 f
s= =0
∂x ∂y
∂2 f
t= =2
∂y 2
Step III At (–3, 0)
rt – s2 = 2(2) – 0 = 4 > 0 and r > 0
Hence, f (x, y) is minimum at (–3, 0).
fmin = (– 3)2 + 0 + 6 (– 3) + 12 = 3

Example 2
a3 a3
Show that the minimum value of f ( x, yy)) = xy
xy + + is 3a2.
x y
Solution
a3 a3
f ( x, yy)) = xy
xy + +
x y
Step I For extreme values,
∂∂ff
=0
∂x
a3
y− 2 =0
x
x 2 y = a3 …(1)
∂∂ff
and =0
∂y
∂y
a3
x− =0
y2
xy2 = a3 …(2)
Solving Eqs (1) and (2),
x=y
Substituting in Eq. (1),
x3 = a3
x=a
\ y=a
Stationary point is (a, a).
8.5 Maximum and Minimum Values by Second Derivative Test 8.23

Step II
∂ 2 f 2a 3
r= = 3
∂x 2 x
∂ f
2
s= =1
∂x ∂y
∂ 2 f 2a 3
t= = 3
∂y 2 y
Step III At (a, a), r = 2, s = 1, t =2
rt – s2 = (2) (2) – (1)2 = 3 > 0
Also, r=2>0
Hence, f (x, y) is minimum at (a, a).
 1 1
f min = a 2 + a 3  +  = 3a 2 .
 a a

Example 3
Discuss the maxima and minima of the function 3x2 – y2 + x3.

Solution
Let f ( x, y ) = 3 x 2 − y 2 + x 3
Step I For extreme values, ∂∂ff
=0
∂x
6 x + 3x 2 = 0
3 x ( x + 2) = 0
x = 0, −2
∂∂ff
and =0
∂y
∂y
−2 y = 0
y=0
Stationary points are (0, 0), (–2, 0).

∂2 f
Step II r= = 6 + 6x
∂x 2
∂2 f
s= =0
∂x ∂y
∂2 f
t= = −2
∂y 2
8.24 Chapter 8 Applications of Partial Derivatives

Step III
(x,y) r s t rt – s2 Conclusion
(0, 0) 6 0 –2 –12 < 0 neither maximum nor minimum
(–2, 0) –6 0 –2 12 > 0 and r < 0 maximum

Hence, f (x, y) is maximum at (–2, 0)


fmax = 3(–2)2 – 0 + (–2)3 = 4

Example 4
Find the stationary value of x3 + y3 – 3axy, a > 0.
Solution
Let f (x, y) = x3 + y3 − 3axy
Step I For extreme values,
∂∂ff
=0
∂x
3 x 2 − 3aayy = 0
x2 − a
ayy = 0 …(1)
∂f
∂f
and =0
∂y
∂y
3 y 2 − 3ax
ax = 0
y 2 − ax
ax = 0 …(2)
From Eq. (1),
x2
y=
a
Substituting in Eq. (2),
x 4 − a3 x = 0
x( x − a )( x 2 + ax + a 2 ) = 0
x = 0, x = a
\ y = 0, y = a.
Hence, stationary points are (0, 0) and (a, a).

Step II ∂2 f
r= = 6x
∂x 2
∂2 f
s= = −3a
∂x ∂y
∂2 f
t= = 6y
∂y 2
8.5 Maximum and Minimum Values by Second Derivative Test 8.25

Step III At (0, 0), r = 0, s = –3a, t = 0


rt – s2 = (0) (0) – (–3a)2 = –9a2 < 0
Hence, f (x, y) is neither maximum nor minimum at (0, 0).
At (a, a), r = 6a, s = –3a, t = 6a
rt – s2 = (6a) (6a) – (–3a)2 = 27a2 > 0
Also, r = 6a > 0
Hence, f (x, y) is minimum at (a, a).
fmin = a3 + a3 – 3a3 = – a3.

Example 5
Find the extreme values of the function x3 + 3xy2 – 3x2 – 3y2 + 4, if any.
[Summer 2017]
Solution
Let f (x, y) = x3 + 3xy2 − 3x2 − 3y2 + 4
Step I For extreme values,
∂∂ff
=0
∂x
3x 2 + 3 y 2 − 6 x = 0
x2 + y 2 − 2x = 0 …(1)
∂∂ff
and =0
∂y
∂y
6 xy − 6 y = 0
6 y ( x − 1) = 0
y = 0, x = 1
Putting y = 0 in Eq. (1),
x2 – 2x = 0,
x = 0, 2
Stationary points are (0, 0), (2, 0).
Putting x = 1 in Eq. (1),
1 + y2 – 2 = 0,
y2 = 1, y = ± 1
Stationary points are (1, 1), (1, –1).
Step II
∂2 f
r= = 6 x − 6 = 6( x − 1)
∂x 2
∂2 f
s= = 6y
∂x ∂y
∂2 f
t= = 6 x − 6 = 6( x − 1)
∂y 2
8.26 Chapter 8 Applications of Partial Derivatives

Step III
(x, y) r s t rt – s2 Conclusion
(0, 0) –6 < 0 0 –6 36 > 0 and r < 0 maximum

(2, 0) 6>0 0 6 36 > 0 and r > 0 minimum


(1, 1) 0 6 0 –36 < 0 neither maximum nor
minimum
(1, –1) 0 –6 0 –36 < 0 neither maximum nor
minimum

Hence, f (x, y) is maximum at (0, 0) and minimum at (2, 0).


fmax = 0 + 4 = 4
and fmin = 23 + 3(2)(0)2 – 3(2)2 – 3(0)2 + 4
= 8 + 0 – 12 + 4
=0

Example 6
Find the extreme values of the function x3 + y3 – 63 (x + y) + 12xy.
Solution
Let f (x, y) = x3 + y3 − 63x − 63y + 12xy

Step I For extreme values,


∂∂ff
=0
∂x
3 x 2 − 63 + 12 y = 0,
3 x 2 + 12 y = 63
x 2 + 4 y = 21 …(1)
∂∂ff
and =0
∂y
∂y
3 y 2 − 63 + 12 x = 0,
12 x + 3 y 2 = 63
4 x + y 2 = 21 …(2)
Equating Eqs (1) and (2),
x2 + 4 y = 4x + y 2
x 2 − y 2 − 44(( x − y ) = 0
( x + y )( x − y ) − 44(( x − y ) = 0
( x − y )( x + y − 4) = 0
x − y = 0,0, x + y − 4 = 0
y = x, y = 4 − x
8.5 Maximum and Minimum Values by Second Derivative Test 8.27

Putting y = x in Eq. (1),


x2 + 4x − 2211 = 0,
( x + 7)( x − 3) = 0
x = −7, 3
∴ y = −7, 3
Stationary points are (–7, –7), (3, 3).
Putting y = 4 – x in Eq. (1),
x 2 + 4(4 − x) = 21
x 2 − 4 x − 5 = 0,
( x + 1)( x − 5) = 0
x = −1, 5
∴ y = 5, −1
Stationary points are (–1, 5), (5, –1).
Step II
∂2 f
r= = 6x
∂x 2
∂2 f
s= = 12
∂x ∂y
∂2 f
t= = 6y
∂y 2
Step III
(x, y) r s t rt – s2 Conclusion
(–7, –7) –42 12 – 42 1620 > 0 and r < 0 maximum
(3, 3) 18 12 18 180 > 0 and r > 0 minimum
(–1, 5) –6 12 30 –324 < 0 neither maximum nor
minimum
(5, –1) 30 12 –6 –324 < 0 neither maximum nor
minimum

Hence, f (x, y) is maximum at (–7, –7).


max = ( −7) + ( −7) − 63( −7 − 7) + 12( −7
− )( −7) = 784.
3 3
f ma (− 7)(
and f (x, y) is minimum at (3, 3).

min = 3 + 3 − 63(
(3 + 3) + 12(3)(3) = −216.
3 3
f mi

Example 7
Find the extreme value of xy (a – x – y).
8.28 Chapter 8 Applications of Partial Derivatives

Solution
Let f (x, y) = xy (a − x − y)
= axy – x2y – xy2
Step I For extreme values,
∂∂ff
=0
∂x
ay − 2 xy − y 2 = 0 …(1)
y (a − 2 x − y ) = 0
y = 0, a − 2 x − y = 0

∂∂ff
and =0
∂y
∂y
…(2)
ax − x 2 − 2 xxyy = 0
x(a − x − 2 y ) = 0
x = 0, a − x − 2 y = 0
Considering four pairs of equations of Eqs (1) and (2),
y=0 x=0
y=0 a − x − 2y = 0
a − 2x − y = 0 x=0
a − 2x − y = 0 a − x − 2y = 0
Solving these equations, the following pairs of values of stationary points are
 a a
), (0, a ),  , 
obtained: (0, 0), (a, 00),
 3 3

Step II
∂2 f
r= = −2 y
∂x 2
∂2 f
s= = a − 2x − 2 y
∂x ∂y
∂2 f
t= = −2 x
∂y 2

Step III
(x, y) r s t rt – s2 Conclusion
2
(0, 0) 0 a 0 –a < 0 neither maximum nor minimum
2
(a, 0) 0 –a –2a –a < 0 neither maximum nor minimum
(0, a) –2a –a 0 –a2 < 0 neither maximum nor minimum

a a −2a −a −2a a2
 ,  >0 maximum or minimum
3 3 3 3 3 3
8.5 Maximum and Minimum Values by Second Derivative Test 8.29

 a a
Hence, f (x, y) is maximum or minimum at  ,  depending on whether a > 0 or a < 0.
 3 3
a a a a  a3
extreme =
f ex ⋅ a − −  = .
3 3 3 3  27

Example 8
Examine the function x3 y2(12 – 3x – 4y) for extreme values.
Solution
Let f (x, y) = 12x3y2 − 3x4y2 − 4x3y3
Step I For extreme values,
∂∂ff
=0
∂x
36 x 2 y 2 −12
− 12 x 3 y 2 − 12 x 2 y 3 = 0
12 x 2 y 2 (3 − x − y ) = 0
x = 0, y = 0, x + y = 3 …(1)

∂∂ff
and =0
∂y
∂y
24 x 3 y − 6 x 4 y − 12 x 3 y 2 = 0
6 x3 y (4 − x − 2 y ) = 0
x = 0, y = 0, x + 2 y = 4 …(2)
Considering six pairs of equations of Eqs (1) and (2),
x=0 y=0
x=0 x + 2y = 4
y=0 x + 2y = 4
x+ y = 3 x=0
x+ y = 3 y=0
x+ y = 3 x + 2y = 4
Solving these equations, the following pairs of stationary points are obtained:
(0, 0), (0, 2), (4, 0), (0, 3), (3, 0), (2, 1)

Step II
∂2 f
r= = 72 xy 2 − 36 x 2 y 2 − 24 xy 3 = 12 xy 2 (6 − 3 x − 2 y )
∂x 2
∂2 f
s= = 72 x 2 y − 2
244 x 3 y − 36x
36 x 2 y 2 = 12 x 2 y (6 − 2x
2 x − 3 y)
∂x ∂y
∂2 f
t= = 24x
24 x 3 − 6 x 4 − 24
4xx 3 y = 6 x 3 (4 − x − 4 y )
24
∂y 2
8.30 Chapter 8 Applications of Partial Derivatives

Step III
(x, y) r s t rt – s2 Conclusion
(0, 0) 0 0 0 0 no conclusion
(0, 2) 0 0 0 0 no conclusion
(4, 0) 0 0 0 0 no conclusion
(0, 3) 0 0 0 0 no conclusion
(3, 0) 0 0 162 0 no conclusion
(2, 1) – 48 – 48 –96 2304 > 0 and r < 0 maximum
Hence, f (x, y) is maximum at (2, 1).
max = ( 2 )( )((12 − 6 − 4) = 16.
3
f ma )(12 )(

Example 9
Find the maxima and minima of x 4 + y 4 − 2 x 2 + 4 xxyy − 2 y 2 .
Solution
Let f ( x, yy)) = x 4 + y 4 − 2 x 2 + 4 xy − 2 y 2
Step I For extreme values,
∂∂ff
=0
∂x
4 x3 − 4 x + 4 y = 0
44(( x 3 − x + y ) = 0
x3 − x + y = 0 …(1)
∂f
∂f
and =0
∂y
∂y
4 y3 + 4 x − 4 y = 0
4( y 3 + x − y ) = 0
y3 + x − y = 0 …(2)
Adding Eqs (1) and (2),
x3 + y 3 = 0
y = −x
Substituting y = − x in Eq. (2),
− x3 + x + x = 0
x3 − 2 x = 0
x ( x 2 − 2) = 0
x = 0, ± 2
y = 0, ∓ 2
8.5 Maximum and Minimum Values by Second Derivative Test 8.31

Stationary points are (0, 0), ( 22,, − 2 ) , ( − 22,, 2 ) .

Step II
∂2 f
r= = 12 x 2 − 4
∂x 2
∂2 f
s= =4
∂x ∂y
∂2 f
t= = 12 y 2 − 4
∂y 2
Step III

(x, y) r s t rt – s2 Conclusion
(0, 0) –4 4 –4 0 no conclusion

( 2, − 2) 20 4 20 384 > 0 and r > 0 minimum

((− 2 , 2 ) 20 4 20 384 > 0 and r > 0 minimum

Hence, f (x, y) is minimum at the point ( 2 , − 2 ) and ( − 22,, 2 ).

At ( 2 , − 2 ), f mi
min = ( 2 ) + ( − 2 ) − 2
4 4
2(( 2 ) 2 + 4 2 ((−
− 2 ) − 2((−
− 2 ) 2 = −8

Example 10
Find the extreme values of the function

f ( x, y ) = x 3 + y 3 − 3 x − 12 y + 20. [Summer 2014]


Solution
f ( x, yy)) = x 3 + y 3 − 3 x − 12 y + 20
Step I For extreme values,

∂∂ff
=0
∂x
3x 2 − 3 = 0
3(( x 2 − 1) = 0
3
x2 − 1 = 0
x2 = 1
x = ±1
8.32 Chapter 8 Applications of Partial Derivatives

∂∂ff
and =0
∂y
∂y
3 y 2 − 12 = 0
33(( y 2 − 4) = 0
y2 − 4 = 0
y2 = 4
y=±2
Stationary points are (1, 2), (1, –2), (–1, 2), (–1, –2).

Step II
∂2 f
r= = 6x
∂x 2
∂2 f
s= =0
∂x ∂y
∂2 f
t= = 6y
∂y 2
Step III
(x, y) r s t rt – s2 Conclusion
(1, 2) 6 0 12 72 > 0 and r > 0 minimum
(1, –2) 6 0 –12 –72 < 0 neither maximum nor
minimum
(–1, 2) –6 0 12 –72 < 0 neither maximum nor
minimum
(–1, –2) –6 0 –12 72 > 0 and r < 0 maximum

Hence, f (x, y) is maximum at (–1, –2) and minimum at (1, 2).


fmax = ( −1)3 + ((−
−2)3 − 3 −1) − 12( −2) + 20 = 38
3(((−
fmin = (1)3 + (2)3 − 3(
3(1) − 12(2) + 20 = 2

Example 11
Find the extreme values of the function x 3 + xxyy 2 + 21x − 12 x 2 − 2 y 2 .
Solution
Let f ( x, yy)) = x 3 + xy 2 + 21x − 12 x 2 − 2 y 2
Step I For extreme values,
∂∂ff
= 0,
∂x
8.5 Maximum and Minimum Values by Second Derivative Test 8.33

3x 2 + y 2 − 2
244 x + 21
21 = 0 …(1)
∂∂ff
and =0
∂y
∂y
2 xy − 4 y = 0
2 y ( x − 2) = 0
y = 0, x = 2

Putting y = 0 in Eq. (1),


3 x 2 − 24
24 x + 21 = 0, x 2 − 8 x + 7 = 0
21
( x − 1)( x − 7) = 0
x = 1, 7
Stationary points are (1, 0), (7, 0).
Putting x = 2 in Eq. (1),
12 + y 2 − 48 + 21 = 0
y 2 = 15, y = ± 15

Stationary points are 2, 15 (


2,, − 15 .
15 , 2 )( )
15 , 22,, − 15 .
Hence, all stationary points are (1, 0), (7, 0), 2, 15 ( )( )
∂2 f
Step II r= = 6 x - 24 = 6( x - 4)
∂x 2
∂2 f
s= = 2y
∂x ∂y
∂2 f
t= = 2 x - 4 = 2( x - 2)
∂y 2
Step III
(x, y) r s t rt – s2 Conclusion
(1, 0) –18 0 –2 36 > 0 and r < 0 maximum
(7, 0) 18 0 10 180 > 0 and r > 0 minimum

(2,2, )
neither maximum nor
15 –12 2 15
1 0 –60 < 0
minimum

(2, - )
neither maximum nor
1155 –12 −2
2 15
1 0 –60 < 0
minimum
Hence, f (x, y) is maximum at (1, 0) and minimum at (7, 0).
max = 1 + (1 × 0 ) + 21 − (12 × 1 ) − (
(22 × 02 ) = 10,
3 2 2
f ma
min = 7 + (7 × 0 ) + ( 21 × 7) − (
(12 × 7 2 ) − (2 × 02 ) = −98
3 2
f mi
8.34 Chapter 8 Applications of Partial Derivatives

Example 12
Find all the stationary points of the function
x3 + 3 xxyy 2 − 15 x 2 − 15 y 2 + 72 x
after examining whether the function is maximum or minimum at those
points. [Summer 2016]
Solution
Let f ( x, yy)) = x 3 + 3 xy 2 − 15 x 2 − 15 y 2 + 72 x

Step I For extreme values,


∂∂ff
=0
∂x

3 x 2 + 3 y 2 − 30 x + 72 = 0 …(1)

∂∂ff
=0
∂y
∂y
6xy – 30y = 0 …(2)
6y (x – 5) = 0
y = 0, x = 5
Putting y = 0 in Eq. (1),
3x2 – 30x + 72 = 0
x = 4, x = 6
Stationary points are (4, 0), (6, 0).
Putting x = 5 in Eq. (1),
3y2 – 3 = 0
y=±1
Stationary points are (5, 1) and (5, –1).
Hence all stationary points are (5, 1) (5, –1), (4, 0) and (6, 0).

Step II

∂2 f
r= = 6 x − 30
∂x 2
∂2 f
s= = 6y
∂x ∂y
∂2 f
t= = 6 x − 30
∂y 2
8.5 Maximum and Minimum Values by Second Derivative Test 8.35

Step III
(x, y) r s t rt – s2 Conclusion
(4, 0) –6 0 –6 36 > 0 and r < 0 maximum
(6, 0) 6 0 6 36 > 0 and r > 0 minimum
(5, 1) 0 6 0 –36 < 0 neither maximum
nor minimum
(5, –1) 0 –6 0 – 36 < 0 neither maximum
nor minimum

Hence, f (x, y) is maximum at (4, 0) and f (x, y) is minimum at (6, 0).


fmax = (4)3 + 0 – 15 (4)2 – 0 + 72 (4) = 112
fmin = (6)3 + 0 – 15 (6)2 – 0 + 72 (6) = 108

Example 13
Find the extreme values of sin x + sin y + sin(x + y).
Solution
Let f (x, y) = sin x + sin y + sin(x + y)

Step I For extreme values,


∂∂ff
=0
∂x
coss x + cos( x + y ) = 0 …(1)
∂f
∂f
and =0
∂y
∂y
coss y + cos( x + y ) = 0 …(2)
Equating Eqs (1) and (2),
coss x + cos
os( x + y ) = cos
ccos(
os( cos y + cos(((xx + y )
coss x = cos
cos y
x= y
Substituting y = x in Eq. (1),
coss x + cos
cos 2 x = 0,
coss x = − ccos
os 2 x
= cos(π − 2 x) or ccos
os(π + 2 x)
os(
x = π − 2 x or π + 2 x
π
x= , −π
3
π
y = , −π
3
8.36 Chapter 8 Applications of Partial Derivatives

π π
Stationary points are  ,  , ( −π , − π ) .
 3 3
Step II
∂2 f
r= = − sin
n x − sin( x + y )
∂x 2
∂2 f
s= = − sin( x + y )
∂x∂y
∂2 f
t= = − sin
n y − siin
n( x + y )
∂y 2
Step III

(x, y) r s t rt – s2 Conclusion

π π − 3 3 − 3
9
> 0 and r < 0 maximum
 ,  −
3 3 2 4
(– p, – p) 0 0 0 0 no conclusion

π π
Hence, f (x, y) is maximum at  ,  .
 3 3

p p 2p 3 3 3 3 3
max = sin
fma sin + sin
in + sin
sin = + + =
3 3 3 2 2 2 2

Example 14
Find the extreme values of sin x sin y sin (x + y).
Solution
Let f (x, y) = sin x sin y sin (x + y)
Step I For extreme values,

∂∂ff
=0
∂x
sinn y [cos x sin(( x + y ) + sinn x cos( x + y )] = 0
sinn y sin(2 x + y ) = 0
1
os 22xx − cos( 2 x + 2 y )] = 0
[cos
2
coss 22xx − cosos(2 x + 2 y ) = 0
ccos(
os( ...(1)
∂∂ff
and =0
∂y
∂y
n x [cos y sin(( x + y ) + sin
sin n y cos( x + y )] = 0
8.5 Maximum and Minimum Values by Second Derivative Test 8.37

sin in(( x + 2 y ) = 0
n x sin
ssin(
1
[cos 2 y − cos(2 x + 2 y )] = 0
2
coss 2 y − cos
os(2 x + 2 y ) = 0
cos( ...(2)
Equating Eqs (1) and (2),
coss 2 x = ccos
os 2 y
x= y

Substituting x = y in Eq. (1),


coss 22xx − cos
os(2 x + 2 x) = 0
ccos(
os(
coss 2 x = cos
cos 4 x = 2 cos
cos 2 2 x − 1
cos 2 2 x − cos
2 cos cos 2 x − 1 = 0
1± 1+ 8
cos 2 x =
4
1
= 1, −
2
1 2π
coss 2 x = 1 = ccos
os 0, cos 2 x = −
cos = cos
2 3
π
x = 0, x=
3
π
y = 0, y=
3
π π
Stationary points are (0, 0),  ,  .
 3 3

Step II
∂2 f
r= = − sin
n22xx + sin 2( x + y ) = 2 sin y cos(2 x + y )
2si
∂x 2
∂2 f
s= = sin 2( x + y )
∂x ∂y
∂2 f
t= = − sin
n 2 y + sin
sin 2( x + y ) = 2
2si os((xx + 2 y )
sin x ccos
sin cos(
os(
∂y 2
Step III
(x, y) r s t rt – s2 Conclusion
(0, 0) 0 0 0 0 no conclusion

π π − 3 3 − 3
9
> 0 and r < 0 maximum
 ,  −
3 3 2 4

π π
Hence, f (x, y) is maximum at  ,  .
 3 3
8.38 Chapter 8 Applications of Partial Derivatives

p p 2p 3 3 3 3 3
max = sin
fma sin ssin
in sin
sin = ◊ ◊ =
3 3 3 2 2 2 8

8.6 MAXIMUM AND MINIMUM VALUES WITH


CONSTRAINED VARIABLES
Sometimes we have to find the extreme values of a function of three (or more)
variables, say f (x, y, z), which are not independent but are connected by some given
relation φ ( x, y, z ) = 0. The extreme values of f (x, y, z) in such a situation are called
constrained extreme values.
In such situations, we use φ ( x, y, z ) = 0 to eliminate one of the variables, say z,
from the given function, thus converting the function as a function of only two
variables and then find the extreme values of the function.

Example 1
Find the minimum value of x2 + y2 + z2 with the constraint x + y + z = 3a .
Solution
Let f = x2 + y2 + z2
x + y + z = 3a
z = 3a − x − y …(1)
Substituting the value of z in Eq. (1),
f = x2 + y2 + (3a − x − y)2
Step I For extreme values,
∂∂ff
=0
∂x
2 x − 2(3a − x − y ) = 0
4 x − 6a + 2 y = 0
2x + y = 33a …(2)

and
∂∂ff
=0
∂y
∂y
2 y − 2(3a − x − y ) = 0
2 y − 6a + 2 x + 2 y = 0
x + 2 y = 3a …(3)
Solving Eqs (2) and (3),
x=y=a
The stationary point is (a, a).
8.6 Maximum and Minimum Values with Constrained Variables 8.39

Step II
∂2 f
r= =4
∂x 2
∂2 f
s= =2
∂x∂y
∂2 f
t= =4
∂y 2
Step III At (a, a), r = 4, s = 2, t = 4
rt − s2 = (4)(4) − (2)2 = 12 > 0
Also, r = 4 > 0
Hence, f (x, y) is minimum at (a, a)
fmin = a2 + a2 + (3a − a − a)2
= 3a2

Example 2
Find the minimum value of x3y2z subject to the condition x + y + z = 1.
Solution

Let f = x3y2z
x+y+z=1
z=1−x−y ...(1)
Substituting the value of z in Eq. (1),
f = x3y2(1 − x − y)
= x3y2 − x4y2 − x3y3
Step I For extreme values,
∂∂ff
=0
∂x
3x2y2 − 4x3y2 − 3x2y3 = 0
x2y2(3 – 4x − 3y) = 0
x = 0, y = 0, 4x + 3y = 3 …(2)
∂∂ff
and =0
∂y
∂y
2x3y − 2x4y − 3x3y2 = 0
x3y(2 − 2x −3y) = 0
x = 0, y = 0, 2x + 3y = 2 …(3)
8.40 Chapter 8 Applications of Partial Derivatives

Considering six pairs of Eqs (2) and (3),


x=0 y=0
x=0 2x +3y = 2
y=0 2x +3y = 2
4x + 3y = 3 x=0
4x + 3y = 3 y=0
4x + 3y = 3 2x + 3y = 2
Solving these equations, the following pairs of stationary points are formed:

 2  3   1 1
(0, 0),  0,  , (1, 0), (0, 1),  , 0 ,  , 
 3  4   2 3
Step II
∂2 f
r= = 6 xy 2 − 12 x 2 y 2 − 6 xy 3 = 6 xy 2 (1 − 2 x − y )
6xy
∂x 2
∂2 f
s= = 6 x 2 y − 8 x3 y − 9
9xx 2 y 2 = x 2 y (6 − 8
8xx − 9 y )
∂x∂y
∂2 f
t= =2
2xx 3 − 2 x 4 − 6
6xx 3 y = 2 x 3 (1
1−− x − 3 y)
∂y 2
Step III

(x, y) r s t rt – s2 Conclusion
(0, 0) 0 0 0 0 no conclusion

 2
 0,  0 0 0 0 no conclusion
3
(1, 0) 0 0 0 0 no conclusion
(0, 1) 0 0 0 0 no conclusion

3  0 0 27 0 no conclusion
 , 0 128
4
 1 1 1 1 1 1 maximum
 ,  − − − > 0 and r < 0
2 3 9 12 8 144

 1 1
Hence, f (x, y) is maximum at  ,  .
 2 3
3 2
 1   1  1 1
max =      1 −
f ma −
 2   3   2 3 
1
=
432
8.6 Maximum and Minimum Values with Constrained Variables 8.41

Example 3
Divide 120 into three parts so that the sum of their products taken two
at a time shall be maximum.
Solution
Step I Let x, y, z be three numbers.
Let x + y + z = 120
f = xy
xy + yyzz + zzxx
= xy + y (120 − x − y ) + (120 − x − y ) x
= xy + 120 y − xxyy − y 2 + 120 x − x 2 − xy
= 120 x +120
+ 120 y − xy − x 2 − y 2
For extreme values,
∂∂ff
=0
∂x
120 − y − 2 x = 0 ...(1)
∂∂ff
and =0
∂y
∂y
120 − x − 2 y = 0 ...(2)
Solving Eqs (1) and (2),
x = 40
y = 40
Stationary point is (40, 40).
Step II
∂2 f
r= = −2
∂x 2
∂2 f
s= = −1
∂x ∂y
∂2 f
t= = −2
∂y 2
Step III At (40, 40).
rt – s2 = (–2) (–2) – (–1)2 = 3 > 0
and r=–2<0
f (x, y) is maximum at (40, 40).
Hence, three parts are 40, 40 and 40.

Example 4
Find a point on the plane 2x + 3y – z = 5 which is nearest to the origin.
[Summer 2017]
8.42 Chapter 8 Applications of Partial Derivatives

Solution
Let P(x, y, z) be any point on the plane 2x + 3y – z = 5.
Its distance from the origin is given by
d = x 2 + y2 + z2
d2 = x2 + y2 + z2
Since P lies on the plane z = 2x + 3y – 5,
d2 = x2 + y2 + (2x + 3y – 5)2
Let f(x, y) = x2 + y2 + 4x2 + 9y2 + 25 + 12xy – 30y – 20x
= 5x2 + 10y2 + 12xy – 30y – 20x + 25
Step I For extreme values,
∂f
=0
∂x
10x + 12y = 20
5x + 6y = 10 ...(1)
∂f
and =0
∂y
20y + 12x = 30
10y + 6x = 15 ...(2)
Solving Eqs (1) and (2),
5 15
x= ,y=
7 14
Ê 5 15 ˆ
Stationary point is Á , ˜ .
Ë 7 14 ¯
∂2 f
Step II r= = 10
∂x 2
∂2 f
s= =6
∂x ∂y
∂2 f
t= = 10
∂y 2
Ê 5 15 ˆ
Step III At Á , ˜ , r = 10, s = 6, t = 10
Ë 7 14 ¯
rt – s2 = 100 – 36 = 64 > 0
Also, r = 10 > 0
Ê 5 15 ˆ Ê 5 15 ˆ
f(x, y) i.e. d2 is minimum at Á , ˜ and hence d is minimum at Á , ˜ .
Ë 7 14 ¯ Ë 7 14 ¯

Ê 5 15 ˆ
At ÁË 7 , 14 ˜¯ ,
Ê 5ˆ Ê 15 ˆ 5
z = 2 x + 3y - 5 = 2 Á ˜ + 3 Á ˜ - 5 = -
Ë 7¯ Ë 14 ¯ 14
8.6 Maximum and Minimum Values with Constrained Variables 8.43

Ê 5 15 5ˆ
Hence, d is minimum at Á , , - ˜ .
Ë 7 14 14 ¯

The point ÊÁ 5 , 15 , - 5 ˆ˜ on the plane 2x + 3y – z = 5 is nearest to the origin.


Ë 7 14 14 ¯

Example 5
Find the points on the surface z2 = xy + 1 nearest to the origin. Also find
that distance.
Solution
Let P (x, y, z) be any point on the surface z2 = xy + 1.
Its distance from the origin is given by

d = ( x2 + y 2 + z 2 )
d 2 = x2 + y 2 + z 2

Since P lies on the surface z2 = xy + 1,

d 2 = x 2 + y 2 + xy
xy + 1
Let f ( x, y ) = x 2 + y 2 + xxyy + 1
Step I For extreme values,
∂∂ff
=0
∂x
2x + y = 0 …(1)
∂f
∂f
and =0
∂y
∂y
2y + x = 0 …(2)
Solving Eqs (1) and (2),
x = 0, y = 0
Stationary point is (0, 0).
Step II
∂2 f
r= =2
∂x 2
∂2 f
s= =1
∂x ∂y
∂2 f
t= =2
∂y 2
Step III At (0, 0), r = 2, t = 2, s = 1
rt – s2 = (2) (2) – (1)2 = 3 > 0
8.44 Chapter 8 Applications of Partial Derivatives

Also, r=2>0
2
f (x, y), i.e., d is minimum at (0, 0) and hence d is minimum at (0, 0).
At (0, 0),
z 2 = xy
xy + 1 = 1
z = ±1
Hence, d is minimum at (0, 0, 1) and (0, 0, –1).
The points (0, 0, 1) and (0, 0, –1) on the surface z2 = xy + 1 are nearest to the origin.
Minimum m distance = 0 + 0 + 1 = 1.

Example 6
A rectangular box open at the top is to have a volume of 108 cubic metres.
Find the dimensions of the box if its total surface area is minimum.
Solution
Let x, y and z be the dimensions of the box. Let V and S be its volume and surface area
respectively.
V = xyz
S = xxyy + 2 xxzz + 2 yz
yz
V
Substituting z = ,
xy
V V
S = xxyy + 2 x ⋅ + 2 y ⋅
xy xy
2V 2V
= xy + +
y x
Step I For extreme values,
∂S
=0
∂x
2V
y− 2 =0 …(1)
x
∂S
and =0
∂y
∂y
2V
x− =0 …(2)
y2
2V
Substituting y = from Eq. (1) in Eq. (2),
x2
 x4 
x − 2V  2  = 0
 4V 
 x3 
x 1 − =0
 2V 
8.6 Maximum and Minimum Values with Constrained Variables 8.45

1
x = ((2
2V ) 3
1
2V 2V
∴ y= 2
= 2
= (2V ) 3 [Since x ≠ 0 being the side of the box]
x
( V )3
(2
 1 1

Stationary point is (2V ) 3 , (2V ) 3  .
Step II ∂ 2 S 4V
r= = 3
∂x 2 x
∂2 S
s= =1
∂x ∂y
∂ 2 S 4V
t= = 3
∂y 2 y

 1 1
 4V 4V
V ) 3 , (2V ) 3  , r =
Step III At (22V = 2 > 0, s = 1, t = =2
2V 2V
rt – s 2 = (2)(2) – (1)2 = 3 > 0 and r = 2 > 0
1
Hence, S is minimum at x = y = (2V ) 3 .
Putting V = 108 m3,
1
x = y = ((2 ×
×108
108) 3 = 6
V 108
and z= = =3
xy 6 × 6
Hence, dimensions of the box which make its total surface area S minimum are x = 6,
y = 6, z = 3.

Example 7
Show that the rectangular solid of maximum volume that can be inscribed
in a given sphere is a cube.
Solution
Let x, y, z be the length, breadth and height of the rectangular solid and V be its
volume.
V = xyz …(1)
Let the given sphere be
x2 + y 2 + z 2 = a2
z 2 = a2 − x2 − y 2
8.46 Chapter 8 Applications of Partial Derivatives

Substituting in Eq. (1),

V = xxyy a 2 − x 2 − y 2
V 2 = x 2 y 2 (a 2 − x 2 − y 2 )
Let f ( x, y ) = V 2 = x 2 y 2 ( a 2 − x 2 − y 2 ) …(2)

Step I For extreme values,


∂∂ff
=0
∂x
y 2 [ 2 x(a 2 − x 2 − y 2 ) + x 2 ( −2 x)] = 0
xy 2 (a 2 − 2 x 2 − y 2 ) = 0
2xy
2
x = 0, y = 0, 2 x 2 + y 2 = a 2 ...(3)
∂f
∂f
and =0
∂y
∂y
x 2 [ 2 y (a 2 − x 2 − y 2 ) + y 2 ( −2 y )] = 0
2 x 2 y (a 2 − x 2 − 2 y 2 ) = 0
x = 0, y = 0, x 2 + 2 y 2 = a 2 …(4)
But x and y are the sides of the rectangular solid, and therefore cannot be zero.
Solving 22xx 2 + y 2 = a 2 and x 2 + 2 y 2 = a 2 ,

a2 2 a2
x2 = ,y =
3 3
a a
x= ,y= [∵side cannot be negative]
3 3
a2 a2 a
z = a2 − − =
3 3 3
 a a a 
Stationary points are  , , .
 3 3 3 
Step II
∂2 f
r= = 2 a 2 y 2 − 12 x 2 y 2 − 2 y 4
∂x 2
∂2 f
s= = 4a 2 xxyy − 8 x 3 y − 8 xy
x 3
∂x ∂y
∂2 f
t= =2
2aa 2 x 2 − 2 x 4 −12
− 12 x 2 y 2
∂y 2
 a a  2a 4 4a 4 2a 4 8a 4
Step III At  , , r = − − =−
 3 3 3 3 9 9
8.6 Maximum and Minimum Values with Constrained Variables 8.47

4a 4 8a 4 8a 4 4a 4
s= − − =−
3 9 9 9
2a 4 2a 4 12a 4 8a 4
t= − − =−
3 9 9 9
8 8 8
64 a 16
1 6 a 48 a
rt − s 2 = − = >0
81 81 81
rt – s2 > 0 and r < 0
2
f (x, y), i.e. V is maximum at x = y = z and hence, V is maximum when
x = y = z, i.e. the rectangular solid is a cube.

EXERCISE 8.3
1. Examine maxima and minima of the following functions and find their
extreme values:
(i) 2 + 2x + 2y − x2 − y 2 (ii) x 2y 2 − 5x 2 − 8xy − 5y 2
(iii) x 2 + y 2 + xy + x − 4y + 5 (iv) x 2 + y 2 + 6x = 12
(v) x 3y 2 (1 − x − y) (vi) xy (3a − x − y)
(vii) x 3 + 3xy 2 − 3x 2 − 3y 2 + 4 (viii) x4 + y4 − 2 (x − y)2
(ix) x 4 + x2y + y2 (x) x 4 + y 4 − 4a2 xy
(xi) y 4 − x 4 + 2(x 2 − y 2) (xii) x 3 + 3x2 + y2 + 4xy
(xiii) x 2y − 3x 2 − 2y 2 − 4y + 3 (xiv) x4 − y4 − x2 − y 2 + 1.

 
 Ans.: (i) Max. at (1, 1); 4 (ii) Max.
Max. at
at (0, 0); 0 
 (iii) Min. at (−
( 2, 3);;−−2
−2 (iv) Min. at ( − 3, 0)
0); 3 
 
  1 1 1 
(v) Max. at  ,  ; (vi) Max. a att (a, a); a 3
  2 3  432 
 


(vii
vii) Max. at (0, 0); 4 (viii) Min. at (
at 2 , − 2 a )
and (
nd − 2, 2 ; −8  ) 
 (ix) Min. att ((0
0, 0); 0 (x) Min. att (a, a) and (−a a,, a)); a 4 
 
 (xi) No extrreme
eme values (xii
xii) No extreme values 
  1 
 1  1
(xiii) Max. att (0, − 1);5 xiv) Max x.. att((0, 0);1,min at  ± ,±  ;
  2 2  2 

2. A rectangular box, open at the top, is to have a volume of 32 cc. Find the
dimensions of the box requiring least materials for its construction.
[Ans. : 4, 4, 2]
8.48 Chapter 8 Applications of Partial Derivatives

3. Divide 120 into three parts so that the sum of their products taken two
at a time shall be maximum.
[Hint: f = xy + yz + zx where x + y + z = 120]
[Ans. : 40, 40, 40]
4. The sum of three positive numbers is ‘a’. Determine the maximum value
of their product.
 a3  a a a 
 Ans.: at  , ,  
 27  3 3 3 
5. Find the volume of the largest rectangular parallelepiped that can be
inscribed in an ellipsoid
x 2 y 2 z2
+ + = 1.
a 2 b2 c 2
Hint : Lett 2 x, 2y , 2 z b
be the sides of the parallelepiped; then ittss volume 
 
 x2 y2 
 v = 8 xyz
xyz = 8 xy 1 − 2 − 2 
a b
 8abc 
 Ans. : 
 3 3
6. Prove that area of a triangle with constant perimeter is maximum when
the triangle is equilateral.

Hint : Area = s(s − a)(s − b)(s − c) 


 
 where 22ss = a + b + c, c = 2s − a − b, s iiss constta
ant 

7. Find the shortest distance from the origin to the surface xyz2 = 2.
[Ans. : 2]
8. Find the shortest distance from the origin to the plane x − 2y − 2z = 3.
[Ans. : 1]
x −3 y −5 z −7
9. Find the shortest distance between the lines = = and
1 −2 1
x +1 y +1 z +1
= = .
7 −6 1
 Ans.: 2 2
29 
10. Find the maximum value of cos A cos B cos C, where A, B, C are angles
of a triangle.
È Ê p p p ˆ 1˘
Í Ans.: max. at ÁË , , ˜¯ ; ˙
Î 3 3 3 8˚
8.7 Method of Lagrangian Multipliers 8.49

8.7 METHOD OF LAGRANGIAN MULTIPLIERS


Let f (x, y, z) be a function of three variables x, y, z, and the variables be connected by
the relation
f (x, y, z) = 0 …(1)
Suppose we wish to find the values of x, y, z, for which f (x, y, z) is stationary (maxi-
mum and minimum).
For this purpose, we construct an auxiliary equation
f (x, y, z) + lf (x, y, z) = 0 …(2)
Differentiating Eq. (2) partially w.r.t. x, y, z,
∂∂ff ∂φ
+λ =0 …(3)
∂x ∂x
∂f
∂f ∂φ
+λ =0 …(4)
∂∂yy ∂y
∂∂ff ∂φ
+λ =0 …(5)
∂z ∂z
Eliminating l from Eqs (3), (4) and (5), the values of x, y, and z are obtained for which
f (x, y, z) has stationary value. This method of obtaining stationary values of f (x, y, z) is
called Lagrange’s method of undetermined multipliers, and equations (3), (4) and (5)
are called Lagrange’s equations. The term l is called undetermined multiplier.

Example 1
Find the minimum value of x2 + y 2, subject to the condition ax + by = c.
Solution
Let f (x, y) = x2 + y2 …(1)
ax + by = c …(2)
Let f (x, y) = ax + by – c = 0
Let the auxiliary equation be

( x 2 + y 2 ) + λ (ax + by − c) = 0 ...(3)

Differentiating Eq. (3) partially w.r.t x,

22xx + λ a = 0
2x
λ=− …(4)
a
Differentiating Eq. (3) partially w.r.t y,
2 y + λb = 0
2y
λ=− …(5)
b
8.50 Chapter 8 Applications of Partial Derivatives

From Eqs (4) and (5),


22xx 2 y
=
a b
b
y= x
a
Substituting y in Eq. (2),
b 
ax + b  x = c
a 
b2
ax + x=c
a
(a 2 + b 2 ) x = ac
ac
x=
a + b2 2

b  ac  bc
∴ y=  2 =
a  a + b2  a 2 + b2

2 2 a2c2 b2c2
Minimum value of x + y = +
(a 2 + b 2 )2 (a 2 + b 2 )2

Example 2
Find the minimum values of x2 yz3, subject to the condition 2x + y + 3z = a.
[Summer 2014]
Solution
Let f (x, y, z) = x2 yz3 …(1)
2 x + y + 3z = a …(2)
Let f (x, y, z) = 2x + y + 3z – a = 0
Let the auxiliary equation be
x 2 yyzz 3 + λ (2 x + y + 3 z − a ) = 0 ...(3)
Differentiating Eq. (3) partially w.r.t x,
xyz 3 + 2λ = 0
22xyz
λ = − xyz 3 …(4)
Differentiating Eq. (3) partially w.r.t y,
x2 z3 + λ = 0
λ = −x 2 z 3 …(5)
Differentiating Eq. (3) partially w.r.t z,
3 x 2 yz 2 + 3λ = 0
8.7 Method of Lagrangian Multipliers 8.51

λ = −x 2 yz
y 2 …(6)
From Eqs (4), (5), and (6),
xyz 3 = x 2 z 3 = x 2 yz
y 2
yz = xz = xy
∴ y = x and z = y
Substituting y = z = x in Eq. (2),

2 x + x + 3x
3x = a
6x = a
a
x=
6
a a
∴ y= ,z=
6 6
2 3 6
 a  a  a  a
Minimum value of x2 yz3 =       =  
 6  6  6  6

Example 3
Find the maximum value of f = x 2y 3z 4, subject to the condition x + y + z = 5.
Solution
Let f (x, y, z) = x 2y 3z 4 …(1)
x+y+z=5 …(2)
Let f (x, y, z) = x + y + z – 5 = 0
Let the auxiliary equation be

x2y3z4 + l (x + y + z – 5) = 0 ...(3)

Differentiating Eq. (3) partially w.r.t x,

xy 3 z 4 + λ = 0
2xy
2
λ = −2 xy 3 z 4 …(4)
Differentiating Eq. (3) partially w.r.t y,

3xx 2 y 2 z 4 + λ = 0
3
λ = −3 x 2 y 2 z 4 …(5)
Differentiating Eq. (3) partially w.r.t z,
4xx 2 y 3 z 3 + λ = 0
4
λ = − 4 x2 y3 z3 …(6)
8.52 Chapter 8 Applications of Partial Derivatives

From Eqs (4), (5), and (6),


xy 3 z 4 = 3 x 2 y 2 z 4 = 4 x 2 y 3 z 3
22xy
2 yz = 3xz
3 xz = 4 xy
3
∴ y= x and z = 2x
2
Substituting y and z in Eq. (2),
3
x+ x + 2x = 5
2
9 x = 10
10
x=
9
3 3  10  5
∴ y= x=   =
2 2 9  3
 10  20
and z = 2x = 2   =
 9 9
2 3 4
 10   5   20  (210 )(59 )
Maximum value of x 2 y 3 z 4 =       =
 9   3  9  315

Example 4
Find the maximum value of x my nz p when x + y + z = a.
Solution
Let f (x, y, z) = x m y n z p …(1)
x+y+z=a …(2)
Let f (x, y, z) = x + y + z – a = 0
Let the auxiliary equation be
x m y n z p + λ ( x + y + z − a) = 0 ...(3)
Differentiating Eq. (3) partially w.r.t x,
mx mm−1 y n z p + λ = 0
λ = − mx mm−1 y n z p …(4)
Differentiating Eq. (3) partially w.r.t y,
ny nn−1 x m z p + λ = 0
λ = −ny n −1 x m z p …(5)
Differentiating Eq. (3) partially w.r.t z,
px m y n z p −1 + λ = 0
8.7 Method of Lagrangian Multipliers 8.53

λ = − px m y n z p −1 …(6)
From Eqs (4), (5) and (6),
mx mm−−−11 y n z p = nx m y n −1 z p = px m y n z p −1
m n p
= =
x y z
n p
∴ y = x and z= x
m m
Substituting y and z in Eq. (2),
n p
x+ x+ x = a
m m
am
x=
m+n+ p

n an
∴ y= x=
m m+n+ p
p ap
and z= x=
m m+n+ p

a m+ n + p mm nn p p
Maximum value of x m y n z p =
(m + n + p) m + n + p

Example 5
Find the minimum value of x2 + y2 + z2 with the constraint xy + yz + zx = 3a2.
Solution
Let f (x, y, z) = x2 + y2 + z2 …(1)
2
xy + yz + zx = 3a …(2)
Let f (x, y, z) = xy + yz + zx – 3a 2

Let the auxiliary equation be


( x 2 + y 2 + z 2 ) + λ ( xy + yz + zx − 3a 2 ) = 0 ...(3)
Differentiating Eq. (3) partially w.r.t x,

2x + λ ( y + z) = 0
2x
l=- …(4)
y+z
Differentiating Eq. (3) partially w.r.t. y,

2y + l (x + z) = 0
8.54 Chapter 8 Applications of Partial Derivatives

2y
l=- …(5)
z+x
Differentiating Eq. (3) partially w.r.t. z,
2z + l (y + x) = 0
2z
l=- …(6)
x+y
From Eqs (4), (5) and (6),
2x 2y 2z 2 x + 2 y + 2z
= = = =1
y+z z+x x+y y+z+z+x+x+y
2x - y - z = 0
-x + 2y - z = 0
- x - y + 2z = 0
Solving these equations,
x=y=z
Substituting y = z = x in Eq. (2),
33xx 2 = 3a 2
x = ±a
x = y = z = ±a
Minimum value of x2 + y2 + z2 = 3a2

Example 6
Using Lagrange’ss method of multiplier
multipliers,
s, show that the stationary
1 1 1
value of a3x2 + b3y2 + c3z2 where + + = 1 occur at
x y z
a+b+c a+b+c a+b+c
x= ,y= ,z= .
a b c
Solution
Let f (x, y, z) = a3x2 + b3y2 + c3z2 …(1)
1 1 1
+ + =1 …(2)
x y z

1 1 1
Let φ ( x, y , z ) = + + −1 = 0
x y z
Let the auxiliary equation be
1 1 1 
(a x
3 2
+ b3 y 2 + c 3 z 2 ) + λ  + + − 1 = 0 ...(3)
x y z 
8.7 Method of Lagrangian Multipliers 8.55

Differentiating Eq. (3) partially w.r.t. x,


λ
22a 3 x − =0
x2
2 3 x3 − λ = 0
2a
λ = 2a 3 x 3 ...(4)
Differentiating Eq. (3) partially w.r.t. y,
λ
22bb3 y − =0
y2
22bb3 y 3 − λ = 0
λ = 2b3 y 3 …(5)
Differentiating Eq. (3) partially w.r.t. z,
λ
2c 3 z − =0
z2
2c 3 z 3 − λ = 0
λ = 2c 3 z 3 …(6)
From Eqs (3), (4) and (5),
22aa 3 x 3 = 2b
2b3 y 3 = 2c 3 z 3
ax = by = cz
ax ax
∴y= and z=
b c
Substituting y and z in Eq. (2),
1 b c
+ + =1
x ax ax
a+b+c
=1
ax
a+b+c
x=
a
ax a + b + c
∴y= =
b b
ax a + b + c
and z= =
c c
Hence, the stationary value of a3x2 + b3y2 + c3z2 occurs at
a+b+c a+b+c a+b+c
x= ,y= ,z=
a b c

Example 7
Find the point on the plane ax + by + cz = p at which the function
f = x 2 + y 2 + z 2 has a minimum value and find this minimum value of f.
[Summer 2015]
8.56 Chapter 8 Applications of Partial Derivatives

Solution
Let f (x, y, z) = x2 + y2 + z2 …(1)
ax + by + cz = p …(2)
Let f (x, y, z) = ax + by + cz – p = 0
Let the auxiliary equation be
(x2 + y2 + z2) + l (ax + by + cz – p) = 0 …(3)
Differentiating Eq. (3) partially w.r.t x,
22xx + λ a = 0
2x
λ=− ...(4)
a
Differentiating Eq. (3) partially w.r.t y,
2 y + λb = 0
2y ...(5)
λ=−
b
Differentiating Eq. (3) partially w.r.t z,
2z + λc = 0
2z ...(6)
λ=−
c
From Eqs (4), (5) and (6),
22xx 2 y 2 z
= =
a b c
bx cx
y= and z =
a a
Substituting y and z in Eq. (2),

b2 x c2 x
ax + + = p
a a
ap
x= 2
a + b2 + c2
bp
∴y= 2
a + b2 + c2
cp
and z= 2
a + b2 + c2
 ap bp cp 
Thus, (x2 + y2 + z2) is minimum at  2 , , 
a + b2 + c2 a 2 + b2 + c2 a 2 + b2 + c2 
a2 p2 b2 p 2 c2 p2
Minimum value of x + y + z = + +
2 2 2

(a 2 + b 2 + c 2 ) 2 (a 2 + b 2 + c 2 ) 2 (a 2 + b 2 + c 2 ) 2

p 2 (a 2 + b 2 + c 2 ) p2
= =
(a 2 + b 2 + c 2 ) 2 a 2 + b2 + c2
8.7 Method of Lagrangian Multipliers 8.57

Example 8
 25 
Find the length of the shortest line from the point  0, 0,  to the sur-
 9
face z = xy.
Solution
Let (x, y, z) be a point on the surface z = xy.
 25 
The distance d between (x, y, z) and  0, 0,  is
 9
2
 25 
d= x2 + y 2 +  z − 
 9
2
 25 
d 2 = x2 + y 2 +  z − 
 9
2
 25 
Let f ( x, y , z ) = d 2 = x 2 + y 2 +  z −  …(1)
 9
z = xy …(2)
Let f (x, y, z) = z – xy = 0

Let the auxiliary equation be


 2  25  
2

+ + −   + λ ( z − xy ) = 0
2
 x y  z ...(3)
 9 
Differentiating Eq. (3) partially w.r.t. x,

2 x + λ (− y) = 0
2x
λ= …(4)
y

Differentiating Eq. (3) partially w.r.t. y,


2 y + λ ( − x) = 0
2y
λ= …(5)
x
Differentiating Eq. (3) partially w.r.t. z,
 25 
2 z −  + λ = 0
 9
 25 
λ = −2  z − 
 9 ...(6)
8.58 Chapter 8 Applications of Partial Derivatives

From Eqs (4) and (5),


2x 2 y
=
y x
x2 = y 2
x = ±y …(6)

Substituting x = y, in Eq. (4),


l=2
Substituting l = 2 in Eq. (6),
 25 
2 = −2  z − 
 9
25
−1 = z −
9
25 16
z = −1 + =
9 9

16
Substituting y = x and z = in Eq. (2),
9
16
= x2
9
4
x=±
3
4
∴y=±
3
34
Similarly, when x = –y, z = . But this gives a complex value of x and y.
9
4 4 16
Thus f (x, y, z), i.e., d 2 is minimum when x = ± , y = ± , z = .
3 3 9
2 2 2
 4   4   16 25  41
Minimum distance d =   +  + −  =
 3  3  9 9 3
 25  41
Hence, the length of the shortest line from  0, 0,  to the surface z = xy is .
 9 3

Example 9
Show that the rectangular solid of maximum volume that can be inscribed
in a sphere is a cube. [Summer 2016]
Solution
Let 2x, 2y, 2z be the length, breadth and height of the rectangular solid.
8.7 Method of Lagrangian Multipliers 8.59

Let r be the radius of the sphere.


Volume of solid, V = 8xyz …(1)
Equation of the sphere, x + y + z2 = r2
2 2
…(2)
Let f (x, y, z) = x2 + y2 + z2 – r2 = 0
Let the auxiliary equation be
xyz + λ ( x 2 + y 2 + z 2 − r 2 ) = 0
88xyz ...(3)
Differentiating Eq. (3) partially w.r.t. x,
8 yz + λ ⋅ 2
2x = 0
4 yz
λ=− …(4)
x
Differentiating Eq. (3) partially w.r.t y,
8xz + λ ⋅ 2 y = 0
8
4 xz
λ=−
y …(5)
Differentiating Eq. (3) partially w.r.t z,
xy + λ ⋅ 2 z = 0
8xy
8
4 xy
λ=−
z …(6)

From Eqs (4), (5) and (6),


4 yz 4 xz 4 xy
= =
x y z
y 2 = x2 and z2 = y2
x2 = y 2 = z 2
x= y=z
Hence, the rectangular solid is a cube.

Example 10
Find the minimum and maximum distance from the point (1, 2, 2) to the
sphere x 2 + y 2 + z 2 = 36.
Solution
Let (x, y, z) be any point on the sphere. Its distance D from the point (1, 2, 2) is

D = ( x − 1) 2 + ( y − 2) 2 + ( z − 2) 2
Let D2 = f (x, y, z) = (x – 1)2 + (y – 2)2 + (z – 2)2 ...(1)
x2 + y2 + z2 = 36 ...(2)
Let f (x, y, z) = x 2 + y 2 + z 2 – 36
8.60 Chapter 8 Applications of Partial Derivatives

Let the auxiliary equation be


( x − 1)2 + ( y − 2)2 + ( z − 2)2  + λ ( x 2 + y 2 + z 2 − 36) = 0 ...(3)
Differentiating Eq. (3) partially w.r.t x,
2( x − 1) + λ (2 x) = 0
x −1 1
λ=− = −1 + …(4)
x x
Differentiating Eq. (3) partially w.r.t y,
2( y − 2) + λ ((22 y ) = 0
y−2 2
λ=− = −1 +
y y …(5)
Differentiating Eq. (3) partially, w.r.t. z,
2( z − 2 ) + λ ( 2 z ) = 0
z−2 2
λ=− = −1 +
z z …(6)
From Eqs (4), (5) and (6),
1 2 2
−1 +
−1 = −1 + = −1
−1 +
x y z
1 2 2
= =
x y z
y = 2x and z = 2x
Substituting y and z in Eq. (2),
x 2 + 4 x 2 + 4 x 2 = 36
9 x 2 = 36
x2 = 4
x = ±2
∴ y = ±4
and z = ±4
Minimum distance = ((22 − 1) 2 + ((44 − 2) 2 + ((44 − 2) 2 = 1 + 4 + 4 = 3

Maximum distance = ( −2 − 1) 2 + ((−


− 4 − 2) 2 + ( − 4 − 2) 2 = 9 + 36 + 36 = 9

Example 11
A rectangular box open at the top is to have a volume of 32 cubic units. Find
the dimensions of the box requiring least material for its construction.
[Winter 2016, 2014]
8.7 Method of Lagrangian Multipliers 8.61

Solution
Let x, y, z be the dimensions of the box.
The box is open at the top.
Surface area S = xy + 2xz + 2yz …(1)
Volume V = xyz = 32 …(2)
Let f (x, y, z) = xyz – 32
Let the auxiliary equation be
( xy + 2 xz + 2 yz ) + λ ( xyz − 32) = 0 ...(3)
Differentiating Eq. (3) partially w.r.t. x,
y + 2z + lyz = 0
y + 2z 1 2
λ=− =− − …(4)
yz z y
Differentiating Eq. (3) partially w.r.t. y,
x + 2z + lxz = 0
x + 2z 1 2
λ=− =− − …(5)
xz z x
Differentiating Eq. (3) partially w.r.t. z,
2x + 2y + lxy = 0
2x + 2 y 2 2
λ=− =− − …(6)
xy y x
From Eqs (4), (5) and (6),
1 2 1 2 2 2
− − =− − =− −
z y z x y x
2 2 1 2
= and =
y x z x
x
y = x and z =
2
Substituting y and z in Eq. (2),
 x
x( x)   = 32
 2
x 3 = 64
x=4
∴ y = 4, z = 2
Hence, dimensions of the box requiring least material for its construction are 4, 4, 2.
8.62 Chapter 8 Applications of Partial Derivatives

Example 12
A rectangular box without a lid is to be made from 12 m2 of cardboard.
Find the maximum volume of such a box. [Winter 2013, 2015]
Solution
Let x, y, z be the dimensions of the box.
The box is open at the top.
Volume V = xyz ...(1)
Surface area S = xy + 2xz + 2yz = 12 ...(2)
Let f (x, y, z) = xy + 2xz + 2yz – 12
Let the auxiliary equation be
xyz + l(xy + 2xz + 2yz – 12) = 0 ...(3)
Differentiating Eq. (3) partially w.r.t. x,
yz + l(y + 2z) = 0
yz
l=-
y + 2z
1 y + 2z 1 2
=- =- - ...(4)
l yz z y
Differentiating Eq. (4) partially w.r.t. y,
xz + l(x + 2z) = 0
xz
l=-
x + 2z
1 x + 2z 1 2
=- =- - ...(5)
l xz z x
Differentiating Eq. (3) partially w.r.t. z,
xy + l(2x + 2y) = 0
xy
l=-
2( x + y )
1 2x + 2y 2 2
=- =- - ...(6)
l xy y x

From Eqs (4), (5) and (6),

1 2 1 2 2 2
- - =- - =- -
z y z x y x
2 2 1 2
= and =
y x z x
x
y = x and z =
2
8.7 Method of Lagrangian Multipliers 8.63

Substituting y and z in Eq. (2),


Ê xˆ Ê xˆ
x( x ) + 2 x Á ˜ + 2 x Á ˜ = 12
Ë 2¯ Ë 2¯
3 x 2 = 12
x2 = 4
x=2
\ y = 2, z = 1
Hence, maximum volume = xyz = 2(2)(1) = 4 m3

Example 13
A wire of length b is cut into two parts which are bent in the form of
a square and circle respectively. Find the least value of the sum of the
areas so found.
Solution
x
Let the piece of length x be bent in the form of a square so that each side is .
4
x x x2
The area of the square, A1 =⋅ = .
4 4 16
Suppose a piece of length y is bent in the form of a circle of radius r; so perimeter of
the circle is y.
2π r = y
y
r=

2
 y y2
The area of the circle, A2 = π   = .
 2π  4π
Let sum of the areas be given as
x2 y 2
f ( x, y ) = + ...(1)
16 4π
Also, x+y=b …(2)
Let f (x, y) = x + y – b
Let the auxiliary equation be
 x2 y 2 
 +  + λ ( x + y − b) = 0 ...(3)
16 4π
Differentiating Eq. (3) partially w.r.t. x,
2x
+l = 0
16
8.64 Chapter 8 Applications of Partial Derivatives

x
l=- ...(4)
8
Differentiating Eq. (3) partially w.r.t. y,
2y
+λ =0

y
λ=− ...(5)

From Eqs (4) and (5),
x y
=
8 2π

π
y= x
4
Substituting y in Eq. (2),
π
x+ x=b
4
4b
x=
4+π
πb
∴y=
4+π
Hence, the least value of the sum of the areas is
2 2
x2 y 2 1  4b  1  πb 
+ =   +  

16 4π 16 4 + π 4π  4 + π 
b2  π2 
= 1 + 4π 
(4 + π ) 2
b 2π (4 + π )
=
4π (4 + π ) 2
b2
=
4(π + 4)

Example 14
A closed rectangular box has length twice its breadth and has constant
volume V. Determine the dimensions of the box requiring least surface
area.
Solution
Let x be the breadth and y be the height of the rectangular box so length of the box
will be 2x.
8.7 Method of Lagrangian Multipliers 8.65

Surface area of the box


S = 2 (2x ◊ x + x ◊ y + y ◊ 2x) = 4x2 + 6xy
Let f (x, y) = 4x2 + 6xy ...(1)
Volume of the box V = x ◊ 2x ◊ y = 2x2y ...(2)
Let f (x, y) = 2x2y – V
Let the auxiliary equation be
(4 x 2
+ 6 xxyy ) + λ (2 x 2 y − V ) = 0 ...(3)
Differentiating Eq. (3) partially w.r.t. x,

8 x + 6 y + λ (4 xy ) = 0
4x + 3y
λ=−
2 xy …(4)
Differentiating Eq. (3) partially w.r.t. y,

6 x + λ (2 x 2 ) = 0
3
λ=− ...(5)
x
From Eqs (4) and (5),
4x + 3y 3
=
2 xy x
4x + 3y = 6 y
4x
y=
3
Substituting y in Eq. (2),
4x
2 x2 ◊ =V
3
3V
x3 =
8
1
Ê 3V ˆ 3
x=Á ˜
Ë 8 ¯
1 1
4 Ê 3V ˆ 3 Ê 8V ˆ 3
\y = Á ˜ = Á ˜
3Ë 8 ¯ Ë 9 ¯
1 1
 3V  3  3V  3
Hence, the dimensions of the box requiring least surface area are 2   ,   ,
1
 8  8
 8V  3
  .
9
8.66 Chapter 8 Applications of Partial Derivatives

Example 15
Show that if the perimeter of a triangle is a constant, the triangle has
maximum area when it is equilateral.
Solution
Let x, y and z be the sides of the triangle.
x+ y+ z
Perimeter of the triangle s=
2
Area of the triangle A = s ( s − x)( s − y )( s − z )
Let f (x, y, z) = A2 = s(s – x)(s – y)(s – z) …(1)

Also, x + y + z = 2s …(2)
Let f (x, y, z) = x + y + z – 2s
Let the auxiliary equation be
[ s( s − x)( s − y)( s − z )] + λ ( x + y + z − 2s) = 0 ...(3)
Differentiating Eq. (3) partially w.r.t x,
−ss ( s − y )( s − z ) + λ = 0

λ = s ( s − y )( s − z ) …(4)
Differentiating Eq. (3) partially w.r.t y,
− s ( s − x)( s − z ) + λ = 0
λ = s ( s − x)( s − z ) …(5)

Differentiating Eq. (3) partially w.r.t z,

− s ( s − x)( s − y ) + λ = 0
λ = s ( s − x)( s − y ) …(6)
From Eqs (4), (5) and (6),
s ( s − y )( s − z ) = s(
s ((ss − x)( s − z ) = s ( s − x)( s − y )
s − y = s − x and s − z = s − y
y=x and z= y
∴x = y = z
Hence, the triangle is equilateral.

Example 16
The temperature u(x, y, z) at any point in space is u = 400 xyz2. Find the
highest temperature on surface of the sphere x2 + y2 + z2 = 1.
8.7 Method of Lagrangian Multipliers 8.67

Solution
Let f (x, y, z) = u = 400 xyz2 …(1)
x2 + y2 + z2 = 1 …(2)
Let f (x, y, z) = x2+y2 + z2 – 1 = 0

Let the auxiliary equation be

400 xyz 2 + λ ( x 2 + y 2 + z 2 − 1) = 0 ...(3)

Differentiating Eq. (3) partially w.r.t. x,


200 yz 2
λ=− …(4)
x
Differentiating Eq. (3) partially w.r.t. y,

400 xz 2 + λ (2 y ) = 0

200 xz 2
λ=− …(5)
y
Differentiating Eq. (3) partially w.r.t. z,

800 xyz + λ (2 z ) = 0
λ = −400 xy
λ = −400xy …(6)
From Eqs (4), (5) and (6),

200 yz 2 200 xz 2
= = 400 xy
x y
200 y 2 z 2 = 200 x 2 z 2 = 400 x 2 y 2
1 1 2
= =
x2 y 2 z 2
z2
x2 = y 2 =
2
Substituting y2 and z2 in Eq. (2),

x2 + x2 + 2x2 = 1
4x2 = 1
1
x2 =
4
8.68 Chapter 8 Applications of Partial Derivatives

1
x=±
2
1 1
∴y=± , z =±
2 2

Considering positive sign,


1 1 1
x= , y= , z=
2 2 2
Highest temperature on the surface of the sphere
 1  1  1
u = 400 xyz 2 = 400       = 50.
 2  2  2

Example 17
Use the method of the Lagrange’s multipliers to find volume of larg-
est rectangular parallelepiped that can be inscribed in the ellipsoid
x2 y 2 z 2
+ + = 1.
a 2 b2 c2
Solution
Let 2x, 2y, 2z be the length, breadth and height of the rectangular parallelepiped
inscribed in the ellipsoid.
Volume of the parallelepiped, V = (2x) (2y) (2z) = 8xyz.
Let f (x, y, z) = 8 xyz ...(1)

x2 y 2 z 2
+ + =1 …(2)
a 2 b2 c2

x2 y 2 z 2
Let φ ( x, y , z ) = + + −1
a 2 b2 c2

Let the auxiliary equation be

 x2 y 2 z 2 
8xyz + λ  2 + 2 + 2 − 1 = 0
8 ...(3)
a b c 
Differentiating Eq. (3) partially w.r.t. x,

2x
8 yz + λ =0
a2
4 yza 2
λ=− …(4)
x
8.7 Method of Lagrangian Multipliers 8.69

Differentiating Eq. (3) partially w.r.t. y,

2y
8 xz + λ =0
b2
4 xzb 2
λ=− …(5)
y
Differentiating Eq. (3) partially w.r.t. z,
2z
8 xy + λ =0
c2
4 xyc 2
λ=− …(6)
z
From Eqs (4), (5) and (6),
4 yza 2 4 xzbb 2 4 xyc 2
= =
x y z
a 2 b2 c2
= =
x2 y 2 z 2
b2 2 c2 2
∴ y2 = x and z2 = x
a2 a2
Substituting y2, z2 in Eq. (2),
x2 x2 x2
+ + =1
a2 a2 a2
3x 2
=1
a2
a2
x2 =
3
a
x=±
3
b c
∴y=± , z=±
3 3
Since sides cannot be negative,
a b c
x= , y= , z=
3 3 3
Volume of the largest rectangular parallelepiped that can be inscribed in the ellipsoid
 a   b   c  8abc
V = 8 xyz = 8       =
 3  3  3 3 3
8.70 Chapter 8 Applications of Partial Derivatives

Example 18
Find the minimum distance from origin to the plane 3x + 2y + z = 12.
Solution
Let (x, y, z) be a point on the plane 3x + 2y + z = 12.
The distance d between (x, y, z) and origin (0, 0, 0) is

d= x2 + y 2 + z 2
d 2 = x2 + y 2 + z 2

Let f (x, y, z) = d 2 = x 2 + y2 + z2 ...(1)


3x + 2y + z = 12 ...(2)

Let f (x, y, z) = 3x + 2y + z – 12 = 0
Let the auxiliary equation be
( x 2 + y 2 + z 2 ) + λ (3 x + 2 y + z − 12) = 0 ...(3)

Differentiating Eq. (3) partially w.r.t. x,


2x + l (3) = 0
2x
λ=− ...(4)
3
Differentiating Eq. (3) partially w.r.t. y,
2y + l (2) = 0
2y
λ=− = −y ...(5)
2
Differentiating Eq. (3) partially w.r.t. z,
2z + l (1) = 0
l = –2z ...(6)

From Eqs (3), (4) and (5),


2x
= y = 2z
3
2x
y=
3
x
z=
3
Substituting y, z in Eq. (2),
4x x
3x + + = 12
3 3
8.7 Method of Lagrangian Multipliers 8.71

14 x
= 12
3
18 12 6
\ x= ,y= ,z=
7 7 7
The minimum distance is
2 2 2
 18   12   6  504
d =   +  +  =
 7   7   7 7

Example 19
Divide 24 into three parts such that the continued product of the first,
square of the second and cube of the third may be maximum.
Solution
Let x, y and z be three parts of 24.
Let f (x, y, z) = xy2 z3 ...(1)
Let x + y + z = 24 ...(2)
φ ( x, y, z ) = x + y + z − 24 = 0
Let the auxiliary equation be
xy2 z3 + l (x + y + z – 24) = 0 ...(3)
Differentiating Eq. (3) partially w.r.t. x,
y2z3 + l = 0
l = – y2z3 ...(4)
Differentiating Eq. (3) partially w.r.t. y,
2xyz3 + l = 0
l = – 2xyz3 ...(5)
Differentiating Eq. (3) partially w.r.t. z,
3xy2z2 + l = 0
l = – 3xy2z2 ...(6)
From Eqs (4), (5), (6),
y2z3 = 2xyz3 = 3xy2z2

Dividing by xy2z3,
1 2 3
= =
x y z
y = 2 x, z = 3 x
8.72 Chapter 8 Applications of Partial Derivatives

Substituting y, z in Eq. (2)


x + 2 x + 3 x = 24
6 x = 24
x=4
∴ y = 8, z = 12
Hence, 4, 8 and 12 are three parts of 24.

Example 20
A space probe in the shape of the ellipsoid 4 x 2 + y 2 + 44z 2 = 16 enters
the earth’s atmosphere and its surface begins to heat. After 1 hour,
the temperature at the point (x, y, z) on the surface of the probe
T ( x, y, z ) = 8 x 2 + 4 yz
y – 16 z + 600. Find the hottest points on the
probe’s surface.
Solution
Let f ( x, y, z ) = 8 x 2 + 4 yyzz − 16 z + 600 …(1)

4x2 + y 2 + 4
4z 2 = 16 …(2)
Let φ ( x, y, z ) = 4 x 2 + y 2 + 4 z 2 − 16 = 0

Let the auxiliary equation be


(8 x 2 + 4 yz − 16 z + 600) + λ (4 x 2 + y 2 + 4 z 2 − 16) = 0 …(3)
Differentiating Eq. (3) partially w.r.t. x,
16x + l (8x) = 0
l = –2 …(4)
Differentiating Eq. (3) partially w.r.t. y,
4z + l(2y) = 0
2z
λ=− …(5)
y
Differentiating Eq. (3) partially w.r.t. z,
4y – 16 + l(8z) = 0
16 − 4 y 4 − y
λ= = …(6)
8z 2z
From Eqs (4) and (5),
2z
−2 = −
−2
y
y=z …(7)
8.7 Method of Lagrangian Multipliers 8.73

From Eqs (4) and (6),


4− y
−2 =
2y
−4 y = 4 − y
−3 y = 4
4
y=−
3
4
z=−
3
Substituting in Eq. (2),
16 64
4x2 + + = 16
9 9
64
4x2 =
9
16
x2 =
9
4
x=±
3
 4 4 4
The hottest points on the probe’s surface are  ± , − , − 
 3 3 3

Example 21
Find the points on the surface z 2 = xxyy + 1 nearest to the origin.
Solution
Let (x, y, z) be any point on the surface z2 = xy + 1.
The distance d between (x, y, z) and origin (0, 0, 0) is
d= x2 + y 2 + z 2
d 2 = x2 + y 2 + z 2
Let f ( x, y , z ) = d 2 = x 2 + y 2 + z 2 …(1)
z2 = xy + 1 …(2)

Let φ ( x, y, z ) = z 2 − xy − 1 = 0

Let the auxiliary equation be


(x2 + y2 + z2) + l(z2 – xy – 1) = 0 …(3)
Differentiating Eq. (3) partially w.r.t. x,
2x – ly = 0
8.74 Chapter 8 Applications of Partial Derivatives

y
λ= …(4)
2x
Differentiating Eq. (3) partially w.r.t. y,
2y – l x = 0
x
λ= …(5)
2y
Differentiating Eq. (3) partially w.r.t. z,
2z + l (2z) = 0
l = –1 …(6)
From Eqs (4) and (6), y
= −1
2x
y = −2 x
From Eqs (5) and (6),
x
= −1
2y
x = –2y = –2(–2x) = 4x
x=0
\ y=0
Substituting in Eq. (2)
z2 = 1
z=±1
The nearest points on the surface from the origin are (0, 0, ±1).

Example 22
x2 y 2 z 2
If u = 3 + 3 + 3 where x + y + z = 1 then prove that stationary value
a b c
a3 b3 c3
of u is given by x = 3 3 3 , y = 3 3 3 , z = 3 3 3
a +b +c a +b +c a +b +c

Solution
x2 y 2 z 2
Let f ( x, y , z ) = u = + + …(1)
a 3 b3 c 3
x+y+z=1 …(2)

Let f (x, y, z) = x + y + z – 1 = 0
8.7 Method of Lagrangian Multipliers 8.75

Let the auxiliary equation be

 x2 y 2 z 2 
 a 3 + b3 + c 3  + λ ( x + y + z − 1) = 0 …(3)

Differentiating Eq. (3) partially w.r.t. x,

2x
+λ =0
a3
2x
λ=− …(4)
a3
Differentiating Eq. (3) partially w.r.t. y,
2y
+λ =0
b3
2y
λ=− …(5)
b3
Differentiating Eq. (3) partially w.r.t. z,
2z
+λ =0
c3
2z
λ=− …(6)
c3
From Eqs (4), (5) and (6),
2x 2 y 2z
= =
a 3 b3 c 3
x y z
3
= 3 = 3
a b c
b3 c3
y= x
x,, z = x
a3 a3

Substituting y, z in Eq. (2),


b3 c3
x+ x + x =1
a3 a3
(a 3 + b3 + c 3 ) x
=1
a3
a3
x=
a 3 + b3 + c 3
b3 c3
∴ y= 3 , z= 3
a +b +c3 3
a + b3 + c 3
8.76 Chapter 8 Applications of Partial Derivatives

Example 23
a 3 b3 c 3
If u = + + where x + y + z = 1 then find the stationary values.
x2 y 2 z 2
Solution
a 3 b3 c 3
Let f ( x, y , z ) = u = + + …(1)
x2 y 2 z 2
x+y+z=1 …(2)
Let f (x, y, z) = x + y + z – 1 = 0
Let the auxiliary equation be
 a 3 b3 c 3 
 x 2 + y 2 + z 2  + λ ( x + y + z − 1) = 0 …(3)

Differentiating Eq. (3) partially w.r.t. x,


2a 3
− +λ =0
x3
2a 3
λ= …(4)
x3

Differentiating Eq. (3) partially w.r.t. y,

2b3
− +λ =0
y3
2b3
λ= …(5)
y3
Differentiating Eq. (3) partially w.r.t. z,
2c 3
− +λ =0
z3
2c 3
λ= …(6)
z3
From Eqs (4), (5) and (6),
22aa 3 2b3 2c 3
= 3 = 3
x3 y z
a 3 b3 c 3
= =
x3 y 3 z 3
a b c
= =
x y z
8.7 Method of Lagrangian Multipliers 8.77

b
y= x
a
c
z= x
a

Substituting y, z in Eq. (2),


b c
x+ x+ x =1
a a
(a + b + c)
x =1
a
a
x=
a+b+c
b c
∴ y= ,z=
a+b+c a+b+c

Example 24
x2 y 2 z 2
Prove that the stationary values of u= + + where
a 4 b4 c4
x2 y 2 z 2
lx + my + nz = 0 and + + = 1 are given by the roots of the equa-
a 2 b2 c2
l 2a 4 m 2b 4 n 2c 4
tion + + =0
1 − a 2u 1 − b 2u 1 − c 2u
Solution
x2 y 2 z 2
Let f ( x, y , z ) = u =
+ + …(1)
a 4 b4 c4
lx + my + nz = 0 …(2)
f (x, y, z) = lx + my + nz = 0
x2 y 2 z 2
+ + =1 …(3)
a 2 b2 c2
x2 y 2 z 2
ψ ( x, y , z ) = + + −1 = 0
a 2 b2 c2

Let the auxiliary equation be

 x2 y 2 z 2   x2 y 2 z 2 
+ +
 a 4 b 4 c 4  + λ1 (lx + my + nz ) + λ 2  2 + 2 + 2 − 1 = 0 …(4)
a b c 
8.78 Chapter 8 Applications of Partial Derivatives

Differentiating Eq. (4) partially w.r.t. x,


2x  2x 
+ λ1l + λ 2  2  = 0 …(5)
a 4
a 

Differentiating Eq. (4) partially w.r.t. y,


2y  2y
+ λ1m + λ 2  2  = 0 …(6)
b4 b 
Differentiating Eq. (4) partially w.r.t. z,

22z  2z 
+ λ1n + λ 2  2  = 0 …(7)
c 4
c 

Multiplying Eq. (5) by x, Eq. (6) by y, Eq. (7) by z and then adding,

 x2 y 2 z 2   x2 y 2 z 2 
2  4 + 4 + 4  + λ1 (lx + my + nz ) + 2λ 2  2 + 2 + 2  = 0
a b c  a b c 
22u + λ1 (0) + 2λ 2 (1) = 0
λ 2 = −u

Substituting l2 = – u in Eq. (5),


22xx 2 xu
+ λ1l − 2 = 0
a4 a
1− a u
2
2x  + λ1l = 0
 a 4 
a 4 λ1l
x=−
2(1 − a 2 u )

b 4 λ1m
Similarly y=−
2(1 − b 2 u )

c 4 λ1n
z=−
2(1 − c 2 u )

Substituting x, y, z in Eq. (2),

l 2 a 4 λ1 m 2 b 4 λ1 n 2 c 4 λ1
− − − =0
2(1 − a 2 u ) 2(1 − b 2 u ) 2(1 − c 2 u )
 l 2a4 m2b4 n2c4 
 1 − a 2 u + 1 − b 2 u + 1 − c 2 u  λ1 = 0
But l1 π 0
8.7 Method of Lagrangian Multipliers 8.79

l 2a4 m2b4 n2c4


∴ + + =0
1 − a 2u 1 − b2u 1 − c 2u

Example 25
In a plane triangle ABC, find the extreme values of cos A cos B cos C.
[Summer 2015]
Solution
Let f (A, B, C) = cos A cos B cos C …(1)
In a triangle ABC,
A + B + C = 180° …(2)
Let f (A, B, C) = A + B + C – 180°
Let the auxiliary equation be
cos A cos B cos C + l (A + B + C – 180°) = 0 …(3)
Differentiating Eq. (3) partially w.r.t. A,
– sin A cos B cos C + l = 0
l = sin A cos B cos C …(4)
Differentiating Eq. (3) partially w.r.t. B,

– cos A sin B cos C + l = 0


l = cos A sin B cos C …(5)
Differentiating Eq. (3) partially w.r.t. C,
– cos A cos B sin C + l = 0
l = cos A cos B sin C …(6)
From Eqs (4), (5) and (6),
sin A cos B cos C = cos A sin B cos C = cos A cos B sin C
Dividing by cos A cos B cos C,
tan A = tan B = tan C
p
A=B=C=
3
p p p
Hence, fmax = cos A cos B cos C = cos cos cos
3 3 3
Ê 1ˆ Ê 1ˆ Ê 1ˆ 1
=Á ˜Á ˜Á ˜ =
Ë 2¯ Ë 2¯ Ë 2¯ 8
8.80 Chapter 8 Applications of Partial Derivatives

EXERCISE 8.4
1. Find stationary values of the function f (x, y, z) = x2 + y2 + z2, given that
z2 = xy + 1.
[Ans. : (0, 0, −1), (0, 0, 1)]
2. Find the stationary value of a3x2 + b3y 2 + c 3z 2 subject to the fulfillment of
1 1 1
the condition + + = 1, given a, b, c are not zero.
x y z

 1 1 1 
 Ans. : x = a (a + b + c), y = b (a + b + c),
c), z = (a + b + c)
 c 
3. Find the largest product of the numbers x, y and z when x + y + z 2 = 16.
 4096 
 Ans. : 
 25 5 
4. Find the largest product of the numbers x, y and z when x 2 + y 2 + z 2 = 9.
 Ans. : 3 3 
 
5. Find a point in the plane x + 2y + 3z = 13 nearest to the point (1, 1, 1).
 3 5 
 Ans. :  , 2,  
 2 2 
6. Find the shortest distance from the point (1, 2, 2) to the sphere
x 2 + y 2 + z 2 = 36. [Ans. : 3]
7. Find the maximum distance from the origin (0, 0) to the curve
3x 2 + 3y 2 + 4xy − 2 = 0.
 Ans. : 2 
 
8. Decompose a positive number a into three parts so that their product is
maximum.
  a a a 
 Ans. :  , ,  
 3 3 3 
9. Find the maximum value of x m y n z p when x + y + z = a.
 a m + n + p mm n n p p 
 Ans. : 
 (m + n + p)m + n + p 

10. Find the dimensions of a rectangular box of maximum capacity whose


surface area is given when
(i) box is open at the top (ii) box is closed

 s s 1 s s s s
 Ans. : (i) , , (ii) , , 
 3 3 2 3 6 6 6 
8.8 Taylor’s Formula for Two Variables 8.81

11. Determine the perpendicular distance of the point (a, b, c) from the
plane lx + my + nz = 0.
 la + mb + nc 
 Ans. : minimum distance 2 
 l + m2 + n2 
12. Find the length and breadth of a rectangle of maximum area that can be
inscribed in the ellipse 4x 2 + y 2 = 36.
 3 2 
 Ans. : , 2, Area = 12
 2 
13. Find the volume of the largest rectangular parallelepiped that can be
inscribed in the ellipsoid of revolution 4x 2 + 4y 2 + 9z 2 = 36.
 Ans. : 16 3 

14. Find the extreme volume of x2 + y2 + z2 + xy + xz + yz subject to the


conditions x + y + z = 1 and x + 2y + 3z = 3.
 1 1 5
 Ans. : 6 , 3 , 6 
 

8.8 TAYLOR’S FORMULA FOR TWO VARIABLES


Theorem If f (x + h, y + k) is a given function which can be expanded into a series of
positive ascending powers of h and k then
2
 ∂ ∂ 1 ∂ ∂
f ( x + h, y + k ) = f ( x, y ) +  h + k  f ( x, yy) +  h + k  f ( x, y )
 ∂x ∂y  2!  ∂x ∂y 
3
1 ∂ ∂
+  h + k  f ( x, y ) + 
3!  ∂x ∂y 
Proof By Taylor’s expansion for function of a single variable, expanding f (x + h, y + k)
as a function of x,
∂ h2 ∂2
f ( x + h, y + k ) = f ( x , y + k ) + h f ( x, y + k)
k) + f ( x, y + k )
∂x 2! ∂x 2
h3 ∂3
+ f ( x, y + k)
k) + …(1)
3! ∂x 3
Again, expanding f (x, y + k) as a function of y,
∂ k 2 ∂2
f ( x, y + k)
k ) = f ( x, y ) + k f ( x, y ) + f ( x, y )
∂y 2! ∂y 2
k 3 ∂3
+ f ( x, y ) +  …(2)
3! ∂y 3
8.82 Chapter 8 Applications of Partial Derivatives

Differentiating Eq. (2) partially w.r.t. x,

∂ ∂ ∂2 2
∂3
f x, y + = f x y +k f x, y ) + f x y ) +  …(3)
∂x x ∂ y y
Differentiating Eq. (3) partially w.r.t. x,
∂2 ∂2 ∂3
f x, y + = f x y +k f x, y ) + …(4)
∂x 2 x ∂ y
Differentiating Eq. (4) partially w.r.t. x,
∂3 ∂3
f x, y + = f x y) +  …(5)
∂x 3 ∂x
Substituting Eqs (2), (3), (4) and (5) in Eq. (1),

 ∂ k2 2
k3 3 
f x +k x +k f x, y ) + x + f x, ) +
 y 2 3! y 3
∂ 2 2
∂3
+ x k f x, y ) + f ,
∂x x y 2 ! ∂x y 2
h2 ∂3 h3 3

+ +k f x + + f( y) + 
2 x x 3 ∂x 3

∂ ∂ 1 2 ∂2 ∂2 ∂2
+ h +k f x, y + h + 2hk + k2 2 f x, y )
x y ∂x 2
∂ y y
1 3 ∂3 ∂3 ∂3 ∂3
+ h + 3h k + 3hk 2 + k3 3 f x, y ) +
3 ∂x 3
∂ y ∂ y 2
y
2
∂ ∂ 1 ∂ ∂
+ h +k f x, y + h +k f x, y )
∂x y ∂x y
1 ∂ ∂
+ h +k f x, y ) + 
3 ∂x y
This is known as Taylor’s series expansion of f (x + h, y + k) in powers of h and k.
Putting x = a and y = b in the above series,
f a +k a + hf a b k ( a b)
1
+ h a b + 2hk f (a b) k f (a b)
2
1
+ a k a b hk f (a b) k f (a b) +
3
f f ∂2 f ∂2 f ∂2 f
where , , , et
y x y ∂x y
8.8 Taylor’s Formula for Two Variables 8.83

Putting a + h = x and b + k = y,

f ( x, y ) = f (a, b) + ( x − a ) f x (a, b


b) + ( y − b) f y (a, b) 
1
+ ((xx − a ) 2 f xx (a b) + 2( x − a )( y − b a,, b) + ( y − b) 2 f yy (a, b)  + 
b)) f xy (a
2! 
This expansion is called the expansion of f (x, y) in powers of (x – a) and (y – b).
Putting a = 0 and b = 0,
1 2
f ( x, y ) = f (00,, 0) +  x f x (0, 0) + y f y (0,
0, 0)  +  x f xx (0, 0) + 2 xy f xy (0 0) + y 2 f yy (0, 0) 
2! 
1 3
+  x f xxx ((0
0, 0) + 3 x 2 y f xxy (0, 0) + 3 xxyy 2 f xyy (0, 0) + y 3 f yyy (0, 0)  + 
3! 

Example 1
Expand ex + y in power of (x – 1) and (y + 1) up to first-degree terms.
Solution
Let f (x, y) = e x+y
By Taylor’s expansion,
f ( x, y ) = f ( a, b) + [( x − a) f x ( a, b) + ( y − b) f y ((aa, b)] +  …(1)

Here, a = 1 and b = –1.


f (x, y) = e x+y f (1, –1) = 1
x+y
fx (x, y) = e fx(1, –1) = 1
fy (x, y) = e x+y fy(1, –1) = 1
Substituting these values in Eq. (1),
f ( x, y ) = 1 + [( x − 1))((1) + ( y + 1)(
) (1))]] + 
e x + y = 1 + ( x − 1) + ( y + 1) + 

Example 2
Expand xy near the point (1, 1) up to the first-degree terms.
Solution
Let f (x, y) = xy
By Taylor’s expansion,
f ( x, y ) = f (a, b) + [( x − a ) f x (a, b) + ( y − b) f y ((a
a, b)] +  …(1)

Here a = 1 and b =1.


f (x, y) = x y, f (1, 1) = 1
fx (x, y) = yxy –1, fx(1, 1) = 1
8.84 Chapter 8 Applications of Partial Derivatives

fy (x, y) = x y log x, fy(1, 1) = 0


Substituting these values in Eq. (1),
f ( x, y ) = 1 + [( x − 1))((1) + ( y − 1)(
)(0))]] + 
x y = 1 + ( x − 1) + 

Example 3
 π
Expand e x cos y near 1,  by Taylor’s series up to first-degree terms only.
 4
Solution
Let f (x, y) = ex cos y
By Taylor’s expansion,
f ( x, y ) = f (a, b) + [( x − a ) f x (a, b) + ( y − b) f y ((a
a, b)] +  …(1)
π
Here, a = 1, and b = .
4
Ê pˆ p e
f ( x, yy)) = e x cos y, f Á 1, ˜ = e cos
cos =
Ë 4¯ 4 2
Ê pˆ p e
f x ( x, yy)) = e x cos y, f x Á 1, ˜ = e cos =
Ë 4¯ 4 2
Ê pˆ p e
f y ( x, y) = -e x sin y, f y Á 1, ˜ = -e si
sin = -
Ë 4¯ 4 2
Substituting these values in Eq. (1),
e   e   π  e 
f ( x, y ) = + ( x − 1)   + y −  −   +
2   2  4  2
e   π 
e x cos y = + ( x − 1) −  y −  
11+
2  4 

Example 4
Expand e xy in the power of (x – 1) and (y – 1) using Taylor’s expansion.
[Summer 2017]
Solution
f (x, y) = exy
By Taylor’s expansion,
f ( x, y ) = f (a, b) + [( x − a)
a ) f x (a, b) + ( y − b) f y (a, b)]
1
+ ((xx − a ) 2 f xx (a, b) + 2( x − a )( y − b) f xy (a, b) + ( y − b) 2 f yy (a, b)  + 
2! 
...(1)
8.8 Taylor’s Formula for Two Variables 8.85

Here, a = 1 and b = 1.
f (x, y) = exy f (1, 1) = e
fx (x, y) = yexy, fx (1, 1) = e
fy (x, y) = xexy, fy (1, 1) = e
fxx (x, y) = y2exy, fxx (1, 1) = e
fxy (x, y) = xyexy + exy, fxy (1, 1) = e + e = 2e
fyy (x, y) = x2exy, fyy (1, 1) = e
Substituting these values in Eq. (1),
1 2
f x, y e [(x e+( e + [( x e x - e - e +
2!
È ( 2 ˘
e +( + +
Î 2 ˚

Example 5
Expand sin xy in powers of (x – 1) and y − up to second-degree terms.
2
[Summer 2014]
Solution
Let f (x, y) = sin xy
By Taylor’s expansion,
x, y a + [( x a a y b (a b)]
1
+ x a) f a x−a y− a b + (y a +
2!
π
Here, a = 1 and b = .
2
p p
f x, y x , 1
2 2
p p p
f c 1 = =0
2 2 2
p p
f x cos 1, =0
2 2
p p2 p p2
f x, y y sin xy, 1, =- sin -
2 4 2 4
p p p p
f x, y - n xy) y xy f 1, sin + cos -
2 2 2 2 2
p p
f x, y x sin xy , = - sin = -1
2 2
8.86 Chapter 8 Applications of Partial Derivatives

Substituting these values in Eq. (1),


  π 
y ) = 1 + ( x − 1)0 +  y −  0
f ( x, y)
  2 
1 
2
2  π2   π  π  π
+ ((xx − 1)  −  + 2( x − 1)  y −   −  +  y −  ( −1)  + 
2 !   4  2   2   2  
2
π2 π  π 1 π
sin xy = 1 − ( x − 1) 2 − ( x − 1)  y −  −  y −  + 
8 2  2 2  2

Example 6
Expand xy2 + xy + 3 in powers of (x – 1) and (y + 2) using Taylor’s
series expansion. [Summer 2016]
Solution
Let f ( x, y) = xy 2 + xy + 3
By Taylor’s expansion,

f ( x, y) = f (a, b) + ÈÎ( x - a ) f x (a, b) + ( y - b) f y (a, b)˘˚


1 È
+ ( x - a )2 f xx (a, b) + 2( x - a )( y - b) f xy (a, b)
2! Î
+( y - b)2 f yy (a, b)˘˚ +  ...(1)

Here, a = 1 and b = –2.

f ( x, y) = xy 2 + xy + 3, f (1, - 2) = 4 - 2 + 3 = 5
2
f x ( x, y) = y + y, f x (1, - 2) = 4 - 2 = 2
f y ( x, y) = 2 xy + x, f y (1, - 2) = -4 + 1 = -3
f xx ( x, y) = 0, f xx (1, - 2) = 0
f yy ( x, y) = 2 x, f yy (1, - 2) = 2
f xy ( x, y) = 2 y + 1, f xy (1, - 2) = - 4 + 1 = - 3
and so on.
Substituting these values in Eq. (1),
f ( x, y) = 5 + [( x - 1)(2) + ( y + 2)( -3)]
1
+ [( x - 1)2 (0) + 2( x - 1)( y + 2)( -3) + ( y + 2)2 ( -3)] +
2!
8.8 Taylor’s Formula for Two Variables 8.87

3
xy 2 + xy + 3 = 5 + 2( x - 1) - 3( y + 2) - 3( x - 1)( y + 2) - ( y + 2)2 +
2

Example 7
Expand x2y + 3y –2 in powers of (x – 1) and ( y + 2) up to second-
degree terms.
Solution
Let f (x, y) = x2y + 3y −2
By Taylor’s expansion,
f ( x, y ) = f (a, b) + [( x − a)
a ) f x (a, b) + ( y − b) f y (a, b)]
1
+ ((xx − a ) 2 f xx (a, b) + 2( x − a )( y − b) f xy (a, b)
2! 
+(( y − b) 2 f yy (a, b)  + 
+ …(1)

Here, a = 1 and b = –2.


f (x, y) = x2y + 3y –2, f (1, –2) = (1)2 (–2) + 3(–2) –2 = –10
fx (x, y) = 2xy, fx (1, –2) = 2(1)(–2) = – 4
fy (x, y) = x2 + 3, fy (1, –2) = (1)2 + 3 = 4
fxx (x, y) = 2y, fxx (1, –2) = 2(–2) = – 4
fxy (x, y) = 2x, fxy (1, –2) = 2(1) = 2
fyy (x, y) = 0, fyy (1, –2) = 0
Substituting these values in Eq. (1),
f ( x, y ) = −10 + [( x − 1)(
)( −4) + ( y + 2)4]
1
+ ( x − 1) 2 ( −4) + 2( x − 1))(( y + 2)(
)(2) + ( y + 2) 2 (0)  + 
2!
x 2 y + 3 y − 2 = −10 − 4( x − 1) + 4( y + 2) − 2( x − 1) 2 + 2( x − 1)( y + 2) + 

Example 8
Expand f (x, y) = y x in the neighbourhood of (1, 1) up to the terms of
second degree.
Solution
f (x, y) = y x
8.88 Chapter 8 Applications of Partial Derivatives

By Taylor’s expansion,
f x, y a b + x a a b (a b)]
1
+ x a ) 2 f a, b − a )( y b) ( a, )
2!
+ ) 2 f ( , b) +  …(1)

Here, a = 1 and b =1.


f (x, y) = y x, f (1, 1) = 1
fx (x, y) = y x log y, fx (1, 1) = 0
fy (x, y) = x y x–1, fy (1, 1) = 1
fxx (x, y) = y x (log y)2, fxx (1, 1) = 0
fxy (x, y) = y x–1 + xy x–1 log y, fxy (1, 1) = 1
fyy (x, y) = x(x–1)y x–2, fyy (1, 1) = 0
Substituting these values in Eq. (1),
f x, y x + )( )]
1
+ x x )( y +( 1 0) + 
2!
yx − + (x 1) + 

Example 9
y
Expand tan −1 in Taylor series at (1, 1) up to second-degree terms.
x
Solution
Let 1 y
y) tan
x
By Taylor’s expansion,
1
f x, y a + [( x a a a + ( x a)2 f a, b
2!
+ )( ( y − b) (a ) +  …(1)

Here, a = 1 and b = 1.
-1 -1 p
f x, y ) , = ta 1
x 4
1 1
)
y x x y 2
1+
x2
8.8 Taylor’s Formula for Two Variables 8.89

1 1 x 1
f y ( x, y ) = 2
◊ = , f y (1, 1) =
y x x 2 + y2 2
1+
x2
y(2 x ) 2 xy 1
f xx ( x, y) = 2 2 2
= 2 2 2
, f xx (1, 1) =
((xx + y ) ((xx + y ) 2
2 2
( x + y )1 - x ◊ 2 x y2 - x 2
f xy ( x, y) = = , f xy (1, 1) = 0
( x 2 + y 2 )2 ( x 2 + y 2 )2
x ◊ 2y 2 xy 1
f yy ( x, y) = - 2 2 2
=- 2 2 2
, f yy (1, 1) = -
((xx + y ) ((xx + y ) 2
Substituting these values in Eq. (1),
π   1  1 
f ( x, yy)) = + ( x −1
− 1)  −  + ( y − 1)   
4   2  2 
1  1  1 
+  ((xx − 1) 2   + 2( x − 1)( y −
−11)(0) + ( y −1
− 1) 2  −   + 
2!   2   2 
y π 1 1 1 1
n −1
tan = − (x −
−11) + ( y −
−11) + ( x − 1) 2 − ( y −
−11) 2 + 
x 4 2 2 4 4

Example 10
Expand x2y + sin y + e x in Taylor’s series about (1, p ) up to second-
degree terms.
Solution
Let f (x, y) = x2y + sin y + ex
By Taylor’s expansion,
f ( x, y ) = f (a, b) + [( x − a)
a ) f x (a, b) + ( y − b) f y (a, b)]
1
+ ((xx − a ) 2 f xx (a, b) + 2( x − a )( y − b) f xy (a, b)
2! 
+(( y − b) 2 f yy (a, b)  + 
+ …(1)

Here, a = 1, b = p.
f (x, y) = x2y + sin y + e x, f (1, p ) = p + sin p + e = p + e
fx (x, y) = 2xy + e x, fx (1, p) = 2p + e
fy (x, y) = x2 + cos y, fy (1, p ) = 1 + cos p = 1 – 1 = 0
fxx (x, y) = 2y + e x, fxx (1, p ) = 2p + e
fxy (x, y) = 2x, fxy (1, p ) = 2
fyy (x, y) = –sin y, fyy (1, p ) = –sin p = 0
8.90 Chapter 8 Applications of Partial Derivatives

Substituting these values in Eq. (1),


(y )( )]
1
x +
2!
1
x2 y x 

Example 11
Expand in powers of h and k up to the second-degree terms.
y k
Solution

Let k) …(1)
y k
By Taylor’s expansion,
f x h +k x + hf k ( x y)
1 …(2)
+ h x + 2hk f (x y) k f ( x y) +
2
Putting h = 0 and k = 0 in Eq. (1),
xy
y)
x y
(x y2
f x, y )
(x y)2 ( x y)2
y x2
y) =
(x y)2 ( x y)2
2
2y
f x, y ) −
( x y )3
(x y)
y) 4
(x y)
2 y x
(x y)4
2 xy
( x + y )3
2x2
f x, y ) = −
( x y )3
8.8 Taylor’s Formula for Two Variables 8.91

Substituting these values in Eq. (2),

xy  y2 x2 
f ( x + h, y + k ) = + h + k 
x + y  ((xx + y ) 2 ( x + y)2 
(x
1  2 2 y2  2xy
2 xy 2  2 x2 
+  h  −  + 2 hk + k  − 3 
+
2 !   ((xx + y )3  ( x + y )3  ( x + y) 
( x + h)( y + k ) xy hy 2 + kx 2 h 2 y 2 − 2hkxy + k 2 x 2
= + − +
x+h+ y+k x + y ( x + y)2 ( x + y )3

Example 12
Expand e y log (1 + x) in powers of x and y upto third degree terms.
[Winter 2015]
Solution
Let f (x, y) = ey log (1 + x)
By Taylor’s expansion,
f ( x, y) = f (00,, 0) + ÈÎ x f x (0, 0) + y f y (0, 0)˘˚
1
+ ÈÎ x 2 f xx (0, 0) + 2 xy f xy (0 0) + y 2 f yy (0, 0)˘˚
2!
1
+ ÈÎ x 3 f xxx ((00, 0) + 3 x 2 y f xxy (0, 0) + 3 xxyy 2 f xyy (0, 0) + y3 f yyy (0, 0)˘˚ +  …(1)
3!
f (x, y) = ey log (1 + x), f (0, 0) = 0
ey
f x ( x, y ) = , f x (0, 0) = 1
1+ x
fy (x, y) = e y log (1 + x), fy (0, 0) = 0
y
e
f xx ( x, y) = - , f xx (0, 0) = -1
(1 + x )2
ey
f xy ( x, y) = , f xy (0, 0) = 1
1+ x
f yy ( x, y) = e y log(1 + x),
x f yy ((xx, y) = 0
2e y
xxx ( x, y ) =
f xxx , f xxx (0, 0) = 2
( + x )3
(1
ey
f xxy ( x, y) = - , f xxy (0, 0) = -1
( + x )2
(1
ey
f xyy ( x, y) = , f xyy (0, 0) = 1
1+ x
8.92 Chapter 8 Applications of Partial Derivatives

f x, y e +x ,0 =0

Substituting these values in Eq. (1),


1
f x, y x( ) y x - y (0 )

1
+ x +3 y xy 2 (1 0) + 
3!
1 1
ex x x x2 + ( x y2 ) +

x2 x3 x2 y y2
= - + + +
2 3 2 2

Example 13
Expand e x cos y in powers of x and y as far as the terms of the third degree.
Solution
Let f (x, y) = e x cos y
By Taylor’s expansion,
x f + )]
1
+ 0, 0)
2!
1
+ +3 2y 0, 0) + …(1)
3
f (x, y) = e x cos y, f (0, 0) = 1
fx (x, y) = e x cos y, fx (0, 0) = 1
fy (x, y) = – e x sin y, fy (0, 0) = 0
fxx (x, y) = e x cos y, fxx (0, 0) = 1
fxy (x, y) = – e x sin y, fxy (0, 0) = 0
fyy (x, y) = – e x cos y, fyy (0, 0) = –1
fxxx (x, y) = e x cos y, fxxx (0, 0) = 1
fxxy (x, y) = – e x sin y, fxxy (0, 0) = 0
fxyy (x, y) = – e x cos y, fxyy (0, 0) = –1
fyyy (x, y) = e x sin y, fyyy (0, 0) = 0
Substituting these values in Eq. (1),
1

1
x +
3
8.8 Taylor’s Formula for Two Variables 8.93

1 1
e x cos y = 1 + x + ( x 2 - y 2 ) + ( x 3 - 3 xy 2 ) + 
2 6

Example 14
Using the Taylor series, show that
1 1
log (1 + x + y) = (x + y) – ( x + y ) + ( x + y ) .
2 3

2 3
Solution
Let f (x) = log (1 + x + y)
By Taylor’s expansion,
f ( x, y ) = f (00,, 0) + [ x f x (0, 0) + y f y (0, 0)]
1 2
+ [ x f xx (00,, 0) + 22xy
xy f xy (0, 00)) + y 2 f yy (0, 0)]
2!
1
+ ÈÎ x 3 f xxx (0, 0) + 33xx 2 yyff xxy (0, 0) + 3 xxyy 2 f xyy (0, 00)) + y 3 f yyy (0, 0) ˘˚ +  …(1)
3!

f ( x, yy)) = log ((11 + x + y), f (0, 0) = 0


1
f x ( x, y ) = , f x (0, 0) = 1
1+ x + y
1
f y ( x, y ) = , f y (0, 0) = 1
1+ x + y
1
f xx ( x, y) = - , f xx (0, 0) = -1
1 + x + y )2
(1+
1
f xy ( x, y ) = − , 0, 0) = −1
f xy (0,
( + x + y)2
(1
1
f yy ( x, y) = - , f yy (0, 0) = -1
( + x + y )2
(1
2
f xxx ( x, y) = , f xxx (0, 0) = 2
( + x + y)3
(1
2
f xxy ( x, y) = , f xxy (0, 0) = 2
( + x + y)3
(1
2
f xyy ( x, y) = , f xyy (0, 0) = 2
( + x + y)3
(1
2
f yyy ( x, y) = , f yyy (0, 0) = 2
( + x + y)3
(1
8.94 Chapter 8 Applications of Partial Derivatives

Substituting these values in Eq. (1),

1
f x, y x )+ x2 + xy + )

1 3
+ x ( ) 2) + y 3 (2) + 
3!
1 1
log(1 + y) = + y) − y y2 + + y3 ) + 

1
= )− )2 + 3
+
2 3

Example 15
Find the Taylor series expansion of e x sin y up to third-degree terms.
[Winter 2014]
Solution
Let f (x, y) = e x sin y
By Taylor’s expansion,

f x, y a + [( x a a b (a b)]
1
+ x a ) 2 f a, b − a )( y − +( ) f ( b)
2
1
+ )3 f ( , x−a y b f ( a b)
3!
+ x a )( y a y f (a b) + …(1)

Here, a = –1 and b .
4
f x, y =0
f x, y 0
f x, y =1

f x, y =0

f x, y 1

f x, y e sin y f (0 ) 0
8.8 Taylor’s Formula for Two Variables 8.95

f x, y =0
f x, y 1

f x, y e sin y f (0 ) 0

f x, y e cos y f (0 ) -1

Substituting these values in Eq. (1),


1
f x, y x ) y( )] + x xy ( )
2
1
+ x x y x 1) +
3!
x y3
= xy + - +
2 6

Example 16
Find the Taylor’s series expansion of x 2 y 2 xy in powers of
(x + 2) and (y − 1) up to the third powers.
Solution
Let f x, y x x y2
By Taylor’s expansion,
f x, y a + [( x a ab b (a b)]
1
+ − )2 , −a y− + 2
f ( , b)
2
+ ( )( )2 f b)
3!
+ )3 f ( , b) +  …(1)

Here a = − 2 and b = 1.
) 6

1 4
( 6
1 10
4
8.96 Chapter 8 Applications of Partial Derivatives

f xxx ( x, y) = 0, f xxx (-2, 1) = 0


f xxy ( x, y) = 4 y + 4, f xxy (-2, 1) = 8
f xyy ( x, y) = 4 x + 6, f xyy (-2, 1) = -2
f yyy ( x, y) = 0, f yyy (-2, 1) = 0
Substituting these values in Eq. (1),
( )]
1
y 0 1) ( 4)
2!
1
3!
- y 1 0) + ◊◊◊
2 2
x y xy
2 2
+

EXERCISE 8.5
1. Expand e y in powers of x and y up to third-degree terms.
 1 
Ans : y x2 y +

2. Expand xy 2 + sin xy at the point 1, up to terms of second degree.


2
 Ans. : 
 
1 + π + π (  π π π
2 2 2
1
+ y− − ( − + − 1) y − + − + 
4 4 2 8 2 2 2 2

3. Expand e y about 1, up to the terms of second degree.


4
 e  π π 1
2

 Ans : 1 x − + x x ) y y− +
2 4 2 4
4. Obtain terms up to the third degree in the Taylor’s series expansion of
e y around the point 1,
2
Ans. :
1 2
 1 2

e x − )e + e + (x e 3e( +
2! 3 2
8.9 Jacobians 8.97

5. Expand x 2 + x
xyy + y 2 in powers of (x – 1) and (y – 2) up to second-degree terms.
 Ans. : (x − 1)2 + (y − 2)2 + ((x
x − 1)(
)(y − 2) + 4 x − 1) + 5(y − 2) + 7 
4(((x

6. Expand sin(x + h)(y + k) by Taylor’s theorem.


 1 
n xy + (hy
 Ans. : sin (hy + kx)cos xy + hk cos
cos xy − (hy + kx)2 sin
sin xy + 
2 

7. Expand x y about (1, 1) up to second-degree terms.


 1 
1+ (x − 1) + (x − 1)(y − 1) + [(x − 1)2 + 2(x − 1)(y − 1)] + 
 Ans. : 1 2! 
8. Using Taylor’s series, verify that
1 1
(i) cos(x + y ) = 1 − (x + y )2 + (x + y )4 − 
2! 4!
1
n−1(x + y ) = (x + y ) + (x + y )3 + 
(ii) tan
3

8.9 JACOBIANS

If u and v are continuous and differentiable functions of two independent variables


∂u ∂u
∂x ∂y
∂y
x and y, i.e., u = f1(x, y) and v = f2(x, y) then the determinant is called the
∂v ∂v
∂x ∂y
∂y
∂(u , v)
Jacobian of u, v with respect to x, y and is denoted as J = .
∂ ( x, y )
Similarly, if u, v and w are continuous and differentiable function of three independent
variables x, y, z then the Jacobian of u, v, w with respect to x, y, z is

∂u ∂u ∂u
∂x ∂y
∂y ∂z
∂(u , v, w) ∂v ∂v ∂v
=
∂ ( x, y , z ) ∂x ∂y
∂y ∂z
∂w ∂w ∂w
∂x ∂y
∂y ∂z

Jacobian is useful in transformation of variables from cartesian to polar, cylindrical


and spherical coordinates in multiple integrals.
8.98 Chapter 8 Applications of Partial Derivatives

Properties of Jacobians
Property 1 If u and v are functions of x and y then
∂(u , v) ∂ ( x, y )
J ◊ J* = 1 where J = and J * =
∂ ( x, y ) ∂(u , v)
Proof Let u and v be two functions of x and y.

u = f1 ( x, y)
y ) and v = f 2 ( x, y ) …(1)

Writing x and y in terms of u and v,

x = φ1 (u , vv)) and y = φ2 (u , v) …(2)

Differentiating Eq. (1) partially w.r.t. u and v,

∂u ∂f
∂f ∂x ∂f
∂f ∂y
∂y
= 1⋅ + 1⋅
∂u ∂x ∂u ∂y
∂y ∂u
∂u ∂x ∂u ∂y
∂y
1= ⋅ + ⋅ …(3)
∂x ∂u ∂y
∂y ∂u

∂u ∂f
∂f ∂x ∂f
∂f ∂y
∂y
= 1⋅ + 1⋅
∂v ∂x ∂v ∂y
∂y ∂v
∂u ∂x ∂u ∂y
∂y
0= ⋅ + ⋅ …(4)
∂x ∂v ∂y
∂y ∂v

∂v ∂f
∂f ∂x ∂f
∂f ∂y
∂y
= 2⋅ + 2⋅
∂u ∂x ∂u ∂y
∂y ∂u
∂v ∂x ∂v ∂y
∂y
0= ⋅ + ⋅ …(5)
∂x ∂u ∂y
∂y ∂u

∂v ∂f
∂f ∂x ∂f
∂f ∂y
∂y
= 2. + 2.
∂v ∂x ∂v ∂y
∂y ∂v
∂v ∂x ∂v ∂y
∂y
1= ⋅ + ⋅ …(6)
∂x ∂v ∂y
∂y ∂v

∂(u , v) ∂( x, y )
J ⋅ J* = ⋅
∂( x, y ) ∂(u , v)
∂u ∂u ∂x ∂x
∂x ∂y ∂u ∂v
=
∂v ∂v ∂y ∂y
∂x ∂y ∂u ∂v
8.9 Jacobians 8.99

∂u ∂u ∂x ∂y
∂y
∂x ∂y
∂y ∂u ∂u  Interchanging rows and columns
ns 
= of second determinant 
∂v ∂v ∂x ∂y
∂y  
∂x ∂y
∂y ∂v ∂v
∂u ∂x ∂u ∂y
∂y ∂u ∂x ∂u ∂y∂y
+ +
∂x ∂u ∂y
∂y ∂u ∂x ∂v ∂y∂y ∂v
=
∂v ∂x ∂v ∂y
∂y ∂v ∂x ∂v ∂y
∂y
+ +
∂x ∂u ∂y
∂y ∂u ∂x ∂v ∂y
∂y ∂v
1 0
= [[Substituting Eqs (33),
), (4), (55),
), (6)]
0 1
=1
Note: If u,v and w are functions of three variables x, y and z respectively then
∂(u , v, w) ∂( x, y, z )
⋅ =1
∂( x, y, z ) ∂(u , v, w)
Property 2 If u, v are functions of r, s and r, s are functions of x, y then
∂(u , v) ∂(u , v) ∂(r , s )
= ⋅ .
∂ ( x, y ) ∂ ( r , s ) ∂ ( x, y ) u v

Proof Let u, v be functions of r, s and r, s be functions


of x, y.
r s r s
∂u ∂u ∂r ∂r
∂(u , v) ∂(r , s ) ∂x ∂y
∂y
. = ∂r ∂s
∂ ( r , s ) ∂ ( x , y ) ∂v ∂v ∂s ∂s x,, y x,, y
∂r ∂s ∂x ∂y
∂y Fig. 8.3

∂u ∂u ∂r ∂s
 Interchanging rows 
∂x ∂x
= ∂r ∂s and columns
umns of
umns
umn 
∂v ∂v ∂r ∂s  
second determinant 
∂r ∂s ∂y∂y ∂y
∂y
∂u ∂r ∂u ∂s ∂u ∂r ∂u ∂s
+ +
∂r ∂x ∂s ∂x ∂r ∂y
∂y ∂s ∂y ∂y
=
∂v ∂r ∂v ∂s ∂v ∂r ∂v ∂s
+ +
∂r ∂x ∂s ∂x ∂r ∂y ∂s ∂y ∂y
∂u ∂u
∂x ∂y∂y
=
∂v ∂v
∂x ∂y∂y

∂(u , v)
=
∂ ( x, y )
8.100 Chapter 8 Applications of Partial Derivatives

Note: If u, v, w are functions of r, s, t and r, s, t are functions of x, y, z then


∂(u , v, w) ∂(u , v, w) ∂(r , s, t )
= ⋅ .
∂ ( x, y , z ) ∂ ( r , s , t ) ∂ ( x, y , z )

Example 1
∂(u , v)
Find the Jacobian for each of the following functions:
∂ ( x, y )

(i) u = x 2 − y 2 , v = 2 xy (ii) u = x sin y, v = y sin x


x+ y
(iii) u = , v = tan −−11 x + tan −1 y
1 − xy
Solution
(i) u = x2 − y2 v = 2xy
∂u ∂v
= 2x = 2y
∂x ∂x
∂u ∂v
= −2 y = 2x
∂y
∂y ∂∂yy
∂u ∂u
∂(u , v) ∂x ∂y
∂y
=
∂ ( x , y ) ∂v ∂v
∂x ∂y
∂y
2 x −2 y
=
2 y 2x
= 4((xx 2 + y 2 )
(ii) u = x sin y v = y sin x
∂u ∂v
= sin
ny = y cos x
∂x ∂x
∂u ∂v
= x cos y = sin x
∂y
∂y ∂y
∂y
∂u ∂u
∂(u , v) ∂x ∂y ∂y
=
∂ ( x , y ) ∂v ∂v
∂x ∂y∂y
sinn y x cos y
=
y cos
cos x sin x
= sin x sin y − xxyy coss x cos y
8.9 Jacobians 8.101

x+ y
(iii) u= v = tan −1 x + tan −1 y
1 − xy
∂u ( − xy ) − ( x + y )( − y )
(1 ∂v 1
= =
∂x ( − xy ) 2
(1 ∂x 1 + x 2
1+ y 2
=
( − xy ) 2
(1
∂u ( − xy ) − ( x + y )( −x
(1 − x) ∂v 1
= =
∂y
∂y ( − xy ) 2
(1 ∂∂yy 1 + y 2
1 + x2
=
( − xy ) 2
(1
∂u ∂u
∂(u , v) ∂x ∂y ∂y
=
∂ ( x , y ) ∂v ∂v
∂x ∂y ∂y
1+ y2 1 + x2
( − xy )
(1 2
( − xy ) 2
(1
=
1 1
1+ x 2
1+ y2
1 1
= −
1 − xy )
(1− 2
1 − xy ) 2
(1−
= 0.

Example 2
Find the Jacobian for the following functions:
x = a cosh q cos f, y = a sinh q sin f
Solution
x = a cosh θ cos φ h θ sin
y = a sinh sin φ
∂x ∂∂yy
= a sinh θ cos φ h θ sin φ
= a cosh
∂θ ∂θ
∂x ∂y
∂y
= − a cosh θ sin φ h θ co
= a sinh oss φ
∂φ ∂φ
∂( x, y)
J=
∂(q , f )
∂x ∂x
∂q ∂f
=
∂y ∂y
∂q ∂f
8.102 Chapter 8 Applications of Partial Derivatives

a sinh q cos f - a cosh q sin f


=
a cosh q sin f a sinh q cos f
= a 2 (sinh 2 θ cos 2 φ + cosh 2 θ sin
sin 2 φ )
= a 2 [sinh 2 θ (1 − si
sinn 2 φ ) + (1 + sinh 2 θ ) ssiin 2 φ ]
(sinh 2 θ + sin
= a 2 (sinh sin 2 φ )
a2
= (coshh 2θ − 1 + 1 − ccos
(cos os 2φ )
2
a2
= (cosh 2θ − ccoos 2φ )
(cosh
2

Example 3
∂(u , v)
If u = x2 – y2, v = 2xy, where x = r cos q and y = r sin q, find .
( , θ)
∂(r
Solution
u = x2 − y2 v = 2xy
∂u ∂v
= 2x = 2y
∂x ∂x
∂u ∂v
= −2 y = 2x
∂y
∂y ∂y
∂y

∂u ∂u
∂(u , v) ∂x ∂y ∂y
=
∂ ( x, y ) ∂ v ∂ v
∂x ∂y
∂y
2 x −2 y
=
2y 2x
= 4((xx 2 + y 2 )
= 4r 2
x = r cos θ y = r sin θ
∂x ∂y
= coss θ = sin θ
∂r ∂r
∂x ∂y

= − r sin = r cos θ
∂θ ∂θ
8.9 Jacobians 8.103

∂x ∂x
∂( x, y) ∂r ∂q
=
( , q ) ∂y
∂(r ∂y
∂r ∂q
q
cosssq -r sin q
=
sin
nq
n q cos q
r cos
= r cos2 q + r ssiin 2 q
=r
Hence, ∂(u, v) ∂(u, v) ∂( x, y)
= ◊
∂(r , q ) ∂( x, y) ∂(r , q )
= 4r 2 ◊ r
= 4r 3

Example 4
Find the Jacobian for each of the following functions:
(i) u = xyz , v = x2 + y 2 + z 2 , w = x + y + z
(ii) x = r sin θ cos φ , y = r sin θ sin
sin φ , z = r cos θ
[Winter 2016; Summer 2015]
Solution
(i) u = xyz v = x2 + y 2 + z 2 w= x+ y+ z
∂u ∂v ∂w
= yz = 2x =1
∂x ∂x ∂x
∂u ∂v ∂w
= xz = 2y =1
∂y ∂y ∂y
∂u ∂v ∂w
= xy = 2z =1
∂z ∂z ∂z

∂(u, v, w)
J=
∂( x, y, z )
∂u ∂u ∂u
∂x ∂y ∂z
∂v ∂v ∂v
=
∂x ∂y ∂z
∂w ∂w ∂w
∂x ∂y ∂z
8.104 Chapter 8 Applications of Partial Derivatives

yz xz xy
= 2x 2 y 2z
1 1 1
= yz (2 y - 2 z ) - xz (2 x - 2 z ) + xy (2 x - 2 y )
= 2 y 2 z - 2 yz 2 - 2 x 2 z + 2 xz 2 + 2 x 2 y - 2 xy 2
2xz
= 2[ yz ( y - z ) + x 2 (- z + yy)) - x( y 2 - z 2 )]
= 2( y - z )[ yz + x 2 - x( y + z )]
= 2( y - z )[ x 2 - x( y + z ) + yz ]
= 2( y - z )[ x( x - z ) - y ( x - z )]
= 2( y - z)(
z )( x - z )( x - y )
(ii) x = r sin θ cos φ n θ sin
y = r sin sin φ z = r cos θ
∂x ∂∂yy ∂z
n θ cos
= sin cos φ n θ sin
= sin sin φ = cos θ
∂r ∂r ∂r
∂x ∂y
∂y ∂z
= r coss θ cos
cos φ = r cos θ sin φ = − r sin θ
∂θ ∂θ ∂θ
∂x ∂y
∂y ∂z
= − r sinn θ sin φ n θ cos φ
= r sin =0
∂φ ∂φ ∂φ

∂x ∂x ∂x
∂r ∂θ ∂φ
∂ ( x , y , z ) ∂y ∂y ∂y
J= =
∂ ( r , θ , φ ) ∂r ∂θ ∂φ
∂z ∂z ∂z
∂r ∂θ ∂φ
sin θ cos
cos φ r cos
cos θ cos
cos φ n θ sin
− r sin sin φ
n θ sin
= sin sin φ r co
coss θ sin
sin φ r sinn θ cos
cos φ
cos θ − r ssiin θ 0
n θ cos
sin cos φ coss θ cos
cos φ n θ sin
− sin sin φ
n θ sin
= r sin2
sin φ coss θ sin
sin φ n θ cos
sin cos φ
os θ
cco sin θ
− sin 0

= r 2 [cos θ (cos θ sin θ cos 2 φ + sinn θ cos


cos θ sinn 2 φ )
+ sin θ (si
sinn 2 θ cos 2 φ + sin
sin 2 θ sin 2 φ )]
= r (sin θ cos 2 θ + sin 3 θ )
2

= r2 sin q (cos2 q + sin2 q)


= r2 sin q
8.9 Jacobians 8.105

Example 5
x y z ∂(u , v, w)
If u = , v= w= , find .
y−z z − x, x− y ∂ ( x, y , z )

Solution
Taking logarithms,
g u = log
log log x − log( y − z ), g v = log
log log y − log( z − x), logg w = log
log z − log( x − y )

1 ∂u 1 1 ∂v 1 1 ∂w 1
= = =−
u ∂x x v ∂x z−x w ∂x x− y
∂u u ∂v v ∂w w
= = =−
∂x x ∂x z−x ∂x x− y
1 ∂u 1 1 ∂v 1
=− = 1 ∂w 1
u ∂y y−z v ∂y y =
w ∂y x− y
∂u u ∂v v
=− = ∂w w
∂y y−z ∂y y =
∂y x− y
1 ∂u 1 1 ∂v 1
= =− 1 ∂w 1
u ∂z y−z v ∂z z−x =
w ∂z z
∂u u ∂v v ∂w
= =− =
w
∂z y−z ∂z z−x ∂z z

∂u ∂u ∂u
∂x ∂y
∂y ∂z
∂(u , v, w) ∂v ∂v ∂v
=
∂ ( x, y , z ) ∂x ∂y
∂y ∂z
∂w ∂w ∂w
∂x ∂y
∂y ∂z
u u u

x y−z y−z
v v v
= −
z−x y z−x
w w w

x− y x− y z
8.106 Chapter 8 Applications of Partial Derivatives

1 1 1

x y−z y−z
1 1 1
= uvw −
z−x y z−x
1 1 1

x− y x− y z
y−z
−1 1
x
uvw z−x
= 1 −1
( x − y )( y − z )( z − x) y
x− y
−1 1
z
=0

Example 6
∂ ( x, y , z )
If x + y + z = u,
u y + z = uv, z = uvw, prove that = u 2 v.
∂(u , v, w)
Solution
x+ y+ z = u
y + z = uv
z = uvw
x = u(
u (1 − v) y = uv
uv − u
uvw z = uuvw
= uv(
uv(1 − w)
∂x ∂∂yy ∂z
= 1− v = v(1 − w) = vw
∂u ∂u ∂u
∂x ∂y
∂y ∂z
= −u = u (1 − w) = uw
∂v ∂v ∂v
∂x ∂y
∂y ∂z
=0 = −uv = uv
∂w ∂w ∂w
∂x ∂x ∂x
∂u ∂v ∂w 1− v −u 0
∂( x, y, z ) ∂y∂y ∂y∂y ∂y
∂y
= = v(1 − w) uu((1 − w) −uv
∂(u , v, w) ∂u ∂v ∂w
vw uw uv
∂z ∂z ∂z
∂u ∂v ∂w
1 0 0
= v − vw u − uw −uv [By R1 + R2 + R3 ]
[By
vw uw uv
8.9 Jacobians 8.107

= 1[(u - uw)(uv) + (uv)(uw)]


= u2 v - u2 vw
w + u2 vw
= u2 v

Example 7 ∂ ( x, y )
If x = a (u + v), y = b(u − v) and u = r 2 cos 2θ , v = r 2 si
sinn 2θ , find .
∂(r , θ )
Solution
y = b(
b(u − v)
x = a(
a (u + v)
∂x ∂y
∂y
=a =b
∂u ∂u
∂x ∂y
∂y
=a = −b
∂v ∂v
∂x ∂x
∂ ( x , y ) ∂u ∂v
=
∂(u , v) ∂∂yy ∂y∂y
∂u ∂v
a a
=
b −b
= −2ab

u = r 2 cos 2θ v = r 2 sin 2θ
∂u ∂v
= 2r cos 2θ = 2r sin 2θ
∂r ∂r
∂u ∂v
= −2r 2 sin 2θ = 2r 2 cos 2θ
∂θ ∂θ
∂u ∂u
∂(u , v) ∂r ∂θ
=
( , θ ) ∂v
∂(r ∂v
∂r ∂θ
cos 2θ
2r cos in 2θ
−2r 2 ssin
=
sin 2θ
2r sin 2r cos 2θ
2

4r 3 (cos 2 2θ + sin
=4 sin 2 2θ )
= 4r 3
∂( x, y ) ∂( x, y ) ∂(u , v)
Hence, = ⋅
∂(r , θ ) ∂(u , v) ∂(r , θ )
= ( −2ab) (4r 3 )
= −8abr 3
8.108 Chapter 8 Applications of Partial Derivatives

Example 8
If x = vw , y = uw , z = uv and u = r sinq cosf, v = r sinq sinf,
∂ ( x, y , z )
w = r cosq, find .
∂(r , θ , φ )
Solution
x = vw y = uw z = uv
∂x ∂y 1 w ∂z 1 v
=0 = =
∂u ∂u 2 u ∂u 2 u
∂x 1 w ∂∂yy ∂z 1 u
= =0 =
∂v 2 v ∂v ∂v 2 v
∂x 1 v ∂y 1 u
∂y ∂z
= = =0
∂w 2 w ∂w 2 w ∂w
∂x ∂x ∂x
∂u ∂v ∂w
∂ ( x , y , z ) ∂y ∂y ∂y
=
∂(uu , v, w) ∂u ∂v ∂w
∂z ∂z ∂z
∂u ∂v ∂w
1 w 1 v
0
2 v 2 w
1 w 1 u
= 0
2 u 2 w
1 v 1 u
0
2 u 2 v
1 w  1 v  1 v  1 w
=− − +
2 v  4 w  2 w  4 v 
1
=
4
u = r sin q cos f v = r sin q sin f w = r cos q
∂u ∂v ∂w
= sin q cos f = sin q sin f = cos q
∂r ∂r ∂r
∂u ∂v ∂w
= r cos q cos f = r cos q sin f = - r sin q
∂q ∂q ∂q
∂u ∂v ∂w
= - r sin q sin f = r sin q cos f =0
∂f ∂f ∂f
8.9 Jacobians 8.109

∂u ∂u ∂u
∂r ∂θ ∂φ
∂(u , v, w) ∂v ∂v ∂v
and =
( , θ, φ)
∂(r ∂r ∂θ ∂φ
∂w ∂w ∂w
∂r ∂θ ∂φ
sinn θ cos
cos φ r cos
cos θ cos
cos φ − r sinn θ sin
sin φ
= sinn θ sin
sin φ r co
coss θ sin
sin φ r sinn θ cos
cos φ
cos θ − r sin θ 0
= sin θ cos φ (r sin
sin θ cos φ ) − r coss θ cos
2 2
cos φ ( −r
− r sin θ cos
cos θ cos φ )
− r ssiin θ sin
sin φ ((− sin 2 θ sin
−rr sin
− sin φ − r cos
cos 2 θ sin
sin φ )
= r 2 sin θ
∂( x, y, z ) ∂( x, y, z ) ∂(u , v, w) 1 2
Hence, = ⋅ = r sin θ
( , θ , φ ) ∂(u , v, w) ∂(r
∂(r ( , θ, φ) 4

Example 9
u
Verify JJ* = 1 for the transformation x = uv, y = .
v
Solution
u
x = uv y=
v
∂x ∂∂yy 1
=v =
∂u ∂u v
∂x ∂y
∂y u
=u =− 2
∂v ∂v v
∂( x, y)
y
J=
∂(u , v)
v
∂x ∂y
∂u ∂u
=
∂x ∂y
∂v ∂v
v u
= 1 u
− 2
v v
u u
=− −
v v
2u
=−
v
8.110 Chapter 8 Applications of Partial Derivatives

Expressing u and v in terms of x, y,

xy = u 2 x
= v2
y

u= x y x
v=
y

∂u y 1 ∂v 1  1  1
= = =  =
∂ x 2 x 2v ∂x y  2 x  2u

∂u x v ∂v  1 −3  1 x
= = = x − y 2 = −
∂y 2 y 2 ∂y  2  2y y
v v2
=− =−
2y 2u

∂(u , v)
v
J* =
∂( x, y)
y
∂u ∂u
∂x ∂y
=
∂v ∂v
∂x ∂y
1 v
2v 2
=
1 v2

2u 2u
v v
=− −
4u 4u
v
=−
2u

 2u   v 
Hence, J ⋅ J =  −   −  = 1
*
 v   2u 

Example 10
For the transformations x = ev sec u , y = ev tan u , prove that
∂( x, y ) ∂(u , v)
= 1.
∂(u , v) ∂( x, y )

Solution
x = ev sec u y = ev tan u
8.9 Jacobians 8.111

∂x ∂y
∂y
= ev sec u tan u = ev sec 2 u
∂u ∂u
∂x ∂y
∂y
= ev sec u = ev tan u
∂v ∂v

∂x ∂x
∂ ( x , y ) ∂u ∂v
=
∂(u , v) ∂∂yy ∂y
∂y
∂u ∂v
ev sec tan u ev sec u
sec u tan
=
ev sec
sec 2 u ev tan u

= e 2 v (sec u tan 2 u − sec3 u )


= −e 2 v sec u

Writing u, and v in terms of x, y,


y x 2 − y 2 = e2v
= sin u
x 2v = log( x 2 − y 2 )
 y 1
u = sin −1   v= log( x 2 − y 2 )
 x 2
∂u y
=− ∂v x
∂x x x − y2
2 = 2
∂x x − y 2
∂u 1 ∂v y
= =− 2
∂y
∂y x − y2
2
∂y
∂y x − y2

∂u ∂u
∂(u , v) ∂x ∂y
∂y
=
∂ ( x , y ) ∂v ∂v
∂x ∂y
∂y
y 1

x x −y2 2
x − y2
2
=
x y

x − y2
2
x − y2
2

y
1 − 1
= 3 x
((xx 2 − y )
2 2
x −y
1  y2 
= 3  x − x
((xx 2 − y )
2 2
8.112 Chapter 8 Applications of Partial Derivatives

1  y 2 − x2 
=  
x 
3
((xx 2 − y ) 2 2

1
=− 1
x( x 2 − y 2 ) 2
x(
1
=− v
xe
1
= − 2v
e sec
sec u
∂( x, y ) ∂(u , v)  1 
Hence, = ( −e 2 v sec
sec u )  − 2 v 
∂(u , v) ∂( x, y )  e sec sec u 
=1

Example 11
Verify J ◊ J * = 1 for the following function:
x = u, y = u tan v, z = w
Solution
x=u y = u tan v z=w
∂x ∂∂yy ∂z
=1 = tan v =0
∂u ∂u ∂u
∂x ∂y
∂y ∂z
=0 = u sec 2 v =0
∂v ∂v ∂v
∂x ∂y
∂y ∂z
=0 =0 =1
∂w ∂w ∂w

∂ ( x, y , z )
J=
∂(u , v, w)
∂x ∂x ∂x
∂u ∂v ∂w
∂y ∂y ∂y
=
∂u ∂v ∂w
∂z ∂z ∂z
∂u ∂v ∂w
1 0 0
= tann v u sec v 0
2

0 0 1
= u sec
sec v2
8.9 Jacobians 8.113

Writing u, v, w in terms of x, y and z,


y y
u=x tan v = = w= z
u x
 y
v = tan −1  
 x

∂u ∂v y ∂w
=1 =− 2 =0
∂x ∂x x + y2 ∂x
∂u ∂v x ∂w
=0 = =0
∂y
∂y ∂y x 2 + y 2
∂y ∂y
∂y
∂u ∂v ∂w
=0 =0 =1
∂z ∂z ∂z
∂(u , v,
v, w)
J* =
∂( x, y,
y, z )
∂u ∂u ∂u
∂x ∂y ∂z
∂v ∂v ∂v
=
∂x ∂y ∂z
∂w ∂w ∂w
∂x ∂y ∂z
1 0 0
y x
= − 2 0
x + y2 x2 + y 2
0 0 1

x
=
x + y2
2

u
= 2
u + u 2 tan 2 v
1
=
u sec 2 v
∂( x, y,
y, z ) ∂(u , v,
v, w) 1
Hence, J ⋅ J* = ⋅ = u sec 2 v ⋅ =1
∂(u , v,
v, w) ∂( x, y,
y, z ) u sec 2 v

EXERCISE 8.6

∂(u, v )
1. Find the Jacobian for each of the following functions:
∂(x, y )
(i) u = x + y, v = x − y (ii) u = x2, v = y2
8.114 Chapter 8 Applications of Partial Derivatives

y−x
(iii) u = 3x + 5y, v = 4x − 3y (iv) u = , v = tan−1 y − tan−1 x
1 + xy
(v) u = x sin y, v = y sin x.
 Ans.: 
 
 (i) −1 (ii) 4 xy (iii) − 29
4xy (iv) 0 (v
(iv ( ) sin
sin x ssin cos y 
in y − xy coss x cos
 2 
2. Find the Jacobian for each of the following functions:
(i) x = eu cos v, y = eu sin v (ii) x = u(1 − v), y = uv
u+v
(iii) x = uv
uv , y=
u−v
 2u 4uv 
 Ans.: (i) e (ii) u (iii) 
 u − v )2 
((u

3. Find the Jacobian for each of the following functions:


yz zx xy
(i) u = ,v = ,w = (ii) u = xyz, v = xy + yz + zx, w = x
x y z +y+z
1
(iii) u = x 2, v = sin y, w = e−3z (iv) x = (u 2 − v 2 ), y = uv , z = w
2

 1 
 Ans.: (i) 4 (ii) (x − y )(
)(y − z))((z − x) (iii) − 6e −3 z x coss y (iv)
 u + v 2 
2

4. Verify that J ◊ J* = 1 for the following functions:


y2 y2
(i) u = x + , v =
x x
(ii) x = u(1 − v), y = uv
(iii) x = sin q cos f, y = sin q sin f
∂(x, y , z)
5. If u = x + y + z, uv = y + z, uvw = z, evaluate .
∂(u, v , w )
[Ans. : u 2 v]
∂(u, v ) 1 (y 2 − x 2 )
6. If u3 + v3 = x + y, u2 + v2 = x3 + y3, show that = .
∂(x, y ) 2 uv((u − v )
∂(u, v , w ) x y z
7. Calculate if u = ,v = ,w = where
∂(x, y , z) 1− r 2
1− r 2
1− r2
r2 = x2 + y2 +z2.
 − 
5

 Ans. : ((1 − r ) 2 
2

∂(u, v , w )
8. If u = x + y + z, u2v = y + z, u3w = z, show that = u −5 .
∂(x, y , z)
Points to Remember 8.115

∂(u, v )
9. Show that sin 2θ, if u = x2 − 2y2, v = 2x2 − y2 and x = r cos q,
= 6r 3 sin
∂(r, θ)
y = r sin q.

POINTS TO REMEMBER

• Tangent Plane and Normal to a Surface

The equation of the tangent plane at P(x0, y0, z0) to the surface
f (x, y, z) = 0 is
(x − x0) fx(x0, y0, z0) + (y − y0) fy (x0, y0, z0) + (z − z0) fz (x0, y0, z0) = 0
The equations of the normal line to the surface at P(x0, y0, z0) are
x − x0 y − y0 z − z0
= =
fx (x 0 , y 0 , z0 ) fy (x 0 , y 0 , z0 ) fz (x 0 , y 0 , z0 )

• Linear Approximation or Linearization


Linear approximation or linearization of f (x, y) near the point
(x0, y0) is
f (x, yy))  f (x 0 , y 0 ) + fx (x 0 , y 0 )(x − x 0 ) + fy (x 0 , y 0 )(
)((yy − y 0 )

Linear approximation or linearization of f (x, y, z) near the point


(x0, y0, z0) is
f (x, y , z)  L(x, y , z) = ff((x 0 , y 0 , z0 ) + fx (x 0 , y 0 , z0 )(x − x 0 )
+ fy (x 0 , y 0 , z0 )(y − y 0 ) + fz (x 0 , y 0 , z0 )(z − z0 )

• Maximum and Minimum Values by Second Derivative Test


To determine the maxima and minima (extreme values) of a function
f (x, y)
∂f ∂f
Step 1: Solve =0 and = 0 simultaneously for x and y.
∂x ∂y
∂2 f ∂2 f ∂2 f
Step 2: Obtain the values of r = ,s = ,t = 2 .
∂x 2
∂x ∂y ∂y
Step 3: (i) If rt – s > 0 and r < 0 (or t < 0) at (a, b) then f (x, y) is
2

maximum at (a, b) and the maximum value of the function


is f (a, b).

(Contd )
8.116 Chapter 8 Applications of Partial Derivatives

(ii) If rt – s2 > 0 and r > 0 (or t > 0) at (a, b) then f (x, y) is


minimum at (a, b) and the minimum value of the function
is f (a, b).
(iii) If rt – s2 < 0 at (a, b) then f (x, y) is neither maximum nor
minimum at (a, b).
(iv) If rt – s2 = 0 at (a, b) then no conclusion can be made
about the extreme values of f (x, y).

• Method of Lagrangian Multipliers


Let f (x, y, z) be a function of three variables x, y, z, and the
variables be connected by the relation
φ(x, y , z) = 0
Let f (x, y, z) + lf (x, y, z) = 0 be an auxiliary equation.
Differentiating this equation partially w.r.t x, y and z
∂f ∂φ
+λ =0
∂x ∂x
∂f ∂φ
+λ =0
∂y ∂y
∂f ∂φ
+λ =0
∂z ∂z
Eliminating l from these equations, the values of x, y and z are
obtained for which f (x, y, z) has a stationary (maximum and
minimum) value.
• Taylor's Formula for Two Variables
If f (x + h, y + k) be a given function which can be expanded into a series
of positive ascending powers of h and k then
 ∂ ∂
f (x + h, y + k) = f (x, y ) +  h + k  f ( x, y )
 ∂x ∂y 
2 3
1 ∂ ∂ 1 ∂ ∂
+ h + k  f ( x, y ) +  h + k  f (x,
x, y ) + 
2!  ∂x ∂y  3!  ∂x ∂y 
Also,
f (x, y ) = f (a, b) + (x − a)fx (a, b
b)) + (y − b)fy (a, b)
1
+ [(x − a)2 fxx (a, b) + 2((x
x − a)(y − b)fxy (a, b)
2!
1
+(((yy − b)2 fyy (a, b
+ )] + (x − a)3 fxxx (a, b)
b)]
3!
+3
+ x − a)2 (y − b)fxxy (a, b
3((x b)) + 3(x − a)(
a)(y − b
b))2 fxy
xyyy (a, b)
Multiple Choice Questions 8.117

(Contd )

• Jacobian
If u, v are functions of x, y then the Jacobian is
∂u ∂u
∂(u, v)
v ∂x ∂y
J= =
∂(x, y)
y ∂v ∂v
∂x ∂y
If u, v, w are functions of x, y, z then the Jacobian is
∂u ∂u ∂u
∂x ∂y ∂z
∂(u, v,
v, w) ∂v ∂v ∂v
J= =
∂(x, y,
y , z) ∂x ∂y ∂z
∂w ∂w ∂w
∂x ∂y ∂z
Property 1. If u and v are functions of x and y then
∂(u, v)
v ∂(x, y)
y
J ◊ J* = 1 where J = and J * =
∂(x, y)
y ∂(u, v)
v
Property 2. Chain Rule: If u, v are functions of r, s and r, s are functions
of x, y then
∂(u, v ) ∂(u, v ) ∂(r, s)
= ⋅ .
∂(x, y ) ∂(r, s) ∂(x, y )

MULTIPLE CHOICE QUESTIONS

Choose the correct alternative in the following questions:


∂( x , y )
1. If x = u(1–v), y = uv then =
∂(u, v)
(a) 1 (b) u (c) v (d) uv
∂( x, y, z )
2. If x = r cosq, y = r sinq, z = 2 then =
∂(r , q , z )
1 1
(a) (b) r (c) r2 (d)
2 r2
∂(r , q )
3. If x = r cosq, y = r sinq then = [Summer 2016, 2015]
∂( x , y )
1 1
(a) r (b) –r (c) (d) -
r r
8.118 Chapter 8 Applications of Partial Derivatives

∂( x , y )
4. If x + y = 2eq cos f, x–y = 2ieq sin f then =
∂(q , f )
(a) –2 ie2q (b) 2 ie2q (c) 2e2q (d) 2eq
5. The functions of y1 = f(x, y); y2 = g(x, y) are functionally dependent if and only
if
∂( y1 , y2 ) ∂( y1 , y2 )
(a) =0 (b) π0
∂( x , y ) ∂( x, y )
∂( y1 , x ) ∂( y1 , x )
(c) =0 (d) π0
∂( y2 , y) ∂( y2 , y)
x x+y
6. The functions u = ;v= are functionally
y x-y
(a) dependent by the relation u + v = 0
u +1
(b) dependent by the relation v =
(c) independent u -1
∂(u, v)
(d) independent because π0
∂( x , y )
∂( x , y ) ∂(r , q )
7. If J = and J * = then the value of JJ* is [Winter 2015]
∂(r , q ) ∂( x , y )
(a) 1 (b) 0 (c) –1 (d) •
∂ ( u, v )
8. If u = x2 –2y, v = x + y then =
∂( x , y )
(a) (x + 1)2 (b) 2 (x + 1) (c) xy (d) x + 1
∂(u, v) ∂( x, y )
9. If u = then =
∂( x , y ) ∂(u, v)
1
(a) u (b) (c) 1 (d) u2
u
x+y ∂(u, v) ∂( x, y)
10. If u = , v = tan–1x + tan–1y then . =
1 - xy ∂( x, y) ∂(u, v)
(a) 0 (b) 1 (c) xy (d) x2y
y Ê u, v ˆ
11. If u = , v = xy then J Á =
x Ë x, y ˜¯
2y 2x 2y 2x
(a) - (b) - (c) (d)
x y x y
∂(u, v)
12. If u = x2y, v = xy2 then is
∂( x , y )
(a) 3x2y2 (b) 4x2y2 (c) 2x2y2 (d) x2y2
Multiple Choice Questions 8.119

∂(u, v)
13. The Jacobian for the function u = ex sin y, v = ex cos y is
∂( x , y )
(a) 0 (b) sin x sin y – xy cos x cos y
ex
(c) – ex (d)
x
14. If the function u, v, w of three independent variables x,y,z are not independent
then the Jacobian of u, v, w w.r.t. x, y, z is always equal to
(a) 1 (b) 0
(c) • (d) Jacobian of x, y, z w.r.t. u, v, w
15. If f (x, y) = x2 + y2 + 3, the minimum value of f(x, y) is
(a) 3 (b) • (c) 0 (d) 1
16. The stationary points of x3 + y3 – 3axy are
(a) (0, 0), (a, a) (b) (0, 0), (a, 0) (c) (a, 0), (0, – a) (d) (0, a), (a, 0)
17. If f(x,y) = xy + (x – y), the stationary points are
(a) (0, 0) (b) (1, –1) (c) (1, 2) (d) (1, –2)
18. The stationary points of x4 + y4 – 2x2 + 4xy – 2y2 are
(a) ( 2, 2 ) (b) ( - 2, - 2 ) (c) ( 2, - 2 ) (d) (0, 2 )
19. In a plane triangle ABC, the maximum value of cos A cos B cos C is
1 3 3
(a) 0 (b) (c) (d)
8 8 8
2 2
20. For the function f(x,y) = x + y + 6x + 12, minima occurs at
(a) (0, 3) (b) (–3, 0) (c) (3, 0) (d) (0, –3)
21. The function f(x, y) = 2x2 + 2xy – y3 has
(a) only one stationary point at (0, 0)
Ê 1 1ˆ
(b) two stationary points at (0, 0) and Á , - ˜
Ë 6 3¯
(c) two stationary points at (0, 0) and (1, –1)
(d) no stationary points
1 18
22. The function z = 5 xy - 4 x 2 + y 2 - 2 x - y + 5 has at x = ,y =
41 41
(a) maxima (b) saddle point (c) minima (d) no conclusion
23. With usual notations, the properties of maxima and minima under various
conditions are
I II
(P) Maxima (i) rt – s2 = 0
(Q) Minima (ii) rt – s2 < 0
(R) Saddle Point (iii) rt – s2 > 0, r > 0
(S) Failure Case (iv) rt – s2 > 0, r < 0
(a) P – i, Q – iii, R – iv, S – ii (b) P – ii Q – i, R – iii, S – iv
(c) P–iv, Q – iii, R–ii, S – i (d) P – iv, Q – ii, R – i, S – iii
8.120 Chapter 8 Applications of Partial Derivatives

24. The minimum value of f(x,y) = x2y2 is [Winter 2015]


(a) 1 (b) 2 (c) 4 (d) no conclusion
25. The sum of the squares of two positive numbers is 200, their minimum product
is
(a) 200 (b) 25 7 (c) 28 (d) 0
26. The coefficient of (x + y)3 in the expansion of sin (x + y) is
1 1 1 1
(a) (b) (c) - (d) -
3 6 6 3
y
27. The coefficient of x – 1 in the expansion of x near the point (1, 1) is
(a) 1 (b) –1 (c) 0 (d 2
28. The coefficients of x – 1 and y + 1 in the expansion of ex + y upto first degree terms
are
(a) (1, 1) (b) (1, –1) (c) (–1, 1) (d) (0, 1)
29. The coefficient of (x – 1)2 in the expansion of sin xy in powers of (x – 1) and
Ê pˆ
ÁË y - 2 ˜¯ is

p -p p2 -p 2
(a) (b) (c) (d)
8 8 8 8
30. The minimum value of x2 + y2 + z2 given that xy + yz + zx = 3a2 is
1 2
(a) 3a (b) 4a2 (c) a (d) 3a2
3
31. For the auxiliary equation F ( x, y, z ) = f ( x, y, z ) + lf ( x, y, z ) = 0, the Lagrange’s
equations are [Winter 2014]
∂F ∂F ∂F ∂f ∂f ∂f
(a) = 0, = 0, =0 (b) = 0, = 0, =0
∂x ∂y ∂z ∂x ∂y ∂z
∂f ∂f ∂f ∂f ∂f ∂f
(c) = 0, = 0, =0 (d) = 0, = 0, =0
∂x ∂y ∂z ∂x ∂f ∂z
32. The equation of the tangent plane of z = x at (2, 0, 2) is
(a) z = x (b) x + y + z = 2 (c) z + x = 0 (d) x + y = 2
33. A point (a, b) is said to be saddle point if at (a, b) [Summer 2016]
(a) rt – s2 > 0 (b) rt – s2 = 0 (c) rt – s2 < 0 (d) rt – s2 ≥ 0
34. The minimum value of f(x, y) = x2 + y2 is [Winter 2016]
(a) 1 (b) 2 (c) 4 (d) 0
∂( x, y)
35. If x = u + 3v, y = –u + v then J = is [Winter 2016; Summer 2017]
∂(u, v)
(a) –1 (b) 4 (c) 5 (d) 7
Answers 8.121

Answers

1.(b) 2.(b) 3.(c) 4.(a) 5.(a) 6.(b) 7.(a) 8.(b) 9.(b) 10.(a)
11.(a) 12.(a) 13.(c) 14.(b) 15.(a) 16.(a) 17.(b) 18.(c) 19.(b) 20.(b)
21.(b) 22.(b) 23.(c) 24.(d) 25.(d) 26.(c) 27.(a) 28.(a) 29.(d) 30.(d)
31.(a) 32.(a) 33.(b) 34.(d) 35.(b)
Unit 6
Multiple Integrals
CHAPTER
Multiple
9
Integrals
9.1 INTRODUCTION

Integration of functions of two or more variables is normally called multiple


integration. The particular case of integration of functions of two variables is called
double integration and that of three variables is called triple integration. Sometimes,
we have to change the variables to simplify the integrand while evaluating the multiple
integrals. Variables can be changed by substitution or by changing the coordinate
system (polar, spherical or cylindrical coordinates). Multiple integrals are useful in
evaluating plane area, mass of a lamina, mass and volume of solid regions, etc.

9.2 DOUBLE INTEGRALS OVER RECTANGLES

Let f ((x, y) be a continuous function defined in a closed and bounded region R in


the xy-plane. Divide the region R into small elementary rectangles by drawing lines
parallel to coordinate axes. Let the total number of complete rectangles which lie
inside the region R be n. Let dA
d r be the area of rth rectangle and (xr, yr) be any point
in this rectangle.
n
Consider the sum S = ∑ f ( xr , yr )δ Ar …(1)
r =1

where d Ar = d xr◊d yr
If we increase the number of elementary rectangles y
then the area of each rectangle decreases. Hence, as dA
A
n Æ •, d Ar Æ 0. The limit of the sum given by the
Eq. (1), if it exists, is called the double integral of f (x, y)
over the region R and is denoted by ∫∫ f ( x, yy))d
R
)d A.

n O x
Hence, ∫∫ f ( x, yy)))dd A = lim ∑ f ( x , y )δ A
R
n→∞
δ Ar →0 r =1
r r r Fig. 9.1

where d A = d x dy
dy
9.2 Chapter 9 Multiple Integrals

9.2.1 Evaluation of Double Integrals by Fubini's Theorem


Double integral of a function f (x, y) over a region R can be evaluated by two successive
integrations. There are two different methods to evaluate a double integral which are
known as Fubini's theorem.
Method-I Let the region R, i.e., PQRS be
y
bounded by the curves y = y1 (x), y = y2 (x)
and the lines x = a, x = b.
In the region PQRS, draw a vertical strip AB. y = y2 (x))
B R
Along the strip AB, y varies from y1 to y2 and S
x is fixed. Therefore, the double integral is
integrated first w.r.t. y between the limits y1 x=a x=b
and y2 treating x as constant.
Now, move the strip AB horizontally from P
PS (i.e., x = a) to QR (i.e., x = b) to cover A Q
y = y1 (x))
the entire region PQRS. The result of the
first integral is integrated w.r.t. x between the O x
limits a and b. Hence, Fig. 9.2

∫∫ f ( x, yy))d x ddyy = ∫ ∫ f ( x, yy))dy  d x


b y2 ( x )
y
R
a y1 ( x ) 

Method-II Let the region R, i.e., PQRS be


bounded by the curves x = x1( y), x = x2( y) S y=d R
and the lines y = c, y = d.
In the region PQRS, draw a horizontal strip A B
x =x1(y ) x = x2 (y)
AB. Along the strip AB, x varies from x1 to x2
P y=c Q
and y is fixed. Therefore, the double integral
is integrated first w.r.t. x between the limits x1
and x2 treating y as constant. O x

Now, move the strip AB vertically from PQ


Fig. 9.3
(i.e., y = c) to RS (i.e., y = d ) to cover the
entire region PQRS. The result of the first
integral is integrated w.r.t. y between the limits c and d.

∫∫ f ( x, yy))d x ddyy = ∫ ∫ f ( x, yy))d x ddyy


d x2 ( y )
Hence, c x1 ( y ) 
R

Note:
(i) If all the four limits are constant then the function f (x, y) can be integrated w.r.t.
any variable first. But if f (x, y) is implicit and is discontinuous within or on the
boundary of the region of integration then the change of the order of integration
will affect the result.
(ii) If all the four limits are constant and f (x, y) is explicit then double integral can be
written as product of two single integrals.
9.2 Double Integrals Over Rectangles 9.3

(iii) If inner limits depends on x then the function f (x, y) is integrated first w.r.t. y and
vice-versa.

9.2.2 Properties of Double Integrals


Various properties of double integrals are analogous to those for single integrals. For f
and g continuous in region R with k as rational number,
(i) ∫∫ ( f + g ) dxdy = ∫∫ f dxdy + ∫∫ g dxdy
R R R

(ii) ∫∫ k f dxdy = k ∫∫ f dxdy , where k is a constant.


R R
For f continuous in region R, where R = R1 » R2 where R1 and R2 are non-overlapping
regions whose union is R:
(iii) ∫∫ R
f dxdy = ∫∫ f dxdy + ∫∫ f dxdy
R1 R2

Example 1
3 1
∫ ∫ (x + 3 y 2 )dy d x.
2
Evaluate
0 0
Solution
3 1 3
∫ ∫ ((xx + 3 y 2 ) dy d x = ∫ x 2 y + y 3 0 d x
2 1

0 0 0
3
= ∫ ( x 2 + 1)d x
0
3
x3
= +x
3 0

= 12

Example 2
1 2
Evaluate ∫∫
0 0
( x 2 + y 2 )dy d x.
Solution 2
1 2 1 y3
∫∫ 0 0
((xx 2 + y 2 ) dy d x = ∫ x 2 y +
0 3 0
dx

1 8
= ∫  2x2 +  dx
0 3
1
2 x3 8 x
= +
3 3 0
10
=
3
9.4 Chapter 9 Multiple Integrals

Example 3
1 2
Ú-1 Ú0 (1 - 6 x
2
Evaluate y) dx dy . [Winter 2016]
Solution
y) dx dy = Ú ÈÍ Ú (1 - 6 x 2 y) dx ˘˙ dy
1 2 1 2
Ú-1 Ú0 (1 - 6 x
2
-1 Î 0 ˚
2
1 x3
=Ú x - 6y ◊ dy
-1 3
0
1 2
=Ú x - 2 yx 3 dy
-1 0
1
= Ú (2 - 16 y) dy
-1
1
= 2 Ú 2 dy
0
=4

Example 4
a b d x dy
Evaluate ∫∫
2 2 xy
.

Solution
a b dxx dy
d aÊ b ddxx ˆ d
dyy a b 1
Ú2 Ú2 xy
= Ú ÁÚ ˜
2 Ë 2 x ¯ y
= Ú log x 2 dy
2 y
a 1
= (log b - logg 2)Ú dy
2 y
Ê bˆ a
= log
g Á ˜ log y 2
Ë 2¯
Ê bˆ
= log
g Á ˜ (logg a - log 2)
Ë 2¯
Ê bˆ Ê aˆ
= log
g Á ˜ log Á ˜
Ë 2¯ Ë 2¯
Another method: Since both the limits are constant and integrand (function) is
explicit in x and y, the integral can be written as
a b d x dy a dy b d x
Ú2 Ú2 xy

2 y Ú2 x

a b
= log
g y 2 log x 2
= (log a - log 2)(log b - logg 2)
9.2 Double Integrals Over Rectangles 9.5

 a  b
= logg   ⋅ log  
 2  2
 b  a
= logg   ⋅ log  
 2  2

Example 5
1 2
Evaluate ∫ ∫ xy d y d x.
0 1
Solution
∫∫
1

0 1
2
xy dy dx = ∫
1

0 { ∫ y dy } x d x
2

1
2
1 y2
=∫ x dx
0 2 1
1 4 1
= ∫  −  x dx
02 2
1
3 x2
=
2 2 0

3 1
⋅ =
2 2
3
=
4
Another method: Since both the limits are constant and integrand (function) is
explicit in x and y, the integral can be written as
1 2 1 2
∫∫
0 1
xy dy d x = ∫ x d x ⋅ ∫ y dy
0 1

2 1 2 2
x y
=
2 0
2 1

1  4 1
=  − 
2  2 2
3
=
4

Example 6
1 x
Evaluate ∫∫
0 0
dy d x.
Solution x
1 x 1
∫∫
0 0
dy d x = ∫ y d x
0 0
9.6 Chapter 9 Multiple Integrals

1
= ∫ x dx
0
1
x2
=
2 0

1
=
2

Example 7
y
1 x
Evaluate ∫∫
0 0
e x ddyy d x.
Solution y
1 x 1 y x
∫∫ 0 0
e x dyd x = ∫
0 xe x
0
dx
1
= ∫ x(e − 1)dxx
0
1
x2
= (e − 1)
2 0

1
= (e − 1)
2

Example 8
1 x2
Evaluate ∫∫
0 x
xy dy dx.
Solution
∫∫
1

0
x2

x
xxyy dy dx = ∫
1

0 {∫ } x2

x
y dy x dx
x2
1 y2
=∫ x dx
0 2 x

1 1 2 2
2 ∫0 
= ( x ) − x 2  x dx

1 1
= ∫ ( x 5 − x 3 )dx
dx
2 0
1
1 x6 x4
= −
2 6 4 0

1  1 1
=  − 
2  6 4
1
=−
24
9.2 Double Integrals Over Rectangles 9.7

Example 9
1 1+ x 2 dx d y
Evaluate ∫∫
0 0 1 + x2 + y 2
.

Solution
1+ x 2
 1+ x2 dy
dy  1 y
∫0 ∫0 ( 1 + x 2 )2 + y 2  dx
1 1
dx = ∫ tan −1 dx
dx
 
0
1+ x 2
1 + x2 0

1 1
=∫ (tan −−11 1 − tan −1 0)dx
0
1 + x2
1 1 π
=∫ ⋅ dx
0
1+ x 4
2

π
log ( x + 1 + x 2 )
1
=
4 0

π
= log (1+
1+ 2)
4

Example 10
1− y 2
1 dxdy
Evaluate ∫∫
0 0
2

1 − x2 − y 2
.

Solution
1 
1− y 2 1− y 2
1 d dy
dx dy dx
dx
∫∫
0 0
2

1 − x2 − y 2
= ∫ ∫ 2
0
 0
 dy
(1 − y 2 ) − x 2 
dy

1− y 2
1 −1 x 2
= ∫ sin dy
dy
0 1 − y2 0

1 1 
= ∫  sin
n −−11 sin −1 0 dy
− sin d
0 
2
π 1
= y0
4
π
=
4
9.8 Chapter 9 Multiple Integrals

Example 11
1 1 x− y 1 1 x− y
Show that ∫ dx∫
0 0 ( x + y) 3
dyy ≠ ∫ dy ∫
d
0 0 ( x + y )3
ddxx.

Solution
1 1 x− y 1 1 2 x − ( x + y)
∫ dx∫
0 0 ( x + y) 3
dyy = ∫ dx ∫
d
0 0 ( x + y )3
ddyy

1 1  2x 1 
=∫ ∫  ( x + y )3 − ( x + y ) 2  dy dx
0 0
 
1
1  1  1
= ∫ 2x  2+ dx
0
 −2( x + y )  x + y0
1 x 1 1 1
= ∫ − + + −  dx
dx
0
 ( x + 1) 2
x + 1 x x
1 1
=∫ dx
dx
0 ( x + 1) 2

1
1
= −
x +1 0
1
=
2

1 1 x− y 1 1  ( x + y) − 2 y 
∫ ddyy ∫
0 0 ( x + y) 3
dxx = ∫ dy ∫ 
d
0 0
 ( x + y) 
3  dx
dx

1 1  1 2y 
=∫ ∫  ( x + y ) 2 − ( x + y )3  dx dy
0 0
 
1
1 1 y
=∫ − + dy
dy
0 x + y ( x + y)2 0
1 1 y 1 1
= ∫ − + + −  dy
dy
 1 + y (1 + y )
2
0 y y
1 1
=∫ − dy
dy
0 (1 + y ) 2
1
1
=
1+ y 0

1
=−
2
9.2 Double Integrals Over Rectangles 9.9

1 1 x− y 1 1 x− y
Hence, ∫ dx∫
0 0 ( x + y) 3
dyy ≠ ∫ dy ∫
d
0 0 ( x + y )3
ddxx

x− y
Since is discontinuous at (0, 0), a point on the boundary of the region (square),
( x + y )3
change of order of integration does not give the same result.

Example 12
y
4 x 3
Sketch the region of integration and evaluate Ú1 Ú0 2
e x dy dx .

[Summer 2017]

Solution
1. Since the inner limits depend on x, the function is integrated first w.r.t. y and then
w.r.t. x.
2. Limits of y: y = 0 to y = x
Limits of x: x = 1 to x=4
3. The region is bounded by the line x = 1, y = 0, x = 4 and parabola y2 = x.
4. The points of intersection of y2 = x and x = 1, x = 4 are (1, 1) (4, 2).

3 ÈÍ x ˘
y y y
4 x 3 4 y2 = 4
Ú1 Ú0 dy dx = Ú dy ˙ dx
2 Í Ú0
x x
e e
2 1 ˙
Î ˚ B
(4, 2)
(1, 1)
x
y
4 3 e x
=Ú dx A
x
1 2 1/ x O
x=1
0
x=4

x
4
4 3
=Ú x e x
dx
1 2
0
Fig. 9.4
4 3
=Ú x (e - 1)dx
1 2
3 4
= (e - 1) Ú x dx
2 1
9.10 Chapter 9 Multiple Integrals

4
3
3 x2
= (e - 1)
2 3
2 1
4
3
3 2
= (e - 1) ◊ x 2
2 3
1

= (e - 1) ÈÍ 2 ˘
3

Î - 1˙˚
4
3
2◊
= (e - 1) [2 2 - 1]
3
= [e – 1] [2 – 1]
= (e – 1) [8 – 1]
= 7(e – 1)

EXERCISE 9.1
Evaluate the following integrals:
2 2−y
1. ∫∫1 − 2−y
2 x 2 y 2 dx dy
 856 
 Ans. : 945 
1 y
2. ∫∫0 0
xy e x −2 dx dy
 1
 Ans. : 4e 
1 x
3. ∫∫0 0
e x + y d x dy
 1 2
 Ans. : 2 (e − 1) 
1
1
4. ∫ ∫
10 0
y
ye xy dx dy
[Ans. : 9(1 – e)]
log 8 log y
5. ∫ ∫
1 0
e x + y d x dy
[Ans. : 8(log 8 – 1)]
1 y
6. ∫∫0 y2
(1 + xy 2 )dx dy
 41 
 Ans. : 210 
9.2 Double Integrals Over Rectangles 9.11

2a 2 ax − x 2
7. ∫ ∫
0 0
xyy dy dx
 2a 4 
 Ans. : 
 3 

9.2.3 Working Rule for Evaluation of Double Integrals


Over a General Region
1. If the region is bounded by more than one curve then find the points of intersection
of all the curves.
2. Draw all the curves and mark their point of intersection.
3. Identify the region of integration.
4. Draw a vertical or horizontal strip in the region whichever makes the integration easier.
5. The vertical strip starts from the lowest part of the region and terminates on the
highest part of the region.
6. For vertical strip: (i) The lower limit of y is obtained from the curve where the verti-
cal strip starts and the upper limit of y is obtained from the curve where it termi-
nates.
(ii) The lower limit of x is obtained from the leftmost point of the region and the
upper limit of x is obtained from the rightmost point of the region.
7. The horizontal strip starts from the left part of the region and terminates on the right
part of the region.
8. For horizontal strip: (i) The lower limit of x is obtained from the curve where the
horizontal strip starts and upper limit is obtained from the curve where it terminates.
(ii) The lower limit of y is obtained from the lowest point of the region and the
upper limit of y is obtained from the highest point of the region.
9. If variation along the strip changes within the region then the region is divided into
parts.

Example 1

∫∫ e
ax + by
Evaluate dx dy,
dy over the triangle bounded by x = 0, y = 0, ax + by = 1.
Solution
1. The region of integration is the DOPQ.
2. The integration can be done w.r.t. any variable first. Draw a vertical strip AB paral-
lel to y-axis which starts from x-axis and terminates on the line ax + by = 1.
1 − ax
3. Limits of y : y = 0 to y =
b
1
Limits of x : x = 0 to x=
a
9.12 Chapter 9 Multiple Integrals

1 1 y
I x y
1 1

∫ a
e ax eby d x (0, 1 )
b
1 ax
1
eby ax + by = 1
∫ a ax
e
b 0 B
1
1
1] dx
b∫
e ax e 1
a ax

( , 0)
1
1 O A x
dx
b
1 Fig. 9.5
1 e a
ex
b a 0
1 e e 1
− +
b a a a
1
ab

Example 2
xy
Evaluate ∫∫ 1− y 2
dx y over the first quadrant of the circle x2 + y2 = 1.
Solution
1. The region of integration is OPQ. y
2. The integration can be done w.r.t any
variable first. Draw a vertical strip AB (0, 1)
parallel to y-axis which starts from x-axis
and terminates on the circle x2 + y 2 = 1. B

3. Limits of y : y = 0 to y x 2
x2 + y2 = 1

Limits of x : x = 0 to x=1
1 1 x2
I dx y
1 y2 (1, 0)
1 A x
1 1− x 2 1 − O
− − −2 y dx
2 Fig. 9.6
1 x2
( x)]n +1 
1
1 1
2∫
− x 1 x ∵∫ ( x)]n f ( )dx
n +1
1 1
− ∫ 2 1)dx
9.2 Double Integrals Over Rectangles 9.13

1
x3 x2
=−
3 2 0

1 1
=−
3 2
1
=
6

Example 3
Evaluate ∫∫( ) 2 dx y over the right half of the circle x2 + y2 = a2.
Solution
1. The region of integration is PQR.
2. The integration can be done w.r.t. any variable first. Draw a vertical strip AB
parallel to y-axis which starts from the part of the circle a 2 below
x-axis and terminates on the part of the y
circle a 2 above x-axis.

3. Limits of
y y 2 2 2
− x2
R
Limits of x : x = 0 to x = a B
2 2
x + y2 a2
a a x
I dx y
a2 x2

a2 x2
= − )2 ∫ 2
d Q (a, 0)
0 −
O x
a2 x2
= −x 2
y− 2 2 x
a
+ ⋅ 2 a2 x 2 dx
a A
2
) x 2 dx P

Putting x a co d
When x

When x = Fig. 9.7


2

I d
0

= + sin c 2
n cos 2
π

=2 4
s sin s c 2
( sin d
9.14 Chapter 9 Multiple Integrals

4  1 π cos 2
⋅ + ⋅ ⋅ +2
2 2 4 2 2 3

0


Using re nf la and d
[ f ( )]n +1 
n +1
p ˘
- cos 0
4 16 3 2
p p 4
4
+
2 8 3
5p 4
a4 -
3

Example 4
n
x2 y 2 2
Evaluate ∫ ∫ xy 2 + 2 over the first quadrant of the ellipse
a b
x2 y 2
+ =1
a2 b
Solution y
1. The region of integration is OPQ.
2. The integration can be done w.r.t any
variable first. Draw a vertical strip AB par-
allel to y-axis which starts from x-axis and
x2 2 (0, b)
terminates on the ellipse 2 + =1 B
a b x2 y2
+ =1
a2 b2
x2
3. Limits of y : y = 0 to
a2
P (a, 0)
Limits of x : x = 0 to x=a
n O A x
x2 2 2
a b 1 2
I d y Fig. 9.8
0
a2 b2
n
x2
a b 1 b2 x 2 y 2 2y
∫ x∫ a2
+
2 a2 b2 b2
d

x2
n 1
+1 a2
b2 a 1 x2 y2 2
[ f y )]n +1 
+
2 ∫ x n
+1 a2 b2
dx ∵ ∫ [ f y )]n f y) y
n +1
2 0
9.2 Double Integrals Over Rectangles 9.15

b2 x2 1 xn+ 4
− n+ 2 ⋅
n+2 2 a n+4 0
b2 a 2 1 an+ 4

n + 2 2 an+ 2 n + 4
a 2 ( + )

( n + 2) 2 + )
2
a
2 + )

Example 5
Evaluate ∫∫( ) over the ellipse 2x2 + y2 = 1.

Solution
2 2
1. The region of integration is PQRS, the ellipse 2 1 or 2
1
1 1
2
1 y
with and 1 as its axes.
2
Q (0, 1)
2. The integration can be done w.r.t any vari- B
able first. Draw a vertical strip AB paral-
lel to y-axis which starts from the part 2x 2 + y 2 1
of the ellipse 2 1 below x-axis
and terminates on the part of the ellipse
2 1 above x-axis. x
1 0
3. Limits of ( √2 ) (√21 , 0)
y y = − 1− 2
= x2
1 1
Limits of x x − to x =
2 2 S (0, 1) A
1 2
1 2x
I dy
1− 2 x 2
2 Fig. 9.9
1 1− 2 x 2
y3
+ dx
2
3 1 2 x2
1
1
2
1 − x2 + − 2x x
2
3
1 3

0
− x2 +
1
3
− ) 2 dx
9.16 Chapter 9 Multiple Integrals

1 1
Putting 2 x 2 = cos 2 θ , x = cos θ , dx = −
cos sin θ dθ
2 2
π
When x = 0, cos θ = 0, θ =
2
1
When x = , cos θ = 1, θ = 0
2

0 È cos q

2
1 Ê 1 ˆ
I = 4Ú p Í 1 - coss 2 q + (1 - cos 2 q ) 2 ˙ Á - sin q dq ˜
2ÍÎ
2 3 ˙˚ Ë 2 ¯

È1 p 1
p ˘
= 2 2 Í Ú 2 cos cos 2 q sin 2 q dq + Ú 2 sin
sin 4 q dq ˙
ÍÎ 2 0 3 0
˙˚
È1 Ê 1 1 p ˆ 1 Ê 3 1 p ˆ˘
= 2 2 Í Á ◊ ◊ ˜ + Á ◊ ◊ ˜˙ [ Using
Using reductio
reductionn fo mula ]
forrmul
Î2 Ë 4 2 2¯ 3Ë 4 2 2¯˚
3 2p
=
16

Example 6
Evaluate ∫ ∫ ( x 2 − y 2 )dx dy over the triangle with the vertices (0, 1),
(1, 1), (1, 2).
Solution
1. The region of integration is DPQR.
2. Equation of the line PQ is y = 1. y
Equation of the line PR is R (1, 2)
2 −1
y −1 = ( x − 0) = x B
1− 0
y = x +1 x +1
y=
3. The integration can be done w.r.t. P (0, 1) Q (1, 1)
any variable first. Draw a vertical A y=1
strip AB parallel to y-axis which
starts from the line y = 1 and termi-
nates on the line y = x + 1.
4. Limits of y : y = 1 to y = x + 1 x
O
Limits of x : x = 0 to x = 1
Fig. 9.10
9.2 Double Integrals Over Rectangles 9.17

1 x +1
I=∫ ∫ ((xx 2 − y 2 )ddyy dx
0 1
x +1
1 y3
= ∫ x2 y − dx
dx
0
3 1

1 ((x + 1)3 1
= ∫  x2 ( x +
+11) − − x 2 +  dx
d
0
 3 3
1
x 4 x 3 ( x +1
+ 1) 4 x 3 x
= + − − +
4 3 12 3 30
1 1 16 1
= + − +
4 3 12 12
2
=−
3

Example 7

∫∫e
y2
Evaluate dx dy
dy over the region bounded by the triangle with vertices

(0, 0), (2, 1), (0, 1). y

Solution
1. The region of integration is
DOPQ.
P (0, 1) Q (2, 1)
2. Equation of the line OQ is
x
y = or x = 2 y. A B
2
3. Here, it is easier to integrate 2y
x=
w.r.t. x first than y. Draw a
horizontal strip AB parallel to O x
x-axis which starts from y-axis
and terminates on the line x = 2y.
4. Limits of x : x = 0 to x = 2y Fig. 9.11
Limits of y : y = 0 to y = 1
1 2y
I=∫ ∫
2
e y dxdy
0 0
1 2y
= ∫ ey ∫
2
dx dy
dy
0 0
1
= ∫ e y x 02 y dy
2
dy
0
1
= ∫ e y ⋅ 2 y dy
2
dy
0

∵ e f ( y ) f ′ ( y )dy = e f ( y ) 
 ∫
1
= e y2 0 
= e −1
9.18 Chapter 9 Multiple Integrals

Example 8
2 xy 5
Evaluate
1+
∫∫ − y4
over the triangle having vertices

(0, 0), (1, 1) and (0, 1).

Solution
y
1. The region of integration is the DOPQ.
2. Equation of the line OP is y = x.
3. Here, it is easier to integrate w.r.t. x f irst than
with y. Draw a horizontal strip AB parallel
(0, 1)
to x-axis which starts from y-axis and termi- (1, 1)
nates on the line y = x.
4. Limits of x : x = 0 to x = y
Limits of y : y = 0 to y = 1 A B

1 y 2 xy 5
I=Ú Ú dx y
1+ x - y4 O x

1
1
= x 2 xy 2 dx y Fig. 9.12
y
1
1
+x y [ f x )]n +1
y )2 0
dy ∵ Ú f x )]n f x )dx =
n +1
1
1 3
◊ ) 2 dy

1
1
- 1 dy
0
1 1
y4 -2
1
1 - y )2
( 4y )
dy
4 0
-4
1
[ ( x )]n +1 ˘
3
1 1 2
+ (1 )2 ∵ Ú [ ( x )]n f ( )d x
2 2 3 0 n +1
1 1
-
2
1
6
9.2 Double Integrals Over Rectangles 9.19

Example 9
Evaluate ∫ ∫(( x 2 + y 2 )dx dy over the region bounded by the lines
y = 4x, x + y = 3, y = 0, y = 2.
Solution
y
1. The region of integration is OPQR.
2. The integration can be done w.r.t. any
variable first. But in case of vertical strip y = 4x
we need to divide the region into three
parts. Therefore, draw a horizontal strip
AB parallel to x-axis which starts from R Q y=2
the line y = 4x and terminates on the line
x + y = 3. A B
x+y=3
y
3. Limits of x : x = to x=3–y
4 O P x
Limits of y : y = 0 to y=2
2 3- y 2
I=Ú Ú
y (x + y 2 )dx dy Fig. 9.13
0
4
3- y
2 x3
=Ú + xy 2 dy
0 3 y
4

2 È (3 - y ) 1 y3 y3 ˘
3
=Ú Í + (3 - y) y 2 - ◊ - ˙ dy
0
ÍÎ 3 3 64 4 ˙˚
2 È (3 - y ) 241 3 ˘
3
=Ú Í + 3 y2 - y ˙ dy
ÎÍ 3 19
922 ˚˙
0

2
1 (3 - y)4 y3 241 y 4
= ◊ + 3◊ - ◊
3 -4 3 192 4
0

1 241 Ê 27 ˆ
=- +8- ◊4 - Á- ˜
12 192 Ë 4¯
463
=
48
9.20 Chapter 9 Multiple Integrals

Example 10
Evaluate Ú Ú ( x + y)dy dx, where R is the region bounded by x = 0, x = 2,
R
y = x, y = x + 2. [Summer 2016]

Solution
1. The region of integration is
OPQR. y
Q
(2, 4)
2. The point of intersection of x = 2 2
and y = x + 2 is obtained as y = 4. x+
y=
The point of intersection is (2, 4). (2, 2)
R P
2)
x=0
3. The integration can be done w.r.t. ( 0,
x x=2
any variable first. Draw a vertical y=
strip AB parallel to y-axis which O x
starts from the line y = x and termi- (–2, 0)
nates on the line y = x + 2.
Fig. 9.14
4. Limits of y: y = x to y = x + 2
Limits of x: x = 0 to x = 2
2
x+2
I = ÚÚ ( x + y) dy dx
x
0
2
= Ú ÈÍ Ú ( x + y) dy ˘˙ dx
x+2

0
Î x ˚
2 x+2
y2
= Ú xy + dx
0
2
x

2
È 1 x2 ˘
= Ú Í x( x + 2) + ( x + 2)2 - x 2 - ˙ dx
0ÎÍ 2 2 ˙˚
2
È x2 x2 ˘
= Ú Í x2 + 2 x + + 2 x + 2 - x 2 - ˙ dx
0ÍÎ 2 2 ˙˚
2
= Ú [4 x + 2] dx
0
2
x2
= 4 + 2x
2
0
2
= 2 x2 + 2 x
0
= 8+4-0
= 12
9.2 Double Integrals Over Rectangles 9.21

Example 11
ÚÚ (2 x - y
2
) dA
A over the triangular region R enclosed between the lines
R
y = –x + 1, y = x + 1 and y = 3. [Summer 2015]
Solution
1. The region of integration is DPQR.
2. The points of intersection of
(i) y = –x + 1 and y = x + 1 is
obtained as
–x + 1 = x + 1
2x = 0, x = 0
y=1
The points of intersection
is P(0, 1).
(ii) y = x + 1 and y = 3 is
obtained as
3=x+1
x = 2, y = 3
The points of intersection
is Q(2, 3). Fig. 9.15
(iii) y = –x + 1 and y = 3 is obtained as
3 = –x + 1
x = –2, y = 3
The points of intersection is R(–2, 3).
3. The integration can be done w.r.t. any variable first. But in case of vertical strip,
we need to divide the region into two parts. Therefore, draw a horizontal strip AB
parallel to x-axis which starts from the line y = –x + 1 and terminates on the line
y = x + 1.
4. Limits of x : x = 1 – y to x = y – 1
Limits of y : y = 1 to y = 3
3 y -1
I=Ú
1 Ú1- y
2x - y
((2 2
) dA
3 y -1
=Ú Ú 2 x - y 2 ) dx dy
((2
1 1- y
3 y -1
= Ú x 2 - xxyy 2 dy
1 1- y
3
= Ú ÈÎ( y - 1)2 - y 2 ( y - 1) - ((1
1 - y)2 + y 2 (1 - y)˘˚ dy
1
9.22 Chapter 9 Multiple Integrals

3
dy
1
3
3 3 4 3
-
3 3 4 3
8 8 81 54 1 2 ˘
- + + -
3 3 2 3 2 3
68
3

Example 12
y
Evaluate ∫∫ ( ) y2
over the region bounded by the parabola

y 2 = x and the line y = x.


Solution
1. The region of integration is OPQ.
y
2. The points of intersection of y2 = x and
y = x are obtained as y2 x
B
(1, 1)
x2 x
Q A
x 01
\ =0 1
The points of intersection are O (0, 0) and O x
P (1, 1).
3. Here, it is easier to integrate w.r.t. y first
than x. Draw a vertical strip AB parallel to
y-axis, which starts from the line y = x and
terminates on the parabola y2 = x.
4. Limits of y : y = x to y x
Limits of x : x = 0 to x=1 Fig. 9.16

1 x y
I=Ú Úx
(a x ) y2
1
1 1 x 1

(a x ) Ú
- ax
2
9.2 Double Integrals Over Rectangles 9.23

1 1 1 1
x
È [ f ( y)]n +1 ˘
2 Ú0 ((aa - x ) 2((ax - y )
=- 2 2 dx Í∵ Ú [ f ( y)] f ¢(
n
¢( y)dy = ˙
x Î n +1 ˚
1 1 È 1 1˘
= -Ú ÎÍ((ax - x ) 2 - (ax - x ) 2 ˚˙ dx
2
0 ((aa - x )

= -Ú
1 x
( a --11 - a - x ) dx
dx
0 a-x
Putting x = a sin2 q, dx = 2a sin q cos q dq
When x = 0, q = 0
1
When x = 1, q = sin -1
a
1
sin -1 a sin q
I = -Ú a ( a - 1 - a cos q ) 2 a sinn q cos
cos q dq
0 a cos2 q
1
sin -1 sin 2 q
= -2 a Ú
0
a
cos q
( a - 1 - a cos q ) dq
sin -1
1
ÈÊ 1 - cos2 q ˆ 2 ˘
= -2 a Ú a ÍÁ ˜¯ a - 1 - a sin q ˙ dq
0
Î Ë cos q ˚
sin -1
1
È a ˘
= -2 a Ú a Í a - 1(sec q - cos
cos q ) - ((11 - cos
cos 2q )˙ dq
0 Î 2 ˚
1
sin -1
= -2 a aq a sin 2q a
a - 1[log(sec
[log(sec q + ttan q ) - sin q ] - +
2 4 0

1
sin -1
È Ê 1 + sin q ˆ ˘ aq sin q ccos q
a sin a
= -2 a a - 1 Í log Á ˜¯ - sin q ˙ - +
Î Ë cos q ˚ 2 2 0

È Ê 1 ˆ 1 ˘
Í Á
1+
˜ a sin -1 ˙
a 1 a a 1 1˙
= -2 a Í a - 1 Á log - ˜ - + ◊ ◊ 1 -
Í Á 1 a˜ 2 2 a a˙
Í ÁË 1 - ˜¯ ˙
Î a ˚
a +1 1
= -2 a(a - 1) log
g + a - 1 + a sin -1
a -1 a
9.24 Chapter 9 Multiple Integrals

Example 13
Evaluate ∫ ∫ y over the region enclosed by the parabola x2 = y and
the line y = x + 2. y
Solution (2, 4)
1. The region of integration is POQ.
B
2. The points of intersection of x2 = y and
y = x + 2 are obtained as x2 = y
x 2
x+2 P ( 1, 1)
2
x x 2 0 A
(x x 1) = 0 O x
x= 1
∴ =41 Fig. 9.17

The points of intersection are P (–1, 1) and Q (2, 4).


3. The integration can be done w.r.t. any variable first. Draw a vertical strip AB parallel
to y-axis which starts from the parabola x2 = y and terminates on the line y = x + 2.
4. Limits of y : y = x2 to y = x + 2
Limits of x : x = –1 to x = 2
2 x 2
I=Ú Ú
1 x
x+2

2 y2 dx
1
2 x
1 2
2Ú 1
= dx

3 5 2
1

3 2 5 1

1 64 32 1 1
=
2 3 5 3 5
36
=
5

Example 14
Evaluate ∫ ∫ xy( x y) , over the region enclosed by the parabolas
x2 = y, y2 = –x.
Solution
1. The region of integration is OPQ.
9.2 Double Integrals Over Rectangles 9.25

2. The points of intersection of the parabola y


x2 = y, and y2 = –x are obtained as
y4 = y Q (−1, 1)
y = 0, 1
\ x = 0, –1. A B
The points of intersection are O (0, 0) and x2 = y y 2 = −x
P
Q (–1, 1).
O x
3. Here, it is easier to integrate w.r.t. x first. Draw
a horizontal strip AB parallel to x-axis, which
starts from the parabola x2 = y and terminates
on the parabola y2 = –x.
4. Limits of x : x = − y to x = –y 2
Limits of y : y = 0 to y = 1
1 − y2 Fig. 9.18
I=∫y ∫ xxyy ( x + y )dx dy
dy
0 − y

1 − y2

=∫y ∫ ( x 2 + xy )dx dy
0 − y

1 − y2
x3 x 2 y
=∫y + dyy
0 3 2 − y

 7 5

−y
1
 y6 y 2 y3 
=∫ + + − dy
dy
 3 2 3 2
0

7 1

y8 y 7 2 y 2 y 4
= − + + −
24 14 21 8 0

=0

Example 15
Evaluate ∫ ∫ xy dx dy over the region enclosed by the x-axis, the line
x = 2a and the parabola x2 = 4ay.
Solution
1. The region of integration is OPQ.
2. The point of intersection of the parabola x2 = 4ay and the line x = 2a is obtained as
4a2 = 4ay
y=a
The point of intersection is Q (2a, a).
3. The integration can be done w.r.t. any variable first. Draw a vertical strip AB parallel
to y-axis, which starts from x-axis and terminates on the parabola x2 = 4ay.
9.26 Chapter 9 Multiple Integrals

x2 y
4. Limits of y : y = 0 to y=
4a
Q (2
(2a, a)
Limits of x : x = 0 to x = 2a
x2 x 2 = 4ay
2a
I=∫ ∫ 4a
xy d
dxx dy
0 0
B x = 2a
x2
2a
= ∫ x∫ 4a
y dy dx P (2a, 0)
0 0
x 2 O A x
2 4a
2a y
=∫ x dx
dx
0
2 0 Fig. 9.19
2a x4
=∫ x⋅ dx
dx
0 a2
32a
2a
1 x6
=
32a 2 6 0

1 64a 6
= ⋅
32a 2 6
a4
=
3

Example 16
Evaluate ∫ ∫ xy dx dy, over the region enclosed by the circle x2 + y2 – 2x = 0,
the parabola y2 = 2x and the line y = x.
Solution
1. The region of integration is OPQRO.
y 2 = 2x
2. (i) The points of intersection of the circle D
Q (2, 2)
x2 + y2 – 2x = 0 and the line y = x are ob- R x
y =
tained as B
C
x2 + x2 – 2x = 0
x = 0, 1 A P x 2 + y 2 − 2x = 0
\ y = 0, 1 (1, 1)
The points of intersection are O (0, 0) and
O
P (1, 1).
(ii) The point of intersection of the circle
x2 + y2 – 2x = 0 and the parabola
y2 = 2x is obtained as
x2 + 2x – 2x = 0
x=0
\y=0
The point of intersection is O (0, 0). Fig. 9.20
9.2 Double Integrals Over Rectangles 9.27

(iii) The points of intersection of the parabola y2 = 2x and the line y = x are obtained
as x2 = 2x
x = 0, 2
\ y = 0, 2
The points of intersection are O (0, 0) and Q (2, 2).
3. The integration can be done w.r.t. any variable first. To integrate w.r.t. y first we
need to draw a vertical strip in the region. But one vertical strip does not cover the
entire region, therefore, divide the region OPQRO into two subregions OPR and
RPQ and draw one vertical strip in each subregion.
4. In the subregion OPR, strip starts from the circle x2 + y2 – 2x = 0 and terminates on
the parabola y2 = 2x.
Limits of y : y = 2 x − x 2 to y = 2 x
Limits of x : x = 0 to x = 1.
5. In the subregion RPQ, strip starts from the line y = x and terminates on the
parabola y2 = 2x.
Limits of y : y = x to y = 2 x
Limits of x : x = 1 to x = 2
I = Ú Ú xy ddxx dy

= ÚÚ xy dx dy + ÚÚ xy dx dy
OPR RPQ
1 2x 2 2x
=Ú Ú xxyy dy dx + Ú Úx xxyy dy dx
0 2 x - x2 1
1 2x 2 2x
= Ú xÚ 2 y dy dx + Ú x Ú y dy dx
0 2x-x 1 x
2x 2x
1 2 2 2
=Ú x y dx + Ú x y dx
0 1
2 2 x - x2 2 x

1 1 1 2
= Ú
2 0
x(2 x - 2 x + x 2 )dx + Ú x(2 x - x 2 )dx
2 1
1 2
1 4 1 2 x3 x 4
= x + -
2 4 0 2 3 4 1

1 8 1 1
= + -2- +
8 3 3 8
7
=
12

Example 17
Evaluate ∫ ∫ x dxdy
2
dy, over the region in the first quadrant enclosed by
the rectangular hyperbola xy = 16, the lines y = x, y = 0 and x = 8.
[Winter 2014]
9.28 Chapter 9 Multiple Integrals

Solution
1. The region of integration is OPQR. y
2. (i) The points of intersection of the
hyperbola xy = 16 and the line
y = x are obtained as x
x2 = 16, x = ± 4
=
y
B
\y=±4 x=8
Hence, R (4, 4) is the point of inter- R (4, 4)
section in the first quadrant. B xy = 16
D Q (8, 2)
(ii) The point of intersection of the
hyperbola xy = 16 and line x = 8 is
obtained as O A M y=0 C P x
8y = 16
y=2
Fig. 9.21
The point of intersection is Q (8, 2).
3. The integration can be done w.r.t. any variable first. To integrate w.r.t. y first we
need to draw a vertical strip in the region. But here one vertical strip cannot cover
the entire region, and therefore divide the region OPQR into two subregions OMR
and RMPQ and draw one vertical strip in each subregion.
4. In the subregion OMR, strip starts from x axis and terminates on the line y = x.
Limits of y : y = 0 to y = x
Limits of x : x = 0 to x = 4
5. In subregion RMPQ, strip starts from x axis and terminates on the rectangular
hyperbola xy = 16
16
Limits of y : y = 0 to y =
x
Limits of x : x = 4 to x = 8
I = Ú Ú x 2 dx dy
dy
= ÚÚ x 2 dx ddyy + ÚÚ x 2 dx dy
OMR
OM RMPQ
16
4 x 8
=Ú Ú0 x 2 dy ddxx + Ú Ú
x x 2 dy dx
0 4 0
16
4 x 8
= Ú x 2 Ú dy dx + Ú x 2 Ú x dy dx
0 0 4 0
16
4 2 8 2
=Ú y 0 dx + Ú
x
x x y 0x dx
0 4
4 3 8 16
= Ú x dx + Ú x 2 ◊ dx
0 4 x
4 8
4 2
= x + 16 x
4 0 2 4
= 64 + 8(64
64 - 16
16)
= 448
9.2 Double Integrals Over Rectangles 9.29

Example 18
dx d y
Evaluate ∫∫ x 4
+ y2
, over the region bounded by the y ≥ x2, x ≥ 1.

Solution
1. The region of integration is bounded by y
y ≥ x2 (the region inside the parabola
B
x2 = y) and x ≥ 1 (the region on the right
of line x = 1).
2. The point of intersection of x2 = y and
x = 1 is obtained as 1 = y.
The point of intersection is P(1, 1).
A
3. Here, it is easier to integrate w.r.t. y x2 = y
first than x. Draw a vertical strip AB
parallel to y-axis in the region which
P (1, 1)
starts from the parabola x2 = y and
O x
extends up to infinity. x=1
4. Limits of y : y = x2 to y Æ •
Limits of x : x = 1 to x Æ •
Fig. 9.22
• • 1
I=Ú Ú 2 4 dyydx
d
1 x x + y2


• 1 y
=Ú tan -1 dx
dx
1 x2 x2 x2
• 1
=Ú (tan -1 • - tan -1 1)dx
1x2
• 1 Êp pˆ
= Ú 2 Á - ˜ dx
1 x Ë 2 4¯

p 1
= -
4 x1
p
=
4

EXERCISE 9.2
Evaluate the following integrals:
1
1. ∫ ∫ xy dx dy, over the rectangle 1 £ x £ 2, 1 £ y £ 2.
[Ans. : (log 2)2]
2. ∫∫ sinn π(ax
( + by))dx dy , over the triangle bounded by the lines x = 0, y = 0
and ax + by = 1.
 1 
 Ans. : π ab 
9.30 Chapter 9 Multiple Integrals

∫∫ e
3x +4 y
3. dx dy , over the triangle bounded by the lines x = 0, y = 0, and
x + y = 1.
 1 
 Ans. : 12 (3e − 4e + 1)
4 3

4. ∫∫ xy − x − y dx dy,
1− y over the triangle bounded by x = 0, y = 0 and x + y = 1.
 16 
 Ans. : 945 

5. ∫∫ xy − y 2 dx dy , over the triangle having vertices (0, 0), (10, 1), (1, 1).
[Ans. : 6]
6. ∫ ∫ (x + y + a)dx dy, over the region bounded by the circle x 2
+ y 2 = a 2.
[Ans. : p a3]

7. ∫ ∫ xydxdy, over the region bounded by the x-axis, the line y = 2x and
x2
the parabola y = .
4a  2048 4 
 Ans. : 3 a 

8. ∫ ∫ 5 − 2x − y (dx) dyy, over the region bounded by x-axis, the line x + 2y = 3


and the parabola y2 = x.
 217 
 Ans. : 60 
1

∫ ∫ (4 x − y )2 dx dy, over the triangle bounded by x-axis, the line y = x


2 2
9.
and x = 1.  1π 3
 Ans. :  + 
 3 3 2 
10. ∫ ∫ xy(x + y)dx dy, over the region bounded by the parabola y 2
= x, x2 = y.
 3
 Ans. : 28 

11. ∫ ∫ xy(x + y)dx dy, over the region bounded by the curve x 2
= y and the
line x = y.
 3
 Ans. : 56 

12. ∫ ∫ xy(x − 1)dx dy, over the region bounded by the rectangular hyperbola
xy = 4, the lines y = 0, x = 1, x = 4 and x-axis.
[Ans. : 8(3 – log 4)]
9.3 Change of Order of Integration 9.31

9.3 CHANGE OF ORDER OF INTEGRATION

Sometimes, evaluation of double integral becomes easier by changing the order of


integration. To change the order of integration, first, we draw the region of integration
with the help of the given limits. Then we draw a vertical or horizontal strip as per the
required order of integration. This change of order also changes the limits of integration.

Type I Change of Order of Integration

Example 1
1 x
Change the order of integration of ∫∫ f x, y x y.

Solution y
1. Since inner limits depends on x, the func-
tion is integrated first w.r.t. y and then Q (1, 1)
w.r.t. x.
B′
The correct form of the integral
1 x
∫∫ x
x=1
2. Limits of y : y = 0 to y = x, along
vertical strip A¢B¢
Limits of x : x = 0 to x = 1
O y 0 A′ P x
3. The region is bounded by the lines y = 0,
y = x, and x = 1.
4. The point of intersection of y = x and Fig. 9.23
x = 1 is Q(1, 1). y
5. To change the order of integration, i.e.,
to integrate first w.r.t. x, draw a horizon- Q (1, 1)
tal strip AB parallel to x-axis which starts
from the line y = x and terminates on the x 1
line x = 1.
Limits of x : x = y to x = 1 A B
Limits of y : y = 0 to y = 1
Hence, the given integral after change of
order is O P x
1
∫∫ f x, y
y
f ( x y )dx y
Fig. 9.24

Example 2
1 y
Change the order of integration of ∫∫ x
9.32 Chapter 9 Multiple Integrals

Solution y

1. Since inner limits depend on y, the function y 1


is integrated first w.r.t. x and then w.r.t. y. P
Q (1, 1)
The correct form of the integral
1 y
=∫ ∫ f x, y x y.
2. Limits of x : x = 0 to x = y, along hori- A′ B
zontal strip A¢B¢
Limits of y : y = 0 to y = 1
3. The region is bounded by the lines x = 0, O x
x = y, and y = 1.
4. The point of intersection of x = y and y = 1
is Q (1, 1). Fig. 9.25
y
5. To change the order of integration, i.e., to
integrate first w.r.t. y, draw a vertical strip B y 1
AB parallel to y-axis which starts from the P
Q (1, 1)
line x = y and terminates on the line y = 1.
Limits of y : y = x to y = 1
Limits of x : x = 0 to x = 1
Hence, the given integral after change of order
is
A
1
∫∫ f x, y x y f ( x y) x O x

Fig. 9.26

Example 3
a a
Change the order of integration of ∫∫ x
x
Solution
y
1. Since inner limits depend on x, the function
B′ y a
is integrated first w.r.t. y and then w.r.t. x. P
Q (a a)
2. Limits of y : y = x to y = a, along verti-
cal strip A¢B¢.
Limits of x : x = 0 to x = a
3. The region is bounded by the lines y = x,
y = a and x = 0.
4. The point of intersection of y = x and A′
y = a is Q (a, a).
O x
5. To change the order of integration i.e., to
integrate first w.r.t. x, draw a horizontal
strip AB parallel to x-axis which starts from Fig. 9.27
9.3 Change of Order of Integration 9.33

the line x = 0 and terminates on the line y


y = x.
y=a
Limits of x : x = 0 to x = y P
Q (a a)
Limits of y : y = 0 to y = a
Hence, the given integral after change of
order is
A B
a a
∫ f x y f ( x y )dx y

O x

Fig. 9.28

Example 4
∞ ∞
Change the order of integration of ∫ ∫ x
f x, y x y.

Solution y yƕ
B

1. Since inner limits depend on x, the function is


integrated first w.r.t. y and then w.r.t. x.
The correct form of the integral
∞ ∞
= ∫ ∫ x
f x, y dx

2. Limits of y : y = x to y Æ •, along vertical


strip A
Limits of x : x = 0 to x Æ •
O x
3. The region is bounded by the lines y = x and
x = 0.
Fig. 9.29
4. Here the only point of intersection is origin
y
O.
5. To change the order of integration, i.e., to
integrate first w.r.t. x, draw a horizontal strip
AB parallel to x-axis which starts from the
line x = 0 and terminates on the line y = x.
Limits of x : x = 0 to x = y
Limits of y : y = 0 to y Æ • A B
Hence, the given integral after change of order
is
∞ ∞ ∞ y O x
∫ f x y x f ( x y )dx y
Fig. 9.30
9.34 Chapter 9 Multiple Integrals

Example 5
1 x
Change the order of integration of ∫∫ x
f x, y x.

Solution
y
1. The function is integrated first w.r.t. y and
then w.r.t. x. (1, 1)
2. Limits of y : y = x to y x
B′
Limits of x : x = 0 to x = 1
3. The region is bounded by the line y = x and
the parabola y2 = x. A′
4. The points of intersection of y2 = x and O x
y = x are obtained as
x2 = x
x = 0, 1
\ y = 0, 1.
The points of intersection are O (0, 0) and
Q (1, 1). Fig. 9.31
5. To change the order of integration, i.e., to y
integrate first w.r.t. x, draw a horizontal y
strip AB parallel to x-axis which starts from (1, 1)
the parabola y2 = x and terminates on the A B
line y = x.
Limits of x : x = y2 to x = y
Limits of y : y = 0 to y = 1
Hence, the given integral after change of order O x
is

x y
∫ y

Fig. 9.32

Example 6
1
1 y
Change the order of integration of ∫∫ y2
f x, y x y.

Solution
1. The function is integrated first w.r.t. x and then w.r.t. y.
9.3 Change of Order of Integration 9.35

1 y
2. Limits of x : x = y2 to x y3
Limits of y : y = 0 to y = 1 (1, 1)
3. The region is bounded by the parabola y2 x y x3
y2 = x and the cubical parabola y = x3. A′
B′
4. The points of intersection of y2 = x and y = x3
are obtained as
O x
x6 = x
x = 0, 1
\ y = 0, 1.
The points of intersection are O (0, 0) and
Q (1, 1).
Fig. 9.33
5. To change the order of integration , i.e.,
y
to integrate first w.r.t. y, draw a vertical
strip parallel to y-axis which starts from the
cubical parabola y = x3 and terminates on
(1, 1)
the parabola y2 = x. B
Limits of y : y = x3 to y x y2 = x
y = x3
Limits of x : x = 0 to x = 1 A
Hence, the given integral after change of order O x
is
1
y x
∫ 2

Fig. 9.34

Example 7
y
Change the order of integration of
y Q y=8
8
∫∫ 4
y 8
4
f x, y x y. (0, 8) (2, 8)

A′ B′
Solution
1. The function is integrated first w.r.t. x and
then w.r.t. y.
y 8 y
2. Limits of x : x to x
4 4
Limits of y : y = 0 to y = 8
3. The region is bounded by the line y = 4x + 8, R
y = 4x, y = 8 and x-axis (y = 0). (−2, 0) O x

Fig. 9.35
9.36 Chapter 9 Multiple Integrals

4. The point of intersection of y = 4x and y


y = 8 is obtained as
Q y=8 D
8 = 4x
(0, 8)
x = 2.
The point of intersection is P (2, 8).
5. To change the order of integration, i.e.,
to integrate first w.r.t. y, divide the region
C
OPQR into two subregions OQR and
OPQ. Draw a vertical strip parallel to B
y-axis in each subregion.
(i) In subregion OQR, strip AB starts
from x-axis and terminates on the line R
y = 4x + 8. ( 2, 0) A O x
Limits of y : y = 0 to y = 4x + 8
Limits of x : x = –2 to x = 0 Fig. 9.36
(ii)In subregion OPQ, strip CD starts from
the line y = 4x and terminates on the line y = 8.
Limits of y : y = 4x to y = 8
Limits of x : x = 0 to x = 2
Hence, the given integral after change of order is
y
+ 8
∫∫ y−
4
f x, y x y x x ∫ x
f x, y dx

Example 8
y2
a
Change the order of integration of ∫ ∫ a
a
f x, y x y.

Solution
1. The function is integrated first w.r.t. x and y
then w.r.t. y. y=a
P
y2 Q (a a)
2. Limits of x : x = 0 to x = A B′
a
Limits of y : y = –a to y = a y 2 = ax
3. The region is bounded by the y-axis, the
parabola y2 = ax, and the line y = – a, and O x
y = a.
4. (i) The point of intersection of y2 = ax and
y = –a is obtained as R (a a)
a2 = ax S
y= a
x=a
The point of intersection is R (a, –a). Fig. 9.37
9.3 Change of Order of Integration 9.37

(ii) The point of intersection of y2 = ax and y


y = a is obtained as D y=a
a2 = ax P
Q (a a)
x=a y 2 = ax
The point of intersection is Q (a, a). C

5. To change the order of integration, i.e.,


to integrate first w.r.t. y, divide the region O x
into two subregions ORS and OPQ. Draw
a vertical strip parallel to y-axis in each B
subregion. R (a a)
S
(i) In subregion ORS, strip AB starts from A y = −a
the line y = –a and terminates on the
parabola y2 = ax. Fig. 9.38
Limits of y : y = –a to y ax (part of the parabola below x-axis)
Limits of x : x = 0 to x = a
(ii) In subregion OPQ, strip CD starts from the parabola y2 = ax and terminates on
the line y = a.
Limits of y : y ax to y = a
Limits of x : x = 0 to x = a
Hence, the given integral after change of order is
y
a ax a a
∫ ∫ a
f x, y x y x x ∫ f x, y dx

Example 9 2 2 4− 2 y
Change the order of integration of ∫∫ y
f x, y x y.
Solution
1. The function is integrated first w.r.t. x and then w.r.t. y.
2. Limits of x : x = y to x y
Limits of y : y = 0 to y = 2
y
3. The region is bounded by the x-axis,
Q (2, 2)
the line y = x and the parabola
(x – 2)2 = 2(2 – y). (x 2) = 2(2 − y)
4. The points of intersection of y = x
and (x – 2)2 = 2(2 – y) are obtained A′ B′
as R (4, 0)
(x – 2)2 = 2(2 – x) O x
x = 0, 2
\ y = 0, 2.
Fig. 9.39
9.38 Chapter 9 Multiple Integrals

The points of intersection are O (0, 0) and Q (2, 2).


5. To change the order of integration, i.e., to integrate first w.r.t. y, divide the region into
two subregions OPQ and PQR. Draw a vertical strip parallel to y-axis in each subre-
gion.
y
(i) In subregion OPQ, strip AB
starts from x-axis and termi- Q (2, 2)
D
nates on the line y = x. B
2
Limits of y : y = 0 to y = x x 2
Limits of x : x = 0 to x = 2
(ii) In subregion PQR, strip CD (4, 0)
starts from x-axis and termi- O A (2, 0) C x
nates on the parabola (x – 2)2 =
2(2 – y).
Fig. 9.40
x2
Limits of y : y = 0 to y −
2
Limits of x : x = 2 to x = 4
Hence, the given integral after change of order is
x2
− y x 2 x−
∫∫ f x, y x y x ∫ 2
f x, y dx

Example 10
a co a2 x2
Change the order of integration of ∫ ∫ tan
x

Solution
1. The function is integrated first w.r.t. y and then w.r.t. x.
2
2. Limits of y : y = x tan a to x2
Limits of x : x = 0 to x = a cos a
3. The region is bounded by the line y
y = x tan a, the circle x2 + y2 = a2 (0, a)
and y-axis. Since given limits of x B′ y = x tan a
and y are positive, the region lies P (a cos a, a sin a )
in the first quadrant.
4. The points of intersection of y = x A′
x2 + y 2 a2
tan a and x2 + y2 = a2 are obtained
as O x
x2 + x2 tan2a = a2
x = ± a cos a
\ y = ± a sin a . P
( a cos a
The points of intersection are P − sin a)
(a cos a, a sin a ) and P ¢(– a cos
a, – a sin a ). Fig. 9.41
9.3 Change of Order of Integration 9.39

5. To change the order of integration, y


i.e., to integrate first w.r.t. x, divide (0, a)
the region into two subregions C D y = x tan a
OPR and PQR. Draw a horizontal R P (a cos a, sin a )
strip in each subregion. A B x2+y2 a2
(i) In subregion OPR, strip AB starts
from y-axis and terminates on O x
the line y = x tan a.
Limits of x : x = 0 to
x = y cot a
P′
Limits of y : y = 0 to (− cos a
y = a sin a a sin a)
(ii) In subregion PQR, strip CD
starts from y-axis and terminates Fig. 9.42
on the circle x2 + y2 = a2.
2
Limits of x : x = 0 to y2
Limits of y : y = a sin a to y = a
Hence, given integral after change of order is
a2 x 2 ot a a2 y2
Ú sin a Ú y

Example 11
4 4x
Change the order of integration of ∫∫ 4 x x2
f x, y x.
Solution
1. The function is integrated first w.r.t. y and then w.r.t. x.
2. Limits of y : = x y
B′ (4, 4)
Limits of x : x = 0 to x = 4.
3. The region is bounded by the circle y 2 = 4x
x2 + y2 – 4x = 0, the parabola y2 = 4x
and the line x = 4. A
4. (i) The point of intersection of
(4, 0)
x2 + y2 – 4x = 0 and y2 = 4x is
O x
obtained as
2
x =0 x 2 + y 2 − 4x 0
x=0
\ y = 0.
The point of intersection is O (0, 0).
(ii) The points of intersection of y2 = 4x
Q ′(4, −4)
and x = 4 are obtained as
y2 = 16 Fig. 9.43
y=±4
The points of intersection are Q (4, 4) and Q ¢ (4, – 4).
9.40 Chapter 9 Multiple Integrals

5. To change the order of integration, i.e., y


to integrate first w.r.t. x, divide the region Q (4, 4)
into three subregions ORT, TPS and E F
y 2 = 4x
RSQ. Draw a horizontal strip parallel to R T (2, 2)
S (4, 2)
x-axis in each subregion. A B C D
(i) In subregion ORT, strip AB starts from
P (4, 0)
the parabola y2 = 4x and terminates on
O (2, 0) x
the circle x2 + y2 – 4x = 0.
Limits of x :
x 2 + y 2 − 4x = 0
y2
x= to x = 2 − 4 − y 2
4
(Part of the circle where x < 2)
Limits of y : y = 0 to y = 2 Q' (4, −4)
(ii) In subregion TPS, strip CD starts
from the circle x2 + y2 – 4x = 0 and Fig. 9.44
terminates on the line x = 4.
Limits of x : x = 2 + 4 − y 2
(Part of circle where x > 2) to x = 4
Limits of y : y = 0 to y = 2
(iii) In subregion RSQ, strip EF starts from the parabola y2 = 4x and terminates on
the line x = 4.
y2
Limits of x : x = to x = 4
4
Limits of y : y = 2 to y = 4
Hence, given integral after change of order is

4 4x 2 2−
2 − 4− y2
∫∫
0 4 x−
x x 2
f ( x, y ))d dx = ∫
dy dx
0 ∫ y2
4
f ( x , y ) dx d y
2 4 4 4
+∫ ∫ dy + ∫
f ( x, yy))dx dy ∫ y2 f ( x, y )dx dy
dy
0 2+
2 + 4− y2 2
4

Example 12
2 ((4 − x )2
Change the order of integration of ∫∫
0 4− x
f ( x, y ))ddy dx
dx.
Solution
1. The function is integrated first w.r.t. y and then w.r.t. x.
2. Limits of y : y = 4 − x to y = (4 − x) 2 .
Limits of x : x = 0 to x = 2
3. The region is enclosed by the parabolas y2 = 4 – x, y = (4 – x)2, the lines x = 0 and x = 2.
4. (i) The points of intersection of x = 2 and y2 = 4 – x are obtained as
9.3 Change of Order of Integration 9.41

y2 = (4 – 2) y

y = ± 2.
U (0, 16)
The points of intersection are
Q Q (2 ). = (4 − x)2
(ii) The point of intersection B
of x = 2 and y = (4 – x)2 is
x=2
obtained as
y = (4 – 2)2 = 4.
The point of intersection is
S (2, 4).
(iii) The points of intersection P
(0, 2) A′
of x = 0 and y2 = 4 – x are Q
(2, 2)
obtained as
y2 = 4
y = ±2. O V (4, 0) x
The points of intersection
are P (0, 2) and P¢ (0, –2). y2 −x
(iv) The point of intersection
Q (2, 2)
of x = 0 and y = (4 – x)2 is P′
obtained as (0, 2)
y = 16.
Fig. 9.45
The point of intersection is
U (0, 16). y

5. To change the order of


integration, i.e., to integrate first U (0, 16)

w.r.t. x, divide the region into


2
three subregions PQR, PRST = (4 − x
and STU. Draw a horizontal strip
in each subregion.
E F
(i) In subregion PQR, strip AB
starts from the parabola y2 = T
4 – x and terminates on the C D
line x = 2. P R (2, 2)
A B
Limits of x : (0, 2) Q
x = 4 – y2 to x = 2 (2, 2)

Limits of y : x=2

y= 2 to y = 2 O V (4, 0) x
(ii) In subregion PRST, strip y 2 = (4 − x
CD starts from y-axis and
terminates on the line x = 2. Q ′(2, − 2)
Limits of x : x = 0 to x = 2 P (0, −2)
Limits of y : y = 2 to y = 4
Fig. 9.46
9.42 Chapter 9 Multiple Integrals

(iii) In subregion STU, strip EF starts from y-axis and terminates on the parabola
y = (4 – x)2.
Limits of x : x = 0 to x y (Part of the parabola where x < 4)
Limits of y : y = 4 to y = 16
Hence, the given integral after change of order is
( x 2
∫∫
0 −
f x, y

x dx ∫ f x, y x y
16 4 y
+∫ ∫ f x, y x y
4

Type II Evaluation of Double Integrals by Changing the Order


of Integration

Example 1
a a
Change the order of integration and evaluate ∫∫ x
( ) .
Solution
1. Since inner limits depend on x, the function y
is integrated first w.r.t. y. B′ y=a
2. Limits of y : y = x to y = a, along verti- P
Q a, a)
cal strip
Limits of x : x = 0 to x = a
3. The region is bounded by the lines y = x,
y = a and x = 0
4. The point of intersection of y = x and y = a
is Q (a, a). A′
5. To change the order of integration, i.e. to O x
integrate first w.r.t. x, draw a horizontal
strip AB parallel to x-axis which starts from
y-axis and terminates on the line y = x. Fig. 9.47
Limits of x : x = 0 to x = y y
Limits of y : y = 0 to y = a y=a
Hence, the given integral after change of P
Q (a a)
order is
a
( )
0
y A B
a x3 2
y x dy
3 0
3 x
a y O
y3 y
3
Fig. 9.48
9.3 Change of Order of Integration 9.43

a 4 3
∫ 3
y y
a
4 y4
3 4 0
4
a
3

Example 2
sin y
Change the order of integration and evaluate ∫ ∫ y
Solution
y
1. The function is integrated first w.r.t.
y, but evaluation becomes easier by y p B¢
changing the order of integration. P (p p )
2. Limits of y : y = x to y = p
Limits of x : x = 0 to x = p
3. The region is bounded by the line

y = x, y = p and x = 0.
4. The point of intersection of the line
y = x and the line y = p is P (p, p).
5. To change the order of integration, i.e.,
O x
to integrate first w.r.t. x, draw a hori-
zontal strip AB parallel to x-axis which
starts from y-axis and terminates on the Fig. 9.49
line y = x.
Limits of x : x = 0 to x = y
Limits of y : y = 0 to y = p
Hence, the given integral after change of order is
y
y y

y=p
sin y y
=∫ x0 y
Q
P (p p )
y
sin y
y y A B
y

∫ sin y y

− cos y 0
O x
cos cos 0
2 Fig. 9.50
9.44 Chapter 9 Multiple Integrals

Example 3
1 1
Ú0 Úx sin y
2
Evaluate the iterated integral dydx. [Winter 2013]

Solution
1. Since the inner limits depend on x, the
function is integrated first w.r.t. y, but
evaluation becomes easier by changing
the order of integration.
2. Limits of y : y = x to y = 1, along
vertical strip
Limits of x : x = 0 to x = 1
3. The region is bounded by the lines y = x,
y = 1 and x = 0.
4. The point of intersection of the line
y = x and the line y = 1 is P (1, 1).
5. To change the order of integration, i.e., Fig. 9.51
to integrate first w.r.t. x, draw a horizontal
strip AB parallel to x-axis which starts
from y-axis and terminates on the line
y = x.
Limits of x : x = 0 to x = y
Limits of y : y = 0 to y = 1
Hence, the given integral after change of
order is

y
Ú0 Ú
2
n s y 2 dx y
Fig. 9.52
1 y
=Ú n y 2 x 0 dy
1 2
y dy

1 1 2
◊ y dy
2
1 1
= - co y 2 y sin ( ) ◊ ( dy - cos ( y)
2 0

1
= -
2
1
= - 1]
2
9.3 Change of Order of Integration 9.45

Example 4
e− y
Change the order of integration and evaluate ∫ ∫ x y
x

[Winter 2015]
y
Solution B′ y Æ •

1. Since inner limits depend on x, the


function is integrated first w.r.t. y but
evaluation becomes easier by changing
the order of integration.
2. Limits of y : y = x to y Æ •, along
vertical strip
Limits of x : x = 0 to x Æ •
A
3. The region is bounded by the lines y = x
and x = 0. O x
4. Here, the only point of intersection is
origin. Fig. 9.53
5. To change the order of integration, i.e. to y
integrate first w.r.t. x, draw a horizontal
strip AB parallel to x-axis which starts
from y-axis and terminates on the line
y = x.
Limits of x : x = 0 to x = y
Limits of y : y = 0 to y = •
A B
Hence, the given integral after change of
order is
y
∞ ∞ y e
∫ x
0 y
dx y
O x

∫ {∫
∞ y e Fig. 9.54
x y
y
∞ e

y
x0
y
−y
e
= ⋅ y
y

∫ e y

= −e y
0

= − (e e )
1
9.46 Chapter 9 Multiple Integrals

Example 5
2 1 x2
Evaluate the integral Ú0 Ú y e dx dy by changing the order of integration.
2 [Summer 2017, 2015]
Solution
1. The function is integrated first w.r.t. x, but
evaluation becomes easier by changing the
order of integration.
y
2. Limits of x : x = to x = 1
2
along horizontal strip A¢B¢
Limits of y : y = 0 to y = 2
3. The region is bounded by the lines y = 2x,
x = 1, y = 2, and y = 0.
4. The point of intersection of y = 2x and x = 1 Fig. 9.55
is x = 1, y = 2.
The point of intersection is Q (1, 2).
5. To change the order of integration, i.e., to
integrate first w.r.t. y, draw a vertical strip
AB parallel to y-axis which starts from x-axis
and terminates on the line y = 2x.
Limits of y : y = 0 to y = 2x
Limits of x : x = 0 to x = 1
Hence, the given integral after change of order
is

2 1 x2 1 2 x x2
Ú0 Ú y e dx dy = Ú
0 0 Ú e dy dx Fig. 9.56
2


1
0 {Ú }2x
0
2
dy e x dx

1 2
= Ú 2 x ◊ e x dx
0

2 1
È∵ e f ( x ) f ¢( x ) dx = e f ( x ) ˘
Î Ú
= ex
0
˚

= e1 - e0
= e -1
9.3 Change of Order of Integration 9.47

Example 6
x2
x −

∫ ∫
y
Change the order of integration and evaluate xe
Solution
1. The function is integrated first w.r.t. y, but y
evaluation becomes easier by changing the
order of integration.
2. Limits of y : y = 0 to y = x.
Limits of x : x = 0 to x Æ •. B′
3. The region is the part of the first quadrant
bounded between the lines y = x and y = 0.
4. To change the order of integration, i.e., to
integrate first w.r.t. x, draw a horizontal strip
parallel to x-axis which starts from the line
y = x and extends up to infinity. O A x
Limits of x : x = y to x Æ •
Limits of y : y = 0 to y Æ •
Hence, the given integral after change of order Fig. 9.57
is
y
x2 x2
x − −

∫ ∫
y y
xe x xe dx y
x
∞ y 2x
=∫ ∫ e y
− y
2 y y
1
2∫
x
=− y y y
e y

∵∫ e f ′ (x x ef x)

O x
1 ∞
2∫
−y
dy
Fig. 9.58
1 y y
0
2
1
2

Example 7
a a x2
Change the order of integration and evaluate ∫∫ y
x2 y2
dx y
9.48 Chapter 9 Multiple Integrals

Solution
1. Since inner limits depend on y, the function is integrated first w.r.t. x but evaluation
becomes easier by changing the order of inte-
y
gration.
2. Limits of x : x = y to x = a, along horizontal Q (a, a)
strip A¢B¢

=a
Limits of y : y = 0 to y = a
3. The region is bounded by the lines y = x, x = a
and y = 0.
A′ B′
4. The point of intersection of y = x and x = a is
x = a, y = a.
O P x
The point of intersection is Q (a, a).
5. To change the order of integration, i.e. to
integrate first w.r.t. y, draw a vertical strip AB Fig. 9.59
parallel to y-axis which starts from x-axis and y
terminates on the line y = x.
Limits of y : y = 0 to y = x Q a a)
Limits of x : x = 0 to x = a
Hence, the given integral after change of order is

a
a
2
x2 B
= dx
x2 y2 x2 y2
O A P x
a x 1
∫ ∫
2
dy
x2 y2 
Fig. 9.60
a x
2
dx
0
a
− og x x 2 dx

a
= ∫ log
(
x 1+ )  2

x 

)∫
a
= + x2 x
0

3 a
= + ) x3
0

= + ) a3
3
9.3 Change of Order of Integration 9.49

Example 8
Change the order of integration and evaluate [Winter 2016]
1 1− x 2 ey y
∫∫
0 0
((ee +
+11) 1 − x − y
y 2 2
dy dx.
Q (0, 1)
B′
x2 + y2 = 1
Solution
P (1, 0)
1. The function is integrated first w.r.t. y, but
O A′ x
evaluation becomes easier by changing the
order of integration.
2. Limits of y : y = 0 to y = 1 − x 2
Limits of x : x = 0 to x = 1
Fig. 9.61
3. Since given limits of x and y are positive, the y
region is the part of circle x2 + y2 = 1 in the
first quadrant.
Q (0, 1)
4. To change the order of integration, i.e., to x2 + y2 = 1
integrate first w.r.t. x, draw a horizontal strip
A B
AB parallel to x-axis which starts from y axis
and terminates on the circle x2 + y2 = 1. O P(1, 0) x
Limits of x : x = 0 to x = 1− y 2

Limits of y : y = 0 to y=1
Hence, the given integral after change of order Fig. 9.62
is
1 1− x 2 ey 1 ey 1− y 2 1
∫∫
0 0
((ee +
y
+11) 1 − x − y
2 2
dy dx = ∫
0 e y + 1 ∫0
((1 − y 2 ) − x 2
dx d y

1− y 2
e y sin −1
1 x
=∫ y dyy
0 e +1 1 − y2 0

ey 1
=∫ (sin −−11 1
1−− sin −1 0)dy
dy
0 e +1 y

1 e
y
π
=∫ y ⋅ dy
dy
0 e +1 2

π  f ′( y) 
∵ ∫ f ( y ) dy
1
= log(e y + 1) 0 dy = log f ( y ) 
2  
π
= og(e +
[log( +11) − log
log 2]
2
π  e + 1
= log 
2  2 
9.50 Chapter 9 Multiple Integrals

Example 9
a a a − y2
Change the order of integration and evaluate ∫∫ 2
y2
d y
Solution
1. Since inner limits depend on y, the func- y
tion is integrated first w.r.t. x.
P (a a)
2
2. Limits of x x y 2 to A′ B
x a + a2 y 2 , along horizontal strip
A¢B¢
Q (2a, 0)
Limits of y : y = 0 to y = a
O a, 0) x
3. The region is bounded by the circle (x –
a)2 + y2 = a2 and the line y = 0. Since
limits of y are positive, the region is the
part of the circle (x – a)2 + y2 = a2 above
x-axis.
4. The points of intersection of the circle
Fig. 9.63
with x-axis are O (0, 0) and Q (2a, 0).
5. To change the order of integration i.e. y
to integrate first w.r.t. y, draw a vertical
strip AB parallel to y-axis which starts B
from x-axis and terminates on the circle
(x – a)2 + y2 = a2
or x2 + y2 – 2ax = 0 Q (2a, 0)

2 O A (a x
Limits of y : y = 0 to
Limits of x : x = 0 to x = 2a
Hence, the given integral after change of
order is
a x2
Fig. 9.64
∫ d y

2a 2 − x2

∫ y
0
2a
= − x2 x
2a
= 2
− x a)2
2a
( x a) 2 a2 x a
= a x a)2 sin −
2 2 a 0

a a2 (0 a a2
= + n 11 0− ( −1)
2 2 2 2
9.3 Change of Order of Integration 9.51

= a 2 sin -1 1 [∵ sin -1
-1
( -1) = - sin -1 (1)]
p
= a2
2
p a2
=
2

Example 10
1 2- x2
x
Evaluate Ú Ú dy dx by changing the order of integration.
0 x x 2 + y2
[Summer 2016]
Solution
1. Since inner limits depend on x, the function is integrated first w.r.t. y.
2. Limits of y : y = x to y = 2 - x 2 along vertical setup A¢B¢
Limits of x : x = 0 to x = 1
3. The region is bounded by y-axis, the line
y = x and the circle x2 + y2 = 2.
4. The point of intersection of the circle
y = 2 - x 2 and y = x is obtained as

x2 = 2 - x2
2 x2 = 2
x2 = 1
x = ±1 Fig. 9.65
\ y=1
Hence, P(1, 1) is the point of intersection.

5. To change the order of integration, i.e. to


integrate first w.r.t. x, divide the region
into two subregions OPR and PQR.
Draw a horizontal strip parallel to x-axis
in each subregion.
(i) In the subregion OPR, strip AB starts
from y-axis and terminates on the
line y = x.
Limit of x: x = 0 to x = y
Limit of y: y = 0 to y = 1 Fig. 9.66
9.52 Chapter 9 Multiple Integrals

(ii) In the subregion PQR, strip CD starts from y-axis and terminates on the circle
y = 2 - x2 .

Limit of x: x = 0 to x = 2 - y 2

Limit of y: y = 1 to y = 2

1 2 - x2
x x x
Ú Ú dy dx = ÚÚ dy dx + ÚÚ dydx
0 x x +y
2 2
OPR x +y
2 2
PQR x + y2
2

2
1 y 2 2- y
x x
= ÚÚ dy dx + Ú Ú dy dx
00 x +y
2 2
1 0 x + y2
2

y 2 - y2
È 1 ˘ È 1 ˘
1 Í ( x 2 + y2 ) 2 ˙ 1 Í ( x 2 + y2 ) 2 ˙
1 2
=Ú Í ˙ dy + Ú Í ˙ dy
2Í 1 ˙ 2Í 1 ˙
0 1
ÎÍ 2 ˚˙ 0 ÎÍ 2 ˚˙ 0
1 2
= Ú ÈÎ 2 y - y ˘˚ dy + Ú ÈÎ 2 - y ˘˚ dy
0 1

1 2
y2 È y2 ˘
= ( 2 - 1) + Í 2y - ˙
2
0 ÎÍ 2 ˙˚
1

Ê1 ˆ Ê 1ˆ
= ( 2 - 1) Á - 0˜ + Á 2 - 1 - 2 + ˜
Ë2 ¯ Ë 2¯

2 1 3
= - + - 2
2 2 2
1
= 1- 2 +
2
1
= 1-
2
9.3 Change of Order of Integration 9.53

Example 11
Sketch the region of integration, reverse the order of integration and
2y
2 4 - x 2 xe
evaluate Ú0 Ú0 dy dx. [Summer 2014]
4-y

Solution
1. Since inner limit depends on x, the
function is integrated first w.r.t. y.
2. Limits of y : y = 0 to y = 4 – x2
along vertical strip A¢B¢
Limits of x : x = 0 to x = 2
3. The region is bounded by the parabola
Fig. 9.67
x2 = 4 – y, x-axis and y-axis.
4. To change the order of integration, i.e., to
integrate first w.r.t. x, draw a horizontal
strip parallel to x-axis which starts from
y-axis and terminates on the parabola
x2 = 4 – y.
Limits of x : x = 0 to x = 4− y
Limits of y : y = 0 to y=4
Hence, the given integral after change of order is Fig. 9.68

2 4 - x2 xe2 y 4 4 - y xe
2y

Ú0 Ú0 4-y
dydx = Ú Ú
0 0 4-y
dx dy

4 e2 y È 4 - y ˘

4 - y ÍÎ Ú0
x dx ˙ dy
0
˚
4- y
4 e2 y x 2
=Ú dy
0 4-y 2
0

4 e2 y È 1 ˘
=Ú Í ( 4 - y ) ˙ dy
0 4 - y Î2 ˚
9.54 Chapter 9 Multiple Integrals

4 e2 y
=Ú dy
0 2
1 2y 4
= e
4 0

1
= (e8 - 1)
4

Example 12
1 2− x
Change the order of integration and evaluate ∫∫
0 x2
xy dx dy.

Solution
1. Since inner limits depend on x, the function P ′(−2, 4) y
is integrated first w.r.t. y.
The correct form of integral
1 2− x
=∫ ∫ xxyy dy dx Q (0, 2)
x2
0
x2 = y
2
2. Limits of y : y = x to y = 2 – x, along B′
vertical strip A¢B¢ P (1, 1)
Limits of x : x = 0 to x = 1 A′ x +y=2
3. The region is bounded by y-axis, the line O x
x + y = 2 and the parabola x2 = y. Since given
limits of x and y are positive, the region lies Fig. 9.69
in the first quadrant.
4. The points of intersection of x + y = 2 and P ′(−2, 4) y
x2 = y are obtained as
x2 = 2 – x
2
x +x–2=0
Q (0, 2)
(x – 1) (x + 2) = 0 x2 = y
C D
x = 1, –2
y = 1, 4 R P (1, 1)
A B x +y=2
The points of intersection are P (1, 1) and O x
P' (–2, 4).
5. To change the order of integration, i.e., to Fig. 9.70
integrate first w.r.t. x, divide the region into two subregions OPR and RPQ. Draw a
horizontal strip parallel to x-axis in each subregion.
(i) In subregion OPR, strip AB starts from y-axis and terminates on the parabola
x2 = y.
Limits of x : x = 0 to x = y
Limits of y : y = 0 to y = 1
9.3 Change of Order of Integration 9.55

(ii) In subregion RPQ, strip CD starts from y-axis and terminates on the line
x + y = 2.
Limits of x : x = 0 to x = 2 – y
Limits of y : y = 1 to y = 2
Hence, the given integral after change of order is
x 2 y
∫∫ y x dx ∫ xy dx y
y 2 y
1 x2 2 x2
+∫ y y
0 2 0
1
0

1 2
= y
2 1

1 2
= + 4 y
2 1

1 4 2
1 3 2 3

2 3 0 2 2 3 4 1
1 1 1 32 4 1
+ − + −
2 3 2 3 3 4
1 5
= +
6 24
9
24
3
=
8

Example 13
4a 2 ax
Change the order of integration and evaluate ∫ ∫ x2
4a
x.
[Winter 2014]
Solution y

1. Since inner limits depend on x, the y 2 = 4ax


function is integrated first w.r.t. y. P(4a, 4a)
x2 B′
2. Limits of y y to x
4a x 2 = 4ay
along vertical strip A¢B¢ A′
3. The region is bounded by the parabolas O x
x2 = 4ay and y2 = 4ax.
4. The points of intersection of x2 = 4ay
and y2 = 4ax are obtained as
x4 = 16a2y2
=16a2 (4ax)
Fig. 9.71
9.56 Chapter 9 Multiple Integrals

x(x3 – 64a3) = 0
x = 0, x = 4a
\ y = 0, y = 4a
The points of intersection are O (0, 0) and P (4a, 4a).
5. To change the order of integration, i.e., to integrate first w.r.t. x, draw a horizontal
strip AB parallel to x-axis which starts from the parabola y2 = 4ax and terminates on
the parabola x2 = 4ay.
y
y2
Limits of x : x = to x = 2 aay
4a y 2 = 4ax
Limits of y : y = 0 to y = 4a P(4a, 4a)
Hence, the given integral after change of
x 2 = 4ay
order is A B
4a 2 ax 4a 2 ay
Ú0 Ú x 2 xy dy dx = Ú
xy
0 Úy 2 dx dy O x
4a 4a
4a 2 ay
=Ú x dy
0 y2
4a

4a Ê y2 ˆ
=Ú Á 2 ay - dy Fig. 9.72
0 Ë 4 a ˜¯
4a
3
4a
y2 1 y3
=2 a -
3 4 3
4a
0
2 0
3 3
4 1
= a ( 4) 2 a 2 - (64 a 3 )
3 12 a
32 2 16 2
= a - a
3 3
16 2
= a
3

Example 14
Change the order of integration and evaluate
a a−
a − a2 − y2 xy log( x + a )
∫∫
0 0 ( x − a)2
dx dy.

Solution
1. The function is integrated first w.r.t. x, but evaluation becomes easier by changing
the order of integration.
9.3 Change of Order of Integration 9.57

2. Limits of x : x = 0 to x = a − a 2 − y 2 y
y=a
Limits of y : y = 0 to y = a P (a, a)
Q
3. The region is bounded by the circle A′ B′
(x − a)2 + y 2 = a 2
(x – a)2 + y2 = a2, the lines y = a and x = 0.
4. The point of intersection of x
O
(x – a)2 + y 2 = a2 and y = a is obtained as
(x – a)2 + a2 = a2
x=a
The point of intersection is P (a, a). Fig. 9.73
5. To change the order of integration, y
i.e., to integrate first w.r.t. y, draw a
y=a
vertical strip AB parallel to y-axis which B P((a, a)
starts from the circle (x – a)2 + y2 = a2 and Q
terminates on the line y = a. A (x − a)2 + y 2 = a 2

Limits of y : y = 2a
axx − x 2 to y=a
O x
Limits of x : x = 0 to x = a
Hence, the given integral after change of
order is
Fig. 9.74
a 2
a− a − y 2
xy log( x + a ) a a x log( x + a )
∫∫
0 0 ( x − a) 2
dx d y = ∫ ∫
0 2 ax − x 2
( x − a)2
y dy d x

a
a x log( x + a ) y 2
=∫ dx
dx
0 ( x − a)2 2 2 ax − x 2

a x log( x + a )  a 2 − 2aaxx + x 2 
=∫  dx
dx
0 ( x − a ) 2  2
1 a
2 ∫0
= x log( x + a)
a )dx

1  x2 
a 2
a x 1
=  log( x + a ) − ∫ ⋅ dx 
dx
2 2 0 2 x+a 
 0 
1  a2 1 a a2  
=  log 2a − ∫0 ( x − a ) +  dx
dx 
2 2 2  x + a 

1  a2 
a
1 x2
=  log 2a − − ax + a 2 log( x + a ) 
2 2 2 2 0
 
1  a 2

=  a 2 log 2a − + a 2 − a 2 log 2a + a 2 log a
4 2 
9.58 Chapter 9 Multiple Integrals

1  a2 
=  + a 2 log a
4 2 
a2
= (1 + 2 log a )
8

Example 15
Change the order of integration and evaluate
1
1 4 y2
1− 1 + x2
∫ ∫
0
2
0
1 − x2 1 − x2 − y 2
dx dy.
y

y
Solution 1
Q 0,
1. The function is integrated first w.r.t. 2
x, but evaluation becomes easier by x 2+ 4y 2 = 1
changing the order of integration. A′ B′
x
O P (1, 0)
2. Limits of x : x = 0 to x = 1 − 4 y2
1
Limits of y : y = 0 to y=
2
3. Since the limits of x and y are positive,
the region is the part of the ellipse in Fig. 9.75
the first quadrant. y
4. To change the order of integration, i.e.,
to integrate first w.r.t. y, draw a verti- 1
Q 0,
cal strip AB parallel to y-axis which 2
B
starts from x-axis and terminates on x 2+ 4y 2 = 1
the ellipse x2 + 4y2 = 1.
x
O A P (1, 0)
1
Limits of y : y = 0 to y= 1 − x2
2
Limits of x : x = 0 to x=1
Hence, the given integral after change of
order is Fig. 9.76

1 1
1- 4 y2 1 + x2 1 1 + x2 1- x 2 1
Ú Ú
2
0 0
dxdy = Ú
0 2 Ú 0
2 dydx
1 - x 2 1 - x 2 - y2 1- x (1 - x 2 ) - y 2
1
1- x 2
1 1 + x2 y 2
=Ú sin -1 dx
0
1- x 2
1 - x2 0
9.3 Change of Order of Integration 9.59

1 1 + x2
=Ú sin - s -1
0 dx
1- x 2 2
12 1- x ) p
=Ú ◊ d
1 x2 6

p 1
=
6 Ú 2
1 x2 x
1 x
1
p - 1 -1
= - x - x
6 2 2 0

x a2 x
∵ Ú a2 - x2 d 2
x2 + n
2 2 a
p 3 -1
= sin 1
6 2
p p
= ◊
4 2
p2
=
8

Example 16
Change the order of integration and evaluate
a y x dx
∫∫ (a )( y )( y x)

Solution
y
1. The function is integrated first w.r.t. x, but
evaluation becomes easier by changing the
order of integration. y=a
2. Limits of x : x = 0 to x = y P (a, a)
Limits of y : y = 0 to y = a
3. The region is bounded by the line y = x,
y = a and x = 0.
4. The point of intersection of y = a and y = x A′ B′
is P (a, a).
5. To change the order of integration, i.e., to
integrate first w.r.t. y, draw a vertical strip O x
AB parallel to y-axis which starts from the
line y = x and terminates on the line y = a.
Fig. 9.77
9.60 Chapter 9 Multiple Integrals

Limits of y : y = x to y = a y
Limits of x : x = 0 to x = a
Hence, the given integral after change of order
is B =a
P (a, a)
a y x dx
∫∫ (a )( y )( y x)
a a x dx
=∫ ∫ (a x )( y )( y x) A
a y 

a
=∫ x
dx
a2 x2 (a )( x) O x

Putting y – x = t2, dy = 2t dt
When y = x, t = 0 Fig. 9.78

When y = a, x
a a x 2t t
∫0
a x ( y y x
=
a x ( x t 2
dx

a x a x dt
= 2∫ ∫ dx
2 2
0
a x ( x) t 2
a−x
a x t
= 2∫ sin −1 dx
0
a2 x2 a x 0

a x
=2 − sin 0)dx
a − x2
2

a  1 −
1

2∫
= ⋅ ( x ) 2 ( 2x) d
2
a
p
1
È [ ( x )]n +1 ˘
- 2( - )2 ∵Ú ( x )]n f ( )dx =
2 n +1
0
pa

Example 17
Change the order of integration and evaluate
1
1 y
∫∫ x
x ( x y )
Solution
1. The function is integrated first w.r.t. y, but evaluation becomes easier by changing
the order of integration.
9.3 Change of Order of Integration 9.61

1 y
2. Limits of y : y = x to y
x
Limits of x : x = 0 to x=1 xy 1
3. The region is bounded by the rectangular
hyperbola xy = 1, the line y = x and y-axis B′
in the first quadrant.
4. The point of intersection of xy = 1 and (1, 1)
y = x in the first quadrant is obtained as
A′
x2 = 1
x=1
\ y=1 O x
The point of intersection is P (1, 1).
5. To change the order of integration, i.e., to Fig. 9.79
integrate first w.r.t. x, divide the region y
R
into two subregions OPQ and QPR. Draw
a horizontal strip parallel to x-axis in each
subregion.
(i) In subregion OPQ, strip AB starts from C D
y-axis and terminates on the line y = x. xy 1
Limits of x : x = 0 to x=y
Limits of y : y = 0 to y=1 Q (1, 1)

(ii) In subregion QPR, strip CD starts from


y-axis and terminates on the rectangular
hyperbola xy = 1. O x
1
Limits of x : x = 0 to x Fig. 9.80
y
Limits of y : y = 1 to y → ∞

Hence, the given integral after change of order is


1 1
1 y y y 1 y 1
∫∫ x
x= ∫ 1+
dx y + y
1+ 2
d y

1
y
∞ 1 y

∫ +
1 1+ y y xy ) 0
y
0

∞ 1 1
=− − 1 y
1 1+ y 2
1  1 1  1 ∞ 1
= −∫ − y
1+ y +y y
9.62 Chapter 9 Multiple Integrals

Putting y = tan q in the first term of first integral, dy = sec2q dq,


When y = 0, q = 0
π
When y = 1, θ =
4
π
1
1
y sec 2 θ dθ 1 1 ∞
∫∫
0
x
x ((1
1+
1 + xy ) (1 + y )
2 2
d y d x = − ∫0
4
sec θ4
+ tan −1 y 0 + tan −1 y 1
tan
2
π
(1
1++ cos 2θ ) 1
= −∫ 4 dθ + (tan −−11 1 − tan −1 0) + (tan −−11 ∞ − tan −1 1)
0 2 2
π
1 sin 2θ 4 π 1π π
=− θ+ + +  − 
2 2 0 4 2  2 4
π 1 π 3π
= − − sin +
8 4 2 8
π −1
=
4

Example 18
Change the order of integration and evaluate
1 1− y 2 cos −1 x
∫∫
0 0
1 − x2 1 − x2 − y 2
dx dy.

Solution
1. The function is integrated first w.r.t. x, but y
evaluation becomes easier by changing the
order of integration. Q (0, 1)

2. Limits of x : x = 0 to x = 1 − y 2
A′ B′
Limits of y : y = 0 to y = 1 x2 + y2 = 1

3. Since given limits of x and y are positive, the O P (1, 0) x


region is the part of the circle x2 + y 2 = 1 in
the first quadrant.
4. To change the order of integration, i.e., to
integrate first w.r.t. y, draw a vertical strip
AB parallel to y-axis in the region. AB starts
Fig. 9.81
from x-axis and terminates on the circle
x2 + y 2 = 1.
Limits of y : y = 0 to y = 1 − x 2
Limits of x : x = 0 to x = 1.
9.3 Change of Order of Integration 9.63

Hence, the given integral after change of order is y


1
1 1 y 2
cos x
∫∫ 1 − x2 y2
dx y
Q
B
1 1− x 2 cos 1 x 1
=∫ ∫ x x2 + y2 1
2 2
1 ) y
1 x2 O A P (1, 0) x
1 cos 1 x
=∫ sin −
dx
1 − x2 0

1
1 cos x
=∫ (sin −1 n − )dx
1− x 2
Fig. 9.82
1 1
2∫
=− cos −1 x − dx
1 − x2
2 1
(cos 1 x ( x)]2 
− ∵ ∫ f ( ) f ( )d x =
2 2 0
2
π
− (cos 1 2

4
2
π π
− 0−
4 2
3

16

EXERCISE 9.3
Change the order of integration of the following integrals:
6 2+ x
1. ∫∫ 2 x
y y dx

 6


1 2x
2. ∫∫ x
y y dx

 y 1 
Ans. :
0 ∫ y
2 2

1 y
3. ∫∫ y
y x dy

y x dy 
1 1
Ans. ∫ ∫ 2
9.64 Chapter 9 Multiple Integrals

y
a
4. ∫ ∫
a
a
y x dy
a a
Ans.
ax

a 2a x
5. ∫∫ x2
a
y y dx


:

3 y
6. ∫ ∫
2 y 6
y x dy

x x 
Ans. :
x

1 y
7. ∫∫ y
y

 x x x2 
: ∫ 2

6 x
2
8. ∫∫ 2
x2 +4
x
4


Ans. :
1 ∫
2 x a
9. ∫∫ 4 x
y y dx

 Ans. : 2 6 +2 2
y
0 ∫ 4 2 ∫
a y +a
10. ∫ ∫ 0 a2 − y 2
f (x , y )dxdy

a a a

Ans. ∫ x a

2a 2 ax
11. ∫ ∫ 2 ax − x
y x dy

 :
2 2
a a 2a 2a
∫ 2a
∫ ∫ y
2a
f x y
9.3 Change of Order of Integration 9.65

a
12. ∫∫ 4
y y dx

a

Ans. 2
∫ 2

a2
a
13. ∫∫ x
x
y y dx
 y
a2 
Ans. :

b mx
14. ∫ ∫ k
x
y y dx

 k
ma b
 Ans. :
a
k ∫ k y y
 b y a m 
x
1 e
15. ∫∫ y y dx

y
e 1
Ans. : ∫ ∫ gy

2 x3
16. ∫∫ y y dx
 8 2 
Ans. ∫∫ 1 y x y
y3

1+ x 2
1
1 4y2
17. ∫ ∫
2

 1
1 x2
1+ x 2 2 
Ans. : ∫ ∫ 2 
1 x y 3
2
x
2 x
18. ∫∫ 2

 2 2 x 1
Ans. ∫∫ 2y
1 4
4)

a a x
19. ∫∫ y

a
4
9.66 Chapter 9 Multiple Integrals

9.4 DOUBLE INTEGRALS IN POLAR COORDINATES


θ2
r2
The integral ∫∫
θ1
r1
f ( r , θ )drdθ represents the polar form of the double integration. This

integral is first integrated w.r.t. r keeping q constant and then the resulting expression
is integrated w.r.t. q.
R
Limits of Integration
If the limits of integration are not given B
then these limits are obtained from = q2 r r (q )
the equations of the given curves. Let
the region be bounded by the curves Q
r = r1(q ), r = r2 (q ) and the lines S
q = q1, q = q2. A
dq = q1
The region of integration is PQRS.
Draw an elementary radius vector AB q2 P
from origin which enters in the region r r (q )
q
from the curve r = r1(q ) and leaves at O =0
the curve r = r2(q ). Therefore, limits for
Fig. 9.83
r are r1(q ) to r2(q ).
To cover the entire region PQRS, rotate elementary radius vector AB from PQ to RS.
Therefore, q varies from q1 to q 2.
(
∫∫ f r, r
1 ( )
f (r r

Type I Evaluation of Double Integrals in Polar Coordinates

Example 1
1
Evaluate ∫ ∫4

Solution
1
∫ ∫
0
4
r rd r r d
1
r2

0
4
2 0
d

1

0
4
2
d

1
4
0
2
9.4 Double Integrals in Polar Coordinates 9.67

1 π
= ⋅
2 4
π
=
8
Another method: Since both the limits are constant and integrand (function) is
explicit in r and q, the integral can be written as
π π
1 1
∫ ∫
0
4
0
r dr dθ = ∫ 4 dθ ⋅ ∫ r dr
0 0
1
π
r2
= θ 04 ⋅
2 0

π 1
= ⋅
4 2
π
=
8

Example 2
π sin θ
Evaluate ∫ ∫
0 0
r dr dθ.

Solution
π sin θ π sin θ
∫ ∫ r dr dθ = ∫  ∫ r dr  dθ
0 0 0 
 0 
sin θ
π r2
=∫ dθ
0 2 0

1 π 2
2 ∫0
= sin θ dθ

1 π 1 − cos 2θ
= ∫ dθ
2 0 2
π
1 sin 2θ
= θ−
4 2 0

1 sin 2π 
= π − 
4 2 
π
=
4
9.68 Chapter 9 Multiple Integrals

Example 3 p
2 1 - sin q r 2
Evaluate the integral Ú Ú
0 0
cos q dr dq . [Summer 2014]
Solution
p p
1 - sin q 2 È 1 - sin q 2
r dr ˘˙ cos q dq
Ú Ú
2
0 0
r cos q dr dq = Ú ÍÎÚ0
0
2
˚
p 1 - sin q
r3
= Ú
0
2
3
cos q dq
0
p
1
=
3 Ú 0
2 (1 - sin q )3 cos q dq
p
1 (1 - sin q )4 2
=
3 4
0

1 È 1˘
= Í0 - 4 ˙
3 Î ˚
1
= -
12

Example 4
π
2 a cos θ
Evaluate ∫ ∫
2
0 0
r 2 sin θ dθ dr.

Solution
Since inner limits depend on q, the function is integrated first w.r.t. r.
π
2 a cos θ
The correct form of the integral = ∫02 ∫0 r 2 sin θ dr dθ

π π
2 a cos θ 2 a cos θ
∫ ∫
2
r 2 sin θ dr dθ = ∫ 2  ∫ r 2 dr  ssin
in θ dθ
0 0 0  0 
π os θ
2 a cco
r3
=∫ 2
sin θ dθ
0 3 0
π
8a 3 2
3 ∫0
= coss3 θ sin
sin θ dθ
π
8a 3 2
3 ∫0
=− coss3 θ ( − sin
sin θ ) dθ
9.4 Double Integrals in Polar Coordinates 9.69

p
8a 3 4
q 2 f (q )]n +1 ˘
=- ∵Ú f ¢ )dq , n π -1
3 4 n +1
0
3
8a
=- s4 - cos 4 0
12 2
2 3
= a
3

Example 5
a (1 sin )
∫ ∫
2 2
Evaluate
Solution
Since inner limits depend on q, the function is integrated first w.r.t. r.
(1+ sin )
The correct form of the integral ∫ ∫ 2
0
r2 r

a a sin )
∫ ∫
2
r r r 2 r cos
3 a (1 sin )

∫ 2
3
cos
0

p
1
+s cos q dq
3 0
p
[ ( )]n +1
a3 s 4 2
f ) dq ,
∵Ú n +1
3 4
0 n π -1
4
a3 π
= + sin − + 0) 4
12

a3
= −1
12
5
= a3
4

Example 6
cos 2 r
Evaluate ∫ ∫
4
( r 2
dr d
9.70 Chapter 9 Multiple Integrals

Solution
p
cos 2q 1 2r
Ú Ú 4
0 0

2 (1 + r 2 )2
dr dq

p
1 4 È cos 2q ˘
= Ú Í Ú (1 + r 2 )-2 ◊ 2r dr ˙ dq
2 0 Î 0 ˚
È [ f (r )] ˘
n +1

dq Í∵ Ú [ f (r )] f ¢(r ) dr = n + 1 ˙
p n
1 4 cos 2q Í ˙
= Ú -(1 + r 2 )-1
2 0 0
Í n π -1 ˙
Î ˚
p
1 4Ê 1 ˆ
2 Ú0 ÁË 1 + cos 2q ˜¯
=- - 1 dq

p
1 4Ê 1 ˆ
2 Ú0 ÁË 2 cos2 q ˜¯
=- - 1 dq

p
1 4Ê1 ˆ
=-
2 Ú0 Á
Ë 2
sec 2 q - 1˜ dq
¯
p
1 1 4
=- tan q - q
2 2 0
1Ê1 p pˆ
= - Á tan - ˜
2Ë2 4 4¯
1
= (p - 2)
8

Example 7
p p
Evaluate Ú Ú
2 2
0 0
sin(q + f )dq df.

Solution
p p p È p ˘
Ú Ú 2 2
0 0
f ) dq df = Ú 2 Í Ú 2 ssin((q + f ) dq ˙ df
sin(q + f)
0
ÍÎ 0 ˙˚
p
p 2
=Ú 2 - cos(q + f ) df
0 0
p
È Êp ˆ ˘
= -Ú 2
Ícoss ÁË + f ˜¯ - cos f ˙ df
0
Î 2 ˚
p
= - Ú 2 ( - sin f - cos f ) df
0
9.4 Double Integrals in Polar Coordinates 9.71

p
= - coss f - ssin
in f 2
0

Ê p p ˆ
= - Á coss - sin - coss 0 + ssin
in 0˜
Ë 2 2 ¯
=2

Type II Evaluation of Double Integrals Over a Given Region


in Polar Coordinates

Example 1
3
Evaluate ÚÚr sin 2q dr dq over the area bounded in the first quadrant
R
between the circle r = 2 and r = 4. [Summer 2016]
Solution
1. The region of integration is the interior of the
circle between r = 2 to r = 4.
2. Draw an elementary radius vector OAB from the
origin which enters in the region from the circle
r = 2 and leaves at the circle r = 4.
3. Limits of r: r = 2 to r = 4
p
Limits of q: q = 0 to q =
2
I = ÚÚ r 3 sin 2q dr dq Fig. 9.84

p
2 4
= Ú Úr
3
sin 2q dr dq
0 2
4 p
r4 cos 2q 2
= -
4 2 0
2

1È 4 Ê 1ˆ
= Î(4) - (2)4 ˘˚ Á ˜ (cos p - cos 0)
2 Ë 2¯
1 Ê 1ˆ
= [256 - 16] Á - ˜ (-2)
2 Ë 2¯
1
= [240]
2
= 120
9.72 Chapter 9 Multiple Integrals

Example 2
Evaluate ÚÚ r a2 2
q over the upper half of the circle
r = a cos q. [Summer 2017]

Solution q p
2
1. The region of integration is the upper half
of the circle r = a cos q. A
2. Draw an elementary radius vector OA which r = a cos q
starts from the origin and terminates on the
circle r = a cosq.
3. Limits of r : r = 0 to r = a cosq
O q =0
Limits of q : q = 0 to =
2
2
I r 2 dr dq
p 1
a q 1
- (a ) (-2r
2 r dq Fig. 9.85
2
a q
3
(r )]n +1
p
1

- 2 [ (r )] )dr
n 1
p
1 2

- ( s -a ) q

p
a 3 3q
-
3 Ú 2
4
- 1 dq

p
a 1 cos 3q 2
- + q
4 3 0

3
a 3 p p p 3 1
=- + cos - + co - cos 0
3 4 2 12 2 2 4 12
a3 p 3 1
=- +
3 2 4 12
a3 2 p
=- -
3 3 2
9.4 Double Integrals in Polar Coordinates 9.73

Example 3
4 3
Evaluate ÚÚ over the interior of the circle r = 2a cos q.

Solution
1. The region of integration is the interior of q p
the circle r a cos
A
2. Draw an elementary radius vector OA
r = 2 cos q
which starts from the origin and termi-
nates on the circle r a cos
3. Limits of r r a cos
O q 0

Limits of =− to
2 2

I r 4 cos3 q r q
p
2 a cosq 4 Fig. 9.86
=Ú 2
p Ú r 3
r q
-
2

p 2a q
r5
=Ú 2
p s3 q q
5
2 0
p
1 2
5Ú p
= (2 cos q cos q dq
2
p
32 a 5
= ◊ 2 Ú 2 cos8 cos8 = co 8
5 0

64 a 5 7 5 3 1 p
= ◊ [Using reduction formula]
5 8 6 4 2 2
7p 5
a
4

Example 4
2
Evaluate ÚÚ r r q over the cardioid r = a (1 + cos q ) above the
initial line.
9.74 Chapter 9 Multiple Integrals

Solution
1. The region of integration is the part of the cardioid r = a (1 + cos q ) above the initial
line (q = 0).
2. Draw an elementary radius vector OA which starts from the origin and terminates
on the cardioid r = a (1 + cos q ).
3. Limits of r : r = 0 to r = a (1 + cos q )
Limits of q : q = 0 to q = p q=p
2

I r dq A
r (1 + cos q )
p a (1+ cosq
Ú Ú r2 n r dq
a 1+ cosq ) O q =0
p r3
Ú 3 0
nq d

1 p
cos q ◊ dq
3
a3 p Fig. 9.87
=-
3 Ú (1 + - n )dq

p
a3 + cos 4
(q )]n +1 ˘
=- ∵Ú f )dq
3 4 n +1
0
3
a
=- ( + - 4
12
a3
=-
12
4 2
= a
3

Example 5
q
Evaluate ÚÚ 2 2
over one loop of the lemniscate r2 = a2 cos 2q.
r a
Solution
1. The region of integration is one loop of the lemniscate r2 = a2 cos 2q bounded
between the lines =− and .
4 4
9.4 Double Integrals in Polar Coordinates 9.75

2. Draw an elementary radius vector OA which q= p


2
starts from the origin and terminates on the
q= p
lemniscate r 2 = a2 cos 2q. 4
r 2 = a 2 cos 2q
3. Limits of r: r = 0 to r = a cos 2θ A
q=0
π π
Limits of θ : θ = − to θ = O
4 4
r dr dq
I = ÚÚ q=− p
4
r 2 + a2
p
a cos 2q r d r dq Fig. 9.88
= Ú 4p Ú
- 0
4 r 2 + a2
p 1
a cos 2q 1 2 -
=Ú 4
p 0 Ú (r + a 2 ) 2 ((2r )d
)dr dq
- 2
4

p 1 a cos 2q
1 È [ f (r )]n +1 ˘
= Ú 4p 2((rr 2 + a 2 ) 2 dq Í∵ Ú [ f (r )] f ¢(
n
¢ ( r ) dr = , n π - 1˙
2 - ÎÍ n +1 ˚˙
4 0

1
p È 1 ˘

= 4
p 2a Í(cos 2q + 1)
1) 2 - 1˙ dq
-
4
ÍÎ ˙˚
p
= a Ú 4p
-
( 2 cos q - 1 dq )
4
p
=a 2 sin q - q 4
p
-
4

È p p Ê p ˆ Ê p ˆ˘
= a Í 2 sin - - 2 sin Á - ˜ + Á - ˜ ˙
Î 4 4 Ë 4¯ Ë 4¯˚
Ê pˆ
= aÁ2- ˜
Ë 2¯
a
= (4 - p )
2

Example 6
2
Evaluate ÚÚ r dr dq over the area between the circles r = a sin q and
r = 2a sin q.
Solution
1. The region of integration is the area bounded between the circle r = a sin q and
r = 2a sin q.
9.76 Chapter 9 Multiple Integrals

2. Draw an elementary radius vector OAB q= p


2
from the origin which enters in the region
from the circle r = a sin q and leaves at
the circle r = 2a sin q.
3. Limits of r : r = a sin q to r = 2a sin q B
Limits of q : q = 0 to q = p
I rd r = 2 sin q

2a n A
∫ ∫ a n
r2 d
2a n r = a sin q
r3
∫ 3
d
a sin O q =0
1
3∫
(8a n 3 s d
Fig. 9.89
7a3
3 ∫
sin 3

7a3 3 3
3 ∫
d
4
7a3 co 3
= −3 co +
12 3 0

7a 3
1
= −3 − + 0)
12 3
7 a 3 16
=
12 3
28 3
= a
9

EXERCISE 9.4
Evaluate the following integrals:
r

∫∫ re a co
2
1. in dr dq over the upper half of the circle r = 2a cos q.

a2 1
Ans. : 3+ 4
16 e

2. ∫∫ r r q over the region between the circles r = 2 sin q and r = 4 sin q.

 45π
Ans.
2
9.5 Change of Variables 9.77

3. ∫∫ r sinq dA over the cardioid r = a (1+ cos q ) above the initial line.
 4 3
 Ans. : 3 a 
 
r
4. ∫∫ r +4
2
dr dq over one loop of the lemniscate r2 = 4 cos 2q.

Ans. :((4 − π )

9.5 CHANGE OF VARIABLES

9.5.1 Change of Variables from Cartesian to


Polar Coordinates
The double integral can be changed from Cartesian coordinates (x, y) to polar
coordinates (r, q ) by putting x = r cos q, y = r sin q. Then ∫∫ f ( x, y )dy dx
dx =
∫∫ f (r cos θ , r ssin
in θ ) J dr dθ where J is the Jacobian (functional determinant)
defined as
∂x ∂x
∂ ( x , y ) ∂r ∂θ
J= =
∂ ( r , θ ) ∂y ∂y
∂r ∂θ
coss θ − r sin θ
=
sinn θ r cos θ
= r (cos 2 θ + sin 2 θ )
=r
Hence, ∫∫ f ( x, y)dy dx
dx = ∫∫ f (r cos θ , r sin
sin θ ) r dr dθ

= ∫∫ f (r cos θ , r ssin
in θ )r dr dθ

Example 1
1 − x2 − y 2
Evaluate ∫∫ 1 + x2 + y 2
dx dy over the first quadrant of the circle

x2 + y 2 = 1.
Solution
1. Putting x = r cos q, y = r sin q, polar form of the circle x2 + y2 = 1 is obtained as r = 1.
2. The region of integration is the part of the circle r = 1 in the first quadrant.
9.78 Chapter 9 Multiple Integrals

3. Draw an elementary radius vector OA which q= p


2
starts from the origin and terminates on the
circle r = 1.
4. Limits of r : r = 0 to r = 1 A
π
Limits of q : q = 0 to θ = r=1
2
Hence, the polar form of the given integral is O q =0

1− x − y 2 2
I = ∫∫ dx dy
1 + x2 + y 2
π
1 1− r2
= ∫2∫ r dr dθ
0 0 1+ r2
Putting r 2 = cos2t, 2r dr = – 2 sin 2t dt Fig. 9.90
π
When r = 0, t =
4
When r = 1, t = 0
π
0 1 − cos 2t
I = ∫ 2 ∫π t )dθ
( − sin 2t dt)
0
4 1 + cos 2t
π π
2 sin 2 t
=∫ 2
∫ 4
sin 2t dt dθ
0 0 2 cos 2 t
π π
sin t
= ∫2∫4 cos t dt dθ
⋅ 2 sinn t cos
0 0 cos t
π π
= ∫ 2 dθ ∫ 4 (1 − cos 2t )dt
0 0
π
π
sin 2t 4
=θ 2
0
t− dθ
2 0

π  π 1
=  − 
2  4 2
π
= (π − 2)
8

Example 2
4 xy - x 2 - y2
Evaluate ÚÚ x 2 + y2 e dx dy over the region bounded by the circle
x2 + y 2 – x = 0 in the first quadrant.
Solution
1. Putting x = r cos q, y = r sin q, polar form of the circle x2 + y2 – x = 0 is r 2 – r cos q = 0,
r = cos q.
9.5 Change of Variables 9.79

2. The region of integration is the part of q p


2
the circle r = cos q in the first quadrant.
3. Draw an elementary radius vector
A
OA which starts from the origin and
terminates on the circle r = cos q.
4. Limits of r : r = 0 to r = cos q r = cos q

Limits of q : q = 0 to O q =0
2
Hence, the polar form of the given integral
is
4 xy
I = ∫∫ 2
2 2

2
e x y
x y
Fig. 9.91
co 4r 2 co sin
=∫ 2
∫ e r2
r dr d
r2
co

= −2 ) dr d

= −2∫ co
2
sin e − r r ef r)

= −2 )d
= −∫ 2 e cos
(2 2

2 cos 2 2
=−e +
2 0

os 0
=− e + −e −
2 2
1
=
e

Example 3
x 2 y2
Evaluate ÚÚ ( x
y )
over the region bounded by the circles
x2 + y2 = a2 and x2 + y2 = b2 (a > b).
Solution
1. Putting x = r cos q, y = r sin q, polar form of
(i) the circle x2 + y2 = a2 is r2 = a2, r = a.
(ii) the circle x2 + y2 = b2 is r 2 = b2, r = b.
9.80 Chapter 9 Multiple Integrals

2. The region of integration is the part bounded q= p


2
between the circles r = a and r = b.
3. Draw an elementary radius vector OAB
from the origin which enters in the region B
from the circle r = b and terminates on the r a
circle r = a.
A r=b
4. Limits of r : r = b to r = a
Limits of q : q = 0 to q = 2p O =0
Hence, the polar form of the given integral is
x2 y 2
I ÚÚ ( x y )
dx y

2p a r 4 co sin q
◊ r r dq
b r2
a
Fig. 9.92
4
2p
Ú cos s q
4
dq
b
4
sin 2q (a
2p )
◊ dq
4 4
4
a 2 ( 4q )
16 Ú 2
d

2p
Êa 4 ˆ n 4q
q-
32 4 0

Êa 4 ˆ
(2p )
32
p
- )
16

Example 4
( )2
Evaluate ÚÚ x 2 y2
over the region common to the circles
x2 + y 2 = ax and x2 + y2 = by (a, b > 0).
Solution
1. Putting x = r cos q, y = r sin q, polar form of
(i) the circle x2 + y 2 = ax is r 2 = ar cos q, r = a cos q.
(ii) the circle x2 + y 2 = by is r 2 = br sin q, r = b sin q.
2. The region of integration is the common part of the circles r = a cos q and
r = b sin q.
9.5 Change of Variables 9.81

3. The point of intersection of the circle q= p


2
r = a cos q and r = b sin q, is obtained
as r = b sin q

b sin θ = a cos θ q = tan−1 a


b
a P
tan θ = B
b r = a cos q
a A
θ = tan −1
b
O q=0
a
Hence, θ = tan −1 at P.
b
4. Divide the region into two subregions
OAP and OBP. Draw an elementary
Fig. 9.93
radius vector OA and OB in each
subregion.
(i) In subregion OAP, elementary radius vector OA starts from the origin and
terminates on the circle r = b sin q.
Limits of r : r = 0 to r = b sin q
a
Limits of q : q = 0 to θ = tan −1
b
(ii) In subregion OBP, elementary radius vector OB starts from the origin and
terminates on the circle r = a cos q.
Limits of r : r = 0 to r = a cos q
a π
Limits of θ : θ = tan −1 to θ =
b 2
Hence, the polar form of the given integral is

( x 2 + y 2 )2
I = ÚÚ dx d y
x 2 y2
a p
tan -1 b sin q r4 a cosq r4
=Ú b
Ú0 ◊ r dr dq + Ú 2 -1 a Ú ◊ r dr dq
0 r 4 sin
sin 2 q cos
cos2 q tan
ta
b
0 r 4 sin
sin 2 q ccoos2 q

a b sin q p cosq
a cos
tan -1 1 r2 1 r2
=Ú b dq + Ú -1 a
2 dq
0 n 2 q cos2 q 2
sin 0
tan
ta
b
sinn 2 q cos2 q 2 0
a p
1 tan -1 b 1 1 1
= Ú
2 0 2 2
n q cos q
sin
◊ b2 sin 2 q dq + Ú 2 -1 a
2 ta
tan sin
n 2
q cos 2
q
◊ a 2 cco
o s 2 q dq
b
a p
b2 tan -1
tan a2
= Ú0 sec 2 q dq + Útatan-1 a cosec q dq
b sec 2 2
2 2
b
9.82 Chapter 9 Multiple Integrals

- p
b a2
+ - cot q 2
a
2 2 tan -1
b

b a ˘ a2 a ˘
tan tan tan t cot tan
2 b 2 2 b
2
b a a È b
-0
2 b 2 a
ab ab
+
2 2
ab

Example 5

∫ ∫
x
Evaluate e x y

Solution y

1. Limits of x : x = 0 to x → ∞
Limits of y : y = 0 to y → ∞
2. The region of integration is the first
quadrant.
3. Putting x = r cos q, y = r sin q, the integral
changes to polar form.
4. Draw an elementary radius vector which
starts from the origin and extends up to
infinity.
O x
Limits of r : r = 0 to r → ∞

Limits of q : q = 0 to = Fig. 9.94


2
Hence, the polar form of the given integral is p
q
2
∞ r→
y
p
• -r
Ú Ú 2 e r dr dq
p
1 2 • -r 2
2 Ú0 Ú
- e r )dr dq
p
1 2 -r 2 •
2 Ú0
- e dq ∵Ú f ¢(r r ef r)
0
p O
1 2 q=0

2 Ú0
- ( e )dq
Fig. 9.95
9.5 Change of Variables 9.83

p
1
= - -q 2
0
2
p
=
4

Example 6

Evaluate ∫ ∫ 3
2
( )
Solution
1. Limits of x x → −∞ to x→∞
Limits of y y → −∞ to y→∞ y

2. The region of integration is the entire coor-


dinate plane.
3. Putting x = r cos q, y = r sin q, integral
changes to polar form.
4. Draw an elementary radius vector which
starts from origin and extends up to •. x
Limits of r : r = 0 to r → ∞
Limits of q : q = 0 to q = 2p
Hence, the polar form of the given integral is
∞ ∞ d y
I=∫ ∫ 3
∞ ∞
( + y )2
Fig. 9.96
2 r d
∫ ∫ 0 3
( + r )2
3

1 2 ∞
= + )dr d q= p
2 2
1
1 2 r→∞
= −2 1 + ) 2
d
2 0

f r )]n +1
∵∫ (r )]n r )dr
n +1
O =0
2 1
= −∫ d
1+ r2 0
2
= −∫ ( )d

= 0

=2 Fig. 9.97
9.84 Chapter 9 Multiple Integrals

Example 7
a a x
Evaluate ∫ ∫ 2
dx y by transforming into polar coordinates.
y x y2
[Winter 2013]
Solution
y
1. Limits of x : x = y to x = a
Limits of y : y = 0 to y = a
2. The region of integration is bounded by
Q (a a)
the lines y = x, x = a and y = 0.
3. Putting x = r cos q, y = r sin q, polar form
of
(i) the line y = x is r sin q = r cos q, tan x=a

q = 1, .
4
(ii) the line x = a is r cos q = a,
O P x
r = a sec q.
4. Draw an elementary radius vector OA
which starts from the origin and terminates
on the line r = a sec q. Fig. 9.98
Limits of r : r = 0 to r = a sec q
Limits of q : q = 0 to =
4
Hence, the polar form of the given integral is
a x
I dx y
y x y
c r cos p
= ⋅r d y =
0 r2 2
a se
∫∫ 4
cos
Q

=∫ r0 4 a se
cos
A
= ∫ 4 a se cos d
r = a secq
= a∫ 4 d
O P =0
a 4
0


4 Fig. 9.99
a
4
9.5 Change of Variables 9.85

Example 8
2 2 x x2 x
Evaluate ∫∫ x 2
y2
x by transforming into polar coordinates.

Solution
y
1. Limits of y : y = 0 to x2
Limits of x : x = 0 to x = 2
2. The region of integration is bounded by
the circle x 2 + y 2 – 2x = 0 and the lines x 2+y 2 2x = 0
y = 0, x = 0. Since the limits of x and y are
positive, the region of integration is the
part of the circle in the first quadrant. (2, 0)
3. Putting x = r cos q, y = r sin q, polar form O x
of the circle x2 + y 2 – 2x = 0 is
r2 – 2r cos q = 0
r = 2 cos q.
4. Draw an elementary radius vector OA
which starts from the origin and termi-
nates on the circle r = 2 cos q. Fig. 9.100
Limits of r : r = 0 to r = 2 cos q

Limits of q : q = 0 to p
2 =
2
Hence, the polar form of the given integral is
2 2 x x2 x A
I 2 2
dy x r = 2 cosq
0 x y
p
2 cosq r cos q
q
0 0 r2 (2, 0)
p O =0
2 co q
Ú Ú 2 cos q q
p
2 q
Ú 2 r0 cos q q
p

Ú 2 2 co
2
q q Fig. 9.101
p
+ cos q q
p
sin 2q 2
+
2 0
9.86 Chapter 9 Multiple Integrals

p 1 1
= + sinn p - sin 0
2 2 2
p
=
2

Example 9
1 2 x − x2
Evaluate ∫∫0 x
((xx 2 + y 2 )dx dy.
y
Solution
y=x
1. Limits of y : y = x to y = 2 x − x 2
P
Limits of x : x = 0 to x = 1 (1, 1) x 2 + y 2 − 2x = 0
2. The region of integration is bound-
ed by the line y = x and the circle
x2 + y 2 – 2x = 0.
O (1, 0) x
3. Putting x = r cos q, y = r sin q, polar
form of
(i) the line y = x is
π
r sin θ = r cos θ , tan
tan θ = 1, θ = .
4
2 2
(ii) the circle x + y – 2x = 0 is Fig. 9.102
r2 – 2r cos q = 0
r = 2 cos q. q= p
2
4. Draw an elementary radius vector OA
q= p
which starts from the origin and termi- A P 4
nates on the circle r = 2 cos q. r = 2 cos q
Limits of r : r = 0 to r = 2 cos q
π π
Limits of θ : θ = to θ =
4 2
Hence, the polar form of the given integral O q=0
is
1 2 x - x2
I=Ú
0 Úx
( x 2 + y 2 )d
dxx dy
p
2 cosq 2
=Ú 2
p 0 Ú r ◊ r dr dq
Fig. 9.103
4
p 4 2 cosq
=Ú 2 r dq
p
4
4 0
p
= 4Ú p2 cos4q dq
4
9.5 Change of Variables 9.87

p 2
Ê 1 + cos 2q ˆ
= Ú 2
4 p Á
Ë 2 ˜¯ dq
4
p
=Ú p (1 + 2 cos 2
2 2qq + cos2 2q ) dq
4
p
2Ê 1 + cos 4q ˆ
=Ú p Á 1 + 2 cos 2q + ˜¯ dq
Ë 2
4
p
3 2 sin 2q sin sin 4q 2
= q+ +
2 2 8 p
4
3Êp pˆ Ê pˆ 1
= - - sin p - sin ˜ + (sin 2p - sin
sin p )
2 ÁË 2 4 ˜¯ ÁË 2¯ 8
3p
= +1
8

Example 10
a a2 - y2
Evaluate the integral Ú0 Ú0 y 2 x 2 + y 2 dy dx by changing into
polar coordinates. [Summer 2014]
Solution
1. Limits of x : x = 0 to x = a2 - y2
Limits of y : y = 0 to y = a
2. The region of integration is bounded by x = 0,
x= a 2 - y 2 , y = 0 and y = a.
3. Putting x = r cos q, y = r sin q, polar form of the
circle x2 + y2 = a2 is
Fig. 9.104
(i) r2 = a2
\r=a
4. Draw an elementary radius vector OA which
starts from the origin and terminates on the
circle r = a.
Limits of r: r = 0 to r = a
p
Limit of q: q = 0 to q =
2

Fig. 9.105
9.88 Chapter 9 Multiple Integrals

Hence, the polar form of the given integral is


p
a 2
I= Ú Ú
2
0 0
r sin 2 q ◊ r ◊ r dr dq
p
sin 2 q ÈÍ Ú r 4 dr ˘˙ dq
a
= Ú
0
2
Î 0 ˚
p a
r5
Ú
2
= 2 sin q dq
0 5
0
p
a5
=
5 Ú
0
2 sin 2 q dq

a5 1 p
= ◊ ◊
5 2 2
a 5p
=
20

Example 11
2 2
1 1- x 2 xye - ( x + y )
Evaluate Ú0 Ú x - x2
x 2 + y2
dx dy.

Solution
1. Limits of y : y = x − x 2 to y = 1 − x2
Limits of x : x = 0 to x = 1
2. The region of integration is the part of y
the first quadrant bounded by the circles
x 2 + y 2 – x = 0 and x 2 + y 2 = 1. x2 + y2 = 1
3. Putting x = r cos q, y = r sin q, polar
form of x2 + y2 – x = 0
(i) the circle x 2 + y 2 – x = 0 is P (1, 0)
r2 – r cos q = 0, r = cos q. O x
(ii) the circle x2 + y2 = 1 is r2 = 1, r = 1.
4. Draw an elementary radius vector OAB
from the origin which enters in the
region from the circle r = cos q and
terminates on the circle r = 1.
Limits of r : r = cos q to r = 1
Fig. 9.106
p
Limits of q : q = 0 to q =
2
9.5 Change of Variables 9.89

Hence, the polar form of the given integral is q p


2
1 1 x2 xye - x y )
I=Ú Ú 2
dx y
x x x2 y2 r
B
p r2
1 r2 e
= ◊ r dq A r cos q
co q r2
p
1 2 1 r2

=- n qÚ e r ) dr dq O q 0
q
p 1
1 2 r2

=- sin cos q e dq
q

Úe f ¢( )
r ef
p
1 2 cos2 q Fig. 9.107

=- sin q (e e )dq
p
1 21 1
2Ú 2 e
=- sin q cos q dq
p
1 1 2q (q )
=- e cos f
4 e 2 0
2p
1 1 - co
cos2 0
=- p - co 0) +e
4 2e

1 1 -1 ˘
=- 2
4 2e
1 1 1
- +
4 e e
1 2˘
4 e

Example 12
4a y x2 y2
Evaluate Ú Úy 2
x2 y2
dx y [Winter 2013]
4a
Solution
y2
1. Limits of x x to x = y
4a
Limits of y : y = 0 to y = 4a.
9.90 Chapter 9 Multiple Integrals

2. The region of integration is bounded by the y


line y = x and the parabola y 2 = 4ax.
y2 4ax P(4a, 4a)
3. Putting x = r cos q, y = r sin q, polar form of
(i) the line y = x is r sin q = r cos q,
p
tan q = 1, q .
4
(ii) the parabola y2 = 4ax is
r 2 sin2 q = 4ar cos q, r = 4a cot q cosec q.
O x
4. Draw an elementary radius vector OA which
starts from the origin and terminates on the
parabola r = 4a cotq cosecq.
Limits of r : r = 0 to r = 4a cot q cosec q
p p
Limits of q q = to q
4 2 Fig. 9.108
Hence, the polar form of the given integral is
q= p
2
4a y x2 2
I dx y
0 y
x2 y2 r = 4a cot q cosec q
4a P
p A
4 a cot q c q r2 2
q - sin )
=Ú p
2
Ú ◊r dq
4
r2 q p
4
p 4 a cot q cosec q
r2
= p - 2 sin 2 q dq
2
4 0
p O
1 =0
p
2 - 2 sin 2 q 4 a ) c 2
q q dq
2
4
p

p co q
4

p
= p - - - +2 q
4 Fig. 9.109
p
cot 3 q 2 f (q )]n +1 ˘
- + +2 f( ] (q )dq =
3 p n +1 ˚
4
p p p p p p
2
- co - cot + - cot + -
3 2 4 2 4 2 4
9.5 Change of Variables 9.91

È 1 p˘
= 8a 2 Í- (-1) + 2(-1) + 2 ◊ ˙
Î 3 4˚
È 5 p˘
= 8a 2 Í - + ˙
Î 3 2˚
Èp 5 ˘
= 8a 2 Í - ˙
Î 2 3˚

Example 13
a 5 ax - x 2 x2 + y 2
Evaluate Ú0 Ú2 ax y2
dx dy.
y

Solution
1. Limits of y : y = 2 ax to y

y = 5a axx − x2 y 2 = 4ax
P
Limits of x : x = 0 to x = a x 2 + y 2 – 5ax = 0
2. Since the limits of x and y are posi-
tive, the region of integration is the
part of the first quadrant bounded O x
by the parabola y 2 = 4ax and the
circle x2 + y 2 – 5ax = 0
3. Putting x = r cos q, y = r sin q, polar
form of
(i) the parabola y2 = 4ax is
r2 sin2 q = 4ar cos q, Fig. 9.110
r = 4a cot q cosecq. q= p
2
(ii) the circle x2 + y2 – 5ax = 0 is r = 4a cotq cosecq
r2 – 5ar cos q = 0, B P
r = 5a cos q. r = 5a cosq
A
4. The points of intersection of
r = 4a cotq cosecq and r = 5a cos q
are obtained as O q=0
4a cot q cosec q = 5a cos q
4
sin 2 q =
5
2
q = ± sin -1
5
Fig. 9.111
2
Hence, q = sin -1 at P.
5
9.92 Chapter 9 Multiple Integrals

5. Draw an elementary radius vector OAB from the origin which enters in the region
from the parabola r = 4a cotq cosecq and terminates on the circle r = 5a cos q.
Limits of r : r = 4a cotq cosecq to r = 5a cos q
2 p
Limits of q sin -1 to q =
5 2
Hence, the polar form of the given integral is
a 5 ax - x x2 y2
I Ú Ú2 ax y2
dx y

p
5 a cosq r
Úsin2
1 2 4 a cot q cosecq r 2 n2 q
r r dq
5
p
q
Úsin2
1 2 cosec q a t
q
5
p
= 1 2 - cosecq )dq
n
5
p
= 1
+ c 2
q cosec dq
n
5
p
cosec q 3 2 È [ f (q )]n +1 ˘
-5 a ∵Ú f q )dq
3 1 2 n+
n
5

p a 2
= -5a acosec sin - 3
sin 1
2 5 3 2 3 5

2 ˆ 5 ˘
È 3˘ ∵ cosec 1
cose cosec -1
5 4a aÊ 5ˆ 5 2
= -5 +
2 3 3 2 5
=
2
=
a
- )
3

Example 14
n
2 2ˆ2
x y
Evaluate ÚÚ xy a 2
+ 2
b
over the first quadrant of the ellipse

x2 y 2
+ =1
a2 b
9.5 Change of Variables 9.93

Solution
Let x = ar cos q, y = br cos q
∂x ∂x
∂( x, y) ∂r ∂q y
J= =
∂(r , q ) ∂y ∂y
∂r ∂q
a cos q - aarr sin q
= = abr
b sin q bbrr cos q
O x
dx dy = J dr dq = abr dr dq

Under the transformation x = ar cos q,


x2 y 2
y = br sin q, the ellipse 2 + 2 = 1 in the
a b
xy-plane gets transformed to r 2 = 1 or r = 1,
Fig. 9.112
circle with centre (0, 0) and radius 1 in the
rq -plane.
p
The region of integration is the part of the q=
2
circle r = 1 in first quadrant in the rq -plane.
Draw an elementary radius vector OA which A
starts from the origin and terminates on the r=1
circle r = 1.
Limits of r : r = 0 to r = 1 O q=0
p
Limits of q : q = 0 to q =
2
Hence, the polar form of the given integral is
n
Ê x 2 y2 ˆ 2
I = ÚÚ xxyy Á 2 + 2 ˜ dx dy Fig. 9.113
Ëa b ¯
p n
1
= Ú 2 Ú abr 2 cos q sin q (r 2 ) 2 abr dr dq
0 0
p
sin 2q 1 n + 3
= a 2 b2 Ú 2
2 Ú0
(r ) dr
0
p 1
a 2 b2 cos 2q 2 r n+4
= -
2 2 0 n+4
0
2 2
a b 1
= (- cos p + cos 0) ◊
4 n+4
a 2 b2
=
2(n + 4)
9.94 Chapter 9 Multiple Integrals

EXERCISE 9.5

Change to polar coordinates and evaluate the following integrals:

1
1. ∫∫ xy
dx dy over the region bounded by the semicircle x 2 + y 2 – x = 0,

y ≥ 0.
 p 
 Ans. : 
 2

∫∫ y
2
2. dx dy over the area outside the circle x 2 + y 2 – ax = 0 and inside the
circle x 2 + y 2 – 2ax = 0.
 15p a 4 
 Ans. : 
 64 

∫∫ sin(x + y 2 )dx dy over the circle x2 + y2 = a2.


2
3.

 Ans. : p((1 − ccos a 2 )


3

∫∫ xy(x + y )2 dx dy over the first quadrant of the circle x2 + y2 = a2.


2 2
4.
 a7 
 Ans. : 
 14 
3 3x dy dx
5. ∫∫ 0 0
x2 + y2
 3 
 Ans. : 2 log 3
 
a x x 3 dx dy
6. Ú Ú 0 0
x2 + y2
 
a4
(
 Ans. : log 1 + 2 
 4 
)
a a2 − x 2 p 
7. ∫ ∫ 0 0
n  2 (a 2 − x 2 − y 2 ) dx dy
sin
a 
 a2 
 Ans. : 
 2
a a2 − x 2
∫ ∫
2
−y2
8. e−x dx dy
0 0



p
(
 Ans. : 4 1 − e
−a 2 


)
9.5 Change of Variables 9.95

2a 2 ax − x 2
9. ∫ ∫
0 0
(x 2 + y 2 )dx dy
È 3p a 4 ˘
Í Ans. : ˙
Î 4 ˚
1 x - x2 4 xy
ÚÚ
2 2
10. e -( x + y ) dx dy
0 0 x +y
2 2

È 1˘
Í Ans. : e ˙
Î ˚
a
a2 - y 2
11. Ú Ú
0
2
y
g e(x 2 + y 2 )dx dy
log
È p 2Ê 1ˆ ˘
Í Ans. : a ÁË log a - ˜¯ ˙
Î 4 2 ˚
a a + a2 - y 2 dx dy
12. Ú Ú
0 y (4 a + x 2 + y 2 )3
2

È 1 Êp 1 1 ˆ˘
Í Ans. : 2 Á - tan-1 ˜˙
Î 8a Ë 4 2 2¯˚
a a2 - x 2 dx dy
13. Ú Ú
0 ax - x 2
a - x2 - y2
2

ÎÈAns. : a ˘˚
a a2 - x 2 xy
Ú Ú
2 2
14. e -( x + y ) dx dy
x +y 22
2
0 ax - x

È 1 È - a2 ˘ ˘
Í Ans. : 4 a 2 Î1 - (1 + a )e ˚ ˙
2

Î ˚
1 x dx dy
15. ÚÚ0 x2
x2 + y2
È Ans. : 2 - 1˘
Î ˚

9.5.2 Change of Variables from Cartesian to


Other Coordinates
In some cases, evaluation of double integral becomes easier by changing the variables.
Let the variables x, y be replaced by new variables u, v by the transformation
x = f1 (u, v), y = f2 (u, v), then

ÚÚ f ( x, y)dx dy = ÚÚ f ( f1 , f2 ) J dudv ... (1)


∂x ∂x
∂( x, y) ∂u ∂v
where Jacobian J = =
∂(u, v) ∂y ∂y
∂u ∂v
Using Eq. (1), the double integral can be transformed to new variables.
9.96 Chapter 9 Multiple Integrals

Example 1
Ê x - yˆ
Using the transformation x - y = u, x + y = v, evaluate ÚÚ cos Á dxdy
Ë x + y ˜¯
over the region bounded by the lines x = 0, y = 0, x + y = 1.
Solution x - y = u, x + y = v
u+v v-u
x= ,y =
2 2

∂x ∂x 1 1
∂( x, y) ∂u ∂v 2 2 1 1 1
J= = = = + =
∂(u, v) ∂y ∂y 1 1 4 4 2
-
∂u ∂v 2 2
1
dxdy = J dudv = dudv
2
The region bounded by the lines x = 0, y = 0 and x + y = 1 in xy-plane is a triangle
OPQ.
Under the transformation x = and = ,
2 2
(i) the line x = 0 gets transformed to the line u = –v
y
v
v=1
Q C
Q' P'
x=0 x+y=1 A B
u = −v u=v
P
O x O u
y=0

Fig. 9.114

(ii) the line y = 0 gets transformed to the line u = v


(iii) the line x + y = 1 gets transformed to the line v = 1
Thus, triangle OPQ in xy-plane gets transformed to triangle OP'Q' in uv-plane bounded
by the lines u = v, u = –v and v = 1.
In the region, draw a horizontal strip AB parallel to u-axis which starts from the line
u = –v and terminates on the line u = v.
Limits of u : u = –v to u = v
Limits of v : v = 0 to v = 1
9.5 Change of Variables 9.97

Ê x - yˆ
I = ÚÚ cos Á dx dy
Ë x + y ˜¯
1 v Ê uˆ 1
=Ú Ú cos Á ˜ du dv
0 -v Ë v¯ 2
v
1 1 Ê uˆ
2 Ú0
= v sin Á ˜ dv
Ë v¯ -v
1 1
v [sin 1 - sin(-1)] dv
2 Ú0
=
1
1 v2
= ◊ 2 sin 1
2 2
0
1
= sin 1
2

Example 2
Using the transformation x 2 - y 2 = u , 2 xy = v, find ÚÚ ( x + y ) dx dy
2 2

over the region in the first quadrant bounded by x2 – y2 = 1, x2 – y2 = 2,


xy = 4, xy = 2.
Solution
x2 – y2 = u, 2xy = v
It is difficult to express x and y in terms of u and v, therefore we write Jacobian of u,
v in terms of x and y.
∂u ∂u
∂ (u, v) ∂x ∂y 2x - 2y
J= = = = 4 ( x 2 + y2 )
∂ ( x , y ) ∂v ∂v 2y 2x
∂x ∂y

(
dudv = J dxdy = 4 x 2 + y 2 dxdy )
1
dxdy = dudv
(
4 x + y2
2
)
The region in xy-plane bounded by the curves x2 – y2 = 1, x2 – y2 = 2, xy = 4, xy = 2 is
transformed to a square in uv-plane bounded by the lines u = 1, u = 2, v = 4, v = 8.
In the region, draw a vertical strip AB parallel to the v-axis which starts from the line
v = 4 and terminates on the line v = 8.

I = ÚÚ ( x 2 + y 2 ) dx dy
9.98 Chapter 9 Multiple Integrals

2 8 1
=Ú Ú4 ( x
2
+ y2 ) du dv
1 4 ( x + y2 )
2

1 2 8
= u v
4 1 4
=1
v
y
xy = 4 v=8
R' B Q'
xy = 2
R x2 − y2 = 1
Q
x2 − y2 = 2
S S' A P'
v=4
P u=2
u=1
O x O u

Fig. 9.115

Example 3
Ú Ú (x + y 2 ) dA by changing the variables, where R is
2
Evaluate
R
the region lying in the first quadrant and bounded by the hyperbola
x2 – y2 = 1, x2 – y2 = 9, xy = 2 and xy = 4. [Summer 2014]
Solution
Let u = x2 – y2 v = xy
∂u ∂u
∂(u, v) ∂x ∂y 2 x -2 y
J= = =
∂( x, y) ∂v ∂v y x
∂x ∂y
= 2x2 + 2y2 = 2(x2 + y2)
du dv = |J| dx dy = 2(x2 + y2) dx dy
1
dx dy = ◊ du dv
2( x + y 2 )
2

The region in the xy-plane bounded by the curves x2 – y2 – 1, x2 – y2 = 9, xy = 2 and xy = 4


is transformed to a square in the uv-plane bounded by the lines u = 1, u = 9, v = 2, v = 4.
9.5 Change of Variables 9.99

v
y
xy = 4 v=8
R' B Q'
xy = 2
R x2 − y2 =1
Q
x2 − y2 = 9
S S' A P'
v=4
P u=2
u=1
O x O u

Fig. 9.116

ÚÚ ( x + y 2 ) dx dy
2
I=
R

1
ÚÚ ( x + y2 ) ◊
2
= du dv
R 2( x + y 2 )
2

1
=
2 ÚÚ du dv
49
1
=
2 Ú Ú du dv
21

1 4 9
= v u1
2 2
1
= (4 - 2) (9 - 1)
2
1
= (2)(8)
2
=8

Example 4
Using the transformation x + y = u, y = uv, show that
y
1 1- x x+ y 1
Ú0 Ú0 e dx =
dy dx
2
(e - 1). [Winter 2014]
9.100 Chapter 9 Multiple Integrals

Solution
x + y = u, y = uv
x = u(1 - v), y = uv
∂x ∂x
∂( x, y) ∂u ∂v 1 - v - u
J= = = = (1 - v)u + uv = u
∂(u, v) ∂y ∂y v u
∂u ∂v
dxdy = J du udv = udu dv
Limits of y : y = 0 to y = 1 – x
Limits of x : x = 0 to x = 1.
The region in xy-plane is the triangle OPQ bounded by the lines x = 0, y = 0 and x + y = 1.
Under the transformation x = u (1 – v) and y = uv,
(i) the line x = 0 gets transformed to the line u = 0 or v = 1
(ii) the line y = 0 gets transformed to the line u = 0 or v = 0
(iii) the line x + y = 1 gets transformed to the line u = 1
v
y
v=1
R' B Q' (1, 1)
Q

u=0 u 1

P
O x O A P' u
v 0

Fig. 9.117
Thus, the triangle OPQ in the xy-plane gets transformed to the square OP'Q'R' in
uv-plane bounded by the lines u = 0, v = 0, u = 1 and v = 1.
In the region, draw a vertical strip AB parallel to the v-axis which starts from the u-axis
and terminates on the line v = 1.
Limits of v : v = 0 to v = 1
Limits of u : u = 0 to u = 1 y
1 1- x
I=Ú Ú e x + y dx dy
0 0
1 1 v
=Ú Ú e udu dv
0 0
1
1 u2
= ev
0 2
0
1
= (e1 - e0 ) ◊
2
1
= (ee -1)
2
9.5 Change of Variables 9.101

Example 5
y
2 2x -
4 +1 y
Evaluate
Ú0 Ú y
2
dx dy by applying the transformations
2

2x - y y
u= , v = . Draw both regions. [Winter 2015]
2 2

Solution
1. The function is integrated first w.r.t. x.
y y
2. Limits of x : x = to x = + 1
2 2
Limits of y : y = 0 to y = 4.
3. The region is the parallelogram bounded
y y
by the lines x = , x = + 1, y = 0 and
2 2
y = 4 in xy-plane.
Applying the transformations
2x - y y
u= , v=
2 2
y Fig. 9.118
= x-
2
y
(i) the line x - = 0 mapped to the line u = 0
2
y
(ii) the line x - = 1 mapped to the line u = 1
2
(iii) the line y = 0 mapped to the line v = 0
(iv) the line y = 4 mapped to the line v = 2
Hence, the parallelogram OABC in the xy-plane
mapped to the rectangle O¢A¢B¢C¢ in uv-plane,
bounded by the lines u = 0, u = 1, v = 0 and v = 2.
In the region, draw a vertical strip AB parallel to the Fig. 9.119
v-axis which starts from the u-axis and terminates on the line v = 2.
Limits of u : u = 0 to u = 1
Limits of v : v = 0 to v = 2
dxdy = J du dv
∂( x, y)
where Jacobian, J =
∂(u, v)
1 ∂(u, v)
J* = =
J ∂( x, y)
9.102 Chapter 9 Multiple Integrals

∂u ∂u
∂x ∂y
=
∂v ∂v
∂x ∂y
1
1 -
2
=
1
0
2
1
=
2
1
\ dxdy = dudv
2
Hence, the new form of the integral is
4 y +1 2 x - y
I = Ú Úy dxdy
0
2 2
2 1 1
=Ú Ú u ◊ du dv
v=0 u=0 2
1
1 2 u2
= Ú dv
2 0 2
0
1 2
= v0
4
1
= (2 )
4
=2

Example 6
Using the transformation x = u (1 + v), y = v(1 + u ), u ≥ 0, v ≥ 0,
1
2 y -
Ú0 Ú0 ÈÎ( x - y) + 2( x + y ) + 1˘˚ dydx.
2 2
evaluate

Solution
x = u(1 + v), y = v(1 + u)

∂x ∂x
∂( x, y) ∂u ∂v 1 + v u
J= = = = 1+ u + v
∂(u, v) ∂y ∂y v 1+ u
∂u ∂v
dxdy = J dudv = (1 + u + v) dudv
9.5 Change of Variables 9.103

y y=x
y=2
v (1 + u) = 2
Q
P
Q' v=u
x=0 B
u = −1
P'(1, 1)
u=0
A
O x O u

Fig. 9.120
Limits of x : x = 0 to x = y
Limits of y : y = 0 to y = 2.
The region in the xy-plane is the DOPQ bounded by the lines x = 0, y = 2 and y = x.
Under the transformation x = u (1 + v), y = v (1 + u), u ≥ 0, v ≥ 0
(i) the line x = 0 gets transformed to the line u = 0
(ii) the line y = 2 gets transformed to the curve v (1 + u) = 2
(iii) the line y = x gets transformed to the line u = v
Thus, the triangle OPQ in the xy-plane gets transformed to the region OP'Q' in uv
plane bounded by the lines u = 0, u = v and the curve v (1 + u) = 2.
The point of intersection of u = v and v (1 + u) = 2 is obtained as u2 + u – 2 = 0, u = 1, –2
and v = 1, –2.
The point of intersection is P' (1,1).
In the region, draw a vertical strip AB parallel to the v-axis which starts from the line
u = v and terminates on the curve v (1 + u) = 2.
2
Limits of v : v = u to v =
1+ u
Limits of u : u = 0 to u = 1
1
2 y -
I=Ú È( x - y) + 2( x + y) + 1˘
Ú
2 2
Î ˚ dy dx
0 0
1
2 -
1 2
=Ú È(u - v)2 + 2(u + v + 2uv) + 1˘
0 u Ú
1+ u
Î ˚ (1 + u + v)du dv
2
1
=Ú 1+ u (1 + u + v ) -1 (1 + u + v ) dv du
0 u Ú
2
1
=Ú Ú
1+ u dv du
u
0 u
2
1 1+ u
=Ú v du
0 u
1Ê 2 ˆ
=Ú Á - u˜ du
0 Ë 1+ u ¯
9.104 Chapter 9 Multiple Integrals

1
u2
= 2 log(1 + u) -
2
0
1
= 2 log 2 -
2

Example 7
Evaluate ÚÚ xy dxdy by changing the variables over the region in the
first quadrant bounded by the hyperbolas x2 – y2 = a2, x2 – y2 = b2 and the
circles x2 + y2 = c2, x2 + y2 = d2 with 0 < a < b < c < d.
Solution
Let x 2 - y 2 = u, x 2 + y 2 = v
u+v 2 v-u
x2 = ,y =
2 2
∂x ∂x 1 1
∂( x, y) ∂u ∂v 4x 4x 1
J= = = =
∂(u, v) ∂y ∂y 1 1 8 xy
-
∂u ∂v 4y 4y
1
dxdy = J dudv = dudv
8 xy
dudv
xydxdy =
8
The region bounded by the hyperbolas x2 – y2 = a2, x2 – y2 = b2 and the circles x2 + y2 = c2, x2
+ y2 = d 2 in xy-plane is the curvilinear rectangle PQRS.
v
y v = d2
S' B R'

x2 − y2 = a 2
R x2 − y2 = b 2
Q u = b2
S u a2
P x2 + y 2 = d2 P' A Q'
v = c2
x
O u
x2 + y 2 = c2

Fig. 9.121
9.5 Change of Variables 9.105

Under the transformation x2 – y2 = u and x2 + y2 = v,


(i) the hyperbolas x2 – y2 = a2, x2 – y2 = b2 get transformed to the lines u = a2,
u = b2 respectively.
(ii) the circles x2 + y2 = c2, x2 + y2 = d 2 get transformed to the lines v = c2, v = d 2
respectively.
Thus, the curvilinear rectangle PQRS in the xy-plane gets transformed to the rectangle
P'Q'R'S' in uv-plane bounded by the lines u = a2, u = b2, v = c2 and v = d 2.
In the region, draw a vertical strip AB parallel to v-axis which starts from the line
v = c2 and terminates on the line v = d 2.
Limits of v : v = c2 to v = d 2
Limits of u : u = a2 to u = b2
I = ÚÚ xydxdy
b2 d2 1
=Ú 2 Úv = c 2 dudv
u=a 8
1 b2 d 2
= u a2 v c2
8
1
= (b 2 - a 2 )(d 2 - c 2 )
8

Example 8
Evaluate ∫∫ ( x + y ) dx dy, by changing the variables
2
over the
parallelogram with vertices (1, 0), (3, 1), (2, 2), (0, 1).
Solution
The region of integration in xy-plane is the parallelogram PQRS.
Equations of the sides of the parallelogram are obtained as
1- 0 y
(i) PQ : y - 0 = ( x -1)
3 -1
2 y = x -1
x - 2y =1 R (2, 2)

2 -1
(ii) RS : y -1 = ( x - 0) S
2 -0 Q (3, 1)
2y -2 = x (0, 1)
x - 2 y = -2
O P (1, 0) x
(iii) PS : y - 0 = 1 - 0 ( x -1)
0 -1
x + y =1
Fig. 9.122
9.106 Chapter 9 Multiple Integrals

2 -1
(iv) QR : y -1 = ( x - 3)
2 -3
y -1 = - x + 3
x+y =4
Let x - 2 y = u, x + y = v
u + 2v v -u
x= ,y=
3 3
∂x ∂x 1 2
∂( x, y) ∂u ∂v 3 3 1
J= = = =
∂(u, v) ∂y ∂y 1 1 3
-
∂u ∂v 3 3
1
dxdy = J dudv = du udv
3
Under the transformation x – 2y = u, and x + y = v
(i) the lines x – 2y = 1, x – 2y = –2 get transformed to the lines u = 1, u = –2
respectively.
(ii) the lines x + y = 1, x + y = 4 get transformed to the lines v = 1, v = 4 respectively
Thus, the parallelogram PQRS in the xy-
v=4 v
plane gets transformed to a square P'Q'R'S'
in uv-plane bounded by the lines u = 1, u = R' B Q'
–2, v = 1 and v = 4.
In the region, draw a vertical strip AB
parallel to v-axis which starts from the line
v = 1 and terminates on the line v = 4.
Limits of v : v = 1 to v = 4
u = −2 u=1
Limits of u : u = –2 to u = 1
I = ÚÚ ( x + y)2 dxdy
1 4 1
=Ú Ú v2 dudv S' A P'
u = 2 v =1 3
4
v=1
1 1 v3 O u
= u -2
3 3
1 Fig. 9.123
= 21

Example 9
n
Ê x2 y 2 ˆ 2
Evaluate ÚÚ xy Á 2 + 2 ˜ over the first quadrant of the ellipse
Ëa b ¯
x2 y 2
+ = 1.
a 2 b2
9.5 Change of Variables 9.107

Solution
Let x = ar cos q, y = br sin q y
∂x ∂x
∂( x, y) ∂r ∂q
J= =
∂(r , q ) ∂y ∂y
∂r ∂q
a cos q - ar sin q
= = abr O x
b sin q br cos q
dx dy = J dr dq = abr dr dq

Under the transformation x = ar cos q,


x2 y2
y = br sin q, the ellipse + = 1. in the Fig. 9.124
a 2 b2
xy-plane gets transformed to r2 = 1 or r =
1, circle with centre (0, 0) and radius 1 in the rq -plane.
The region of integration is the part of the circle r = 1 in first quadrant in the
rq-plane. In the region, draw an elementary radius vector OA from the pole which
terminates on the circle r = 1.
Limits of r : r = 0 to r = 1
p
Limits of q : q = 0 to q = q=p
2 2
n
Ê x 2 y2 ˆ 2 A
I = ÚÚ xy Á 2 + 2 ˜ dxdy r=1
Ëa b ¯
p n O
1 q=0
=Ú 2
Ú abr 2 2 2
cos q sin q (r ) abr dr dq
0 0
p
sin 2q 1 n +3
= a 2 b2 Ú 2 Ú0 (r ) dr
0 2
p 1
a 2 b2 cos 2q 2 r n+4 Fig. 9.125
= -
2 2 0 n+4
0
2 2
a b 1
= (- cos p + cos 0) ◊
4 n+4
a 2 b2
=
2(n + 4)
9.108 Chapter 9 Multiple Integrals

EXERCISE 9.6
x -y

ÚÚ e
x +y
1. Using the transformation x + y = u, x – y = v, evaluate dxdy over
the region bounded by x = 0, y = 0 and x + y = 1.
È 1Ê 1ˆ ˘
Í Ans.: ÁË e - ˜¯ ˙
Î 4 e ˚

2. Using the transformation x2 – y2 = u, 2xy = v, evaluate ÚÚ (x - y )dxdy


2 2

over the region bounded by the hyperbolas x2 – y2 = 1, x2 – y2 = 9, xy = 2


and xy = 4.
[Ans.: 4]
3. Using the transformation x + y = u, y = uv, evaluate
• •
Ú Ú
0 0
e -( x + y ) x p -1y q -1dx dy .
È Ans.: p q ˘
Î ˚

1 x
4. Using the transformation x = u, y = uv, evaluate ∫∫
0 0
x 2 + y 2 dx dy .

È 1È 2 1 ˘˘
Í Ans.: Í
ÍÎ 3 ÍÎ 2
+ log 1 + 2 ˙ ˙
2 ˙˚ ˙˚
( )

∫∫ (x + y) dx dy by changing the variables over the region bounded


2
5. Evaluate
by the parallelogram with sides x + y = 0, x + y = 2, 3x – 2y = 0 and 3x – 2y = 3.
 8
 Ans. : 5 
 

6. Evaluate ∫∫ (x − y )4 e x + y dx dy , by changing the variables over the region


bounded by the square with vertices at (1, 0), (2, 1), (1, 2), (0, 1).
 e3 − e 
 Ans. : 
 5 
1
7. Evaluate ∫∫ [ xy (1 − x − y )]2 dx dy , by changing the variables over the region
bounded by the triangle with sides x = 0, y = 0, x + y = 1.

È p˘
2p
2
Í Ans.: 105 ˙
Î ˚
9.6 Triple Integrals 9.109

9.6 TRIPLE INTEGRALS

Let f (x, y, z) be a continuous function defined in a closed and bounded region V in


3-dimensional space. Divide the region V into small elementary parallelopipeds by
drawing planes parallel to the coordinate planes. Let the total number of complete
parallelopipeds which lie inside the region V be n. Let d Vr be the volume of the rth
parallelopiped and (xr, yr, zr) be any point in this parallelopiped. Consider the sum
n
S = Â f ( xr , yr , zr )d Vr …(1)
r =1

where, d Vr = d xr ◊ d yr ◊ d zr
If we increase the number of elementary parallelopipeds, n, then the volume of each
parallelopiped decreases. Hence as n Æ •, d Vr Æ 0.
The limit of the sum given by Eq. (1), if it exists is called the triple integral of
f (x, y, z) over the region V and is denoted by ÚÚÚÚ f ( x, y, z )dV
V

n
Hence, ÚÚÚÚ f ( x, y, z)dV = lim
n Æ•
 f ( xr , yr , zr )d Vr
V d Vr Æ 0 r =1

where dV = dx dy dz

9.6.1 Triple Integrals in Cartesian Coordinates


Triple integral of a continuous function f (x, y, z) over a region V can be evaluated by
three successive integrations.
Let the region V be bounded below by a surface z = z1 (x, y) and above by a surface
z = z2 (x, y). Let the projection of region V in xy-plane be R which be bounded by the
curves y = y1(x), y = y2(x) and x = a, x = b. Then the triple integral is defined as
b È y2 ( x )
I = Ú ÍÚ
a Î y1 ( x ) {Ú z2 ( x , y )
z1 ( x , y ) } ˘
f ( x, y, z ) dz dy ˙ dx
˚
Note: The order of variables in dx dy dz indicates the order of integration. In some
cases this order is not maintained. Therefore, it is advisable to identify the order of
integration with the help of the limits.

9.6.2 Triple Integrals in Cylindrical Coordinates


Cylindrical coordinates r, q, z are used to evaluate the integral in the regions which
are bounded by cylinders along z-axis, planes through z-axis, planes perpendicular to
the z-axis.
9.110 Chapter 9 Multiple Integrals

Relations between Cartesian (rectangular) coordinates (x, y, z) and cylindrical


coordinates (r, q, f ) are given as x = r cos q
y = r sin q
z=z
z

Then ÚÚÚÚ f ( x, y, z )dx dy dz

= ÚÚÚ f (r cos q , r sin q , z ) J dz dr dq


P (r,, q,, z)
∂x ∂x ∂x
∂r ∂q ∂z z=z
∂( x, y,
y, z ) ∂y ∂y ∂y
where, J= =
∂(r , q , z ) ∂r ∂q ∂z O
y = r sin q N
y
∂z ∂z ∂z

q
90°

os
q

rc
∂r ∂q ∂z r

=
x
90°
q -r sin q 0
cosssq M Q
= sin
nq
n q r cos cos q 0 x
0 0 1 Fig. 9.126
= cos q (r co
coss q ) + r sin
n q (si
(sinnq )
=r

Hence, ÚÚÚ f ( x, y, z)dx dy dz = ÚÚÚ f (r cos q , r sin q , z)r dz dr dq

9.6.3 Triple Integrals in Spherical Coordinates


Spherical coordinates (r, q, f ) are used z
to evaluate the integral in the regions
which are bounded by the sphere with
P (x,, y,, z)
centre at the origin.
Relations between cartesian (rectangu-
lar) coordinates (x, y, z) and spherical r z = r cos q
q
coordinates (r, q, f ) are given as
y = r sin q sin f N
x = r sin q cos f O
y
f

y = r sin q sin f
s

90°
co

f
rs
q
in

in
z = r cos q
rs

q
=

90°
x

Then M Q

ÚÚÚÚf ( x, y, z )dx dy dz x

= ÚÚÚÚ f (r sin q cos


cos f , r sin q sin f , r cos q ) J dr dq df Fig. 9.127
9.6 Triple Integrals 9.111

∂x ∂x ∂x
∂r ∂q ∂f
∂( x, y, z ) ∂y ∂y ∂y
where J= =
∂(r
( , q , f ) ∂r ∂q ∂f
∂z ∂z ∂z
∂r ∂q ∂f
sinn q cos
cos f r cos q cos
cossq cos f -r sinn q sin
sin f
= sinn q sin
sin f r cos q sin
cossq sin f r sinn q cos
cos f
cos q -r sin q 0
= sin q cos f (r 2 sin
sin 2 q cos f ) - r coss q cos
cos f (-r
-r sin q cos
cos f cos q )
- r ssiin q sin -rr sin
-
sin f ((- sin f - r cos
sin 2 q sin cos2 q sin
sin f )
= r 2 sin cos2 f (sin 2 q + cos
q cos
sinnq cos2 q ) + r 2 sin q sin 2 f
= r 2 sin q
ÚÚÚÚ f ( x, y, z)dx dy ddzz = ÚÚÚÚ
ÚÚ f (r sin q cos f, r sin
2
Hence, sin q sin
sin f , r cos q )r sin q dr dq df.
Note: If the region of integration is a sphere x2 + y2 + z2 = a2 with centre at (0, 0, 0)
and radius a, then limits of r, q, f are
(i) For positive octant of the sphere,
r : r = 0 to r = a
p
q : q = 0 to q =
2
p
f : f = 0 to f =
2
(ii) For hemisphere,
r : r = 0 to r = a
p
q : q = 0 to q =
2
f : f = 0 to f = 2p

(iii) For complete sphere,


r : r = 0 to r = a
q : q = 0 to q = p
f : f = 0 to f = 2p

9.6.4 Change of Variables


In some cases, evaluation of a triple integral becomes easier by changing the variables.
Let the variables x, y, z be replaced by new variables u, v, w by the transformation x =
f1 (u, v, w), y = f2 (u, v, w), z = f3 (u, v, w).
9.112 Chapter 9 Multiple Integrals

Then ÚÚÚ f ( x, y, z)dx dy ddzz = ÚÚÚ f ( f1 , f2 , f3 ) J du dv dw


∂x ∂x ∂x
∂u ∂v ∂w
∂( x, y, z ) ∂y ∂y ∂y
where, J= =
∂(u, v, w) ∂u ∂v ∂w
∂z ∂z ∂z
∂u ∂v ∂w

9.6.5 Working Rule for Evaluation of Triple Integrals


1. Draw all the planes and surfaces and identify the region of integration.
2. Draw an elementary volume parallel to z ( y or x) axis.
3. Find the variation of z ( y or x) along the elementary volume.
4. Lower and upper limits of z ( y or x) are obtained from the equation of the surface
(or plane) where elementary volume starts and terminates respectively.
5. Find the projection of the region on xy (zx or yz) plane.
6. Draw the region of projection in xy (zx or yz) plane.
7. Follow the steps of double integration to find the limits of x and y (z and x or y
and z).
Note: (1) If the region is bounded by the cylinders along the z-axis, planes through
z-axis, the planes perpendicular to the z-axis, then the cylindrical coordinates are used.
(2) If the region is bounded by the sphere, then the spherical coordinates are used.

Type I Evaluation of Triple Integrals when Limits are Given

Example 1
1 2 e
Evaluate Ú0 Ú0 Ú0 dy dx dz.
Solution
1 2 e 1 2È e ˘
Ú0 Ú0 Ú0 dy dx dz = Ú0 Ú0 ÍÎÚ0 dy ˙˚dx dz
1 2 e
=Ú Ú y dx dz
0 0 0

= Ú Í Ú e dx ˘˙ dz
1È 2
0Î 0 ˚
1 2
= e Ú x dz
0 0
1
= e Ú 2 dz
0
9.6 Triple Integrals 9.113

1
= 2e z 0
= 2e
Another method:
Since all the limits are constant and integrand (function) is explicit in x, y and z, the
integral can be written as
1 2 e 1 2 e
Ú0 Ú0 Ú0 dy dx ddzz = Ú0 dz ◊Ú0 ddxx ◊ Ú0 dy
1 2 e
= z0◊ x0◊ y0
= 1◊ 2 ◊ e
= 2e.

Example 2
2 3 2
Evaluate Ú0 Ú1 Ú1 xy 2 z dz dy dx.
Solution
Since all the limits are constant and integrand (function) is explicit in x, y and z, the
integral can be written as
2 3 2 2 3 2
Ú0 Ú1 Ú1 xy z dz dy dx = Ú x dx ◊Ú y 2 dy ◊ Ú z dz
2
0 1 1
2 3 2
x2 y3 z2
= ◊ ◊
2 3 2
0 1 1
26 3
= 2◊ ◊
3 2
= 26

Example 3
1 p p
Evaluate Ú0 Ú0 Ú0 y sin z dx dy dz. [Winter 2013]

Solution
Since all the limits are constant and integrand (function) is explicit in x, y, and z, the
integral can be written as
1 p p 1 p p
Ú0 Ú0 Ú0 y sin z dx dy dz = Ú0 sin z dzÚ0 y ddyyÚ0 dx
p
1 y2 p
= - cos z 0 ◊ ◊ x0
2
0
9.114 Chapter 9 Multiple Integrals

Ê p2 ˆ
= ( - cos 1 + cos 0) Á ˜ (p )
Ë 2 ¯
p3
= (1 - cos 1)
2

Example 4
1 z 2p
Evaluate Ú0 Ú0 Ú0 (r 2 cos2 q + z 2 )rdq drdz. [Winter 2016]

Solution
1 z
Ú0 Ú0 Ú0
2p
(r 2 cos2 q + z 2 )rdq drdz = Ú
1
0 0 Ú
z
{Ú 0
2p
}
[r 2 cos2 q + z 2 ] dq r drdz

1 z ÏÔ 2p È 2 Ê 1 + cos 2q ˆ ˘ ¸Ô

0 Ú0
Ì Ú0 Ír ÁË ˜ + z 2 ˙ dq ˝ r drdz
Ô Ó Î 2 ¯ ˚ ˛Ô

ÏÔ È 1 Ê 2p ¸
1 z sin 2q ˆ 2 ˘ Ô
=Ú Ú Ì Í
r r 2
ÁË q + ˜¯ + z q ˙ ˝ d r dz
0 0
ÔÓ Î 2 2 ˚ 0 Ô˛

1 z È1 3 2 ˘
= 2p Ú
0 Ú0 Í 2 r + z r ˙ drdz
Î ˚
2
1 r4 r2
= 2p Ú + z2 dz
0 8 2
0

1 z2 z3
= 2p Ú + dz
0 8 2
1
È z3 z 4 ˘
= 2p Í + ˙
ÍÎ 24 8 ˙˚ 0

È 1 1˘
= 2p Í + ˙
Î 24 8 ˚
È 3 + 1˘
= 2p Í ˙
Î 24 ˚
9.6 Triple Integrals 9.115

4
= 2p ◊
24
p
=
3

Example 5
1 1 x x y
Evaluate Ú Ú Ú d y dz

Solution
z˘d
x+ y
ÚÚ Ú d yd x

1 1- x x y
Ú Ú0 z
0
dx
1 1 x
y)d x
0
1- x
1 y2
+ d
2
0

1 ( x )2 ˘
+ dx
2
1 ( x )2 ˘
2
+ dx
2
1
x2 3
1 (1 x )3
- + ◊
2 3 2 ( 3)
0
1 1 1
- +
2 3 6
1
3

Example 6
1 1- x x y
Evaluate Ú Ú Ú0 e x dy

Solution
e z˘ d
1 x x x
ÚÚ Ú e x y x
9.116 Chapter 9 Multiple Integrals

1 1- x x y
=Ú Ú e dx
0
1 1- x
= - e )d x

1 1 x
dx
0
1
1
0
1
x2
+ - ex
2
0
1
+e 0
2
1
=
2

Example 7
a x x y x +
Evaluate Ú Ú0 Ú e dy x
Solution
e z˘ d
a x a x y
Ú ex z
y x e x
0

x y x y
Ú e ez
0
x
x
- )d x
È - e )d
a x x x y
x
x
a x e2 y
x
◊ e
2
0

a È e 2x ˘
= x x

Î 2 2 ˚
a 1
+ e x dx
2 2
a
1 e e
= ◊ - ◊ + ex
2 4 2 2
0

=
1
- - - )+ - )
8 4
1 3
= 4
- e a
+ -
8 4 8
9.6 Triple Integrals 9.117

Example 8
2 2 yz
Evaluate the integral Ú0 Ú1 Ú0 xyz dx dy dz. [Summer 2014]

Solution
2 2 yz 2 2 È yz
x dx ˘˙ dy dz
Ú0 Ú1 Ú0 xyz dx dy dz = Ú0 Ú1 yz ÍÎÚ0 ˚
yz
2 2 x2
= Ú0 Ú1 yz
2
dy dz
0

2 2 1
Ú0 Ú1 2 y
3 3
= z dy dz
2 2
1 y4 z4
=
2 4 4
1 0

1 Ê 1ˆ 1
= Á ˜ [16 - 1] [16 - 0]
2 Ë 4¯ 4
15
=
2

Example 9
1
3 xy
Evaluate Ú Ú Ú
x
1 1 0
xy dz dy dx.

Solution
1 1
3 xy 3 È xy ˘
Ú Ú Ú x
1 1 0
xy dz dy ddxx = Ú
1 1
x
Ú ÍÎÚ0 dz ˙ xxyy dy dx
˚
1
3 xy
=Ú x
Ú z 0 dy
dy dx
1 1

3È ˘
1
= Ú Í Ú x xy dy ˙ dx
1 Î 1 ˚
1
3 x
3 2y 2
=Ú x dx
1 3 1

Ê ˆ
2 3 Á 1 - 1˜ dx
3 Ú1
= x
Á 3 ˜
Ë x2 ¯
9.118 Chapter 9 Multiple Integrals

2 3Ê 1

3 Ú1 Ë x
= Á - x ˜¯
2 dx

3
3
2 2x 2
= log x -
3 3 1

=


2
3 ˘
Í( log 3 - log 1) - 3 2 - 1 ˙
3 ˚
( )
2È 2˘
= Ílog 3 - 2 3 + ˙
3Î 3˚

Example 10
2 z yz
Evaluate Ú0 Ú1 Ú0 xyz dx dy dz. [Summer 2017]

Solution
The innermost limits depend on y and z. Hence, integrating first w.r.t. x,
yz
2 z yz 2 z x2
Ú0 Ú1 Ú0 xyz dx dy ddzz = Ú
0 1 Ú 2
yz dy dz
0

1 2 z 2 2
2 Ú0 Ú1
= ((yy z ) yz dy dz

= Ú z 3 ÈÍ Ú y3 dy ˘˙ dz
1 2 z

2 0 Î1 ˚
z
1 2 3 y4
2 Ú0
= z dzz
4
1
1 2
= Ú z 3 ( z 4 - 1)d
)dz
8 0
2
1 z8 z 4
= -
8 8 4
0
1
= (32 - 4)
8
7
=
2
9.6 Triple Integrals 9.119

Example 11
1 1- x 1- x - y 1
Evaluate Ú0 Ú0 Ú0 ((xx + y + z + 1)3
dx dy dz.

Solution
The innermost limits depend on x and y. Hence, integrating first w.r.t. z,
1 1- x 1- x - y 1
Ú0 Ú0 Ú0 ((xx + y + z + 1)3
dx dy dz

1 1- x È 1- x - y 1 ˘
=Ú Ú Í Ú0 dz ˙ dy dx
0 0
Î ((xx + y + z +
+11)3
˚
1- x - y
1 1- x 1
=Ú Ú dy dx
0 0 -2
-2(((xx + y + z + 1)2
2 0

1 1 1- x È 1 1 ˘
=-
2 Ú0 Ú0
Í
ÍÎ {x + y + (1
1-- x - y) + 1}2
- ˙ dy dx
( x + y + 1) ˙˚
2

1 1 È 1- x ÏÔ 1 1 ¸Ô̧Ô ˘
=- Ú ÍÚ Ì - ˝ dyy˙˙ dx
2 0 ÍÎ 0 ÓÔ 4 ( x + y + 1)2 ˛Ô ˙˚
1- x
1 1 y 1
=- Ú + dx
2 4 x + y +1 0
0

1 1 È1 - x 1 1 ˘
=- Ú Í
2 Î 4
0
+ -
x + (1 - x ) + 1 x + 1 ˙˚
dx

1 1Ê 1 - x 1 1 ˆ
=- Ú Á + - ˜ dx
2 0Ë 4 2 x + 1¯
1
1 x x2 x
=- - + - log( x + 1)
2 4 8 2
0
1Ê5 ˆ
= - Á - log 2˜
2Ë8 ¯

Example 12
1 1- x 2 1- x 2 - y2
Evaluate Ú0 Ú0 Ú0 xyz dz dy dx. [Winter 2014]

Solution
1- x 2 - y2
1 1- x 2 È 1- x2 - y2 ˘ 1 1- x 2 z2
Ú0 Ú0 xy Í Ú
Î 0
z dz ˙ dx dy =
˚ Ú0 Ú0 xy
z 0
dx dy
9.120 Chapter 9 Multiple Integrals

1 1 1- x2
2 Ú0 Ú0
= xy(1 - x 2 - y 2 ) dx dy

1 1 È 1- x2 È ˘
= Ú x ÍÚ Î (1 - x 2 )y - y3 ˘˚ ˙ dy dx
2 0 Î 0 ˚
1- x 2
1 1 y2 y 4
= Ú x (1 - x 2 ) - dx
2 0 2 4 0

1 1 È 2 (1 - x )
2 Ê 1 - x 2 )2 ˆ ˘
2 Ú0 Î
= x Í (1 - x ) - ÁË ˜ ˙ dx
2 4 ¯˚

1 1 È (1 - x 2 )2 ˘
2 Ú0 Î
= xÍ ˙ dx
4 ˚
1 1
8 Ú0
= x(1 - x 2 )2 dx

1 1
8 Ú0
= {x(1 - 2 x 2 + x 4 )} dx

1 1
8 Ú0
= ( x - 2 x 3 + x 5 ) dx

1
1 x2 2 x 4 x6
= - +
8 2 4 6 0

1 Ê 1 1 1ˆ
= Á - + ˜
8 Ë 2 2 6¯
1
=
48

Example 13
e log y e x
Evaluate Ú1 Ú1 Ú1 logg z dx dy dz. [Summer 2016]

Solution
The inner most limit depends on x and middle limit depends on y. Hence, integrating
first w.r.t. z,
e log
log y ex e llog
og y ex
Ú1 Ú1 Ú1 log z dx dy dz = Ú
log
1 1 Ú Ú1 llog
og z dz dx dy
9.6 Triple Integrals 9.121

x
e log y x 1
1
-Ú z dz x dy
1 1 1 z
e log y ex
x
- log 1 z1 d y
1 1
e È log y
- dy
1 1
e gy
x
- ex - +x dy
1 1
e lo
-2 e 1 dy
1

- 2 ) + g y e - ] dy
e
1

] dy
e
e
1

2 ˆ 1Ê 2 ˆ e
+ - + - + e - 1) y 1
y 1
1
e
2 2
e 1
log e +e +1 +y 2
-1 1]
1

e2 1
+1
2 4
e2 13
2e +
4 4

Example 14
• • • d y dz
Evaluate Ú Ú Ú ( x y z 2

Solution z
1. It is difficult to integrate this integral in r →∞
cartesian form. Putting x = r sin q cos f,
y = r sinq sin f, z = r cosq integral changes to
spherical form.
2. Limits of x x = xƕ
O
Limits of y y = Æ• y

Limits of z z •
The region of integration is the positive octant of
the plane. x
Limits of r r r • Fig. 9.128
9.122 Chapter 9 Multiple Integrals

p
Limits of q = 0 to q =
2
p
Limits of f : f = 0 to f =
2
Hence, the spherical form of the given integral is
• • • dxdydz
I=Ú Ú0 Ú0
0 ((1 + x + y 2 + z 2 )2
2

p p
• r 2 sin q dr dq df
=Ú Ú Ú
2 2
0 0 0 ((1 + r 2 )2
p p
• r 2 dr
=Ú Ú 2 sin q dq Ú 2 df
0 (1 + r 2 )2 0 0

Putting r = tan t , dr = sec 2 t dt


When r = 0, t = 0
p
When r Æ •, t =
2
p p p
tan 2 t
I = Ú 2 sin q dq Ú 2 sec 2 t dt Ú 2 df
0 0 sec 4 t 0
p p
Ê p 2 ˆ
= - cos q 2
0 ÁË Ú 2 sin t dt ˜¯ f
2
0
0

Ê p ˆ Ê p ˆ p
= Á - coss + cos 0˜ ◊ Á 2 1 - cos 2t dt ˜ ◊
Ë 2 ¯ Ú
Ë 0 2 ¯ 2
p
1 sin 2t 2 p
= t-
2 2 0 2
1 Èp 1 ˘p
= Í - (sin p - sin 0 )˙
2 Î2 2 ˚2
2
p
=
8

Example 15
a a2 - x2 a2 - x 2 - y2
Evaluate Ú0 Ú0 Ú0 xyz dx dy dz.
Solution
1. It is difficult to integrate this integral in cartesian form. Putting
x = r sin q coss f , y = r sin
sin q sin f,, z = r ccos
sin f os q integral changes to spherical form.
9.6 Triple Integrals 9.123

2. Limits of z : z = 0 to z = a - x - y
2 2 2
z
Limits of y : y = 0 to y = a - x 2 2

Limits of x : x = 0 to x = a
The region of integration is the positive octant P
r
of the sphere x 2 + y 2 + z 2 = a 2 .
q
Limits of r : r = 0 to r = a O y
p φ
Limits of q : q = 0 to q =
2
p
Limits of f : f = 0 to f = x
2
Hence, the spherical form of the given integral is Fig. 9.129
a a2 - x 2 a2 - x 2 - y2
I=Ú Ú0 Ú0 xyz dx dy dz
0
p p
a
=Ú2 2
Ú Ú r 3 sin 2 q cos q ◊ cos f sin f ◊ r 2 sin q dr dq df
f =0 q =0 r =0

p p
2f
sin 2f a
=Ú2 df Ú 2 sin os q dq Ú r 5 dr
sin 3q ccos
0 2 0 0

p p
È [ f (q )]n +1 ˘˙
a
1 - coss 2f sin 4 q 2 r6
Í∵ Ú [ f (q )]n f ¢(q )dq =
2
= ◊
2 2 4 6 Í n +1 ˙
0 0 0 Î ˚
1 1 1Ê p ˆ a6
= ◊ (- cos p + co
oss 0) ◊ Á ssin
in - sin 0˜ ◊
2 2 4Ë 2 ¯ 6
1 2 1 a6
= ◊ ◊ ◊
2 2 4 6
a6
=
48

Example 16
1 1- x 2 1 dz dy dx
dx
Evaluate Ú0 Ú0 Ú 2
x +y 2 by transforming into spherical
x 2 + y2 + z2
polar coordinates.
Solution
1. Limits of z : z = x + y to
2 2
z=1
Limits of y : y = 0 to y = 1 − x2
Limits of x : x = 0 to x=1
9.124 Chapter 9 Multiple Integrals

2. The region of integration is the part of the cone z2 = x2 + y 2 bounded above by the
plane z = 1 in the positive octant (since all three limits are positive).
3. Putting x = r sinq cos f, y = r sinq sin f, z = r cosq, spherical polar form of
(i) the cone z2 = x2 + y 2 is
r2 cos2q = r2 sin2q (cos2f + sin2f )
= r2 sin2q
cosq = sin q
tanq = 1
p
q=
4
(ii) the plane z = 1 is r cos q = 1
r = secq
z

r = sec q
A
y
P p
q=
4
f O
O x f y
P

Fig. 9.130
4. Draw an elementary radius vector OA which starts from the origin and terminates
on the plane r = secq.
Limits of r : r = 0 to r = secq
p
Limits of q : q = 0 to q =
4
p
Limits of f : f = 0 to f = (in positive octant)
2
Hence, the spherical form of the given integral is
1 1- x 2 1 dz dy dx
I=Ú
0 Ú0 Ú x 2 + y2
x 2 + y2 + z2
p p
sec q r 2 sin q dr dq df
=Ú2
f = 0 Úq = 0 Úr = 0
4
r
p p
= Ú 2 Ú 4 ÈÍ Ú r dr ˘˙ sin q dq df
sec q
0 0 Î 0 ˚
9.6 Triple Integrals 9.125

p p sec q
r2
Ú Ú2 4
2
sin
0
p È p
sec 2 q ˘
Ú2 Ú4 2
sin q q df

p p
1

4 sec q q

p p
1
f q 04
2
1 p p
- c0
2 2 4
p
4
- )
Type II Evaluation of Triple Integrals Over the Given Region

Example 1
Evaluate Ú x 2 y over the region bounded by the planes
x = 0, y = 0, z = 0 and x + y + z = 1.
Solution
1. Draw an elementary volume AB parallel to z-axis in the region. AB starts from
xy-plane and terminates on the plane x + y + z = 1.
z

y
R (0, 0, 1)
Q (0, 1)
x+y z 1
B
B

Q (0, 1, 0)
O
A y O A′ (1, 0) x

P (1, 0, 0)
x

Fig. 9.131
9.126 Chapter 9 Multiple Integrals

Limits of z : z = 0 to z = 1 – x – y
2. Projection of the plane x + y + z = 1 in xy-plane is DOPQ. Putting z = 0 in x + y + z = 1,
the equation of the line PQ is obtained as x + y = 1.
3. Draw a vertical strip A¢B¢ in the region OPQ. A¢B¢ starts from the x-axis and
terminates on the line x + y = 1.
Limits of y : y = 0 to y = 1 – x
Limits of x : x = 0 to x = 1
1 1- x 1- x - y 2
I=Ú Ú Ú x yzz dz dy dx
0 0 0
1- x - y
1 1- x z2
=Ú Ú x2 y dy dx
0 0 2
0

=
1 1 2 È 1- x
2 Ú0 ÍÎ Ú0
x {
y (1 - x )2 + y 2 - 2 y(1 - x ) dy ˘˙ dx
˚ }
1 1 2 È 1- x
= Ú x ÍÚ
2 0 Î 0
{
y(1 - x )2 + y3 - 2y
˚ }
2 y 2 (1 - x ) dy ˘˙ dx
1- x
1 1 2 y2 y 4 y3
= Ú
2 0
x (1 - x )2
2
+
4
- 2(1 - x )
3 0
dx

1 1 2È 2 (1 - x )
2
(1 - x ) 4 (1 - x )3 ˘
2 Ú0 Î
= x Í(1 - x ) ◊ + - 2 (1 - x ) ◊ ˙ dx
2 4 3 ˚
1 1 x2
2 Ú0 12
= (1 - x )4 dx
1
1 (1 - x )5 2 (1 - x )6 (1 - x )7
= ◊x - ◊ 2x + ◊2
24 -5 30 -210
0
1 Ê 1 ˆ
= Á 0+
24 Ë 105 ˜¯
1
=
2520

Example 2
Evaluate ÚÚÚ 2xx dV , where E is the region under the plane
E
2x + 3y + z = 6 that lies in the first octant. [Winter 2015]
Solution
1. Draw an elementary volume AB parallel to z-axis in the region. AB starts from xy-plane
and terminates on the plane 2x + 3y + z = 6.
Limits of z : z = 0 to z = 6 – 2x – 3y
9.6 Triple Integrals 9.127

2. Projection of the plane 2x + 3y + z = 6 in xy-plane is DOPQ. Putting z = 0 in


2x + 3y + z = 6, the equation of the line PQ is obtained as 2x + 3y = 6.
z

y
R (0, 0, 6)
Q (0, 2)

B′
B

Q (0, 2, 0)
O
A y O A′ (3, 0) x

P (3, 0, 0)
x

Fig. 9.132

3. Draw a vertical strip A¢B¢ in the region OPQ. A¢B¢ starts from the x-axis and
terminates on the line 2x + 3y = 6.
6 - 2x
Limits of y : y = 0 to y=
3
Limits of x : x = 0 to x=3
I = ÚÚÚ 2 x dV
E
6 -2 x
3 6 -2 x -3 y
=Ú Ú3
Ú 2 x dz dy dx
0 0 0
6 -2 x
3 6 -2 x -3 y
=Ú Ú3 2x z 0 dy dx
0 0

6 -2 x
3
=2Ú Ú 3 x(6 - 2 x - 3 y) dy dx
0 0
6 -2 x
3 3 y2 3
=2Ú x (6 - 2 x ) y - dx
0 2
0

3 È (6 - 2 x ) 3 Ê 6 - 2 x ˆ ˘
2
= 2Ú x Í(6 - 2 x ) - Á ˙ dx
0
ÍÎ 3 2 Ë 3 ˜¯ ˙
˚
9.128 Chapter 9 Multiple Integrals

3 (6 - 2 x )2
= 2Ú x dx
0 6
4 3
3 Ú0
= x (9 + x 2 - 6 x )dx

4 3
= Ú (9 x + x 3 - 6 x 2 ) dx
3 0
3
4 x2 x4 x3
= 9 + -6
3 2 4 3
0
=9

Example 3
Evaluate ÚÚÚ xyz dx dy dz over the positive octant of the sphere
x2 + y2 + z2 = 4.
Solution
Putting x = r sin q cos f, y = r sin q sin f, z
z = r cos q, the equation of the sphere
x2 + y2 + z2 = 4 reduces to r2 sin2q cos2f + r2 sin2q
sin2f + r2 cos2 q = 4, r2 = 4, r = 2.
The region is the positive octant of the sphere r P
r = 2. q
Limits of r : r = 0 to r = 2 O y
π φ
Limits of q=0 to θ=
2
π
Limits of f=0 to φ= x
2 Fig. 9.133
Hence, the spherical form of the given integral is

I = ÚÚÚ xyz dx dy dz
p p
2
= Ú 2 Ú 2 Ú (r 3 sin 2 q cos q cos f si
sinn f )r 2 sin q dr dq df
0 0 0
p p
sin 2f 2
= Ú sinn 3 q cos
2 cos q dqÚ 2
dq df Ú r 5 dr
0 0 2 0
p p 2
sinn 4 q 2 - cos 2f r6 È [ f (q )]n +1 ˘
Í∵ Ú [ f (q )] f ¢(q )dq =
2 n
= , n π 1˙
4 4 0 6 Î n +1 ˚
0 0
9.6 Triple Integrals 9.129

1Ê 4 p ˆÈ 1 ˘ Ê 26 ˆ
= Á sin - sin 0˜ Í - (cos p - coss 0)˙ Á ˜
4Ë 2 ¯Î 4 ˚Ë 6 ¯
4
=
3

Example 4
dx dy dz
Evaluate ÚÚÚÚ over the region bounded by the sphere
a2 - x 2 - y2 - z2
x2 + y2 + z2 = a2.
Solution
1. Putting x = r sin q cos f, y = r sin q sin f, z = r cos q, the equation of the sphere
x2 + y2 + z2 = a2 reduces to r = a.
2. For the complete sphere, limits of r : r = 0 to r = a
limits of q : q = 0 to q = p
limits of f : f = 0 to f = 2p
Hence, the spherical form of the given integral is
dx dy dz
I = ÚÚÚ
a - x 2 - y2 - z2
2

2p p a r2 sin q dr dq df
=Ú Ú0 Ú0
0
a2 - r 2
2p p a r 2 + a2 - a2
=Ú df
d f Ú sin q dq Ú dr
0 0 0
a2 - r 2
2p p aÊ a2 ˆ
= f 0 ◊ - cos q 0 ◊ Ú Á - a 2 - r 2 ˜ dr
0
Ë a2 - r 2 ¯
a
-1 r r a2 -1 r
= (22p
p )(- cos p + cos 0) a sin 2
- a2 - r 2 - sin -1
a 2 2 a
0

Ê a2 ˆ
sin -1 1 -
= 2p (2) Á a 2 sin sin -1 1˜
Ë 2 ¯
Ê a2 ˆ
= 4p Á sin -1 1˜
Ë 2 ¯
a2 p
= 4p ◊ ◊
2 2
= p 2 a2
9.130 Chapter 9 Multiple Integrals

Example 5
d
Evaluate ∫∫∫ 1 over the region bounded by the spheres
2
( z )
x + y + z = a and x + y 2 + z 2 = b2, a > b > 0.
2 2 2 2 2

z
Solution
1. Putting x = r sinq cos f, y = r sinq sin f,
z = r cosq, equations of the spheres
x2 + y2 + z2 = a2 and x 2 + y 2 + z 2 = b2 reduce
to r = a and r = b respectively. r a
2. Draw an elementary radius vector OAB A r b
from the origin in the region. This radius O
y
vector enters in the region from the sphere
r = b and terminates on the sphere r = a.
3. Limits of r : r = b to r = a.
For the complete sphere,
limits of q : q = 0 to q = p
limits of f : f = 0 to f = 2p x
Hence, the spherical form of the given
integral is Fig. 9.134
dx y dz
I ÚÚÚ 1
(x + z )2
2p p a r 2 sinq
Ú Ú Ú r
dr dq df
2p a
Ú0 d dq Ú r r
b
a
2
p r
◊- 0

2
b

(a - )
- + 0)
2
2
2p ( ).

Example 6
ÚÚÚ z
2
Evaluate x dy z over the region common to the sphere
x2 + y2 + z2 = 4 and the cylinder x2 + y2 = 2x.
9.6 Triple Integrals 9.131

Solution
1. Putting x = r cos q, y = r sin q, z = z, the q p
equation of 2
(i) the sphere x 2 + y 2 + z 2 = 4 reduces to
r2 + z2 = 4
z 2 = 4 – r 2. A

(ii) the cylinder x2 + y 2 = 2x reduces to r = 2cos q


r 2 = 2r cosq, r = 2 cosq. O q=0
2. Draw an elementary volume parallel to z-axis
in the region. This elementary volume starts
from the part of the sphere z2 = 4 – r 2, below xy-
plane and terminates on the part of the sphere
z2 = 4 – r 2, above xy-plane.
Fig. 9.135
Limits of r z = - r
3. Projection of the region in rq -plane is the circle r = 2 cosq.
4. Draw an elementary radius vector OA in the region (r = 2 cosq ) which starts from
the origin and terminates on the circle r = 2 cosq
Limits of r : r = 0 to r = 2 cos q
p p
Limits of q q to q =
2 2
Hence, the cylindrical form of the given integral is
I z 2 dx y dz
p
2 cosq 4 r
= Ú -2p Ú Ú z2r d r dq
4 r2
2

p 4 -r
2 cosq z3
=Ú 2
p Ú r r dq
3
2 -r
p 3
1 2 cosq
= p r r dr dq
3
2
p 3
1 2 cosq
= p -(4 r ) 2 ( 2r r dq
3
2
2 q
5
p
1 - [ (r )]n +1 ˘
3 Ú- Ú
=- (r )dr
+1
2 0
p
= - Ú 2p È
2 5 5
dq
15 - (4 c 2
q ) - ( 4) 2
2
9.132 Chapter 9 Multiple Integrals

p
2 2 5 5
15 Ú- p
=- (2 sin q - 25 ) dq
2

È∵ a f (q )dq = 0, iiff f (-q ) = - f (q )˘


2 È ˘
p
= - Í0 - 2 5 q 2 ˙ Í Ú- a ˙
15 Í p
- ˙ Í ˙
ÎHere sinn (--q
q ) = - sin
5 5
Î 2˚ sin q ˚
26 p
=
15
64p
=
15

Example 7
Evaluate ÚÚÚÚ xyz dx dy dz over the region bounded by the planes
x = 0, y = 0, z = 0, z = 1 and the cylinder x2 + y 2 = 1.
Solution
1. Putting x = r cosq, y = r sinq, z = z, equation of the cylinder x2 + y2 = 1 reduces to
r 2 = 1, r = 1.

B z=1 q= p
2

A
A′
r=1
O
y O q=0

Fig. 9.136

2. Draw an elementary volume AB parallel to z-axis in the region. This elementary


volume AB starts from xy-plane and terminates on the plane z = 1.
Limits of z : z = 0 to z = 1
9.6 Triple Integrals 9.133

3. Projection of the region in rq -plane is the part of the circle r = 1 in the first quadrant.
4. Draw an elementary radius vector OA¢ in the region in the rq -plane which starts
from the origin and terminates on the circle r = 1.
Limits of r : r = 0 tor=1
p
Limits of q : q = 0 to q =
2
Hence, the cylindrical form of the given integral is

I = ÚÚÚ xyz dx dy dz
p
1 1
=Ú Ú Ú
2 r2 cos q sin q ◊ zzrr dz dr dq
z=0 q =0 r =0
p
1 sin
si n 2q 1
= Ú z dz Ú 2 dq Ú r 3 dr
0 0 2 0

1 p 1
z2 cos 2q 2 r4
= -
2 4 0 4
0 0
1
=
16

Example 8
Evaluate ∫∫∫∫ x 2 + y 2 dx dy dz
dz over the region bounded by the right
circular cone x2 + y2 = z2, z > 0 and the planes z = 0 and z = 1.
Solution
1. Putting x = r cosq, y = r sinq, z = z, the equation of the cone x 2 + y 2 = z 2 reduces to
r 2 = z 2, r = z.
2. Draw an elementary volume AB parallel to z-axis in the region, which starts from
the cone r = z and terminates on the plane z = 1.
Limits of z : z = r to z = 1.
3. Projection of the region in rq -plane is the curve of intersection of the cone r = z
and the plane z = 1 which is obtained as r = 1, a circle with centre at the origin and
radius 1.
4. Draw an elementary radius vector OA¢ in the region which starts from the origin and
terminates on the circle r = 1.
Limits of r : r = 0 to r = 1
Limits of q : q = 0 to q = 2p
9.134 Chapter 9 Multiple Integrals

B z 1
= p
2
r z

A A′
r 1

O y
=0

Fig. 9.137

Hence, the cylindrical form of the given integral is

I dx dz
2p 1 1
◊r d r dq
=0 r =0 z =r
2p 1 2 1
Ú r z r d dq
2p 1
= - dr
1
2 r3 r4
0

3 4
1

12

Example 9
Evaluate Ú ( ) dz over the region bounded by the parabo-
2 2
loid x + y = 3z and the plane z = 3.
Solution
1. Putting x = r cos q, y = r sin q, z = z, the equation of the paraboloid x2 + y2 = 3z
reduces to r 2 = 3z.
2. Draw an elementary volume AB parallel to z-axis in the region which starts from the
paraboloid r 2 = 3z and terminates on the plane z = 3.
9.6 Triple Integrals 9.135

r2
Limits of z : z = to z = 3
3
3. Projection of the region in rq -plane is the curve of intersection of the paraboloid
r 2 = 3z and the plane z = 3 which is obtained as r 2 = 9, r = 3, a circle with centre at
the origin and radius 1.

z=3 q= p
2

r 2 = 3z A′
r=3
A q
O y O q=0

Fig. 9.138

4. Draw an elementary radius vector OA¢ in the region (circle r = 3) which starts from
origin and terminates on the circle r = 3.
Limits of r : r = 0 to r = 3
Limits of q : q = 0 to q = 2p
Hence, the cylindrical form of the given integral is
I = ÚÚÚ ( x 2 + y 2 )dx dy dz
2p 3 3
=Ú Ú0 Ú r 2 r 2 ◊ r dz dr dq
0
3
2p 3 3 3
=Ú Ú r z dr dq
0 0 r2
3

2p 3 Ê r2 ˆ
=Ú dq
d q ◊ Ú r 3 Á 3 - ˜ dr
0 0 Ë 3¯
3
2p 3r 4 r 6
=q0 -
4 18
0

Ê3 3 ˆ 5 6
= 2p Á - ˜
Ë 4 18 ¯
81p
=
2
9.136 Chapter 9 Multiple Integrals

Example 10
x 2 y2 z2
Evaluate ÚÚÚ 1- - - dx dy dz, where V is the volume of the
a 2 b2 c 2
V

x 2 y2 z2
ellipsoid + + = 1.
a 2 b2 c 2
Solution
It is difficult to integrate this integral in cartesian form. Therefore, transforming the
ellipsoid into a sphere using following change of variables.
x y z x 2 y2 z2
Putting = u, = v, = w, equation of the ellipsoid 2 + 2 + 2 = 1 reduces to
a b c a b c
u 2 + v 2 + w2 = 1, which is a sphere of radius 1 and centre at the origin,

dx dy dz = |J| du dv dw
∂x ∂x ∂x
∂u ∂v ∂w
∂( x, y,
y, z ) ∂y ∂y ∂y
where, J= =
∂(u, v,
v, w) ∂u ∂v ∂w
∂z ∂z ∂z
∂u ∂v ∂w
a 0 0
= 0 b 0 = abc
0 0 c
Therefore, dzz = abc du dv dw
dx dy d
New form of the integral is
x2 y2 z2
I = Ú Ú Ú 1- 2
- 2
- dx dy dz
a b c2

= Ú Ú Ú 1 - u2 - v 2 - w 2 ◊ abc du dv dw
Since in the new coordinate system u, v, w, the region of integration is a sphere,
therefore using spherical coordinates u = r sinq cos f, v = r sin q sin f, w = r cosq and
du dv dw = r2 sin q dr dq df, the equation of the sphere u2 + v2 + w2 = 1 reduce to r2 =1, r = 1.
For complete sphere limits of r : r = 0 to r = 1 (radius of sphere)
limits of θ : θ = 0 to θ = π
limits of φ : φ = 0 to φ = 2π
9.6 Triple Integrals 9.137

Hence, the spherical form of the given integral is


2p p 1
I=Ú Ú0 Ú0 1 - r 2 abc ◊ r 2 sin q dr dq df
0
2p p 1
= abc Ú df Ú sin q dq Ú r 2 1 - r 2 dr
0 0 0
Putting r = sin t, ddrr = cos t dt
When r = 0, t=0
p
When r = 1, t=
2
p
2p p
\ I = abc f 0 - cos q 0 Ú02 sin t ◊ cos t ◊ cos t dt
2

1 1 π
= abc(2π )(2) ⋅  ⋅ ⋅  [Using reduction formula]
 4 2 2
π 2 abc
=
4

Example 11
Evaluate ÚÚÚÚ x 2 y 2 z 2 dx dy dz over the region bounded by the surfaces
xy = 4, xy = 9, yz = 1, yz = 4, zx = 25, zx = 49.
Solution
Evaluation of integral becomes easier by changing the variables. Under the trans-
formation xy = u, yz = v, zx = w, the surfaces get transformed to u = 4, u = 9,
v = 1, v = 4, w = 25, w = 49.
These equations represent the planes parallel to v w, w u and uv planes in the new
coordinate system.
It is easier to find partial derivatives of u, v, w w.r.t. x, y and z.
∂u ∂u ∂u
∂x ∂y ∂z
∂(u, v, w) ∂v ∂v ∂v
=
∂( x, y, z ) ∂x ∂y ∂z
∂w ∂w ∂w
∂x ∂y ∂z
y x 0
= 0 z y
z 0 x
= y( zzxx - 0) - x(0 - yz )
= 2 xyz
9.138 Chapter 9 Multiple Integrals

w = J dx ddyy dz = 2 xyz ddxx dy dz


du dv ddw
1
dzz =
dx dy d du dv dw
2 xyz
1
du dv dw = [∵ x 2 y 2 z 2 = uvw]
2 uvw
Limits of u : u = 4 to u = 9
Limits of v : v = 1 to v = 4
Limits of w : w = 25 to w = 49
Hence, the new form of the integral is
I = ÚÚÚ x 2 y 2 z 2 dx dy dz
49 4 9 1

w = 25 Úv =1 Úu = 4
uvw ◊ du dv dw
2 uvw
1 1 1
1 49 2 4 9
= Ú
2 25
w dw Ú v 2 dv Ú u 2 du
1 4
49 4 9
3 3 3
1 2w 2 2v 2 2u 2
=
2 3 25 3 1 3 4
4
= (343 -125
- 125)(8 - 1)(27 - 8)
27
115976
=
27

EXERCISE 9.7
(I) Evaluate the following integrals:
1 2 2
1. ∫ dx ∫
0 0
dy ∫ x 2 yyzz dz
1
Ans. :1
log 2 x x +y
2. ∫ ∫ ∫
0 0 0
e x + y + z dz dy d x
 5
 Ans. : 8 
 
π
a cos θ a2 −r 2
3. ∫ ∫
0
2
0 ∫ 0
r dz dr d θ
 a3  π 2  
 Ans. :  −  
 3 2 3 
9.6 Triple Integrals 9.139

π 1+ cos θ )
a(1+ h  r 
4. ∫ ∫
0 0 ∫ 0
2 1 −
1 + cos θ) 
 a(1+
r dz dr d θ

 π a 2h 
 Ans. : 
 2 
4 2 z 4 z−x2
5. ∫∫ ∫
0 0 0
dy dx dz

Ans. : 8π 
2 2
π a −r
a sin θ
6. ∫ ∫
0
2
0 ∫ 0
a
r dz dr dθ
 5a 3 
 Ans. : 
 64 
2 y x +y
7. ∫∫ ∫
0 0 x −y
(x + y + z)dz dx dy

Ans. :16 
a a2 − x 2 a2 − x 2 − y 2
8. ∫∫
0 0 ∫ 0
xyzz dz dy dx.
 a6 
 Ans. : 
 48 

(II) Evaluate the following integrals over the given region of integration:

1. ∫∫∫ (x + y + z)dx dy dz over the tetrahedron bounded by the planes


x = 0, y = 0, z = 0 and x + y + z = 1.
 1
 Ans. : 8 
 
dx dy dz
2. ∫∫∫ (1 + x + y + z) 3 over the tetrahedron bounded by the planes x = 0,

y = 0, z = 0 and x + y + z = 1.
 1 5 
 Ans. :  log 2 −  
 2 8 

3. ∫∫∫ xyzz dx dy dz over the positive octant of the sphere x2 + y2 + z2 = a2.


 a6 
 Ans. : 
 48 

∫∫∫ xyzz(x + y 2 + z 2 )dx dy dz over the positive octant of the sphere


2
4.
x 2 + y 2 + z 2 = a2.
 a8 
 Ans. : 
 64 
9.140 Chapter 9 Multiple Integrals

∫∫∫ ((yy z 2 + z 2 x 2 + x 2 y 2 ) dx dy dz over the sphere of radius a and centre


2
5.
at the origin.
 4π a 7 
 Ans. : 
 35 
z2
6. ∫∫∫ x 2 + y 2 + z 2 dx dy dz over the sphere x 2 + y 2 + z 2 = 2.
 8π 2 
 Ans. : 
 9 
dx dy dz
7. ∫∫∫ 3
over the region bounded by the spheres x2 + y2 + z2 = a2
(x 2 + y 2 + z ) 2 2

and x2 + y 2 + z2 = b2, a > b > 0.


  a
 Ans. : 4π log   
 b 

∫∫∫ z dx dy dz
2
8. over the region common to the spheres x 2 + y 2 + z 2 = a 2
and cylinder x 2 + y 2 = ax.
 2π a 5 
 Ans. : 
 15 

∫∫∫ (x + y 2 )dx dy dz over the region bounded by the paraboloid x2 + y2 = 2z


2
9.
and the plane z = 2.
 16π 
 Ans. : 3 
 

∫∫∫ x
2
10. y z dx dy dz over the tetrahedron bounded by the planes x = 0,
x y z
y = 0, z = 0 and + + = 1.
a b c
 a 3 b 2c 2 
 Ans. : 
 2520 

11. ∫∫∫ xyzz dx dy dz over the positive octant of the ellipsoid


2 2 2
x y z
2
+ 2 + 2 ≤ 1.
a b c  a 2 b 2c 2 
 Ans. : 
 48 

x 2 y 2 z2
12. ∫∫∫ 1 + 4 + 9 dx dy dz over the region bounded by the ellipsoid
x 2 y 2 z2
+ + = 1.
1 4 9
Ans. : 8π 
9.7 Area as Double Integral 9.141

9.7 AREA AS DOUBLE INTEGRAL

9.7.1 Area in Cartesian Coordinates


(i) The area A bounded by the curves y = y1(x) y
and y = y2(x) intersecting at the points
P (a, b) and Q (c, d ) is

x Q c, d )

(ii) If equation of the curves are represented


as x = x1(y) and x = x2(y) then
d P (a b)
y

Note: Consider the symmetricity of the region O x


while calculating area.
Fig. 9.139

Example 1
x2 y2
Find the area bounded by the ellipse + = 1, above x-axis.
a2 b
Solution
1. The region is symmetric about y-axis. Total y
area = 2 (area bounded by the ellipse in the
first quadrant) 2
x +y 1
2. Draw a vertical strip AB in the region (0, b)
B a b2
which lies in the first quadrant. AB starts
from the x-axis and terminates on the
x2 2
x
ellipse 2 + =1 O A P (a, 0)
a b
x2
Limits of y : y = 0 to
a2
Limits of x : x = 0 to x = a
x
a b 1
x Fig. 9.140

x
a b 1
= 2∫ y 0
2
dx

a x2
= − dx
a2
2b a
2 2
x
a
9.142 Chapter 9 Multiple Integrals

a
2b x 2 a2 x
= a − x 2 + sin −1
a 2 2 a0
2b  a 2 −1 
=  sin 1
a  2
2b  a 2 π 
=  ⋅ 
a  2 2
π ab
=
2

Example 2
Find the area bounded by the parabola y 2 = 4x and the line 2x –3y + 4 = 0.
Solution
1. The points of intersection of the parabola y2 = 4x and the line 2x – 3y + 4 = 0 are
obtained as
2
 2x + 4
  = 4 x
3
( x + 2) 2 = 9 x y

x2 − 5x + 4 = 0
B Q (4, 4)
x = 1, 4
∴ y = 2, 4 A
2xx – 3yy + 4 = 0 P (1, 2)
The points of intersection are P (1, 2) and Q (4, 4).
2. Draw a vertical strip AB which starts from the
O x
line 2x – 3y + 4 = 0 and terminates on the pa-
rabola y2 = 4x. y 2 = 4x
2x + 4
Limits of y : y = to y = 2 x
3
Limits of x : x = 1 to x = 4
4 2 x
A = 2∫ ∫ 2 x+4 dy dx
dx
1
3
4 Fig. 9.141
= ∫ y 2 x+
2 x
x + 4 dx
dx
1
3
4  2x + 4
=∫  2 x −  dx
dx
1 3 
3 4
2
= 2 ⋅ 2 ⋅ x − x − 4x 2

3 3 3 1
4 1 4
= (8 − 1) − (16 −1
− 1) − (4 − 1)
3 3 3
1
=
3
9.7 Area as Double Integral 9.143

Example 3
Find the area enclosed by the curves y = x2 and y = x.
Solution
1. The points of intersection of the parabola y
y = x2 and the line y = x are obtained as
x = x2 y =x P(1,1)
x = 0, 1
\ y = 0, 1 B y = x2
The points of intersection are O (0, 0) and
P (1, 1). A
O x
2. Draw a vertical strip AB which starts from
the parabola y = x2 and terminates on the
line y = x.
Limits of y : y = x2 to y = x Fig. 9.142
Limits of x : x = 0 to x = 1
1 x
A=∫ ∫ dy dx
dx
0 x2

1 x
= ∫ y 2 dx
0 x
1
= ∫ ( x − x 2 ) dx
dx
0
1
x 2 x3
= −
2 3 0

1 1
= −
2 3
1
=
6

Example 4
Find the area enclosed by the parabola y2 = 4ax and the lines
x + y = 3a, y = 0 in the first quadrant.
Solution
1. The points of intersection of the parabola y2 = 4ax and the line x + y = 3a are
obtained as
y2 = 4a(3a – y)
y + 4ay – 12a2 = 0
2

y = 2a, – 6a
\ x = a, 9a
The point of intersection is Q (a, 2a) which lies in the first quadrant.
2. Area enclosed in the first quadrant is OPQ.
9.144 Chapter 9 Multiple Integrals

Draw a horizontal strip AB which starts from the parabola y2 = 4ax and terminates
on the line x + y = 3a. y
y2
Limits of x x to x = 3a – y
4a x + y = 3a
Limits of y : y = 0 to y = 2a y 2 = 4ax
2a 3
y Q(a 2a)
4a
B
2a 3a y P
=∫ x y O x
4a

2 y2
− y
4a
2a
y2 y3
= −
2 4 3 0
Fig. 9.143
3
1 8a
− ⋅
4a 3
10 2
= a
3
Note: In case of vertical strip, two vertical strips are required to cover the entire region.
Therefore one horizontal strip is preferred over vertical strip.

Example 5
Find the area bounded by the parabolas y2 = 4ax and x2 = 4ay.
Solution
1. The points of intersection of the parabolas y2 = 4ax and x2 = 4ay are obtained as
2
x2
= 4ax y
4a
x4 = 16a2 (4ax) y = 4ax
x(x3 – 64a3) = 0 B
P(4a a)
x = 0, x = 4a
\ y = 0, y = 4a
x 2 = 4ay
The points of intersection are O (0, 0) and
A
P (4a, 4a). O x
2. Draw a vertical strip AB which starts
from the parabola x2 = 4ay and termi-
nates on the parabola y2 = 4ax.
x2
Limits of y y = to y ax
4a
Limits of x : x = 0 to x = 4a Fig. 9.144
9.7 Area as Double Integral 9.145

4a 2 ax
A=∫ ∫ x2 dy dx
dx
0
4a
4a
=∫
2 ax
y x2
dx
0
4a

4a  x2 
= ∫  2 ax −  dxdx
0  4a 
4a
2 3 1 x3
= 2 a ⋅ x2 − ⋅
3 4a 3 0
3
4 1
= a ( 4a ) 2 − 4a )3
(4a
3 12a
32 2 16 2
= a − a
3 3
16
= a2
3

Example 6
Find the area enclosed by the curves y = 2 – x and y2 = 2(2 – x).
Solution
1. The points of intersection of the line y = 2 – x and the parabola y2 = 2(2 – x) are
obtained as y
(2 – x)2 = 2(2 – x)
(2 – x)(2 – x – 2) = 0 Q (0, 2)
(2 – x)(–x) = 0 y =2−x
B
x = 2, 0 y 2 = 2 (2 −x)
y = 0, 2 A P (2, 0)
The points of intersection are P (2, 0) and
O x
Q (0, 2).
2. Draw a vertical strip AB which starts
from the line y = 2 – x and terminates
on the parabola y2 = 2 (2 – x).
Limits of y : y = 2 – x to y = 2(2 − x)
Limits of x : x = 0 to x = 2 Fig. 9.145
2 2( 2− x )
A=∫ ∫ dy dx
dx
0 2− x

2 2( 2− x )
=∫ y dx
0 2− x

2 
1
= ∫  2 (2 − x) 2 − (2 − x)  dx
dx
0
 
9.146 Chapter 9 Multiple Integrals

3 2

2(2 − x) 2 x2
= 2⋅ − 2x +
−3 2
0

 8 1
=  0 +  − 2( 2 − 0 ) + ( 4 − 0 )
 3  2
2
=
3

Example 7
Find the area bounded between the parabolas x2 = 4ay and
x2 = – 4a (y – 2a).
Solution
1. The parabola x2 = 4ay has vertex (0, 0) and the parabola x2 = – 4a(y – 2a) has vertex
(0, 2a). Both the parabolas are y
symmetric about the y-axis.
2. The points of intersection of
Q (0, 2a)
x2 = 4ay and x2 = – 4a(y – 2a) are B x 2 = 4ay
obtained as
4ay = −44aa ( y − 2a ) R P (2a, a)
(−2a, a)
8ay = 8a 2 x 2 = −4a (y − 2a)
y=a A
O x
∴ x = ± 2a

The points of intersection are


P (2a, a) and R (–2a, a). Fig. 9.146
3. The region is symmetric about
y-axis.
Total area = 2 (Area in the first quadrant)
4. Draw a vertical strip AB in the region which lies in the first quadrant. AB starts from
the parabola x2 = 4ay and terminates on the parabola x2 = – 4a(y – 2a).
x2 x2
Limit of y : y = to y = 2a −
4a 4a
Limits of x : x = 0 to x = 2a
x2
2a 2a −
A = 2∫ ∫ x2
4a
dy dx
dx
0
4a
x2
2a 2a −
= 2∫ y x2
4a
dx
dx
0
4a
2a  x2 x2 
= 2∫  2 a − −  dxd
0 
4a 4a 
9.7 Area as Double Integral 9.147

2a
x3
= 2 2ax −
6a 0
 4 
= 2  4a 2 − a 2 
 3 
16
= a2
3

Example 8
Find smaller of the area enclosed by the curves y = 2 – x and x2 + y2 = 4.
Solution
1. The points of intersection of the line y = 2 – x and the circle x2 + y2 = 4 are obtained as
x2 + (2 – x)2 = 4 y
x + 4 – 4x + x2 = 4
2

2x2 = 4x
x = 2, 0 Q (0, 2)
\ y = 0, 2 B
y

The points of intersection are P (2, 0) x2 + y2 = 4


=
2

and Q (0, 2).


x

A P (2, 0)
2. Draw a vertical strip AB which starts
from the line y = 2 – x and terminates O x
on the circle x2 + y2 = 4.
Limits of y : y = 2 – x to y = 4 − x 2
Limits of x : x = 0 to x = 2
2 4 − x2
A=∫ ∫ dy dx
dx Fig. 9.147
0 2− x
2
2 4− x
=∫ y dx
0 2− x

= ∫  4 − x 2 − (2 − x)  dx
2
dx
0  
2
x 4 x x2
= 4 − x 2 + sin −1 − 2 x +
2 2 2 2 0

= 2 sin −1 1 − 2
=π −2

Example 9
Find the area of the loop of the curve x(x2 + y2) = a (x2 – y2).
9.148 Chapter 9 Multiple Integrals

Solution
 a − x
The equation of the curve can be rewritten as y 2 = x 2 
 a + x 
1. The points of intersection of the curve with x-axis ( y = 0) are obtained as
x2(a – x) = 0
x = 0, x = a.
y
The loop of the curve lies between the
points O (0, 0) and P (a, 0).
2. The region is symmetric about x-axis
Total area = 2 (Area above x-axis) B
P (a, 0)
3. Draw a vertical strip AB in the region x
A
above x-axis. AB starts from x-axis and O
a − x
terminates on the curve y 2 = x 2 
x=−a
 .
a + x

a−x
Limits of y : y = 0 to y=x
a+ x
Fig. 9.148
Limits of x : x = 0 to x=a
a− x
a x
A = 2∫ ∫ a+ x
dyy dx
d
0 0

a−x
a x
= 2∫ y 0 a+ x
dx
dx
0

a a−x
= 2∫ x dx
dx
0 a+ x
Putting x = a cosq, dx = – a sin q dq
π
When x = 0, θ =
2
When x = a, q = 0
0 a - a cos q
A = 2 Ú p a cos q (- a sin q )dq
2
a + a cos q
q
p 2 sin 2
= 2 a Ú cos q sin q ◊
2 2 2 dq
0 q
oss2
2 co
2
q
p sin
q q 2 dq
= 2 a Ú cos
2 2
cos q ◊ 2 ssin cos ◊
0 2 2 q
cos
2
p
= 2 a 2 Ú 2 coss q (1 - coss q ) dq
0
9.7 Area as Double Integral 9.149

p
Ê 1 + cos 2q ˆ
= 2 a 2 Ú 2 Á cos q - ˜¯ dq
0 Ë 2
p
q sin
sin 2q 2
= 2 a sin q - -
2
2 4 0

ÈÊ p ˆ 1Êp ˆ 1 ˘
n 0˜ - Á - 0˜ - (sin p - sin 0 )˙
= 2 a 2 ÍÁ sin - sin
Î Ë 2 ¯ 2 Ë 2 ¯ 4 ˚
Ê p ˆ
= 2a 2 Á 1 - ˜
Ë 4¯

Example 10
Find the area included between the curve y2(2a – x) = x3 and its
asymptote. [Summer 2017]
Solution
The equation of the curve can be rewritten as
x3 y
y2 =
2a - x
B Asymptote
1. The point of intersection of the curve
with x-axis (y = 0) is x = 0.
2. The region is symmetric about
x-axis. x
Total area = 2 (Area above x-axis) O A x = 2a
3. Draw a vertical strip AB in the re-
gion above x-axis. AB starts from
x-axis and terminates on the curve
x3
y2 = .
2a - x Fig. 9.149
x
Limits of y : y = 0 to y = x
2a - x
Limits of x : x = 0 to x = 2 a
x
2a x
A = 2Ú Ú0 2a - x dy dx
0
x
2a x
2a - x
= 2Ú y 0
dx
0

2a x
= 2Ú x dx
0 2a - x
9.150 Chapter 9 Multiple Integrals

Putting x = 2a sin2q,
dx = 2a (2 sinq cosq dq)
When x = 0, q = 0
p
When x = 2 a, q =
2
p
2 a sin 2 q
A = 2 Ú 2 2 a sin 2 q ◊ 4 a sin q cos q dq
0 2 a cos2 q
p
sin q
= 2 Ú 2 2 a sin 2 q ◊ ◊ 4 a sin q cos q dq
0 cos q
p
= 16 a 2 Ú 2 sin 4 q dq
0

3 1 p
= 16 a 2 ◊ ◊ ◊
4 4 4
= 3p a 2

Example 11
Find the area between the rectangular hyperbola 3xy = 2 and the line
12x + y = 6.
Solution
1. The points of intersection of the rectangular hyperbola 3xy = 2 and the line
12x + y = 6 are obtained as y
3 x (6 − 12 x) = 2
1 ,
18 x 2 − 9 x + 1 = 0 Q
6
4

1 1
x= , 12x + y = 6
3 6
B
∴ y = 2, 4
The points of intersection are 3xy = 2
A
1  1  P 1, 2
P  , 2 and Q  , 4 .
3  6  3
O x
2. Draw a vertical strip AB in the region
which starts from the rectangular hyper-
bola 3xy = 2 and terminates on the line
12x + y = 6. Fig. 9.150
2
Limits of y : y = to y = 6 – 12x
3x
1 1
Limits of x : x = to x =
6 3
9.7 Area as Double Integral 9.151

1
2x
x
6 3x
1
= ∫ 13 y
6 12 x
2 dx
6 3x

1
2
= ∫ 13 6 12 x dx
6 3x
1
2 3
= 2
log x
3 1
6

1 1 2 1 1
−6 − log − log
9 36 3 3 6
1 2
= − log 2
2 3

Example 12 2 2

Find the area bounded by the hypocycloid x y


3 3
+ 1.
a b
Solution
1. The hypocycloid is symmetric in all the y
quadrants.
Q (0, b) 2 2
Total area = 4 (area in the first quadrant) x 3 y 3
a =
2. Draw a vertical strip AB parallel to y-axis b
B
in the region which lies in the first quad-
rant. AB starts from x-axis and terminates P
2 2
O A (a x
x 3 y 3
on the curve + = 1.
a b
3
2 2
x 3
Limits of y : y = 0 to
a
Fig. 9.151
Limits of x : x = 0 to x = a
2 2
x
a b 1−
x
3
 2 2
x
a b1
4∫ a

3
2
2
x 3
dx
a
9.152 Chapter 9 Multiple Integrals

Putting x = a cos3 t, dx = 3a cos2 t (–sin t) dt


π
When x = 0, t =
2
When x = a, t = 0
3
0
A = 4∫π b(1 − cos 2 t ) 2 ( −3a coss 2 t sin t)
t dt
2
π
= 12ab∫ 2 sin
n 4 t cos 2 t dt
0

3 1 1 π
= 12ab ⋅ ⋅ ⋅ [ Using reduction for
ormu
mula ]
6 4 2 2
3
= π ab
8

EXERCISE 9.8

1. Find the area bounded by y-axis, the line y = 2x and the line y = 4.
[Ans. : 4]

2. Find the area bounded by the lines y = 2 + x, y = 2 – x and x = 5.


[Ans. : 25]
3. Find the area bounded by the parabola y2 + x = 0, and the line y = x + 2.
 9
 Ans. : 2 
 
4. Find the area bounded by the parabola x = y – y 2 and the line x + y = 0.
 4
 Ans. : 3 
 

5. Find the area bounded by the curves y 2 = 4x and 2x – 3y + 4 = 0.


 1
 Ans. : 3 
 
6. Find the area bounded by the parabola y = x2 – 3x and the line y = 2x.
 125 
 Ans. : 6 
 
9.7 Area as Double Integral 9.153

7. Find the area bounded by the parabolas y2 = x, x2 = –8y.


 8
 Ans. : 3 
 
x2
8. Find the area bounded by the parabolas y = ax2 and y = 1 − , where
a
a > 0.
 4 a 
 Ans. : 
 3 a + 1 
2

 a + x
9. Find the area of the loop of the curve y 2 = x 2 
 a − x 

 2 π 
 Ans. : 2a  − 1 
 4 
10. Find the area of one of the loops of x4 + y4 = 2a2xy.
 π a2 
 Ans. : 
 4 

11. Find the area enclosed by the curve 9xy = 4 and the line 2x + y = 2.

 1 4 
 Ans. : 3 − 9 log 2
 
12. Find the area of the smaller region bounded by the circle x2 + y2 = 9
and a straight line x = 3 – y.
  2π 3
 Ans. : 4  − 
  3 2  

13. Find the area bounded by the x-axis, circle x2 + y2 = 16 and the line y = x.
[Ans. : 2p ]
14. Find the area bounded between the curves y = 3x2 – x – 3 and y = – 2x2
+ 4x + 7.
 45 
 Ans. : 2 
 
2 2 2
15. Find the area bounded by the asteroid (x) 3 + (y ) 3 = (a) 3 .
 3 2
 Ans. : 8 π a 
 
9.154 Chapter 9 Multiple Integrals

9.7.2 Area in Polar Coordinates p


q = q = q2
2
The area A bounded by the curves r = r1 (q ), r = r2 R
(q ) and the lines q = q1 and q = q2 is
q = q1
θ2 r2 (θ )
A=∫ ∫ r dr dθ S
Q
θ1 r1 (θ ) r = r2(q )
q2 r = r1(q )
q1 P
Note: Consider the symmetricity of the region O q =0
while calculating the area.
Fig. 9.152

Example 1
Find the area between the circles r = 2 sinq and r = 4 sinq.
Solution
π
1. The region is symmetric about the line θ = .
2
Total area = 2 (area in the first quadrant)
q= p
2. Draw an elementary radius vector OAB 2
from the origin in the region which lies
in the first quadrant. OAB enters in the P
region from the circle r = 2sinq and B
terminates on at the circle r = 4 sinq. r = 4 sinq
Limits of r : r = 2sinq to r = 4sinq Q A
π
Limits of θ : θ = 0 to θ =
2
π
4 sin θ
A = 2∫ 2 ∫ r dr dθ r = 2 sinq
0 2 sin θ
q
π 4 sin θ
r2 O
= 2∫ 2 dθ q =0
0 2 2 sin θ
π
Fig. 9.153
= ∫ (16 sin 2 θ − 4 sin
2
sin 2 θ ) dθ
0
π
= ∫ 2 12 sin 2 θ dθ
0
π
= ∫ 2 6(1 − cos 2θ ) dθ
0
π
ssiin 2θ 2
=6θ− dθ
2 0

 π sin
sin π − ssin 0 
= 6 − 
2 2
= 3π
9.7 Area as Double Integral 9.155

Example 2
Use double integral in polar form to find the area enclosed by the three
petalled rose r = sin 3q. [Winter 2015]
Solution
1. This curve consists of three similar loops.
Total area = 3 (area of the loop in the first quadrant)
2. When r = 0, sin 3q = 0
3q = 0, p, 2p, 3p, ...
p 2p
q = 0, , , p , ...
3 3
Since, in the first quadrant,
p
r = 0 at q = 0,
3 Fig. 9.154
p
loop exists between q = 0 and q = .
3
3. Draw an elementary radius vector OA from the origin in the loop which lies in the
first quadrant. OA starts from the origin and terminates on the curve = sin q.
Limits of r : r = 0 to r = sin 3q
p
Limits of q : q = 0 to q =
3
p
sin 3q
A = 3Ú 3 Ú r dr dq
0 0
p sin 3q
r2
= 3Ú 3 dq
0 2
0
p
3

= sin 2 3q dq
3
0
p
3 3 Ê 1 - coss 6q ˆ
2 Ú0 ÁË
= ˜¯ dq
2
p
3 sin 6q 3
= q-
4 6 0
3Êp 1 ˆ
= Á - sin 2p ˜
Ë
4 3 6 ¯
3Êpˆ
=
4 ÁË 3 ˜¯
p
=
4
9.156 Chapter 9 Multiple Integrals

Example 3
Find the area of the crescent bounded by the circles r = 3 and
r = 2cosq.
Solution
p
1. The points of intersection of r = 3 q=
2
and r = 2cosq are obtained as r = √3 q=
p
6
3 = 2 cos θ P
r = 2 cosq
3 A B
cos θ =
2 q
π O q =0
θ=±
6

π
Hence, θ = at P. p
6 q =−
6
2. The region is symmetric about the
initial line, q = 0. Fig. 9.155
Area of the crescent = 2 (area above the initial line, q = 0)
3. Draw an elementary radius vector OAB from the origin in the region above the
initial line. OAB enters in the region from the circle r = 3 and terminates on at
the circle r = 2 cosq.
Limits of r : r = 3 to r = 2 cos θ
π
Limits of θ : θ = 0 to θ =
6
π
2 cos θ
A = 2∫ 6 ∫ r dr dθ
0 3
π 2 2 cos θ
r
= 2∫ 6 dθ
0 2 3
π
= ∫ 6 (4 cos 2 θ − 3)dθ
0
π
= ∫ 6 [2(1 + coss 2θ ) − 3]
3]dθ
0
π
n 2θ
siin 6
= 2 −θ
2 0

π π
= sin
sin −
3 6
3 π
= −
2 6
9.7 Area as Double Integral 9.157

Example 4
Find the area which lies inside the circle r = 3a cosq and outside the
cardioid r = a (1 + cosq ).
p p
q= q=
Solution 2 3
1. The points of intersection of the circle r = 3a
a cosq
R
r = 3a cos q and the cardioid
r = a (1 + cos q ) are obtained as B

3a cos θ = a (1 + cos
cos θ ) A
q Q
1
cos θ = O P q =0
2 r = a (1 + cosq )
π
θ=±
3
π p
Hence, θ = at R. q =−
3
3
2. The region is symmetric about the ini-
Fig. 9.156
tial line q = 0.
Total area = 2 (area above the initial line)
3. Draw an elementary radius vector OAB from the origin in the region above the ini-
tial line. OAB enters in the region from the cardioid r = a (1 + cosq ) and terminates
on the circle r = 3a cosq.
Limits of r : r = a (1 + cosq ) to r = 3a cosq
π
Limits of θ : θ = 0 to θ =
3
p
3 a cosq
A = 2Ú 3 Ú r dr dq
0 a (1+ cosq )

p 3 a cosq
r2
= 2Ú 3 dq
0 2
a (1+ cosq )
p
= Ú 3 ÈÎ9a 2 cos2 q - a 2 (1 + cos q )2 ˘˚ dq
0
p
= Ú 3 ÈÎ8a 2 cos2 q - a 2 - 2
2aa 2 cos q ˘˚ dq
0
p
= a 2 Ú 3 [ 4(1 + cos 2q ) - 1 - 2 cos q ] dq
0
p
4 sin 2q 3
= a 3q +
2
- 2 sin q
2 0
9.158 Chapter 9 Multiple Integrals

Ê p 2p pˆ
= a 2 Á 3 + 2 sin - 2 sin ˜
Ë 3 3 3¯
= p a2

Example 5
Find the area lying inside the circle r = a sin q and outside the cardioid
r = a (1 – cos q). [Summer 2016]
Solution
1. The points of intersection of circle r = a sinq and the cardioid r = a (1 – cosq) are
obtained as
a sinq = a(1 – cosq)
q q q
2 sin cos = 2 sin 2
2 2 2
q q
sin = 0, tan = 1
2 2
q q p
= 0, =
2 2 4
p
q = 0, q = Fig. 9.157
2
p
Hence, q = 0 at origin and q = at P.
2
2. Draw an elementary radius vector OAB from origin in the region. OAB enters
in the region from the cardioid r = a(1 – cos q) and terminates on the circle
r = a sinq.
Limit of r: r = a(1 – cos q) to r = a sin q
p
Limit of q: q = 0 to q =
2
p
a sin q
A=Ú2Ú r dr dq
0 a (1- cosq )

p a sin q
r2
=Ú2 dq
0 2
a (1- cosq )
p
1 2È 2 2
a sin q - a 2 (1 - cos q )2 ˘˚ dq
2 Ú0 Î
=
p
a2 Èsin 2 q - (1 - 2 cos q + cos2 q )˘ dq
=
2 Ú
0
2
Î ˚
9.7 Area as Double Integral 9.159

p
a2
= Ú02 ÈÎsin q - cos2 q + 2 cos q - 1˘˚ dq
2
2
a2 È 2 ˘
p p p
= Í Ú (- cos 2q )dq + 2 Ú 2 cos q dq - Ú 2 dq ˙
2 Î 0 0 0 ˚
È p
p p ˘
a 2 Í sin 2q 2 2 - q 2˙
= - + 2 sin q
2 ÍÎ 2 0
0 0 ˙
˚
a2 È 1 p p˘
= ÍÎ- 2 (sin p - sin q ) + 2(sin 2 - sin 0) - 2 ˙˚
2
a2È p˘
= Í2 - 2 ˙
2 Î ˚
Ê p ˆ
= a2 Á1 - ˜
Ë 4¯

Example 6
Find the area common to the cardioids r = a(1 + cosq ) and
r = a(1 – cos q ).

Solution
1. The points of intersection of the p
cardioids q=
2
a (1 + cos θ ) and r = a (1 − cos
r = a( cos θ )
r = a (1 − cosq ) r = a (1 + cosq )
are obtained as P
A
a (1 + cosθ ) = a(
a (1 − cosθ )
cosθ = 0
π O
θ=± q =0
2

π
Hence, θ = at P.
2
2. The region is symmetric in all the
quadrants
Total area = 4 (area in the first Fig. 9.158
quadrant)
3. Draw an elementary radius vector OA from the origin in the region which lies
in the first quadrant. OA starts from the origin and terminates on the cardioid
r = a(1 – cos q ).
9.160 Chapter 9 Multiple Integrals

Limits of r : r = 0 to r = a (1 − coss θ )
π
Limits of θ : θ = 0 to θ =
2
p
a (1- cosq )
A = 4Ú 2 Ú r dr dq
0 0
p a (1- cosq )
r2
= 4Ú 2 dq
0 2
0
p
= 2 Ú 2 a 2 (1 - cos q )2 dq
0
p
= 2 a 2 Ú 2 (1 - 2 cos q + cos2 q )dq
0
p
Ê 1 + cos 2q ˆ
= 2 a 2 Ú 2 Á 1 - 2 cos q + ˜¯ dq
0 Ë 2
π
3 sin 2θ 2
= 2a θ − 2 sin θ +
2

2 4 0

 3π 
= 2a 2  − 2
 4 

Example 7
Find the area inside the cardioid r = 3(1 + cos θ ) and outside the
3
parabola r = .
1 + cos θ
Solution
1. The points of intersection of the p
q=
2
cardioid r = 3(1 + cos θ ) and the
3 P
parabola r = are ob- B
1 + cos θ
tained as A
r = 3 (1 + cosq )
3
cos θ ) =
3(1 + cos
(1 + cos θ ) O q =0

(1 + cos θ ) 2 = 1
cos θ = 0 3
r=
π 1 + cosq
θ=±
2
π
Hence, θ = at P.
2 Fig. 9.159
9.7 Area as Double Integral 9.161

2. The region is symmetric about the initial line q = 0.


Total area = 2 (area above the initial line)
3. Draw an elementary radius vector OAB from the origin in the region above the
3
initial line q = 0. OAB enters in the region from the parabola r = and
terminates on the cardioid r = 3(1 + cos θ ). 1 + cos θ
3
Limits of r : r = to r = 3(1 + ccos q )
1 + cos q
π
Limits of θ : θ = 0 to θ =
2
p
3(1+ cosq )
A = 2Ú 2 Ú 3 r dr dq
0
1+ cosq
p 3(1+ cosq )
r2
= 2Ú 2 dq
0 2 3
1+ cosq

p
È 1 ˘
= Ú 2 9 Í(1 + cos q )2 - ˙ dq
0
Î (1 + cos q ) ˚
2

p
È 1 + cos 2q 1 ˘
= 9 Ú 2 Í1 + 2 cos q + - 2 ˙ dq
0 2 Ê 2 qˆ ˙
Í
ÍÎ ÁË 2 cos 2 ˜¯ ˙
˚
p
È3 cos 2q 1 Ê qˆ q˘
= 9 Ú 2 Í + 2 cos q + - Á 1 + tan 2 ˜ sec 2 ˙ dq
0 2
Î 2 4Ë 2¯ 2˚

p
È3 cos 2q 1 q 1 q Ê1 q ˆ˘
= 9 Ú 2 Í + 2 cos q + - secc 2 - ◊ tan 2 Á sec 2 ˜ ˙ dq
0 2
Î 2 4 2 2 2Ë2 2¯˚
p
q 3 2
tan
3q sin 2q 1 q 1 2
=9 + 2 sin q + - ◊ 2 tan -
2 4 4 2 2 3
0

È
Í∵ Ú [ f (q )]n f ¢(q )dq =
[ f (q )]n+1 ˘˙
Í n +1 ˙
Î ˚
Ê 3p p sin p 1 p 1 3 pˆ
= 9Á + 2 sin + - tan n - tan ˜
Ë 4 2 4 2 4 6 4¯
Ê 3p 4 ˆ
= 9Á +
Ë 4 3 ˜¯
9.162 Chapter 9 Multiple Integrals

Example 8
Find the area common to both the circles r = cos q and r = sin q.
[Winter 2013]
Solution
p
q=
1. So the point of intersection of the circles 2
r = cos q and r = sin q is obtained as r = sin q
p
q=
cos q = sin q P 4
B
tan q = 1
p A
q= r = cos q
4
O q =0
p
Hence, q = is the point of intersection
4
2. The point of intersection divides the region
into two subregions OAP and OBP. Fig. 9.160
3. Draw an elementary radius in each subre-
gion.
(i) In subregion OAP, radius vector OA starts from the origin and terminates on
the circle r = sin q.
Limit of r: r = 0 to r = sin q
p
Limit of q: q = 0 to q =
4
(ii) In the subregion OBP, the radius vector OB starts from the origin and terminates
on the circle r = cos q.
Limit of r: r = 0 to r = cos q
p p
Limit of q: q = to q =
4 2
p p
sin q cosq
A=
Ú Ú
0
4
0
r dr dq + Ú Ú 2
0 0
r dr dq

p sin q p cosq
r2 r2
=
Ú
0
4
2
0
dq + Ú p
4
2
2
0
dq

1È ˘
p p
= Í
2Í Ú
0
4
sin 2 q dq + Ú p
2
cos2 q dq ˙
˙
Î 4 ˚

1È Ê 1 + cos 2q ˆ ˘
p p
Ê 1 - cos 2q ˆ
= Í
2Í Ú
0
4
ÁË 2 ˜¯ dq + Ú p
2
ÁË 2 ˜¯ dq ˙
˙
Î 4 ˚
9.7 Area as Double Integral 9.163

1È ˘
p p
= Í
4Í Ú
0
4
(1 - cos 2q ) dq + Ú p
2
(1 + cos 2q ) dq ˙
˙
Î 4 ˚
È p p ˘
1Í sin 2q 4 sin 2q 2 ˙
= Íq - +q+
4 2 2 p ˙
ÍÎ 0
4˙˚
1 ÈÊ p 1 ˆ Ê p p 1 ˆ ˘
= Í - + - - ˙
4 ÎÁË 4 2 ˜¯ ÁË 2 4 2 ˜¯ ˚

1Êp ˆ
= -1
4 ÁË 2 ˜¯
p 1
= -
8 4

Example 9
Find the area common to the circles r = cos θ and r = 3 sin θ .

Solution
1. The point of intersection of the circles r = cos θ and r = 3 sin θ is obtained as
n θ = ccos
3 sin os θ p
q=
1 2
tan θ =
3
π
θ=
6
p
q=
π 6
Hence, θ = at P.
6
B P
2. Divide the region OAPBO into two r = √3 sin q
subregions OAP and OBP. Draw A r = cos q
an elementary radius vector in each
O q =0
subregion.
(i) In subregion OAP, radius vec-
tor OA starts from the origin
and terminates on the circle Fig. 9.161
r = 3 sin θ.
Limits of r : r = 0 to r = 3 sin θ
π
Limits of θ : θ = 0 to θ =
6
9.164 Chapter 9 Multiple Integrals

(ii) In the subregion OBP, the radius vector OB starts from the origin and termi-
nates on the circle r = cos q.
Limits of r : r = 0 to r = cos q
p p
Limits of q : q = to q =
6 2
p p
3 sin q cosq
A=Ú 6
Ú r dr dq + Ú 2
p 0 Ú r dr dq
0 0
6

p 3 sinn q p cosq
r2 r2
=Ú 6 dq + Ú p
2 dq
0 2 2
0 6 0

p p
1 6 1
2 Ú0
= 3 sin 2 q dq + Ú p2 cos2 q dq
2
6

p p
3 Ê 1 - cos 2q ˆ 1 2 Ê 1 + ccos
os 2q ˆ
2 Ú ÁË ˜¯ dq + 2 Ú p ÁË
= 6
˜¯ dq
02 2
6
p p
3 n 2q
sin 6 1 sin 2q 2
= q- + q+
4 2 0 4 2 p
6

3Êp 1 pˆ 1Êp p 1 1 pˆ
= - sin ˜ + Á - + sin p - sin ˜
4 ÁË 6 2 3¯ 4 Ë 2 6 2 2 3¯
5p 3
= -
24 4

Example 10
Find the area common to the circle r = a and the cardioid
r = a(1 + cos q ).
Solution
1. The points of intersection of the circle r = a and the cardioid r = a (1 + cos q ) are
obtained as
a = a(1 + cos q )
cos q = 0
p
q=±
2
π
Hence, θ = at Q.
2
9.7 Area as Double Integral 9.165

2. The region is symmetric about the initial line q = 0.


Total area = 2 (area above the initial line)
3. Divide the region OPQO above the initial line into two subregions OPQ and OBQ.
Draw an elementary radius vector in each subregion.
(i) In the subregion OPQ the radius vector OA starts from the origin and termi-
nates on the circle r = a.
Limits of r r
p
q
2
Q

B A
r a
r (1 + cos q )

O P q =0

Fig. 9.162

Limits of
2
(ii) In the subregion OBQ, radius vector OB starts from the origin and terminates
on the cardioid r = a (1 + cos q ).
Limits of + cos )

Limits of
2
Ê p
1+ osq ) ˆ
dr d
2

Ê p a a (1+ cosq ) ˆ

0
p
p 2
1+ o
2
9.166 Chapter 9 Multiple Integrals

p
p 1+ 2q
dq
2
2
p
3 s 2q
2 2 4 p
2

p a2 a2 p p a2
p in + (sin 2
2 2 2 2 4
2 5p
2
4

Example 11
Find the area between the curve q = p
2
r = a (secq + cosq ) and its asymptote
r = a sec q.
r = a (sec q + cos q )
Solution
B
1. The region is symmetric about the ini- r= sec q
tial line q = 0 A
Total area = 2(area above the initial line)
O q =0
2. Draw an elementary radius vector OAB
in the region above the initial line.
OAB enters in the region from the line
r = a sec q and terminates on the curve
r = a (secq + cosq ).
Limits of r : r = a sec q to
r = a (sec q + cos q )
Fig. 9.163
Limits of q : q = 0 to
2
p
a (sec +
A r dr dq
sec q

p a (sec +co
r2
2Ú 2 dq
2
a sec q
p
- ec q q
p
2
9.7 Area as Double Integral 9.167

È1 p p ˘
= a 2 Í ◊ + 2q 02 ˙ [ Using reduction fformula
ormula ]
ÍÎ 2 2 ˙˚
È p 2p ˘
= a2 Í + ˙
Î4 2 ˚
5p 2
= a
4

Example 12
Find the area of the loop of the curve x 4 + y 4 = 8 xy.
Solution
1. The equation of the curve in polar form is r 4 (cos 4 θ + si
sinn 4 θ ) = 8r 2 coss θ sin
sin θ
8 coss θ sin
sin θ p
r2 = q=
coss θ + sin 4 θ
4 2
q =
p
A 4
2. Draw an elementary radius vector
OA from the origin in the region 8 cos q sin q
r2=
which lies in the first quadrant. OA cos4 q + sin4 q
starts from the origin and terminates
O q =0
8 coss θ sin
sin θ
on the curve r 2 = .
coss θ + sin 4 θ
4

Limits of
8 coss θ sin
sin θ
r : r = 0 to r =
coss θ + ssin
4
in 4 θ
π Fig. 9.164
Limits of θ : θ = 0 to θ =
2

p 8 cossq sin q
cos4 q + sin 4 q
A=Ú 2
Ú r dr dq
0 0
8 cossq sin q
p 2
r coss4 q + sin 4 q
=Ú 2 dq
0 2
0
p
1 2 Ê 8 coss q sin q ˆ
2 Ú0 ÁË coss4 q + sin 4 q ˜¯
= dq

π
 tan sec 2 θ 
n θ sec
= 4∫ 2  dθ
0  1 + tan 4 θ  
9.168 Chapter 9 Multiple Integrals

n 2 θ = t , 2 tan
Putting tan tan θ ssec
ec 2 θ dθ = dt
When θ = 0, t = 0
π
When θ= ,t → ∞
2
• dt
A = 2Ú
0 1 + t2

= 2 tan -1 t
0

Êpˆ
= 2Á ˜
Ë 2¯
=p

EXERCISE 9.9

1. Find the area common to the circles r = a and r = 2acos q.

 
2 2π 3
 Ans. : a  − 
  3 2  

2. Find the area of the crescent bounded by the circles


r = 2 and r = 2 cos θ.
Ans. :1

3. Find the area which lies inside the cardioid r = 2a (1 + cos q ) and
2a
outside the parabola r = .
1 + cos θ
 16a 2 
 Ans. : 3π a 2
+ 
 3 

4. Find the area bounded between the circles r = 2a sin q, r = 2b sin q (b > a).
 Ans. : π(b 2 − a 2 )

5. Find the area outside the circle r = a and inside the cardioid r = a(1 + cos q ).
 a2 
 Ans. : (π + 8)
 4 
9.8 Volume as Triple Integral 9.169

9.8 VOLUME AS TRIPLE INTEGRAL

Volume, V of a solid contained in the region V is given as

V = ÚÚÚ dx dy dz
V
In cylindrical coordinates,
V = ÚÚÚ r dr dq dz
V
In spherical coordinates,

V = ÚÚÚ r 2 sinq
q dr dq df
V
Note: Consider symmetricity of the region while calculating the volume.

Example 1
Find the volume of the tetrahedron bounded by the plane x + y + z = 2
and the planes x = 0, y = 0, z = 0. [Winter 2014]

Solution
1. Draw an elementary volume AB parallel to z-axis in the region. AB starts from
xy-plane and terminates on the plane x + y + z = 2.
Limits of z: z = 0 to z = 2 – x – y
2. Projection of the plane x + y + z = 2 in xy-plane is the DOPQ. Equation of the line
PQ is obtained by putting z = 0 in x + y + z = 2.
3. Draw a vertical strip A¢B¢ in the region OPQ in the xy-plane. A¢B¢ starts from the
x-axis and terminates on the line x + y = 2.

Fig. 9.165

Limits of y : y = 0 to y=2–x
Limits of x : x = 0 to x=2
9.170 Chapter 9 Multiple Integrals

2 2- x 2- x - y
V =Ú Ú Ú0 dz dy dx
0 0
2 2- x 2- x - y
=Ú Ú z0 dy dx
0 0
2 2- x

0 0 Ú [(2 - x) - y ] dy dx
2- x
2 y2
=Ú (2 - x ) y - dx
0 2
0

2È ( 2 - x )2 ˘
= Ú Í ( 2 - x )2 - ˙ dx
0
ÍÎ 2 ˙˚
2 ( 2 - x )2
=Ú dx
0 2
2
1 (2 - x )3
=
2 -3
0
1 2
= - (2 - x )3
6 0


= - Î0 - (2)3 ˘˚
6
4
=
3

Example 2
Find the volume of the tetrahedron bounded by the planes x = 0, y = 0,
x y z
z = 0 and + + = 1.
a b c
Solution
1. Draw an elementary volume AB parallel to z-axis in the region. AB starts from
x y z
xy-plane and terminates on the plane + + = 1.
a b c
Ê x yˆ
Limits of z : z = 0 to z = c Á 1 - - ˜ .
Ë a b¯
Ê x yˆ
c Á 1- - ˜
V =Ú Ë a b¯ dz dy dx
0
9.8 Volume as Triple Integral 9.171

x y z
2. Projection of the plane + + 1 in xy-plane is the ∆OPQ Equation of the line
a b c
x y z x
PQ is obtained by putting z = 0 in + + .
a b c a b
3. Draw a vertical strip A¢B¢ in the region OPQ. DOPQ starts from the x-axis and
x y
terminates on the line + = 1.
a b
x
Limits of y y
a
Limits of x x
z

R (0, 0, c)
y

y (0, b)
x z
+ + 1
a b c B
B y
x
a + b 1

A (0, b, 0) y O A P (a, 0) x

P (a, 0, 0)

Fig. 9.166

x x y
a b 1- c 1- -
y x
0
x y
a b 1
x c 1- -
Ú Ú a a b
x
x
a b x y
= -
a b
x
b 1
a x y2 a
y dx
a 2b
0
9.172 Chapter 9 Multiple Integrals

2 2˘
a x b2 x
= - - 1- x
a 2b a
a
3
x
- 1-
bc a
=
2 3 0
abc
=
6

Example 3
Find the volume cut off from the sphere z2 a 2 by the cone
z2
Solution
1. The region is bounded by the cone and sphere. Putting x = r cos q, y = r sin q, z = z,
(i) equation of the sphere z2 a 2 reduces to a2 .
(ii) equation of the cone z 2 reduces to 2 2
, .
2. Draw an elementary volume AB parallel to z-axis in the region. AB starts from the
cone r = z and terminates on the sphere a2 .

Limits of a2 r2
a2 r 2
V r dq
r

B
r + z2 a2 q p
2
r z A
a
r
A √2
q 0
O y

Fig. 9.167
9.8 Volume as Triple Integral 9.173

3. Projection of the region in rq -plane is the curve obtained by the intersection of the
sphere r 2 + z 2 = a 2 and the cone r = z as
r 2 + r 2 = a2
a
r=
2
a
which is a circle with centre at the origin and radius .
2
4. The region (circle) is symmetric in all the quadrants. Draw an elementary radius
vector OA¢ in the first quadrant of the region. OA¢ starts from the origin and termi-
a
nates on the circle r = .
2
a
Limits of r : r = 0 to r =
2
π
Limits of θ : θ = 0 to θ = (first quadrant)
2
p a
a2 - r 2
V = 4Ú 2 Ú 2
Úr r dz dr dq
0 0
p a
a2 - r 2
= 4Ú 2 Ú 2 r zr dr dq
0 0

( )
p a
È ˘
= 4Ú 2 Ú 2
ÍÎr a 2 - r 2 - r ˙ dr
0 0 ˚
p a
È 1 2 2( 2˘
= 4Ú 2 dq Ú 2
Í - 2 a - r -2r ) - r ˙ dr
0 0 Î ˚
a
3 2
p
1 È
2(a 2 - r 2 ) 2 r3 [ f (r )]n +1 ˘
Í∵ Ú [ f (r )] f ¢(
n
=4q 2 - ◊ -
¢(r )dr = , n π -1˙
0 2 3 0 ÍÎ 3 n +1 ˙˚
È Ï 3 ¸ ˘
p Í 1 ÔÊ 2 a 2 ˆ 2 Ô a 3 ˙
= 4 ◊ Í - ÌÁ a - ˜ - a 3 ˝ - ˙
2 Í 3 ÔË 2¯ Ô 6 2˙
Î Ó ˛ ˚
È a 3
a ˘
3
= 2p Í - + ˙
ÍÎ 3 2 3 ˙˚
p a3
= (- 2 + 2)
3
9.174 Chapter 9 Multiple Integrals

Example 4
Find the volume bounded by the cone x2 + y2 = z2 and the paraboloid
x2 + y2 = z.
Solution
1. The region is bounded by the cone and the paraboloid. Putting x = r cos q,
y = r sin q, z = z,
(i) equation of the cone x2 + y2 = z2 reduces to r2 = z2, r = z.
(ii) equation of the paraboloid x2 + y2 = z reduces to r2 = z.
2. Draw an elementary volume AB in the region. AB starts from the paraboloid r2 = z
and terminates on the cone r = z.
2
Limits of z z r
r
V r dq
r

r=z
= p
2
B
r2 = z
A′
A r=1
=0
O y O

Fig. 9.168

3. Projection of the region in rq -plane is the curve obtained by the intersection of the cone
r = z and the paraboloid r 2 = z as
r = r 2, r = 1, which is a circle with centre at the origin and radius 1.
4. The region (circle) is symmetric in all the quadrants. Draw an elementary radius
vector OA¢ in the first quadrant of the region. OA¢ starts from the origin and termi-
nates on the circle r = 1.
Limits of r r
p
Limits of q q (first quadrant)
2
9.8 Volume as Triple Integral 9.175

p
1 r
V r dq
0 r
p
1 r
4Ú 2 Ú r z r 2 dr dq
p
1

p 1
r3 r4
0 3 4
0
p 1 1
.
2 3 4
p
6

Example 5
Find the volume of the solid bounded by the paraboloids y
and y
Solution
1. The region is bounded by the paraboloids. Putting si z z,
2
(i) equation of the paraboloid y reduces to z = r .
(ii) equation of the paraboloid y reduces to z = 4 – 3r2.
2. Draw an elementary volume AB in the region. AB starts from the paraboloid z = r2
and terminates on the paraboloid z = 4 – 3r2.
2
Limits of r2
4 - 3r 2
V r dq

z
q= p
2

B
z 4 3r 2 A
z r2 r 1
A =0
O y O

Fig. 9.169
9.176 Chapter 9 Multiple Integrals

3. Projection of the region in all the quadrants is the curve obtained by the intersection
of the paraboloids z = r2 and z = 4 – 3r2 as
r 2 = 4 - 3r 2
r2 = 1
r=1
which is a circle with centre at the origin and radius 1.
4. The region (circle) is symmetric in all the quadrants. Draw an elementary radius
vector OA¢ in the first quadrant of the region. OA¢ starts from the origin and termi-
nates on the circle r = 1.
Limits of r : r = 0 to r = 1
p
Limits of q : q = 0 to q =
2
p
1 4 - 3r 2
V = 4Ú 2 Ú rÚ dz dr dq
0 0 r2
p
1 4 - 3r 2
= 4Ú 2
Ú
r z r2 dr dq
0 0
p
1
= 4Ú 2 Ú r (4 - 3r 2 - r 2 )dr dq
0 0
p
1
= 4Ú 2 dq Ú (4r - 4r 3 )dr
0 0
p
1
= 4 q 02 2
2rr 2 - r 4
0
p
= 4◊
2
= 2p

Example 6
Find the volume of the cylinder x2 + y2 = 2ax intercepted between the
x 2 + y2
paraboloid z = and the xy-plane.
2a
Solution
1. The region is bounded by the cylinder, paraboloid and xy-plane. Putting x = r cos q,
y = r sin q, z = z
(i) equation of the cylinder x2 + y2 = 2ax reduces to
r2 = 2ar cos q, r = 2a cos q.
x 2 + y2 r2
(ii) equation of the paraboloid z = reduces to z = .
2a 2a
9.8 Volume as Triple Integral 9.177

2. Draw an elementary volume AB parallel to z-axis in the region. AB starts from


r2
xy-plane and terminates on the paraboloid z
2a
r2
Limits of z : z = 0 to z
2a
r2
V r dq

3. Projection of the region in rq -plane is the curve obtained by the intersection of the
cylinder r = 2a cos q and rq -plane (z = 0) as
r = 2a cos q, which is circle with centre (a, 0) and radius a.
4. The region (circle) is symmetric about the initial line q = 0. Draw an elementary
radius vector OA¢ in the region above initial line q = 0. OA¢ starts from the origin
and terminates on the circle r = 2a cos q.
Limits of r : r = 0 to r = 2a cos q

Limits of q : q = 0 to (first quadrant)


2
z

q= p
r2 2
z=
2a
A
B
r = 2 cos q
y O =0

r = 2a cos q

Fig. 9.170

p r2
2 a co q
V r dq
0

p r2
2 a cosq
2Ú 2
Ú rz 2 a dr dq
0
0

r2 ˆ
p
2 a cosq
2Ú 2 Ú r dr dq
0 2a
p 2a sq
1 2 r
aÚ 4
dq
0
9.178 Chapter 9 Multiple Integrals

p
= 4 a 3 Ú 2 cos4 q dq
0

3 1 p
= 4a3 ◊ ◊ ◊
4 2 2
[ Using reduction formula ]
3 3p
=a ◊
4
3
= p a3
4

Example 7
Find the volume bounded by the cylinder x2 + y2 = 4 and the planes
y + z = 3, z = 0. [Summer 2014]
Solution
1. The region is bounded by the cylinder x2 + y2 = 4 and the planes y + z = 3, z = 0.
2. Draw an elementary volume AB parallel to z-axis in the region. AB starts on
xy-plane and terminates on the plane y + z = 3.
Limits of z: z = 0 to z = 3 – y
3. Projection of the region in xy-plane is the circle x2 + y2 = 4.

Fig. 9.171

4. The region in xy-plane is symmetric in all the quadrants.


5. Draw a horizontal strip A¢B¢ in the first quadrant of the circle. A¢B¢ starts from y-axis
and terminates on the circle x2 + y2 = 4.
Limits of x : x = 0 to x = 4 - y2
Limits of y : y = 0 to y = 2
9.8 Volume as Triple Integral 9.179

2 4 - y2 3- y
V = 4Ú Ú Ú0 dz dx dy
0 0

2 4 - y2 3- y
= 4Ú Ú z0 dx dy
0 0

2 4 - y2
= 4Ú Ú (3 - y) dx dy
0 0
2 4 - y2
= 4Ú (3 - y) x 0 dy
0
2
= 4Ú (3 - y) 4 - y 2 dy
0

= 4 ÈÍ Ú 3 4 - y 2 dy - Ú y 4 - y 2 dy ˘˙
2 2

Î 0 0 ˚
È 2 3

y 4 y 1
= 4 Í3 4 - y 2 + sin -1 + (4 - y 2 ) 2 ˙
Í 2 2 20 3 ˙
ÍÎ 0˙
˚
È 3˘
Í Ê -1 1 ˆ2˙
= 4 Í3 Á 2 sin 1 + (4)˜ ˙
Ë 3 ¯
ÎÍ ˙˚
È 8˘
= 4 Í3p + ˙
Î 3˚
4
= [ 9p + 8]
3

Example 8
Find the volume bounded by the cylinders x2 + y2 = 2ax and z2 = 2ax.
Solution
1. The region is bounded by the circular and parabolic cylinders. Putting x = r cos q,
y = r sin q, z = z,
(i) equation of the cylinder x2 + y2 = 2ax reduces to
r2 = 2ar cos q, r = 2a cos q.
(ii) equation of the cylinder z2 = 2ax reduces to z2 = 2ar cos q.
2. The region is bounded by the cylinder z2 = 2ar cos q.
Limits of z : z = - 2 ar cos q to z = 2 a cos q
arr cos
2 ar cosq
V = ÚÚ Ú r dz dr dq
- 2 ar cosq

3. Projection of the region in rq -plane is the circle r = 2a cos q.


9.180 Chapter 9 Multiple Integrals

4. The region in rq -plane (circle) is symmetric about the initial line q = 0. Draw an
elementary radius vector OA in the region above the initial line q = 0. OA starts from
the origin and terminates on the circle
p
r = 2a cos q. q =
2
Limits of r : r = 0 to r = 2a cosq
p
Limits of q : q = 0 to q = (first
quadrant) 2
A
p r = 2a cos q
2 a cosq 2 ar cosq
V = 2Ú 2
Ú Ú r dz dr dq
0 0 - 2 ar cosq
O q =0
p
2 a cosq 2 ar cosq
= 2Ú 2
Ú z - 2ar
2 cosq
r dr dq
0 0
p 1
ossq
2 a co
= 2Ú 2 Ú 2 ar cos q ) 2 r dr dq
2(2ar Fig. 9.172
0 0

2 a cosq
5
p 1 1 2
2r
= 4Ú 2 (2 a ) 2 (cos q ) 2
0 5 0
p 1 1 5
4

= 2 (2 a ) 2 (cos q ) 2 2(2 a cos
cos q ) 2 dq
0

p
64 a 3
=
5 Ú0
2 cos3 q dq

64 a 3 2
= ◊ [ Using reduction fformula
ormula ]
5 3
128a 3
=
15

Example 9
Use triple integral to find the volume of the solid within the cylinder
x2 + y2 = 9 between the planes z = 1 and x + z = 1.
[Summer 2017, 2015]
Solution
1. The region is bounded below by the plane x + z = 1 and above by the plane z = 1.
Limits of z: z = 1 – x to z = 1
2. Draw an elementary volume AB parallel to z-axis in the region. AB starts from
the plane x + z = 1 and terminates on the plane z = 1.
3. Projection of the region in xy-plane is the right half of the circle x2 + y2 = 9.
9.8 Volume as Triple Integral 9.181

Fig. 9.173

4. The region in xy-plane is symmetric about x-axis. Draw a horizontal strip in A¢B¢ in
the region. A¢B¢ starts from y-axis and terminates on the circle x2 + y2 = 9.
Limits of x : x = 0 to x= 9 - y2
Limits of y : y = 0 to y = 3 (first quadrant)
3 9 - y2 1
V = 2Ú Ú Ú1- x dz dx dy
0 0

3 9 - y2 1
= 2Ú
0 Ú0
z 1- x dx dy

3 9 - y2
= 2Ú
0 Ú0
(1 - 1 + x ) dx dy

9 - y2
3 x2
= 2Ú dy
0 2
0
3
= Ú (9 - y 2 ) dy
0
3
y3
= 9y -
3
0
= 27 - 9
= 18

Example 10
Find the volume in the first octant bounded by the circular cylinder
x2 + y2 = 2 and planes z = x + y, y = x, z = 0, x = 0.
Solution
1. The region is bounded below by xy-plane (i.e. z = 0) and above by the plane z = x + y.
9.182 Chapter 9 Multiple Integrals

Limits of z : z = 0 to z = x + y
x+ y
V = ÚÚ Ú dz dy dx
0
2. Projection of the region in xy-plane is the region bounded by the circle x2 + y2 = 2,
the line y = x and x = 0.
3. The point of intersection of the circle y
x2 + y2 = 2 and the line y = x is obtained as y=x
Q B
x2 + x2 = 2
P(1, 1)
2 x2 = 2 x2 + y2 = 2
A
x = ±1 O x
\ y = ±1
The point of intersection is P (1, 1).
4. Draw a vertical strip AB parallel to y-
axis which starts from the line y = x and
terminates on the circle x2 + y2 = 2.
Fig. 9.174
Limits of y : y = x to y = 2 - x 2
Limits of x : x = 0 to x = 1
1 2 - x2 x+ y
V =Ú Ú Ú0 dz dy dx
0 x

1 2 - x2 x+ y
=Ú Ú z0 dy dx
0 x

1 2 - x2
=Ú Ú ( x + y)dy dx
0 x

2 - x2
1 y2
=Ú xy + dx
0 2
x
1È ˘

(
= Ú Í x 2 - x 2 - x + ( 2 - x 2 - x 2 )˙ dx
1
2 ˚
)
1È 1 ˘
= Ú Í-
0Î 2
(
2 - x 2 ( -2 x ) - 2 x + 1˙ dx
2
˚
)
1
3
1 2(2 -
= - .
x2 ) 2
-
2x
+x
3 È
Í∵ Ú [ f ( x )] f ¢( x )dx =
n [ f ( x )]
n +1 ˘
, n π -1˙
2 3 3 0 Î n +1 ˚

=-
1
(
3 1- 2
3
2
2
- +1
3 )
2 2
=
3
9.8 Volume as Triple Integral 9.183

Example 11
Find the volume of the wedge intercepted between the cylinder
x2 + y2 = 2ax and the planes z = mx, z = nx where n > m.
Solution
1. The region is bounded below by the plane y
z = mx and above by the plane z = nx.
Limits of z : z = mx to z = nx B
nx
y x 2 + y = 2ax
mx

2. Projection of the region in the xy-plane is P (2a, 0)


bounded by the circle x2 + y2 = 2ax. O A x
3. The region (circle) in xy-plane is symmetric
about x-axis. Draw a vertical strip AB
parallel to y-axis in the region above
x-axis. AB starts from the x-axis and
terminates on the circle x2 + y2 = 2ax.
2 Fig. 9.175
Limits of y : y = 0 to
Limits of x : x = 0 to x = 2a
2a 2 ax x 2 nx
y x
x=0 y 0 z mx

2a 2 ax - x 2 nx
2Ú Ú x

2a 2 ax - x 2
mx

2a 2 ax x 2 ˘
Ú x y dx

2a 2 ax x 2
Ú x y0 dx
2a
x x 2 dx
3
2a
x dx

Putting x = 2a cos2 q, dx = – 4a cosq sinq dq


p
When x = 0, cos q = 0, q
2
When x = 2a, cos q = 1, q = 0
3
0
os2 ) 2 c ( 4 a cos dq
2
9.184 Chapter 9 Multiple Integrals

p
6a3 co 4
in dq
1 3 1 p
Using r nf ]
6 4 2 2
a3

Example 12
A cylindrical hole of radius b is bored through a sphere of radius a. Find
the volume of the remaining solid.
Solution
Let the equation of the cylinder with radius b is x2 + y2 = b2 and equation of the sphere
with radius a is x2 + y2 + z2 = a2.
1. The region is bounded by the sphere and cylinder. Putting x = r cos q, y = r sin q,
z = z,
(i) equation of the cylinder x2 + y2 = b2 reduces to r2 = b2, r = b.
(ii) equation of the sphere x2 + y2 + z2 = a2 reduces to r2 + z2 = a2.
2. The region is symmetric in all the octants.
Volume = 8 (volume in the positive octant)
3. Draw an elementary volume AB in the positive octant of the region. AB starts from
xy-plane and terminates on the sphere r2 + z2 = a2.
2
Limits of z z r2
a2 r 2
V r dq

q p
2
r b
r 2 + z2 a

B B′
A′ r b
O
O y q =0

A r a

Fig. 9.176
9.8 Volume as Triple Integral 9.185

4. Projection of sphere r2 + z2 = a2 in rq -plane (z = 0) is r2 = a2, r = a. Projection of the


region in rq -plane is the region bounded by the circles r = b and r = a.
5. Draw an elementary radius vector OA¢B¢ in the region in the first quadrant. OA¢B¢
enters in the region from the circle r = b and terminates on the circle r = a.
Limits of r : r = b to r = a
p
Limits of q : q = 0 to q = (first quadrant)
2
p
a a2 - r 2
V = 8Ú 2 Ú
b Ú0
r dz dr dq
0

p
a a2 - r 2
= 8Ú 2 Ú r z dr dq
0 b 0
p
a
= 8 Ú 2 Ú r a 2 - r 2 d r dq
0 b
p
È a 1 ˘
= 8 Ú dq Í Ú -
0
2
Î b 2
( )
a 2 - r 2 ( -2r )˙ dr
˚
a
3
p
2(a 2
- r2 )2 È [ f (r )]n +1 ˘
= -4 q Í∵ Ú [ f (r )] f ¢(r ) dr =
2 n
˙
0 3 b Î n +1 ˚
È 3 ˘
p Í 2(a - b )
2 2 2
˙
= - 4 ◊ ◊ Í0 - ˙
2 Î 3 ˚
3
4p 2
= (a - b2 ) 2 .
3

Example 13 2 2 2
Ê xˆ 3 Ê yˆ 3 Ê zˆ 3
Find the volume bounded by the solid Á ˜ + Á ˜ + Á ˜ = 1.
Ë a¯ Ë b¯ Ë c¯
Solution
It is difficult to integrate this integral in Cartesian form. Therefore, we transform the
solid into a sphere using following change of variables.
Putting x = au3, y = bv3, z = cw3
2 2 2
Ê xˆ 3 Ê yˆ 3 Ê zˆ 3
ÁË a ˜¯ + ÁË b ˜¯ + ÁË c ˜¯ = 1 reduces to

u2 + v2 + w2 = 1, which is a sphere of radius 1 and centre at the origin.

and dx dy dz = |J | du dv dw
9.186 Chapter 9 Multiple Integrals

∂x ∂x ∂x
∂u ∂v ∂w
∂( x, y, z ) ∂y ∂y ∂y
where J= =
∂(u, v, w) ∂u ∂v ∂w
∂z ∂z ∂z
∂u ∂v ∂w

3au2 0 0
= 0 3bv 2
0
0 0 3cw 2

= 27abc u2 v 2 w2
\ dx dy dz = 27 abc u 2 v 2 w2 du dv dw
In the new coordinate system u, v, w, the region is bounded by a sphere.
Putting u = r sin q cos f, v = r sin q sin f, w = r cos q,
sphere u2 + v2 + w2 = 1 reduces to r = 1.
Since region is symmetric in all the octants.
Volume = 8 (volume in the positive octant)
Limits in the positive octant of the sphere are
Limits of r : r = 0 to r = 1
π
Limits of q : q = 0 to θ=
2
π
Limits of f : f = 0 to φ=
2

V = 8ÚÚ
ÚÚÚÚ dx dy dz
= 8ÚÚÚ
ÚÚÚ 27abc u v w du dv dw
2 2 2

p p
1
= 216 abc Ú 2 Ú 2 Ú (r 2 sin 2 q cos2 f )(r 2 sin 2 q ssiin 2 f )(
)(r 2 cos
cos2 q )r 2 ssin q dr dq df
0 0 0
p p
1
= 216 abc Ú 2 coss2f sin
sin 2 f df Ú 2 sin
sin 5q cos2 q dq Ú r 8 dr
0 0 0

1
Ê 1 1 p ˆ Ê 4 2 1ˆ r9
= 216 abc ◊ Á ◊ ◊ ˜ ◊ Á ◊ ◊ ˜ [Using reduction formula]
Ë 4 2 2 ¯ Ë 7 5 3¯ 9
0

4
= p abc
35
9.8 Volume as Triple Integral 9.187

EXERCISE 9.10
1. Find the volume of the sphere x 2 + y 2 + z 2 = a 2 cut off by the planes
z = 0 and the cylinder x2 + y2 = ax.
 2 3  π 2 
 Ans. : 3 a  2 − 3  
 
2. Find the volume common to the sphere x 2 + y 2 + z 2 = a 2 and the
cylinder x 2 + y 2 = b 2 , (a > b).
 Ans. : 2π b 2 a 2 − b 2 

3. Find the volume bounded by the cylinders y 2 = x, x 2 = y and the


planes z = 0, x + y + z = 2.
 11 
 Ans. : 30 
 
4. Find the volume of the cylinder x 2 + y 2 − 4 x = 0 cut by the cylinder
z 2 = 4 x.
x
 1024 
 Ans. : 15 
 
5. Find the volume of the paraboloid x + y = 4 z cut off by the plane z = 4.
2 2

[ Ans. : 32π ]
2
y
6. Find the volume of the paraboloid x 2 + + z = 1 cut off by the plane z = 0.
9
 3π 
 Ans. : 2 
 
7. Find the volume of the solid bounded by the paraboloids
y2 y4
z = 4 − x2 − and z = 3x 2 + .
4 4
 Ans. : 4 2π 

8. Find the volume of the solid bounded by the plane z = 0, the


paraboloid 3z = x 2 + y 2 and the cylinder x 2 + y 2 = 9.
 27π 
 Ans. : 2 
 
9. Find the volume of the solid bounded by the planes z = 0, z = 3 and the
cylinders y = x 2 , x 2 = y .
[Ans. : 1]

10. Find the volume of the solid which is above the cone z 2 + x 2 + y 2 and
inside the sphere x 2 + y 2 + (z − a)2 = a 2 .
[Ans. : p a3]
9.188 Chapter 9 Multiple Integrals

POINTS TO REMEMBER

• Double Integrals
The double integral of a function f (x, y) over the region R is denoted
by ∫∫ f(x, y)dx dy.
R

• Evaluation of Double Integrals


Method-I: When the region R is bounded by the curves y = y1(x),
y = y2(x) and x = a, x = b,

∫∫ f(x, y)dx dy = ∫ ∫ f (x, y )dy  dx


b y2 (x)

R
a y1 ( x ) 

Method-II: When the region R is bounded by the curves x = x1(y),


x = x2(y) and y = c, y = d,

∫∫ f(x, y)dx dy = ∫ ∫ f (x, y )dx  dy


d x2 ( y )

R
c x1( y ) 

If all the four limits are constant and f (x, y) is explicit, then the
f (x, y) can be integrated w.r.t. any variable first and also can be
written as product of two single integrals.
• Change of Order of Integration
Sometimes evaluation of double integral becomes easier by changing
the order of integration. To change the order of integration,
1. Draw the region of integration with the help of the given limits.
2. Draw vertical or horizontal strip as per the required order of
integration
3. Find the limits of integration

∫ ∫ f (x, y )dy  dx = ∫ ∫ f (x, y )dx  dy


b y2 (x) d x2 ( y )

a y1 ( x )  c x1( y ) 
• Double Integrals in Polar Coordinates
Putting x = r cos q, y = r sin q,

∫∫ f(x, y)dy dx = ∫∫ f(r cos θ,r sin


sin θ) J dr dθ
where Jacobian, J = r
Hence, ∫∫ f(x, y)dy dx = ∫∫ f(r cos θ,r sin
sin θ)r dr dθ

(Contd )
Points to Remember 9.189

(Contd )

• Triple Integrals
The triple integral of a continuous function f (x, y, z) over a region
V is denoted by ÚÚÚÚ f (x, y , z)dx dy dz.
V

• Triple Integrals in Cartesian Coordinates


If the region V is bounded below by a surface z = z1 (x, y) and
above by a surface z = z2 (x, y) and if the projection of region V in
xy-plane is R which is bounded by the curves y = y1 (x), y = y2 (x)
and x = a, x = b then

∫∫∫ f(x, y, z)dx dy dz = ∫ ∫


V
b

a
 y2 (x)

y1 ( x ) {∫ z2 ( x , y )

z1( x , y )
ff((x, y, }
y , z)dz dy  dx

• Triple Integrals in Cylindrical Coordinates


Putting x = r cos q, y = r sin q, z = z,

ÚÚÚ f(x, y, z)dx dy dz = ÚÚÚ f(r ccos


os q , r sin q , z) J d
dzz dr dq

where Jacobian, J = r
Hence, ÚÚÚ f(x, y, z)dx dy dz = ÚÚÚ f(r ccos
os q , r sin q , z)r d
dzz dr dq

• Triple Integrals in Spherical Coordinates


Putting x = r sin q cos f, y = r sin q sin f, z = r cos q,

∫∫∫ f(x, y, z)dx dy dz = ∫∫∫ f(r sinθ co


coss φ, r sin
sin θ sin
sin φ, r cos θ) J dr dθ dφ

where Jacobian,J = r 2 sin q

Hence, ∫∫∫∫ f(x, y, z)dx dy dz


= ∫∫∫ f (r sin θ cos
cos φ, r sin θ sin φ, r cos θ) ⋅ r 2
sin q dr dq df

Area as Double Integral


• Area in Cartesian Coordinates
(i) The area bounded by the curves y = y1(x) and y = y2(x) intersecting
at the points P (a, b) and Q (c, d ) is
c y2 (x)
A= ÚÚ
a y1 ( x )
dy dx

(Contd )
9.190 Chapter 9 Multiple Integrals

(Contd )

(ii) The area bounded by the curves x = x1(y) and x = x2(y) and
intersecting at the points P (a, b) and Q (c, d ) is
c y2 (x)
A= ÚÚ
a y1 ( x )
dy dx

• Area in Polar Coordinates


The area bounded by the curves r = r1(q ), r = r2(q ) and the lines
q = q1 and q = q2 is
q2 r2 (q )
A= Ú Ú
q1 r1(q )
r dr dq

• Volume in Cartesian Coordinates


V = ÚÚÚ dx dy dz
V

• Volume in Cylindrical Coordinates

V = ÚÚÚ r dr dq dz
V

• Volume in Spherical Coordinates

V = ÚÚÚ r
2
sinq dr dq df
V

MULTIPLE CHOICE QUESTIONS


Choose the correct alternative in the following questions:
x2
• x -
Ú0 Ú0 xe
y
1. To evaluate dx dy by change of order of integration, the lower limit
for the variable x is equal to
(a) y2 (b) 0 (c) • (d) y
4 3 2
2. Ú0 Ú0 Ú0 dx dy dz =
(a) 9 (b) 24 (c) 1 (d) 0
2 ex
3. By changing the order of integration, the integral Ú0 Ú1 dy dx is equivalent to
the double integral _______.
e 2 e2
Ú1 Úlog y dx dy
2
(a) (b) Ú1 Úlog y dx dy
1 log y e2 log y
(c) Úe Ú2
2 dx dy (d) Ú1 Ú2 dx dy
Multiple Choice Questions 9.191

4. By changing to spherical polar co-ordinates, ÚÚÚ dy dx dz, where R is the region


R
of hemisphere x 2 + y 2 + z 2 = a 2 is equivalent to triple integral ______
p
2p a 2 2p p a 2
(a) Ú0 Ú02 Ú0 r sin q dr dq df (b) Ú0 Ú0 Ú0 r sin q dr dq df
p p p p
a 2 a 2
(c) Ú Ú Ú
2 2
0 0 0
r sin q dr dq df (d) Ú Ú Ú
2 2
0 0 0
r cos q dr dq df

a x y
5. Ú0 Ú0 Ú0 xyz dz dy dx =

a6 a6 a4 a4
(a) (b) (c) (d)
24 48 48 24
6. In evaluating Ú Ú xy ( x + y) dx dy over the region between y = x2 and y = x, the
limits are
(a) x = 0 to 1, xy = 0 to 1 (b) x = 0 to 1, y = 0 to x
(c) x = 0 to 1, y = 0 to x2 (d) x = 0 to 1, y = x2 to x
a a2 - y2
7. Ú0 Ú0 ( a 2 - x 2 - y 2 ) dx dy =

p a4 p a4 p a4
(a) (b) (c) (d) pa4
8 4 2
• • - ( x 2 + y2 )
8. After transforming to polar co-ordinates Ú0 Ú0 e dx dy =
p p
1 -r 2 1 -r 2
(a) Ú Ú2
0 0
e dr dq (b) Ú Ú
2
0 0
e r d r dq
p p
• -r 2 • - y2
(c) Ú02 Ú0 e r dr dq (d) Ú02 Ú0 e r dr dq

p a cosq
9. Ú0 Ú0 r sin q dr dq =

a2 a2 a2 a2
(a) (b) (c) (d)
4 3 2 6
1 2
Ú0 Ú0 xy
2
10. dy dx =

5 1 2 4
(a) (b) (c) (d)
3 3 3 3
p p
11. Ú02 Ú02 sin ( x + y) dx dy is
p
(a) 0 (b) p (c) (d) 2
2
9.192 Chapter 9 Multiple Integrals

12. The value of the integral Ú Ú xy dx dy over the region bounded by the x-axis,
ordinate at x = 2a and the parabola x2 = 4ay is
a4 a4 a4 a4
(a) (b) (c) (d)
3 5 7 9
13. The triple integral ÚÚÚ dx dy dz gives
R
(a) volume (b) area (c) surface area (d) density
1 1 1- x
14. The value of Ú0 Úy Ú0
2 x dz dx dy is

4 3 8 6
(a) (b) (c) (d)
35 35 35 35
p
2p 1 2
15. The value of the integral Ú0 Ú03 Ú0 r sin q dr dq df is

p p 2p p
(a) (b) (c) (d)
3 6 3 4
• • - x 2 (1 + y2 )
16. The value of the integral Ú0 Ú0 e x dx dy is
p p p p
(a) (b) (c) (d)
2 3 4 6
• 2
17. Changing the order of integration in the double integral
s q
Ú0 Ú x f ( x, y) dy dx leads
4
to Úr Ú p f ( x, y) dx dy , then q is
(a) 4y (b) 16y2 (c) x (d) 8
18. The limits of integration of Ú Ú +
2 2
( x y ) d x d y over the domain bounded by
y = x2 and y2 = x are
(a) x = 0 to 1, y = x2 to x (b) x = 0 to 1, y = 0 to 1
2
(c) x = y to y , y = 0 to 1 (d) x = 0 to y, y = x to x2
xy
19. ÚÚ dx dy over the positive quadrant of the circle x2 + y2 = 1 is
1- y 2

1 2 5 5
(a) (b) (c) (d)
6 3 6 3

ÚÚr
20. 3
dr dq over the region included between the circles r = 2 sinq and
r = 4 sinq is
p
p 4sin q 3 4sin q 3
(a) Ú0 Ú 2sin q
r dr dq (b) Ú Ú
2
0 2sin q
r dr dq
Multiple Choice Questions 9.193

p
p 4sin q 3 4sin q 3
(c) Ú Ú
- p 2sin q
r dr dq (d) Ú Ú
2
0 sin q
r dr dq

a a2 - x 2
21. On converting into polar co-ordinates Ú0 Ú0 ( x 2 + y 2 ) dy dx =

p p
a
Ú0 Ú
a 2 r3
(a) Ú Ú
0 0
2 r2 dr d q (b) 0
dr dq
p p
a a
(c) Ú Ú
0 0
4 r3 dr dq (d)
Ú0 Ú 0
4 r2 dr d q

22. In spherical co-ordinates, dx dy dz is equal to


(a) r dq df dr (d) r sinq dq df dr (c) r2 sinq dq df dr (d) r2 dq df dr
1 1 1
Ú0 Ú0 Ú0 ( x + y 2 + z 2 ) dz dy dx is
23. The value of the integral 2

1 2
(a) 1 (b) (c) (d) 3
3 3
a a2 - x 2 a2 - x 2 - y2
24. The value of Ú0 dx Ú
0
dy Ú
0
dz is

p a3 p 2
(a) 4pa2 (c) 4pa3
(b) (d) a
6 3
25. The transformations x + y = u, y = uv transform the area element dy dx into
J du dv, where J is equal to
(a) 1 (b) u (c) –1 (d) u2
x2 y2
ÚÚ x y dx dy, where R is region enclosed by the ellipse + =1
3
26. The value of
R a2 b2
in the first quadrant is
b2 a 4 b3 a 4 ba 4 b2 a 2
(a) (b) (c) (d)
24 24 24 24
1 x
27. By changing the order of integration,
Ú0 Ú0 f ( x, y)dy dx =
0 y 0 1
(a) Ú1 Ú1 f ( x, y)dx dy (b) Ú1 Úy f ( x, y)dx dy
1 y 1 1
(c) Ú0 Ú1 f ( x, y)dx dy (d) Ú0 Úy f ( x, y)dx dy
2p 1
28. Ú0 dq Ú e2r dr is equal to
0

p 2
(a) e2 – 1 (b) (e - 1) (c) p(e2 – 1) (d) 2p(e2 – 1)
2
9.194 Chapter 9 Multiple Integrals

ÚÚ x
2 3
29. The value of y dx dy, where R is the region bounded by the rectangle
R
0 £ x £ 1 and 0 £ y £ 3 is
27 27 29 29
(a) (b) (c) (d)
4 8 4 8
30. The value of Ú Ú 3 y dx dy over the triangle with vertices (–1, 1), (0, 0) and (1, 1)
is [Winter 2015]
(a) 0 (b) 1 (c) 2 (d) 3
31. The area of the curve y = x2 + 1 bounded by the x-axis and the line x = 1 and
x = 2 is [Summer 2014]
3 10 1
(a) (b) (c) 6 (d)
10 3 6
32. The equation of a cylindrical surface x2 + y2 = 9 becomes ____ when converted
to cylindrical polar coordinates. [Summer 2016]
(a) r = 9 (b) r2 = 9 (c) r = ± 3 (d) r = 3
y
2 x2
33. Ú0 Ú0 e x dydx is equal to [Summer 2016]

(a) e2 – 1 (b) e2 (c) e2 + 1 (d) e–2


2 2 1
34. Ú Ú dxdy = [Winter 2016]
1 1 xy

(a) 0 (b) (log 2)2 (c) 1 (d) log 2


4 6
35. The region of Ú1 Ú2 dx dy represents [Winter 2016]

(a) rectangle (b) square (c) circle (d) triangle


2 2
36. The region Ú1 Ú1 dx dy represents [Summer 2017]
(a) rectangle (b) square (c) circle (d) triangle
1 1
Ú0 Ú0 (3 x - 2 y 2 ) dx dy is
37. The value of 2 [Summer 2017]
1
(a) 0 (b) 1 (c) –1 (d)
3

Answers

1.(d) 2.(b) 3.(b) 4.(a) 5.(b) 6.(d) 7.(a) 8.(c) 9.(b) 10.(d)
11.(d) 12.(a) 13.(a) 14.(a) 15.(a) 16.(c) 17.(a) 18.(a) 19.(a) 20.(a)
21.(b) 22.(c) 23.(a) 24.(b) 25.(b) 26.(a) 27.(d) 28.(c) 29.(a) 30.(c)
31.(b) 32.(d) 33.(a) 34.(b) 35.(a) 36.(a) 37.(d)
Appendix
Differential
1
Formulae

d n d 1
1. (x ) = nx n–1 14. (cot–1 x) = −
dx dx 1 + x2
d 1 d 1
2. (log x) = 15. (sec–1 x) =
dx x dx x x2 − 1
d x d 1
3. (e ) = e x 16. (cosec–1 x) = −
dx dx x x2 − 1
d x
4. (a ) = ax log a d dv du
dx 17. (uv) = u +v
d dx dx dx
5. (sin x) = cos x du dv
dx v −u
d  u dx dx
d 18.   =
6. (cos x) = –sin x dx  v  v 2
dx
d
7. (tan x) = sec2 x
dx
d
8. (cot x) = –cosec2 x
dx
d
9. (sec x) = sec x tan x
dx
d
10. (cosec x) = –cosec x cot x
dx
d 1
11. (sin–1 x) =
dx 1 − x2
d 1
12. (cos–1 x) = -
dx 1 - x2
d
13. (tan–1 x) = 1 + 12
dx x
Appendix
Integral
2
Formulae

xn +1 1
∫ x dx (n π –1) ∫
n
1. = 16. dx =
n +1 x − a2
2

1  x
2. ∫ x dx = log x log ( x + x 2 − a 2 ) = cosh–1  
a
∫ e dx = e x
17. ∫ 2
1
x
3. dx =
ax x + a2
4. ∫ a dx = , a > 0, a π 1
x
log a  x
log ( x + x 2 + a 2 ) = sinh–1  
a
5. ∫ sin x dx = – cos x 1 1  a + x
6. ∫ cos x dx = sin x
18. ∫a− x2
2
dx =
2a
log 
 a − x 
1 −1  x 
7. ∫ tan x dx = –log cos x = tanh   , x2 < a2
a a
 x − a
8. ∫ cot x dx = log sin x 19. ∫ 2
1
dx =
1
log 
x − a2 2a  x + a 
9. ∫ sec x dx = log (sec x + tan x) = −
1  x
coth −1   , x2 > a2
a  a
10. ∫ cosec x dx = log (cosec x – cot x) 1 1  x
11. ∫ sec x dx = tan x
2 20. ∫a 2
+ x2
d x = tan −1  
a  a

∫ cosec x dx = – cot x ∫
2
12. 21. a 2 − x 2 dx
a2  x
13. ∫ sec x tan x dx = sec x =
x 2
2
a − x2 +
2
sin −1  
 a
14. ∫ cosec x cot x dx = –cosec x 22. ∫ a 2 + x 2 dx
d x = sin–1  
1

x
15. x 2 2 a2
a −x
2 2  a = a +x + log ( x + x 2 + a 2 )
2 2
A2.2 Integral Formulae

23. ∫ x 2 − a 2 dx 31. ∫ sin[ f ( x)] f ′( x) d x = – cos f (x)


=
x 2 2 a2
x −a − log ( x + x2 − a2 ) 32. ∫ cos[ f ( x)] f ′( x) d x = sin f (x)
2 2 a a

24. ∫e
ax
sin bx d x 33. ∫
0
f ( x) d x = ∫ f (a − x) d x
0
e ax 2a
= 2 (a sin bx – b cos bx)
a + b2 34. ∫
0
f ( x) d x

∫e
ax
25. cos bx d x a a

e ax
= ∫ f ( x ) d x + ∫ f ( 2a − x ) d x
= 2 (a cos bx + b sin bx) 0 0
a + b2 a

 du  35. ∫ f ( x) d x
26. ∫ uv dx = u ∫ v d x − ∫  dx ∫ v d x dx −a
a

= 2 ∫ f ( x) d x , if f (x) is even
∫ [ f ( x)] f ′ ( x) d x
n
27. 0

[ f ( x)]n +1 = 0, if f (x) is odd


= , n π –1 2a
n +1
f ′ ( x) 36. ∫ f ( x) d x
28. ∫ f ( x)
d x = log | f (x) | 0
a
= 2 Ú f ( x) d x, if f (x) = f (2a – x)
∫ e f ′ ( x) d x = e
f ( x) f (x)
29.
0
= 0, if f (x) = –f (2a – x)
∫ e [ f ( x) + f ′( x)] dx = e f (x)
x x
30.

reduction formulAe

1 cos x sinn –1 x tan x sec n − 2 x


1. ∫ sin x dx = –
∫ sec x dx =
n
5. n
n
n −1 n −1
∫ sin x dx n−2
n−2
+
∫ sec x dx
n−2
+
n n −1
1
2. ∫ cos n x dx = sin x cosn –1 x 6. ∫ cosec x dx
n
n
n −1 − cot x cosec n − 2 x
∫ cos x dx
n−2
+ =
n n −1
n −1
tan x n−2

3. tan n x dx =
n −1
+
n −1 ∫
cosec n − 2 x dx
– ∫ tan n − 2 x dx
7. (a) ∫ sin
m
x cos n x dx
n −1
cot x cos n + 1 x sin m −1 x
4. ∫ cot x dx = −
n
= −
n −1 m+ n
– ∫ cot n − 2 x dx m −1
m+n ∫
+ sin m − 2 x cos n x dx
Integral Formulae A2.3

(b) ∫ sin n −1 n − 3 n − 5 1 p
m
x cos n x dx = ◊ ◊ ◊…◊ ◊ ,
sin x cos n −1 x
m +1 n n−2 n−4 2 2
=
m+ n if n is even
n −1
m+ n ∫
+ sin m x cos n − 2 x dx p
9. ∫ 2
cos n x dx
0
(c) ∫ sin
m
x cos n x dx n −1 n − 3 n−5 2
= ◊ ◊ ◊…◊ ,
cos n + 1 x sin m −1 x n n−2 n−4 3
=−
n +1 if n is odd
m −1
n +1 ∫
+ sin m − 2 x cos n + 2 x dx n −1 n − 3 n − 5 1 p
= ◊ ◊ ◊…◊ ◊ ,
n n−2 n−4 2 2
∫ sin
m
(d) x cos n x dx
if n is even
sin m + 1 x cos n −1 x
= p
m +1
n −1
10. ∫ 2
sin m x cos n x dx
m +1 ∫
0
+ sin m + 2 x cos n − 2 x dx m −1 m−3
= ◊ ◊…◊
m+n m+ n−2

(e) sin m x cos n x dx 2

1
,
cos n + 1 x sin m + 1 x
=− 3 + n n +1
m +1
m+ n+ 2 if m is odd and n may be odd or even
∫ sin x cos x dx
m+ 2 n
+
m +1 m −1 m−3 m−5
= ◊ ◊
m+ n m+ n−2 m+ n−4
(f) ∫ sin
m
x cos n x dx
sin m + 1 x cos n + 1 x 1 n −1 n − 3 2
… × ◊ … ,
= 2+n n n−2 3
m +1
m+ n+ 2 if m is even and n is odd
m +1 ∫
+ sin m x cos n + 2 x dx
m −1 m − 3 m−5
p = ◊ …
∫ 2 m+ n m+ n−2 m+ n−4
n
8. sin x dx
0
1 n −1 n − 3 1 p
n −1 n − 3 n − 5 2 × ◊ … ◊ ,
= ◊ ◊ ◊ …◊ , 2+n n n−2 2 2
n n−2 n−4 3 if m is even and n is even
if n is odd,
Appendix
Standard
3
Curves

Rectangular hyperbola xy = a Catenary y = c cosh ( cx )


y
y

x
O c
x
O

Cissoid of Diocles y2 (2a – x) = x3 Cubical parabola a2y = x3


y
x = 2a
Asymptote y

x
O (2a, 0) x
O

Semi-cubical parabola ay2 = x3 ay2 = x(a2 + x2)


y

x
O
x
O
A3.2 Standard Curves

Witch of Agnessi xy2 = 4a2 (a – x) x (x2 + y2) = a(x2 – y2)


y

x
y=
x
x
O A (a, 0) O
y=
�x

y(x2 + 4a2) = 8a3 2y2 = x(4 + x2)


y

(0, 2a) x
O (4, 0)
(2, 0)
x
O

a2 y2 = x3 (2a – x) y2 = x5 (2a – x)
y
y

x
(2a, 0) O (2a, 0)
x
O

y2 (a2 – x2) = a3x Cycloid x = a (q + sin q ), y = a (1 – cos q)


y
y

x
O 2a x
O
q = –p q=p
q=0
Standard Curves A3.3

Cycloid x = a (q + sin q ), y = a (1 + cos q ) Cycloid x = a (q – sin q ), y = a (1 + cos q )


y y

�p p x x
O O

q=0 q = 2p
q=�p q=p q=p
2 2 2
Cycloid x = a (q – sin q), y = a (1 – cos q) Astroid x 3 + y 3 = a 3
y y

(0, a)

x x
O (a, 0)
q=0 q = 2p
q=p
2 2
Ê xˆ 3 Ê yˆ 3
Hypocycloid Á ˜ + Á ˜ = 1 Folium of Descartes x3 + y3 = 3axy
Ë a¯ Ë b¯
y
x + y = 3a
x
y y=
(0, b) 3a (3a/2, 3a/2)

x
O
x
O (0, a)

x+y+a=0

Cardioid r = a(1 + cos q ) Cardioid r = a(1 – cos q )


p p
q= q=
2 2

q=0 q=0
O O
A3.4 Standard Curves

Cardioid r = a(1 + sin q ) Cardioid r = a(1 – sin q )


p p
q= q=
2 2
O q=0

O q=0

Lemniscate of Bernoulli r2 = a2 cos 2q r2 = a2 sin 2q


p
q=
2

p
q=
2
O
q=0
q=0
O

r = aq mq Hyperbolic spiral rq = a
p
q=
2

q=0

O q=0

Spiral of Archimidies r = aq r 2q = a2
p
q= p
2 q=
2

O q=0 q=0

r = aq
Standard Curves A3.5

Four leaved rose r = a cos 2q , a > 0 Three leaved rose r = a sin 3q , a > 0

q=p q=p
2 2
q = 5p q=p
6 6

q=0 q=0
O
GUJARAT TECHNOLOGICAL UNIVERSITY
BE SEMESTER-1st/2nd (New) Examination – Winter 2016

Subject Code: 2110014 Date: 24/01/2017


Subject Name: Calculus

Q.1 Multiple-Choice Questions


(a) Choose the appropriate answer out of the four options given in each of the
following questions. [07]
Ê p 1ˆ
1. The sequence sin Á + ˜ converges to
Ë 6 n¯
(a) 0 (b) 1 (c) –1 (d) 0.5

Ê p 1ˆ
Solution: The sequence sin Á + ˜ converges to 0.5.
Ë 6 n¯
Ans. (d)
• n
Ê eˆ
2. The sum of the series  ÁË p ˜¯ is
n =1

p e p e
(a) (b) (c) (d)
p -e p -e e-p p

e e
a p p e
Solution: Sum of the series = = = =
1- r e e p -e
1- p-
Ans. (b) p p

sin( x - 6)
3. The value of lim is
x Æ6 x-6
(a) 0 (b) 1 (c) –1 (d) 0.5
1 È ( x - 6) ( x - 6)5
3 ˘
Solution lim Í( x - 6) - + + ˙
x Æ6 x - 6 Î 3! 5! ˚
È ˘
= lim Í1 - ( x - 6) + ( x - 6) + ˙
2 4

x Æ6 Î 3! 5! ˚
=1–0+0
=1
Ans. (b)
SQP.2 Solved Question Papers Winter 2016

x2
4. Asymptote parallel to y-axis of the curve y = is the line
x-3
(a) x = 0 (b) y = 3 (c) x = 3 (d) does not exist
Solution: Asymptotes parallel to y-axis are obtained by equating the coefficient
of highest degree term of y in the equation to zero.
Equating the coefficient of highest degree term of y i.e. x – 3 to 0,
x–3=0
x=3
Ans. (c)
5. Let f(x, y) = y sin(xy). The value of fx(p, 1) is
(a) 0 (b) 1 (c) –1 (d) 2.5
Solution: f(x, y) = y sin(xy)
fx(x, y) = y2 cos(xy)
fx(p, 1) = (1)2 cos(p) = –1
Ans. (c)
22
1
6. Ú Ú xy dxdy =
11
(a) 0 (b) (log2)2 (c) 1 (d) log2
Solution:
1È 1 ˘
2 2 2
2 1
Ú y ÍÚ x ˙
Í d x ˙ d y = Ú log x 1 ◊ y dy
1 Î1 ˚ 1

2 2
= log x 1
log y 1

= (log2 – log1)2
= (log2)2
Ans. (b)

7. The coefficient of x5 in the expansion of ex is


1 1 1
(a) (b) (c) (d) 5
5 4! 5!
x2 x3 x 4 x5
Solution: Expansion of ex = 1 + x + + + + +
2! 3! 4! 5!
1
The coefficient of x5 in ex =
5!
Ans. (c)
Solved Question Papers Winter 2016 SQP.3

(II) Choose the appropriate answer out of the four options given in each of the
following questions.

2n
1. Â 3n - 1
is
n =1

(a) convergent and sum is 0 (b) convergent and sum is 1


(c) divergent (d) oscillating
2n
Solution: un =
3n - 1
2 n +1
un +1 =
3n + 2
un 2n 3n + 2
= ◊
un +1 3n - 1 2 n +1
Ê 2ˆ
n 3+
un 1 ÁË ˜

lim = lim ◊
n Æ• un + 1 n Æ• 2 Ê 1ˆ
nÁ3 - ˜
Ë n¯
1
=
<1
2
Hence, the series is divergent.
Ans. (c)
x y
Ê xˆ Ê yˆ ∂f ∂f
2. If f = e tan Á ˜ + e x cot Á ˜ then value of x
y
+y is
Ë y¯ Ë x¯ ∂x ∂y
(a) 0 (b) f (c) –f (d) 2f
Solution: f(tx, ty) = t0 f(x, y) is a homogenous function of degree 0.
∂f ∂f
x +y = nf = 0. f = 0
∂x ∂x
Ans. (a)

3. The curve x3y + y3x – 3 is symmetric about the


(a) x-axis (b) y-axis (c) origin (d) line y = x
Solution: f(x, y) = x3y + y3x – 3
On interchanging x and y, the equation remains unchanged, i.e. f(x, y) = f(y, x).
Hence, the curve is symmetric about the line y = x.
Ans. (d)
46
4. What does the region of Ú Ú dxdy represent?
12
(a) Rectangle (b) Square (c) Circle (d) Triangle
SQP.4 Solved Question Papers Winter 2016

Solution: All four limits are constant. Hence, the region represents a rectangle.
Ans. (a)

1
5. The value of Ú 1 + x 2 dx is
0

p
(a) p (b) (c) 0 (d) 1
2
Solution:
• Èb 1 ˘
1
dx ˙ = lim ÍÈ tan -1 x ˙˘
b
Ú 1 + x2 dx = lim Í Ú
b Æ• Í 1 + x 2
˙˚ b Æ• Î 0 ˚
0 Î0
p
= lim [tan -1 (b) - tan -1 (0)] = lim tan -1 (b) = tan -1 (•) =
b Æ• b Æ• 2
Ans. (b)
6. The minimum value of f(x, y) = x2 + y2 is

(a) 1 (b) 2 (c) 4 (d) 0


Solution: f(x, y) = x2 + y2
fx = 2x
fy = 2y
When fx = 0 and fy = 0,
x = 0 and y = 0
Hence, the stationary point is (0, 0).
fxx = r = 2 > 0, fyy = t = 2, fxy = s = 0
rt – s2 = 4 – 0 = 4 > 0
2
rt – s > 0 and r > 0. Hence, f(x, y) is minimum at point (0,0).
Minimum value f(x, y) = f(0, 0) = 0 + 0 = 0
Ans. (d)

∂( x, y)
7. If x = u + 3v and y = –u + v then J = is
∂(u, v)
(a) –1 (b) 4 (c) 5 (d) 7
∂x ∂x
∂( x, y) ∂u ∂v = 1 3 = 1 + 3 = 4
Solution: J= =
∂(u, v) ∂y ∂y -1 1
∂u ∂v
Ans. (b)
Solved Question Papers Winter 2016 SQP.5

Q2 (a) You drop a ball from ‘a’ meters above a flat surface. Each time the ball hits the
surface after falling a distance h, it rebounds a distance rh where 0 < r < 1. Find
2
the total distance the ball travels up and down, when a = 6 m and r = m.
3
Solution: Refer Example 6 on page 1.14.

Q2 (b) Evaluate

e x + e- x - x 2 - 2
i. lim
xÆ0 sin 2 x - x 2
Solution: Refer Example 5 on page 4.5.

ii. lim Ê x - 1 ˆ
Á
x Æ1 Ë x - 1 log x ˜¯
Solution: Refer Example 3 on page 4.39.

Q2 (c) Obtain the Maclaurin’s series of loge(1 + x) and hence find the series of
Ê1 + xˆ Ê 11ˆ
loge Á ˜ . Also obtain the approximate value of loge Á ˜ .
Ë1- x¯ Ë 9¯

Solution: Refer Example 2 on page 2.31.

Ï 2 x2 y
Ô ( x, y ) π 0
Q3 (a) Show that f ( x, y) = Ì x 3 + y3
Ô 0 ( x, y ) = 0
Ó
is not continuous at the origin.
Solution: Refer Example 11 on page 7.8.
-r 2
1 ∂ Ê 2 ∂q ˆ ∂q
Q3 (b) If q = t ˜=
n
e 4t then find the value of n for which Ár .
r 2 ∂r Ë ∂r ¯ ∂t
Solution: Refer Example 32 on page 7.39.

Q3 (c) State Euler’s theorem on homogenous function of two variables. If


Ê x 2 + y2 ˆ
u = tan -1 Á , prove that
Ë x + y ˜¯
∂2 u ∂2 u ∂2 u
x2 + 2 xy + y 2 2 = - 2 sin 3 u cos u
∂x 2 ∂x ∂y ∂y
Solution: Refer Example 22 on page 7.153.
SQP.6 Solved Question Papers Winter 2016

Q4 (a) Find the Jacobian the transformation


x = r sin f cos q, y = r sin f sin q, z = r cos f
Solution: Refer Example 4 page 8.103.

Q4 (b) Find the equations of the tangent plane and normal line to the surface f(x, y, z)
= x2 + y2 + z – 9 = 0 at the point p(1, 2, 4).
Solution: Refer Example 2 on page 8.2.

Q4 (c) A rectangular box open at the top is to have a volume of 32 cubic units. Find
the dimensions of the box requiring least material for its construction.
Solution: Refer Example 11 on page 8.60.
1 2

Ú Ú (1 - 6 x
2
Q5 (a) Evaluate y)dxdy.
-1 0
Solution: Refer Example 3 on page 9.4.
1 z 2p

Ú Ú Ú (r cos2 q + z 2 )rdq drdz .


2
Q5 (b) Evaluate
0 0 0
Solution: Refer Example 4 on page 9.114.

Q5 (c) Change the order of integration and evaluate


2
1 1- x
ey
Ú Ú (e y + 1) 1 - x 2 - y 2
dy dx
0 0

Solution: Refer Example 8 on page 9.49.


Q6 (a) Let S = Â na n where |a | < 1. Find the value of a in (0, 1) such that S = 2a.
n =1

Solution: Refer Example 4 on page 1.13.

Q6 (b) Check for convergence/divergence



5n3 - 3n
i. Â n2 (n - 2)(n2 + 5)
.
n =1

Solution: Refer Example 19 on page 1.29.


2n
ii. Â n3 + 1.
n =1

Solution: Refer Example 3 on page 1.39.


Solved Question Papers Winter 2016 SQP.7


(-1)n
Q6 (c) 1. Test the series for absolute or conditional convergence  n + n +1
.
n =1
Solution: Refer Example 3 on page 1.86.

OR

x 2 n -1
Q6 (c) 2. For the series  (-1)n -1 2 n -1
, find the radius and internal of convergence.
n =1
Solution: Refer Example 6 on page 1.100.
Q7 (a) The graph of y = x2 between x = 1 and x = 2 is rotated around the x-axis. Find
the volume of the solid so generated.
Solution: Refer Example 3 on page 6.9.
Q7 (b) Test the convergence of the improper integrals. If convergent, then evaluate the
same.
1
dx
i. Ú1- x
0
Solution: Refer Example 3 on page 5.11.

1
ii. Ú (1 + x 2 )(1 + tan -1 x) dx
0

Solution: Refer Example 10 on page 5.6.


Q7 (c) Trace the curve y2(a + x) = x2(a – x) a > 0.
Solution: Refer Example 4 on page 3.8.
GUJARAT TECHNOLOGICAL UNIVERSITY
BE Semester-1st/2nd (New) Examination – Summer 2017

Subject Code: 2110014 Date: 01/06/2017


Subject Name: Calculus
Q.1 Multiple-Choice Questions
(I) Choose the appropriate answer out of the four options given in each of the
following questions.
1 1 1
1. Infinite series 1 + + 2 + 3 + ... is
2 2 2
(a) divergent (b) convergent (c) oscillatory (d) none of these

1 1 1
Solution: 1+ + + +
2 2 2 23
1
a = 1, r = , |r |<1
2
Hence, the series is convergent.

Ans. (b)

x2 x3 x 4
2. The series x - + - + represents an expansion of
2 3 4
(a) ex (b) log(1 + x) (c) sin x (d) cos x

x2 x3 x 4
Solution: x- + - + = log(1 + x )
2 3 4
Ans. (b)

Ê sin 2 x ˆ
3. The value of lim Á ˜ is
xÆ0 Ë x ¯
(a) 2 (b) 1 (c) –1 (d) 1
2
Ê sin 2 x ˆ Ê sin 2 x ˆ
Solution: lim Á
xÆ0 Ë
˜ = lim Á ˜ = 2 ◊1 = 2
x ¯ xÆ0 Ë 2x ¯
Ans. (b)

3x2
4. Asymptote parallel to y-axis of the curve y = is the line
x-2
(a) x = 0 (b) y = 0 (c) x = 2 (d) y = 2
SQP.2 Solved Question Papers Summer 2017

Solution: Equating the coefficient of highest degree term of y i.e. x – 2 to zero,


x–2=0
x=2
Ans. (c)

5. f(x) = | x | is _____ at origin.


(a) continuous (b) discontinuous (c) differentiable (d) none of these
Solution: f(x) = | x | is continuous at origin.
Ï x x>0
f ( x) = Ì
Ó- x x < 0
lim f ( x ) = lim x = 0
x Æ 0+ x Æ 0+

lim f ( x ) = lim (- x ) = 0
x Æ 0- x Æ 0-

Ans. (a)

6. Curve y2(a + x) = x2(b – x) is symmetric about _____


(a) x-axis (b) y-axis (c) line x = b (d) line x = –a
Solution: The powers of y occurring in the equation are all even. Hence, the
curve is symmetric about x-axis.
Ans. (a)
dy
7. The curve increases strictly in interval in which is
dx
(a) < 0 (b) > 0 (c) = 0 (d) none of these
dy
Solution: >0
dx
Ans. (b)

(b) Choose the appropriate answer out of the four options given in each of the
following questions.

-x
1. The value of Úe cos(2 x )dx is
0

1 1 2
(a) 0 (b) - (c) (d)
5 5 5
• Èb -x ˘
-x
Solution: Ú e cos2 x d x = lim Í Ú e cos(2 x )dx ˙
b Æ• Í ˙˚
0 Î0
b
ÔÏ e x Ô¸
-
= lim Ì (- cos2 x + 2sin 2 x ) ˝
b Æ• Ô 1 + 4 Ô˛0
Ó
Solved Question Papers Summer 2017 SQP.3

ÏÔ e - x ¸Ô 1
= lim Ì (- cos 2 x + 2 sin 2 x ) ˝ - (-1)
b Æ• Ô 5
Ó ˛Ô 5
1
=
5
Ans. (a)
22
2. What does the region Ú Ú dxdy represent?
11
(a) Rectangle (b) Square (c) Circle (d) Triangle
Solution: All four limits are constant. Hence, the region represents a
rectangle.
Ans. (a)

Ê cos x ˆ
3. The value of lim Á p ˜ is
p x-
xÆ Á ˜
2 Ë 2¯
p
(a) 0 (b) 1 (c) –1 (d)
2
cos x Ê0 ˆ
Solution: lim
p p ÁË form˜¯
xÆ x- 0
2
2
Ê sin x ˆ
= lim Á - ˜ (L’Hospital’s rule)
pË 1 ¯

2

= lim (- sin x )
p

2

Êpˆ
= - sin Á ˜
Ë 2¯
= -1
Ans. (c)
Ê yˆ
4. The function f(x, y) = x2y f Á ˜ is homogenous of degree
Ë x¯
(a) 0 (b) 1 (c) 2 (d) 3
Ê yˆ
Solution: f ( x, y = x 2 y f Á ˜
Ë x¯
Replacing x by xt and y by yt,
Ê yˆ
f ( xt , yt ) = t 3 x 2 y f Á ˜
Ë x¯
SQP.4 Solved Question Papers Summer 2017

= t3 f(x, y)
which is a homogenous function of degree 3.
Ans. (d)
dy
5. The equation of the form f(xy) = c then = _______
dx
fx fx fy fy
(a) - (b) (c) - (d)
fy fy fx fx
∂f
f
= - ∂x = - x
dy
Solution:
dx ∂f fy
∂y
Ans. (a)

11

Ú Ú (3 x - 2 y 2 )dx dy is
2
6. The value of
00
1
(a) 0 (b) 1 (c) –1 (d)
3
11 1 È1 ˘
ÚÚ - = Ú ÍÍÚ (3 x - 2 y )dx ˙˙ dy
2 2 2 2
Solution: (3 x 2 y )d x d y
00 0 Î0 ˚
1 1
3
= Ú 3 x - 2 xy 2 dy
0 3 0

1
= Ú (1 - 2 y 2 )dy
0
1
3
= y - 2y
3 0

2
=1-
3
1
=
3
Ans. (d)
∂( x, y)
7. If x = u + 3v and y = v – u then the value of
∂(u, v)
(a) 4 (b) –1 (c) 5 (d) 7
Solved Question Papers Summer 2017 SQP.5

∂x ∂x
∂( x, y) ∂u ∂v = 1 3 = 1 + 3 = 4
Solution: =
∂(u, v) ∂y ∂y -1 1
∂u ∂v
Ans. (a)

Q2 (a) Expand log(sec x) in power of x.


Solution: Refer Example 1 on page 2.57.

1
Ê 1x + 2 x + 3 x ˆ x
Q2 (b) Evaluate lim Á ˜¯
xÆ0 Ë 3
Solution: Refer Example 5 on page 4.48.

Q2 (c)
(i) Trace the curve y2(2a – x) = x3.
Solution: Refer Example 2 on page 3.6.
3
1
(ii) Determine Ú 3- x
dx converge or diverges.
0

Solution: Refer Example 1 on page 5.10.

Q3 (a) If f(x, y) = x2y + xy2 then find fx(1, 2) and fy(1, 2) by definition.
Solution: Refer Example 1 on page 7.12.
Q3 (b) Check the continuity for the following function at (0, 0).
Ï 2 xy
Ô ( x, y) π (0, 0)
f ( x, y ) = Ì x 2 + y 2
Ô 0 ( x, y) = (0, 0)
Ó
Solution: Refer Example 10 on page 7.7.
Ê 1 1 ˆ
4 + y4 ∂u ∂u
Á x ˜
Q3 (c) (i) If u = sin -1 Á 1 1 ˜
then find the values of x +y and
∂x ∂y
ÁË x 5 + y 5 ˜¯

∂2 u ∂2 u 2 ∂ u
2
x2 + 2 xy + y .
∂x 2 ∂x ∂y ∂y 2
Solution: Refer Example 9 on page 7.142.
SQP.6 Solved Question Papers Summer 2017

ex + y + z ∂u ∂u ∂u
Q3 (c) (ii) If u = then show that + + = 24 .
e x + e y + ez ∂x ∂y ∂z
Solution: Refer Example 4 on page 7.14.

Q4 (a) Find the extreme value of x3 + 3xy2 – 3x2 – 3y2 + (4), if any.
Solution: Refer Example 5 on page 8.25.

Q4 (b) Find the equation of the tangent plane and normal line to the surface
2x2 + y2 + 2z = 3 at (2, 1, –3).
Solution: Refer Example 4 on page 8.3.

Q4 (c) (i) Find a point on the plane 2x + 3y – z = 5 which is nearest to the origin.
Solution: Refer Example 4 on page 8.41.
Q4 (c) (ii) Expand exy in the power of x – 1 and y – 1 using Taylor’s expansion.
Solution: Refer Example 4 on page 8.84.

1 x x2
Q5 (a) Test the convergence of the series + + + .
1◊ 2 ◊ 3 4 ◊ 5 ◊ 6 7 ◊8 ◊ 9
Solution: Refer Example 25 on page 1.56.

• 1
Q5 (b) Test the convergence of the series  [(n3 + 1) 3 - n] .
n =1

Solution: Refer Example 22 on page 1.31.


Q5 (c) (i) Determine absolute or conditional convergence of the series

n2
 (-1)n n3 + 1
n =1

Solution: Refer Example 4 on page 1.87.

10 20 40
(ii) Test the convergence for 5 - + - + .
3 9 47
Solution: Refer Example 2 on page 1.12.

Q6 (a) Evaluate ÚÚ r a 2 - r 2 drdq over the upper half of the circle r = a cos q.
Solution: Refer Example 2 on page 9.72.
Solved Question Papers Summer 2017 SQP.7

4 x y
3
Q6 (b) Sketch the region of integration and evaluate ÚÚ 2
e x
dy dx.
1 0

Solution: Refer Example 12 on page 9.9.


21
x2
Q6 (c) (i) Evaluate the integral ÚÚe dxdy by changing the order of integration.
0 y
2
Solution: Refer Example 5 on page 9.46.
2 2 yz
(ii) Evaluate the integral Ú Ú Ú xyz dxdydz.
01 0

Solution: Refer Example 10 on page 9.118.

Q7 (a) Find the area included between the curve y2(2a – x) = x3 and its asymptote.
Solution: Refer Example 10 on page 9.149.
Q7 (b) Find the volume of the solid generated by revolving the cardioid r = a(1 – cos
q) about the initial line.
Solution: Refer Example 1 on page 6.25.

Q7 (c) Use triple integration to find the volume of the solid within the cylinder
x2 + y2 = 9 between the planes z = 1 and x + z = 1.
Solution: Refer Example 9 on page 9.180.
Index

A Convergence and Divergence of


Improper Integrals 5.16
Absolute Convergence of a Series 1.84 Convergence, Divergence and
Alternating Series 1.77 Oscillation of a Sequence 1.1
Area as Double Integral 9.141 Convergence, Divergence and
Area in Cartesian Coordinates 9.141 Oscillation of Infinite Series 1.8
Area in Polar Coordinates 9.154
D
B D’Alembert’s Ratio Test 1.36
Bounded Sequence 1.2 Deductions from Euler’s Theorem 7.115
Direct Comparison Test 5.17
C Double Integrals in Polar Coordinates
9.66
Cauchy’s Integral Test 1.71 Double Integrals Over Rectangles 9.1
Cauchy’s Root Test 1.63
Chain Rule 7.70
E
Change of Order of Integration 9.31
Change of Variables 9.77 Errors and Approximations 8.12
Change of Variables from Cartesian to Euler’s Theorem for a Function of Three
Other Coordinates 9.95 Variables 7.114
Change of Variables from Cartesian to Euler’s Theorem for a Function of Two
Polar Coordinates 9.77 Variables 7.114
Clairaut’s Theorem 7.12 Euler’s Theorem for Homogeneous
Comparison Test 1.17 Functions 7.114
Composite Function of One Variable Evaluation of Double Integrals by
7.70 Fubini’s Theorem 9.2
Composite Function of Two Variables
7.77 F
Concavity, Convexity and Points of
First Derivative Test for Local Extremum
Inflection of a Curve 3.1
3.2
Conditional Convergence of a Series
Functions of Two or More Variables 7.1
1.84
I.2 Index

G Maximum and Minimum Values by


Second Derivative Test 8.21
Geometric Series 1.10 Maximum and Minimum Values with
Graphs and Level Curves 7.1 Constrained Variables 8.38
Method of Lagrangian Multipliers 8.49
H Monotonic Functions 3.1
Monotonic Sequence 1.2
Higher-Order Partial Derivatives 7.11

I N
nth Term Test for Divergence 1.9
Implicit Differentiation 7.105
Improper Integral of the Third Kind
5.16 P
Improper Integrals 5.1 Partial Derivatives 7.11
Improper Integrals of the First Kind 5.1 Power Series 1.94
Improper Integrals of the Second Kind Properties of Double Integrals 9.3
5.9
Properties of Infinite Series 1.8
Infinite Series 1.8
Properties of Jacobians 8.98
Interval and Radius of Convergence
1.94
R
J Relation between Polar and Cartesian
Coordinates 3.31
Jacobians 8.97

S
L
Sandwich Theorem for Sequences 1.6
L’Hospital’s Rule 4.1
Second Derivative Test for Local
Leibnitz’s Test for Alternating Series Extremum 3.3
1.77
Sequence 1.1
Limit and Continuity of Functions of
Several Variables 7.2
Limit Comparison Test 5.18 T
Limit of a Sequence 1.1 Tangent Plane and Normal to a Surface
Linear Approximation or Linearization 8.1
8.10 Taylor’s Formula for Two Variables 8.81
Taylor’s Series 2.1
M Total Derivatives 7.70
Tracing of Cartesian Curves 3.4
Maclaurin’s Series 2.27
Tracing of Parametric Curves 3.24
Maxima and Minima 3.2
Index I.3

Tracing of Polar Curves 3.31 Volume by Slicing 6.1


Triple Integrals 9.109 Volume of Solid of Revolution 6.6
Triple Integrals in Cartesian Coordinates Volume of Solid of Revolution in
9.109 Cartesian Form 6.6
Triple Integrals in Cylindrical Volume of Solid of Revolution in
Coordinates 9.109 Parametric Form 6.7
Triple Integrals in Spherical Coordinates Volume of Solid of Revolution in Polar
9.110 Form 6.7
Volume of Solids of Revolution by Disk
U Method 6.6
Volume of Solids of Revolution by
Uniform Convergence of a Series 1.85 Washer Method 6.6

V W
Volume as Triple Integral 9.169 Weierstrass’s M-Test 1.91
Volume by Cylindrical Shells 6.33

You might also like